Sunteți pe pagina 1din 517

https://telegram.

me/aedahamlibra
https://telegram.me/aedahamlibra

Subjectwise-Chapterwise
Based On Previous Papers

©Rainbow Publishers
Email: rainbowpublishers123@gmail.com
The copyright of this book is entirely with the Rainbow Publishers. The reproduction of this book or a part of this will
be punishable under the Copyright Act.All disputes subject to Hisar jurisdiction.

Page 1 of 516
https://telegram.me/aedahamlibra
Table of contents
QUANTITATIVE APTITUDE.............. 3 ANCIENT INDIA ................................................... 299
MEDIEVAL INDIA ................................................ 304
PERCENTAGE ...........................................................3
MODERN INDIA ................................................... 308
PROFT, LOSS & DISCOUNT ......................................8
INDIAN ART & CULTURE .................................... 319
NUMBER SYSTEM & LCM HCF ............................27
INDIAN POLITY & CONSTITUTION ....................... 322
SIMPLIFICATION ....................................................35
PHYSICAL GEOGRAPHY ....................................... 339
ALGEBRA...............................................................45
GEOGRAPHY OF INDIA ........................................ 346
AVERAGE ..............................................................71
WORLD GEOGRAPHY .......................................... 359
SIMPLE & COMPOUND INTEREST ...........................78
INDIAN ECONOMY ............................................... 363
RATIO & PROPORTION ...........................................86
ECONOMICS ........................................................ 369
TIME & WORK .......................................................97
PHYSICS .............................................................. 380
SPEED TIME AND DISTANCE ................................108
CHEMISTRY ......................................................... 394
MENSURATION ....................................................117
BIOLOGY ............................................................. 407
TRIGONOMETRY ..................................................135
COMPUTER & IT ................................................. 428
GEOMETRY ..........................................................156
REASONING ABILITY .................... 190 DISCOVERIES & INVENTIONS .............................. 433
ENGLISH LANGUAGE .................... 435
ANALOGY...............................................................190
ERROR DETECTION ............................................. 435
CLASSIFICATION ..................................................205
IMPROVEMENT OF SENTENCES ........................... 438
SERIES .................................................................217
ACTIVE/PASSIVE VOICE ...................................... 444
ALPHABET TEST ..................................................228
DIRECT/INDIRECT SPEECH .................................. 451
CODING DECODING .............................................240
FILL IN THE BLANKS........................................... 461
BLOOD RELATIONSHIP .........................................249
SYNONYMS ......................................................... 466
DIRECTION AND DISTANCE .................................251
ANTONYMS ......................................................... 470
SEQUENCE & RANKING TEST ..............................254
ONE-WORD SUBSTITUTION ................................. 474
NUMBER PUZZLE .................................................257
IDIOMS/PHRASES................................................. 479
VENN DIAGRAM ..................................................266
SPELLING MISTAKES ........................................... 485
SITTING ARRANGEMENT ......................................271
ARRANGEMENT OF SENTENCES .......................... 489
ARGUMENTS & SYLLOGISM.................................273
CLOZE TEST ........................................................ 500
COMPLETION & EMBEDDED FIGURES ..................281
COMPREHENSION TEST ....................................... 505
FIGURE COUNTING & ANALYSIS .........................284
PAPER CUTTING AND FOLDING ...........................287
MIRROR & WATER IMAGES .................................291
VISUAL & NON VERBAL REASONING ..................295
GENERAL AWARENESS ................ 299

Page 2 of 516
https://telegram.me/aedahamlibra
QUANTITATIVE APTITUDE
Percentage Ans: (d)
3A = B
3A = A ×
Q1. If A's income is 50% less than that of B's, then B's
300 = 2x x = 150
income is what per cent more than that of A?
(a) 125 (b) 100
Q7. If an electricity bill is paid before due date, one
(c) 75 (d) 50
gets a reduction of 4% on the amount of the bill. By
Ans: (b) Required precentage = 100% paying the bill before due date a person got a
reduction of ₹ 13. The amount of his electricity bill
Q2. If 60% of A = of B, then A : B is was
(a) 9 : 20 (b) 20 : 9 (a) ₹ 125 (b) ₹ 225
(c) 4 : 5 (d) 5 : 4 (c) ₹ 325 (d) ₹ 425
Ans: (c) Let the amount of the bill be ₹ x.
Ans: (d)
∴ = 13
x= = ₹ 325

Q8. If 90% of A = 30% of B and B = x% of A, then


the value of x is
Q3. 1.14 expressed as a per cent of 1.9 is
(a) 800 (b) 300
(a) 6% (b) 10%
(c) 700 (d) 400
(c) 60% (d) 90%
Ans: (b) A ×
Ans: (c) Required percentage
A×3=B
A × x% = A × 3
Q4. If A's income is 25% less than B's income, by how
much percent is B's income more than that of A? = 3 x = 300
(a) 25 (b) 30
(c) Q9. If 30% of A is added to 40% of B, the answer is
80% of B. What percentage of A is B?
Ans: (c) Required percentage
(a) 30% (b) 40%
= % (c) 70% (d) 75%
Ans: (d) A ×
Q5. Two successive price increases of 10% and 10%
A × 30 = B × 40
of an article are equivalent to a single price increase
of
(a) 19% (b) 20% = 75%
(c) 21% (d) 22%
Ans: (c) Single equivalent percentage increase in price Q10. The population of a town is 15000. If the number
( ) of males increases by 8% and that of females by 10%,
then the population would increase to 16300. Find the
number of females in the town.
Q6. If 9 0% of A = 30% of B and B = 2x % of A, then
(a) 4000 (b) 6000
the value of x is
(c) 3000 (d) 5000
(a) 450 (b) 400
(c) 300 (d) 150
Page 3 of 516
https://telegram.me/aedahamlibra
Ans: (d) If the number of females be x, then, number of
males = 15000 – x

= 16300 – 15000
10x + 120000 – 8x
= 1300 × 100
2x = 130000 – 120000
= 10000
x = 5000

Q11. First and second numbers are less than a third x = 250
number by 30% and 37% respectively. The second Alternate Method:
number is less than the first by Number persons in both = number of persons in 1st + no.
(a) 7% (b) 4% of persons in 2nd – Total no. of persons in any one
(c) 3% (d) 10% % of persons in both = 72 + 44 – 100 = 16%
Ans: (d) Let the third number = 100.
Total no. of persons × = 40
First number = 70
Second number = 63 Total no. = 250
∴ Required per cent = = 10%
Q15. The salary of an employee increases every year
in the month of July by 10%. If his salary in May
Q12. The number of seats in an auditorium is
2000 was ₹ 15,000, his salary in October 2001 was
increased by 25%. The price of a ticket is also
(a) ₹ 16,500 (b) ₹ 18,000
increased by 12%. Then the increase in revenue
(c) ₹ 18,150 (d) ₹ 19,965
collection will be
Ans: (c) Salary in May 2000 = ₹ 15000
(a) 40% (b) 35%
(c) 45% (d) 48% Salary in July 2000 15000 + 10% of 15000 = ₹16500
Ans: (a) Required increase Salary in October 2001 = 16500 + 10% of 16500 =
₹18150
( ) = 40%
Q16. If 125% of x is 100, then x is :
Q13. The number 20% more than 80 is (a) 80 (b) 150
(a) 36 (b) 30 (c) 400 (d) 125
(c) 90 (d) 96
Ans: (a)
Ans: (d) Required number

Q14. 72% of the students of a certain class took


Biology and 44% took Mathematics. If each student Q17. A team played 40 games in a season and won in
took Biology or Mathematics and 40 t ook both, the 24 of them. What percent of games played did the
total number of students in the class was team win ?
(a) 200 (b) 230 (a) 70% (b) 40%
(c) 250 (d) 320 (c) 60% (d) 35%
Ans: (c) Let the total number of students in the class be Ans: (c) Required percentage =
x.
Q18. A certain amount of money is divided among x, y
and z. If x receives 25% more than y and y receives
25% less than z, then x : y : z is equal to
(a) 12 : 10 : 11 (b) 14 : 12 : 13
(c) 15 : 12 : 16 (d) 10 : 9 : 12
Ans: (c) or or

Page 4 of 516
or or https://telegram.me/aedahamlibra
B = 125
A is larger than C by
= = 20%

Q23. In the annual examination Mahuya got 10% less


marks than Supriyo in Mathematics. Mahuya got 81
Then, x: y: z is equal to 15: 12: 16 marks. The marks of Supriyo are
(a) 89 (b) 90
Q19. In a big garden 60% of the trees are coconut (c) 87 (d) 88
trees, 25% of the number of coconut trees are mango Ans: (b) Marks of Supriyo = x marks
trees and 20% of the number of mango trees are Accoding to question
apple trees. If the number of apple trees are 1500. Mahuya marks = Supriyo marks – 10% of Supriyo marks
then the number of trees in the garden is :
81 = x – 10% of x ( )
(a) 48000 (b) 50000
(c) 51000 (d) 45000 81 =
Ans: (b) If the number of trees in the garden be x, then ∴ x = 90 marks
= 1500
= 1500 Q24. Rakesh got 273 marks in an examination and
scored 5% more than the pass %. If Lokesh got 312
x= = 50000 marks, then by what % above the pass mark d id he
pass the examination?
Q20. A number when reduced by 10% gives 30. The (a) 20% (b) 27%
number is (c) 25% (d) 15%
(a) 35 (b) Ans: (a) Let passing marks be represented by p.
p × 1.05 = 273
(c) 33− (d) 40
p = 260
Ans: (c) Let the number is x.
According to question Lokesh passing
x – 10% of x = 30 = 20%

Q25. A box has 100 blue balls, 50 red balls, 50 black


( ) ball. 25% of blue balls a nd 50% of red balls are taken
away. percentage of black balls at present is
(a) % (b) 40%

Hence, the number is (c) 50% (d) 25%


Ans: (a) After taking away respective balls,
Number of balls in the box
Q21. Two persons contested an election of Parliament.
= 75 + 25 + 50 = 150
The winning candidate secured 57% of the total votes
∴Percentage of black balls
polled and won by a majority of 42,000 votes. The
number of total votes polled is
(a) 4,00,000 (b) 5,00,000
%
(c) 6,00,000 (d) 3,00,000
Ans: (d)
Q26. A sales representative will receive a 15%
Q22. Given A is 50% larger than C and B is 25% commission on a sale of ₹ 2,800. If he has already
large than C, then A is what percent larger than B? received an advance of ₹ 150 on that commission, the
(a) 20% (b) 25% remaining amount of commission is
(c) 50% (d) 75% (a) ₹ 320 (b) ₹ 420
Ans: (a) C = 100 (c) ₹ 120 (d) ₹ 270
A = 150 Ans: (d) Sales representative will receive total amount
Page 5 of 516
https://telegram.me/aedahamlibra
Savings of A = (100 – 85)% of x = 0.15x
Savings of B = (100 – 95)% of (40000 – x)
Remaining amount = 420 – 150 = 270 = 0.05 (40000 – x)
0.15 x = 0.05 (40000 – x)
Q27. In a school, 10% of number of girls is equal to 0.15x + 0.05x = 40000 × 0.05
20th of number of boys. Ratio between the number of 0.2x = 2000
boys to number of girls is x = 10000
(a) 1 : 2 (b) 2 : 1 OR
(c) 1 : 4 (d) 4 : l
Ans: (b) If boys = x and girls = y, then
y× ∴A:B=1:3
Salary of A = 40000 × = 10000

Q31. 1% of 1% of 25% of 1000 is


Q28. If 40% of of - of a number is 48, then what is (a) .025 (b) .0025
1% of the same number ? (c) .25 (d) .000025
(a) 20 (b) 2 Ans: (a)
(c) 10 (d) 1
Ans: (b) = 48 Q32. A number is increased by x%, to get back to the
= 48 original number, it is to be reduced by :

x= (a) % (b) %

1% of 200 is 2. (c) x % (d)


Ans: (b) Cumulative % change = a + b +
Q29. In an e xamination 75% candidates passed in
Cumulative change to be 0
English and 60% passed in Mathematics. 25% failed
in both and 240 passed the examination. Find the total So a + b + =0
number of candidates. Here a = x%
(a) 492 (b) 300 So =0
(c) 500 (d) 400
( ) = –x
Ans: (d) Let the total number of students be x.
Let A and B represent the sets of students who passed in b= =
English and Mathematics respectively. – ve sign means decrease
Then, number of students passed in one or both the
So we need to decrease the number by %
subjects
= n (A ∪ B) = n (A) + n (B) – n (A ∩ B)
= 75% of x + 60% of x – (x – 25% of x) Q33. The height of a triangle is increased by 10%. To
retain the original area of the triangle, its
= =( )
corresponding base must be decreased by:
So, = 240 (a) %
x= (c) 10% (d)
Ans: (c) Let original, area height, and base of triangle is
Q30. The monthly salaries of A and B together a, h and b
amount to ₹ 40,000. A spends 85% of his salary and New area, height and base of triangle is A, H, B
B, 95% of his salary. If now their savings are the H=
same, then the salary (in ₹) of A is
H = 1.1 h
(a) 10,000 (b) 12,000
(c) 16,000 (d) 18,000 Original area (a) =
Ans: (a) Let the monthly salary of A be x,, New area (A) =
monthly salary of B is (40000 – x).
Page 6 of 516
A= https://telegram.me/aedahamlibra
Distributed % =
But A = a Percentage left in the basket = 75%

Q38. The price of rice has increased by 60%. In order


to restore the original price, the new pric e must be
reduced by
B = 0.9b
(a) % (b) %
∴ Corresponding base must be decreased by
(c) 40% (d) 45%
Ans: (b) By using x + y + =0

Q34. In an examination, a student must get 36% Let Price be reduced by = x%


marks to pass. A student who gets 190 marks failed by 60 + x +
35 marks. The total marks in that examination is :
(a) 500 (b) 625
(c) 810 (d) 450 x= (– shows reduction)
Ans: (b) Let total mark of Examination be x.
= 190 + 35 Q39. of % of 800 equals
(a) 100 (b) 200
x = 625 (c) 300 (d) 400
Ans: (b) of 1600 + % of 800
Q35. % of ₹90 is equal to 60% of ?
(a) ₹ 124 (b) ₹ 125
(c) ₹ 123 (d) ₹ 122
Q40. A number is increased by 15% and then
Ans: (b) of 90
decreased by 25% and the number becomes 22 less
= 90 × than the original number. The original number is
(a) 120 (b) 140
60% of x = x
(c) 100 (d) 160
So, 90 × = x Ans: (d)
x=
Q41. In a motor of 120 machine parts, 5% parts were
x= = 125. defective. In another motor of 80 machine parts, 10%
parts were defective. For the two motors considered
Q36. If 35% of A‘s income is equal to 25% of B‘s together, the percentage of defective mac hine parts
income, then the ratio of A‘s income to B‘s income is were
(a) 7 : 5 (b) 5 : 7 (a) 7 (b) 6.5
(c) 4 : 7 (d) 4 : 3 (c) 7.5 (d) 8
Ans: (b) 35 % A‘s Salary = 25% of B‘s Salary Ans: (a) Total deffective part =
= 6 + 8 = 14

or 5 : 7 Deffective % =

Q37. A basket contains 300 mangoes. 75 mangoes Q42. After deducting 60% from a certain number and
were distributed among some students. Find the then deducting 15% from the remainder, 1428 is left.
percentage of mangoes left in the basket What was the initial number?
(a) 70% (b) 72% (a) 4200 (b) 3962
(c) 76% (d) 75% (c) 4150 (d) 4300
Ans: (d) Total mango = 300 Ans: (a) Let initial number be x.
Distribution = 75 According to question,

Page 7 of 516
https://telegram.me/aedahamlibra ( )
∴ = 4200. ∴
= 320,000
Q43. 32% of a number exceeds 17% of the same ∴Population of city 3 years was = 320000.
number by 120. What is the value of the number?
(a) 900 (b) 860 Q47. A person spends 25% of his annual income on
(c) 940 (d) 800 house rent. 15% on education of children and 45% on
Ans: (d) Required number other items. If he saves ₹14,400 annually, then the
person's total income is:
(a) ₹98,000 (b) ₹1,00,000
(c) ₹96,000 (d) ₹1,20,000
∴ Ans: (c) Total spend of his annual income
= (15% + 25% + 45%)
Q44. 80 litre mixture of milk and water contains 10%
milk. How much milk (in lit res) must be added to Q48. If 40% of a number is 290, then what is the
make water percentage in the mixture as 80%? number which is 20% more than the initial number?
(a) 8 (b) 9 (a) 870 (b) 725
(c) 10 (d) 12 (c) 825 (d) 680
Ans: (c) According to question, Ans: (a) Let original number = x
According to question,
Volume of water litres
Volume of milk litres
Now, ∴
∴Required number

640 + 80x = 1440


Q49. What will be the net discount (in percentage)
∴ litres.
after two successive discounts of 40% and 20%?
(a) 60 (b) 68
Q45. If A has got 20% more marks than B, then by (c) 52 (d) 42
what percent marks of B are less than the marks of
Ans: (c) Required net discount ( )%
A?
(a) 16.66 (b) 20 = 60 – 8 = 52%
(c) 33.33 (d) 14.28
Ans: (a) Required percent of marks Proft, Loss & Discount
= 16.66%

Q46. The population of a city increases at the rate of


Q1. If the ratio of cost price and selling price of an
5% per annum. If the present population of the city is
article be as 10 : 11, the percentage of profit is
3,70,440.
(a) 8 (b) 10
It population 3 years ago was:
(c) 11 (d) 15
(a) 2,80,000 (b) 3,60,000
(c) 3,20,000 (d) 30,000 Ans: (b) Gain = 11x – 10x = ₹x
Ans: (c) Present population = 370440 p%
Rate = 5%
Time = 3 years Q2. A shopkeeper earns a profit of 12% on selling a
According to question, book at 10% discount on the printed price. The ratio
fo the cost price and the printed price of the book is
( )
(a) 45 : 56 (b) 45 : 51
Page 8 of 516
(c) 47 : 56
Ans: (a) Let the CP be ₹100.
https://telegram.me/aedahamlibra
(d) 47 : 51 Ans: (d) If the CP of each book be ₹1, then
SP of 20 books = ₹15
∴ SP = ₹112 CP of 20 books = ₹20
If the marked price be ₹x, then ∴ L% = = 25%
90% of x = 112
=₹ Q7. If there is a profit of 20% on the cost price of an
∴ Required ratio article, the percent age of profit calculated on its
selling price will be
= 900 : 1120 = 45 : 56
Shortcut Method: (a) 24 (b)
Marked price = Cost Price × (c) (d) 20
Required ratio = Cost Price : Cost Price Ans: (b) If the CP = ₹100, then SP = ₹120 and gain = ₹
20
= 90 : 112 = 45 : 56
Gain % = = %
Q3. A manufacturer marked an article at ₹50 and
sold it allowing 20% discount. If his profit was 25% Q8. If on a marked price, the difference of selling
then the cost price of the article was prices wit h a discount of 30% and two successive
(a) ₹40 (b) ₹35 discounts of 20% and 10% is ₹ 72, then the marked
(c) ₹32 (d) ₹30 price (in rupees) is
Ans: (c) Marked price = ₹50 (a) 3,600 (b) 3,000
S.P. after discount = 80% of 50 = ₹40 (c) 2,500 (d) 2,400
If the CP of article be ₹x, then Ans: (a) Let the marked price be ₹ x.
∴ In case I, SP = ₹
Single discount equivalent to succe ssive discounts of
= ₹32 20% and 10%.
=( )% = 28%
Q4. By selling an article, a man makes a profit of 25%
∴ S.P. in this case = ₹
of its selling price. His profit per cent is
(a) 20 (b) 25 ∴ = ₹ 72
(c) = 72
Ans: (d) If the S.P. of article be ₹x,
∴x= = ₹ 3600
then its CP = x ₹

% Q9. If an article is sold at 200% profit, then the ratio


of its cost price to its selling price will be
(a) 1 : 2 (b) 2 : 1
Q5. By selling a bicycle for ₹ 2,850, Aa shopkeeper (c) 1 : 3 (d) 3 : 1
gains 14%. If the profit is re duced to 8%, then the Ans: (c) Let CP = 100 P = 200
selling price will be SP = CP + P = 300
(a) ₹ 2,600 (b) ₹ 2,700 = =
(c) ₹ 2,800 (d) ₹ 3,000
Ans: (b) C.P. of bicycle ₹2500 Q10. The price of an article was first increased by
S.P. for a profit of 8% ₹2700 10% and then again by 20%. If the last increased
price be ₹ 33, the original price was
Q6. If the cost price of 15 books is equal to the selling (a) ₹ 30 (b) ₹ 27.50
price of 20 books, the loss percent is (c) ₹ 26.50 (d) ₹ 25
(a) 16 (b) 20 Ans: (d) Net increase percentage
(c) 24 (d) 25 =( )% = 32%

Page 9 of 516
∴x× = 33 https://telegram.me/aedahamlibra
Q15. If the selling price o f 10 articles is equal to the
cost price of 11 articles, then the gain percent is
x= = ₹ 25 (a) 10 (b) 11
(c) 15 (d) 25
Q11. Successive discounts of 10%, 20% and 30% is Ans: (a) Let the C.P. of each article be ₹ 1.
equivalent to a single discount of ∴ C.P. of articles = ₹ 10
(a) 60% (b) 49.6% and S.P. of 10 articles = ₹ 11
(c) 40.5% (d) 36%
∴ Profit percent = = 10%
Ans: (b) Single equivalent discount for successive
discounts of 10% and 20%.
Q16. A trader bought two horses for ₹19,500. He sold
=( ) % = 28%
one at a loss of 20% and the other at a profit of 15%.
Single equivalent discount for 28% and 30% If the selling price of each horse is the same, then their
=( ) % = 49.6% cost prices are respectively.
Shortcut Method: (a) ₹ 10,000 and ₹ 9,500 (b) ₹ 11,500 and ₹ 8,000
Let marked price be 100% (c) ₹ 12,000 and ₹ 7,500 (d) ₹ 10,500 and ₹ 9,000
Final discount = 100 – 100 × Ans: (b) The sum of cost prices of two articles is ₹ x.
One of them is sold at a loss of a% and other is sold at a
Here D1, D2, D3 ........ are successive discounts.
gain of b% and their S.P. is same.
Required value = 100 –
∴ C.P. of article sold at a loss of a%
50.4 = 49.6%.
= =

Q12. What single discount is equivalent to two = = ₹ 11500


successive discounts of 20% and 15%? ∴ C.P. of second article = ₹ 8000
(a) 35% (b) 32% Alternate Method:
(c) 34% (d) 30% C.P of first horse × = C.P of second horse
Ans: (b) Single equivalent discount
C.P of first horse : C.P of second horse = 115 : 80 =
=( ) =( )% = 32% 23:16
C.P of first horse
Q13. A shopkeeper allows a discount of 10% to his
C.P of seond horse = 19500 – 11500 = 8000
customers and still gains. 20%. Find the marked price
of the article which costs ₹ 450.
Q17. Krishna purchased a number of articles at ₹10
(a) ₹ 600 (b) ₹ 540
for each and the same number for ₹ 14 each. He
(c) ₹ 660 (d) ₹ 580
mixed them together and sold them for ₹13 each.
Ans: (a) Let the marked price of the article be ₹ x.
Then his gain or loss percent is
∴x×
(a) Loss (b) Gain
= 540
(c) Loss (d) Gain
x= = ₹ 600
Ans: (a) Average cost of = = 12
QP = 13
Q14. While selling a watch, a shopkeeper gives a
P% = =
discount of 5%. If he gives a discount of 6%, he earns
₹ 15 less as profit. What is the marked price of the
watch? Q18. When the price of sugar decreases by 10%, a
(a) ₹ 1,250 (b) ₹ 1,400 man could buy 1 kg more for ₹ 270. Then the original
(c) ₹ 1,500 (d) ₹ 750 price of sugar per kg is
Ans: (c) Difference in discount = 1% (a) ₹ 25 (b) ₹ 30
(c) ₹ 27 (d) ₹ 32
= 15
Ans: (b) Let the original price of sugar be ₹ x/kg.
x = 1500
∴ New price = ₹ /kg
Page 10 of 516
∴ =1 https://telegram.me/aedahamlibra
Q23. 20% loss on selling pr ice is what per cent loss on
the cost price?
x = ₹ 30/kg (a) 25% (b) 15%
(c) % (d) %
Q19. The cost price of an article is 40% of the s elling Ans: (c) Let SP = 100
price. What percent of the cost price is the selling Loss% on SP = 20%
price? CP = 100 + 20 = 120
(a) 140% (b) 200%
L% of cp =
(c) 220% (d) 250%
Ans: (d) Let the S.P. of the article = ₹ 100
Q24. Marked price of an article is ₹275. Shopkeeper
∴ C.P. = ₹ 40
allows a discount of 5% and he gets a profit of 4.5%.
∴ Required percentage = = 250% The actual cost of the article is
(a) 250 (b) 225
Q20. X sells two articles for ₹ 4,000 each with no loss (c) 215 (d) 210
and no gain in the interaction. If one was sold at a Ans: (a) MP = 275
gain of 25% the other is sold at a loss of SP after Discount of 5% =
(a) 25% (b) %
CP where P % of 4.5 = = ₹250
(c) % (d) 20%
Ans: (c) Cost price of first article Q25. The price of a commodity rises from ₹ 6 per kg
to ₹ 7.50 per kg. If the expenditure cannot increase,
Total cost price of both articles = Total sell ing price of the percentage of reduction in consumption is
both articles = 4000 + 4000 = 8000 (a) 15 (b) 20
Cost price of second articles = 8000 – 3200 = 4800 (c) 25 (d) 30
Selling price of second articles = 4000 Ans: (b) Percentage increase = = 25
Loss on second articles = 4800 – 4000 = 800 ∴ Percentage decrease in consumption
Required % = % = = 20%

Q21. If the price of sug ar is raised by 25%, find by Q26. If the cost price of 15 articles is equal to the
how much percent a householder must reduce his selling price of 12 articles, find gain %
consumption of sugar so as not to increase his (a) 20 (b) 25
expenditure? (c) 18 (d) 21
(a) 10 (b) 20
Ans: (b) Percentage profit = = 25
(c) 18 (d) 25
Ans: (b) Percentage decrease = × 100 = 20
Q27. The difference between a discount of 40% on
₹500 and two successive discounts of 36%, 4% on the
Q22. A reduction of 20% in the price of sugar enables same amount is
me to purchase 5 kg more for ₹ 600. Find the price of (a) ₹ 0 (b) ₹ 2
sugar per kg before reduction of price. (c) ₹ 1.93 (d) ₹ 7.20
(a) ₹ 24 (b) ₹ 30 Ans: (d) Single equivalent discount for 36% and 4%
(c) ₹ 32 (d) ₹ 36
=( ) = (40 – 1.44)% = 38.56%
Ans: (b) Let CP = x, Total ₹ = 600, Sugar bought
∴ Required difference = 1.44% of 500
=
= = ₹7.20
ATQ * + = 600
480 + 4x = 600
4x = 120
x = 30
Page 11 of 516
https://telegram.me/aedahamlibra
Q28. A man purchased some eggs at 3 for ₹ 5 and sold
them at 5 for ₹ 12. Thus he gained ₹ 143 in all. The
Cost price : Marked price

number of eggs he bought is


(a) 210 (b) 200 Shortcut Method:
(c) 195 (d) 190 M.P =
Ans: (c) Let he buy 15 eggs. C.P : M.P = (100 – 10) : (100 + 8) = 90 : 108 = 5 : 6
∴ CP of 15 15 eggs = ₹ 25
∴ SP of 15 eggs = ₹ 36 Q32. A man sold two articles at ₹ 375 each. On one, he
∴ Gain = 36 – 25 = ₹ 11 gains 25% and on the other, he loses 25%. The gain or
₹ 11 ≡15 eggs loss% on the whole transaction is :
∴ ₹ 143 ≡ = 195 eggs. (a) 6% (b)
(c) ₹ 50 (d)
Q29. The cost price of an article is 64% of the marked
Ans: (d) In such type of question,
price. The gain percentage after allowing a discount of
12% on the marked price is Required % loss = %
(a) 37.5% (b) 48% =
(c) 50.5% (d) 52%
Ans: (a) Marked price of article = ₹ 100 (let)
Q33. Rahul bought two cycles for a total sum of ₹
∴ C.P. of article = ₹ 64 1,500. He sold one cylce at 20% loss and the other
∴ S.P. of article = ₹ 88 cycle at 20% gai n. If the selling price of both the
∴ Profit per cent = = 37.5% cycles is the same, find the cost price of the two cycles.
(a) ₹ 500, ₹ 1,000 (b) ₹ 600, ₹ 900
Q30. By selling an artcile for ₹ 21,000, a man gains (c) ₹ 750 each (d) ₹ 550, ₹ 950
5%. To get a profit of 15%, he has to sell it for Ans: (b)
(a) ₹ 19,800 (b) ₹ 20,700 Shortcut Method:
(c) ₹ 23,000 (d) ₹ 25,000 Cost price of first cycle = cost price of second
Ans: (c) C.P. = cycles ×
C.P = ₹ 20000 C.P of first cycle : C.P of second cycle = 2 :3
New profit = 15% C.P of first cycle
New S.P = = ₹ 230000 C.P of second cycle = 1500 – 600 = 900
∴ To get 15% profit he has to sell an article at ₹23000.
Q34. The profit percent of a bookseller if he sells book
Q31. A bookseller makes 8% profit after selling the at marked price after enjoying a commission of 25%
book at 10% discount. The ratio o f the cost price to on marked price will be:
the marked price is (a) 30% (b) 25%
(a) 4 : 5 (b) 5 : 4 (c) 20% (d) %
(c) 5 : 6 (d) 6 : 5 Ans: (d) Let MP = 100
Ans: (c) So, SP = 100 – 25% of 100
= 100 – 25 = 75
So, Profit percent =
= %
Let cost price (C.P.) = ₹ x
& Marked price (M.P.) = ₹ 100 Q35. A bought an article, paying 5% less than the
original price. A sold it with 20% profit on the price
he had paid. What percent of profit did A earn on the
original price ?
(a) 10 (b) 13
Page 12 of 516
(c) 14 https://telegram.me/aedahamlibra
(d)
Q40. If a commission of 10% is given on the marked
Ans: (c) Required % earned by A
price of a work, the publisher gains 20%. If the
=, - commission is increased to 15%, the gain present is:
=, - (a) 15% (b) %
= (114 –100)% = 14% (c) % (d) %
Ans: (c) C.P. of article = ₹ 100
Q36. A sells an article to B at a gain of 10% , B sells it
to C at a gain of 5%. If C pays ₹ 462 for it, what did it Marked price
cost to A ? 85%of
(a) ₹ 500 (b) ₹ 450
Gain %
(c) ₹ 600 (d) ₹ 400
Ans: (d) Cost to A → cost to B→ cost to C Q41. A watch is sold at a profit of 30%. Had it been
Cost to A × sold for ₹ 80 less, there would have been a loss of
Cost to A = 10%. What is the cost price of rupees?
(a) 150 (b) 200
(c) 400 (d) 800
Q37. The printed price of a book is ₹ 320. A retailer Ans: (b) Difference of P% and L% = 30 – (– 10) = 40%
pays ₹ 244.80 for it. He gets successive discounts of
10% and an another rate. His second rate is :
(a) 15% (b) 16% x = 200
(c) 14% (d) 12%
Ans: (a) Printed price = ₹ 320, d1 = 10% Q42. A dealer offered a machine for sale for ₹27,500
Let d2 = x%, Amount actually paid = ₹ 244.80 but even if he had charged 10% less, he would have
So, amount actually paid = ( )( ) made a profit of 10%. The actual cost of the machine
is
(a) ₹22,000 (b) ₹24,250
(c) ₹22,500 (d) ₹22,275
100 – x = = 85 x = 15%
Ans: (c) If the C.P.of machine by ₹x, then

Q38. 'A' sells an article to 'B' at a profit of 20% and


'B' sells it to 'C' at a profit of 25%. If 'C' pays ₹1200,
the cost price of the article originally (in ₹) is
(a) 700 (b) 600 ₹22500
(c) 1,000 (d) 800
Ans: (d) Effective profit percent ( )= Q43. By selling 9 articles for a rupee, a man incurred
50% a loss of 4%. To make a gain of 44%, the number of
∴ Original cost price articles to be sold for a rupee is:
(a) 5 (b) 3
(c) 4 (d) 6
Ans: (d) C.P. of 9 articles
Q39. A discount of 30% on the marked price of a toy
∴ S.P. for a gain of 44 %
reduces its selling price by ₹30. What is the new
selling price (in ₹) ?
(a) 70 (b) 21
∴ Required number of articles
(c) 130 (d) 100
Ans: (a)
∴ 100% ≡ ₹ 100 Q44. A businessman allows a discount of 10% on the
written price. How much above the cost price must he
∴ New S.P. = 100 – 30 = ₹ 70
mark his goods to make a profit of 17%?
Page 13 of 516
(a) 30%
(c) 27%
https://telegram.me/aedahamlibra
(b) 20%
(d) 18%
'–' denotes discount. Hence, single discount equivalent to
79%
Ans: (a) Let CP = ₹100
Then, S.P = ₹117 Q49. A merchant purchases a wrist watch for ₹ 450
Let marked price be Rs x. and fixes its list price in such a way that after allowing
Then, 90% of x = 117 ( ) a discount of 10%, he earns a profit of 20%. Then the
list price of the watch is
∴ Marked price = 30% above C.P.
(a) ₹ 600 (b) ₹ 650
(c) ₹ 700 (d) ₹ 550
Q45. A man makes a profit of 20% on the sale by
Ans: (a) C.P. = ₹ 450; profit = 20%
selling 20 articles for ₹1. The number of a rticles he
bought by ₹1cs ∴ S.P. =
(a) 20 (b) 24 Let the list price of the wrist watch be ₹x.
(c) 25 (d) 30 Then discount = 10% = ₹
Ans: (b) C.P. of 20 articles
∴ S.P. =
∴ Number of articles bought for Re.1.
According to quesiton,

Q46. A trader has a weighing balance that shows x=


1,200 gm for a kilogram. He further marks u p his cost
price by 10%. Then the net profit percentage is Q50. A dishonest grocer sells rice at a profit of 10%
(a) 32% (b) 23% and also uses weights which are 20% less than the
(c) 31.75% (d) 23.5% marked weight. The total gain earned by him will be
Ans: (a) The trader professes to sell 1200 kg but sells (a) 35% (b) 37.5%
only 1000 kg. (c) 40% (d) 30.5%
So profit = 20% Ans: (b) Let us consider a packet or rice marked 1kg. It's
Markup = 10% actualweight is 80% of 1000 gm = 800 gm
Total profit Let C.P. of each gm be ₹1.
Then, C.P. of this packet = ₹ 800
Q47. A man sold an article at a loss of 20%. If he sells S.P. of this packet = 110% of C.P. of 1kg =
the article for ₹ 12 more, he would have gained 10%.
The cost price of the article is
(a) ₹ 60 (b) ₹ 40 Q51. A merchant allows a discount of 10% on marked
(c) ₹ 30 (d) ₹ 22 price for the cash payment. To make a profit of 17%,
he must mark his goods higher than their cost price
Ans: (b) S.P = C.P ( ) S.P C.P ... (1)
by
( ) C.P 12 (a) 30% (b) 33%
... (2) (c) 40% (d) 27%
From eqn. (1) and (2) Ans: (a) Solving this type of question by short cut.
C.P C.P 12 Net profit% =
17% = [∴ '–' for dicount]
27 =
Q48. Two successive discounts of 70% and 30% are 27 × 10 = 9y
equivalent to a single discount of y = 30%
(a) 89% (b) 75% Hence, He must mark his goods 30% higher than their
(c) 79% (d) 100% cost price.
Ans: (c) Single discount =
Q52. The cost price of a radio is ₹ 600. 5% of the cost
= = – 100 + 21 = –79% price is charged towards transportation. After adding

Page 14 of 516
https://telegram.me/aedahamlibra
that, if the net profit to be made is 15%, then the
selling price of the radio must be
= ₹ (126 + 39) = ₹ 165
CP of 100 g = ₹ 16.5
(a) ₹ 684.50 (b) ₹ 704.50 S.P. of 100 g = ₹ 18.15
(c) ₹ 724.50 (d) ₹ 664.50 Profit = ₹ (18.15 – 16.5) = ₹ 1.65
Ans: (c) C.P. of a radio = 600 % gain = = 10%
New C.P. after adding transportation charges
= ( ) Q57. An article was sold at 16% gain. Had it been sold
S.P. = ( ) for ₹ 200 more, the gain would have been 20%. Then
the cost price of the article is :
= (a) ₹ 5000 (b) ₹ 4800
(c) ₹ 4500 (d) ₹ 5200
Q53. Ans: (a) If the C.P. of article be ₹x, then
=
Gain % =
Ans:
Q54. A man sold 250 chairs and had a gain equal to = ₹5000
selling price of 50 chairs. His profit per cent is:
(a) 20% (b) 25% Q58. The cost price of a boo k is ₹ 150. At what price
(c) 50% (d) 15% should it be sold to gain 20% ?
Ans: (b) Percentage profit = (a) ₹ 80 (b) ₹ 120
(c) ₹ 180 (d) ₹ 100
Q55. By selling a fan for ₹ 600, a man loses 10%. To Ans: (c) = ₹180
make a gain of 20%, the selling price of the fan should S.P = ₹180
be
(a) ₹ 800 (b) ₹ 900 Q59. A got 30% concession on the label price of an
(c) ₹ 1000 (d) ₹ 700 article sold for ₹ 8,750 with 25% profit on the price he
Ans: (a) S.P bought. The label price was
(a) 10,000 (b) 13,000
(c) 16,000 (d) 12,000
Ans: (a) Let the labelled price be ₹ x
Now, C.P =
To make a gian of 20%, the S.P. of Fan should be C.P = = ₹ 7000
Now, (1 – 30% concession) label price = C.P
( ) = 7000

S.P = = = 7000
Hence, S.P. should be x=
x = ₹ 10, 000
800.
Q60. A shopkeeper marks the price of an article at ₹
Q56. A shopkeeper blends two varieties of tea costing 80. What will be the selling price, if he allows two
₹ 18 and ₹ 13 per 100 gm in the ratio 7:3. He sells the successive discounts at 5% each ?
blended variety at the rate of ₹ 18.15 per 100 gm. His (a) ₹ 7.2 (b) ₹ 72.2
percentage gain in the transaction is (c) ₹ 72 (d) ₹ 85
(a) 8% (b) 10%
Ans: (b) Net discount = ( )% = %
(c) 12% (d) 14%
Ans: (b) CP of 1000 gm tea = 18 × 7 + 13 × 3 ∴ S.P. = 80 × = ₹72.2

Page 15 of 516
https://telegram.me/aedahamlibra
Q61. If books bought at prices ranging from ₹ 150 to
∴ Gain percent = = 19.6%

₹ 300 are sold at prices ranging from ₹ 250 to ₹ 350, Q65. On selling an article for ₹ 170, a shopkeeper
what is the greatest possible profit that might be made loses 15%. In order to gain 20%, he must sell that
in selling 15 books ? article at rupees :
(a) ₹ 3,000 (b) Cannot be determined (a) 210 (b) 215.50
(c) ₹ 750 (d) ₹ 4,250 (c) 212.50 (d) 240
Ans: (a) Least cost price = ₹ (150×15) = ₹2250
Ans: (d) C.P. of article = = ₹200
Greatest selling price = ₹ (350×15) = ₹5250
Required profit = ₹ (5250 – 2250) ₹3000 ∴ Required S.P. = = ₹240

Q62. Which of the following successive discounts is Q66. A fruit seller buys some oranges at the rate of 4
better to a customer? for ₹ 10 and an equal number more at 5 for ₹ 10. He
(A) 20%, 15%, 10% or sells the whole lot at 9 for ₹ 20. What is his loss or
(B) 25%, 12%, 8% gain percent?
(a) (A) is better (b) (B) is better (a) Loss percent % (b) Gain percent %
(c) (A) or (B) (both are same) (d) None of these
(c) No loss or no profit (d) Loss percent 2%
Ans: (b) (a) Net discount for 20% and 15%
Ans: (a) Let 20 apples of each type be bought.
=( )% = 32% C.P. of 40 apples
Net discount for 32% and 10% =( ) = ₹ 90
=( ) = 38.8%
Total S.P. = =₹
(b) Net discount for 25% and 12%
Loss = 90 –
=( ) = 34%
Net discount for 34% and 8% ∴ Loss per cent = = %
=( )% = 42 – 2.72 = 39.28% Alternate Method:
Cost price of 2 apples 1 of each typle =
Q63. The marked price of a mixie is ₹ 1600. The Selling price of 2 apples
shopkeeper gives successive discount of 10% and x%
Loss =
to the customer. I f the customer pays ₹1224 for the
mixie, find the value of x : Required % = %
(a) 8% (b) 10% Or, C.P : S.P =
(c) 12% (d) 15%
Loss % =
Ans: (d) First discount = = ₹160
Price after it = 1600 – 160 = ₹1440
Q67. A reduction in the price of apples enables a
∴ = 1440 – 1224 = 216 person to purchase 3 apples for ₹ 1 instead of ₹ 1.25.
∴x= = 15% What is the % of reduction in price (approximately)?
(a) 20 (b) 25
Q64. A retail er purchased radiosets at the rate of ₹ (c) 30 (d)
400 each from a wholesaler. He raised the price by Ans: (a) Percentage decrease
30% and then allowed a discount of 8% on each set. = × 100 = 20%
His profit will be
(a) 19% (b) 78.4%
Q68. A dozen pairs of socks quoted at ₹ 180 are
(c) 22% (d) 19.6%
available at discount of 20%. How many pairs of
Ans: (d) Marked price of a ratio set
socks can be bought for ₹ 48?
= = ₹ 520
(a) 2 pairs (b) 5 pairs
S.P. = = ₹ 478.4 (c) 3 pairs (d) 4 pairs

Page 16 of 516
Ans: (d) https://telegram.me/aedahamlibra
(c) ₹ 750 (d) ₹ 775
Ans: (a) Let the marked price be ₹ x.
S.P. of a dozen pairs of socks S.P. = (x – 25% of x) =
But, S.P = ₹ 525
∴ S.P. of 1 pair of socks ∴
∴ No of pairs available for
Q73. Kabir buys an article with 25% discount on its
₹ 48 =
marked price. He makes a profit of 10% by selling it
at ₹ 660. The marked price is
Q69. An article is sold for ₹ 300 at a profit of 20%. (a) ₹ 600 (b) ₹ 685
Had it been sold ₹ 235, the loss percentage would have (c) ₹ 700 (d) ₹ 800
been
Ans: (d) Let the marked price be ₹x.
(a) 5 (b) 6
(c) 16 (d) 3 = (x – 25% of x) =

Ans: (b) C.P. of the article = ₹ 250 ( )


On selling at ₹ 235,
But, x = 800.
Loss per cent
Q74. A shopkeeper marks his goods 20% above his
Q70. The marked price of a radio set is ₹480. The cost price and gives 15% discou nt on the marked
shopkeeper allows a discount of 10% and gains 8%. If price. His gain percent is
no discount is allowed, his gain percent would be (a) 5% (b) 4%
(a) 18.5% (b) 20% (c) 2% (d) 1%
(c) 25% (d) 18% Ans: (c) If the C.P. of goods be ₹100, then
Ans: (b) If the C.P. of radio be ₹ x, Marked price = ₹120
then
∴ S.P. = = ₹102
∴ Profit per cent = 2%

Q75. A CD was sold at a profit of %. If it had


If no discount is allowed,
been sold at a profit of 15%, it would have gained him
Gain = 480 – 400 = ₹ 80
₹ 10 more. The cost prices of CD is (in ₹)
Gain per cent (a) 450 (b) 500
(c) 400 (d) 550
Q71. The marked price of a table is ₹ 12,000. If it was Ans: (c) Ist case :
sold for ₹10,500 after allowing a certain discount,
then the rate of discount is
(a) 12.5% (b) 15% ( )
(c) 17.5% (d) 10%
Ans: (a) Discount = 12000 – 10500 = ₹ 1500
CP... (1)
If the discount per cent be x,
IInd case :
Then

Q72. On the eve of Gandhi Jayanti, Gandhi Ashram C.P... (2)


declared a 25% discount on silk. If selling price of a
Dividing equation (1) by (2)
silk saree is ₹ 525, what is its marked price ?
(a) ₹ 700 (b) ₹ 725

Page 17 of 516
( ) https://telegram.me/aedahamlibra
(a) 100
(c) 400
(b) 200
(d) 500
115 S.P = 112.5 SP + 1125 Ans: (c) C.P. of article = ₹ x
S.P = 450
∴ First S.P. = =₹
∴ Case II,
Alternate Method:
Let cost price be 100%
1st selling price = 100 + 12.5 = 112.5% = 100
2nd selling price = 100 + 15 = 115%
Difference = 115 – 112.5 = 2.5% = 100
5x = 2000
x= = ₹ 400

Q76. The list price of an article is ₹160 and a


Q80. Aman sells two watches at ₹ 99 each. On one he
customer buys it for ₹122.40 after two successive
gets 10% profit and on the other he loses 10%. His net
discounts. If the first discount is 10%, then second
gain or loss percent is
discount is (a) loss of 1% (b) no profit no loss
(a) 12% (b) 10%
(c) profit of 10% (d) loss of 10%
(c) 14% (d) 15%
Ans: (a) Cost price of watch on which he get 10% Profit,
Ans: (d) S.P. after a discount of 10% = = ₹ 144
Second discount = 144 – 122.40 = ₹ 21.6 Cost Price of watch on which he losses 10%,
If the second discount be x %, then
= 21.6 x= = 15%
Net loss%
Q77. A shopkeeper earns a profit of 12% on selling a
book at 10% discount on printed price. The ratio of
the cost price to printed price of the book is
(a) 45 : 56 (b) 50 : 61 Q81. A bookseller sells a book at a profit of 10%. If he
(c) 90 : 97 (d) 99 : 125 had bought it at 4% less and sold it for ₹ 6 more, he
Ans: (a) C.P. of the book = ₹ x
would have gained %. The cost price of the book
Printed price = ₹ y
is
∴ (a) ₹ 160 (b) ₹ 170
(c) ₹ 150 (d) ₹ 155
Ans: (c) Let C be the cost price of book
Q78. The price of an article is first decreased by 20% Selling price, S = C +
and then increased by 30%. if the resu lting price is ₹ If cost price is 6% less, C′ = C –
416, the original price of the article is.
S′ = 1.1C + 6
(a) ₹ 350 (b) ₹ 405
(c) ₹400 (d) ₹ 450
Ans: (c) If the original price of article be ₹ x, then
x× = 416
x= = ₹ 400
14C + 600 = 18C
Q79. A tradesmen sold an article at a loss of 20%. if 4C = 600
the selling price had been increased by ₹100. there C = ₹150
would have been a gain of 5%. The cost price of the Alternate Method:
article (in ₹) was Let Cost price of book be x

Page 18 of 516
First S.P = https://telegram.me/aedahamlibra
(c) ₹ 180 (d) ₹ 200
Ans: (b) Let M be the marked price.
New Cost price =
New Selling price =

Cost Price, C = 1800 – 200 = 1600


If no discount is given
Profit ₹ 2000 – ₹ 1600 = ₹ 400
Q82. A cyc le merchant allows 25% discount on the
marked price of the cycles and still makes a profit of Q85. Rita purchased a car with a marked price of ₹
20%. If he gains ₹ 360 over the sale of one cycle, find 2,10,000 at a discount of 5%. If the sales tax charged
the marked price of the cycle. is 10%, find the amount she has to pay.
(a) ₹ 2,920 (b) ₹ 2,800 (a) ₹ 2,19,500 (b) ₹ 2,19,000
(c) ₹ 2,880 (d) ₹ 2,900 (c) ₹ 2,19,450 (d) ₹ 2,20,000
Ans: (c) Let M be the mar ket price and C be the cost Ans: (c) Selling price of car; S.P. = 2, 10,000 –
price of the cycle.
Selling price, or = 1,99,500
Sales tax charged is 10%
Total cost for Rita = 1,99,500 +
= ₹ 2, 19, 450

Q86. Mahesh earned a profit of 20% by selling 60


apples at the rate of ₹ 42.50 for 5 apples. Then the
total cost, at which the apples were bought is
(a) ₹ 452 (b) ₹ 425
Also, (c) ₹ 450 (d) ₹ 485
Ans: (b) Selling price of 5 apples = ₹ 42.50
Selling price of 60 apples =
[ ] C.P + Profit = S.P
M = 360 × 4 × 2 = ₹ 2,880 C.P +
Alternate Method:
C.P. = = ₹ 425
Cost price =
Marked price Q87. A tea -merchant professes to sell tea at cost price
= but uses a false weight of 900 gram for a kilogram.
The profit percent in his transaction is
Q83. If a person lost 8% by selling an article for ₹ (a) % (b) 10%
1,035, he bought the article for (c) % (d) 15%
(a) ₹ 1,135 (b) ₹ 1,152
(c) ₹ 1,105 (d) ₹ 1,125 Ans: (a) Profit % = %

Ans: (d) Person bought the article for =₹


Q88. The marked price of a saree is ₹ 200. After
1,125
allowing a discount of 20% on the marked price , the
shopkeeper makes a profit of ₹ 16. Find the gain
Q84. A shopkeeper sold an item for ₹ 1,800 at a
percent.
discount of 10% and gained ₹ 200. Had he not given
(a) % (b) %
the discount, his gain would be
(a) ₹ 300 (b) ₹ 400 (c) 11% (d) 8%
Ans: (a) Selling price = Marked price – Discount
Page 19 of 516
= 200 – 20% of 200 = 160
Cost Price = 160 – 16 = 144
https://telegram.me/aedahamlibra
Q92. A dealer of scientific instruments allo ws 20%
Gain% = % discount on the marked price of the instruments and
still makes a profit of 25%. If his gain over the sale of
Q89. A retailer buys a sewing machine at a discount of an instrument is ₹ 150, find the marked price of the
instrument.
15% and sells it for ₹1955. Thus he makes a profit of
(a) ₹ 938.50 (b) ₹ 940
15%. The discount is
(a) ₹ 270 (b) ₹ 290 (c) ₹ 938 (d) ₹ 937.50
(c) ₹ 300 (d) ₹ 310 Ans: (a) Let marked price of the instrument be ₹ x
Ans: (c) Let original price of sewing machine be ₹ x Selling price, S.P. = x – x = 0.8x
Retailer bought it at = 0.85x Cost price, C.P. = C.P. + C.P. = 0.8x
= 1955 C.P. = x
1.15 × 0.85 x = 1955 x= C.P.
x= = 2000
Given that C.P = 150
Discount is = ₹ 300
C.P. = = 600
Marked price x = = ₹ 938.50
Q90. A man sold his watch at a loss of 5%. Had he
sold it for ₹ 56.25 more, he would have gained 10%.
What is the cost price of the watch (in ₹) ? Q93. A shopkeeper allows 10% discount on goods
(a) 370 (b) 365 when he sells without credit. Cost price of his goods is
(c) 375 (d) 390 80% of his selling price. If he sells his goods by cash,
Ans: (c) Let Cost Price of watch be ₹ x then his profit is
(a) 50% (b) 70%
S.P = x – = .95 x
(c) 25% (d) 40%
If S.P = 0.95x + 56.25 then profit = 10% Ans: (c) Let marked price of goods be ₹ 100.
Selling price of goods = = ₹ 90
Cost price of goods is 80% of its selling price
C.P. = = 72
Profit on goods = (90 – 72) = ₹ 18
Profit % = = 25%

Q94. A sold an article to B at 20% profit and B sold it


Q91. The marked price of an item is twice the cost to C at 15% loss. If A sold it to C at the selling price of
price. For a gain of 15%, the discount should be B, then A would make
(a) 7.5% (b) 20.5% (a) 5% profit (b) 2% profit
(c) 32.5% (d) 42.5% (c) 2% profit (d) 5% loss
Ans: (d) Marked Price, M = 2C, where C is cost price for Ans: (b) Let ₹ 100 be the cost price for A.
15% gain, S.P. = S.P. for A = 100 + 20% of 100 = 120
Let discount be x% S.P. for B = 120 – 15% of 120 = 102
2C – × 2C = 1.15C x = 42.5% Profit % = = 2%
Alternate Method:
Let Cost Price be 100 Q95. Ram bought a T.V. with 20% discount on the
Marked price = 100 ×2 = 200 labelled price. Had he bought it with 30% discount he
Selling price = 115 would have saved ₹ 800. The value of the T.V. set that
Discount = 200 – 115 = 85 he bought is
% of Discount = (a) ₹ 5,000 (b) ₹ 8,000
(c) ₹ 9,000 (d) ₹ 10,000
Page 20 of 516
https://telegram.me/aedahamlibra
Ans: (b) Let labelled price of T.V. be ₹ x Q99. A man bought a watch at 25% discount on the
original price. He got ₹ 40 more than the original
Price after 20% discount, x – = 0.8x
price by selling it at 140% of the price at which he
Price after 30% discount, x – = 0.7x
bought. The price of buying the watch was:
According to question (a) ₹ 900 (b) ₹ 600
0.8x – 0.7x = 800 (c) ₹ 800 (d) ₹ 700
x = 800 × 10 = 8000 Ans: (b) CP = 0.75 × MP
SP = MP + 40
Q96. If a shopkeeper purchases cashewnut at ₹ 250 1.4 CP = M.P + 40
per kg and sells it at ₹ 10 per 50 grams, then he will 1.4 (0.75) MP = M.P + 40
have: 1.05 MP = M.P. + 40
(a) 25% profit (b) 20% profit (1.05 – 1) MP = 40
(c) 20% loss (d) 25% loss M.P. = = ₹ 800
Ans: (c) C.P. = ₹ 250
∴ CP = 0.75 M.P. = 0.75 × 800 = ₹ 600
S.P. = ₹ 10 per 50 g = = ₹ 200
∴ Loss = Q100. A fruit seller buys oranges at the rate of ₹ 10
per dozen and sells at the r ate of ₹12 per dozen. His
Q97. A trader marks his goods 20% above C.P. but gain percent is :
allows his customers a discount of 10. The C.P. of a (a) 15% (b) 20%
blackboard, which is sold for ₹ 216, is: (c) (d) 12%
(a) ₹ 200 (b) ₹ 180 Ans: (b) CP = = ₹ 0.833
(c) ₹ 108 (d) ₹ 196
Ans: (a) Marked price = 20 % above CP SP = =₹1
= So, Gain% = × 100
Discount = 10% = × 100 = 20%.
∴S.P. =
= 0.9 × 1.2 × CP = 1.08 CP Q101. By selling an article for ₹450. I lose 20%. For
S.P. = ₹ 216 what amount, should I sell it to gain 20%?
C.P. = = ₹200 (a) ₹490 (b) ₹ 470
(c) ₹ 562.50 (d) ₹ 675
Ans: (d) SP = ₹ 450
Q98. A fruit seller buys 240 apples for ₹ 600. Some of
Loss = 20%
these apples are bad and are thrown away. He sells
the remaining apples at ₹ 3.50 each and makes a
profit of ₹ 198. The % of apples thrown away are : CP = 450 × = 450 × = ₹ 562.50
(a) 8% (b) 7% SP for getting 20% gain = 562.50 × = ₹ 675.
(c) 6% (d) 5%
Ans: (d) C.P = ₹ 600 Q102. Successive discounts of 20 % and 10% are
Let number of apples thrown = x equivalent to a single discount of :
So number of apples left = 240 – x (a) 28% (b) 25%
So S.P. = (240 – x) 3.50 (c) 30% (d) 15%
= 840 – 3.5x Ans: (a) Equivalent discount of 2 successive discounts
So840 – 3.5 x – 600 = 198
=A+B– = 20 + 10 –
240 – 3.5 x = 198
= 30 – 2 = 28%
x=
So % age of apples thrown = Q103. A house was sold for ₹ y by giving a discount of
x%, then the list price was :
(a)

Page 21 of 516
(c) https://telegram.me/aedahamlibra
kg. If he mixed the two brand of rice and sold the
mixture at ₹80.50 per kg. his gain is
Ans: (a) Let list price = ₹ z (a) ₹510 (b) ₹525
So, Sale price (y) = =z= (c) ₹485 (d) ₹450
Ans: (a) Solving by alligation
Q104. After allowing a discount of 20%, a radio is
available for ₹ 1200. Its marked price was :
(a) ₹ 1500 (b) ₹ 1800
(c) ₹ 1400 (d) ₹ 1550
Ans: (a) MP = SP × x=

= = 300 × 5 = ₹ 1500 Hence, cost price = 70.3 × 50 = ₹ 3515


Selling price = 80.5 × 50 = ₹ 4025
Required gain = 4025 – 3515 = 510
Q105. A man purchased an article for ₹ 1500 and sold
it at 25% above the cost p rice. If he has to pay ₹ 75 as
Q109. Allowing 20% and 15% successive discounts,
tax on it, his net profit percentage will be :
the selling price of an article becomes ₹3,060; then the
(a) 25% (b) 30%
marked price will be
(c) 15% (d) 20%
(a) ₹4,400 (b) ₹5,000
Ans: (d) CP = ₹ 1500
(c) ₹4,500 (d) ₹4,000
SP = 1500 × = ₹ 1875 Ans: (c) S.P. of a n article = 20% and 15% successive
Tax paid = ₹ 75 discount
So, actual SP = 1875 – 75 = ₹1800 × marked price of an article
Net profit = × 100 = 20% 3060 = marked price of an article
∴ Marked of an article
Q106. The marked price of a watch was ₹720. A man
bought the same for ₹550.80 after getting two
successive discounts, the first being 10%. The second
discount rate is Q110. Find a simple discount equivalent to a discount
(a) 12% (b) 14% series of 10%, 20% and 25%
(c) 15% (d) 18% (a) 45% (b) 55%
Ans: (c) Let the second discount be x%. Then (c) 52% (d) 46%
(100 – x)% of 90% of 720 = 550.80 Ans: (d) → → →
Required single discount = (100 – 54)% = 46%

Q111. The difference between successive discounts of


40% followed by 30% and 45% followed by 20% on
the maked price of an article i s ₹ 12. The marked
price of the article is:
Q107. 10% discount and then 20% discount in
(a) ₹400 (b) ₹200
succession is equivalent to total discount of
(c) ₹800 (d) ₹600
(a) 15% (b) 30%
Ans: (d) 1st successive discount final rate
(c) 24% (d) 28%
Ans: (d) Successive discount can be given by = = =
= – 70 + 12 = – 58%
2nd successive discount final rate
= = –30 + 2 = 28%
= = –65 + 9 = – 56%
Hence, the successive dicount in equal to 28%
Let mare price be MP
Q108. A shopkeeper bought 30 kg of rice at the rate of then
₹70 per kg and 20 kg of rice at the rate of ₹70.75 per
Page 22 of 516
https://telegram.me/aedahamlibra
(a) 60%
(c) 49%
(b) 47%
(d) 40%
MP = ₹ 600 Ans: (c) Successive discount = 15, 20, 25
net discount when, 15, 20 taken together
Q112. An article which is marked ₹ 975 is sold for = 35 – 3 = 32
₹897. The discount % is ?
Now taking 22 and 25
(a) 6% (b) 10%
(c) 12% (d) 8%
Ans: (d) Now, Let discount be x%
= 897
57 – 8 = 49%
100 – x =
x= = Q117. 115The marked price of a ceiling fan is ₹ 1200
and the shopkeeper allows a discount of 5 % on it.
Q113. Cost price of 100 books is equal to the selling Then selling price of the fan is
price of 60 books. The gain or loss percentage will be : (a) ₹ 1410 (b) ₹ 1400
(a) % (b) % (c) ₹ 1140 (d) ₹ 1104
Ans: (c) Marked price = 1200, Discount % =
(c) 66% (d) % 5% Selling price = ?
Ans: (a) Now according to question. Selling price 1140
100 × CP = 60 × SP

Q118. A dealer marks a washing machine for ₹ 7500,


Both sides substract 1 and allows a discount of 6% on it. Find the selling
price
(a) 6850 (b) 7050
Percentage loss = %
(c) 7250 (d) 6950
Ans: (b) Market Price = 7500,Discount = 6%
Q114. The selling price of 6 bananas is equal to the
Selling Price = = 7050 ₹
cost price of 8 bananas. Then the per centage of profit
is
Q119. If the ratio of cost price and selling price be
(a) 20 (b)
10:11, then the profit percentage is
(c) 25 (d) 30 (a) 1% (b) 10%
Ans: (b) Let CP of 8 banana = ₹ 8 (c) 5% (d) 8%
CP of 6 banana = ₹ 6 Ans: (b) Let CP = 10, SP = 11
Sp of 6 banana = ₹ 8
P%= = 10%
Profit % = %
Q120. A man sells an article at 5% above the cost
Q115. If the successive discounts be 20% , 10% and price. If he had bought it at 5% less than what he paid
5%, then the single equivalent rate of discount is for it and sold it for ₹ 2 less, he would have gained
(a) 31.6% (b) 31.5% 10%. The cost price of the article is
(c) 31% (d) 31.4% (a) ₹ 250 (b) ₹ 400
Ans: (a) For 3 discont 20%, 10% and 5% (c) ₹ 350 (d) ₹ 200
Now take 20% and 10% Ans: (b) Let C.P. = x
= 30 – 2 = 28% then S. P.
Now take 28 % and 5%
If new C.P.
= 33 – 1.4 = 31.6%
then S.P.
Q116. The successive discount of 15%, 20% and 25% Profit = 10% of
on an article is equivalent to the single discount of
Page 23 of 516
Profit = SP – CP https://telegram.me/aedahamlibra
( )
Q125. A merchant purchase a wrist watch for ₹1,200
and fixes its list price in such a way that after allowing
95x = 100 x – 2000 a discount of 10%, he earns a profit of 20%. The list
– 5x = – 2000 price of the watch is
x = ₹400 (a) ₹ 1, 600 (b) ₹ 12,000
C.P. = ₹400 (c) ₹ 1, 400 (d) ₹ 1, 800
Ans: (a) Marked price of the wrist watch = ₹x
Q121. Loss of 20% on selling price is equal to x% loss
in cost price. What is x?
(a) 20% (b) 20
(c) % (d) 16
x = ₹1600
Ans: (c) Let Selling Price = 100Loss = 20%
Cost price = 120
Q126. After two successive discount of 20% and 35%,
Loss% of cost price = = % an article is sold for ₹ 50700. What is the marked
price (in ₹) of the article?
Q122. By selling an article at of selling price, a trader (a) 92500 (b) 98500
incurred a loss of 10%. The profit/loss percentage, (c) 97500 (d) 94000
when it is sold at the original selling prices, is Ans: (c) Let the mark price be x.
(a) 120% profit (b) 32.5% loss after first discount, price
(c) 20% loss (d) 20% profit after the second discount, price
Ans: (d) Let selling price be 100
Now selling price =
Loss = 10% According to question,

Cost price
Profit at original price = ∴ = 97500.
% Profit = ∴marked price of article is ₹. 97500.

Q123. A trader lists his article 20% above the cost Q127. For an article the profit is 170% of the cost
price and allows a discount of 10% on cash payment. price. If the cost price increases by 20% but the
His gain percent is selling price remains same, then what is the new profit
(a) 6% (b) 10% percentage?
(c) 5% (d) 8% (a) 41 (b) 50
Ans: (d) Let C.P. = ₹100 (c) 75 (d) 125
Ans: (d) Let cost price of an article = 100
Then, M.P. = ₹120
∴Profit = 170
S.P. = 90% of ₹20 = ₹108
∴S.P. = 100 + 170 = 270
Gain = 8%
Now,
Cost price increased by 20%, then
Q124. After a discount of 34% an article is sold for ₹
Cost price = 120
3168. What is the marked price (in ₹) of the article?
S.P. = 270
(a) 4750 (b) 4800
∴Profit = 270 – 120 = 150
(c) 4850 (d) 5000
∴Profit percentage
Ans: (b)
= 125%
Page 24 of 516
https://telegram.me/aedahamlibra
Q128. The marked price of a sofa set is ₹ 4800 which
Q131. By selling 175 pineapples, the gain is equal to
the selling pric e of 50 pineapples. What is the gain
is sold at ₹ 3672 at two successive discounts. If the percentage?
first discount is 10%, then what will be the second (a) 28 (b) 30
discount (in%)? (c) 32 (d) 40
(a) 13 (b) 14 Ans: (d) Let S.P. of each pineapple = Re 1
(c) 15 (d) 17 ∴Gain = 50, SP = 175
Ans: (c) First discount ∴CP = (175 – 50) = 125
Price after it = 4800 – 480 = 4320 ∴Required percentage
∴ = 40%

∴ Q132. Marked price of an item is Rs 500. On purchase


∴Second discount = 15% of 2 items discount is 8%, on purchase of 3 items
discount is 16%. Radha buys 5 items, what is the
Q129. If the price of pen decreases by 20%, then a effective discount?
man can buy 10 more pens for 100. What is the new (a) 20.4 percent (b) 23.25 percent
price of each pen? (c) 12.8 percent (d) 35 percent
(a) 1 (b) 2 Ans: (c) Marked price of each item = 500
(c) 4 (d) 5 No. of items = 5
Ans: (b) Let the cost price of pen = x ∴Total marked price = 500 × 5 = 2500
∴Number of pens he can purchase for ₹ 100 with the Total discount
actual price of pen
Number of pens he can purchase for ₹ 100 with the ∴Effective discount
reduced price of pen
= 12.8%
( )

According to question, Q133. A person bought pens at 25 for a rupee and


sold at 15 for a rupee. What is his profit percentage?
(a)
(c) (d) 40
10 x = 25 Ans: (c) According question,
∴ C.P of a pen

∴New price of pen S. P of a pen


∴profit =
Q130. The marked price of an article is 20% more ∴Profit percentage
than its cost price. If 5% discount is given on the
maked price, then what is the profit percentage? %
(a) 5 (b) 14
(c) 15 (d) 25 Q134. A shopkeeper by selling 13 Titan watches,
Ans: (b) Let C.P of an article = 100 earns a profit equal to the selling price of 3 Titan
∴M.P of an article = 120 watches. His profit percentage is
After discount 5% on marked price, then (a) 30 percent (b) 23 percent
S.P of an article = 120 × 0.95 = 114. (c) 46 percent (d) 16 percent
∴Profit = 114 – 100 = 14 Ans: (a) Let selling price of each watches = ₹ 1
∴Profit percentage ∴Selling price of 13 watches = 13
Profit = 3 × selling pric of watches
= 3 × 1 = 3.
∴Cost price of 13 wateres = (13 – 3) = 10

Page 25 of 516
∴Profit percentage https://telegram.me/aedahamlibra
at ₹40 per kg and sells all the mangoes at ₹45 per kg.
The profit percentage of the fruit seller is:
Q135. The price of an article is cut by 33%, to restore (a) (b) 12.5
to its original value, the new price must be increased (c) 8 (d) 25
by Ans: (c) Total CP of the mangoes = (100 × 45) + (200 ×
(a) 33 percent (b) 49.25 percent 40)
(c) 24.81 percent (d) 41.25 percent = 4500 + 8000 = 12500
Ans: (b) Let the price of the article = ₹ 100 Total SP of the mangoes = (300 × 45) = 13500
New Price = 100 – 33 = 67 ∴Required profit percentage
Therefore the new price must be increased by
%
= 8%
Q136. A man sells an article at a loss of 10%. If he
had sold it for ₹75 more he would have gained 20%. Q139. A fan is listed at ₹150/- with a discount of 20%.
The cost price of the article is (in ₹): What additional discount must be offered to the
(a) 225 (b) 300 customer to bring the net price to ₹108/- ?
(c) 250 (d) 150 (a) 15% (b) 5%
Ans: (c) Let CP of an article = x (c) 10% (d) 20%
According to question, Ans: (c) M.P. of a fan = 150
Discount = 20%
Price of a fan after 20% discount
= ₹ 120
Let x% is additional discount
3x = 750
Then,

∴The cost price of an article = ₹ 250.

Q137. A merchant buys 20 kgs of a variety of rice at


₹14 per kg and another 40 kgs of rice at ₹10 per kg. ∴
He mixes them and sells 1/3 of the mixture at ₹12.50 ∴Additional discount = 10%
per kg. At what rate should be sell the remaining
mixture so as to earn a profit of 25% on the whole Q140. At what percent above the cost price must a
outlay? person mark the price of an article so that he can
(a) ₹12/- (b) ₹15/- enjoy 20% profit after allowing 20% discount?
(c) ₹12.50/- (d) ₹13/- (a) 60% (b) 30%
Ans: (b) CP = (20 × 14 + 40 × 10) (c) 50% (d) 40%
(280 + 400) = 680 Ans: (c) Let C.P = ₹ 100 Then S.P = ₹ 120
Profit = 25% Let marked price be x
Then, 80% of x = 120
∴ ( )
∴Marked price = 50% above C.P.
SP of part of mixture = 20 × 12.5 = 250
∴SP of remaining mixture = (850 – 250) = 600 Q141. A shopkeeper sold a TV set for ₹17940 with a
discount of 8% and earned a profit of 19.6%. What
∴SP of remaining mixture per kg 15per kg.
should have been the percentage of profit earned if no
discount was offered?
Q138. A fruit seller buys 100 kg of superior variety of (a) 23.07% (b) 24.6%
mangoes at ₹45 per kg and 200 kgs of inferior variety (c) 24.05% (d) 30%
Ans: (d) According to question,
Page 26 of 516
https://telegram.me/aedahamlibra
Selling price of the TV set = ₹ 17940
% discount = 8% and % gain = 19.6%
(c)
Ans: (c) Let the numbers be 3x and 3y.
Let the cost price be ₹ 100 ∴ 3x + 3y = 36
Gain = ₹ 19.6 x + y = 12... (i)
∴SP = 100 + 19.6 = 119.6 and 3xy = 105... (ii)
If SP is 119.6, CP = 100 Dividing equation (i) by (ii), we have
ifSP is 17940, CP 15000
Now, % discount = 8%
Marked price
if no discount is given, the selling price = 19500 Shortcut Method:
∴Profit = 19500 – 15000 = 4500
Therefore,
%Profit
Q2. The H.C.F. and L.C.M. of two numebrs are 8 and
So,the gain percent is = 30%
48 respectively. If one of the numbers is 24, then the
other number is
Q142. A man bought 15 mangoes for a rupee. How (a) 48 (b) 36
many mangoes were sold for a rupee so that there is a (c) 24 (d) 16
loss of 25%? Ans: (d) p × q = HCF × LCM
(a) 10 (b) 12
∴ Second number
(c) 18 (d) 20
Ans: (d) CP of a mango
Q3. A number, when divided by 114, leaves remainder
Loss = 25%
21. If the same number is divided by 19, then the
∴ remainder will be
So,A man sold 20 mangoes for a rupee. (a) 1 (b) 2
(c) 7 (d) 17
Q143. An article is sold for ₹ 6552 after a discount of Ans: (b) If the first divisor is a multiple of second
22%. What is the marked price (in ₹) of the article? divisor. Then, remainder by the second divisor.
(a) 8450 (b) 8425 ∴ Remainder = 21 ÷ 19 = 2
(b) 8400 (d) 8750
Ans: (c) Let mark price be x. Q4. The ninth term of the sequence 0, 3, 8, 15, 24, 35,
According to question, .... is
(a) 63 (b) 70
(c)
80 (d) 99
Ans: (c) 0 + 3 = 3
3+5=8
∴ 8 + 7 = 15
∴Marked price of the article = ₹ 8400. 15 + 9 = 24
24 + 11 = 35
35 + 13 = 48
Number System & LCM 48 + 15 = 63
63 + 17 = 80
HCF
Q5. The sixth term of the sequence 2, 6, 11, 17, ..... is
(a) 24 (b) 30
Q1. The sum of two numbers is 36 and their H.C.F (c) 32 (d) 36
and L.C.M. are 3 and 105 respectively. The sum of the Ans: (c) 2 + 4 = 6
reciprocals of two numbers is 6 + 5 = 11
(a) 11 + 6 = 17

Page 27 of 516
17 + 7 = 24
24 + 8 = 32
https://telegram.me/aedahamlibra
= 5834 – 1120 = 4714

Q10. The H.C.F. and L.C.M. of two numbers are 12


Q6. Two numbers are in the ratio 3 : 4. Their L.C.M. and 336 respectively. If one of the numbers is 84, the
is 84. The greater number is other is
(a) 21 (b) 24 (a) 36 (b) 48
(c) 28 (d) 84 (c) 72 (d) 96
Ans: (c) Let the numbers be 3x and 4x. Ans: (b) First number × second number
∴ Their LCM = 12x = HCF × LCM
∴ 12x = 84 84 × second number = 12 × 336
∴ Second number
= = 48
∴ Larger number
p × q = HCF × LCM
= 4x = 4 × 7 = 28
q= = 48
Q7. A 4–digit number is formed by repeating a 2–digit
number such as 1515, 3737, etc. Any number of this Q11. How many perfect squares lie between 120 and
form is exactly divisible by 300 ?
(a) 7 (b) 11 (a) 5 (b) 6
(c) 13 (d) 101 (c) 7 (d) 8
Ans: (d) xyxy = xy × 100 + xy Ans: (c) 112 = 121, 122 = 144, 132 = 169, 142 = 196
= xy (100 + 1) = 101 × xy 152 = 225, 162 = 256, 172 = 289
Hence, the number is exactly divisible by 101. Square no above 120 = 121 of 11
Square no below 300 = 289 of 17
Q8. A number, when divided by 136, leaves remainder Total 11, 12, 13, 14, 15, 16, 17, i.e. 7 no.
36. If the same number is divided by 17, the Alternate Method:
remainder will be First square number above 120 is 121
(a) 9 (b) 7 112 > 120 and 182 > 300
(c) 3 (d) 2 Hence, required number of squares between 120 to 300
Ans: (d) If the f irst divisor be a multiple of the second = 18 – 11 = 7
divisor, then required remainder = remainder obtained by
dividing the first remainder (36) by the second divisor Q12. If ' n' be any natural number, then by which
(17) = 2 largest number (n3 – n) is always divisible ?
(a) 3 (b) 6
17 is a factor of 136 (c) 12 (d) 18
∴ Remainder when 36 is divided by 17 = 2 Ans: (b) n3 – n = (n2 – 1)
n (n +1) (n – 1)
Q9. The greatest number, which when subtracted For n = 2, n3 – n = 6
from 5834, gives a number exactly divisible by each of 23 – 2 = 6
20, 28, 32 and 35, is i.e. n3 – n is always divisible by 6.
(a) 1120 (b) 4714
(c) 5200 (d) 5600 Q13. The last digit of (1001)2008 + 1002 is
Ans: (b) (a) 0 (b) 3
(b) 4 (d) 6
Ans: (b) Last digit of (1001)2008 + 1002 = 1 + 2 = 3

Q14. The remainder when 321 is divided by 5 is


(a) 1 (b) 2
(c) 3 (d) 4
∴ LCM = 2 × 2 × 5 × 7 × 8 = 1120
Ans: (c) 31 = 3; 32 = 9; 33 = 27; 34 = 81; 34 = 243
∴ Required number
i.e. unit‘s digit is repeated after index 4.
Page 28 of 516
https://telegram.me/aedahamlibra
Remainder after dividing 21 by 4 = 1
∴ Unit‘s digit in the expansion of (3) 21 = 3
an – bn is completely divisible by a + b. If n is an even
number in the case of 17200 ÷ 18
∴ Remainder after dividing by 5 = 3 17200 − 1200 is completely divisible by 17 + 1 = 18
Here, 1 is remainder.
Q15. T he L.C.M. of three different numbers is 120. Or in other words if a n is divided by a + 1 and n is even
Which of the following cannot be their H.C.F.? number then it always left 1 as remainder.
(a) 8 (b) 12
(c) 24 (d) 35 Q20. The least number, which is to be added to the
Ans: (d) HCF must be a factor of LCM from option 35 is greatest number of 4 digits so that the sum may be
not factor of 120. divisible by 345, is
Alternate Method: (a) 50 (b) 6
If two number are in the form of ax and bx then x is (c) 60 (d) 5
H.C.F and a × b × x is their L.C.M Ans: (b) The largest 4-digit number = 9999
Hence L.C.M is always divisible by H.C.F.

Q16. If x * y = (x + 3)2 (y –1), then the value of 5 * 4 is


(a) 192 (b) 182
(c) √ (d) 356
Ans: (c) = (x + 3)2 (y – 1)
∴ = (5 + 3)2 (4 – 1)
= 64 × 3 = 192 ∴ Required number = 345 – 339 = 6

Q17. The traffic lights at three different road Q21. If a * b = ab, then the value of 5 * 3 is
crossings change after 24 seconds, 36 seconds and 54 (a) 125 (b) 243
seconds respectively. If they all change simultaneously (c) 53 (d) 15
at 10 : 15 :00 AM, then at wha t time will they again Ans: (a) a * b = ab
change simultaneously? ∴ 5 * 3 = 53 = 5 × 5 × 5 = 125
(a) 10 : 16 : 54 AM (b) 10 : 18 : 36 AM
(c) 10 : 17 : 02 AM (d) 10 : 22 : 12 AM Q22. The unit digit in the sum of (124)372 + (124)373 is
Ans: (b) LCM of 24, 36 and 54 seconds (a) 5 (b) 4
= 216 seconds = 3 minutes 36 seconds (c) 2 (d) 0
∴ Required time = 10 : 15 : 00 + Ans: (d) 41 = 4; 42 = 16; 43 = 64; 44 = 256; 45 = 1024
3 minutes 36 seconds = 10 : 18 : 36 a.m. Remainder on dividing 372 by 4 = 0
Remainder on dividing 373 by 4 = 1
Q18. A number when divided by 49 leaves 32 as ∴ Required unit digit
remainder. This number when divided by 7 will have = Unit‘s digit of the sum = 6 + 4 = 0
the remainder as
(a) 4 (b) 3 Q23. L.C.M. of two numbers is 120 and their H.C.F. is
(c) 2 (d) 5 10. Which of the following can be the sum of those two
Ans: (a) Here, the first divisor i.e. 49 is multiple of numbers?
second divisor i.e. 7. (a) 140 (b) 80
∴ Required remainder = Remainder obtained on dividing (c) 60 (d) 70
32 by 7 = 4 Ans: (d) Let the numbers be 10x and 10y where x and y
are prime to each other.
Q19. If 17200 is divided by 18, the remainder is ∴ LCM = 10 xy
(a) 1 (b) 2 10xy = 120 xy = 12
(c) 16 (d) 17 Posssible pairs = (3, 4) or (1, 12)
Ans: (a) Remainder when (x – 1)n is divided by x is (–1)n ∴ Sum of the numbers = 30 + 40 = 70
∴ (17)200 = (18 – 1)200 Alternate Method:
∴ Remainder = (–1)200 = 1 If two different numbers a re in form of ax and bx H.C.F
Alternate Method: of these numbers is x and L.C.M of these numbers is abx
Page 29 of 516
https://telegram.me/aedahamlibra
Now a and b are co-prime terms in L.C.M.
10 × a × b = 120
∴ a = 55, d = + 10
series would be :
a × b = 12 → 1 × 12 25, 45, 65, 85
2 × 6 this is not a pair of co prime terms. IIIrd term would be 65.
3×4
Q28. The greatest number that can divide 140, 176,
Q24. Which one of the following will comp letely 264 leaving remainders of 4, 6, and 9 respectively is
divide 571 + 572 + 573? (a) 85 (b) 34
(a) 150 (b) 160 (c) 17 (d) 2
(c) 155 (d) 30 Ans: (c) Required number = H.C.F of (140 – 4), (176 –
Ans: (c) 571 + 572 + 573 6) and (264 – 9) = H.C.F. of 136, 170 and 255.
= 571 (1 + 5 + 52) = 570 × 5 × 31
= 571 × 155 which is exactly divisible by 155.

Q25. Four runners started running simultaneously


from a point on a circular track. They took 200
seconds, 300 seconds, 360 seconds and 450 seconds to
complete one round. After how much time they meet
at the starting point for the first time?
(a) 1800 seconds (b) 3600 seconds
(c) 2400 seconds (d) 4800 seconds
Ans: (a) Required time = LCM of 200, 300, 360 and 450
seconds = 1800 seconds.

Q26. When 'n' is divisible by 5 the remainder is 2.


What is the remainder when n2 is divided by 5?
(a) 2 (b) 3
(c) 1 (d) 4 ∴ Required number = 17
Ans: (d) Required remainder = Remainder obtained by Alternate Method:
dividing 22 by 5. Here divisible terms are 140 – 4 = 136, 176 – 6 = 170 and
Remainder = 4 264 – 9 = 255
Now, difference between these numbers
Q27. There are 4 terms in an A.P. such that the sum 170 – 136 = 34
of two means is 110 and product of their extremes is 225 – 170 = 85
2125. The 3rd term is H.C.F of difference = 17
(a) 65 (b) 75 Hence required number = 17.
(c) 55 (d) 45
Ans: (a) Let the 4 terms in A.P are a – 3d, a – d, a + d, a Q29. The least number which when divided by 35, 45,
+ 3d 55 leaves the remainder 18, 28, 38 respectively is
According to question (a) 3448 (b) 3482
a – d + a + d = 110... (1) (c) 2468 (d) 3265
(a – 3d) (a + 3d) = 2125... (2) Ans: (a) 35 – 18 = 17
From equation (1) 45 – 28 = 17
a – d + a + d = 110 55 – 38 = 17
2a = 110 a = 55 i.e., difference between the divisor and correseponding
From equation (2) remainder is same.
(a – 3d) (a + 3d) = 2125 LCM of 35, 45 and 55 = 3465
a2 – 9d2 = 2125 ∴ Required number
(55)2 – 9d2 = 2125 = 3465 – 17 = 3448
3025 – 9d2 = 2125
900 = 9d2 d2 = 100 d = 10
Page 30 of 516
https://telegram.me/aedahamlibra
Q30. The number nearest to 75070 which is divisible
by 65, is
Ans: (c) The pattern is :
13 – 2 = –1
(a) 75070 (b) 75075 23 – 2 = 6
(c) 75010 (d) 75065 33 – 2 = 25
Ans: (b) 43 – 2 = 62
53 – 2 = 123
63 – 2 = 214
73 – 2 = 341

Q35. What would be the sum of


1 + 3 + 5 + 7 + 9 + 11 + 13 + 15 + ....... up to 15th term?
(a) 250 (b) 240
(c) 225 (d) 265
Ans: (c) The sum forms A.P.
First term (a) = 1
∴ Required number Common difference (d) = 2
= 75070 + (65 – 60) = 75075 Sum of 15 term =
Sum =
Q31. The greatest number that will divide 19, 35 and
59 to leave the same remainder in each case is: =
(a) 9 (b) 6
(c) 7 (d) 8 Q36. The least multiple of 13 which when divided by
Ans: (d) Required number = HCF of 4, 5, 6, 7 leaves remainder 3 in each case is
(35 – 19), (59 – 35) and (59 – 19) (a) 3780 (b) 3783
= HCF 16, 24 and 40 = 8 (c) 2520 (d) 2522
Ans: (b) LCM of 4, 5, 6 and 7 = 420
Q32. A three -digit number 4a3 is added to another ∴ Required number
three-digit number 984 to give the fo ur digit number = 420k + 3 which is exactly divisible by 13.
13b7 which is divisible by 11. Then the value of (a + b) = 32 × 13k + 4k + 3
is: Hence, 4k + 3 should be divisible by 13 for some value of
(a) 11 (b) 12 k.
(c) 9 (d) 10 For k = 9, 4k + 3 = 39 which is divisible by 13.
Ans: (d) 4a3+984 = 13b7 ∴ Required number = 420 × 9 + 3 = 3783
13b7 is exactly divisible by 11.
∴b=9∴a=1 Q37. If 1 3 + 2 3 + ....... + 9 3 = 2025, then the approx.
∴ a + b = 9 + 1 = 10 value of
(0.11)3 + (0.22)3 + ....... + (0.99)3 is
Q33. The next term of the series 1, 5 12, 24, 43 is (a) 0.2695 (b) 0.3695
(a) 51 (b) 62 (c) 2.695 (d) 3.695
(c) 71 (d) 78 Ans: (c) (0. 11)3 (13 + 23 +...... +93)
Ans: (c) The pattern is : = 0 .001331 × 2025
1+4=5
=
5 + 7 ( = 4 + 3) = 12
12 + 12 ( = 7 + 5) = 24
24 + 19 ( = 12 + 7) = 43 Q38. The least number which when divided by 48, 64,
43 + 28 ( = 19 + 9) = 71 90, 120 wi ll leave the remainders 38, 54, 80, 110
respectively, is
(a) 2870 (b) 2860
Q34. The next term of the series
(c) 2890 (d) 2880
– 1, 6, 25, 62, 123, 214, _____ is:
(a) 345 (b) 143 Ans: (a) Here, (48 – 38) = 10, (64 – 54) = 10, (90 – 80) =
(c) 341 (d) 343 10 and (120 – 110) = 10.

Page 31 of 516
= 2870
https://telegram.me/aedahamlibra
∴ Required number = (L.C.M of 48, 64, 90 and 120) – 10 = 3 × 4 × 5 = 60

Q43. If the sum of the digits of any integer lying


√ √ √ √ 3 between 100 and 1000 is sub tracted from the number,
Q39. If x = and y = , then the value of x +
√ √ √ √ the result always is
3
y is: (a) divisible by 5 (b) divisible by 6
(a) 950 (b) 730 (c) divisible by 2 (d) divisible by 9
(c) 650 (d) 970 Ans: (d) (100x + 10y + z) – (x + y + z) = 99x + 9y
√ √ (√ √ )(√ √ )
Ans: (d) x = = (√ = 9 (11x + y)
√ √ √ )(√ √ )
(√ √ )
= = √ √ = √ Q44. 'a' divides 228 leaving a remainder 18. The
√ √ biggest two-digit value of ‗a‘ is
∴y= = √
√ √ (a) 30 (b) 70
∴x+y= √ √ (c) 21 (d) 35
xy = ( √ ) ( √ ) Ans: (b) 228 – 18 = 210 is exactly divisible biggest two
= 25 – 24 = 1 digit no. i.e. 70
∴ x3 + y3 = (x + y)3 – 3xy (x + y)
= (10)3 – 3 (10) = 1000–30 = 970 Q45. Product of two co -prime numbers is 117. Then
their L.C.M. is
Q40. With a two digit prime number, if 18 is added, (a) 13 (b) 39
we get another prime number with digits reversed. (c) 117 (d) 9
How many such numbers are possible? Ans: (c) HCF of two-prime numbers = 1
(a) 2 (b) 3 ∴ Product of numbers = their LCM = 117
(c) 0 (d) 1
Ans: (a) Let the number be 10x + y. Q46. The fifth term of the sequence for which t 1 = 1, t2
According to condition = 2 and tn + 2 = tn + tn + 1, is
10x + y + 18 = 10y + x (a) 5 (b) 10
y–x=2 (c) 6 (d) 8
So those numbers are 02, 13, 24, 35, 46, 57, 68, 79, 80 Ans: (d) tn + 2 = tn + tn + 1
But 13 and 79 are prime numbers. t 2 = t 2 + t2 = 3
t 4 = t 3 + t2 = 3 + 2 = 5
Q41. L.C.M. of is t 5 = t 4 + t3 = 3 + 5 = 8

(a) Q47. The sum of the squares of the digits of the


(c) largest prime number in two digits is
(a) 148 (b) 130
Ans: (c) LCM of
(c) 97 (d) 118
Ans: (b) Largest two digit prime number is 97
92 + 72 = 81 + 49 = 130

Q42. The ratio of two numbers is 3 : 4 and their HCF Q48. A number x when divided by 289 leaves 18 as the
is 5. Their LCM is: remainder. The same number when divided by 17
(a) 10 (b) 60 leaves y as a remainder. The value of y is
(c) 15 (d) 12 (a) 3 (b) 1
Ans: (b) If the numbers be 3x and 4x, then (c) 5 (d) 2
HCF = x = 5 Ans: (b) Here, the first divisor (289) is a multiple of
∴ Number = 15 and 20 second divisor (17).
∴ LCM = 12x = 12 × 5 = 60 ∴Required remainder = Remainder obtained on dividing
Alternate Method: 18 by 17 = 1
If two numbers are in the form of 3x and 4x then their
L.C.M = 3 ×4 × x Q49. The next term of the sequence,
Page 32 of 516
( ) ( )( ) ( https://telegram.me/aedahamlibra
)( )( ) is (c) 63 (d) 47
Ans: (a) If they are equal number of rows and columns
(a) 3 (b) ( ) then,
(c) 5 (d) ( )( ) √ = 37
Ans: (a) Next term will be
Q55. The smallest five digit number which is divisible
( )( )( )( ) by 12, 18 and 21 is :
(a) 50321 (b) 10224
(c) 30256 (d) 10080
Ans: (d) Lowest 5 digit number = 10,000
Q50. Find the number lying between 900 and 1000 The number which is divisible by 12, 18 and 21 is LCM
which when divided by 38 and 57 leaves in each case a of 12, 18, 12 which is 252.
remainder 23. gives 172 as remainder
(a) 912 (b) 926 So, 252 – 172 = 80
(c) 935 (d) 962 10,000 + 80 = 10080
Ans: (c) L.C.M of (38, 57) = 114 If 10080 when divided by 12, 18 and 21 g ives 0 as
Multiple of 114 between 900 and 1000 = 912 remainder
number which leaves 23 = 912 + 23 = 935 So, 10080 is the least 5–digit number.

Q51. The H.C.F. and L.C.M. of two numbers are 44 Q56. If the product of first fifty positive consecutive
and 264 respectively. If the first number is divided by integers be divisible by 7 n, where n is an integer, then
2, the quotient is 44. The other number is the largest possible value of n is
(a) 147 (b) 528 (a) 7 (b) 8
(c) 132 (d) 264 (c) 10 (d) 5
Ans: (c) First number = 2 × 44 = 88 Ans: (b) Product of f irst fifty positive consecutive
Other number = integers = 1 × 2 × .... × 50 = factorial 50
Largest possible value of n
Q52. Three tankers contain 403 litres, 434 litres, 465 =* + * +=7+1=8
litres of diesel respectively. Then the maximum
capacity of a container that can measure the diesel of
Q57. The least number that should be added to 2055
the three container exact number of times is
so that the sum is exactly divisible by 27 :
(a) 31 litres (b) 62 litres
(a) 24 (b) 27
(c) 41 litres (d) 84 litres
(c) 31 (d) 28
Ans: (a) H.C.F. of 403, 434 and 465 is 31.
Ans: (a) Number has to be less than 27. Let the number
be x. On Dividing 2055 by 27, we get remainder as 3
Q53. The first term of an Arithmetic Progression is 22
Now,3 + x = 27
and the last term is –11. If the sum is 66, the number
∴x = 24
of terms in the sequence are :
(a) 10 (b) 12
Q58. If 13 + 23 + ..... + 103 – 3025, then the value of 23 +
(c) 9 (d) 8
43 +..... + 203 is :
Ans: (b) The sum of Arithmetic Progression is given by
(a) 5060 (b) 12100
(c) 24200 (d) 7590
Ans: (c) 23 + 43 + 63 + ------+ 203
= 23 (13 + 23 + 33 + ------ + 103)
n = 12
= ( ) ( )
Q54. A teacher wants to arrange his students in an = 24200
equal number of rows and columns. If there are 1369
students, the number of students in the last row are Q59. What least value must be assigned to '*' so that
(a) 37 (b) 33 the numbers 451*603 is exactly divisible by 9?
Page 33 of 516
(a) 7
(c) 5
https://telegram.me/aedahamlibra
(b) 8
(d) 9
5000 ÷ 2 = 2500
Hence, 2500 is a perfect square of 50.
Ans: (b) To divide 451 * 603 by 9
(4 + 5 + 1 + * + 6 + 0 + 3) = (19 + *) Q64. Which value among √ √ √ √ is the
(19 + *) must be multiple of 9 largest?
∴ 19 + * = 27 (a) √ √
*=8 (c) √ √

Q60. The least number which when divided by 6, 9, Ans: (a) √


12, 15, 18 leaves the same remainder 2 in each case is: √
(a) 178 (b) 182
(c) 176 (d) 180 √
Ans: (b) LCM of 6, 9, 12, 15 and 18 √
So, option (a) is correct.

Q65. Product of digits of a 2 –digit number is 27. If we


add 54 to the number, the new number obtained is a
number formed by interchange of the digits. Find the
number.
LCM = 2 × 3 × 3 × 2 × 5 = 180 (a) 39 (b) 93
Least number = 180 + 2 = 182 (c) 63 (d) 36
Ans: (a) Let digit at ten's place be x and digit at unit's
Q61. How many numbers are there from 300 to 650 place be y.
which are completely divisible by both 5 and 7? ∴The number = 10x + y
(a) 8 (b) 9 When digit are interchanged, the new number
(c) 10 (d) 12 = 10y + x
Ans: (c) LCM of 5 and 7 = 35 According to question,
So, the numbers divisible by both 5 and 7 are multilpe of Product of digits = 27 i.e., xy = 27... (i)
35. Between 300 and 650. We have 10 multiple of 35. Also,
They are : 315, 350, 385, 420, 455, 490, 525, 560, 595, 10x+ y + 54 = 10y + x
630. 9x – 9y = – 54
x–y=–6
Q62. If X and Y are the two digits of the number ∴x = y – 6... (ii)
347XY such t hat the number is completely divisible From (i) and (ii),
by 80, then what is the value of X + Y? y (y – 6) = 27
(a) 2 (b) 4 y2 – 6y – 27 = 0
(c) 6 (d) 8 y2 – 9y + 3y – 27 = 0
Ans: (a) 347XY as 347X0. Since 8 is a factor of 80. (y – 9) (y + 3) = 0
347X0 is divisible by 8. It means last three digits 7X0 is ∴y = 9 or y – 3
divisible by 8. ∴x = 3
Hence, X is 2 or 6 When x = 3, and y = 9
if X = 6, number is 34760. But this is not divisible by 80. ∴Required number = 10x + y
if X = 2, number is 34720, which is divisible by 80. = 10 × 3 + 9
Therefore, number is 34720 with X = 2 and Y = 0. 30 + 9 = 39.
∴x + y = 2 + 0 = 2.
Q66. What is the LCM (least common multiple) of 57
Q63. By which least number should 5000 be divided and 93?
so that it becomes a perfect square? (a) 1767 (b) 1567
(a) 2 (b) 5 (c) 1576 (d) 1919
(c) 10 (d) 25 Ans: (a) LCM of 57 and 93,
Ans: (a) According to option,
Page 34 of 516
https://telegram.me/aedahamlibra
Now, (27)2 = 729
∴ 729 – 709 = 20
∴20 must be added to 709 to make it a perfect square.
3 × 19 × 31 = 1767.
So, Required answer is 1767.
Simplification
Q67. Of the three numbers, the first is twice the
second, and the second is twice the third. The average
of the reciprocal of the numbers is 7/12. The numbers Q1. is equal to
are: (a) 0.3 (b) 0.5
(a) 20, 10, 5 (b) 4, 2, 1 (c) 0.8 (d) 0.9
(c) 36, 18, 9 (d) 16, 8, 4 Ans: (c) If 0.5 = a and 0.3 = b then,
Ans: (c) Let third number = x
Expression
then,
second number = 2x = 0.5 + 0.3 = 0.8
first number = 4 x
According to question
Q2. √ is equal to
( )
(a) 34 (b) 36
∴x=9 (c) 38 (d) 39
∴first number = 4x = 4 × 9 = 36 Ans: (b) Expression
second number = 2x = 2 × 9 = 18
third number = x = 9 √ √

= 3 × 2 × 3 × 2 = 36
Q68. The least number of five digits exactly divisible
by 88 is:
Q3. The sum of the series
(a) 10088 (b) 10023
(1 + 0.6 + 0.06 + 0.006 + 0.0006 + ....) is
(c) 10132 (d) 10032
Ans: (d) The smallest number of 5 digits = 10000 (a)

Now, and remainder is 56 (c)


∴Required number = 10000 + (88 – 56) = (10000 + 32) Ans: (a) 1 + 0.6 + 0.06 + 0.006 + 0.0006 + ... = 1.666...
= 10032. = 1. ̅

Q69. Which one among √ √ √ √ √ √ is Q4. The number 0.121212.... in the form - is equal to
the smallest number? (a)
(a) √ √ √ √
(c)
(c) √ √ (d) All are equal
Ans: (b) Here, Ans: (c) 0.121212……….
√ √
√ √ Q5. The square root of 0.09 is
√ √ (a) 0.30 (b) 0.03
(c) 0.81 (d) 0.081
So,√ √ is the smallest number.
Ans: (a) √ √
Q70. What is the smallest value that must be added to
709, so that the resultant is a perfect square? Q6. ( ) is equal to
(a) 8 (b) 12
(a) 1236 (b)
(c) 20 (d) 32
Ans: (c) According to question (c) 618 (d) 617

√ • Ans: (a) = 1234 + 2 = 1236

Page 35 of 516
https://telegram.me/aedahamlibra
Q7. By what least number should 675 be multiplied so
(c)

as to obtain a perfect cube number ? Ans: (a) ( )( )( )( )( )


(a) 3 (b) 5 =
(c) 24 (d) 40
Ans: (b) 675 = 5 × 5 × 3 × 3 × 3 = 5
Q14. The least among the fractions is
No to be multiplied
(a)
Q8. is simplified to (c)
(a) 1 (b) 0.1
Ans: (b) = 0.94; = 0.95
(c) 0.01 (d) 10
= 0.96; = 0.97
Ans: (a)

Q15. (0.1 × 0.01 × 0.001 × 107) is equal to


Q9. is equal to
(a) 100 (b)
(a)
(c) (d) 10
(c)
Ans: (d) 0.1 × 0.01 × 0.001 × 107 = 10–6 × 107 = 10
Ans: (b)

Q16. is equal to
Q10. (12 + 22 + 32 + ...... + 102) is equal to
(a) 380 (b) 385 (a) 1 (b)
(c) 390 (d) 392 (c) (d) 10
Ans: (b) 12 + 22 + 32 + .... + n2 = Ans: (d)
∴ 12 + 22 + 32 + .... + 102 = = 385 = = 10

Q11. is equal to Q17. in the form is equal to


(a) 420 (b) 400
(a)
(c) 360 (d) 320
Ans: (b) If 256 = a and 144 = b, then (c)
Ans: (c) =

[a – b = 256 – 144 = 112]


= 256 + 144 = 400 Q18. , - is equal to
(a) 1010 (b) 110
(c) 101 (d) 100
Ans: (d) = 99 + 1 = 100
Q12. Simplified form of [( √ ) ] is

Q19. Out of six consecutive natura l numbers, if the


(a) x5 (b) x–5 sum of first three is 27, what is the sum of the other
(c)
x (d) three ?
(a) 36 (b) 35
(c) 25 (d) 24
Ans: (c) [( √ ) ] =( ) = =x
Ans: (a) 8 + 9 + 10 = 27
11 + 12 + 13 = 36
So, let 3 consecutive no x, x + 1, x + 2
Q13. ( )( )( ) ( ) is equal to Next 3 consecutive no x + 3; x + 4, x + 5
i.e. sum of last 3 consecutive no. is 9 more than sum of
(a)
first 3.
Page 36 of 516
= 27 + 9 = 36 https://telegram.me/aedahamlibra Q24. If √
(a) 2
√ √ , then x = ?
(b) 3
√ √ (c) 4 (d) 5
Q20. The square root of ( ) is
√ √
Ans: (b) √ √ √
(a) √ √ √ √
√ √ √ √
(c) √ √ √ √
(√ √ )
Ans: (a) Expression = (√
√ )
Rationalising the denominator, Q25. is
(√ √ )(√ √ ) (√ √ )
(√
= (√ √ ) (a) 10 (b) 100
√ )(√ √ )
(c) 1000 (d) None of these
√ √
∴√ √(√ √ ) √ √ Ans: (b)
√ √

= 100
Q21. √ √ √ =?
(a) 2.3 (b) 3 Q26. If the sum of two numbers be multiplied by each
(c) 6 (d) 6.3 number separately, the products so obtained are 247
and 114. The sum of the numbers is
Ans: (b) √ √ √ (a) 19 (b) 20
6=3×2 (c) 21 (d) 23
By trick = 3 answer Ans: (a) Let the numbers be x and y.
∴ x (x + y) = 247
√ √ √ and y (x + y) = 114
Q22. The value of is
√ √ √ √ √ √ x2 + xy = 247 and xy + y2 = 114
(a) 4 (b) 0
On adding;
(c) √ √ x2 + xy + xy + y2 = 247 + 114
Ans: (b) Expression x2 + 2xy + y2 = 361
√ √ √
= (x + y)2 = 192 x + y = 19
√ √ √ √ √ √
√ (√ √ ) √ (√ √ )
= (√ √ )(√ √ ) (√ √ )(√ √ )
+
Q27. √ is equal to
√ √ √
(√ √ ) √ √ (a)
√ (√ √ ) √ (√ √ ) √ (√ √ ) (c)
=
= √ (√ √ ) √ (√ √ ) √ (√ √ ) Ans: (b) = √ √
=√ √ √ √ √ √ =0
=√ √

Q23. The value of is


√ √
Q28. If = a + b √ then the values of a and b
√ √
(a) are respectively

(c) 1 (d) (a)

Ans: (a) Expression (c)


√ √
= Ans: (d)
√ √
√ √ √

= = √ √

√ √
Page 37 of 516
By Rationalising
√ ( √ √ )
https://telegram.me/aedahamlibra
∴ Correct answer = = 110 + 12 = 122

( √ √ )( √ √ ) Q34. The value of √ ×√ is


√ √ √ (a) 116 (b) 126
, ,
(c) 106 (d) 136
√ √ √ √ Ans: (b) √ √
√ √ √ = 126

Q35. What is the value of (2.1)2 × √ ?


Q29. Find a number, one –seventh of which exceeds its (a) 0.9261 (b) 92.61
eleventh part by 100. (c) 92.51 (d) 0.9251
(a) 1925 (b) 1825
Ans: (a) Expression is (2.1) 2 × √ = 4.41 × 0.21 =
(c) 1540 (d) 1340
0.9261
Ans: (a) Let the number be x.
∴ = 100
Q36. If √ , then the value of n is:
= 100 (a) 9 (b) 6
4x = 77 × 100 (c) 1 (d) 3
x= = 1925 Ans: (a) [ ]

Q30. The value of is Comparing,


√ √ √
(a) √ (b) 3 x=9
(c) 1 (d) 0
Ans: (b) ( ) Q37. If , then the value is :
√ √ √
√ √
(a) 10x (b) 100x
= (( )
√ √ )( √ ) (c)
√ √
= = Ans: (d)
√ √ √ √

Q32. Last year my age was a perfect square number.


Next year it will be a cubic number. What is my
present age?
(a) 25 years (b) 27 years Q38. The value of 4√ √ √ 5 is
(c) 26 years (d) 24 years
Ans: (c) By going options, 26 years is the pre sent age. equal to
Present age be 26, then last year age was 25 which (a) 3 (b) 10
represents a perfect square and next year age would be 27 (c) 8 (d) 2
which represents a cubic number.
Ans: (a) √ √ √
Q33. A student was asked to divide a number by 6 On squaring,
and add 12 to the quotient. He, however, first added
√ √
12 to the number and then divided it by 6, getting 112
2
as the answer. The correct answer should have been x =6+x
(a) 124 (b) 122 x2 – x – 6 = 0
(c) 118 (d) 114 x2 – 3x + 2x – 6 = 0
Ans: (b) Let the number be x x (x – 3) + 2 (x –3) = 0
∴ = 112 (x – 3) (x + 2) = 0
x + 12 = 672 x = 3 because
x = 672 – 12 = 660 Bytrick

Page 38 of 516
Q39. From 9.00 AM to 2.00 PM, the temperature rose
https://telegram.me/aedahamlibra
the second one –fourth, the third son and the fourth
son 7 cows. Then the value of n is
at a constant rate from 21ºC to 36ºC. What was the
(a) 240 (b) 100
temperature at noon ?
(c) 180 (d) 140
(a) 27ºC (b) 30ºC
Ans: (d) According to the question,
(c) 32ºC (d) 28.5ºC
Ans: (b) Time difference between 9.00 A.M & 2.00 P.M
= 5 hours
Temperature difference between 21°C & 36°C
= 36 – 21 = 15°C
Now, Time difference between 9.00 A.M & 12.00 Noon
= 3 hrs.
In 5 hours →
So, In 3 hours → ( ) = 9°C Q44. is equal to
So, temperature at noon = 21 + 9 = 30°C (a) 3 (b) 0
(c) 1 (d) 2

Q40. is equal to Ans: (c)

(a) 0 (b)

(c) √
√ √ √ √
√ Q45. If √ , then the value of x is:
Ans: (b) √ √ √ √
√ ( √ )( √ )
(a) 6 (b) 30
√ √
(c) √ (d) 15
√ √ √ √
√ √ Ans: (d) √
√ √ √ √

(√ √ ) (√ √ ) √ √
√ √ √
(√ √ )(√ √ )
√ √
Q41. If √ √ √ , then the value of x is
(a) 3 (b) ± 3 Q46. If 21 is added to a number, it becomes 7 less than
thrice of the number. Then the number is
(c) √ √
(a) 14 (b) 16
Ans: (a) √ √ √
(c) 18 (d) 19
√ √ √ Ans: (a) If the number be x, then
√ √ √ x + 21 = 3x – 7
√ √ 3x – x = 21 + 7
x=3 2x = 28
x = 14
Q42. By what least number shou ld 675 be multiplied
to obtain a number which is a perfect cube? Q47. The simplest value of
(a) 7 (b) 8 is
√ √ √ √ √ √ √ √
(c) 5 (d) 6
Ans: (c) 675 = 5 × 5 × 3 × 3 × 3 (a) √ (√ ) √ (√ ) √ √
= 33 × 52 Ans: (b)
√ √
∴ Required number = 5

Q43. A farmer divides his herd of n cows among his


four sons, so that the first son gets one –half the herd,

Page 39 of 516
√ √ √ https://telegram.me/aedahamlibra
√ (c) 9 (d) 7
√ √ √ √ Ans: (d) √ √ √ =7
√ √
√ √ Q53. The greatest among the following numbers
√ √ is:
√ √
(a) (b) 1
√ √ √ √
√ √ (c)
∴ Expression Ans: (d) LCM of 3, 2 and 6 = 6
√ √ √ √ √ √ √ √ ∴
√ √ √ (√ )

Q48. √ equals
(a) 5 (b) 50 Q54. Find the value of
(c) 500 (d) 0.05
√ √ √
Ans: (b) √ √
(a) 6 (b) 3
√ = 50 (c) √
(√ )

√ √
Ans: (b)
√ √ √
Q49. The simplified value of is
√ √ √ √ √
(a) 4 (b) 3 √ √ √ √ √ √
(c) 2 (d) 6 √ √ √
√ √ √ √
Ans: (c) √ √ √
√ √ √ √
√ √ ( √ √ )
√ √
√ √ ( √ √ ) 3

Q50. The value of


Q55. Which is greater √ or √ ?
√ √ √ √ (a) Equal (b) Cannot be compared
is: (c) √ √
(a) 4.8484 (b) 4.8694
(c) 4.8884 (d) 4.8234 Ans: (d) √ √
Ans: (c) √ √ √ √ or √
√ √ √
= 4.4 + 0.44 + 0.044 + 0.0044 = 4.8884
Q56. If ( ) ( ) ( ) , then x is :
Q51. The value of is:
(a) 2½ (b) –2
(c) 2 (d) 5
(a)
Ans: (d) ( ) ( ) ( )
(c)
Ans: (c) = ( ) ( ) ( )

( ) ( )
= = =
2x = 10 x=5

Q57. If a number is as much greater than 31 as it is


Q52. The value of √ √ √ is less than 75, then the number is.
(a) 11 (b) √ (a) 53 (b) 106
Page 40 of 516
(c) 44
Ans: (a) x – 31 = 75 – x
https://telegram.me/aedahamlibra
(d) 74 (a)
(c)
2x = 106
x = 53 Ans: (b) (ab + ba)–1 = (23 + 32)–1 = (8 + 9)–1 = (17)–1

Q58. A rational number between and is Q63. If x 2 = y + z, y 2 = z + x and z 2


= x + y, then the
(a) value of is

(c) (a) 2 (b) 0


(c) – 1 (d) 1
Ans: (b) Ans: (d) x2 = y + z
x2 + x = x + y + z
x (x + 1) = x + y + z

∴ Required rational number Similarly,

Q59. Number of digits in the square root of 62478076


is :
(a) 3 (b) 4
(c) 5 (d) 6
Ans: (b) When no. of digit in a no. is 7 or 8 then in
square root will be 4.

Q60. The numerator o f a fraction is 4 less than its Q64. The fourth root of 24010000 is
denominator. If the numerator is decreased by 2 and (a) 7 (b) 49
the denominator is increased by 1, then the (c) 490 (d) 70
denominator becomes eight times the numerator. Find
Ans: (d) √ = 4900
the fraction.
Again, √ = 70
(a)
∴√ = 70
(c)
Ans: (d) Original fraction Q65. The smallest positive integer which when
multiplied by 392, gives a perfect square is
In case II,
(a) 2 (b) 3
8 (x – 4 – 2) = x + 1
(c) 5 (d) 7
8x – 48 = x + 1
Ans: (a) 392 × 2 = 784 (28)2
7x = 49 x = 7
Hence, 2 can be multiplied by 392 which gives perfect
Original fraction square.

Q61. Find the simplest value of √ √ √ √


Q66. The value of is

(given √ ).
(a) √ √
(a) 10.312 (b) 8.484
(c) √ √
(c) 4.242 (d) 9.898

Ans: (d) Expression Ans: (a) Expression =

√ √ √ Rationalising the denominator.
√ √ √ =(
( √ )( √ )
=
√ √
√ )( √ )
√ √ √ √
= √ √

Q62. If a = 2, b = 3, then (ab + ba)–l is


Page 41 of 516
https://telegram.me/aedahamlibra
Q67. The greatest 4 digit member which is a perfect
square, is
Ans: (a) Next term will be

( )( )( )( )
(a) 9999 (b) 9909
(c) 9801 (d) 9081
Ans: (c) 99 × 99 = 9801
Alternate Method: Q73. If '+' means ' ÷', '×' means ' –', '÷' means '×' and
1002 = 10000 which is a 5 digits number '–' means '+', what will be the value of the following
Hence, 992 is required number. expression?
9 + 3÷ 4 – 8 × 2 = ?
Q68. If (23)2 = 4x then 3x is equal to (a)
(a) 3 (b) 6
(c) (d) 18
(c) 9 (d) 27
Ans: (d) (23)2 = 4x Ans: (d) 9 + 3 ÷ 4 – 8 × 2 = ?
26 = 22x Applying rules
6 = 2x 9÷3×4+8–2=?
x=3 3×4+8–2=?
33 = 27 20 – 2 = ?
? = 18
Q69. Which one of the following is the minimum value
of the sum of two integers whose product is 24? Q74. Ram left of his property to his widow and of
(a) 25 (b) 11 the remainder to his daughter. He gave the rest to his
(c) 8 (d) 10 son who received ₹ 6,400. How m uch was his original
Ans: (d) Product of 2 no is = 24 property worth?
Possible pair of factor = (1, 24) (2, 12) (3, 8) (4, 6) i,e., 4 (a) ₹ 16,000 (b) ₹ 32,000
+ 6 is minimum = 10 (c) ₹ 24,000 (d) ₹ 1,600
Ans: (c) Let original property worth ₹ x
Q70. The value of
Property left for Ram‘s widow =
* , ( )-+ is
Property left for his daughter =
(a)
Remaining property = ( )
(c)
Ans: (b) * , -+

[ ]

[ ] Q75. The simplified value of


(√ √ √ √ )(√ √ √ √ )
is
(a) 13 (b) 12
√ √ (c) 11 (d) 10
Q71. Evaluate
√ √
(a) 2 (b) 3 Ans: (d) [(√ √ ) (√ √ )] *(√
(c) 4 (d) 5 √ ) (√ √ )+
√ √ √ √ √
Ans: (b)
√ √ √ √ √ = (√ √ ) (√ √ )
= √ √ √ √
Q72. The next term of the sequence,
= √ √ = 10
( ) ( )( ) ( )( )( ) is
(a) 3 (b) ( ) Q76. Arrange the following in ascending order
334, 251, 717, we get
(c) 5 (d) ( )( ) (a) 334 > 251 > 717 (b) 717 > 251 > 334
Page 42 of 516
(c) 334 > 717> 251
Ans: (b) 334 = (32)17 = 917
https://telegram.me/aedahamlibra
(d) 251 > 334> 717 (a) 8 (b)

251 = (23)17 = 817 (c) (d) 4
717 > 817 > 917
or 717 > 251 > 334 √ √ √ √ √
Ans: (c)

Q77. Which one of the following is true ?
(a) √ √ √ √ √ √ √ √ √ √ √ √

(c) √ √ √ √ (√ √ )(√ √ )=1 = ( √ )



Ans: (a) √ √ √ √ √ √ √
Squaring both sides = (√ )

5+3+ √ >6+2+ √ =
√ √
(√ )

√ √ which is true

= =
√ √
Q78. The simplified value of √ √ √ √
= ( √ ) √ √
is:
(a) 59.96 (b) 599.6
Q82. In an exam the sum of the scores of A and B is
(c) 0.5996 (d) 5.996
120, that of B and C is 130 and that of C and A is 140.
Ans: (d) 0.0539 – 0.002 = 0.0519
Then the score of C is :
0.56 × 0.07 = 0.0392
(a) 65 (b) 60
0.0519 × 0.4 = 0.02076
(c) 70 (d) 75
0.04 × 0.25 = 0.01
Ans: (d) A + B = 120… (i)
So B + C = 130 … (ii)
= = = 5.996 C + A = 140 … (iii)
Adding all three equations
2A + 2B + 2C = 390
Q79. 2km 5m is equal to:
A + B + C = 195
(a) 2.005 km (b) 2. 0005 km
But, A + B = 120
(c) 2.5 km (d) 2.05 km
So, C = 195 – 120 = 75
Ans: (a) 2km 5m = = 2.005 km
Q83. The simplified value of following is :
Q80. If = √ , then the value of
( )
will be :
(a) (a)
√ √
(c) √ (d) 3 (c)

Ans: (c) 32x – y = 3x + y = √ Ans: (c) ( )÷ a2 b c3


2x – y = x + y = = a6 b11 c7 ÷ a2 b c 3
4x – 2y = 3 ... (i)
= a6 b11 c7× a–2 b–1 c-3 = a6–2 b11–1 c7–3
2x + 2y = 3 ... (ii)
Solving equation (i) and (ii) = a4 b10 c4
x=1y=
Q84. The value of

is
√ √ √ √ √ √ √ √
(a) 0 (b) 1
√ √ √ √
(c) 5 (d) 7

Q81. ? Ans: (c)
√ √ √ √ √ √ √ √ √

Page 43 of 516
√ √



√ √ √
https://telegram.me/aedahamlibra



√ √
Ans: (c) By squ aring the given relations, we get (i) and
(iii) are incorrect relations from the given statement.
√ √ √ √
√ √ √ √ √ √ √ Q88. The value of
√ √ (√ √ ) √
(a) 2 (b) 2.27
(√ √ ) √ (c) 2.33 (d) 2.6
Ans: (d)
√ √ √ √ √ √ √ √
[ ]
5 [ ]
2.3 + 0.3 = 2.6
Q85. The sum of four numbers is 48. When 5 and 1
are added to the first two; and 3 & 7 are subtracted
Q89. If of a number is 7 more than of the number,
from the 3rd & 4th, the numbers will be equal. The
numbers are then of the number is :
(a) 4, 12, 12, 20 (b) 5, 11, 13, 19 (a) 15 (b) 18
(c) 6, 10, 14, 18 (d) 9, 7, 15, 17 (c) 12 (d) 20
Ans: (c) Let four numbers are a, b, c, d, then Ans: (d) Let the number be x
a + b + c + d = 48... (i)
anda + 5 = b + 1... (ii)
or,a = b – 4... (iii)
andc – 3 = d – 7... (iv) =
c = d – 4... (v)
∴ x = 12
Substituting equation (iii) and (v) in equation (i) we get
b – 4 + b + d – 4 + d = 48 Now, of x =
b + d = 28... (vi)
But we know,
Q90. If , then the value of a + b + c
b+1=d–7
∴b = d – 8
Substituting in equation (vi) we get (a) 6 (b) 3
d – 8 ≠ d = 28 (c) 9 (d) 12
d = 18 Ans: (c)
Solving this way we get a = 6, b = 10, c = 14 and d = 18
Q91. If the numbers √ √ √ are arrange d in
Q86. If x = ascending order, then the right arrangement is

(a) √ √ √ √ √ √
then (x + 1) equals to
(c) √ √ √ √ √ √
(a) 2 (b) √
Ans: (d) √ √ √
(c) √ √
LCM of 3, 4, 6 = 24

Ans: (d) x = √
√ √ √ √ √
Now, x + 1 = √ =√ √ √ √
i.e.√ √ √
Q87. Choose the incorrect relation (s) from the
Alternate Method:
following:
(i) √ √ √ √
L.C.M of Numbers of Powers = 12
(ii)√ √ √ √
(iii)√ √ √ √ √ √ √
(a) (i) (b) (ii) √ √ √
(c) (i) and (iii) (d) (ii) and (iii) √ √ √

Page 44 of 516
https://telegram.me/aedahamlibra
Q92. If 2x – 3 (2x – 2) > x – 1 < 2 + 2x, then x can take
which of the following values?


(a) 2 (b) –2 √ ( √ )
=
(c) 4 (d) –4 √ √
Ans: (b) Here,
2x – 3 (2x – 2) > x – 1 < 2 + 2x Q95. If 1 .5x = 0.04y, then the value of is
2x – 6x + 6 > x – 1
2x – 6x – x > –7 (a)
–5x > –7 (c)
x < 7/5... (i)
Ans: (b) 1.5x = 0.04y =
(x – 1) < (2 + 2x)
x – 1 < 2 + 2x
– 3 < x... (ii)
Now,
From (i) and (ii),
x = –2.
= = =

Q93. The sum of two numbers is and their


difference is . The product of the numbers is Algebra
(a) 50 (b)
(c) (d) 60 Q1. If √ , then the value of x is
Ans: (a) Let the numbers are x and y.
According to question (a) (b) 160
x+y= ... (i) (c) (d) 169
x–y ... (ii) Ans: (b) √
Adding eqn. (i) & (ii)
Squaring on both sides,
2x =
x = 10
10 + y =
y= x = 160

Product of number
Q2. If √ = 0.3 × 0.3 × √ value of is
(a) 0.009 (b) 0.09
√ √
Q94. If then what is the value of N+ (c) 0.09 (d) 0.08
√ √
(1/N)? Ans: (b) √ = 0.3 × 0.3 √
(a) √ (b) 5 On squaring,
(c) 10 (d) 13 0.03 × 0.3 × a = 0.09 × 0.09 × b
Ans: (b) Here, = 0.9
√ √
?
√ √
Now, Q3. If x = (0.08)2, y = and z = (1 – 0.08)2 – 1,
√ √ √ √ (√ √ ) then out of the following the true relation is
√ √ √ √ (a) y < x and x = z (b) x < y and x = z
(√ ) (√ )
(c) y < z < x (d) z < x < y
√ √ √ √
Ans: (d) x = (0.08)2
∴ y

√ Z = (1 – 0.08)2 – 1
∴( )

Page 45 of 516
Clearly, z < x < y
https://telegram.me/aedahamlibra
= 1 + (0.08)2 – 2 × 0.08 – 1 = (0.08)2 – 2 × 0.08 (c) –1
Ans: (b) If a + b + c = 0
(d) 1

then a3 + b3 + c3 – 3abc = 0
Q4. The sum of two numbers is 24 and their product
is 143. The sum of their squares is Q8. If = 3, the value of is
(a) 296 (b) 295
(a) 123 (b) 126
(c) 290 (d) 228
(c) 113 (d) 129
Ans: (c) Let the two numbers be x and y.
∴ x + y = 24 Ans: (a)
and, xy = 143 On squaring,
∴ x2 + y2 = (x + y)2 – 2xy
( )
= (24)2 – 2 × 143 = 576 – 286 = 290

Q5. If = 23, then the value of ( ) will be


Again, ( )
(a) 7 (b) –7
(c) –3 (d) 3 ( )
Ans: (d)

( )
( )( )
( ) = 7 × 18
( )

( ) =5–2=3

Q6. In xy –plane, P and Q are two points having co – Q9. If (a – b) = 3, (b – c) = 5 and (c – a) = 1, then t he
ordinates (2, 0) and (5, 4) respectively. Then the value of is
numerical value of the area of the ci rcle with radius
(a) 17.5 (b) 20.5
PQ is
(c) 10.5 (d) 15.5
(a) 16 π (b) 32 π
(c) 14 π (d) 25 π Ans: (a)
Ans: (d) a3 + b3 + c3 – 3abc
= [ ]

[ ]

(9+25+1)

Q10. In the xy –coordinate system, if (a, b) and (a + 3,


b + k) are two points on the line defined by the
√ equation x = 3y – 7, then k = ?
√ (a) (b) 1
∴ Area of circle = πr2
(c) 9 (d) 3
= 25 π sq. units
Ans: (b) Points (a, b) and [ (a + 3), (b + k)] will satisfy
3 the equation x – 3y + 7 = 0.
Q7. If a + b + 1 = 0, then the value of (a + b3 + 1 –
∴ a – 3b + 7 = 0...... (i)
3ab) is
and a + 3 – 3 (b + k) + 7 = 0
(a) 3 (b) 0
Page 46 of 516
a + 3 – 3b – 3 k + 7 = 0
a – 3b + 7 + 3 – 3 k = 0
https://telegram.me/aedahamlibra
a3 + 1
a3 = –1
3 – 3k = 0 3k = 3
Q14. If ax + by = 6, bx – ay = 2 and x 2 + y 2 = 4, then
[ ]
the value of (a2 + b2) would be:
(a) 10 (b) 2
√ √ √ √ (c) 4 (d) 5
Q11. If , , then the value of
√ √ √ √
Ans: (a) ax + by = 6... (i)
is: bx – ay = 2... (ii)
(a) 900 (b) 970 On squaring and adding,
(c) 1030 (d) 930 a2x2 + b2y2 + 2abxy + b2x2 + a2y2 – 2abxy
√ √ = 36 + 4
Ans: (b)
√ √ x2 (a2 + b2) + y2 (a2 + b2) = 40
√ √ √ √ (a2 + b2) (x2 + y2) = 40
√ √ √ √ (a2 + b2) × 4 = 40
(√ √ ) a2 + b2 = 10
√ = 5 – 2√
√ √ Q15. If a 3 – b3 = 56 and a – b = 2, then the value of (a 2

√ √ + b2) is:
a + b = 10; (a) – 10 (b) – 12
ab ( √ )( √ ) = 25 – 24 = 1 (c) 20 (d) 18
Ans: (c) (a – b)3 = a3 – b3 – 3ab (a –b)
8 = 56 – 3ab (2)
6ab = 56 – 8 = 48
2ab = 16... (i)
= 103 – 3 × 10 = 1000 – 30 = 970 ∴ a2 + b2 = (a – b)2 + 2ab
= 4 + 16 = 20
Q12. If (5x 2 – 3y2) : xy = 11 : 2, then the postive value
of x/y is:
Q16. If √ , then the value of √ is:
(a) 7/2 (b) 5/2 √

(c) 3/2 (d) 5/3 (a) √ √


(c) √ (d) 6
Ans: (c)
Ans: (b) √
10x2 – 6y2 = 11xy

10x2 – 11xy – 6y2 = 0
10x2 – 15xy + 4xy – 6y2 = 0 √ √ √
5x (2x – 3y) + 2y (2x – 3y) = 0 √

(5x + 2y) (2x – 3y)
5x ≠ 2y, 2x = 3y (√ ) √ √

.√ √

Q13. If , then the value of a3 is: Q17. Area of the triangle formed by the graph of the
(a) – 2 (b) 2 line 2x – 3y + 6 = 0 along with the coordinate axes is:
(c) – 1 (d) 4 (a) 1/2 sq. units (b) 3/2 sq. units
Ans: (c) (c) 3 sq. units (d) 6 sq. units
Ans: (c) Putting x = 0 in
a2 + 1 = a
2x – 3y + 6 = 0
a2 – a + 1 = 0
y=2
Multiplying both side by (a + 1)
Putting y = 0 in 2x + 3y + 6 = 0
(a + 1) (a2 – a + 1) = 0
x=–3
Page 47 of 516
https://telegram.me/aedahamlibra
(c) 790
Ans: (d)
(d) 970

( )

( )

On cubing both sides,

( )

( )
Area of ∆ OAB
sq. units.

Q18. Area of the trapezium formed by x -axis; y -axis Q20. If , then the value of
and the lines 3x + 4y = 12 and 6x + 8y = 60 is:
(a) 37.5 sq. unit (b) 31.5 sq. unit is:
(c) 48 sq. unit (d) 36.5 sq. unit (a) 2 (b) 6
Ans: (b) For 3x + 4y = 12, (c) 4 (d) 3
By putting x = 0, y = 3 Ans: (a)
By putting, y = 0, x = 4
For 6x + 8y = 60,
By putting x = 0, On cubing,
By putting y = 0, x = 10
[ ]

( )

Q21. If the distance between two points (0, –5) and


(x,0) is 13 unit, then x =
(a) 10 (b)
∴ Area of ∆ OCD
(c) 12 (d)
Ans: (c) √
x2 + 25 = 169
∴ Area of ∆ OAB
x2 = 169 – 25 = 144
∴ Area of trapezium ∴x=√
= 31.5 sq. units
Q22. If x = 1 + √ √ then the value of (2x 4 – 8x3 –
5x2 + 26x – 28) is
Q19. If , then the value of
(a) √ (b) 0
will be
(c) √ √
(a) 535 (b) 1030 Ans: (a) x – 1 = √ √
Page 48 of 516
On squaring,
x2 – 2x + 1 = 2 + 3 + √
https://telegram.me/aedahamlibra
b+1=0 b=–1
c+1=0 c=–1
√ ∴ 2a – 3b + 4c = 2 + 3 – 4 = 1
On squaring again,
x4 + 4x2 + 16 – 4x3 – 8x2 + 16x = 24 Q26. If x = √ √ then the value of
x4 – 4x3 – 4x2 + 16x – 8 = 0 is
2x4 – 8x3 – 8x2 + 32x – 16 = 0
2x4 – 8x3 – 5x2 + 26x – 28 – 3x2 (a)
+ 6x + 12 = 0 (c)
2x4 – 8x3 – 5x2 + 26x – 28
Ans: (d) √ √
= 3x2 – 6x – 12
x + y = 4; xy = 4 – 3 = 1
= 3 (x2 – 2x – 4)
√ √

Q23. If , t hen the value of is


(a) 70 (b) 50
(c) 110 (d) 55 Q27. If the value of is

Ans: (d) (a)


2
x – 5x + 1 = 0 (c)
x2 – 3x + 1 = 2x
Ans: (b)

4 5 ( ) On multiplying by ,

0( ) ( )1 On squaring,

Q24. If 4x = 18y, then the value of ( )is

(a)
(c) Q28. If 2x – then the value of is
Ans: (b) 4x = 18y
(a)
(c)

( ) Ans: (a)
[on dividing by 2]
2 2 2
Q25. If a + b + c = 2 (a – b – c) – 3 then the value of
2a – 3b + 4c is
(a) 3 (b) 1 [On Squaring]
(c) 2 (d) 4
Ans: (b) a2 + b2 + c2 = 2 (a – b –c) –3
a2 + b2 + c2 = 2a + 2b + 2c + 3 = 0
a2 – 2a + 1 + b2 + 2b + 1 + c2 + 2c + 1 = 0 Q29. If , then the value of
(a –1)2 + (b + 1)2 + (c + 1)2 = 0
is
[If x2 + y2 + z2 = 0 x = 0; y = 0; z = 0]
(a) a (b) b
∴a–1=0 a=1
(c) 0 (d) 2
Page 49 of 516
Ans: (d) https://telegram.me/aedahamlibra
(c) 16 (d) 4
Ans: (a) x2 + y2 + z2 – xy – yz – zx
Applying compodendo and Dividendo

Q30. The area of the triangle formed by the lines5x + [ ]


7y = 35, 4x + 3y = 12 and x-axis is
[ ]
(a) .unit (b) .unit
(c) .unit (d) 10 sq. unit [ ]
Ans: (a) 5x + 7y = 35... (i)
4x + 3y = 12... (ii) Q33. If √ , √ , then the value of
√ √
By equation (i) × 4 – (ii) × 5 4 4 2 2
x + y – 2x y is
on subtracting (20x+28y = 140) (20x+15y = 60) = 13y =
(a) 16 (b) 20
80
(c) 10 (d) 5
= Height of triangle Ans: (a) x4 + y4 – 2x2y2
Point of intersection on x-axis of equation (x2 – y2)2 [ (x + y) (x – y)]2
5x + 7y = 35
5x + 7 × 0 = 35 [(√ √ ) (√ √ )]
√ √ √ √
5x = 35
x=7 ( √ )

∴ (7, 0) Similarly, point of intersection of
4x + 3y = 12 = (3, 0)
Q34. If a + b + c = 8, then the value of
∴ Base = 7 – 3 = 4 (a – 4)3 + (b – 3)3 + (c – 1)3 – 3 (a – 4) (b – 3) (c – 1) is
∴ Area = sq.unit (a) 2 (b) 4
(c) 1 (d) 0
Q31. The area (in sq. unit) of the triangle formed by Ans: (d) We have x3 + y3 + z3 – 3xyz = (x + y + z)
the three graphs of the equations x = 4, y = 3, and 3x + (x2 + y2 + z2–xy – yz - zx)
4y = 12, is Here x = a – 4, y = b – 3, z = c –1
(a) 12 (b) 10 So, given expression is (x + y + z)
(c) 6 (d) 8 (x2 + y2 + z2 – xy – yz – zx)
Ans: (c) x = 4... (1) = (a – 4 + b – 3 + c – 1) (x2 + y2 + z2 – xy – yz – zx)
y = 3... (2) = (a + b + c – 8) (x2 + y2 + z2 – xy – yz – zx)
3x + 4y = 12... (3) = (8 – 8) (x2 + y2 + z2 – xy – yz – zx)
Putting x = 0 in 3rd equation we get y = 4 =0
Putting y = 0 in 3rd equation we get x = 3
The triangle will be formed by joining the points (3, 0) Q35. If √ , then the value of √ is

and (0, 4).
(a) √
So, base = 3 and altitude = 4
(c) √
Area =
Ans: (b) √
2
Q32. If x = 997, y = 998, z = 999, then the value of x + √
y2 + z2 √ √
– xy – yz – zx will be √
(a) 3 (b) 9 √

Page 50 of 516
(√ )
https://telegram.me/aedahamlibra
Q40. If = 0, then the value of

is
(√ ) √ √ =4
√ (a) 15 (b) 3
(√ ) √ (c) 5 (d) 10
√ Ans: (c)

Q36. If , then the value of is


(a)
(c)
Ans: (d) [ ]

Multiply by on both sides [ ]


( )

Squaring on both sides Q41. If (3x – y) : (x + 5y) = 5 : 7, then the value of (x +


y) : (x – y) is
(a) 3 : 1 (b) 1 : 3
(c) 2 : 3 (d) 3 : 2
Ans: (a)
Q37. If a, b, c are real and a 3 + b 3 + c 3 = 3abc and 16x = 32y
, then the relation between a, b, c will be x = 2y or ₹... (1)
(a) a + b = c (b) a + c = b
Now, to calculate value of Divide numerator &
(c) a = b = c (d) b + c = a
Ans: (c) denominator by y.

Q38. If , the value of


( ) is
Putting value of from equation (1)
(a) 2 (b) 3
(c) 4 (d) 5
Ans: (b) If a + b + c = 0
then a3 + b3 + c3 = 3abc
Dividing both sides by abc
Q42. If a + b = 5, a 2 + b 2 = 13, the value of a – b
(where a > b) is
(a) 2 (b) – 1
(c) 1 (d) – 2
Ans: (c) a + b = 5
Q39. Minimum value of is Squaring on both sides
(a) – 3 (b) – 2 (a + b)2 = (5) 2
(c) 0 (d) – 1 a2 + b2 + 2ab = 25
13 + 2ab = 25
Ans: (b)
2ab = 25 – 13 = 12... (1)
is minimum when x = 0 Again, a2 + b2 = 13
Subtracting (– 2ab) from both sides
a2 + b2 – 2ab = 13 – 2ab
(a – b)2 = 13 – 12 from equation (1)
Page 51 of 516
(a – b)2 = 1
TRICK a = 3
https://telegram.me/aedahamlibra
b = 2 (a > b)
a–b=1

Q43. The shaded region represents

(a) y = – x (b) y = – 3x
(a) y ≤ x (b) y ≥ – x
(c) y = x (d) y = 3x
(c) y ≥ x (d) y ≤ – x
Ans: (d)
Ans: (a)

Q44. The line passing through the points ( – 2, 8) and Q47. If , then the value of x is :
(5, 7) (a) 11 (b) 19
(a) does not cut any axes (b) cuts x-axis only (c) 23 (d) 7
(c) cuts y-axis only (d) cuts both the axes Ans: (b)
Ans: (c)
9x + 15 = 10x – 4
15 + 4 = 10x – 9x
x = 19

Q48. If , then :
(a)
(c) (d) none of the above is
true
Ans: (a) (say)
So, x = k (b + c)
x – y = k (b + c) – k (c + a)
= k (b – a)
y = k (c + a)
As indicated in the graph, the line passing through the y – z = k (c + a) – k (a + b)
points cuts Y-axis only. = k (c – b)
z = k (a + b) = z – x = k (a + b) – k (b + c) = k (a – c)
Q45. The equation of this graph is So,check option (a)

k=k=k
option (a) is true.

Page 52 of 516
Q49. If √ https://telegram.me/aedahamlibra
√ , then the value of is :
This is possible when and
(a) √ √
(c) √ √
a = 1, b = –1, c = –1
Ans: (c) √ √ Thus, 2a – 3b + 4c = 2 (1) – 3 (–1) + 4 (–1)
√ √ √ √ = 2 + 3 – 4 = 1.
√ √
√ √ √ √
Now, (√ √ ) (√ √ ) Q53. If xy + yz + zx = 0, then

= (a + b)3 – (a – b)3 [Let √ = a and √ = b] ( ) is equal to


= a3 + b3 + 3a2b + 3b2 – (a3 – b3 – 3a2b + 3b2a) (a) 0 (b) 3
= a3 + b3 + 3a2b + 3b2 – a3 + b3 + 3a2b – 3b2a (c) 1 (d) x + y + z
= 2b3 + 6a2b = (√ ) (√ ) (√ ) Ans: (d)
= √ √ √

Q50. If the difference of two numbers is 3 and the


difference of their squares is 39; then the larger
number is :
(a) 9 (b) 12
(c) 13 (d) 8
Ans: (d) Let the numbers are x, y.
x – y = 3… (1)
x2 – y2 = 39 Q54. If a = then the value of is
(x – y) (x + y) = 39 (a) 2 (b) 6 ab
x + y = 13… (2) (c) 0 (d) 1
Adding eqn (1) and (2) Ans: (c) Given a = or or
x + y + x – y = 16
We know,
x=8
∴ (a + b) (ab – ab) 0 (using given)
∴y = 3
Hence, 8 is the larger number.
Q55. The linear equation such that each point on its
graph has an ordinate four times its abscissa is:
Q51. Let √ √ √ √ (a) y + 4x = 0 (b) y = 4x
Then point out the correct alternative among the four
(c) x = 4y (d) x + 4y = 0
alternatives given below.
Ans: (b) y = 4x,
(a) a < b < c (b) b < a < c
When, x = 1, y = 4
(c) a < c < b (d) b < c < a ;
Ans: (a) √ √ √ Q56. If x = a – b, y = b – c, z = c – a, then the
a=√ √ numerical value of the algebraic expression
b=√ x3 + y3 + z3 – 3xyz will be
c= √ (a) a + b + c (b) 0
(c) 4 (a + b + c) (d) 3 abc
Q52. If a2 + b2 + c2 = 2 (a – b – c) – 3, then the value of Ans: (b) x + y + z = a – b + b – c + c – a = 0
2a – 3b + 4c is ∴ x3 + y3 + z3 –3xyz = 0
(a) 1 (b) 7
(c) 2 (d) 3 Q57. If the square of the sum of two numbers is equal
Ans: (a) to 4 times of their product. then the ratio of these
numbers is :
(a) 2 : 1 (b) 1 : 3
(c) 1 : 1 (d) 1 : 2
Ans: (c) Let the number be x and y.
Page 53 of 516
According to question,
(x + y)2 = 4xy
https://telegram.me/aedahamlibra
Q62. In a two -digit number, the digit at the unit's
place is 1 less than twice the digit at the ten‘s place. If
x2 + y2 + 2xy – 4xy = 0 the digits at unit‘s and ten‘s place are interchanged,
(x – y)2 = 0 the differen ce between the new and the original
x=y number is less than the original number by 20. The
original number is
Q58. One of the factors of the expression (a) 47 (b) 59
√ √ is: (c) 23 (d) 35
(a) √ (b) 4 x + 3 Ans: (a) Since two digit number = 10x + y
According to question → y = 2x – 1.. (i)
(c) 4x – 3 (d) 4x – √
When digits are interchanged then new number
Ans: (d) √ √
= 10y + x
= √ √ then original number – [new number – original number]
(√ ) √ (√ ) = 20
=( √ )(√ ) → 10x + y – [10y + x – (10x + y)] = 20
→ 10x + y – 10y – x + 10x + y = 20
Q59. Divide 5 0 into two parts so that the sum of their 19x – 8y = 20
reciprocals is 1/12. 19x – 8 (2x – 1) = 20 (Using eq. (i) )
(a) 28, 22 (b) 35, 15 19x – 16x + 8 = 20
(c) 20, 30 (d) 24, 36 3x = 12 →x = 4
Ans: (c) From (i) y = 2 × 4 – 1 = y = 7
∴original number = 10x + y = 10×4 + 7 = 47
x2 – 50x + 600 = 0
x2 – 30x – 20x + 600 = 0
Q63. A man buys 3 cows and 18 goats in ₹ 47,200.
x (x – 30) – 20 (x – 30) = 0
Instead if he would have bought 8 cows and 3 goats,
x = 30, 20
he had to pay ₹ 53,000 mroe. Cost of one cow is :
(a) ₹ 10,000 (b) ₹ 11,000
Q60. If a2 + b2 = 5ab, then the value of ( )is : (c) ₹ 12,000 (d) ₹ 13,000
(a) 32 (b) 16 Ans: (c) C.P of 1 cow = ₹ x
(c) 23 (d) –23 C.P of a goat = ₹ y
Ans: (c) a2 + b2 = 5ab 3x + 8y = 47200... (i)
8x + 3y = 100200... (ii)
By equation (i) × 3 – (ii) × 8, 9x + 24y – 64x – 24y
= 141600 – 801600
On squaring both sides. 55x = 660000
∴( ) x= = ₹12000
= 25
Q64. If a + b + c = 9 (where a, b, c a re real numbers),
= 25 – 2 = 23 then the minimum value of
a2 + b2 + c2 is
Q61. The equation cos2θ = is only possible when (a) 81 (b) 100
(c) 9 (d) 27
(a) x < y (b) x = –y
Ans: (d)
(c) x > y (d) x = y
Ans: (d) =
will be minimum if ab + bc + ca is
maximum.
sin2θ cannot be – ve ab + bc + ca is maximum when a = 3, b = 3, and c = 3.
Both sin2θ and cos2θ will be +ve when x = y [

Page 54 of 516
a + b + c = 9]
∴ minimum value of
https://telegram.me/aedahamlibra
c + 1 = 0 c = –1
∴ 2a – b + c = 2 + 1 – 1 = 2
= 81 – 2 (3 × 3 + 3 × 3 + 3 × 3)
= 81 – 54 = 27 Q69. If ( ) = 4, then the value of is :
(a) 124 (b) 64
Q65. If , then the value of is: (c) 194 (d) 81
(a) 27 (b) 81 Ans: (c) ( )=4
(c) 110 (d) 125
On squaring both sides
Ans: (c)
= 16
=3
x2 + = 14
=3
On squaring again
=5
= 196
On cubing both sides
= 194
( ) = 125
= 125 – 3 × 5 = 110
Q70. If ( ) be a perfect square, then the
values of t are :
Q66. If p – 2q = 4, then the value of p 3 – 8q3 – 24pq –
(a) ±1 (b) ±2
64 is :
(c) 1, 2 (d) 2, 3
(a) –1 (b) 2
(c) 0 (d) 3 Ans: (a) For nr – tn + to be a perfect square,
Ans: (c) p – 21 = 4 r = 2 and t = 1
cubing both sides, Look :
(p – 2q)3 = 64 n2 – n + = n2 – 2.n. =( )
p3 – 8q3 + 3p. 4q2 – 3p2 . 2q = 64
p3 – 8q3 + 12pq2 – 6p2q = 64 =( )
p3 – 8q3 – 6pq (p – 2q) = 64
p3 – 8q3 – 6pq × 4 = 64 Q71. If (x + 7954 × 7956) be a square number, then
p3 – 8q3 – 24pq – 64 = 0 the value of ‗x‘ is
(a) 1 (b) 16
Q67. If , then the value of x – x2 is : (c) 9 (d) 4
(a) a (b) – a Ans: (a) x + 7954 × 7956
x + 7954 (7954 + 2)
(c) (d) –
x + 79542 + 2 × 7954 × 1
Ans: (a) Putting x = 1
(x + 7954)2 or (1 + 7954)2
x2 = x – a x – x2 = a
Q72. Equation of the straight line parallel to x -axis
2 2 2 and also 3 units below x-axis is :
Q68. If a + b + c + 3 = (2 (a – b – c), then the value
(a) x = 3 (b) x = –3
of 2a – b + c is :
(c) y = 3 (d) y = –3
(a) 2 (b) 3
Ans: (d)
(c) 4 (d) 0
Ans: (a) a2 + b2 + c2 + 3
= 2a – 2b – 2c
a2 – 2a + 1 + b2 + 2b + 1 + c2 * + 2c + 1 = 0
(a – 1)2 + (b + 1)2 + (c + 1)2 = 0
∴a–1=0 a=1
b + 1 = 0 b = –1

Page 55 of 516
https://telegram.me/aedahamlibra
Q76. If a + b + c = 2s, then
is equal to
2 2 2
(a) a + b + c (b) 0
(c) 1 (d) 2
Ans: (c) Expression
=

=
∴ Equation is : y = –3
= =1
Q73. If , then the value of
Q77. The minimum value of (x – 2) (x – 9) is
( ) is
(a)
(a) 0 (b) 2
(c) – 2 (d) 4 (c) 0 (d)
Ans: (b) a + =4 Ans: (d) Expression = (x – 2) (x – 9)
= x2 – 11x + 18 = ax2 + bx + c
(a – 2) + =4–2=2
Minimum value =
On squaring,
=
(a – 2)2 + =4
Alternate Method:
(a – 2)2 + In this type of questions take (x – a) (x – b) = 0
and value of x = for minimum value and x = a +b for
Q74. If the number p is 5 more than q and the sum of
maximum value of equation.
the squares of p and q is 55, then the pr oduct of p and
q is Required value = ( )( )= =
(a) 10 (b) – 10
(c) 15 (d) – 15 Q78. If a 3 – b3 – c3 = 0 then the value of a 9 – b9 – c9 –
Ans: (c) p = q + 5 3a3b3c3 is
p–q=5 (a) 1 (b) 2
p2 + q2 = 55 (c) 0 (d) – 1
∴ (p – q)2 + 2pq = 55 Ans: (c) If a + b + c = 0, then
25 + 2 pq = 55 a3 + b3 + c3 – 3abc = 0
2 pq = 30
pq = 15 Q79. The third proportional to
( ) and √ is
Q75. If xy (x + y) = 1 then, the value of
(a) xy (b) √
is
(c) √ √
(a) 3 (b) – 3 Ans: (a) Third proportional of a and b
(c) 1 (d) – 1 a:b:c
Ans: (a) xy (x + y) = 1
x+y=
Cubing both sides, c=
x3 + y3 + 3xy (x + y) = (√ )
= = xy
3 3
x +y +3×1=
Alternate Method:
– x3 + y3 = 3
Page 56 of 516
Proportional)2
https://telegram.me/aedahamlibra
First Proportional × Third Proportional = (Mid Q83. An equation of the form ax + by + c = 0 where a
≠ 0, b ≠ 0, c = 0 represents a straight line which passes
∴ Third Proportional = through
(a) (0, 0) (b) (3, 2)
Required value (c) (2, 4) (d) None of these
(√ ) Ans: (a) ax + by +c = 0
=
When c = 0,
ax + by = 0
Q80. If x + y + z = 6 and x 2 + y 2 + z 2 = 20 then the
value of x3 + y3 + z3 – 3xyz is
(a) 64 (b) 70 When x = 0, y = 0 i,e. this line passes through the origin
(c) 72 (d) 76 (0, 0).
Ans: (c) x + y + z = 6
On squaring, Q84. If a 2 + b2 + 4c2 = 2 (a + b – 2c) – 3 and a, b, c are
x2 + y2 + z2 + 2xy + 2zy + 2zx = 36 real, then the value of (a2 + b2 + c2) is
20 + 2 (xy + yz + zx) = 36 (a) 2 (b)
xy + yz + zx = 8 (c) 3 (d)
∴ x3 + y3 + z3 – 3xyz
Ans: (b) a2 + b2 + 4c2 = 2a +2b – 4c – 3
= (x + y + z) (x2 + y2 + z2 – xy – yz – zx)
a2 + b2 + 4c2 – 2a –2b + 4c + 3 = 0
= 6 (20 – 8) = 72
a2 – 2a + 1 + b2 – 2b + 1 + 4c2 + 4c + 1 = 0
(a –1)2 + (b – 1)2 + (2c + 1)2 = 0
Q81. If , then the value of ∴ a – 1 = 0 a = 1;
is b – 1 = 0 b = 1;
(a) 4 (b) 6 2c + 1 = 0 c
(c) 9 (d) 3
Ans: (a)

Q85. If 3x – = 6, then the value of 4x – is


(a) 2 (b) 4
(c) 6 (d) 8
Ans: (d)
2
Q82. If x – 3x + 1 = 0, then the value of 12 xy – 1 = 24y
Now,
is
(a) 6 (b) 8
(c) 10 (d) 2
Ans: (c) x2 – 3x + 1 = 0 Q86. The expression x 4 – 2x2 + k will be a perfect
x2 + 1 = 3x square when the value of k is
Dividing both sides by x, (a) 1 (b) 2
(c)
Ans: (a) x4 – 2x2 + k
(x2)2 – 2x2 + k (x2)2 – 2.1. x2 + k
For above ex pression to make a perfect square, the k
( ) ( ) ( ) ( )
value is equal to 1.
= 9 – 2 + 3 = 10
Q87. If x + = 4, find the value of .

Page 57 of 516
(a) 8 https://telegram.me/aedahamlibra
(b) 8 Ans: (a) Let Puneet‘s age = x yr.
Let Puneet‘s father age = y yr.
(c) 16 (d) 16
x + y = 45 y = (45 – x)
Ans: (b) x + xy = 126
x2 + 4 = 4x x2 – 4x + 4 = 0 (x – 2)2 = 0 Putting the value of y.
x=2 (x) (45 – x) = 126
45x – x2 = 126
x2 – 45x + 126 = 0
x2 – 42x – 3x + 126 = 0
x (x – 42) – 3 (x – 42) = 0
x = 3, x = 42
Q88. If a + b + c = 0, find the value of . Hence, Puneet‘s age in 3yr.
(a) 0 (b) l
(c) – 1 (d) 2 Q92. If a = 0, b ≠ 0, c ≠ 0, then the equation ax + by +
Ans: (a) a + b + c = 0 c = 0 represents a line parallel to
i.e. a = – (b + c); b = – (c + a); c = – (a + b) (a) x + y = 0 (b) x-axis
Now, (c) y-axis (d) none of these
[ ]
Ans: (b) It a = 0, b ≠ 0, c ≠ 0, then equation ax + by + c =
0
–1+2–1=0 represents a line parallel to x-axis.

Q89. If ‗a‘ be a positive number, then the least value Q93. If = 1, then the value of
of a + is is
(a) 1 (b) 0 (a) 1 (b) 3
(c) 2 (d) (c) 4 (d) 0
Ans: (c) The least value of is 2 where a = 1. Ans: (c) =1
=4
Q90. If x = 3 + 2√ , then the value of (√ ) is =4

(a) 1 (b) 2
=4
(c) 2√ (d) 3√
Ans: (b) x = 3 + 2√
Q94. The total cost of 8 buckets and 5 mugs is ₹ 92
x = 2 + 1 + 2√
and the total cost of 5 buckets and 8 mugs is ₹ 77.
x = (√ ) √ Find the cost of 2 mugs and 3 buckets.
x = (√ ) (a) ₹ 35 (b) ₹ 70
(√ )... (1) (c) ₹ 30 (d) ₹ 38

Ans: (a) C.P. of 1 bucket = ₹ x
√ √
√ C.P. of 1 mug = ₹ y
√ √ √
∴ 8x + 5y = 92... (i)
Now, √ √ (√ ) 5x + 8y = 77... (ii)

√ √ By equation (i) × 5 – equation (ii) × 8,
40x + 25y – 40x – 64y = 460 – 616
√ –39y = –156

y=4
Q91. The sum of the ages of Puneet and his father is From equation (i) ,
45 years and the product of their ages is 126. What is 8x + 20 = 92
the age of Puneet ? 8x = 92 – 20 = 72
(a) 3 years (b) 5 years x=9
(c) 10 years (d) 45 years ∴ C.P. of 2 mugs and 3 buckets
Page 58 of 516
=2×4+3×9
= 8 + 27 = ₹ 35
https://telegram.me/aedahamlibra
=

Q99. If a2 + b2 + c2 = 2a – 2b – 2, then the value of 3a –


Q95. If + 2P = 12 for what value of P, x = 6 ? 2b + c is
(a) 6 (b) 14 (a) 0 (b) 3
(c) 2 (d) 1 (c) 5 (d) 2
Ans: (c) When x = 6. Ans: (c) a2 + b2 + c2 = 2a – 2b – 2
+ 2P = 12 (a2 – 2a + 1) + (b2 + 2b + 1) + c2 = 0
(a – 1)2 + (b + 1)2 + c2 = 0
8 + 2P = 12
This equation is possible if
2P = 12 – 8 = 4 P=2
a – 1 = 0, b + 1 = 0 and c = 0
a = 1, b = –1, c = 0
Q96. If (x – 3)2 + (y – 5)2 + (z – 4)2 = 0, then the value
3a – 2b + c = 3 × 1 – 2 × (–1) + 0 = 3 + 2 = 5
of
is Q100. The straight line 2x + 3y = 12 passes through :
(a) 12 (b) 9 (a) lst, 2nd and 3rd quadrant (b) lst, 2nd and 4th
(c) 3 (d) 1 quadrant
Ans: (c) (x – 3)2 + (y – 5)2 + (z – 42) = 0 (c) 2nd, 3rd and 4th quadrant (d) lst, 3rd and 4th
x–3=0 x=3 quadrant
y–5=0 y=5 Ans: (b) Putting y = 0 in 4x + 3y = 12, we get x = 3
z–4=0 z=4 Putting x = 0 in 4x + 3y = 12, we get, y = 4

= =1+1+1=3

Q97. What number must be added to the expression


16a2 –12a to make it a perfect square ?
(a)
(c) (d) 16
Ans: (a) a2 – 2ab + b2 = (a – b)2
∴ 16a2 – 12a
Q101. If a 2 – 4a – 1 = 0, a ≠ 0, then the value of a2 + 3a
= (4a)2 – 2 × 4a ×
+ – is
Hence, on adding ( ) (a) 24 (b) 26
Expression will be a perfect square. (c) 28 (d) 30
Ans: (d) a2 – 4a – 1 = 0
Q98. If ( ) , then the value of is a2 – 4a = 1
a (a – 4) = 1
(a)
a–4=
(c)
= 4 ... (1)
Ans: (b) 3x – 2 = 3x – = 2
We have a2 + 3a +
x–
On squaring both sides ( ) ( )

( ) ( ) ( )
2
4 + 3 × 4 + 2 = 30

Page 59 of 516
https://telegram.me/aedahamlibra
Q102. If a + b + c = 3, a2+ b2 + c2 = 6 and = 1, (a) –2
(c) 0
(b) –1
(d) 9.932
where a, b, c are all non-zero, then 'abc' is equal to
Ans: (c) a = 4.965 ≈ 5, b = 2.343 ≈ 2
(a) c = 2.622
(c) a–b=c
Ans: (b) a + b + c = 3 taking cube both sides
Squaring both sides a3 – b3 – 3a2b + 3ab2 = c3
a2 + b2 + c2 + 2 (ab + bc + ac) = 9 a3 – b3 – c3 – 3ab (a – b) = 0
6 + 2 (ab + bc + ca) = 9 a3 – b3 – c3 – 3abc = 0
ab + bc + ca = ... (1)
Q106. If x = 2 + √ , then x2 + is equal to
given =1
(a) 10 (b) 12
ab + bc + ac = abc = [from (1) ] (c) –12 (d) 14
Ans: (d) x = √
Q103. For what value of k, the system of equations kx √
= = √
+ 2y = 2 and 3x + y = 1 will be coincident? √ √

(a) 2 (b) 3 =( ) –2
(c) 5 (d) 6
Ans: (d) Kx + 2y = 2... (1) =( √ √ )
3x + y = 1... (2) = 16 – 2 = 14
divide eqn (1) by (2)
Q107. If x : y ::2 : 3 and 2 : x :: 4 : 8 the value of y is
=1
(a) 6 (b) 8
for system of equation to be coincident (c) 4 (d) 12
=3 Ans: (a) ;
K=6
x=4
=
Q104. The total area (in sq. unit) of the triangles
formed by the graph of 4x + 5y = 40, x-axis, y-axis and
x = 5 and y = 4 is Q108. If x + y + z = 0, then the value of is
(a) 10 (b) 20
(a) –1 (b) 0
(c) 30 (d) 40
(c) 1 (d) 2
Ans: (a)
Ans: (d) x + y + z = 0
y + z = –x
y2 + z2 + 2yz = x2
y2 + z2 = x2 – 2yz... (1)
= =2

Q109. If p = then 27 p3 – is equal to


(a)
(c)
Ans: (c)
Area of ∆ABC = × BC × AC
= × (10 – 5) × 4 = × 5 × 4 = ( ) ( )
Area = 10 sq unit. =( ) ₹

Q105. If a = 4.965, b = 2.343 and c = 2.622, then the =( )


value of a3 – b3 – c3 – 3abc is
Page 60 of 516
Q110. If a = √
2
2b =
√ b =√ https://telegram.me/aedahamlibra
√ then 2a 2 – 5ab + (c)
Ans: (d)
(a) 38 (b) 39
(c) 40 (d) 41
Ans: (b) 2a2 – 5ab + 2b2 Q116. If x = 3 + √ , then is equal to
2 (a2 – 2ab + b2) – ab (a) 216 (b) 192
2 (a – b)2 – ab (c) 198 (d) 204

[√ √ √ √ ] (√ √ )(√ √ ) Ans: (d) We have, √


2 × 4 × 5 – 1 = 39 √

√ √
Q111. If a = 331, b = 336 and c = – 667, then the value
of
a3 + b3 + c3 – 3abc is
(a) 1 (b) 6
(c) 3 (d) 0 =( ) ( )
Ans: (d) Here, a + b + c = 0 =( )( ) ( )
∴ a3 + b3 + c3 – 3abc = 0
=( ) [( ) ] ( )
2013 2
Q112. If = 2, then x + =? = 6[6 – 3] + 6 = 198 + 6 = 204
(a) 0 (b) 1
(c) –1 (d) 2 Q117. If = √ , then the value of
Ans: (d) will be :
x2 –2x + 1 = 0; (x – 1)2 = 0 ; x = 1 (a)
√ √
=1+1=2 (c) √ (d) 3
Ans: (c) 3 2x – y
=3 x+y
=√
4 2
Q113. If x = p + and y = p – , then value of x - 2x 2x – y = x + y =
y2 + y4 is 4x – 2y = 3 ... (i)
(a) 24 (b) 4 2x + 2y = 3 ... (ii)
(c) 16 (d) 8 Solving equation (i) and (ii)
Ans: (c) x4 – 2x2y2 + y4 = (x2 – y2)2 = [ (x + y) (x – y)]2 x=1y=
=( ) √

2
Q114. If = 3, then the value of is Q118. The mean of x and is M. Then the mean of x

(a) and is.

(c) (a) M2 (b) M2–2


(c) 4M2–2 (d) 2M2–1
Ans: (c) Ans: (d) Mean of x and = M
( )=M
( )
=M
( )
Mean of x2 and = ( )

Q115. If = and x + y + z =
0, then each ratio is =
(a)
=

Page 61 of 516
= https://telegram.me/aedahamlibra
Now multiplying both sides by x

= 2M2 –1 ( ) = 2x = 2x
=2
Q119. The term that should be added to (4x 2 + 8x) so 2
x +1 = 2x
that resulting expression be a perfect square is: x2 – 2x + 1 = 0
(a) 4 (b) 1
x2 – x – x +1 = 0
(c) 2x (d) 2
x (x – 1) –1 (x – 1)
Ans: (a) 4x2 + 8x = (2x)2 + 2 (2x) (2) + (2) 2 (x –1)2 = 0
= (2x + 2)2 ∴ x2 + y2 + 2xy = (x + y)2 x=1
So, 4 should be added to make if perfect square.
So = 2 (1) = 2
Q120. If999x + 888y = 1332
888x + 999y = 555 Q123. If for non –zero x, x 2 – 4x – 1 = 0 the value of
then the value of x+y is: is :
(a) 888 (b) 1 (a) 10 (b) 4
(c) 999 (d) 555 (c) 12 (d) 18
Ans: (b) 999x + 888y = 1332 Ans: (d) – 4x – 1 = 0 can be written as – 4x + 4 – 1
111 (9x + 8y) = 1332 =4
9x + 8y = = 12 So –1=4
9x + 8y = 12 ... (i) x–2=√
888x + 999y = 555 x=√
8x + 9y = 5 ... (ii)
So, =( √ ) +
Solving (i) and (ii) ( √ )
x=4y=–3 =4+5+ √ +

x+y=4–3=1
= √ +

Q121. If p = –0.12, q = –0.01 & r = –0.015, then the ( √ )
=
correct relationship among the three is : √

(a) q > p > r (b) p > q > r =

(c) p > r > q (d) p < r < q √
Ans: (d) p = – 0.12; q = – 0.01; r = – 0.015 =

So, p < r ;p < q √ √
= ×
√ √
=p<r<q
( √ )( √ )
=
Q122. If x + = 2, then the value of is: = (√ ) √
(a) 25 (b) 212 = 1458 – 648√ + 648 √ – 1440
(c)
2 (d) 27 = 1458 – 1440 = 18.
Ans: (c) =2
√ √ √ √
Take cube on both sides Q124. If a = and b = , then the value of
√ √ √ √
( ) = 23 is :
( )( )=8 (a) 970 (b) 930
(c) 1030 (d) 1025
=8
√ √ √ √
Ans: (a) a = ×
=2 √ √ √ √
(√ √ ) √
Squaring on both sides = = = √ .
=4 √ √ √ √
b= ×
√ √ √ √
=2

Page 62 of 516
=
(√ √ )
= √
https://telegram.me/aedahamlibra = c– (c – 1)
=c–c+1
( √ ) ( √ ) =1
So, + = +
√ √
( √ ) ( √ )
= Q128. If (2a – 1)2 + (4b – 3)2 + (4c + 5) 2 = 0 then the
( √ )
= value of is:
( √ ) ( √ )( √ ) ( √ ) ( √ )( √ ) (a)
√ √ √ √ (c) (d) 0
=
Ans: (d) (2a – 1) + (4b – 3) + (4c + 5)2 = 0
2 2
= 125 + 125 + 360 + 360
= 250 + 720 = ; ; ; ;
= 970
a3 + b 3 + c 3 – 3abc = (a + b + c) (a 2 + b 2+ c 2 – ab – bc –
Q125. Two positive whole numbers are such that the
ca)
sum of the first and twice the second number is 8 and
But a + b + c = 0
their difference is 2. The numbers are :
So, a3 + b3 +c3 – 3abc = 0
(a) 7,5 (b) 6,4
(c) 3,5 (d) 4,2 So, =0
Ans: (d) Let the two numbers are x and y
So, x + 2y = 8 … (i) Q129. If m = –4, n = –2, then the value of m 3 – 3m2 +
x – y = 2… (ii) 3m + 3n + 3n2 + n3 is
Solving both equations (a) 124 (b) –124
x = 4; y = 2 (c) 126 (d) –126
So, numbers are 4,2 Ans: (d) m3 – 3m2 + 3m + 3n + 3n2 + n3
(– 4)3 – 3 (– 4)2 + 3 (– 4) + 3 (– 2) + 3 (– 2)2 + (– 2)3
Q126. A number of boys raised ₹ 12,544 for a famine – 64 – 48 – 12 – 6 + 12 – 8
fund, each boy has given as many rupees as there – 126
were boys. The number of boys was:
(a) 122 (b) 132 Q130. The length of the portion of the straight line 3x
(c) 112 (d) 102 + 4y = 12 intercepted between the axes is
Ans: (c) Contribution of each boy = Number of boys (a) 3 (b) 4
Total contribution raised = ₹ 12544 (c) 7 (d) 5
So, number of boys = √ = 112 Ans: (d) Intercept can represent in the form of

Q127. If and then is equal To get x and y intercept, we have


to 3x + 4y = 12
(a) 0 (b) 1
(c) 2 (d) So, triplets of 3, 4 and 5.
Ans: (b) a + = 1 and b + = 1 Hence, 5 is the length of portion of straight line.

b=
Q131. If = 3, then the value of m
So, a + =1
is
a+ =1 (a) a2 + b2 (b) a2 + b2 + c2
(c) a2 – b2 – c2 (d) a2 + b2 – c2
a=1–
Ans: (b)
a= =

So, c + = c + . /

Page 63 of 516
https://telegram.me/aedahamlibra
(a) 2x – 3y = 0
(c) x + 2y + 9 = 0
(b) 5x – 4 = 0
(d) 2y – 13 = 0
Ans: (a) 2x – 3y = 0 is passing through origion because
its satisfy x = 0 and y = 0.
[ ]
Q136. The area of the triangle formed by the graphs
=0 of the equations x = 0, 2x + 3y = 6 and x + y = 3 is
(a) 3 sq. unit (b) sq. unit
(c) 1 sq. unit (d) sq. unit
Q132. 2x – ky + 7 = 0 and 6x – 12y + 15 = 0 has no Ans: (b) x = 0... (1)
solution for 2x + 3y = 6... (2)
(a) k = – 4 (b) k = 4 2x + y = 3... (3)
(c) k = 1 (d) k = – 1
Ans: (b) For no solution, a = b

k= =4

Q133. If √ √
, then the simplest value of x
is
(a) 1 (b) –2
(c) 2 (d) –1
Ans: (d) √

( √ )( √ )
√ √ 2x + 3y = 6 ... (ii)
√ √ √ x = 0, y = 2
( √ ) ( √ ) x = 3, y = 0
x=–1 From eqn. (iii) x+ y = 3
x = 0, y = 3
Q134. If x = 332, y = 333, z = 335, then the value of x 3 x = 3, y = 0
+ y3 + z3 – 3xyz is Area made by these three lines
(a) 7000 (b) 8000 = Area of triangle OBC – Area of OAC
(c) 9000 (d) 10000 =
Ans: (a) Using the formula,
=

[ ] Q137. If x = then the value of


(a) 432 (b) 433
[ ] (c) 343 (d) 322
[ ] Ans: (d) Given that

[ ]

On cubing both sides we get


Q135. Among the equations ( )
x + 2y + 9 = 0; 5x – 4 = 0; 2y – 13 = 0; 2x – 3y = 0, the
equation of the straight line passing through origin is–

Page 64 of 516
= 343 – 21 = 322 https://telegram.me/aedahamlibra
Q142. If x = 2 then the value of x3 + 27x2 + 243x + 631
(a) 1233 (b) 1231
Q138. The HCF of x8 – 1 and x4 + 2x3 – 2x – 1 is:
(c) 1321 (d) 1211
(a) x2 + 1 (b) x + 1
Ans: (a) Given x = 2
(c) x2 – 1 (d) x – 1
then x3 + 27x2 + 243x + 631
Ans: (c) Given fraction
= 8 + 108 + 486 + 631
= 1233

HCF of Given fractor


Q143. If √ √ √ √ then √
=
?
Q139. If p = 99 then the value of p (p2 + 3p + 3) (a) √ (b) 72
(a) 999999 (b) 988899 (c) 32 (d) 24
(c) 989898 (d) 998889 Ans: (b) √ √
Ans: (a) GivenP = 99 √ √
thenP (P2 + 3P + 3) By adding both equations
= P ( (P + 1)2 + P + 2) √ ,√
= 99 ( (99 + 1)2 + 101)

= 99 × (10000 + 101)
= 999999 √

Q140. If 5x + 9y = 5 and 125x 3 + 729y 3 = 120 then the √ √ = 8 × 9 = 72
value of the product of x and y is
(a) 135 (b) Q144. If then the value of is
(c) (d) 45 (a) 0 (b) 1
Ans: (b) ... (i) (c) 1/4 (d) 2
... (ii) Ans: (a) dividing Eq. by 2
Now cube both sides of equation (i) , we get
Squaring on both sides

=0
Now, put the value of equation (i)
120 + 135 xy × 5 = 125
135 × 5xy = 125 – 120 Q145. If a 3 – b3 = 56 and a – b = 2 then what is the
135 × 5xy = 5 value of a2 + b2 ?
(a) 12 (b) 20
xy =
(c) 28 (d) 32
Ans: (b) (a – b) = 2, a3 – b3 = 56
Q141. If , then the value of is (a – b)3 = a3 – b3 – 3ab (a – b)
(a) 2 (b) 0 8 = 56 – 3ab (2)
(c) 3 (d) 1 – 48 = – 6ab
Ans: (b) ∴ ab = 8
(a – b)2 = a2 + b2 – 2ab
4 = a2 + b2 – 16
20 = a2 + b2
C–3=
158
(C – 3)7 = ( ) (c) x2 – 3x + 1 = 0
Dividing Equation by x
(C – 3)7 ( ) =0

Page 65 of 516
https://telegram.me/aedahamlibra Q149. If x + y + z
3 3 3
= 1, and xyz = – 1,
then x + y + z is equal to
√ (a) –1 (b) 1
Q146. If √ then the val ue of ( ) (c) –2 (d) 2
√ √
Ans: (c) (x – 2) (x – p) = x2 – ax + b
. / is

x2 + (– 2 – p)x + (– P) (–2) = x2 – ax + b
√ √ – (2 + p) = – a
(a)
(c) 5/6 (d) 2/3 2=a–p

Ans: (c) √

Q150. If then the v alue of

√ √ is
Now (a) 2 (b) 1
√ (c) 0 (d)
( )0 √
1

Ans: (b) x + y + z = 1, , xyz = –1
√ √
( ). /
√ √ √
√ √

( √ )( ) xy + yz + zx = – 1
√ √
(x + y + z)2 = 1
( )( )
√ √ x2 + y2 + z2 + 2xy + 2yz + 2zx = 1
x2 + y2 +z2 + 2 (–1) = 1
√ √
x2 + y2 +z2 = 3
Q147. The length of the base of an isosceles triangle is x3 + y3 + z3 – 3xyz
2x – 2y + 4z, and its perimeter is 4x – 2y + 6z. Then = (x + y + z) (x2 + y2 +z2 + xy – yz – zx)
the length of each of the equal sides is x3 + y3 + z3 – 3 (–1) = (1) [ (3 – (–1)]
(a) x + y (b) x + y + z x3 + y3 + z3 + 3 = 4
(c) 2 (x + y) (d) x + z =4–3=1
Ans: (d)
Q151. If x = √a + ,y =√ , (a > 0), then the
√ √
4 4 2 2
value of x + y – 2x y is
(a) 16 (b) 20
(c) 10 (d) 5
[ ] Ans: (a) x = √ y=√
√ √
4 4 2 2
x +y –2xy
= (x2 – y2)2
x2 =
2
Q148. If x – 3x + 1 = 0, ,then the value of
y2 =
is
(a) 1 (b) 0 (x2 – y2)2 = * + = (4) 2 = 16
(c) 3 (d) 2
Ans: (d) Base side of isosceles ∆ = 2x – 2y + 4z Q152. If (x – 2) (x – p) = x2 – ax + 6, then the value of
Perimeter = 4x – 2y + 6z (a – p) is (a) 0
Remaining two sides are (b) 1

(c) 2
(d) 3
163.When then value of is

Page 66 of 516
(a) 2/7
(c) 7/2
https://telegram.me/aedahamlibra
(b) 7/8
(d) 8/7
(c) 1/√3 (d) 6√3
Ans: (a) √
Ans: (b) Cubing both Sides
Dividing eq by 2
(√ ) √
Cubing both sides ( )

( )

( ) Q157. If, √ then the value of

( )is

(a) – 1 (b) 1
= =
(c) 0 (d)
Ans: (c)
Q153. If then the value of
is Q158. If x = 31/3 – 3–1/3 then 3x3 + 9x is equal to
(a) 1/2 (b) 1/3 (a) 5 (b) 6
(c) 2/3 (d) 3 (c) 7 (d) 8
Ans: (b) Ans: (d) x = 31/3 – 3–1/3
4x2 + 1 = 5x Cubing on both sides
x3 = * +
3
x =
x3 + 3x =
Q154. If √ then the value of x 3 + 5x2 3x3 + 9x = 8
+ 12x is
(a) 0 (b) 3 Q159. If , then each of them is equal to
(c) 7 (d) 11
(a)
Ans: (b) √
(x + 2) = √ (Cubing both sides) (c)
(x + 2)3 = x2 + 11 Ans: (a)
x3 + 8 + 6x2 + 12x = x2 + 11
x3 + 5x2 + 12x = 3 Q160. If p, q, r are all real numbers, then (p – q)3 + (q
– r)3 + (r – p)3 is equal to
Q155. If x = 222, y = 223, z = 225 then the value of x3 + (a) 0 (b) 3 (p – q) (q – r) (r – p)
y3 + z3 – 3xyz is (c) (p – q) (q – r) (r – p) (d) 1
(a) 4590 (b) 4690 Ans: (b)
(c) 4950 (d) 4960
Ans: (b) x3 + y3 + z3 – 3xyz = Q161. If √ √ and √ √ , then the
2 2 2
[ (x – y) + (y – z) + (z – x) ] value of is
= (a) 1 (b) √
[ (222 – 223)2 + (223 – 225)2 + (225 – 222)2] (c) 6 (d) ( √ )
[ ][ ] = 335 [14] = 4690 Ans: (c)

Q156. If √ then the value of is equal Q162. If 3x + y = 81 and 81x + y = 3, then the value of is
to (a)
(a) 0 (b) 3√3
(c)
Page 67 of 516
Ans: (c) https://telegram.me/aedahamlibra
… (i)
√ √
… (ii)Solving equ. … (i) & (ii) √ √

√(√ ) √
√(√ )

Q163. If , then what is the value of ?
√ √
(a) 14 (b) √
(c) 0 (d) 18
64
Ans: (a) Here, Q167. If x + (1/x) = 2, then what is the value of x +
√ √ x121?
(a) 0 (b) 1
√ √
(
(c) 2 (d) –2
√ ) √
(√ )
Ans: (c) According to question,
√ ,

x2 + 1 = 2x
x2 – 2x + 1 = (x – 1)2
√ ∴ √
∴x – 1 = 0∴x = 1
∴ √ √ ∴x64 + x121 = 1 + 1 = 2.

Q164. If where then the value of Q168. If , then what is the value of

is ?
(a) 5 (b) 7 (a)
(c) 2 (d) 3
Ans: (d) (c)
Ans: (c)
Q165. If , then what is the value of ?
Q169. If a + b + c = 27, then what is the value of (a –
(a) 52 (b) 256
7)3 + (b – 9)3 + (c – 11)3 – 3 (a – 7) (b – 9) (c – 11)?
(c) 1026 (d) 2702
(a) 0 (b) 9
Ans: (d) (c) 27 (d) 81
then, Ans: (a) According to question,
? a + b + c = 27
(a – 7) + (b – 9) + (c – 11) = 27 – 7 – 9 – 11
Let (a – 7) + (b – 9) + (c – 11) = 0
∴a + b + c = 0
then
( (4) 3 – 3 × 4)2 – 2 a3 + b3 + c3 – 3abc = 0
(64 – 12)2 – 2 = (2704 – 2) = 2702 ∴Required answer = 0.

Q166. If √ , then what is the value of Q170. If √ , then what is the value of
√ √
? √ ?

(a) √ √ (a) √ √

(c) √ (c) √ √
Ans: (c) √ Ans: (a) √

Subtraction by 1 in both side.
√ ( √ ) √
√ ( √ ) √ √

Page 68 of 516
(√ ) (√ ) https://telegram.me/aedahamlibra
√ √ 12xy (x2 – y2)
√ *( √ ) +
(√ √ )
√ √ √ √ √
Now,
Q174. If (x – y) = 7, then what is the value of (x – 15)3
√ – (y – 8)3?

√ √ √ √ (a) 0 (b) 343
√ . (c) 392 (d) 2863
(√ ) (√ )
Ans: (a) Here,
Q171. If (x – 2) and (x + 3) are the factors of the (x – y) = 7
equation x 2 + k 1x + k 2 = 0, then what are the values of then,
k1 and k2?
(a) k1 = 6, k2 = –1 (b) k1 = 1, k2 = –6 a3 – b3 = (a – b) (a2 + ab + b2)
(c) K1 = 1, k2 = 6 (d) k1 = – 6, k2 = 1 (x – 15)3 – (y – 8)3 = ?
Ans: (b) (x – 15 – y + 8) [ (x –15)2 + (x – 15) (y – 8) + (y – 8)2]
(x – y – 7)[ (x – 15)2 + (x – 15) (y – 8) + (y – 8)2]
If x – 2 = 0 (7 – 7) [ (x – 15)2 + (x – 15) (y – 8) + (y – 8)2]
∴x=2 0 × [ (x – 15)2 + (x – 15) (y – 8) + (y – 8)2]
then, 0
x2 + k1x + k2 = 0
(2) 2 + k1 × 2 + k2 = 0 Q175. What is the value of ?
2k1 + k2 = – 4... (i)
(a) 1/x (b) x2 – 1
If x + 3 = 0
(c) x + 1 (d) x
∴ x = –3
then, Ans: (a)
x2 + k1x + k2 = 0
(–3)2 + k1x – 3 + k2 = 0
∴ 3k1 – k2 = 9... (ii)
From equation (i) and (ii),
Solve To get Answer.
we get k 1 = 1 and k2 = – 6

√ Q176. If p/q = r/s = t/u = √ , then what is the value of


Q172. If , then what is the value of
√ √ [ (3p2 + 4r2 + 5t2) / (3q2 + 4s2 + 5u2)]?
√ √ (a) 1/5 (b) 5
?
√ √
(c) 25 (d) 60
(a) √ √
Ans: (b) Here,
(c) √ (d) 2

Ans: (d)
then,
Q173. If x – y – √ = – 1 and x + y – √ = 1, then
what is the value of 12xy (x2 – y2)?
(a) 0 (b) 1
(c) √ √ (√ ) (√ ) (√ )
Ans: (d) According to question,
√ ... (i)
√ ... (ii)
From equation (i) and (ii),
We get,
Q177. What is the value of x in the equation
√ and
then,
Page 69 of 516
√ √ √
?
https://telegram.me/aedahamlibra

(a) – 2 (b) 3
(c) 2 (d) none of these Q180. If * + * + , then what
two values of ?
Ans: (c) √ √

(a) – 1 or 2 (b) 1 or – 2
By squaring both sides (c) – 1 or – 2 (d) 1 or 2
Ans: (a)
4√ √ 5 ( )

Q181. The line passing through ( –2, 5) and (6,b) is
perpendicular to the line 20x + 5y = 3. Find b?
√ √
(a) –7 (b) 4
(c) 7 (d) –4
Ans: (c) Here,
20x + 5y = 3
5y = – 20x + 3
∴y = – 4x +
13x2 + 13x = 12x2 + 12x + 6 Slope of 20x + 5y = 3 –4
x2 + 3x – 2x – 6 = 0 We know, product of slopes = –1 for perpendicular lines
x (x + 3) – 2 (x + 3) = 0 Hence, the slope of the line which passes through ( –2, 5)
(x – 2) (x + 3) = 0
and (6, b)
x–2=0∴x=2
x+3=0∴x=–3 Now,

Q178. If , then what is the value of


b–5=2
? ∴b = 5 + 2 = 7

(a) 2431/ 7 (b) 3375/7


Q182. If x – y = 6 and xy = 40, then find x2 + y2 ?
(c) 3375 / 14 (d) 3985 / 9
(a) 116 (b) 80
Ans: (a) Here,
(c) 89 (d) 146
Ans: (a) Here,
x – y = 6, xy = 40, x2 + y2 = ?
(x – y)2 = (6) 2
x2 – 17x + 1 = 0 x2 + y2 – 2. x. y = 36
x2 – 3x + 1 = 14x ∴x2 + y2 = 36 + 2xy
36 + 2 × 40
∴ . / ( )
= 116

[( ) ( )]
Q183. If (p 2 + q 2) / (r 2 + s 2) = (pq) / (rs), then what is
= the value of (p – q) (p + q) in terms of r and s?
(a) (r + s) / (r – s) (b) (r – s) / (r + s)
(c) (r + s)/ (rs) (d) (rs) / (r – s)
Ans: (b)
Q179. If 3x – 8 (2 – x) = –19, then the value of x is
(a) –3/11 (b) –33/11 Q184. If 'a' and 'b' are positive integers such that a 2 –
(c) –3/5 (d) –33/5 b2 = 19, then the value of 'a' is :
Ans: (a) 3x – 8 (2 – x) = –19 (a) 20 (b) 19
3x – 16 + 8x = – 19 (c) 10 (d) 9
11x = – 3 Ans: (c) According to question,
a2 – b2 = 19
Page 70 of 516
(a + b) (a – b) = 19 https://telegram.me/aedahamlibra
Since 19 is prime, one of (a + b) (a – b) is 19
?
(a) 1 (b) 3
Therefore,
(c) 0 (d) 21
(10)2 – (9) 2 = 19
Ans: (d) Here,
∴a = 10

Q185. If a = 2017, b = 2016 and c = 2015, then what is


Then,
the value of a2 + b2 + c2 – ab – bc – ca?
(a) –2 (b) 0 ?
(c) 3 Now,
(c) 4
Ans: (c) Here, a = 2017, b = 2016, and c = 2015
∴a2 + b2 + c2 – ab – bc – ca
(2017)2 + (2016) 2 + (2015) 2 – 2017× 2016 – 2016 × ( )
2015 – 2015 × 2017 = 3

3 2
Q186. If the expression (px – 8x – qx + 1) is
completely divisible by the expression (3x 2 – 4x + 1), Q188. If x 2
– 3x + 1 = 0, then what is the value of
then what will be the value of p and q respectively? ?
(a) (21/4,15/8) (b) (6, 1)
(a) 3 (b) 7
(c) (33/4, 5/4) (d) (1, 6)
(c) 11 (d) 18
Ans: (c) Let p (x) = px3 – 8x2 – qx + 1
Since, (3x2 – 4x + 1) is factor of p (x) , so p (a) = 0 Ans: (d) If x2 – 3x + 1 = 0 then ?
Dividing equation by x
3x2 – 4x + 1 = 0
3x2 – 3x – x + 1 = 0
3x (x – 1) – 1 (x – 1) = 0
(3x – 1) (x – 1) = 0
∴ ∴ ( ) ( )

∴ = (3) 3 – 3 × 3
= 27 – 9
i.e.,
= 18
( )

( ) ( ) Average
p – 24 – 9q + 27 Q1. In a family, the average age of a father and a
p – 9q = –3... (i) mother is 35 years. The average age of the father,
p (1) = px3 – 8x2 – qx + 1 mother and their only son is 27 years. What is the age
p (1) 3 – 8 (1) 2 – q × 1 + 1 of the son ?
p–8–q+1 (a) 12 years (b) 11 years
p – q = 7.... (ii)From Eq. (i) and (ii), (c) 10.5 years (d) 10 years
Ans: (b) Father + mother
= 2 × 35 = 70 years
Father + mother + son
= 27 × 3 = 81 years
∴ Son‘s age = 81 – 70 = 11 years
Q187. If , then what is the
Q2. The average of the first 100 positive integers is
value of
(a) 100 (b) 51
Page 71 of 516
(c) 50.5 https://telegram.me/aedahamlibra
(d) 49.5 = 12
Ans: (c) 1 + 2 + 3 + ......+ n = 2x + 6 = 36 x = = 15
∴ Average last no = 15 + 4 = 19
Alternate Method:
Average of consecutive numbers or A.P. Q7. If the average of 39, 48, 51, 63, 75, 83, x and 69 is
= 60, then the value of x is
(a) 52 (b) 53
Required average = (c) 50 (d) 51
Ans: (a) 39 + 48 + 51 + 63 + 75 + 83 + x + 69 = 60 × 8
Q3. The mean of 50 numbers is 30. Later it was 428 + x = 480
discovered that two entries were wrongly entered as x = 480 – 428 = 52
82 and 13 instead of 28 and 31. Find the correct mean.
(a) 36.12 (b) 30.66 Q8. The average age of four boys, five years ago was 9
(c) 29.28 (d) 38.21 years. On including a new boy, the present average
Ans: (c) Required average age of all the five is 15 years. The present age of the
= 30 + new boy is
(a) 14 years (b) 6 years
= ( ) = 30 – 0.72 = 29.28 (c) 15 years (d) 19 years
Ans: (d) Sum of the present ages of four boys
Q4. The average of odd numbers upto 100 is = 9 × 4 + 20 = 56 years
(a) 50.5 (b) 50 Sum of the present ages of five boys
(c) 49.5 (d) 49 = 15 × 5 = 75 years
Ans: (b) Average of the first n natural odd numbers = n ∴ Present age of new boy
Number of odd numbers upto 100 = 50 = required = 75 – 56 = 19 years
average.
No. of odd no upto 100 = 50 Q9. The mean of 19 observation is 24. If the mean of
Sum of 50 odd no = 502 the first 10 observations is 17 and that of the last 10
Average = = 50. observations is 24, find the 10th observation.
(a) 65 (b) 37
Q5. The average of 18 observations is recorded as 124. (c) –46 (d) 53
Later it was found that two observations with values Ans: (c) 10th observation
64 and 28 were entered wrongly as 46 and 82. Find = 24 × 10 + 17 × 10 – 19 × 24
the correct average of the 18 observations. = 240 + 170 – 456 = – 46
(a) (b) 122
Q10. The average temperature of Monday, Tuesday
(c) 123 (d) and Wednesday was 30°C and that of Tuesday,
Ans: (b) Difference in observations Wednesday and Thursday was 33°C. If the
= 64 + 28 – 46 – 82 = –36 temperature on Monday was 32°C, then the
∴ Correct average temperature on Thursday was:
= = 122 (a) 33°C (b) 30°C
(c) 41°C (d) 32°C
Ans: (c) M + T + W = 90° ... (i)
Q6. The average of three consecutive odd numbers is
T + W + Th = 99°... (ii)
12 more than one third of the first of these numbers.
By equation (ii) – (i)
What is the last of the three numbers ?
Th – M = 9° Th – 32 = 9
(a) 15 (b) 17
Th = 9° + 32 = 41°
(c) 19 (d) Data inadequate
Ans: (c) Let 3 consecutive odd no. be x, x + 2 and x + 4
Q11. The average of 5 consecutive numbers is n. If the
ATQ
next two numbers are also included, the average of
the 7 numbers will
Page 72 of 516
(a) increase by 2
(c) remain the same
https://telegram.me/aedahamlibra
(b) increase by 1
(d) increase by 1.4
He is playing 12th inning increment made in all pre vious
innings
Ans: (b) Let the numbers be n – 2, n – 1, n, n + 1 and n + Total increment = 2 × 11 = 22 runs
2. Their average = n. His current score = 63
Next two consecutive numbers are n + 3 and n + 4. Required average + increment = current score
Therefore the average of 7 consecutive numbers Required average = 63 – 22 = 41

Q15. Out of 10 teachers of a school, one teacher


retires and in his place, a new teacher of age 25 years
joins. As a result, average age of teachers is reduced
by 3 years. The age (in years) of the retired teacher is
:
Q12. The average age of a jury of 5 is 40. If a member (a) 50 (b) 58
aged 35 resigns and a man aged 25 becomes a (c) 60 (d) 55
member, then the average age of the new jury is Ans: (d) Age of retired teacher = 25 + 3 × 10 = 55 years
(a) 30 (b) 38
(c) 40 (d) 42 Q16. The average weight of 12 crewmen in a boat is
Ans: (b) Required average
increased by kg, when one of the crewmen whose
years
weight is 55 kg is replaced by a new man. What is the
weight of that new men ?
Q13. The average of four consecutive even numbers is (a) 58 (b) 60
9. Find the largest number. (c) 57 (d) 59
(a) 12 (b) 6 Ans: (d) Short-cut method:
(c) 8 (d) 10 Weight of new crewmen
Ans: (a) Let the consecutive even numbers are 2n, 2n + = Replaced man's weight + [No. of crew men ×
2, 2n + 4 and 2n + 6 increment in average]
Average = = kg
8n + 12 = 4 × 9 n = 3
Hence, the numbers are 6, 8, 10 and 21. Largest among Q17. A cricket player after playing 10 tests scored 100
them is 12. runs in the 11th test. As a result, the average of his
runs is increased by 5. The present average of runs is
Q14. A batsman in his 12th innings makes a score of (a) 45 (b) 40
63 runs and there by increases his average scores by 2. (c) 50 (d) 55
What is his average after the 12th innings? Ans: (c) If the average in 10 tests be x, then,
(a) 13 (b) 41
=x+5
(c) 49 (d) 87
Ans: (b) Let the average of batsman after 11th innings = x × 10 + 100 = (x + 5) × 11
A 11x – 10x = 100 – 55
x = 45
∴ Required average = 50

∴ Total score after 11th innings = 11 A Q18. The average of 50 numbers is 38. If two numbers
Now, namely 45 and 55 are discarded, the average of the
remaining numbers is :
(a) 36 (b) 35
(c) 32.5 (d) 37.5
11A + 63 = (A + 2) × 12 Ans: (d) New average
11A – 12 A = 24 – 63
= = = 37.5
A = 39
12th innings average = 39 + 2 = 41
Alternate Method:
Page 73 of 516
3 is
https://telegram.me/aedahamlibra
Q19. The average of the first nine integral multiples of 15x = 300 – 60 = 240
x= = 16
(a) 15 (b) 18
(c) 21 (d) 12
Q24. If a, b, c, d, e are five consecutive odd numbers,
Ans: (a) Required average
their average is
(a) 5 (a + 4) (b)
(c) 5 (a + b + c + d + e) (d) a + 4
Q20. The average of nine numbers is 50. The average Ans: (d) b = a + 2
of the first five numbers is 54 and that of the last three c=b+2=a+4
numbers is 52. Then the sixth number is d=c+2=a+6
(a) 24 (b) 44 e=d+2=a+8
(c) 30 (d) 34 ∴ Required average
Ans: (a) The sixth number = 9 × 50 – 5 × 54 – 3 × 52 =
= 450 – 270 – 156 = 24
= =a+4
Q21. The average of the first five multiples of 7 will be
(a) 14 (b) 21 Q25. Average of first five prime numbers is
(c) 17.5 (d) 24.5 (a) 3.6 (b) 5.3
Ans: (b) Average (c) 5.6 (d) 5
Ans: (c) First five prime numbers are 2, 3, 5, 7, 11

Q22. Out of 40 boys in a class, average weight of 30 is Average =


60 kg and the average weight of the remaining is 56
kg. The average weight (in kilogra m) of the whole Q26. Out of 20 boys, 6 are ea ch of 1 m 15 cm height, 8
class is are of 1 m 10 cm and rest of 1 m 12 cm. The average
(a) 58.5 (b) 58 height of all of them is
(c) 57 (d) 59 (a) 1 m 12 cm (b) 1 m 12.1 cm
Ans: (d) Average weight of 30 = 60 kg (c) 1 m 21.1 cm (d) 1 m 21 cm
Sum of weight of 30 boys = 1800 Ans: (b) Average height =
Average weight of 10 = 56 kg
= or 1m 12.1cm
Sum of weight of 10 boys = 560
Average weight of the whole class
Q27. On a journey across Kolkata, a taxi averages 50
=
km per hour for 50% of the distance. 40 km per hour
for 40% of it and 20 km per hour for the remaining.
The average speed in km/hour, for the whole journey
is :
(a) 42 (b) 40
(c) 35 (d) 45
Alternate Method:
Ans: (b) Total distance = 100 km.
30 : 10 or 3 : 1 (Total 40 boys)
Averages 60 and 56 kgs. Total time = = = hours
Final average = kg ∴Average speed = = 40 kmph

Q23. The average of 20 numbers is 15 and the average Q28. The batting average for 30 inings of a cricket
of first five is 12. The average of the rest is player is 40 runs. His highest score exceeds his lowest
(a) 16 (b) 15 score by 100 runs. If t hese two innings are not
(c) 14 (d) 13 included, the average of the remaining 28 innings is 38
Ans: (a) If the average of remaining numbers be x, then runs. The lowest score of the player is:
20 × 15 = 5 × 12 + 15x (a) 15 (b) 18
300 = 60 + 15x (c) 20 (d) 12
Page 74 of 516
Ans: (b) Lowest score = x
Highest score = x + 100
https://telegram.me/aedahamlibra
Total marks of (B + C) = 87.5 (n2 + n3)
total marks of A = 70 × n1
∴ 28 × 38 + x + x + 100 = 30 × 40 n1 + n2 + n3 = 100... (1)
1064 + 2x + 100 = 1200 70n1 + 87.5 n2 + 87.5 n3 = 8400... (2)
2x = 1200 – 1164 = 36 Multiplying equation (1) by 87.5 and subtract from
x = 18 equation (2)
We get 17.5 n1 = 350
Q29. The average age of boys in the class is twice the n1 = 20
number of girls in the class. The ratio of boys and
girls in the class of 50 is 4 : 1. The total of the ages (in Q33. The average height of 8 students is 152 cm. Two
years) of the boys in the class is more students of heights 144 cm and 155 cm join the
(a) 2000 (b) 2500 group. What is the new average height ?
(c) 800 (d) 400 (a) 151.5 cm (b) 152.5 cm
Ans: (c) Number of boys = = 40 (c) 151 cm (d) 150.5 cm
Ans: (a) Total height of 8 students = 8 × 152 cm = 1216
Number of girls = = 10 cm
Average age of boys = 2 × 20 = 40 Total height of 10 students = 1216 cm + 144 cm + 155
Total ages of the boys = 40 × 20 = 800 cm = 1515 cm
new average =
Q30. The average of 30 numbers is 40 and that of
other 40 numbers is 30. The average of all the
Q34. What is the Arithmetic mean of the first 'n'
numbers is
natural numbers?
(a) 34.5 (b)
(a)
(c) 35 (d) 34
Ans: (b) Sum of 30 numbers = 30 × 40 = 1200 (c) 2 (n + 1) (d)
Sum of 40 numbers = 40 × 30 = 1200 Ans: (a) Arithmetic mean of first ‗n‘ natural number
Average of 70 numbers = = =

Q31. The average weight of 15 oarsmen in a boat is Q35. A boy found that the average of 20 numbers is
increased by 1.6 kg when one of the crew, who weighs 35 when he writes a number '61' instead of '16'. The
42 kg is replaced by a new man. Find the weight of the correct average of 20 numbers is
new man (in kg). (a) 32.75 (b) 37.25
(a) 65 (b) 66 (c) 34.75 (d) 34.25
(c) 43 (d) 67 Ans: (a) Sum of 20 numbers = 20 × 35 = 700
Ans: (b) Let the average weight of 15 Oarsmen at the Sum of 20 numbers when 61 is replace by 16
start = x kg 700 – 61 + 16 = 655
Let the new man‘s weight = y kg
Correct average of 20 numbers =
According to question
15x – 42 = 15 (x + 1.6) – y
15x – 42 = 15x + 24 – y Q36. The batting average of a cricket player for 30
y = 24 + 42 = 66 kg innings is 40 runs. His highest score exceeds his lowest
score by 100 runs. If these two innings are not
Q32. There are 100 students in 3 sections A, B and C included, the average of the remaining 28 innings is 38
of a class. The average marks of all the 3 sections was runs. The lowest score of the player is
84. The average of B and C was 87.5 and the average (a) 18 (b) 20
marks of A is 70. The number of students in A was (c) 12 (d) 15
(a) 30 (b) 35 Ans: (a) Total runs in 30 innings = 30× 40 = 1200
(c) 20 (d) 25 total runs in 28 innings = 28 ×38 = 1064
Ans: (c) Total marks of all three sections = 84 × 100 = Sum of height and lowest score
8400 H + L = 1200 – 1064

Page 75 of 516
H + L = 136... (1)
Also, H – L = 100... (2)
https://telegram.me/aedahamlibra
So 10 natural numbers are there.

adding (1) and (2) Q40. What is the arithmetic mean of first 20 odd
2H = 236 natural numbers?
H = 118 (a) 17 (b) 19
Lowest run, L = 118 – 100 = 18 (c) 22 (d) 20
Ans: (d) For 20 natural last odd number
Q37. 3 years ago the average age of a family of 5 = 1 + (20 – 1) × 2 = 39
members was 17 years. A baby having been born, the Arithmetic mean of odd number = =
average age of the family is the same today. The
present age of the baby is
Q41. The average of 100 observations was calculated
(a) 1 year (b) 1½ years
as 35.It was found later, that one of the observation
(c) 2 years (d) 3 years
was misread as 83 instead of 53. The correct average
Ans: (c) Let total age of family be S years
is :
3 years ago, total age = S – 3 × 5 = S – 15
(a) 32.7 (b) 34.7
(c) 35.7 (d) 36.7
Ans: (b) Average of difference
S = 17 × 5 + 15 = 100
Let present age of baby be x years =
Correct Average = 35 – 0 3 = 34.7

100 + x = 17 × 6 Q42. The average of 13 results is 70. The average of


x = 102 – 100 = 2 years first seven is 65 and that of the last seven is 75, the
seventh result is :
Q38. The average salary of all the workers in a (a) 70 (b) 70.5
workshop is ₹ 8,000. The average salary of 7 (c) 68 (d) 67
technicians is ₹ 12,000 and the average salary of the Ans: (a) Sum of 13 results = 13 × 70 = 910
rest is ₹ 6,000. The total number of workers in the Sum of 7 results = 7 × 65 = 455
workshop is Sum of last 7 results = 7 × 75 = 525
(a) 20 (b) 21 So, 7th result (455 + 525) – 910 = 70
(c) 22 (d) 23
Ans: (b) Let total number of workers be n Q43. Visitors to a show were charged ₹ 15 each on the
total salary of all workers = 8000 n first day, ₹ 7.50 on the second day, ₹ 2.50 on the third
total salary of 7 technicians = 7 × 12000 = 84,000 day and total attendance on three days were in the
total salary of remaining workers = (n – 7) × 6000 ratio 2:5:13 respectively. The average charge per
84000 + (n – 7) × 6000 = 8000 n person for the entire three days is
84 + 6n – 42 = 8n (a) ₹ 5 (b) ₹ 5.50
42 = 2n (c) ₹ 6 (d) ₹ 7
n = 21 Ans: (*) Ratio of ₹ charged per day × Ratio of no. of
men = Total ₹ Ratio
Q39. The average of some natural numbers is 15. If 30 Ratio of ₹ charged per day
is added to first number and 5 is subtracted from the
= 15 : 7.50 : 2.50 = 6 : 3 : 1
last number the average becomes 17.5 then the
Attendance Ratio = 2 : 5 : 13
number of natural number is:
Total ₹ Ratio = 6 × 2 : 5 × 3 : x 13 × 1
(a) 20 (b) 30
= 12 : 15 : 13
(c) 15 (d) 10
Average charge per person
Ans: (d) Let number of natural numbers be x
∴ 15x + 30 – 5 = 17.5 x
25 = 2.5x
x=

Page 76 of 516
https://telegram.me/aedahamlibra
Q44. At an average of 80 km/hr Shatabdi Express
reaches Ranchi from Kolkata in 7 hrs. Then the
(a) ₹15, 900
(c) ₹14,900
(b) ₹15, 200
(d) ₹15,500
distance between Kolkata and Ranchi is Ans: (c) Sum of monthly income of A + B = 30100... (i)
(a) 560 Km (b) 506 Km Sum of monthly income of B + C = 30700... (ii)
(c) 560 m (d) 650 m Sum of monthly income of A + C = 30400 ... (iii)
Ans: (a) Distance = Speed × Time Subtracting eqn. (i) from (ii)
= 80 × 7 = 560 km B + C – A – B = 30700 – 30100
C – A = 600 ... (iv)
Q45. The average of 9 observations was found to be C + A = 30400 ... (v)
35. Later on, it was de tected that an observation 81 Subtracting eqn. (iv) from (v)
was misread as 18. The correct average of the C – A – C – A = 600 – 30400
observations is – 2A = – 29800
(a) 28 (b) 42 A = 14900
(c) 32 (d) 45
Ans: (b) Average of 9 observations = 35 Q49. The average age of 120 members of a society i s
Total = 9 × 35 = 315 60.7 years. By addition of 30 new members, the
Misread Difference = 81 – 18 = 63 average age becomes 56.3 years. What is the average
New total = 315 + 63 = 378 age (in years) of newly joined members?
Average = = 42 (a) 36.5 (b) 37.2
Shortcut Method: (c) 38.3 (d) 38.7
Ans: (d) Let average age of newly joined member = x
81 – 18 = 63 ∴ =7
According to question,
35 + 7 = 42 120 × 60.7 + 30 × x = 150 × 56.3
7284 + 30x = 8445
Q46. The average marks obtained by a class of 60 ∴30x = 8445 – 7284
students is 65. The average marks of half of the 30x = 1161
students is found to be 85. The average marks of the

remaining students is
(a) 35 (b) 45
(c) 55 (d) 65 Q50. The average of 5 members of a family is 24
Ans: (b) Total students = 60 years. If the youngest member is 8 years old, then
what was the average age (in years) of the family at
the time of the birth of the youngest member?
30x = 65 × 60 – 30 × 85 (a) 16 (b) 20
30x = 30[65 × 2 – 85] (c) 24 (d) 32
x= [ ] Ans: (b) Required average age
years
Q47. The average of 17 results is 60. If the average of
first 9 results is 57 and that of the last 9 results is 65, Q51. The mean of marks secured by 60 students in
then what will be the value of 9th result? division A of class X is 64, 40 students of division B is
(a) 39 (b) 78 60 and that of 60 students of division C is 58. Find the
(c) 117 (d) 156 mean of marks of the students of three divisions of
Ans: (b) Value of 9th result = (9×57+9×65) – (17×60) Class X.
= 1098 – 1020 (a) 60.05 (b) 59.35
= 78 (c) 62.15 (d) 60.75
Ans: (d) Mean of marks of the students
Q48. The average monthly income of A and B is ₹15,
050, the average monthly income of B and C is ₹15,
350 and the average income of A and C is ₹15, 200.
The monthly income of A is

Page 77 of 516
https://telegram.me/aedahamlibra
Q52. The average runs conceded by a bowler in 5
matches is 45 and 15.75 in other 4 matches. What is Q2. What annual payment will discharge a debt of
the average runs conceded by the bowler in 9 ₹6,450 due in 4 years at 5% per annum simple
matches? interest ?
(a) 15 (b) 32 (a) ₹1,400 (b) ₹1,500
(c) 35 (d) 53.5 (c) ₹1,550 (d) ₹1,600
Ans: (b) Required average runs Ans: (b) Let the annual instalment be ₹x.
.
( )

Q53. What is the average of the squares of the first 19 ( ) ( )


natural numbers?
(a) 124 (b) 127.5
(c) 130 (d) 133.5 115x + 110x + 105x + 100x
Ans: (c) = 6450 ×100
430x = 6450 × 100
Average of the square of the first 19 natural number

Q3. At what rate per cent per annum will a sum of


₹1,000 amount to ₹ 1,102.50 in 2 years at compound
interest?
130 (a) 5 (b) 5.5
(c) 6 (d) 6.5
Q54. The average age of 30 students is 20 years and
Ans: (a) ( )
average age of 20 other students is 30 years. The
average age of total number of students is: ( )
(a) 24 years (b) 48 years
(c) 25 years (d) 50 years ( )
Ans: (a) The average age of total number of students
( ) ( )

years.
–1
Simple & Compound r = 5%

Interest Q4. In how many years will a sum of money double


itself at simple interest per annum ?
(a) 24 (b) 20
Q1. A sum of money at compound interest doubles
(c) 16 (d) 12
itself in 15 years. It will become eight times of itself in
(a) 45 years (b) 48 years Ans: (c) T = (For double SI = P = x)
(c) 54 years (d) 60 years year
Ans: (a) ( )
Q5. In how many years will a sum of ₹ 800 at 10% per
( )
annum compound interest, compounded semi -
Cubing on both sides, we have annually becomes ₹ 926.10 ?
( ) (a)

Required time = 45 years (c)


Page 78 of 516
https://telegram.me/aedahamlibra
Ans: (a) Rate = 10% per annum = 5% half yearly Ans: (a)
A=P( ) ∴ Rate =
926.10 = 800 ( ) = = 5 % per annum
( )
Q10. Simple interest on a certain sum for 6 years is
( ) ( )
of the sum. The rate of interest is
∴ Time = 3 half years = years (a) 6% (b)
(c) 8% (d)
Q6. A sum of money placed at compound interest
doubles itself in 4 years. In how many years will it Ans: (a) Rate =
amount to four times itself?
= = 6% per annum
(a) 12 years (b) 13 years
(c) 8 years (d) 16 years
Q11. A sum of ₹ 12,000 deposited at compound
Ans: (c) A = P ( ) interest becomes double after 5 years. After 20 years,
it will become
2=1( )
(a) ₹ 48,000 (b) ₹ 96,000
22 = ( ) (c) ₹ 1,90,000 (d) ₹ 1,92,000
= 8 yrs Ans: (d) A = P ( )

Q7. A sum of ₹ 12,000, deposited at compound ( )


interest becomes double after 5 years. How much will
2=( )
it be after 20 years ?
(a) ₹ 1,44,000 (b) ₹ 1,20,000 24 = ( ) 16 = ( )
(c) ₹ 1,50,000 (d) ₹ 1,92,000 Hence, the principal will become 16 times i.e.
Ans: (d) A = P ( ) ₹ (16 × 12000)
= ₹192000
24000 = 12000 ( )

2=( ) Q12. If ₹5,000 becomes ₹5,700 in a year's time, what


will ₹7,000 become at the end of 5 years at the same
24 = ( ) : = 16 times rate of simple interest?
i.e. The sum amounts to ₹192000. (a) ₹10,500 (b) ₹11,900
(c) ₹12,700 (d) ₹7, 700
Q8. The difference between the compound interest Ans: (b) Interest = 5700 – 5000 = ₹ 700
and simple interest on ₹ 10,000 for 2 years is ₹ 25. The Rate
rate of interest per annum is Case II,
(a) 5% (b) 7% Interest
(c) 10% (d) 12%
Ans: (a) Difference =

25 =
R = 5% Amount = 7000 + 4900 = ₹ 11900

Q9. The simple interest on a sum for 5 years is one Q13. The differenc e between the compound interest
fourth of the sum. The rate of interest per annum is and simple interest for the amount ₹ 5,000 in 2 years
(a) 5% (b) 6% is ₹ 32. The rate of interest is
(c) 4% (d) 8% (a) 5% (b) 8%
(c) 10% (d) 12%
Page 79 of 516
Ans: (b) Difference of 2 years https://telegram.me/aedahamlibra
per annum during the first year and 12% per annum
during the second year (in rupees) will amount to:
( )
(a) 12,000 (b) 12,320
32 = (c) 12,500 (d) 11,320

r2 = = 64 Ans: (b) ( )( )

r=√ = 8% ( )( )

Q14. A person takes a loan of ₹10,000 partly from a ( )( )


bank at 8% p.a. and remaining from another bank at A = 12320
10% p.a. He pays a total interest of ₹950 per annum.
Amount of loan taken from the first bank (in ₹) is Q17. The difference between C. I. (Compound
(a) 2500 (b) 5200 Interest) and S.I. (Simple Interest) on a sum of ₹ 4,000
(c) 2050 (d) 5020 for 2 years at 5% p.a. payable yearly is
Ans: (a) If the amount of loan taken from the first bank (a) ₹ 20 (b) ₹ 10
be ₹x, then (c) ₹ 50 (d) ₹ 60
Ans: (b) Required difference
8x + 100000 – 10x = 95000
2x = 100000 – 95000 = 5000
x = ₹2500
Alternate Method: Q18. A sum becomes ₹ 2,916 in 2 years at 8% per
annum compound interest. The simple interest at 9%
Final rate or interest
per annum for 3 years on the same amount will be
using all igation method (a) ₹ 625 (b) ₹ 600
(c) ₹ 675 (d) ₹ 650
Ans: (c) 2916 = ( )
P= = 2500

S.I = = 675
First amount : Second amount = 1 : 3
Received amount = Q19. The difference between the interests received
from two different banks on ₹ 500 for 2 years is ₹
Q15. Prabhat look a certain amount as a loan from a 2.50. The difference between their rates is:
bank at the rate of 8% p.a. simple interest and gave (a) 0.5% (b) 2.5%
the same amount to Ashish as a loan at the rate of (c) 0.25% (d) 1%
12% p.a. If at the end of 12 years, he made a profit of Ans: (c) Let r1 and r2 are the rates of interests.
So, the difference in S.I
₹ 960 in the deal, then the original amount was:
(a) ₹ 3356 (b) ₹ 1000 =
(c) ₹ 2000 (d) ₹ 3000 2.50 =
Ans: (c) Difference of rates = 4%
So, (r1 – r2) =
= 960

Q20. A person deposited ₹ 500 for 4 years and ₹ 600


for 3 years at the same rate of simple interest in a
2000
bank. Altogether he received ₹ 190 as interest. The
rate of simple interest per annum was
Q16. A principal of ₹ 10,000, after 2 years (a) 3% (b) 4%
compounded annually, the rate of interest being 10% (c) 5% (d) 2%
Ans: (c) S.I (Simple Interest)
Page 80 of 516
= https://telegram.me/aedahamlibra
Q25. The time in which ₹ 80,000 amounts to ₹ 92,610
at 10% p.a. at compound interest, interest being
190 =
compounded semi annually is :
190 = 20R + 18R 38R = 190 R = 5% (a) 3 years (b) years

Q21. A sum o f money becomes 1.331 times in 3 years (c) 2 years (d) years
as compound interest. The rate of interest is Ans: (b) Time = t half year
(a) 50% (b) 8% = R = 5% per half year
(c) 7.5% (d) 10%
∴A=P( )
Ans: (d) ( ) ( )
( )

( )
( ) ( )
T = 3 half years
( ) ( )
R = 10% ∴ T = 3 or year

Q22. The value of a machine depreciates every year


Q26. The simple interest an ₹ 4,000 in 3 years at the
by 10%. if its present value is ₹50,000 then the value
rate of x% per annum equals the simple interest on ₹
of the machine after 2 years is __________.
5,000 at the rate of 12% per annum in 2 years. The
(a) ₹40,500 (b) ₹40,050
value of x is
(c) ₹45,000 (d) ₹40,005
(a) 8% (b) 9%
Ans: (a) Depriciated value
(c) 10% (d) 6%
= 50000 ( ) Ans: (c) S.I.
= 50000 × = ₹ 40500

Q23. A sum of money placed at compound interest


doubles itself in 5 years. It will amount to eight times
of itself in :
(a) 15 years (b) 12 years = 10% per annum
(c) 10 years (d) 20 years
Ans: (a) A = ( ) Q27. There is 100% increase to an amount in 8 years,
at simple interest. Find the compound interest of ₹
2= ( ) 8000 after 2 years at the same rate of interest.
Cubing both sides. (a) ₹ 2500 (b) ₹ 2000
23 = 1( ) (c) ₹ 2250 (d) ₹ 2125
Ans: (d) S.I. = ₹ 100%
Therefore, T = 15 years.
Principal = ₹ 8000
SI = 8000
Q24. The interest on a certain sum of money is ₹ 22
and the true discount on the same sum for the same ∴ Rate =
time and at the same rate is ₹ 20, find the sum. = %
(a) ₹ 220 (b) ₹ 200
(c) ₹ 210 (d) ₹ 212 ∴ C.I. = [( ) ]
Ans: (a) Sum = = 8000 [( ) ]
= = ₹ 220
= ( ) = ₹ 2125

Page 81 of 516
https://telegram.me/aedahamlibra
Q28. If the compound interest on a certain sum for
two years at 12% per annum is ₹ 2,544, the simple
[ ]

interest on it at the same rate for 2 years will be [ ]


(a) ₹ 2,400 (b) ₹ 2,500
(c) ₹ 2,480 (d) ₹ 2.440
Ans: (a) C.I. = P [( ) ]

2544 = [( ) ] = ₹10,0000

2544 = P [( ) ]
Q31. X borrowed some money from a source at 8%
2544 = P ( ) simple interest and lent it to Y at 12% simple interest
on the same day and gained ₹ 4,800 after 3 years. The
2544 = P ( )=
amount X borrowed, in ₹, is
P= = ₹10000 (a) 42,000 (b) 60,000
∴ S.I. = (c) 1,20,000 (d) 40,000
Ans: (d) Let X borrow ₹ P
= = ₹ 2400
He has to return total amount,

Q29. Two equal sums were borrowed at 8% simple


interest per annum for 2 years and 3 years Total amount x get from y, A′ =
respectively. The difference in the interest was ₹ 56.
According to question
The sums borrowed were A′ – A = 4800
(a) ₹ 800 (b) ₹ 700
(c) ₹ 560 (d) ₹ 350 ( ) ( )
Ans: (b) Let principal be represented by P.
Ist Case :
S.I.
IInd Case :
Q32. A sum amounts double in 8 years by simple
interest. Then the rate of simple interest p.a. is
According to question
(a) 10% (b) 12.5%
(c) 15% (d) 20%
Ans: (b) Let P be the principle amount and R be rate of
interest.
2P =
Q30. A man borrowed some money from a private
R= = 12.5%
organisation at 5% simple int erest per annum. He
lended 50% of this money to another person at 10%
compound interest per annum and thereby the man Q33. Ram borrows a certain sum of money a t 8% per
made a profit of ₹ 3205 in 4 years. The man borrowed annum simple interest and Rahim borrows ₹ 2,000 at
(a) ₹ 80,000 (b) ₹ 1,00,000 5% per annum simple interest. If the interest at the
end of 3 years is equal, then the amount borrowed by
(c) ₹ 1,20,000 (d) ₹ 1,50,000
Ram is
Ans: (b) Let the required amount = p
(a) ₹ 1,250 (b) ₹ 1,500
0( ) 1 (c) ₹ 2,000 (d) ₹ 1,000
Ans: (a) Let Ram borrowed ₹ P
0( ) 1

= ₹1,250
Page 82 of 516
https://telegram.me/aedahamlibra
Q34. The population of a village increases by 5%
interest be 10% compounded annually, then the value
of each instalment is
annually. If its present population is 4410, then its (a) ₹ 127 (b) ₹ 121
population 2 years ago was (c) ₹ 210 (d) ₹ 225
(a) 4500 (b) 4000 Ans: (a) Principal (P) = 210
(c) 3800 (d) 3500 Ratio (R) = 10%
Ans: (b) Population 2 years ago = Loan has to be paid in the instalments i.e., is it take two
( )
years to pay.
= 4000
( )
Q35. Rekha invested a sum of ₹ 12000 at 5% per
annum compound interest. She received an amount of ( )
₹ 13230 after n years. Find n. So, equal instalment
(a) 2.8 years (b) 3.0 years
(c) 2.5 years (d) 2.0 years
Q38. The income of a compan y increases 20% per
Ans: (d) P = ₹ 12000, Rate = 5 %, Time (n) = ?, Amount
annum. If its income is ₹ 26,64,000 in the year 2012,
= 13230
then its income in the year 2010 was :
( ) ( ) (a) ₹ 28,20,000 (b) ₹ 28,55,,000
(c) ₹ 18,50,000 (d) ₹ 21,20,000
( ) ( ) Ans: (c) Income in 2012 = ₹ 26, 64,000
Every year % of increase in income = 20%
( ) So, income of company in 2012 = 26,64,000 × ×
= ₹ 18,50,000
( ) ( )
∴ n = 2 years Q39. The compound interest on ₹12000 for 9 months
at 20% per annum, interest being compounded
Q36. A certain a mount of money earns ₹ 540 as quarterly is :
Simple Interest in 3 years. If it earns a Compound (a) ₹ 1750 (b) ₹ 1891.10
Interest of ₹ 376.20 at the same rate of interest in 2 (c) ₹ 2136.40 (d) ₹2089.70
years, find the amount. (in rupees) Ans: (b) P = ₹ 12000;
(a) 2100 (b) 1600 R = 20% per annum = 5% per quarter
(c) 1800 (d) 2000 T = 9 months = 3 quarters
Ans: 37 (d) S.I of 3 years = ₹ 540
So, A = 12000 ( ) = 12000 × 1.05 × 1.05 × 1.05
S.I. of 1 year = = ₹ 180
= ₹ 13891.1
S.I. of 2 years = 2 × 180 = ₹ 360
So, CI = 13891.10 – 12000 = ₹ 1891.10
CI of 2 years = ₹ 376.20
Difference between CI and SI = 376.20 – 360
Q40. In certain years a sum of money is doubled itself
= ₹16.20
If we take P = ₹ 180 R = R% and Time = 1 year at % simple interest per annum, then the required
time will be
(a) years (b) 8 years
R=
(c) years (d) 16 years
So S.I. = 540 = P × 9 × Ans: (d) Let x be the principal amount
P= = ₹ 2000 ‗y‘ be the time to double the money.
Then interest will also be ‗x‘.

Q37. A sum of ₹ 210 was taken as a loan. This is to be ∴x =


paid back in two equal instalments. If the rate of 400 = 25y
y = 16 years
Page 83 of 516
https://telegram.me/aedahamlibra
Q41. The population of a town increases by 5% every
year. If the present population is 9261, the population T = 5 years
3 years ago was
(a) 5700 (b) 6000 Q45. What would be the compound interest of ₹
(c) 7500 (d) 8000 25000 for 2 yrs. at 5% per annum
Ans: (d) Population 3 yrs. ago = (a) 2500 (b) 2562.5
( ) (c) 2425.25 (d) 5512.5
= Ans: (b) CI = * +

Q42. The simple interest on a certain sum of money at = 25000* +


the rate of 5% per annum for 8 years is Rs. 840. Rate = 25000[( ) ]
of interest for which the same amount of interest can
be received on the same sum after 5 years is = 25000* +
(a) 7% (b) 8%
(c) 9% (d) 10%
Ans: (b) When P = P, R = 5%, T = 8yr then SI = 840
Q46. A sum of ₹ x was put at simple interest at a
certain rate for 2 years. Had it been put at 3% higher
rate, it would have fetched ₹ 300 more. The value of
4x is
P = 2100
(a) ₹ 16, 000 (b) ₹ 20,000
Case II: When P = 2100, R = ?, T = 5 SI = 840
(c) ₹ 36, 000 (d) ₹ 24,000
P% Ans: (b) Let the sum be ₹x & original rate R%, then,
R = 8%
. / ( )
Q43. A certain sum will amount to ₹12,100 in 2 years 6x = 300 × 100 x = 5000
at 10% per annum of compound int erest, interest The value of 4x = 4 ×5000 = 20000
being compounded annually. The sum is:
(a) ₹12000 (b) ₹6000 Q47. A sum of money placed at compound interes t
(c) ₹8000 (d) ₹10000 double itself at 2 years. The year it will take to
Ans: 42 (d) Final rate of interest for amount 4 times itself is
two pens (a) 6 (b) 4
=x+y+ (c) 8 (d) 3
= Ans: (b) Let the sum be ₹1 which becomes ₹2 after 2
years
Let principal be P.
( ) ... (i)
Let the sum of ₹1 becomes ₹4 after 'n' years
P = 100 × 100 = ₹ 10000
( )
Q44. Alipta got some amount of money from her ( )
father. In how many years will the ratio of the money
and the interest obtained from it be 10:3 at 6% simple 0 ( ) 1 ( )
interest per annum?
(a) 7 years (b) 3 years n = 4 years
(c) 5 years (d) 4 years
Ans: (c) Let principal = 10x Q48. A certain sum of money triples itself in 5 years at
Interest = 3x simple interest. In how many years it will be five
times?
(a) 5 (b) 8
Page 84 of 516
(c) 10
Ans: (c)
https://telegram.me/aedahamlibra
(d) 15 interest for 1 year on a certain sum of money lent out
at 8% per annum is ₹ 64. What is the sum (in ₹)?
∴S.I = 3x – x = 2x (a) 40000 (b) 42000
∴ r = 40% (c) 44000 (d) 44800
Ans: (a) According to question,
Now, P = x A = 5x
∴S.I. = 5x – x = 4x 0 ( ) 1 [ ]
T=?

∴ = 10 years.

Q49. A person lent certain sum of money at 5% per ∴


annum simple interest and in 15 years the interest ∴P = 625 × 64 = 40,000.
amounted to ₹ 250 l ess than the sum lent. What was
the sum lent (in ₹)? Q52. What is the compound interest (in ₹) on ₹ 12500
(a) 1000 (b) 1500 at the rate of 12% per annum compounded yearly for
(c) 2400 (d) 3000 2 years?
Ans: (a) Let principal = x (a) 3000 (b) 2980
Then, (c) 3050 (d) 3180
∴Simple Interest = x – 250 Ans: (d) Here,
According to question, P = 12500
R = 12%
T = 2 years
100 x – 25000 = 75x C.I. = ?
25x = 25000
∴ C.I. = ( )

Q50. A sum of ₹ 400 becomes ₹ 448 at simple interest ( )


in 2 years. In how many years will the sum of ₹ 550
amounts to ₹ 682 at the same rate?
(a) 2 (b) 3 = 15680 – 12500
(c) 3.5 (d) 4 = 3180
Ans: (d) Here, P = 400 A = 448 T = 2 years ∴Compound interest = ₹ 3180
R=?
A = P + S. I Q53. The simple interest on sum for 5 years is 3/5 th
∴S. I = A – P of the sum. The rate of interest per annum is:
= 448 – 400 = 48 (a) % (b) 10%
(c) 12% (d) 8%

∴ Ans: (c) Here Let P = x S.I

NowP = 550 A = 682 R = 6% T=5R=?


T=? According to question,
S-I = 682 – 550 = 132


∴ years ∴The rate of interest per annum is = 12%

Q51. The difference between the compound interest


compounding half yearly for 1 year and the s imple

Page 85 of 516
https://telegram.me/aedahamlibra
Ratio & Proportion (a) ₹400
(c) ₹600
(b) ₹500
(d) ₹800
Ans: (c) A‘s share

Q1. A drum of kerosene is full. When 30 litres of ( )

kerosene is drawn from it, it remains full. The


capacity of the drum is Q5. If 1.5a = 0.04 b then is equal to
(a) 120 l (b) 135 l (a)
(c) 150 l (d) 180 l
Ans: (d) Let the capacity of the drum be x litres. (c)
Ans: (a) 1.5a = 0.04 b

By componendo and dividendo,

= x = 6 × 30 = 180 litres
Q6. Two number are in the ratio 7 : 11. If 7 is added
Q2. An amount was lent for two years at the rate of to each of the numbers, the ratio becomes 2 : 3. The
20% per annum compounding annually. Had the smaller number is
compounding been done half yearly, the interest (a) 39 (b) 49
would have increased by 241. What was the amount (c) 66 (d) 77
(in ₹) lent? Ans: (b) Let the numbers be 7x and 11x respectively.
(a) 10000 (b) 12000 ∴
(c) 20000 (d) 24000 ∴ 22x + 14 = 21x + 21
Ans: (a) Let sum ₹ = x x=7
C.I. when compounded half yearly ∴ Smaller number = 7x = 7 × 7 = 49
0 ( ) 1
Q7. Two numbers are in the ratio 1 : 3. If their sum is
C.I. when compound annually 240, then their difference is
(a) 120 (b) 108
0 ( ) 1 (c) 100 (d) 96
Now, Ans: (a) Let the numbers be 3x and x.
3x + x = 240
4x = 240
∴x = 10000 x= = 60
∴sum = ₹ 10000 ∴ Difference = 3x – x = 2x = 2 × 60 = 120

Q3. What must be added to each term of the ratio 7 : Q8. The ratio of milk and water in mixtures of four
11, so as to make it equal to 3 : 4 ? containers are 5 : 3, 2 : 1, 3 : 2 and 7 : 4 respectively,
(a) 8 (b) 7.5 in which container is the quantity of milk, relative to
(c) 6.5 (d) 5 water, minimum ?
Ans: (d) Let the required number be x. (a) First (b) Second
(c) Third (d) Fourth
Ans: (c) Milk in V1 = = 0.625
28 + 4x = 33 + 3x
x = 33 – 28 = 5 Milk in V2 = = 0.66
Milk in V3 = = 0.6
Q4. If ₹1000 is divided between A and B in the ratio 3
: 2, then A will receive Milk in V4 = = 0.636

Page 86 of 516
https://telegram.me/aedahamlibra
Q9. If W 1 : W 2 = 2 : 3 and W 1 : W 3 = 1 : 2 then W 2 :
=
If the total profit is ₹ 4,00,000, the share of C is
W3 is
(a) 3 : 4 (b) 4 : 3 (a) ₹ 1,12,500 (b) ₹ 1,37,500
(c) 2 : 3 (d) 4 : 5 (c) ₹ 90,000 (d) ₹ 2,70,000
Ans: (c) A : B = 1 : 3
Ans: (a)
B:C=1:3=3:9
and C : D = 1 : 3 = 9 : 27
∴ A : B : C : D = 1 : 3 : 9 : 27

Sum of ratios = 1 + 3 + 9 + 27 = 40
∴ C‘s share of profit
Q10. The ratio of income and expenditure of a person = × 400000 = ₹ 90000
is 11 : 10. If he saves ₹ 9,000 per annum, his month ly
income is
Q13. The ratio of weekly incomes of A and B is 9 : 7
(a) ₹ 8,000 (b) ₹ 8,800
and the ratio of their expenditures is 4 : 3. If each
(c) ₹ 8,500 (d) ₹ 8,250
saves ₹ 200 per week, then the sum of their weekly
Ans: (d) Let the income of man be Rs. = 11x and his
incomes is
expenditure be ₹ 10x.
(a) ₹ 3,600 (b) ₹ 3,200
∴ Savings x = ₹ 9000
(c) ₹ 4,800 (d) ₹ 5,600
∴ Monthly income of man = = ₹ 8250 Ans: (b) Let monthly income of A and B be 9x and 7x
let income = 11x Expenditure = Income – Saving
expenditure = 10x ATQ
Saving = 11x – 10x = x
x = 9000
Monthly income = 27x – 6.00 = 28x – 800
x = 200
= 8,250
Sum = 200 × 16 = 3200

Q11. A can contains a mixture of two liquids A and B


Q14. What number should be added to or subtracted
in the ratio 7 : 5. When 9 litres of mixture are drawn
from each term of the ratio 17 : 24 so that it becomes
off and the can is filled with B, the ratio of A and B
equal to 1 : 2?
becomes 7 : 9. Litres of liquid A contained by the can
(a) 5 is subtracted (b) 10 is added
initially was
(c) 7 is added (d) 10 is subtracted
(a) 10 (b) 20
Ans: (d) Let the number x be added
(c) 21 (d) 25
Ans: (c) A = 7x litre, B = 5x litre (let) ∴
In 9 litres of mixure, 34 + 2x = 24 + x
= = litre 2x – x = 24 – 34
x = –10
B= litre Hence, 10 should be subtracted.
In new situation,
Q15. If x : y = 4 : 5, then (3x + y) : (5x + 3y) =
=
(a) 3 : 5 (b) 5 : 3
(c) 17 : 35 (d) 35 : 17
252x – 189 = 140x + 147 Ans: (c)
112x = 336 x = 3 ( )
∴ Initial quantity of liquid A ∴
( )
= 7x = 7 × 3 = 21 litre
= =
Q12. In a business partnership among A, B, C and D,
the profit is shared as follows:
Page 87 of 516
https://telegram.me/aedahamlibra
Q16. Among three numbers, the first is twice the
second and thrice the third. If the average of the three
(c)
Ans: (a) Amounts of water in two jars are equal; the jar
numbers is 49.5, then the difference between the first
with the greater capacity is full, and the Jar with lesser
and the third number is
(a) 54 (b) 28 capacity is full.
(c) 39.5 (d) 41.5 ∴ When the water in smaller jar is poured into the larger
Ans: (a) Let the second number be x. Jar, the addition of an equal amount of water will double
∴ First number = 2x
the amount in the larger jar, which will then be
∴ Third number =
full.
∴ 2x + x + = 49.5 × 3
6x + 3x + 2x = 49.5 × 9 = 445.5 Q20. Monthly incomes of A and B are in the ratio of 4
11x = 445.5 x= = 40.5 : 3 and their expenses bear the ratio 3 : 2. Each of
∴ Requried difference them saves ₹ 6,000 at the end of the month, then the
= 2x – monthly income of A is
(a) ₹ 12,000 (b) ₹ 24,000
= = 54 (c) ₹ 30,000 (d) ₹ 60,000
Ans: (b) Let salary of A and B be = 4x and 3x
Q17. The ratio between two numbers is 2 : 3. If each expenditure = income-salary
number is increased by 4, the ratio between them ATQ
becomes 5 : 7. The difference between the numbers is
(a) 8 (b) 6
(c) 4 (d) 2 8x – 12000 = 9x – 18000
Ans: (a) Let the numbers be 2x and 3x. x = 6000
∴ A‘s salary = 4x = 4 × 6000 = 24000

∴ 15x + 20 = 14x + 28
Q21. A man leaves ₹ 12,600 to be divided among 7
x = 28 – 20 = 8 = Required Difference
sons,3 daughters and 5 nephews. If each daughter
receives three times as much as each nephew and each
Q18. The ratio of the quantities of an acid and water
son seven times as much as each nephew, then each
in a mixture is 1 : 3. If 5 litres of acid is further added
daughter‘s share is
to the mixture, the new ratio becomes 1 : 2. The
(a) ₹ 700 (b) ₹ 650
quantity of new mixture in litres is
(c) ₹ 600 (d) ₹ 750
(a) 32 (b) 40
(c) 42 (d) 45 Ans: (c) 7S + 3D + 5N = ₹ 12600... (1)
Ans: (d) Let the quantity of acid in original mixture be x According to Question
litre and that of water be 3x litre. D = 3N
S = 7N

Putting these values in equation (1)
2x + 10 = 3x x = 10 49 N + 9N + 5N = 12600
∴ Quantity of new mixture 63N = 12600
= 4x + 5 = 45 litres N = 200
Each daughter‘s share
Q19. Equal amou nts of water were poured into two = 3 × Nephew‘s share = 3 × 200 = ₹ 600
empty jars of different capacities, which made one jar
full and the other jar full. If the water in the jar Q22. A certain sum of money is distributed to A and B
with lesser capacity is then poured into the jar with in the ratio 2 : 5. If A received ₹100, then the money
greater capacity, then the part of the Large r jar filled received by B is
with water is (a) ₹ 200 (b) ₹ 150
(a) (c) ₹ 250 (d) ₹ 300
Ans: (c) Let ‗x‘ be the total sum of money.

Page 88 of 516
Money received by A = https://telegram.me/aedahamlibra 2016
100 = x = ₹ 350 ∴ Loss = 5184 – 2016
= ₹ 3168
∴ Money received by B = = ₹250

Q26. The ratio in which a man must mix rice at ₹


Q23. A, B and C are batsmen. The ratio of the runs 10.20 per kg and ₹ 14.40 per kg so as to make a
scored by them in a certain match are given below:
mixture worth ₹ 12.60 per kg, is
A : B = 5 : 3 and B : C = 4 : 5. In all they scored 564
(a) 3 : 4 (b) 4 : 3
runs. The number of runs scored by B is:
(c) 2 : 5 (d) 18 : 24
(a) 124 (b) 104
Ans: (a) By the rule of alligation:
(c) 114 (d) 144
Cost of 1 kg rice of 1st kindCost of 1 kg rice of 2nd kind
Ans: (d) A : B = 5 : 3
B:C=4:5

No. of runs scored by B = 144

∴ Required ratio = 1.80 : 2.40 = 3 : 4.


Q24. In a school, the ratio of boys to girls is 4 : 3 and
the ratio of girls to teachers is 8 : 1. The ratio of
Q27. A jar contains a mixture of two liquids A and B
student to teachers is :
in the ratio 4 : 1. When 10 litre of the mixture is
(a) 56 : 3 (b) 55 : 1
replaced with liquid B, the ratio becomes 2 : 3. The
(c) 49 : 3 (d) 56 : 1
volume of liquid A present in the jar earlier was:
Ans: (a) Boys (4) :girls (3) :girls (8) :teacher (1)
(a) 20 lt (b) 10 lt
So,
(c) 16 lt (d) 15 lt
Ans: (c) Liquid A = 4x litre
Liquid B = x litre
In 10 litres of mixture,
Liquid A litre
Liquid B = 2 litre
So, Student : teacher
(boys + girls) :teacher
(32 + 24) :3 12x – 24 = 2x + 16
56 :3 10x = 40 x = 4
∴ Initial quantity of liquid A = 16 litre
Q25. The cost of a piece of diamond varies with the
square of its weight. A diamond of ₹5,184 value is cut
Q28. In a partnership business, A invests thof the
into 3 pieces whose weights are in the ratio 1 : 2 : 3.
Find the loss involved in the cutting. capital for of the total time, B invests of the capital
(a) ₹3, 068 (b) ₹3, 088 for of the total time and C, the rest of the capital for
(c) ₹3, 175 (d) ₹3, 168
the whole time. Out of a prof it of ₹ 19,400, B‘s share
Ans: (d) If the weight of a piece of diamond be 6 x units,
is:
then
(a) ₹ 2000 (b) ₹ 1200
Original price α (6x)2 = 36kx2
(c) ₹ 1600 (d) ₹ 1800
∴ 36. kx2 = 5184..... (i)
Ans: (d) Ratio of profit sharing
Again,
New price = k (x2 + 4x2+ 9x2) = 14 kx2
Page 89 of 516
https://telegram.me/aedahamlibra
= 4 : 9 : 84 A + A – 4000 + A – 9000 = 50000
So, A = 21000
Sum of ratios = 97
B = 17000
share = ₹ 1800 C = 12000
∴ A : B : C = 21000 : 17000 : 12000 = 21 : 17 : 12
Q29. An employer reduces the number of employees A‘s Profit = = ₹14700
in the ratio 8: 5 and increases their wages in the ratio
7:9. As a result, the overall wages bill is
Q33. A man ordered 4 pairs of black socks and some
(a) Increased in the ratio 56 : 69
pairs of brown socks. The price of a black socks is
(b) Decreased in the ratio 56 : 45
double that of a brow n pair. While preparing the bill
(c) Increased i n the ratio 13 : 17
the clerk interchanged the number of black and
(d) Decreased in the ratio 17 : 13
brown pairs by mistake which increased the bill by
Ans: (b) Required ratio = 8 × 7 : 5 × 9 = 56 : 45
50%. The ratio of the number of black and brown
pairs of socks in the original order was :
Q30. A and B are partners in a business. A
(a) 2 : 1 (b) 1 : 4
contributes of the capital for 15 months and B (c) 1 : 2 (d) 4 : 1
received of the profit. Find for how long B ‘s money Ans: (b) Number of brown socks = x
was used? Price of brown socks = Rs. y per pair
(a) 6 months (b) 8 months Price of black socks = Rs. 2y per pair
(c) 10 months (d) 12 months ∴ 4y + x × 2y =
Ans: (c) A‘s profit : B‘s profit 4 + 2x =
8 + 4x = 24 + 3x
x = 24 – 8 = 16
∴ Requried ratio = 4 : 16 = 1 : 4

Q34. The present ages of two persons are 36 and 50


years respectively, if after n years the ratio of their
n = 10 months ages will be 3 : 4, then the value of n is
(a) 3 (b) 4
Q31. 465 coins consists of 1 rupee, 50 paise and 25 (c) 7 (d) 6
paise coins. Their values are in the ratio 5: 3 : 1. The Ans: (d) According to question
number of each type of coins respectively is
(a) 155, 186, 124 (b) 154, 187, 124
36 × 4 + 4n = 50 × 3 + 3n
(c) 154, 185, 126 (d) 150, 140, 175
4n – 3n = 150 – 144
Ans: (a) The ratio of number of coins = 5 : 6 : 4
n=6
∴ The number of one rupee coins
The number of 50 paise coins Q35. The ratio of age of two boys is 5 : 6. After two
years the ratio will be 7 : 8. The ratio of their ages
The number of 25 paise coins
after 12 years will be
(a) 11/12 (b) 22/24
Q32. A, B, C subscribe together ₹ 50,000 for a (c) 15/16 (d) 17/18
business. A subscribes ₹ 4,000 more than B and B ₹
Ans: (d) ... (1)
5,000 more than C. Out of a total profit of ₹ 35,000, A
receives = 8A + 16 = 7B + 14 7B – 8A = 2... (2)
(a) ₹ 8,500 (b) ₹ 11,998 From (1) and (2) , A = 5, B = 6
(c) ₹ 12,600 (d) ₹ 14,700
Ans: (d) A = B + 4000
B = C + 5000
A + B + C = 50000
Page 90 of 516
https://telegram.me/aedahamlibra
Q36. The proport ion of acid and water in three
samples is 2 : 1, 3 : 2 and 5 : 3. A mixture containing
By equation (i) × 3 – (ii) × 4
18x – 12y – 20x + 12y
equal quantities of all three samples is made. The = 1200 – 1600
ratio of water and acid in the mixture is: 2x = 400 x = 200
(a) 120 : 133 (b) 227 : 133 ∴ Total income
(c) 227 : 120 (d) 133 : 227 = 6x + 5x = 11x = ₹ 2200
Ans: (b) Required ratio = ( ) ( )
Q40. ₹ 700 is divided among A, B, C in such a way
=( ) ( ) = 227 : 133
that the ratio of the amount of A and B is 2 : 3 and
that of B and C is 4 : 5. Find the amounts in ₹ each
Q37. Three numbers are in the ratio 1 : 2 : 3. By received, in the order A, B, C.
adding 5 to each of them, the new numbers are in the (a) 150, 250, 300 (b) 160, 240, 300
ratio 2 : 3 : 4. The numbers are : (c) 150, 250, 290 (d) 150, 240, 310
(a) 5, 10, 15 (b) 10, 20, 30 Ans: (b) A : B = 2 : 3 = 8 : 12
(c) 15, 30, 45 (d) 1, 2, 3 B : C = 4 : 5 = 12 : 15
Ans: (a) Number = x, 2x and 3x ∴ A : B : C = 8 : 12 : 15
∴ Sum of ratio = 35
4x +10 ∴ A‘s share =
3x +15 = ₹ 160
x=5
B‘s share = × 700 = ₹ 240
Number = 5,
10 and 15, C‘s share = × 700 = ₹ 300

Q38. A invests ₹ 64,000 in a business. After few Q41. 15 litres of a mixture contains alcohol and water
months B joined him with ₹ 48,000. At the end of in the ratio 1 : 4. If 3 litres of water is mixed in it, the
year, the tot al profit was divided between them in the percentage of alcohol in the new mixture will be
ratio 2 : 1. After how many months did B join ? (a) 15 (b)
(a) 7 (b) 8
(c) 17 (d)
(c) 4 (d) 6
Ans: (c) Suppose, B Joined after x month Ans: (b) Alcohol = 3 litres
Then B's money was invested for (12 – x) months Water = 12 litres
∴ According to question ∴ Required percentage
=
=
16 = 24 – 2x
2x = 24 – 16 = x = 4 Q42. A and B have together three times what B and C
Hence, B joined after 4 months have, while A, B, C together have thirty rupees more
than that of A. If B has 5 times that of C, then A has
Q39. The ratio of monthly incomes of A, B is 6 : 5 and (a) ₹ 60 (b) ₹ 65
their monthly expenditures are in the ratio 4 : 3. If (c) ₹ 75 (d) ₹ 45
each of them saves ₹ 400 per month, find the sum of Ans: (b) A + B = 3 (B + C)
their monthly incomes. A + B + C = A + 30
(a) 2300 (b) 2400 B = 5C
(c) 2200 (d) 2500 ∴ A + B = 3 (B + C)
Ans: (c) Incomes of A and B A + 5C = 18C A = 13C
= ₹ 6x and 5x ∴ A + B + C = A + 30
Expenses of A and B A + 5C + C = A + 30
= ₹ 4y and 3y
A+ = A + 30
∴ 6x – 4y = 400... (i)
5x – 3y = 400... (ii) 6A = 30 × 13
Page 91 of 516
A = ₹ 65
Alternate Method:
https://telegram.me/aedahamlibra
Let C = CB = 5C
A + B = 3 (B + C)
A + 5C = 3 × 6 C∴ A = 13 C A‘s share
A + B + C = A + 30
SC + C = 30∴ C = 15
A = 13 ×5 = 65 Shortcut Method:
4A = 6B = 3C
A : B : C = 18 : 12 : 24 = 3 : 2 : 4
Q43. A sum of ₹ 300 is divided among P, Q and R in
such a way that Q gets ₹ 30 more than P and R gets ₹ Share of A
60 more than Q. The ratio of their share is
(a) 3 : 2 : 5 (b) 2 : 5 : 3 Q47. If 2x = 3y = 4z, find x : y : z.
(c) 5 : 3 : 2 (d) 2 : 3 : 5 (a) 3 : 4 : 6 (b) 6 : 4 : 3
Ans: (d) Q = P + 30 Q – P = 30 and (c) 4 : 3 : 2 (d) 2 : 3 : 4
R – Q = 60 = 2 × 30 Ans: (b) 2x = 3y = 4z
∴ Required ratio = 2 : 3 : 5 x = 2z, y =
Look : 3 – 2 = 1, 5 – 3 = 2
x : y : z = 2z : :z=6:4:3
Q44. The prices of a school bag and a shoe are in the
ratio 7 : 5. The price of a school bag is ₹ 200 more Q48. The ratio of 252.5 : 53 is same as
than the price of a shoe. Then the price of a shoe is (a) 5 : 3 (b) 5 : 6
(a) ₹ 500 (b) ₹ 1,200 (c) 1 : 25 (d) 25 : l
Ans: (d) (52)2.5 : 53 252.5 : 53
(c) ₹ 200 (d) ₹ 700
= 55 : 53 = 52 : 1 = 25 : 1
Ans: (a) 7x – 5x = 200
2x = 200 x = 100
Q49. A box contain 280 coins of one rupee, 50 paise
∴ Price of a pair of shoes = 5x = 500
and 25 paise. The values of each kind of coin are in
the ratio of 8 : 4 : 3. The number of one rupee coins
Q45. Divide 81 into three parts so that of 1 st, of 2 nd will be
and of 3rd are equal. (a) 52 (b) 81
(a) 36, 27, 18 (b) 27, 18, 36 (c) 60 (d) 80
(c) 18, 27, 36 (d) 30, 27, 24 Ans: (d) Ratio of number of coins
Ans: (c) Let lst, 2nd and 3rd part represented by x, y, z = 8 : 4 × 2 : 3 × 4 = 8 : 8 : 12 = 2 : 2 : 3

Let Number of one rupee coin

∴x = 2k, y = 3k, = 4k
According to question Q50. The ratio of the ages of A, B and C is 5 : 8 : 9. If
x + y + z = 81 the sum of the ages of A and C is 56 years, the age of
2k + 3k + 4k = 81 9k = 81 k=9 B will be
(a) 12 years (b) 23 years
Hence, parts are 18, 27, 36.
(c) 21 years (d) 32 years
Ans: (d) Let ages of A, B and C are 5x, 8x and 9x
Q46. The prize money of ₹ 1,800 is divided among 3
respectively.
students A, B and C in such a way that 4 times the
5x + 9x = 56
share of A is equal to 6 times the share of B, which is
x=4
equal to 3 times the share of C. Then A‘s share is
Age of B = 8 × 4 = 32 years
(a) ₹ 400 (b) ₹ 600
(c) ₹ 700 (d) ₹ 800
Q51. ₹ 730 were divided among A, B, C in such a way
Ans: (b) 4A = 6B 2A = 3B A : B = 3 : 2
that if A gets ₹ 3, then B gets ₹ 4 and if B gets ₹ 3.50
B = 3C 2 B = C B : C = 1 : 2
then C gets ₹ 3. The share of B exceeds that of C by

Page 92 of 516
(a) ₹ 30
(c) ₹ 70
https://telegram.me/aedahamlibra
(b) ₹ 40
(d) ₹ 210
Ans: (b) ;
and A + B + C = 730
= 730
( ) = 730 ∴ Required ratio = = 3 : 2

B= = 280
Q56. If x : y = 5 : 2, then (8x + 9y) : (8x + 2y) is
C= = 240 (a) 22 : 29 (b) 26 : 61
B exceeds that of C by (280 – 240) = ₹ 40 (c) 29 : 22 (d) 61 : 26
Ans: (c)
Q52. If x : y ::2 : 3 and 2 : x :: 4 : 8 the value of y is
(a) 6 (b) 8
(c) 4 (d) 12
Ans: (a) ;
x=4
Q57. The current ages of Sonali and Monali are in the
= ratio 5 : 3. Five years from now, their ages will be in
the ratio 10 : 7. Then, Monali's current age is:
Q53. Tw o numbers are in the ratio 3 : 5. If 9 is (a) 9 years (b) 15 years
subtracted from each, the new numbers are in the (c) 3 years (d) 5 years
ratio 12 : 23. The small number is Ans: (a) Ratio of present ages of Sonali and Mo nali = 5 :
(a) 27 (b) 33 3
(c) 49 (d) 55 After 5 years ratio of ages of both girls = 10:7
Ans: (b) Let two numbers be 3x and 5x Let actual present ages are 5x and 3x years.
= =

23 (3x – 9) = 12 (5x – 9) = 35x + 35 = 30x + 50


69x – 207 = 60x – 108 5x = 15
9x = 99 x=3
x = 11 So, Monali age = 3 × 3 = 9 years
Hence, the small number will be 3 ×11 = 33
Q58. Incomes of x and y are in the ratio 4 : 3. Their
expenditures are in the ratio 12 : 7. Both save ₹ 3200
Q54. If , then the value of ( ) is
at the end of the month, then the income of x is :
(a) (a) ₹ 6000 (b) ₹ 8000
(c) 1 (d) 2 (c) ₹ 2000 (d) ₹ 4000
Ans: (b) Ratio of income of x and y = 4 : 3
Ans: (c) Ratio of expenditure of x and y = 12 : 7
Saving of x = 4a – 12b = 3200... (i)
Saving of y = 3a – 7b = 3200.. (ii)
Q55. Gold is 19 times as heavy as water and copper is
Solving (i) and (ii)
9 times as heavy as water. In what ratio should these
on
be mixed to get an alloy 15 times as heavy as water?
(a) 1 : 1 (b) 1 : 2 subtracting b = 400
(c) 2 : 3 (d) 3 : 2 Now 3a = 3200 + 7b = 3200 + 7 (400) = 6000
Ans: (d) By rule of alligation, we have a = ₹ 2000
So income of x = ₹ 2000 × 4 = 8000

Page 93 of 516
https://telegram.me/aedahamlibra
Q59. Eighteen years ago, the ratio of A's age to B's
age was 8 : 13. Their present ratio's are 5 : 7. What is
Q62. 729 ml of a mixture contains milk a nd water in
the ratio 7 : 2. How much more water is to be added
the present age of A? to get a new mixture containing milk and water in the
(a) 70 years (b) 50 years ratio 7 : 3 ?
(c) 40 years (d) 60 years (a) 60 ml (b) 71 ml
Ans: (b) Let the A‘s age and B‘s age was 8x and 13x (c) 52 ml (d) 81 ml
According to question Ans: (d) Quantity of milk = = 567 ml
Quantity of water = = 162 ml
56x + 18 × 7 = 65x + 18 × 5 Let ‗x‘ be the quantity that should be added to make the
65x – 56x = 18 × 7 – 18 × 5 ratio 7 : 3
9x = 18 × 2 According the question
x=4
Hence, the present age of A
= 8 × 4 + 18 = 50 yrs. 1701 = 1134 + 7x
7x = 1701 – 1134
Q60. A vessel contains 60 litres of milk. 12 litres of x = 81 ml
milk taken out from it and replaced by water. Then
again from mixture, 12 litres are again taken out and Q63. The ratio of number of boys and girls in a school
replaced by water. The ratio of milk and water in the of 720 students is 7 : 5 . How many more girls should
resultant mixture is : be admitted to make the ratio 1 : 1 ?
(a) 16 : 10 (b) 9 : 5 (a) 90 (b) 120
(c) 15 : 10 (d) 16 : 9 (c) 220 (d) 240
Ans: (d) 12 of milk taken out of 60 milk So 20% water is Ans: (b) Ratio of boy and girl = 7 : 5
added to milk Number of student – 720
Milk = 48;Water = 12 Number of Girl =
Now, again 20% water is added to this mixture Number of Boy = 720 – 300 = 420
48 (milk)→ 38.4 (milk)and 21.6 (water) Number of girl needed = 420 – 300 = 120
So, ratio of milk and water = 38.4 : 21.6
= 16 : 9 Q64. If A and B are in the ratio 4 : 5 and the
Alternate Method: difference of their squares is 81, what is the valu e of
Final Volume of milk = Initial volume A?
× (a) 36 (b) 15
(c) 45 (d) 12
Ans: (d)

Water = 60 –
Squaring both sides, we get
Required ratio =

Q61. What must be added to each term of the ratio 2 : both sides substract 1
5 so that it may equal to 5 : 6 ?
(a) 12 (b) 78
(c) 65 (d) 13
Ans: (d) According to question A2 = 16 × 9

Shortcut Method:
12 + 6x = 25 + 5x Let A be 4x and B be 5x
x = 25 – 12 = 13. (5x)2 – (4x)2 = 81 (given)
9x2 = 81 ∴ x = 3
A = 3 × 4 = 12

Page 94 of 516
https://telegram.me/aedahamlibra
Q65. The numbers x, y, z are respectively
proportional to 2, 3, 5 and the sum of x, y and z is 80.
112x = 336
x=3
If the number z is given by the equation z = ax – 8, So, the can contained 21 litres of A.
then a is
(a) 6 (b) 3/2 Q68. The sum of the cubes of two numbers in the ratio
(c) 3 (d) 5/2 3 : 4 is 5824. The sum of the numbers is:
Ans: (c) Let x, y and z be 2m, 3m and 5m (a) (5824)1/3 (b) 28
According to Question (c) 24 (d) 14
2m + 3m +5m = 80 Ans: (b) Let the number be 3x and 4x
10m = 80, m = 8 (3x)3 + (4x)3 = 5824
z=ax–8 27x3 + 64x3 = 5824
5 (8) = a (2) (8) – 8 91x3 = 5824
40 + 8 = 16 a x3 = 64
48 = 16 a x=4
a=3 Sum of numbers are = x (4 + 3) = 4× 7 = 28

Q66. A and B invest ₹ 3000 and ₹ 2400 respectively in Q69. If , then what is the ratio of P and Q
a business. If after one year there i s a loss of ₹ 720, respectively?
how much loss will B bear? (Loss or Profit is in (a) 12 : 77 (b) 12 : 33
proportion to their investments) (c) 28 : 33 (d) 3 : 28
(a) ₹ 72 (b) ₹ 320 Ans: (c) According to question,
(c) ₹ 400 (d) ₹ 360
Ans: (b) A invested = ₹ 3000
B invested = ₹ 2400 ∴

Q70. The ratio of the radii of two cylinders is 2:1 and


According to Question, (A + B) = ₹ 720
their heights are in the ratio 3:2. Then their volumes
A = 720 – B
are in the ratio
(a) 3 : 1 (b) 4 : 3
(c) 6 : 5 (d) 6 : 1
Ans: (d) Ratio of volumes
9 B = 2880
B = 320

Q67. A can is full of a mixture of a two liquids A and


B in the ratio of 7:5. When 9 litres of mixture are or 6:1
drawn off from the can and replaced by the same
quantity of liquid B, the ratio of A and B in the can
Q71. A, B and C invested amounts in the ratio 3 : 4 : 5
becomes 7:9. The capacity of the can is
respectively. It the schemes offered compound interest
(a) 10 litres (b) 21 litres
at the rate of 20% per annum, 15% per annum and
(c) 20 litres (d) 36 litres
10% per annum respectively, then what will be the
Ans: (b) Suppose the can initially contains 7 x and 5x of
ratio of their amounts after 1 year?
mixtures A and B respectively.
(a) 3 : 15 : 25 (b) 6 : 6 : 5
ATQ
(c) 36 : 46 : 55 (d) 12 : 23 : 11
Ans: (c) Let A, B, and C in vested amounts in the ratio
300 : 400 : 500 respectively.
( )
then,
∴Required ratio
252 x – 189 = 140x + 147 = 36 : 46 : 55.

Page 95 of 516
Q72.
https://telegram.me/aedahamlibra
Ans: (d) Let present age of father = x
present age of son = y
According to question,
3200 is divided among A, B and C in the ratio of 3 : x + y = 90... (i)
5 : 8 respectively. What is the difference (in ₹) Now,
between the share of B and C?
(a) 400 (b) 600
(c) 800 (d) 900 2x – 20 = 5y – 50
2x – 5y = –30... (ii)
Ans: (b) Share of A
from eq. (i) and (ii)
Share of B x = 60
Share of C y = 30
∴The present age of mother = 60 years.
∴Different between B and C = (1600 – 1000) = 600.

Q77. A litre of water weighs a kilogram, a litre of


Q73. The third proportional of two numbers 9 and 24
another liquid weighs 1.340 kilograms. A mixture of
is
the two weighs 1.270 kilograms per litre. The ratio of
(a) 39 (b) 48
their volumes in a litre of the mixture is:
(c) 72 (d) 64
(a) 27 : 34 (b) 7 : 27
Ans: (d) Here, a = 9, b = 24 and c = ?
(c) 17 : 24 (d) 7 : 17
So,
Ans: (b) According to question,

Q74. If A : B = 2 : 5, B : C = 4 : 3 and C : D = 2 : 1,
then what is value of A : C : D?
(a) 6 : 5 : 2 (b) 7 : 20 : 10
(c) 8 : 30 : 15 (d) 16 : 30 : 15
Ans: (d) A : B = 2 : 5
B:C=4:3=( )
∴The ratio of their volumes = 7 : 27.
C:D=2:1=( )
∴A : B : C : D = 2 : 5 : Q78. Of the three numbers, the first is twice the
∴Value of A : C : D = 16 : 30 : 15 second, and the second is twice the third. The average
of the reciprocal of the numbers is 7/12. The numbers
Q75. 1/2 of A = 2/5 of B = 1/3 of C, then A : B : C is: are:
(a) 4 : 6 : 5 (b) 6 : 4 : 5 (a) 20, 10, 5 (b) 4, 2, 1
(c) 4 : 5 : 6 (d) 5 : 4 : 6 (c) 36, 18, 9 (d) 16, 8, 4
Ans: (c) According to question, Ans: (c) Let third number = x
A = 2k = 4 k then,
B = 5k second number = 2x
first number = 4 x
C=3k=6k
According to question
∴A : B : C = 4 : 5 : 6
( )
Q76. The sum of the present ages of father and son is
90 years. 10 years earlier the ratio of their ages was 5 : ∴x = 9
2. The present age of the father is: ∴first number = 4x = 4 × 9 = 36
(a) 65 (b) 68 second number = 2x = 2 × 9 = 18
(c) 70 (d) 60 third number = x = 9
Page 96 of 516
https://telegram.me/aedahamlibra
Q79. Raman, Manan, and Kamal are partners and
Total work unit = efficiency of worker × time of worker
= 15 × 4 = 60
invest in a business such that Raman invests 2/5th of efficiency of A = 4 – 3 = 1
total and Manan invest 3/8th of the total. What is the Required time = days
ratio of profit of Raman, Manan and Kamal
respectively?
Q2. If 5 men or 7 women can earn ₹ 5,250 per day,
(a) 16 : 15 : 9 (b) 16 : 15 : 31
how much would 7 men and 13 women earn per day ?
(c) 2 : 3 : 5 (d) 15 : 16 : 9
(a) ₹ 11,600 (b) ₹ 11,700
Ans: (a) Let total investment = x
(c) ₹ 16,100 (d) ₹ 17,100
Investment by Raman Ans: (d) 5 men ≡ 7 women
Investment by Manan 7men women
Investment by Kamal ( ) ∴ 7 men + 13 women women
Now,
7women
Ratio of invest of Raman, Manan and Kamal
women

∴Ratio of profit of Raman, Manan and Kamal


= 16 : 15 : 19 Alternate Method:
5 Men = 7 women
Q80. If 2A = 3B and 3B = 2C, then what is A : B : C? Work efficiency of Man: Woman = 7: 5
(a) 3 : 2 : 3 (b) 2 : 3 : 2 Work efficiency of 5 Men = 5 ×7 = 35
(c) 1 : 3 : 1 (d) 2 : 3 : 4 Work efficiency of 7 Men and 13 Women = 7 × 7 + 5 ×
Ans: (a) Here, 65 = 114
2A = 3B3B = 2C Required amount = = 17100
∴ and ∴
∴ Q3. A can compete of a work in 5 days and B, of
the work in 10 days. In how many days both A and B
Let
together can complete the work ?
Then A = 3k, B = 2k, C = 3k,
(a) 10 (b)
∴A : B : C = 3 : 2 : 3
(c)

Time & Work Ans: (b) Total time taken by A = 15 days


Total time taken by B = = 25 days
∴ (A + B)‘s 1 day‘s work
Q1. If A and B together can complete a piece of work
=
in 15 days and B alone in 20 days, in how many days
can A alone complete the work ? ∴ the work will be completed in days.
(a) 60 (b) 45
(c) 40 (d) 30 Q4. A can complete a piece of work in 18 days, B in 20
Ans: (a) (A + B)‘s 1 day‘s work days and C in 30 days, B and C together start the
work and are forced to leave after 2 days. The time
B‘s 1 day‘s work
taken by A alone to complete the remaining work is
∴ A‘s 1 day‘s work (a) 10 days (b) 12 days
∴ A alone will do the work in 60 days (c) 15 days (d) 16 days
Alternate Method: Ans: (c) (B + C)‘s 2 days‘work
Work efficiency of A + B : working efficiency of ( ) ( ) part
Remaining work part

Page 97 of 516
https://telegram.me/aedahamlibra
∴ Time taken by A to complete this part of work Q8. A can do a work in 12 days. When he had worked
for 3 days, B joined him. If they complete the work in
days
3 more days, in how many days can B alone finish the
work?
Q5. One pipe fills a water tank three times faster than
(a) 6 days (b) 12 days
another pipe. If the two pipes together can fill the
(c) 4 days (d) 8 days
empty tank in 36 minutes, then how much time will
Ans: (a) According to Question
the slower pipe alone take to fill the tank ?
(a) 1 hour 21 minutes (b) 1 hour 48 minutes = 1, =1
(c) 2 hours (d) 2 hour 24 minutes
Ans: (d) If time taken by the pipe at faster rate to fill the
tank be x minutes then B = 6 days

Q9. 'x' number of men can finish a piece of work in 30


3x = 4 × 36 days. If there were 6 men more, the work could be
x = 48 minutes finished in 10 days less. The original number of men is
∴ Time taken by the slower pipe (a) 6 (b) 10
= 48 × 3 = 144 minutes = 2 hours 24 minutes (c) 12 (d) 15
Alternate Method: Ans: (c) m1d1 = m2d2
Work efficiency of first pipe : work efficiency of second x (30) = (x + 6) 20
pipe = 3 : 1 2x + 12 = 3x
Total work unit = (3 + 1) 36 = 144 3x – 2x = 12
Time required for slow pipe = min = 2 hrs 24 x = 12 men
min.
Q10. X is 3 times as fast as Y and is able to complete
the work in 40 days less than Y. Then the time in
Q6. 7 men can complete a piece of work in 12 days.
which they can complete the work together is
How many additional men will be required to
(a) 15 days (b) 10 days
complete double the work in 8 days ?
(a) 28 (b) 21 (c) days (d) 5 days
(c) 14 (d) 7 Ans: (a) If X completes a work in x days, Y will do the
Ans: (c) M1D1W2 = M2D2W1 same in 3x days.
7 × 12 × 2 = M2 × 8 × 1 ∴ 3x – x = 40 x = 20
M2 = = 21 ∴ Y will finish the work in 60 days.
∴ No. of additional men = 21 – 7 = 14 ∴ (X + Y)‘s 1 days work =
∴ Both together will complete the work in 15 days.
Q7. A and B can complete a piece of work in 8 days, B
and C can do it in 12 days, C and A can do it in 8 Q11. A is thrice as good a workman as B and is,
days. A, B and C together can complete it in therefore, able to finish a piece of work in 60 days less
(a) 4 days (b) 5 days than B. The time (in days) in which they can do it
(c) 6 days (d) 7 days working together is
Ans: (c) (A + B)‘s 1 day‘s work = (a) 22 (b)
(B + C)‘s 1 day‘s work = (c) 23 (d)
(C + A)‘s 1 day‘s work = Ans: (b) If a co mpletes the work in x days, B will do the
same in 3x days.
On adding,
∴ 3x – x = 60
2 (A + B + C)‘s 1 day‘s work
2x = 60
= =
x = 30 and 3x = 90
∴ (A + B + C)‘s 1 day‘s work = ∴ (A + B)‘s day‘s work
Hence, the work will be compelted in 6 days. = =

Page 98 of 516
https://telegram.me/aedahamlibra
∴ A and B together will do the work in days. (c) 4 hours (d) hours
Ans: (d) Part of the tank filled by both pipes in two hours
Q12. A work can be completed by P and Q in 12 days,
( ) ( )
Q and R in 15 days, R and P in 20 days. In how many
days P alone can finish the work? Remaining part =
(a) 10 (b) 20 Time taken by B in filling the remaining part
(c) 30 (d) 60
hours
Ans: (c) (P + Q)‘s 1 day‘s work = ... (i)
(Q + R)‘s 1 day‘s work = ... (ii) Q15. X and Y can do a piece of work in 30 days. They
(R + P)‘s 1 day‘s work = ... (iii) work together for 6 days and then X quits and Y
finishes the work in 32 more days. In how many days
Adding all three equations, 2 (P + Q + R)‘s 1 day‘s work
can Y do the piece of work alone?
= = (a) 30 days (b) 32 days
∴ (P + Q + R)‘s 1 day‘s work = ... (iv) (c) 34 days (d) 40 days
∴ P‘s 1 day‘s work Ans: (d) (x + y) ‗s 6 days‘ work = ( ) .
= Equation (iv) – equation (ii) Remaining work = ( )
=
Now, work is done by y in 32 days.
∴ P alone will complete the work in 30 days,
Alternate Method: Whole work will be done by y in ( ) = 40 days.
Work efficiency of P + Q : Q + R : P + R =
Q16. Two men A and B started a job in which A was
thrice as good as B and therefore took 60 days less
Working efficiency of P+ Q + R = unit
than B to finish the job. How many days will they take
Total work unit = 5 × 12 = 60 to finish the job, if they start working together?
Work efficiency of P = (P + Q + R) – (Q+ R) = 6 – 4 = 2 (a) 15 days (b) 20 days
units
(c) days (d) 25 days
Required time = days. Ans: (c) If time taken by A be x days,
then time taken by B = 3x days
Q13. If 12 men or 18 women can reap a field in 14 ∴ 3x – x = 60
days, then work ing at the same rate, 8 men and 16 2x – 60
women can reap the same field in: x = 30
(a) 9 days (b) 5 days Time taken by B = 90 days
(c) 7 days (d) 8 days ∴ (A + B)‘s 1 day‘s work
Ans: (a)

12 men ≡18 women ∴The work will be completed in


∴ 2 men ≡ 3 women i.e. days
∴ 8 men + 16 women = 28 women
∴ M1D1 = M2D2
Q17. In a fort, there was sufficient food for 200
18 × 14 = 28 × D2
soldiers for 31 days. After 27 days, 120 soldiers left
days the fort. For how many extra days will be rest of the
food last for the remaining soldiers?
Q14. Pipe A alone can fill a tank in 8 hours. Pipe B (a) 10 days (b) 6 days
alone can fill it in 6 hours. If both the pipes are (c) 4 days (d) 12 days
opened and after 2 h ours pipe A is closed, then the Ans: (b) Ratio of new number of persons in fort : original
other pipe will fill the tank in number of persons in fort = 80 : 200 = 2 : 5
(a) 6 hours (b) hours Hence the food will last for 5/2 days of the o riginal (4
days = 31 days – 27 days)
Page 99 of 516
days https://telegram.me/aedahamlibra
Q21. A, B and C can do a piece of work in 10, 12 and
15 days respectively. A leaves 5 days before the
So, extra days = 6 days
completion of the work and B leaves 2 days after A.
The whole work lasts for
Q18. If 10 men or 18 boys can do a work in 15 days,
(a) 7 days (b) 6 days
then the number of days required by 15 men and 33
(c) 12 days (d) 13 days
boys to do twice the work is
Ans: (a) Suppose, the work was finished in x days. Then,
(a) (b) 8 A‘s (x – 5) day‘s work + B‘s (x – 3) day‘s work
(c) 9 (d) 36 + C‘s x day‘s work = 1.
Ans: (c) 10 men in 15 days
6 (x – 5) + 5 (x – 3) + 4x = 60.
1 man can do the work in 150 days
6x – 30 + 5x – 15 + 4x = 60
1 man can do twice the work in 300 days Similarly, 18
15x = 60 + 30 + 15
boys in 15 days
15x = 105 x = 7 days.
1 boy can do the work in 270 days
1 boy can do twice the work in 540 days
Q22. Two pipes A and B can fill a tank in 6 hours and
Now, if there are 15 men and 33 boys trying to do twic e
4 hours respectively. If they are opened on alternate
the work then
hours and if pipe A is opened first, then the tank shall
( ) ( ) be full in
(a) hrs (b) 5 hrs
(c) hrs (d) 6 hrs
It will take 9 days for 15 men and 33 Boys to do twice
the work. Ans: (b) A‘s work in 1 hour =
B‘s work in 1 hour =
Q19. A is thrice as good a workman as B and takes 60
(A + B)‘s 2 hour‘s work when opened alternately
days less than B for doing a job. The time in which
they can do it together is: =( )
(a) 15 days (b) 30 days (A + B)‘s 4 hour‘s work when opened alternately
(c) days (d) 60 days =
Ans: (c) If A can finish a work = x days Remaining part = ( )
B will do this work = 3x days
From question, A – B = 3x – x = 60 x = 30 Now, it is A‘s turn and part is filled by A in 1 hour.
A = 30 days, B = 90 days ∴ Total time taken to fill the tank = (4 + 1) hrs. = 5 hrs.
A‘s 1 day work + B‘s 1 day work =
Q23. Water flows at the rate of 10 metres per minute
So, A and B working together can complete work = = from a cylindrical pipe 5 mm in diameter. How long it
22.5 days lake to fill up a conical vessel whose diameter at the
base is 30 cm and depth 24 cm?
Q20. A can do a piece of work in 20 days which B can (a) 28 minutes 48 seconds (b) 51 minutes 12 seconds
do in 12 days. B worked at it for 9 days. A can finish (c) 51 minutes 24 seconds (d) 28 minutes 36 seconds
the remaining work in : Ans: (a) Volume of water flowing from the pipe in 1
(a) 5 days (b) 7 days minute
(c) 11 days (d) 3 days Volume of conical vessel = cu.cm.
Ans: (a) B‘s 1 day work =
∴ Required time =
B‘s 9 day‘s work = = 28 minutes 48 seconds
Remaining work =
Q24. A can do as much work as B and C together can
∴ A can finish this work in days = 5 days
do. A and B can together do a piece of work in 9 hours
36 minutes and C can do it in 48 hours. The time (in
hours) that B needs to do the work alone, is :
Page 100 of 516
(a) 18
(c) 30
(b) 21
(d) 12
https://telegram.me/aedahamlibra
Ans: (c) 200 men do work in 50 days.

Ans: (b) 9 hours 36 minutes
= = hours = hours
(A + B)‘s 1 hour‘s work = hours M2 × 100 = 200 × 50 × 3
C‘s 1 hour‘s work = M2 = 300
∴ Additional men = 100
(A + B + C)‘s 1 hour‘s work = ... (i)
A‘s 1 hours work = (B + C)‘s 1 hour‘s work... (ii) Q28. If 10 men or 20 women or 40 children can do a
2 ×A‘s 1 hour‘s work = piece of work in 7 months, then 5 men, 5 women and 5
children together can do half of the work in:
A‘s 1 hour‘s work =
(a) 8 months (b) 6 months
∴ B‘s 1 hour‘s work = = (c) 4 months (d) 5 months
∴ B alone will finish the work in 24 hours Ans: (c) 10 men = 20 women = 40 children
i.e. 1m = 2w = 4 m
Q25. A can do a work in 20 days and B can do the ∴ Sm + Sw + Sch
same work in 30 days. In how many days can A and B 5 × 4 + 5 × 2 + 5 = 35
together do the work ?
(a) 15 (b) 16
(c) 10 (d) 12
Ans: (d) A's 1 day's work =
B's 1 day‘s work = = D2

(A + B)'s 1 day's work = ( ) D2 = 4 months.

∴ Both A and B will finish the work in = 12 days. Q29. A can do a piece of work in 12 days whil eB
alone can do it in 15 days. With the help of C they can
Q26. 3 men and 7 women can do a job in 5 days. while finish it in 5 days. If they are paid ₹ 960 for the whole
4 men and 6 women can do it in 4 days . The number work how much money A gets?
of days required for a group of 10 women working (a) ₹ 480 (b) ₹ 240
together, at the same rate as before, to finish the same (c) ₹ 320 (d) ₹ 400
job is: Ans: (d) Work done by A and B in 5 days
(a) 30 (b) 36
= ( ) ( )=
(c) 40 (d) 20
Ans: (d) 3 × 5 men + 7 × 5 women Time taken by C in doing
= 4 × 4 men + 6 ×4 women Work = 5 days
16 men – 15 men = 35 women – 24 women ∴ C will complete in 20 days.
∴ 1 man = 11 women ∴ Ratio of wages = =5:4:3
∴ 3 men + 7 women = 40 women
∴ M1 D1 = M2 D2 ∴ Amount received by A = = ₹ 400
40 × 5 = 10 × D2
D2 = 20 days Q30. A can finish a work in 18 days and B can do the
same work in 15 days. B worked for 10 days and left
Q27. A man undertakes to do a certain work in 150 the job. In how many days. A alone can finish the
days. He employs 200 men. He finds that only a remaining work ?
quarter of the work is done in 50 days. The number of (a) 8 (b) 6
additional men that should be appointed so that the (c) 5½ (d) 5
whole work will be finished in time is : Ans: (b) Work done by B in 10 days =
(a) 50 (b) 75
Remaining work = 1 –
(c) 100 (d) 125
Page 101 of 516
∴ Time taken by A = https://telegram.me/aedahamlibra
= 6 days. ∴ Time taken by A in completing the work
= 6 days.
Q31. One man, 3 women and 4 boys can do a piece of ∴ B‘s 1 day‘s work =
work in 96 hours, 2 men and 8 boys can do it in 80 ∴ B alone will complete the work in 6 days.
hours, 2 men and 3 women can do it in 120 hours. 5
men and 12 boys can do it in Q34. A and B together can complete a piece of work
(a) hours (b) hours in 12 days, B and C can do it in 20 days and C and A
can do it in 15 days. A, B and C together can complete
(c) hours (d) 44 hours
it in
Ans: (c) 1 hr‘s work of 1 man and 4 boys = (a) 8 days (b) 10 days
(c) 12 days (d) 6 days
1 hr‘s work of 1 man and 3 women =
Ans: (b) (A + B)‘s 1 day‘s work =
1 hr work of 3 women
= (B + C)‘s 1 day‘s work =

1 hr work of 2 men = (C + A)‘s 1 day‘s work =


On adding all three,
1 hr work of 4 boys = =
2 (A + B + C)‘s 1 day‘s work
∴ 2 men = 3 women = 4 boys
∴ 2 men + 8 boys = 12 boys
5 men + 12 boys = 22 boys
∴ By M1 D1 = M2D2 ∴ (A + B + C)‘s 1 day‘x work
12 × 80 = 22 × D2 Hence, the work will be finished in 10 days.
D2 = = hours
Q35. A can do a certain work in the same time in
which B and C together can do it. If A and B together
Q32. Ronald and Elan are working on an Assignment.
could do it in 10 days and C alone in 50 days, then B
Ronald takes 6 hours to type 32 pages on a computer,
alone could do it in
While Elan takes 5 hours to type 40 pages. How much
(a) 15 days (b) 20 days
time will they take working together on two differen t
(c) 25 days (d) 30 days
computers to type an assignment of 110 pages?
(a) 7 hrs. 30 min. (b) 8 hrs. Ans: (c) (A + B)‘s 1 day‘s work ;
(c) 8 hrs. 15 min. (d) 8 hrs. 25 min. C‘s 1day‘s work
Ans: (c) Ronald‘s 1 hour‘s work = pages (A + B + C)‘s 1 day‘s work
Elan‘s 1 hour‘s work = 8 pages ( ) ... (1)
1 hour‘s work of the both
Also, A‘s 1 day‘s work = (B + C)‘s 1 day‘s work ... (2)
= pages
From (1) and (2) , we get : 2 × (A‘s 1 day‘s work)
∴ Requried time
A‘s 1 day‘s work
= hours = 8 hours 15 minutes
∴B‘s 1 day‘s work

Q33. A and B together can complete a work in 3 days. ( )


They start together. But, after 2 days, B left the work. So, B alone could do the work in 25 days.
If the work is completed after 2 more days, B alone
could do the work in Q36. A does 20% less work than B. If A can complete
(a) 6 days (b) 8 days
a piece of work in hours, then B can do it in
(c) 10 days (d) 4 days
(a) 6 hours (b) 8 hours
Ans: (a) (A + B)‘s 2 days‘ work
(c) 10 hours (d) 4 hours
Remaining work Ans: (a) Efficiency of A and B = 4 : 5
Time taken by A in doing work = 2 days Ratio of respective time = 5 : 4
∴ Time taken by B
Page 102 of 516
hours https://telegram.me/aedahamlibra
1 day's work
= =
Q37. 12 men construct 1.5 km of road in 7 days. 28 ∴ (Ram's + Shyam's + Hari's)
men will construct 12 km of roads in 1 day's work =
(a) 20 days (b) 24 days
(c) 28 days (d) 38 days ∴ Ram's 1 day's work =
Ans: (b) Let the required number of days be x. ∴ Ram alone will do the work in 30 days.
Then, more men, more km (Direct proportion)
more days, more km (Direct proportion) Q41. Raju can do a piece of work in 20 days, while
men Ram can do it in 30 days. If both of them work at it
together, th en the number of days in which they will
}
be able to finish the work is
∴12 × 7 × 12 = 28 × x × 1.5 (a) 12 days (b) 10 days
(c) 50 days (d) 25 days
Ans: (a) Raju and Ram together can finish the work in
( ) days
Q38. A can do a piece of work in 10 days. B can do the
same work in 15 days. How long would both of them
take to do the same work ? Q42. A and B can do a job in 12 days. B and C in 15
(a) 2 days (b) 4 days days and C and A in 20 days. How long would A take
(c) 6 days (d) 8 days to do that work ?
(a) 20 days (b) 60 days
Ans: (c) A‘s 1 day‘s work and B‘s 1 day‘s work
(c) 30 days (d) 40 days
Ans: (c) (A + B)'s 1 day's work =
∴ (A + B)‘s 1 day‘s work ( ) (B + C)' 1 day's work =
So both together will finish the work in 6 days.
(C + A)' 1 days' work =

Q39. 3 men or 5 women can do'a work in 12 days. ∴ (A + B + C)'s 1 day's work =
How long will 6 men and 5 women take to finish the ∴ A' 1 day's work = =
work?
(a) 4 days (b) 5 days
(c) 6 days (d) 7 days Q43. A can do of work in 28 days, B can do of the
Ans: (a) 3 men = 5 women same work in 20 days. The number of days they will
6 men + 5 women = 15 women take to complete if they do it together is
∴ By M1D1 = M2D2 (a) days (b) days
5 × 12 = 15 × D2
(c) days (d) days
D2 = = 4 days
Ans: (d) A can complete whole work in = 32 days
Q40. A piece of work can be done by Ram and Shyam B can complete whole work in = 24 days
in 12 days, by Shyam and Hari in 15 days and by Hari A and B together can complete whole work in
and Ram in 20 days. Ram alone will complete the days
work in
(a) 30 days (b) 32 days
Q44. A is twice as good a workman as B and together
(c) 36 days (d) 42 days
they finish a piece of work in 20 days. In how many
Ans: (a) (Ram's + Shyam's) 1 day's work = days will A alone finish the work?
(Shyam's + Hari's) 1 day's work = (a) 30 days (b) 25 days
(c) 40 days (d) 35 days
(Hari's + Ram's) 1 day's work =
Ans: (a) If A can finish the work in x days, B finish th e
Adding all three, same work in 2x days.
2 (Ram's + Shyam's + Hari's)
Page 103 of 516
https://telegram.me/aedahamlibra
(A + B) together finish work in 20 days Ans: (c) Let Pratibha can finish the work in x days then,
Sonia can finish the same work in 3x days
According to question
x = 30 days 3x – x = 60
2x = 60 x = 30
Q45. A, B and C together can do a piece of work in 40 Pratibha and Sonia can ind ividually complete the work in
days. After working with B and C for 16 days, A 30 days and 90 days respectively.
leaves and then B and C complete the remaining work
in 40 days more. A alone could do the work in Q48. Three pipes A, B and C can fill a tank in 6 hours.
(a) 80 days (b) 90 days After working it together for 2 hours, C is closed and
(c) 100 days (d) 120 days A and B can fill the remaining part in 7 hours. The
Ans: (c) (A + B + C)'s 1 day's work =( ) part of number of hours taken by C alone to fill the tank is
(a) 10 (b) 12
whole work
(c) 14 (d) 16
(A + B + C)'s 16 days work = of whole work (B
Ans: (c)
+ C) completes remaining wo rk in 40 days (B + C)
(A + B + C) can do part of work in 2 days.
completes ( ) part of work in 40 days.
Remaining work =
(B + C) completes whole work in days.
In one hour (A + B) can do part of work

( )

C = 14 hours

Q49. Two pipes A and B can fill a tank in 36 min. and


A alone can complete whole work in 100 days. 45 min. respectively. Another pipe C can empty the
tank in 30 min. First A and B are opened. After 7
Q46. Seventy -five men are employed to lay down a minutes, C is also opened. The tank is filled up in
railway line in 3 months. Due to certain emergency (a) 39 min. (b) 46 min.
conditions, the work was to be finished in 18 days. (c) 40 min. (d) 45 min.
How many more men should be employed to complete Ans: (a) In one minute (A + B) can together fill
the work in the desired time ?
part.
(a) 300 (b) 325
(c) 350 (d) 375 In 7 minutes part of tank filled =
Ans: (a) More the no. of men less time they take to Remaining part
complete work.
In 8th minutes, part filled by A, B and C altogether =
Let x men are added
(Inverse Proportion)
part of tank filled by (A + B + C)
= = 39 minutes
375 – 75 = x
x = 300
Q50. A and B working separately can do a piece of
work in 9 and 15 days respectively. If they work for a
Q47. Pratibha is thrice as efficient as Sonia and is
day alternately, with A beginning, then the work will
therefore able to finish a piece of work in 60 days less
completed in
than Sonia. Pratibha and Sonia can individually
(a) 10 days (b) 11 days
complete the work respectively in
(c) 9 days (d) 12 days
(a) 30, 60 days (b) 60, 90 days
(c) 30, 90 days (d) 40, 120 days Ans: (b) Two days work =

Page 104 of 516


Ten days work = https://telegram.me/aedahamlibra
operate simultaneously, how much time is needed to
empty the tank?
Remaining work = which is done by A on 11th (a) 18 minutes (b) 14 minutes
day. (c) 15 minutes (d) 30 minutes
Hence, the work will be completed in 11 days. Ans: (a) Work done by 1st tap in one minute =

Q51. A pipe can fill a tank in x hours and another can Work done by 2nd tap in one minute =
empty it in y hours. They can together fill it in (y > x) Both tap one minute work =
(a) x – y (b) y – x
=
(c)
=
Ans: (d) Work done by A in one hour =
Work done by B in one hour = Q55. If 4 men or 8 women can do a piece of work in 15
Both A & B together in work one hour days, in how many days can 6 men and 12 women do
= = the same piece of work ?
(a) 5 days (b) 20 days
Both A & B fill tank in hours. (c) 15 days (d) 30 days
Ans: (a) 4 M = 8W
Q52. A and B together can do a piece of work in 6 1M = 2W
days. If A can alone do the work in 18 days, then the Now, 6M + 12 W = 6 × 2W + 12W
number of days required for B to finish the work is = 12W + 12W
(a) 12 (b) 9 = 24 W
(c) 15 (d) 10 Now, M1 d1 = M2 d2
Ans: (b) A and B can complete work in 6 days 8W × 15 = 24 W × d2
A can complete in 18 days ∴ d2 = = 5 days
Let B can complete in x days
∴ One day work be equal to
Q56. 4 men and 6 women complete a work in 8 days, 2
men and 9 women also complete in 8 days. The
number of days 18 women complete the work is :
(a) days (b) days
3x = x + 18 (c) days (d) days
x = 9 days
Ans: (d) Now, M1d1 = M2d2
(4M + 6W) × 8 = (2M + 9W) × 8
Q53. A's 2 days work is equal to B's 3 days work. If A
4M + 6W = 2M + 9W
can complete the work in 8 days then to complete the
2M = 3W
work B will take :
(a) 14 days (b) 15 days 1M=
(c) 16 days (d) 12 days Now, 4M + 6W =
Ans: (d) A‘s 2 days work = B‘s 3 days work.
Here 12W complete a work in 8 days.
A complete a work in 8 days
So, let 18 women complete a work in x days
A‘s 1 days work = days 12W × 8 = 18W × x
A‘s 2 days work = work. x= days = days
Now, B‘s 3 days work = work
Q57. 12 monkeys can eat 12 bananas in 12 minutes. In
B‘s 1 days work = work
how many minutes can 4 monkeys eat 4 bananas ?
∴ B‘s will take 12 days to complet the work. (a) 12 (b) 10
(c) 4 (d) 8
Q54. A tap can empt y a tank in 30 minutes. A second Ans: (a) 12 monkeys can eat 12 bananas in 12 minutes
tap can empty it in 45 minutes. If both the taps So 1 monkey can eat 1 banana in 12 minutes
Page 105 of 516
https://telegram.me/aedahamlibra
4 monkeys can eat 4 banana in 12 minutes Q62. A can finish a work in 7 days . B can finish the
same work i n 9 days . The days required to finish the
Q58. 20 men can do a piece of work in 18 days. They work by both of them together.
worked together for 3 days, then 5 men joined them. (a)
In how many more days is the work completed ?
(a) 12 (b) 14 (c)
(c) 15 (d) 13 Ans: (c) A can do work in = 7 day
Ans: (a) 1 men 1 day's work = B can do work in = 9 day
Both can do work in =
20 men 3 days work =
days
Remaining work =
25 men 1 days work = Q63. A,B and C contract a work for ₹ 440. Together
Now work is done by them in 1 day A and B are to do 9/11 of the work. The share o fC
should be:
work is done by them in days
(a) 75 (b) 90
(c) 100 (d) 80
Q59. If 20 women can lay a road of length 100m in 10
Ans: (d) Remaining work =
days. 10 women can lay the same road of length 50 m
in : C will get = = 80
(a) 20 days (b) 10 days
(c) 5 days (d) 15 days Q64. A is twice as good as B and together they finish a
Ans: (b) Required number of days = = 10 days piece of work in 16 days. The number of days taken
by A alone to finish the work is
(a) 20 days (b) 21 days
Q60. A contractor was engaged to construct a road in
(c) 22 days (d) 24 days
16 days. After working for 12 days with 20 labours it
Ans: (d) Let B takes day = 2x
was found that only 5/8th of th e road had been
A takes = x
constructed. To complete the work in stipulated time
the number of extra labours required is : , x = 24
(a) 12 (b) 10
(c) 18 (d) 16 Q65. If 4 men and 6 women can complete a work in 8
Ans: (d) days, while 3 men and 7 women can complete it in 10
m1 D1.w2 = m2 D2 W1 days, then 10 women complete it in
(a) 40 days (b) 45 days
(c) 35 days (d) 50 days
Ans: (a) Let 1 man's 1 day's work = x
and 1 woman's 1 day's work = y
Hence, 36 – 20 = 16 more men needed to complete the
Then, 4x + 6y and
remaining work in 4 days.
Solving two equations,
Q61. A can do 1/3rd of a work in 5 days and B can do We get
2/5th of this work in 10 days. Both A and B, together ∴ 1 women's 1 day's work
can do the work in
(a) days (b) days 10 women's 1 day's work ( )
Hence, 10 women will complete the work in 40 days.
(c) days (d) 10 days
Ans: (c) A can do work in = 5 × 3 = 15 days Q66. A and B can separately finish a piece of work in
B can do work in = days 20 days and 15 days respectively. They worked
together for 6 days, after whi ch B was replaced by C.
Together, or days
If the work was finished in next 4 days, then the
number of days in which C alone could do the work is
Page 106 of 516
(a) 50 days
(c) 40 days
https://telegram.me/aedahamlibra
(b) 30 days
(d) 60 days
(c) 2400
Ans: (c)
(d) 2000

Ans: (c) (A + B)'s 6 day's work ( )


Ratio of wages of A, B and C
(A + C)'s 4 day's work =
= 15 : 12 : 10
(A + C)'s 1 day's work =
∴Amount received by B
A's 1 day's work
C's 1 days work ( ) Q70. A piece of work was finished by A, B, and C
together. A and B together finished 60% of the work
Hence C alone can finish the work in 40 days.
and B and C together finished 70% of work. Who
among the three is the most efficient?
Q67. Raman can do a work in 5 days, Jatin can do the
(a) A (b) B
same work in 7 days and Sachin can do the same work
(c) C (d) A or B
in 9 days. If they do the same work together and they
Ans: (c) According to question,
are paid ₹ 2860, then what is th e share (in ₹) of
A + B = 60%
Raman?
∴ (A + B) + (B + C) – (A + B + C) = B
(a) 1260 (b) 700
(60 + 70 – 100) = 30
(c) 900 (d) 870
∴B = 30%
Ans: (a) Raman's 1 day's work A = 30% and
Jatin's 1 day's work C = 40%
Hence, C is most efficient.
Sachin's 1 day's work
∴Ratio of their wages = 63 : 45 : 35 Q71. A can do a piece of work in 5 days and B in 4
∴Raman's share = 1260. days. How long will they take to do the same work
when working together?
Q68. A, B and C can complete a work in 20, 24 and 30 (a)
days respectively. All three of them starts together but
(c) (d) 9
after 4 days A leaves the job and B left the job 6 days
before the work was completed. C completed the Ans: (b) According to question,
remaining work alone. In how many days was the A's 1 day's work
total work completed? B's 1 day's work
(a) 10 (b) 12
(c) 14 (d) 16 (A + B)'s day's work ( )
Ans: (c) Suppose, the work was finished in x days. Then, ∴A and B can do work together in or days.
A's 4 day's work + B's (x – 6) day's work + C's x day's
work = 1
Q72. A does 80% of a work in 20 days. He then calls
in B and they together finish the remaining work in 4
days. How long B alone would take to do the whole
work?
24 + 5x – 30 + 4x = 120 (a) 12.5 days (b) 100 days
9x = 126 (c) 22.5 days (d) 35 days

∴ days Ans: (b) Work done by A in 20 days


Work done by A in 1 day ... (i)
Q69. A can do a work in 8 days, B can do the same Work done by A and B in 4 days
work in 10 days and C can do the same work in 12
(Because remaining 20% is done in 4 days by A and B).
days. If all three of them do the same work together
and they are paid ₹ 7400, then what is the share (in ₹) ∴Work done by A and B in 1 day ... (ii)
of B? ∴Work done by B in 1 day
(a) 2600 (b) 3000
Page 107 of 516
https://telegram.me/aedahamlibra
∴B can complete the work in 100 days. Suppose the waste pip e can empty the full tank in x
hours.
Q73. P is four times as efficient as Q.P can complete a Then, part emptied by waste pipe in 1 hour
work in 45 days less than Q. If both of them work All the three pipes can fill the tank in 20 hours
together, then in how many days the work will be
i.e. part filled by all three pipes in hour
completed?
(a) 10 (b) 12 now,
(c) 15 (d) 30
Ans: (b) According to question,
If P can complete a work in 1 day, Q can complete the
same work in 4 days. ∴
Hence, if the difference is 3 days, Q can complete the i.e, the waste pipe can empty the full tank in hours
work in 4 days
Given that the waste pipe can empty 80 gallons per hour.
If the difference is 45 days, Q can complete the work
in 60 days Therefore, in hours, it can empty =
∴Q's 1 day's work 600gallons.
Hence, volume of the tank = 600 gallons.
∴P's 1 day's work
∴ (P + Q)'s 1 day's work
Speed Time and Distance
( )
∴P and Q together can do work in 12 days.
Q1. Buses start from a bus terminal with a speed of 20
km/hr at intervals of 10 minutes. What is the speed of
Q74. A certain number of men complete a piece of
a man coming from the opposite directiion towards
work in 60 days. If there were 8 men more, the work
the bus terminal if he meets the buses at intervals of 8
can be finished in 10 days less. The number of men
minutes ?
originally is :
(a) 3 km/hr (b) 4 km/hr
(a) 32 (b) 40
(c) 5 km/hr (d) 7 km/hr
(c) 36 (d) 30
Ans: (c) Distance coverd in 10 minute s at 20kmph =
Ans: (b) Here,
distance covered in 8 minutes at (20 + x) kmph
M1 = x,D1 = 60 days
M2 = (x + 8),D2 = 50 days
Now, 200 = 160 + 8x
M1D1 = M2D2 8x = 40
x × 60 = (x + 8) × 50
kmph
60x = 50x + 400
10x = 400
∴x = 40 Q2. A train, 300 m long, passed a man, walking along
∴The number of men originally is = 40. the line in the same direction at the rate of 3 km/hr in
33 seconds. The speed of the train is
(a) 30 km/hr (b) 32 km/hr
Q75. Two inlet pipes can fill a cistern in 10 and 12
hours respectively and an outlet pipe can empty 80 (c) km/hr (d) km/hr
gallons of water per hour. All the three pipes wor king Ans: (d) If the speed of the train be x kmph, then relative
together can fill the empty cistern in 20 hours. What is speed
the capacity (in gallons) of the tank? = (x – 3) kmph.
(a) 360 (b) 300 = (x – 3) × m/sec
(c) 600 (d) 900
Ans: (c) Part filled by first pipe in 1 hour
Part filled by second pipe in 1 hour
5400 = 33 × 5 (x – 3)
360 = 11 (x – 3)
Page 108 of 516
11x – 33 = 360 https://telegram.me/aedahamlibra
Q7. A boatman rows 1 km in 5 minutes, along the
kmph
stream and 6 km in 1 hour against the stream. The
speed of the stream is
Q3. By walking at of his usual speed, a man reaches (a) 3 kmph (b) 6 kmph
his office 20 minutes later than his usual time. The (c) 10 kmph (d) 12 kmph
usual time taken by him to reach his office is Ans: (a) Speed of current
(a) 75 minutes (b) 60 minutes = (Rate downstream – Rate upstream)
(c) 40 minutes (d) 30 minutes
= (12 – 6) kmph [Rate downstream]
Ans: (b) of usual time = Usual time + 20 minutes
rd = = 3 kmph
of usual time = 20 minutes
Usual time = 60 minutes
Q8. Walking at of his usual speed a man is 25
Q4. A train, 240 m long crosses a man walking along minutes too late. His usual time to cover this distance
the line in opposite direction at the rate of 3 kmp h in is
10 seconds. The speed of the train is (a) 2 hours 30 minutes (b) 2 hours 15 minutes
(a) 63 kmph (b) 75 kmph (c) 2 hours 25 minutes (d) 2 hours 10 minutes
(c) 83.4 kmph (d) 86.4 kmph Ans: (a)
Ans: (c) Let train speed be x
∴ 7x – 6x = 25
relative speed = (x + 3) kmph
x = 25 min
∴ Time =
T1 6x = hr
= 2hr 30 min
x + 3 = 86.4
x = 83.4 kmph Q9. A student goes to school at the rate of km/h
and reaches 6 minutes late. If he travels at the speed
Q5. In a 100m race, Kamal defeats Bimal by 5 of 3 km/h. he is 10 minutes early. The distance (in km)
seconds. If the speed of Kamal is 18 Kmph, then the between the school and his house is
speed of Bimal is (a) 5 (b) 4
(a) 15.4 kmph (b) 14.5 kmph (c) 3 (d) 1
(c) 14.4 kmph (d) 14 kmph Ans: (b) Let the required distance be x km.
Ans: (c) Time taken by Kamal

∴ Time taken by Bimal = 20 + 5 = 25


∴ Bimal‘s speed = =4m
= kmph = 14.4 kmph. x = 4 km.

Q6. A man can row 6 km/h in sti ll water. If the speed Q10. A thief is noticed by a policeman from a distance
of the current is 2 km/h, it takes 3 hours more in of 200 m. The thief starts running and the policeman
upstream than in the down –stream for the same chases him. The thief and the policeman run at the
distance. The distance is rate of 10 km and 11 km per hour respectively. The
(a) 30 km (b) 24 km distance (in metres) between them after 6 minutes is
(c) 20 km (d) 32 km (a) 190 (b) 200
Ans: (b) Let the required distance be x km. (c) 100 (d) 150
Ans: (c) Relative speed = 11 – 10 = 1 kmph
∴ =3
Distance covered in 6 minutes
=3
metre = 100metre
=3 ∴ Remaining distance
x = 3 × 8 = 24 km. = 200 – 100 = 100 metre
Page 109 of 516
https://telegram.me/aedahamlibra
Q11. Walking at 5 km/hr a student reaches his school
According to question

from his house 15 minutes early and walking at 3


km/hr he is late by 9 minutes. What is the distance 9x – 18 = 10x – 40x = 22km/h
between his school and his house? Required length = = 50 m
(a) 5 km (b) 8 km
(c) 3 km (d) 2 km Q14. A car covers four successive 6 km stretches at
Ans: (c) Let the required distance be x km. speeds of 25 kmph, 50 kmph, 75 kmph and 150 kmph
∴ respectively. Its average speed over this distance is
=2 (a) 25 kmph (b) 50 kmph
(c) 75 kmph (d) 150 kmph
2x = 2 × 3 x = 3 km
Ans: (b) AverageSpeed =
Q12. With average speed of 40 km/hour, a train
reaches its‘ destination in time. If it goes with an * +
average speed of 35 km/hour, it is late by 15 minutes.
The total journey is km/hr
(a) 30 km (b) 40 km
(c) 70 km (d) 80 km Q15. A ship is moving at a speed of 30 km/hr. To
Ans: (c) If the total length of journey be x km, then know the depth of the ocean beneath it, it sends a
radiowave which travels at a speed 200 m/s. The ship
receives the signal after it has moved 500 m. The
depth of the ocean is
(a) 6 km (b) 12 km
(c) √ m (d) 8 km
Ans: (b) Speed = 30 km/h = m/s
km
Distance covered by ship = 500m

Q13. A train overtakes two persons who are walking Time =


in the same direction in wh ich the train is running, at Distance covered by wave = 60 × 200 = 12000 m
the rate of 2 kmph and 4 kmph and passes them Depth of sea =
completely in 9 and 10 seconds respectively. The
length of the train (in metres): Q16. A man walks ‗a‗ km in ‗b‘ hours. The time taken
(a) 72 (b) 45 to walk 200 metres is:
(c) 54 (d) 50
(a) hours (b) hours
Ans: (d) Let the length of train be x km and its speed by
kmph. (c) hours (d) hours
... (i) Ans: (b) 1 km = 1000 m
D=S×T
... (ii)
a km = S × b hr
By dividing equation (i) by (ii),
S=
Now, D = 200 m
10y – 40 = 9y – 18 Time taken =
y = 40 – 18 = 22 = hrs
From equation (i) ,
Q17. A man walks a certain distance and rides back
taking a total time of 37 minutes. He could walk both
metre
ways in 55 minutes. How long would he take to ride
Alternate Method: both ways?
Let Speed of train be x km/h (a) 9.5 minutes (b) 18 minutes
Page 110 of 516
(c) 19 minutes https://telegram.me/aedahamlibra
(d) 20 minutes
Ans: (c) To walk both ways, duration = 55 minutes
Ans: (c) Let the required distnace be x km.
Difference in the times taken at two speeds
∴ To walk one way, duration = minutes = 8min =
To walk one way + To ride one way = 37 minutes
∴ To ride both ways = 19 minutes

Q18. The minute hand of a big wall -clock is 35 cm


long. Taking , length of the arc, its extremity
moves in 18 seconds is : Hence, the required distance is 4km.
(a) 11 cm (b) 1.1 cm
(c) 6.6 cm (d) 6 cm Q22. A train 270 metre long is running at a speed of
Ans: (b) Length of arc in 18 seconds = ( ) × 36 km per hour, then it w ill cross a bridge of length
circumference 180 metres in :
(a) 40 sec (b) 45 sec
= = 1.1 cm
(c) 50 sec (d) 35 sec
Ans: (b) Speed of train = 36 kmph
Q19. A tra in 100 metres long meets a man going in
=( )m/sec. = 10 m/sec.
opposite Directions at 5 km/hr and passes him in
seconds. What is the speed of the train in km/hr ? Required time =
(a) 45 km/hr (b) 60 km/hr = = 45 seconds
(c) 55 km/hr (d) 50 km/hr
Ans: (a) Let speed of train = x km/hr
Q23. On a journey across Kolkata, a taxi averages 50
Distance travelled by train
km per hour for 50% of the distance. 40 km per hour
= Relative speed of train × Time
for 40% of it and 20 km per hour for the remaining.
100 m = (x + 5) km/hr × seconds The average speed in km/hour, for the whole journey
km = (x + 5) × ( )hrs is :
(a) 42 (b) 40
x + 5 = 50
(c) 35 (d) 45
∴x = 45 km/hr
Ans: (b) Total distance = 100 km.

Q20. Two trains 108 m and 112 m in length are Total time = = = hours
running towards each other on the parallel lines a t a ∴ Average speed = = 40 kmph
speed of 45 km/hr and 54 km/hr respectively. To cross
each other after they meet, it will take Q24. A train is moving at a speed of 80 km/h and
(a) 10 sec (b) 12 sec covers a certain distance in 4.5 hours. The speed of
(c) 9 sec (d) 8 sec the train to cover the same distance in 4 hours is
Ans: (d) Relative speed = (45+54) = 99km/hr = (a) 90 km/h (b) 100 km/h
m/sec (c) 70 km/h (d) 85 km/h
Distance covered in crossing each other Ans: (a) Distance covered in 4.5 h = 80 × 4.5 = 360 km
= (108+112) = 220m Speed = km = 90 km/h
Required time =
Q25. Water flows at the rate of 10 metres per minute
Q21. Walking at a speed of 5 km/hr, a man reaches from a cylindrical pipe 5 mm in diameter. How long it
his office 6 minutes late. Walking at 6 km/hr, he lake to fill up a conical vessel whose diameter at the
reaches there 2 minutes early. The distance of his base is 30 cm and depth 24 cm?
office is (a) 28 minutes 48 seconds (b) 51 minutes 12 seconds
(a) 2 km (b) 3 km (c) 51 minutes 24 seconds (d) 28 minutes 36 seconds
(c) 4 km (d) 3.5 km Ans: (a) Volume of water flowing from the pipe in 1
minute
Page 111 of 516
Volume of conical vessel = https://telegram.me/aedahamlibra
cu.cm.
Q30. A certain distance is covered at a certain speed.
∴ Required time = If half of this distance is covered in double the time,
= 28 minutes 48 seconds the ratio of the two speeds is
(a) 1 : 4 (b) 2 : 1
Q26. A train moving at a rate of 36 km/hr. crosses a (c) 1 : 2 (d) 4 : 1
standing man in 10 seconds. It will cross a platform 55 Ans: (d) If the original speed be S 1 units and time = t1
metres long, in: units and distance = D, then
(a) seconds (b) 6 seconds
(c) 7 seconds (d) seconds
Ans: (d) Speeds of train = 36 kmph = 36 × = 10 m/sec
Length of train = 10 × 10 = 100 metre
∴ Required time =
= 15.5 seconds seconds.

Q27. A man can swim 3 km/hr. in still water. If the Q31. Two cars are moving with speeds v 1, v 2 towards
velocity of the stream is 2 km/hr. the time taken by a crossing along two roads. If their distances from the
him to swim to a place 10 km upstream and back is : crossing be 40 metres and 50 metres at an instant of
(a) time then they do not collide if their speeds are such
(c) 10 hr. (d) 12 hr. that
Ans: (d) Downstream speed = 5 kmph (a) v1 : v2 ≠ 5 : 4 (b) v1 : v2 = 25 : 16
Upstream speed = 1 kmph (c) v1 : v2 = 16 : 25 (d) v1 : v2 ≠ 4 : 5
Ans: (d)
∴ Required time = = 12 hours

Q28. A man rows dow n a river 15 km in 3 hrs. with


the stream and returns in hrs. The rate at which he
rows in still water is
(a) 2.5 km/hr (b) 1.5 km/hr If then they will collide i,e. cars will reach at the
(c) 3.5 km/hr (d) 4.5 km/hr
same time.
Ans: (c) Speed of person in still water = x kmph and
speed of current = y kmph
∴x+y= = 5 kmph
x–y= = 2 kmph Q32. A train 100 metres long moving at a speed of 50
km/hr. crosses a train 120 metres long coming from
On adding,
opposite direction in 6 sec. The speed of the second
2x = 7 x = = 3.5 kmph train is
(a) 60 km/hr. (b) 82 km/hr.
Q29. A man rides at the rate of 18 km/hr, but stops (c) 70 km/hr. (d) 74 km/hr.
for 6 mins. to change horses at the end of every 7th Ans: (b) Let speed of the second train = x km/hr.
km. The time that he will take to cover a distance of Relative speed of trains = (50 + x) km/hr.
90 km is Distance travelled by trains = (100 + 120) = 220 metres
(a) 6 hrs. (b) 6 hrs. 12 min. Distance = Speed × Time
(c) 6 hrs. 18 min. (d) 6 hrs 24 min. ( ) ( )hr
Ans: (b) Number of stoppages =
50 + x =
∴ Total time = ( ) hours + minutes 50 + x = 132
= 6 hours 12 minutes x = 132 – 50 = 82 km/hr
Page 112 of 516
https://telegram.me/aedahamlibra
Q33. Anil calculated that it will take 45 minutes to
Time
Speed = = 70 km/h
cover a distance of 60 km by his car. How long will it
take to cover the same distance if the speed of his car
is reduced by 15 km/hr? Q37. A railway train 100 metres long is running at the
(a) 36 min (b) 55.38 min speed of 30 km/hr. In what time does it pass a man
(c) 48 min (d) 40 min standing near a line ?
Ans: (b) D = S × T (a) 10 seconds (b) 13 seconds
60 = ( ) (c) 12 seconds (d) 15 seconds

S= 80km/hr Ans: (c) Time taken by train = S = 12 seconds

Now, new speed = 80 – 15 = 65 km/hr.


∴Time Q38. A train 50 metre long passes a platform 100
metre long in 10 sec. The speed of the train in km/hr is
or (a) 10 (b) 54
Hence, Time to taken by car to travel same distance is (c) 15 (d) 100
55.38 min. Ans: (b) Total distance = (100 + 50) m = 150 m
Speed of the train = 54km/hr
Q34. By walking at of his usual speed, a man reaches
his oflice 20 minutes later than usual. His usual time is Q39. A student goes to school at the rate of km/hr
(a) 30 min. (b) 75 min.
and reaches 6 minutes late. If he travels at the speed
(c) 90 min. (d) 60 min.
of 3 km/hr, he reaches 10 minutes earlier. The
Ans: (d) New speed = × usual speed distance of the school is
∴ New time = × usual time (a) 45 km (b) 20 km
(c) 10 km (d) 4 km
∴ × usual time = 20 minutes
Ans: (d) Let original time taken by student be x hours.
Usual time = 3 × 20 = 60 minutes
( ) ( )
Q35. A man performs of the total journey by train, = 6x –1
by bus and the remaining 10 km on foot. His total x = hours
journey in km is ∴ distance of school = ( ) km
(a) 15.6 (b) 24
(c) 16.4 (d) 12.8
Q40. A horse take 2½ seconds to complete a round
Ans: (b) If the total journey be of x km, then
around a circular field. If the speed of the horse was
66 m/sec, then the radius of the field is, [Given
x– = 10 ]
(a) 25.62 m (b) 26.52 m
= 10 = 10
(c) 25.26 m (d) 26.25 m
x= = 24 km Ans: (d) Total distance covered by horse in seconds
=
Q36. Raju has to cover a distanc e of 240 km in 4
hours. If he covers one -third of the journey in th Radius of the field =

time, what is his speed at the beginning of the journey


? Q41. A is twice as fast as B and B is thrice as fast as C
(a) 70 km/hr (b) 75 km/hr is. The journey covered by C in 1½ hours will be
(c) 60 km/hr (d) 65 km/hr covered by A in
(a) 15 minutes (b) 20 minutes
Ans: (a) Distance = km = 80 km
(c) 30 minutes (d) 1 hour
Page 113 of 516
Ans: (a) Let C‘s speed = x km/h
Then, B‘s speed = 3x km/h
https://telegram.me/aedahamlibra
Ans: (a) Total distance = Length of train + Length of
bridge
and A‘s speed = 6x km/h = 120 + 360 = 480 m
Ratio of speeds of A, B, C = 6x : 3x : x = 6 : 3 : 1 Speed = 36 km/h = 10m/sec
Ratio of time taken
So time =
=1:2:6
It C‘s 90 minutes
Q45. A bus moving at 40km per hour covers a
Hence, 6x = 90
distance in 6 hours 15 minutes. If it travels the same
x = 15 minutes
distance at 50 km per hour how long will it take to
Hence, A should take 15 minutes.
cover the distance?
(a) 6 hours (b) 2 hours
Q42. If a train runs at 70 km/hour, it reaches its
(c) 4 hours (d) 5 hours
destination late by 12 minutes. But if it runs at 80
km/hour, it is late by 3 minutes. The correct time to Ans: (d) Distance covered = Speed × Time =
cover the journey is km
(a) 58 minutes (b) 2 hours New speed = 50 km/hour
(c) 1 hour (d) 59 minutes ∴ Time taken to cover same distance =
Ans: (c) Let correct time to cover journey be t hours
70( ) = 80( ) Q46. The speed of a car is 54 km/hr. What is its speed
70t + 14 = 80t+ 4 in m/sec?
10t = 10 (a) 150 m/sec (b) 19.44 m/sec
t = 1 hour (c) 194.4 m/sec (d) 15 m/sec
Ans: (d) Speed of car in m/sec = 54 × = 15m/sec
Q43. It takes 8 hours for a 600 km journey, if 120 km
is done by train and the rest by car. It takes 20 Q47. A train 150 m long passes a km stone in 30
minutes more if 200 km is down by train and the rest seconds and another train of the same length
by car. The ratio of the speed of the train to that of travelling in opposite direction in 10 seconds. The
the car is speed of the second train is :
(a) 2 : 3 (b) 3 : 2 (a) 125 km/hr (b) 25 km/hr
(c) 3 : 4 (d) 4 : 3 (c) 90 km/hr (d) 75 km/hr
Ans: (c) Let T be the speed of train and C be the speed of
Ans: (c) Speed of first train = = 5m/sec
car.
Let the speed of second train be x m/sec
=8 ... (1)
Relative speed = (5 + x) m /sec
= ... (2) = 10
Subtracting (2) from (1) 50 + 10x = 300
x= = 25m/sec = 25 × = 90km/h
= C = 80
Q48. A farmer travelled a distance of 61 km in 9 hrs.
= – He travelled partly on foot at the rate of 4 km/hr and
= T = 60 partly on bicycle at the rate of 9 km/hr. The distance
Required ratio = 60 : 80 = 3 : 4 travelled on foot is
(a) 17 km (b) 16 km
Q44. How many seconds will a train 120 metres long (c) 15km (d) 14 km
running the rate of 36km/ hr take to cross a bridge of Ans: (b) Let the distance travelled as foot be x km.
360 metres in length? Then, distance travelled by bicycle = (61 –x) km
(a) 48 sec (b) 36 sec So,
(c) 40 sec (d) 46 sec 9x + 4 (61 –x) = 9 × 36
9x – 4x = 324 – 244
Page 114 of 516
5x = 80
x = 16 km
https://telegram.me/aedahamlibra
(c) 6 (d) 4
Ans: (d) Average speed when speed x and y are given =

Q49. A ship after sail ing 12km towards south from a


particulari place covered 5 km more towards east.
Then the straightway distance of the ship from that 32 x = 6.4 (x + 16)
place is x = 0.2 (x + 16)
(a) 18 km (b) 15 km 0.8x = 3.2
(c) 13 km (d) 11km x=4
Ans: (c) Given, AB = 12 km, BC = 5 km
Q53. A train 180 mt long is running at a speed of 90
km/hr. How long will it take to pass a post ?
(a) 8.2 secs (b) 7.8 secs
(c) 8 secs (d) 7.2 secs
Ans: (d) Speed = = 25 m/sec
Time = = 7.2 sec.

Q54. A train, 200 m long, is running at a speed of 54


Straight way distance of ship km/hr. The time in seconds that will be taken by train
to cross a 175 m long bridge is
AC = √ =√ =√ = 13 km
(a) 12.5 (b) 20
(c) 25 (d) 10
Q50. A train runs at an average speed of 75 km/hr. If
the distance to be covered is 1050 Kms. How long will Ans: (c) Time =
the train take to cover it ? Time = = = 25 sec
(a) 13 hrs (b) 12 hrs
(c) 14 hrs (d) 15 hrs
Ans: (c) Average speed = 75km/h Q55. A man covers a total distance of 100 km on
Distance = 1050 kms bicycle. For the first 2 hours, the speed was 20 km/hr
and for the rest of the journey, it came d own to 10
Time taken to cover the distance = = 14 hrs.
km/hr. The average speed will be
(a) km/hr (b) 13 km/hr
Q51. Walking at the rate of 4 kmph a man covers
certain distance in 2hrs 45 min. Running at a speed of (c) 15 1/8 km/hr (d) 20 Km/hr
16.5 kmph the man will cover the same distance in Ans: (a) Total Distance = 100 km
how many minutes ? distance travelled in first 2hr = 2 × 20 = 40
(a) 35 min. (b) 40 min. Remaining distance = 100 – 40 = 60 km
(c) 45 min. (d) 50 min. Time taken = = 6 hr
Ans: (b) When distance is contant, then speed is
Av speed = = = km/hr
inversely proportional
S1 : S2 = T2 : T1
4 : 16.5 = T2 : 165 Q56. A man travelled a distance 72 km in 12 hour. He
tavelled partly on foot at 5 km/hour and partly on
or
bicycle at 10 km/hour. The distance travelled foot is
T2 = (a) 46 km (b) 52 km
(c) 50 km (d) 48 km
Q52. A man goes to a place on bicycle at speed of 16 Ans: (d) Let distance travelled by foot = x km/hr
km/hr and comes back at lower speed . If the average Let distance travelled by bicycle = 72 – x km/hr
speed is 6.4 km/hr in total , then the return speed (in
km/hr) is
(a) 10 (b) 8 2x + 72 – x = 120
Page 115 of 516
x = 120 – 72 = 48 km https://telegram.me/aedahamlibra
Then,
upstream speed = (x – y) km/h
Q57. Points A and B are 100 km apart on a highway. and down stream speed = (x + y) km/h
One car starts from A and another from B at the same Now,
time. If the cars travel in the same direction, they ... (i)
meet in 5 hours. If the cars travel towards each other,
they meet in 1 hour. What is the speed of the faster ... (ii)
car? From Equation (i) and (ii)
(a) 70 km/hour (b) 40 km/hour x = 10 km/hr and y = 4 km/hr.
(c) 60 km/hour (d) 80 km/hour
Ans: (c) Let the speed of car be x and other be y Q60. A bus starts running with the initial speed of 21
Distance covered from A in 5 hrs = 5x km/hr and its speed increases every hour by 3 km/hr.
Distance covered from B in 5 hrs = 5y How many hours will it take to cover a distance of 252
ATQ km?
when they travel in same direction (a) 3 (b5
then 5x – 5y = 100 (c) 8 (d) 10
x – y = 20... (i) Ans: (c) Since speed of bus increases every hour by 3
When they travel towards each other km/hr.
then ATQ
x + y = 100... (ii) Initial speed = 21 km/hr
Now, adding eqn (i) , & (ii) Total distance = 252 km
2x = 120 According to Arithmatic Progression
x = 60 km/hr a = 21, d = 3, sn = 252 n = ?
[ ]
Q58. A train travels 40% faster than a car. Both start
from point A at the same time and reach point B, 140
km away at the same time. On the way the train takes
25 minutes for stopping at the stations. What is the
3n2 + 39n – 504 = 0
speed (in km/hr) of the train?
n2 + 21n – 8n – 168 = 0
(a) 67 (b) 134.4
n (n + 21) – 8 (n + 21) = 0
(c) 145.9 (d) 160
∴ (n – 8) (n + 21)
Ans: (b) Let speed of car = x
n = 8, n ≠ – 21
∴Speed of train ∴So, 8 hours will it take to cover a distance of 252 km.
According to question,
Q61. A train leaves Del hi at 10 a.m. and reaches
Jaipur at 4 p.m. on same day. Another train leaves
Jaipur at 12 p.m. and reaches Delhi at 5 p.m. on same
day. What is the time of day (approximately) when
= 96 km/hr the two trains will meet?
∴Speed of train = 134.4 km/hr (a) 1 : 42 p.m. (b) 1 : 27 p.m.
(c) 2 : 04 p.m. (d) 1 : 49 p.m.
Ans: (d) Suppose distance between Delhi and Jaipur is 30
Q59. A boat goes 15 km upstream and km kms.
downstream in 3 hours 15 minutes. It goes 12 km Then,
upstream and 14 km downstream in 3 hours. What is Speed of first train 5km/hr
the speed of the boat in still water?
(a) 4 (b) 6 Speed of second train = 6km/hr
(c) 10 (d) 14 If trains met after t hours from 10 a.m. then
Ans: (c) Let speed of the boat in still water be x km/h and 5t + 6 × (t – 2) = 30
speed of current be y km/h. 5t + 6t – 12 = 30

Page 116 of 516


11t = 42
hours = 3hours 49minutes
https://telegram.me/aedahamlibra
x2 + 12x = 2268
∴Trains meet 3 hours 49 minutes after 10 a.m. i.e. at 1 : x2 + 12x – 2268 = 0
49 pm. x2 + 54x – 42x – 2268 = 0
x (x + 54) – 42 (x + 54)
Q62. If a train maintains an average speed of 42 km (x – 42) (x + 54)
per hour, it arrives at its destination at the right time, ∴x – 42 = 0∴x = 42
if however, the average speed is 40 km per hour, it x + 54 = 0∴x = –54
arrives 15 minutes late. Find the length of the ∴Usual speed of motorcyclist = 42 km/hr.
journey?
(a) 210 km (b) 205 km Q65. A train covers a distance of 12 km in 10 minutes.
(c) 209 km (d) 200 km If it takes 6 seco nds to pass a telegraph post, then the
Ans: (a) Let Length of Journey = x km. length of the train is:
According to question, (a) 120 m (b) 90 m
(c) 100 m (d) 140 m
Ans: (a) Speed of train km/hr

20m/sec
km
∴Length of train = speed × time
∴Length of Journey is = 210 km = 20 × 6 = 120 m.

Q63. Two cars travel from city A to city B at a speed


of 36 km/hr and 48 km/hr respectively. If one car Mensuration
takes 3 hours lesser time than the other car for the
journey, then the distance between City A and City B
Q1. An equilateral triangle of side 6 cm has its corners
is 2
cut off to form a regular hexagon. Area (in cm ) of
(a) 518 km (b) 432 km
this regular hexagon will be
(c) 648 km (d) 346 km
Ans: (b) According to question, (a) √ √

Speed of cars = 36 km/hr and 48 km/hr (c) √
So, Ans: (c)
36 × t = 48 (t – 3)
36t = 48t – 48 × 3
12t = 48 × 3
hours
∴Distance = 12 × 36 = 432 km.

Q64. A motorcylist left minutes later than the


scheduled time but in order to reach its destination
Side of the regular hexagon
21km away in time, he had to increase his speed by 12
km/hr from the usual speed. What is u sual speed (in cm
km/hr) of the motorcyclist? √
Area of the hexagon
(a) 28 (b) 35
(c) 42 (d) 64 √
Ans: (c) Let usual speed of the motorcyclist = x
According to question, √ sq.cm.
hour
Q2. If the circumference of a circle is decreased by
50% then the percentage of decrease in its area is
(a) 25 (b) 50
Page 117 of 516
(c) 60 https://telegram.me/aedahamlibra
(d) 75
Ans: (d) Circumference = 2πr (one variable)
(a) 4 : 3
(c) 2 : 3
(b) 3 : 4
(d) 3 : 2
∴ The decrease in area = – 50 – 50 + Ans: (b) Ratio of the circumferences
= Ratio of radii = 3 : 4
= –75%

Q3. The length (in metres) of the longest rod that can
be put in a room of dimensions 10 m × 10 m × 5 m is
(a) √ (b) 15
=
(c) √ √
Ans: (b) Length of the longest rod
Q8. The volume (in m 3) of rain water that can be

collected from 1.5 hectares of ground in a rainfall of 5
√ cm is
√ metre (a) 75 (b) 750
(c) 7500 (d) 75000
Q4. A copper wire of length 36 m and diameter 2 mm Ans: (b) 1 hectare = 10000 sq. metre
is melted to form a sphere. The radius of the sphere ∴ Area of the ground = 15000 sq. metre
(in cm) is ∴ Required volume = 15000 ×
(a) 2.5 (b) 3
= 750 m3
(c) 3.5 (d) 4
Ans: (b) Volume of the wire = πr2h
Q9. A copper wire is bent in the shape of a square of
∴ π × 0.1 × 0.1 × 3600 cm3
area 81 cm 2. If the same wire is bent form of a
36π cm3
semicircle, the radius (in cm) of the semicircle is
Volume cylinder = vol. sphere
(Take )
Volume of the sphere =
(a) 16 (b) 14
= 36 π
(c) 10 (d) 7
R3 = = 27 Ans: (d) Side of a square
∴R= √ = 3 cm = √ = 9 cm
∴ Length of the wire
Q5. If each side of a square is increased by 10%, its = 4 × 9 = 36 cm.
area will be increased by ∴ Perimeter of semi-circle = (π + 2)r
(a) 10% (b) 21% where r = radius
(c) 44% (d) 100% ( )
Ans: (b) Increase percent in area
=( )% = 21%
r= = 7 cm.
Q6. If the length of a rectangle is increased by 10%
and its breadth is decreased by 10%, the change in its Q10. A bicycle wheel makes 5000 revolutions in
area will be moving 11 km. Then the radius of the wheel (in cm) is
(a) 1% increase (b) 1% decrease (Take )
(c) 10% increase (d) No change
(a) 70 (b) 35
Ans: (b) Required change in area (c) 17.5 (d) 140
( ) Ans: (b) Distance covered by wheel in one revolution
= Circumference of wheel
Negative sign shows a decrease.
= m
Shortcut:
= cm = 220 cm
Q7. The ratio of the radil of two wheels is 3 : 4. The ∴ 2πr = 220
ratio of their circumferences is
Page 118 of 516
2× = 220 https://telegram.me/aedahamlibra
Q14. Water is flowing at the rate of 5 km/h through a
pipe of diameter 14cm into a rectangular tank which
r= = 35 cm
is 50 m long, 44m wide. The time taken, in hours, for
the rise in the level of water in the tank to be 7 cm is
Q11. A river 3 m deep and 40 m wide is flowing at the (a) 2 (b)
rate of 2 km per hour. How much water (in litres) will
fall into the sea in a minute? (c) 3 (d)
(a) 4,00,000 (b) 40,00,000 Ans: (a) Water flowed by the pipe in 1 hr.
(c) 40,000 (d) 4,000 = πr2h
Ans: (b) Volume of water flowed in an hour = metre3
= 2000 × 40 × 3 m3 3
= 77 m
= 240000 m3
Volume of expected water in the tank
∴ Volume of water flowed in 1 minute.
= 154 m3
= = 4000 m3
∴ Required time
= 4000000 litre
= = 2 hr.
Q12. A copper wire is bent in the form of square with
an area of 12 1 cm 2. It the same wire is bent in the Q15. The areas of three consecutive faces of a cuboid
form of a circle, the radius (in cum) of the circle is are 12 cm 2, 20 cm 2 and 15 cm 2, then the volume (in
cm3) of the cuboid is
(Take π = )
(a) 3600 (b) 100
(a) 7 (b) 10
(c) 80 (d) 60
(c) 11 (d) 14
Ans: (d) If the length, breadth and height of the cuboid
Ans: (a) Side of square = √ = 11 cm be x, y and z cm respectively, then
∴ Length of wire = 4 × 11 = 44 cm xy = 12; yz = 20; zx = 15
∴ 2πr = 44 ∴ x2y2z2 = 12 × 20 × 15
2× = 44 = 3600 cm6
ν= = 7 cm ∴ ν = xyz = √ = 60 cm3

Q16. The sides of a triangles are in the ratio 2:3:4. the


Q13. The perimeter of a triangle is 40cm and its area
perimeter of the triangle is 18cm. The area (in cm 2) of
is 60 cm 2. If the largest side measures 17cm, then the
the triangle is
length (in cm) of the smallest side of the triangle is
(a) 9 (b) 36
(a) 4 (b) 6
(c) 8 (d) 15 (c) √ √
Ans: (c) Smallest side of the triangle = x cm (let) Ans: (d) Ratio = 2 : 3 : 4
∴ Second side of triangle =4:6:8
= 40 – 17 – x = 23 – x Perimeter = 18 cm

Semi-perimeter, = s = = 20 ∴ Semi-perimeter (s) = =9


∴ Area of triangle
∴√ = 60
=√
√ = 60
(20 – x) (x – 3) = 60 =√
20x – 60 – x2 + 3x = 60 =√ √ sq. cm.
x2 – 23x + 120 = 0
x2 – 15x – 8x + 120 = 0 Q17. The wheel of a motor car makes 1000
x (x – 15) – 8 (x – 15) = 0 revolutions in moving 440 m. The diamete r (in metre)
(x – 8) (x – 15) = 0 of the wheel is
x = 8 or 15 (a) 0.44 (b) 0.14
(c) 0.24 (d) 0.34
Ans: (b) Distance covered by wheel in one revolution

Page 119 of 516


= Circumference of wheel
∴ π × diameter =
https://telegram.me/aedahamlibra
Q20. Volume of a right circular cone is numerically
equal to its slant surface area. Then value of ( ),
× diameter = where h and r are height and radius of the cone
respectively, is
Diameter =
(a) 9 units (b) unit
= 0.14 cm
(c) 4 units (d) unit
Q18. ABCDEF is a regular hexagon of side 2 feet. The Ans: (b) According to condition given
area, in square feet of the rectangle BCEF is Volume of right circular cone = Slant surface area
(a) 4 (b) √ [where, r → radius; h → height;
(c) 8 (d) 4 + √ l → slant height]
Ans: (b)

√ [ ]
Squaring on both sides

Dividing equation by r2h2 on both sides

units
Given BC & EF are each 2 feet. Since area of rectangle is
length × width. Q21. The area of a semi -circular field is 308 sq. m;
To find out BF or CE, Take ∆ABF. It has two equal sides then takingπ = , the length of the railing to surround
(AB = AF), so the perpendicular from A to line BF it has to be
divides ABF into two congruent ∆s. (a) 44 m (b) 72 m
So, each of the two triangles is 30° -60°-90° right angle ∆ (c) 88 m (d) 80 m
with hypotenuse 2. Ans: (b)

In ∆ABM cos 30° = √
So, BF = 2 × BM = √
Area of rectangle = √ × 2 = √

Q19. The base of a triangle is 2 cm more than twice its Length of railing to surround
altitude. If the area is 12 sq. cm, its altitude will be = Length of Are + Length of diameter
(a) 6 cm (b) 5 cm Area of semicircular = 308
(c) 4 cm (d) 3 cm
308 =
Ans: (d) Base = 2 + 2 × altitude
Let, altitude be A 308 = × r2
Area of ∆ = × Base × Altitude
12 = × (2 + 2A) × A r = 14 m
12 = A × (1 + A) Length of railing = πr + 2r
12 = A + A2 = = 44 + 28 = 72 m
A2 +A – 12 = 0
(A – 3) (A + 4) = 0
Q22. The volume of a cubical box is 3.375 cubic
A = 3, A = – 4
meters. The length of edge of the box is:
Altitude = 3 cm
(a) 75 cm (b) 1.5 m
(c) 1.125 m (d) 2.5 m
Page 120 of 516
Length of edge of the box = √
https://telegram.me/aedahamlibra
Ans: (b) Volume of cubical box = 3.375 m3 (a) 2 : 1
(c) 1 : 1
(b) 1 : 6
(d) 1 : 2
Ans: (c) Diagonal of a cube = √
Q23. If the numerical value of the volume of a right √ × side = √
circular cylinder and its curved surface area are ∴ Side of a cube = 6
equal, then its radius is Surface area of cube = 6 × (side)2 = 6 × 62
(a) 2 units (b) 4 units Volume of cube = (side)3 = (6) 3
(c) 3 units (d) 6 units
Required ratio = or1:1
Ans: (a) Volume of right circular cylinder = Curved
surface area of cylinder
πr2h = 2πrh [where, r → radius; h → height] : Q28. The capacities of two hemispherical vessels are
r = 2 units 6.4 litres and 21.6 litres. The ratio of their inner radii
is
Q24. The circumference o f the base of a 16 cm height (a) 4 : 9 (b) 16 : 81
solid cone is 33 cm. What is the volume of the cone in (c) √ √ (d) 2 : 3
cm³ ? Ans: (d)
(a) 1028 (b) 616
(c) 462 (d) 828
( ) ( )
Ans: (c) Circumference = 33 cm
2πr = 33
∴r =
Volume = =
Q29. The perimeter of a semi -circular area is 18 cm,
then the radius is : (using )
Q25. The length of a minute hand of a clock is 7 cm.
The area swept by the minute hand in 30 minutes is: (a) cm (b) cm
(a) 210 sq. cm (b) 154 sq. cm (c) 6 cm (d) 4 cm
(c) 77 sq. cm (d) 147 sq. cm Ans: (b) Perimeter of a semicircular area = 18 cm
Ans: (c) Angle made by clock in 30 minutes = 180° perimeter of semicircular = 18 cm
∴ Area of sector covered by minute hand =
sq.cm r (π + 2) = 18

Q26. The ratio of the edges of rectangular


parallelopiped is 1 : 2 : 3 and its volume is 1296 cubic
cm. The area of the whole surface in sq. cm is :
(a) 696 (b) 792
(c) 824 (d) 548
Ans: (b) Volume of rectangular parallelopiped = 1296 cm
Ratio of edges = 1 : 2 : 3
∴ x, 2x and 3x are length, breadth and height of Q30. A rectangular garden is 100 m × 80 m. There is a
parallelopiped respectively. path along the garden and just outside it. Width of
x × 2x × 3x = 1296 path is 10 m. The area of the path is
6x3 = 1296 x3 = 216 (a) 1900 sq m (b) 2400 sq m
x= √ (c) 3660 sq m (d) 4000 sq m
Length = 6, Breadth = 12, Height = 18 Ans: (d)
Required surface area = 2 (lb + bh + hl)
= 2 (6 × 12 + 12 × 18 + 18 × 6) = 792 sq.cm

Q27. Diagonal of a cube is √ cm. Ratio of its total


surface area and volume (numerically) is:
Page 121 of 516
https://telegram.me/aedahamlibra
(c) 11480 (d) 10080
Ans: (a) Curved surface area of cylinder = 2πrh
sq.cm

Q34. The perimeter of the base of a right circular


cone is 8 cm. If the height of the cone is 21 cm, then its
volume is :
(a) (b) 108 π cm3
Area of the shaded region
= (100 + 2 × 10) (80 + 2 × 10) – 100 × 80 (c) (d) 112 π cm3
= 120 × 100 – 8000 Ans: (c) 2πr = 8 πr = 4
= 4000 sq. metre r=
∴V= = cu.cm.
Q31. The area of the largest triangle that can be
inscribed in a semicircle of radius x in square units is:
SSC CGL 2nd Sit. 2012) Q35. The ratio of inradius and circumradius of a
(a) 4x2 (b) x2 square is :
(c)
2x 2
(d) 3x2 (a) 1 : 2 (b) 1 : √
Ans: (b) (c) √ : √ (d) 1 : 3
Ans: (b)

radius
Area of the largest triangle Radius of circum-circle
=x 2 √
=

Radius of in-circle =
Q32. The length and breadth of a square are
increased by 30% and 20% respectively. The area of ∴ Rate = : = 1:√

the rectangle so formed exceeds the area of the square
by: Q36. A circle is inscribed in an equilateral triangle
(a) 46% (b) 66% and a square is inscribed in that circle. The ratio of
(c) 42% (d) 56% the areas of the triangle and the square is
Ans: (d) Let the side of square = ‗x‘ (a) √ √
Area of square = x2
(c) √ √
New length of rectangle = Ans: (d)
New Breadth of rectangle =
Hence, Area of so formed rectangle =

Therefore, area of rectangle exceeds the area of square by


56%

Q33. A metal pipe of negligible thickness has radius In the given figure ABC is an equilateral ∆ of a side with
21 cm and length 90 cm. The outer curved surface a circle inscribed in it and a square inscribed in the circle.
area of the pipe in square cm is
(a) 11880 (b) 11680
Page 122 of 516
∠C and O is the centre of the circle.
https://telegram.me/aedahamlibra
AD, BO and CO are the angle bisectors of ∠A, ∠B and Ans: (c) Radius of cylinder = r units and height = r units
∴ Required ratio = 2πr2 + 2πr2 = 4πr2
We know that the angle bisector from the vertex of an : 2πr2 + πr2 = 3πr2 = 4 : 3
equilateral triangle is the perpendicular bisector of the
opposite side. Q39. If the sum of the length, breadth and height of a
AD is the perpendicular bisector of BC. rectangular parallelopiped is 24 cm and the length of
BD = and ∠DOB = ∠ its diagonal is 15 cm, then its total surface area is
(a) 351 cm2 (b) 256 cm2
Now in ∆BOD 2
(c) 265 cm (d) 315 cm2
tan 30° = Ans: (a) Let length = l, breadth = b, height = h.
l + b + h = 24 (given) ... (i)
Radius of circle =
√ √ Diagonal of parallellopiped = 15 cm
Now in right ∆EDG
√ = 15 or l2 + b2 + h2 = 225
EG2 + GD2 = ED2 (Pythagoras theorem)
Squaring eqn. (i) on both sides
2 (EG)2 = 2 (OD)2 = ( ) l2 + b2 + h2 + 2 lb + 2bh + 2hl = 576

2 (lb + bh + hl) = 576 – 225 = 351
Side of the square = √ [

Now ar (∆ABC) : ar (∆EFG)
√ √ Surface area of parallellopiped = 2 (lb + bh + hl)]
= √
√ √
Q40. A copper sphere of diameter 18 cm is drawn into
a wire of diameter 4 mm. The length of the wire, in
Q37. A circular road runs around a circular ground.
metre, is :
If the difference between the circumferences of the
(a) 2.43 (b) 243
outer circle and the inner circle is 66 metres, the
(c) 2430 (d) 24.3
width of the road is : (Take )
Ans: (b) Volume of sphere = =
(a) 21 metres (b) 10.5 metres
= 972 π cubic.cm.
(c) 7 metres (d) 5.25 metres
If the length of wire be h cm., then
Ans: (b)
π × (0.2)2 × h = 972 π
h= = 24300 cm = 243 metre

Q41. If area of an equilateral triangle is a and height


b, then value of is:
(a) 3 (b)
(c) √

Ans: (c) Let side of triangle = x

∴ = a... (i)
Breadth of road = r2 – r1
C2 – C1 = 66 √
and =b
∴ 2πr2 = 2πr1 = 66
x= ... (ii)
2π (r2 – r1) = 66 √
Putting x in equation (i)
r2 – r1 = = 10.5 metre
√ √
( )

Q38. A solid right circular cylinder and a solid √
hemisphere stand on equal bases and have the same

height. The ratio of their whole surface areas is:
(a) 3: 2 (b) 3: 4 Q42. Three ci rcles of equal radius 'a' cm touch each
(c) 4: 3 (d) 2: 3 other. The area of the shaded region is :
Page 123 of 516
https://telegram.me/aedahamlibra
(c) 64 : 25 (d) 16: 25
Ans: (c)

Q45. A godown is 15 m long and 12 m broad. The sum


of the areas of the floor and the ceiling is equal to the
√ √ sum of areas of the four walls. The volume (in m 3) of
(a) ( ) ( )
the godown is:

(c) (√ ) ( ) (a) 900 (b) 1200
Ans: (d) AB = BC = CA = 2a cm. (c) 1800 (d) 720
∠BAC = ∠ACB = ∠ABC = 60° Ans: (b) If the height of the godown be h meter, then
√ 2 (15 × 12) = 2 × h (15 + 12)
Area of ∆ABC = 27h = 15 × 12

= =√ sq.cm. h= meter
Area of three sectors ∴ Volume of the godown
= = 1200 cu.meter

Q46. The volumes of a cylinder and a cone are in the


ratio 3 : 1. Find their diameters and then compare
them when their heights are equal.
(a) Diameter o f cylinder < Diameter of cone
(b) Diameter of cylinder = 2 times of diameter of cone
(c) Diameter of cylinder = Diameter of cone
= (d) Diameter of cylinder > Diameter of cone
Ans: (c) Volume of cylinder = 3 × volume of cone
sqcm.
(heights are equal)
Area of the shaded region

r1 = r2
=√ =( ) sq.cm. d1 = d2

Q43. Water f lows at the rate of 10 metres per minute Q47. The base of a right pyramid is an equilateral
from a cylindrical pipe 5 mm in diameter. How long it triangle of side 10 √ cm. If the total surface area of
lake to fill up a conical vessel whose diameter at the the pyramid is 270 √ sq. cm, its height is
base is 30 cm and depth 24 cm?
(a) 12 cm (b) √ cm
(a) 28 minutes 48 seconds (b) 51 minutes 12 seconds
(c) 10 cm (d) √ cm
(c) 51 minutes 24 seconds (d) 28 minutes 36 seconds
Ans: (a) Now, T.S.A of pyramid = ar (∆ABD) + ar
Ans: (a) Volume of water flowing from the pipe in 1
(∆ADC) + ar (∆ABC) + ar (∆BDC)
minute
Volume of conical vessel ∴T.S.A of pyramid =
= cu.cm. √

∴ Required time =
= 28 minutes 48 seconds

Q44. If the volumes of two right cir -cular cones are in


the ratio 4 : 1 and their diameters are in t he ratio 5 : 4
then the ratio of their heights is :
(a) 25: 16 (b) 25: 64
Page 124 of 516
https://telegram.me/aedahamlibra

= 18 cm, b = 12 cm, h = 3 cm
S = 2 (h + bh) + b{Box is open from upper side}
(∴ AE = AF = AG = height of isosceles ∆ (h) ) = 2 (54 + 36) + 216
[ ] √ = 396 cm2

√ [ √ √ √ ] Q50. The volume of air in a room is 204 m 3
. The
√ height of the room is 6 m. What is the floor area of the
( √ )
room?
√ √ √ (a) 34 m2 (b) 32 m2
√ √ (c) 46 m 2
(d) 44 m2
H = 13m... (1) Ans: (a) Volume of air in room = 204 m3
Now to find height of pyramid (H) , we use Area of floor × height of room = 204 m3
Area of floor ×6 = 204 m3
∴Area of floor = = 34 m2

Q51. What is th e height of a cylinder that has the


same volume and radius as a sphere of diameter 12
cm?
(a) 8 cm (b) 7 cm
In ∆ODE, tan30° =
√ (c) 10 cm (d) 9 cm
... (2) Ans: (a) Volume of cylinder = volume of sphere (Given)
√ √
From (1) & (2) , we use pythagoral theorem, in ∆ AEO
(AE)2 = (EO)2 + (AO)2 or h2 = (OE)2 + H2
(13)2 – (5) 2 = H2 144 H = 12cm h=
h= = 8 cm
Q48. The sides of a triangle are 16 cm, 12 cm and 20
cm. Find the area Q52. The l ength and breadth of a rectangle are
(a) 81 cm2 (b) 64 cm2 doubled. Percentage increase in area is
2
(c) 112 cm (d) 96 cm2 (a) 400% (b) 150%
Ans: (d) (16) + (12) = 400 = (20)2
2 2
(c) 200% (d) 300%
A= = 96 cm2 Ans: (d) A = b
A' = (2) (2b) = 4b = 4A
Q49. A square of side 3 cm is cut off from each corner % Change =
of a rectangular sheet of length 24 cm and breadth 18
cm and the remaining sheet is folded to form an open Q53. If the total surface area of a cube is 96 cm 2
, its
rectangular box. The surface area of the box is volume is
(a) 423 cm2 (b) 468 cm2 (a) 36 cm3 (b) 56 cm3
2
(c) 396 cm (d) 612 cm2 (c) 16 cm 3
(d) 64 cm3
Ans: (c) Ans: (d) Total surface area of cube = 96 cm2
6a2 = 96 cm2
a2 = 16 cm2 cm
Now, volume of cube = a3 (4) 3 = 64 cm3
Page 125 of 516
https://telegram.me/aedahamlibra
Q54. A square ABCD is inscribed in a circle of unit
radius. Semicircles are described o n each side as a
diameter. The area of the region bounded by the four
semicircles and the circle is
(a) 1 sq. unit (b) 2 sq. unit
(c) 1.5 sq. unit (d) 2.5 sq. unit
Ans: (b)

θ = 72°

180° = π radius
∴ 72° =

BD = 2 units = radians
AB = √ units Arc AB = s = 88 metre
Area of square = 2 square units ∴θ=
Area of four semicircles

=4× = = π sq. units 2πr = 88 × 5


∴ Required area r= = = 70 metre
= 2 + π – π = 2 sq. units.
Q57. If the perimeters of a recta ngle and a square are
Q55. The base of a right prism is a triangle whose equal and the ratio of two adjacent sides of the
perimeter is 28 cm and the inradius of the triangle is 4 rectangle is 1 : 2 then the ratio of area of the rectangle
cm. If the volume of the prism is 366 cc, then its and that of the square is
height is (a) 1 : 1 (b) 1 : 2
(a) 4 cm (b) 8 cm (c) 2 : 3 (d) 8 : 9
(c) 6 cm (d) None of these Ans: (d) Sides of rectangle are 2x and x units.
Ans: (d) Area of base = Side of square = y units
∴ 4y = 6x
=
= ∴
[Where r = inradius, s = semi-perimeter]
=8:9
volume of prism = area of base × height
366 = 56 × h
Q58. The total surface area of a sphere is 8 π square
h = 6.5 cm [approx]
unit. The volume of the sphere is

Q56. A horse is tied to a post by a rope. If the horse (a) √ cubic unit (b) cubic unit
moves along a circular path always keeping the rope √
(c) cubic unit (d) cubic unit
stretched and describes 88 metres when it has traced
out 72° at the centre the length of the rope is (Take Ans: (c) Surface area of sphere = 4πr2
r2 = 2 r √ units
)
∴ Volume of sphere
(a) 70 m (b) 75 m
(c) 80 m (d) 65 m (√ )
Ans: (a) √
cubic units

Page 126 of 516


https://telegram.me/aedahamlibra
Q59. The diameters of two circles are the side of a
square and the diagonal of the square. The ratio of the
Q62. If the diagonal of a square is doubled, then its
area will be
areas of the smaller circle and the larger circle is (a) three times (b) four times
(a) √ √ (b) 1:√ (c) same (d) none of these
(c) 1 : 2 (d) 1 : 4 Ans: (b) Diagonal of a square (d) = √ × side of square
Ans: (c) Side of square = x units (a) .
Diagonal of square = √ units d=√

Radius of smaller circle = units
Area of square a2 =
Radius of larger circle
Now, diagonal gests doubled

units

∴ Required ratio of areas √
=2:4=1:2 Area of square ( ) ( )

is area of square
Q60. A square is inscribed in a circle of radius 8 cm.
The area of the square is Therefore, area will he four times.
(a) 16 square cm (b) 64 square cm
(c) 128 square cm (d) 148 square cm Q63. The biggest possible circle is inscribed in a
Ans: (c) rectangle of length 16 cm and breadth 6 cm. Then its
area is
(a) 3π cm2 (b) 4π cm2
2
(c) 5π cm (d) 9π cm2
Ans: (d) Radius of square = breadth of rectangle
cm
Alternate Method:
The area of circle is 9π cm2.

Diagonal of square = Diameter of circle Q64. The perimeter of the base of a right circular
√ side of square = 16 cm cylinder is 'a' unit. if the volume of the cylinder is V
Squaring on both sides cubic unit. then the height of the cylinder is
(√ ) (a) unit (b) unit
2
(side of square) =
(c) unit (d) unit
Area of square = 128 sq. cm Ans: (d) If the radius of base of cylinder be r units and its
height be h units, then
Q61. A conical flask is full of wate r. The flask has
2π r = a r = units
base radius r and height h. This water is poured into a
cylindrical flask of base radius mr. The height of ∴ Volume of cylinder = πr2h
water in the cylindrical flask is V=π× h= units
(a)
Q65. The difference of perimeter and diameter of a
(c)
circle is X unit. The diameter of the circle is
Ans: (b) Volume of water in conical flask
(a) unit (b) unit
If the height of water level in cylindrical flask be
H units, then (c) unit (d) ( )unit
Ans: (a) If the diameter of the circle be d units, then
πd–d=x
d (π –1) = x
d= units

Page 127 of 516


https://telegram.me/aedahamlibra
Q66. The length and breadth of a rectangle are 20 m
Q70. A wooden box of d imension 8 metre × 7 metre ×
6 metre is to carry rectangular boxes of dimensions 8
and 15 m respectively. If length is increased by 20% cm × 7 cm × 6 cm. The maximum number of boxes
and the breadth by 30%, the percentage increase in that can be carried in 1 wooden box is
its area is (a) 7500000 (b) 9800000
(a) 54% (b) 56% (c) 1200000 (d) 1000000
(c) 50% (d) 52% Ans: (d) Maximum number of boxes =
Ans: (b) Area of rectangle, A = 20 m × 15 m = 300m2.
= 1000000
increased area, ( )( )
= 24 × 19.5 = 468 m2. Q71. The radius of a right circular cone is 3 cm and
% increase in area = = 56% its height is 4 cm. The total surface area of the cone is
(a) 48.4 sq.cm (b) 64.4 sq.cm
(c) 96.4 sq.cm (d) 75.4 sq.cm
Q67. A sphere of diameter 6 cm is dropped in a right
Ans: (d) Total surface are of cone = πr (l + r)
circular cycindrical ve ssel partly filled with water.
The diameter of the Cylindrical vessel is 12 cm. If the S= (√ )
sphere is just completely submerged in water, then the =
rise of water level in the cylindrical vessel is
S = 75.4 sq. cm
(a) 2 cm (b) 1 cm
(c) 3 cm (d) 4 cm
Q72. A rectangular piece of paper of dimensions 22
Ans: (b) Volume of sphere
cm by 12 cm is rolled along its length to form a
= = cylinder. The volume (in cm 3
) of the cylinder so
= 36 π cu. cm. formed is (use )
If the water level rises by h cm, then
(a) 562 (b) 412
πR2h = 36π
(c) 462 (d) 362
6 × 6 × h = 36
Ans: (c) 2πr = 22 cm
h = 1 cm
r= cm
Q68. A spherical ball of lead of radius 14 cm is melted Height, h = 12 cm
and recast into spheres of radius 2 cm. The number of Volume of cylinder =
the small spheres is
(a) 300 (b) 525 Q73. Two circular cylinders of equal volume have
(c) 343 (d) 450 their heights in the ratio 1 : 2; Ratio of their ra dii is
Ans: (c) Numebr of small spheres,
(Take )
(a) 1 : 4 (b) 1:√
(c) √ (d) 1 : 2
Ans: (c) =

=√ √

Q69. Length of each equal side of an isosceles triangle r1 : r 2 = √


is 10 cm and the included angle between those two
sides is 45°. Find the area of the triangle. Q74. Area of a regular hexagon with side 'a' is
(a) √ √ (a)

sq. unit (b) sq. unit

(c) √ √
(c) sq. unit (d) sq. unit
Ans: (a) Area of triangle √ √
√ √
Ans: (c) Area of hexagon = or

√ √

Page 128 of 516


https://telegram.me/aedahamlibra
Q75. A sphere is placed inside a right circular
cylinder so as to touch the top, base a nd the lateral
Q79. The area of a square park is 25 sq. km. The time
taken to complete a round of the field once, at a speed
surface of the cylinder. If the radius of the sphere is R, of 3 km/hour is
the volume of the cylinder is (a) 4 hours 60 minutes (b) 4 hours 50 minutes
(a) 2πR3 (b) 4πr3 (c) 6 hours 40 minutes (d) 5 hours 40 minutes
(c) 8πR3 (d) Ans: (c) Side of square park = √ km = 5 km
Ans: (a) Radius of cylinder = Radius of sphere = R Perimeter of park = 4 × 5 = 20 km
Height of cylinder = 2R Time taken = = 6 hours 40 minutes
Volume of cylinder =
Q80. There are two cones. The curved surface area of
Q76. A flask in the shape of a right circular cone of one is twice that of the other. The slant height of the
height 24 cm is filled with water. The water is poured latter is twice that of the former. The ratio of their
in a right circular cylindrical flask whose radius is radii is
of the radius of the base of the circular cone. (a) 4 : 1 (b) 4 : 3
Then the height of the water in the cylindrical flask is (c) 3 : 4 (d) 1 : 4
(a) 32 cm (b) 24 cm Ans: (a) C1 = 2C2
(c) 48 cm (d) 72 cm πr1l1 = 2πr2l2
Ans: (d) Let radius of base of cone be r and height of also, l2 = 2l1
cylinder be h. πr1l1 = 2 × 26 πr2l1
Vol. of cone = Vol. of cylinder

( )
h = 72 cm Q81. The base of a right pyramid is an equilateral
triangle of side 4 cm each. Each slant edge is 5 cm
Q77. If the sum of the dimensions of a rectangular long. The volume of the pyramid is
parallelepiped is 24 cm and the length of the diagonal √ √
(a) cm3 (b) cm3
is 15 cm, then the total surface area of it is
(c) √ √
(a) 420 cm2 (b) 275 cm2 cm3 (d) cm3
2
(c) 351 cm (d) 378 cm2 √
Ans: (c) Height of base = where a = 4
Ans: (c) Let length, breadth and height of parallelopiped

be l, b and h respectively. = = √
l + b + h = 24 cm √
Ao = × × 4 = cm
√ l2 + b2 + h2 = 225 cm2 √
2
(l + b + h) – 2 (lb + hb + lh) = 225
(24)2 – 225 = 2 (lb + bh + hl)
351 = 2 (lb + bh + hl)
Total surface area is 351 cm2.

Q78. The external fencing of a circular path around a


circular plot of land is 33 m more than its interior Slant edge BD = 5cm
fencing. The width of the path around the plot is Verticle height Do2 = (5) 2 –( ) = 25 –

(a) 5.52 m (b) 5.25 m

(c) 2.55 m (d) 2.25 m = Do

Ans: (b) Let radius of internal and external circular Volume of Pyramid = ar of base × height
Plot be r and R respectively.
√ √ √
2πR – 2πr = 33 m = × ×4×4×

=

Width of path, (R – r) =
Q82. A vessel full of pure acid contains 10 litres of it,
of which 2 litres are withdrawn. The vessel is then
filled with water. Next 2 litres o f the mixture are
Page 129 of 516
https://telegram.me/aedahamlibra
withdrawn, and again the vessel is filled up with
water. The ratio of the acid left in the vessel with that
of the original quantity is
(a) 1 : 5 (b) 4 : 5
(c) 4 : 25 (d) 16 : 25
Ans: (d) Quantity of acid left = 10( ) =
Required ratio = = 16 : 25 Regular equilateral triangular pyramid
Total surface area of pyramid
= 4 × Area of Equilateral triangle
Q83. A wire is bent into the form of a circle, whose

area is 154 cm 2. If the same wire is bent into the form = = √ sq.cm
of an equilateral triangle, the approximate area of the
equilateral triangle is Q86. The length of two parallel sides of a trapezium
(a) 93.14 cm2 (b) 90.14 cm2 are 15 cm and 20 cm. If its area is 175 sq. cm, then its
2
(c) 83.14 cm (d) 39.14 cm2 height is :
Ans: (b) Let r be the radius of circle. (a) 10 cm (b) 15 cm
πr2 = 154 cm2 (c) 25 cm (d) 20 cm
r2 = = 49 Ans: (a) Area of trapezium
r = 7 cm =
length of wire = circumference of circle
175 = ×H
= = 44 cm
H= = 10cm.
Now, Perimeter of equilateral triangle = 44 cm
side = cm
Q87. If the base of right prism remains same and the

Area of equilateral triangle = ( ) measures of the lateral edges are halved, then its
√ 2
volume will be reduced by:
= = 91.42 cm (a) 50% (b) 25%
Area of equilateral triangle is nearly equal to 90.14 cm2 (c) 66% (d) 33.33%
Hence, option (b) is correct. Ans: (a) Volume of prism = base are × height
=A×h
Q84. The perimeter of a sheet of paper in the shape of New volume =
a quadrant of a circle is 75 cm. Its area would be
( ): ∴ % decrease in volume =
2 2
(a) 346.5 cm (b) 100 cm =
(c) 693 cm2 (d) 512.25 cm2
Ans: (a) Perimeter of quadrant of a circle = 75 cm Q88. If water is freezed to become ice, its volume is
= 75 increased by 10%, then if the ice is melted to water
again, its volume will be decreased by :
( ) = 75
(a) 8% (b)
2r =
(c) 9% (d) %
r= = 21 cm
Ans: (d) Let initial volume = 100
Volume after increase = 100× = 110
Q85. The total surface area of a regular triangular
pyramid with each edges of length 1cm is: So, decrease = ×100
(a) √ sq.cm (b) √ sq.cm = = %
(c) 4sq.cm (d) √ sq.cm
Ans: (a)

Page 130 of 516


https://telegram.me/aedahamlibra
Q89. A hem ispherical bowl has internal radius of 6
cm. The internal surface area would be : (take π =
So, Volume of cone = × × 3.5×3.5×7
= 89.8cm3.
3.14)
(a) 400 cm2 (b) 289.75 cm2
2 Q92. The perimeter of one face of a cube is 20 cm. Its
(c) 225 cm (d) 226.08 cm2
volume will be
Ans: (d) Internal Radius of hemisphere = 60
(a) 100 cm3 (b) 125 cm3
Internal surface area = 2πr2 3
(c) 400 cm (d) 625 cm3
= 2 × 3.14 × (6) 2
Ans: (b) Perimeter of one face, 4a = 20 cm
= 226.08 cm2.
Therefore, side of cube, a = 5 cm
Volume of cube = a3 = 53 = 125 cm3
Q90. The outer circumference of a circular race–track
is 528 metre. The track is everywhere 14 metre wide. 2
Q93. The surface area of a sphere is 616 cm . The
Cost of levelling the track at the rate of ₹ 10 per sq. volume of the sphere would be :
metre is: (a) 2100 cm2 (b) 2500 cm2
(a) ₹ 77660 (b) ₹ 76760
(c) cm2 (d) cm2
(c) ₹ 66760 (d) ₹67760 2
Ans: (d) Ans: (c) Surface area of sphere = 616cm

4 πr2 = 616 r = √ = 7cm

So, volume of sphere = π (7) 3


= × × 7 × 7 × 7 = 1437 cm3

Q94. A square is inscribed in a quarter –circle in such


a manner that two of its adjacent vertices lie on the
two radii at an equal dista nce from the centre, while
the other two vertices lie on the circular arc. If the
square has sides of length x, then the radius of the
circle is
Outer circumference = 528m
(a)


Outer radius = = 84m (c) √

∴ Inner radius = 84 – 14 = 70m Ans: (d)
Outer area of circular race – track = × 84 × 84
= 22176 m2
Inner area of circular race – track = × 70 × 70
= 15400m2.
So area of track = 22176 – 15400 = 6776m2
Cost of levelling the circular track = 6776 × 10 = ₹ 67760

Q91. The volume of the largest right circular cone OA = AD = x


that can be cut out of a cube of edge 7 cm? ( ∴ √ √
So (d) option is correct.
)
(a) 13.6 cm3 (b) 121 cm3
3
Q95. If the area of a circle is A, radius of the circle is r
(c) 147.68 cm (d) 89.8 cm3
and circumference of it is C, then
Ans: (d) Volume of cube = a = (7) 3 = 343 cm3
3
(a) rC = 2A (b)
Volume of right circular cone =
Radius of cone = 3.5cm (c) AC = =C
Height of cone = 7cm Ans: (a) Area of circle, A = πr2... (i)
Page 131 of 516
https://telegram.me/aedahamlibra
Circumference of circle, C = 2πr... (ii)
Multiplying eq. (i) by 2, we get, 2A = 2πr2
Multiplying eq. (ii), by ‗r‘, we get rC = 2πr2
∴ rC = 2A

Q96. 5 persons will live in a tent. If each person Side of square = Diameter of sphere
requires 16m 2 of floor area and 100 m 3 space for air
Radius of sphere cm
then the height of the cone of smallest size to
accomodate these persons would be ? Area of sphere × π × 9 × 9 × 9 = 972π
(a) 18.75 m (b) 16 m
(c) 10.25 m (d) 20 m Q100. The radius and the height of a cone are each
Ans: (a) For surface Area of 5 persen increased by 20%. Then the volume of the cone
increases by
(a) 20% (b) 20.5%
(c) 62% (d) 72.8%
Now, volume of cone = volume of air space of 5 person Ans: (d) Let radius and height of cone = 100
New radius and height of cone = 120
Ratio =

∴h= Volume Ratio = ( )


Difference = 216 – 125 = 91
Q97. If the volume of a sphere is numerically equal to Difference % = =
its surface area then its diameter is :
(a) 4 cm (b) 2 cm = = 72.8%
(c) 3 cm (d) 6 cm
Ans: (d) According to question Q101. A solid sphere of radius 9 cm is melt ed to form
Volume of sphere = surfae area of shpere a sphere of radius 6 cm and a right circular cylinder
of same radius. The height of the cylinder so formed is
(a) 19 cm (b) 21 cm
(c) 23 cm (d) 25 cm
diameter = 6 cm Ans: (a) Radius of large sphere = 9 cm
Radius of smaller sphere = 6 cm
Q98. The base area of a right pyramid is 57 sq. units Radius of cylinder = 6 cm
and height is 10 units. Then the volume of the Radius height of cylinder = h
pyramid is
(a) 190 c. units (b) 380 c.units
(c) 540 c.units (d) 570 c.units
Ans: (a) Volume of Pyramid
[ ]
h=
[ ]
= 190 c. units h=
h= = 19 cm
2
Q99. The area of the largest sphere (in cm that can
be drawn inside a square of side 18 cm is Q102. The radii of the base of a cylender and a cone
(a) 972 π (b) 11664 π are equal and their volumes are also equal. Then the
(c) 36 π (d) 288 π ratio of their heights is
Ans: (a) (a) 2 : 1 (b) 1 : 2
(c) 1 : 3 (d) 1 : 4
Ans: (c) Vol. of cylinder = Vol. of cone

Page 132 of 516


r = R (Given) https://telegram.me/aedahamlibra
(a) 12
(c) 16
(b) 14
(d) 28
Ans: (d) Total surface area of three solid spheres

or 1 : 3 = 628.57 cm2
Now,
Q103. The curved s urface area of a cylinder with its Volume of new sphere
height equal to the radius, is equal to the curved
surface area of a sphere. The ratio of volume of the
cylinder to that of the sphere is ∴
(a) √ √ R3 = 216
(c) 3: √ (d) 3:√ ∴ √ cm
Ans: (d) ∴Surface Area of new solid sphere

Q104. A solid sphere of d iameter 17.5 cm is cut into


two equal. halves. What will be the increase (in cm 2) = 452.5 cm2
in the total surface area? ∴Required percentage
(a) 289 (b) 361.5
(c) 481.25 (d) 962.5
Ans: (c) Here, = 28%.
Radius of sphere cm = 8.75cm
Q107. If the diameter of a sphere is 14 cm, then what
∴Total surface Area of sphere = 4πr2
is the surface area (in cm2) of the sphere?
(a) 616 (b) 308
= 962.5 cm2 (c) 462 (d) 636
After cut in two equal halves. Ans: (a) Diameter of sphere = 14 cm
∴radius cm
∴Surface area of sphere = 4 πr2
= 616 cm2.
∴Total surface of both hemisphere = 2 × 3 πr2
Q108. The total surface area of a hemisphere is 166.32
1443.75 cm 2 sq cm, find its radius?
(a) 4.2 cm (b) 8.4 cm
∴Required increased area = (1443.75 – 962.5)
(c) 1.4 cm (d) 2.1 cm
= 481.25 cm2
Ans: (a) Here,
Total surface area of hemisphere = 166.32 sq cm.
Q105. A cylindrical rod of radius 30 cm and length 40
r=?
cm is melted and made into spherical balls of radius 1
cm. The number of spherical balls is
(a) 36000 (b) 27000
(c) 90000 (d) 40000
Ans: (b) Total number of spherical balls
∴r = 4.2 cm.

= 27000 Q109. If the radius of the cylinder is increased by


25%, then by how much percent the height must be
reduced, so that the volume of the cylinder remains
Q106. Three solid spheres of radius 3 cm, 4 cm, and 5
same?
cm are melted and recasted into a solid sphere. What
(a) 36 (b) 56
will be the percentage decrease in the surface area?
Page 133 of 516
(c) 64
Ans: (a)
https://telegram.me/aedahamlibra
(d) 46

Q110. If diagonals of a rhombus are 16 cm and 30 cm. Q113. The ratio of curved surface area of two cones is
then what is the perimeter (in cm) of the rhombus? 1 : 4 and the ratio of slant height of the two cones is 2 :
(a) 32 (b) 64 1. What is the ratio of the radius of the two cones?
(c) 34 (d) 68 (a) 1 : 2 (b) 1 : 4
Ans: (d) Let side of rhombus be x (c) 1 : 8 (d) 1 : 1
Ans: (c) Here,
The ratio of curved surface area of two cones = 1 : 4
Let curved surface area of first cone be x and curved
surface area of second cone be 4x.
Let slant height of first cone be 2x and slant height of
second cone be x.
According to question,
By pythagorean theorem,
x2 = (15)2 + (8) 2
x2 = 225 + 64 = 289 ∴
∴ √ cm ∴The ratio of the radius of the two cones = 1 : 8
∴Perimeter of rhombus = 4 × side
= 4 × 17 Q114. If the perimeter of a rhombus is 80 cm and one
= 68 cm of its diagonal is 24 cm, then what is the area (in cm 2)
of the rhombus?
Q111. A sphere has a total surface area 9πcm2. Its (a) 218 (b) 192
volume is: (c) 384 (d) 768
(a) 36πcm3 (b) 18πcm3 Ans: (c)
(c)
πcm3 (d) πcm3
Ans: (d) Total surface area of sphere = 9π cm2

4πr2 = 9π cm2

∴ √

∴Volume of sphere perimeter of rhombus = 80 cm


∴Side of rhombus cm
( )
By pythagorean theorem,
(20)2 = (12)2 + (x2)
∴x2 = 400 – 144 = 256
∴ √ cm
∴Diagonal of rhombus = 2x = 2 × 16 = 32 cm and
other diagonal = 24 cm
Q112. Radius of hemisphere is twice that of a sphere.
What is the ratio of total surface area of hemisphere ∴Area of rhombus
and sphere?
(a) 3 : 1 (b) 12 : 1
(c) 4 : 1 (d) 6 : 1
Ans: (a) Let radius of hemisphere = 2x Q115. A solid cylinder having radius of base as 7 cm
Radius of sphere = x and length as 20 cm is bisected from its height to get
∴Ratio of total surface area of hemisphere and sphere

Page 134 of 516


https://telegram.me/aedahamlibra
two identical cylinders. What will be the percentage
increase in the total surface area?
θ1 + θ2 = 60°... (1)
sec (θ1 – θ2) =

(a) 29.78 (b) 25.93
or sec (θ1 – θ2) = sec 30°... (2)
(c) 27.62 (d) 32.83
θ1 – θ2 = 30°
Ans: (b) Here,
Adding equation (1) & (2)
Radius of cylinder = 7 cm
θ1 + θ2 + θ1 – θ2 = 90°
Height of cylinder = 20 cm
θ1 = 45° & θ2 = 15°
Now, sin 2 × 45° + tan 3 × 15°
Total surface area of cylinder = 2πrh + 2πr2
= sin 90° + tan 45° = 1 + 1 = 2

= 880 + 308 = 1188 cm2 Q3. The value of


When cylinder is cut ing along height, two new cylinders (sec θ – cos θ) (cosec θ – sin θ) (tan θ + cot θ)
are generated–radius of new cylinder = 7cm is
height of new cylinder = 10cm (a) 2 (b) 0
(c) 1 (d)
Total surface area of new cylinder = 2πrh + 2πr2 Ans: (c) (sec θ – cos θ) (cosec θ – sin θ) (tan θ + cot θ)

( )( )( )
440 + 308
= 748 cm2
∴Total surface area of two new cylinders
= 748 × 2 = 1496 cm2 [ ]
∴Percentage increase in surface area ( 1
)% = 25.93%
Q4. If ( ) √ , value of cos θ is:

Trigonometry (a) 0 (b)



(c) (d) 1

Q1. tan 7° tan 23° tan 60° tan 67° tan 83° is equal to Ans: (c) ( ) √ , ( )
(a) (b) 1

(c) 0 (d) √
∴ cos θ =
Ans: (d) tan 7° tan23° tan 60° tan 67° tan 83°
tan (90° – 83°) tan (90° – 67°) tan 60° tan 67° tan 83°
cot 83° cot 67° tan 60° tan 67° tan 83° Q5. If , then value of cotθ is:
[ (a)
(c) 3 (d) 2
tan (90° – θ) = cotθ]
Ans: (a)
Dividing numerator and denominator by cos θ.
tan 60° = √

Q2. If tan (θ1+ θ2) = √ and sec , then



the value of is equal to
(Assume that ) 2 tan θ – 1 = 1 + tan θ tan θ = 2
(a) 1 (b) 2 Hence,
(c) 0 (d) 3
Ans: (b) √ Q6. If A and B are complementary angles, then the
or tan value of
Page 135 of 516
B is
https://telegram.me/aedahamlibra
sin A cos B + cos A sin B – tan A tan B + se c2 A – cot2 (a) 3
(c) 1
(b) 5
(d) 2
(a) 2 (b) 0 Ans: (d) 2sin2Α θ + 3 cos2 θ = 2 sin2 θ + 2cos2 θ + cos2 θ
(c) 1 (d) –1 = 2 (sin2θ + cos2θ) + cos2θ = 2 + cos2θ
Ans: (c) A + B = 90° A = 90° – B ∴ Last value = 2 + 0 = 2
sin A = sin (90° – B) = cos B Similarly, [ ]
cos A = sin B, tan A = cot B
∴ sin A . cos B + cos A. sin B – tan A. tanB + sec 2 A– Q10. If sin θ + cos θ = √ cos (90 – θ), then cot θ is
cot2 B = cos2 B+ sin2 B – cotB. tanB + sec2A – tan2A (a) √ (b) 0
= 1 – 1 +1 = 1[ (c) √ √ –1
]
Ans: (d) sinθ + cosθ = √
sinθ + cosθ = √ sinθ
Q7. P and Q are two points observed from the top of a
Divide eq. by sin θ
building √ m high. If the angles of depression of
1 + cotθ = √
the points are complementary and PQ = 20 m, then
the distance of P from the building is cotθ = √
(a) 25 m (b) 45 m
(c) 30 m (d) 40 m Q11. If 2 – cos2 θ = 3 sin θ cos θ, sin θ ≠ cos θ then tan
Ans: (c) θ is
(a) (b) 0
(c)
Ans: (a) 2 – cos2θ = 3sinθ. cosθ
Dividing by cos2θ

2 sec2θ – 1 = 3 tanθ
2 (1 + tan2θ) – 1 = 3 tanθ
2 tan2θ + 2 – 1 = 3 tanθ
In this type of Q just put 2 tan2θ – 3 tanθ + 1 = 0
h2 = ab 2 tan2θ – 2 tanθ – tanθ + 1
( √ ) =0
300 = x (x + 20) 2 tan θ (tan θ – 1) – 1 (tan θ – 1) = 0
10 (30) = x (x + 20) (2 tanθ – 1) (θ – 1) = 0
x + 20 = 30 or1

Q8. If 4x = sec θ and = tan θ then 8( ) is Q12. sec4 θ – sec2 θ is equal to


(a) tan2 θ – tan4 θ (b) tan2 θ + tan4 θ
(a) 4 2
(c) cos θ – cos θ (d) cos2 θ – cos4 θ
(c) Ans: (b) sec θ – sec θ = sec θ (sec2 θ – 1)
4 2 2

Ans: (c) 4x = sec θ = (1 + tan2 θ) (1 + tan2 θ –1) = tan2 θ + tan4 θ

Q13. If x sin 3 θ + y cos3 θ = sin θ cos θ and x sin θ = y


Again, cos θ, sin θ ≠ 0, cos θ ≠ 0, then x2 + y2 is
(a)
( ) √
(c) 1 (d) √
. / Ans: (c) xsin3θ + ycos3θ = sinθ.cosθ
(x sin θ) . sin2 θ + (y cos θ)
cos2 θ = sin θ.cos θ
Q9. The least value of 2 sin2 θ + 3 cos2θ is
xsinθ . sin2θ + xsin θ. cos2θ
Page 136 of 516
= sin θ.cos θ
x sin θ (sin2θ + cos2 θ)
https://telegram.me/aedahamlibra
Ans: (b)
Let BC = x
= sin θ.cos θ ∴ AB = x + 2
x = cos θ ∴ AB2 + BC 2 = AC2
∴ x sinθ = y cosθ (x + 2)2 + x2 = ( √ )
cos θ . sin θ = y cos θ x2 + 4x + 4 + x2 = 20
y = sin θ 2x2 + 4x – 16 = 0
∴ x2 + y2 = cos2 θ + sin2θ = 1 x2 + 2x – 8 = 0
x2 + 4x – 2x – 8 = 0
Q14. If cos A + cos 2 A = 1, then sin2 A + sin4 A is equal
x (x+ 4) – 2 (x + 4) = 0
to
(x – 2) (x + 4) = 0
(a) 1 (b) x = 2 = BC
(c) 0 (d) –1 ∴ AB = 2 + 2 = 4cm
Ans: (a) cos A = 1 – cos2 A = sin2A
∴ sin2A + sin4A = sin2A + cos2A = 1

Q15. A tree is broken by the wind. If the top of the


tree struck the ground at an angle of 30° and at a
distance of 30 m from the root, then the height of the Q17. If cot A + cosec A = 3 and A is an acute angle,
tree is then the value of cos A is:
(a) √ √ (a) (b) 1
(c) √ √ (c)
Ans: (c)
Ans: (a) cot A + cosec A = 3
cosec2 A – cot2 A = 1
(cosec A – cot A) (cosec A + cot A) = 1
cosec A – cot A =
cosec A + cot A = 3
By Adding
2cosec A =
cosec A
AB = tree √
BC = broken part cos A
∴ BC = CD
AD = 30 metre
Q18. If tan (x + y) tan (x – y) = 1, then the value of tan
From ∆ACD,
x is:
tan 30° (a) √ (b) 1
√ metre (c) 1/2 (d)
√ √
CD = AC sin 30° √ √ Ans: (b) tan (x + y) tan (x – y) = 1
∴ AB = AC + BC tan (x + y) = cot (x – y)
= tan (90° – (x – y))
√ √ √ metre
x + y = 90° – (x – y)
2x = 90° x = 45°
Q16. In a right -angled triangle ABC, ∠B is the right
∴ tan x = tan 45° = 1
angle and AC = √ cm. If AB – BC = 2 cm, then the
value of (cos2A – cos2C) is:
Q19. A boy standing in the middle of a field, observes
(a) a flying bird in the north at an angle of elevation of
(c) 30° and after 2 minutes, he observes the same bird in
Page 137 of 516
https://telegram.me/aedahamlibra
the south at an angle of elevation of 60°. If the bird
flies all along in a straight line at a height of √ m,
(a) 4
(c) 2
(b) 1
(d) 3
then its speed in km/h is: Ans: (a) Put θ = 45°
(a) 4.5 (b) 3 a = tan 45° – cot 45°, b = sin 45° – cos 45°
(c) 9 (d) 6 a=1–1=0
Ans: (d) b=
√ √
Put in equation
(a2 + 4) (b2 –1)2 = (0 + 4) (0 – 1)2 = 4

Q23. If a 3 – b3 = 56 and a – b = 2, then the value of (a 2


+ b2) is:
AB = CD = 50√ metre (a) – 10 (b) – 12
From OAB, (c) 20 (d) 18
Ans: (c) (a – b)3 = a3 – b3 – 3ab (a –b)
tan 30°
8 = 56 – 3ab (2)
√ 6ab = 56 – 8 = 48
√ 2ab = 16... (i)
OA = √ √ metre ∴ a2 + b2 = (a – b)2 + 2ab
From ∆OCD, = 4 + 16 = 20
tan 60°
Q24. If α is a positive acute angle and 2sinα + 15cos2α

√ metre = 7, then the value of cotα is:
∴ BD = AC = 150 + 50 = 200 metre (a) 3/4 (b) 4/3
∴ Speed of bird (c)


m/minute kmph = 6 kmph Ans: (a) 2 sin α + 15 cos2 α = 7
2 sin α + 15 (1 –sin2 α) = 7
Q20. The simplied value of 2 sin α + 15 – 15 sin2 α = 7
is: 15 sin2 α – 2 sin2 α – 8 = 0
(a) cosA (b) 0 15 sin2 α – 12 sin α + 10 sin α – 8 = 0
(c) 1 (d) sinA 3 sin α (5 sin α – 4) + 2 (5 sin α – 4) = 0
(3sin α + 2) (5 sin α – 4) = 0
Ans: (a)
5 sin α – 4 = 0
5 sin α
∴cosec α

cosA √ √ √

Q21. The least value of 4 cosec2α + 9sin2α is:


(a) 14 (b) 10 Q25. If (a 2 – b2) sinθ + 2ab cosθ = a2 + b 2, then the
(c) 11 (d) 12 value of tanθ is
Ans: (d) 4 cosec2α + 9 sin2α (a)
= 4 cosec2α + 4 sin2α + 5 sin2α (c)
= 4 [cosecα – sinα)2+2] + 5 sin2α
Ans: (c) (a2 –b2) sin θ + 2ab cos θ = a2 + b2
= 12 [

cosecα – sinα ≥1]

Q22. If tanθ – cotθ = a and cosθ – sinθ = b, then the


{sin2θ + cos2θ = 1}
value of (a2 + 4) (b2 – 1)2 is:
Page 138 of 516
https://telegram.me/aedahamlibra
= 4 tan2θ + 9cot2θ + 12 = 12 + 13
= (2tan2θ – 3Cot2θ)2 + 25
(2 + tan2θ – 3Cot2θ) = 0
Q26. If 5 tanθ = 4, then the value of ( ) is the minimum value is 25.

(a)
Q30. If sinθ and 2x secθ – y
(c) cosecθ = 3, then the value of x + 4y2 is
2

Ans: (a) 5 tanθ = 4 (a) 1 (b) 2


tanθ (c) 3 (d) 4
Ans: (d) 2y cosθ = xsinθ
x secθ = 2y cosecθ
∴ 2x secθ – y cosecθ = 3
4y cosecθ – y cosecθ = 3
3y cosecθ = 3
y cosecθ = 1
y = sinθ
∴ x secθ = 2y cosecθ
Q27. If x, y are positive acute angles, x + y < 90° and = 2sinθ.cosecθ = 2
sin (2x – 20°) = cos (2y + 20°), then the value of sec (x x = 2cosθ
+ y) is ∴ x2 + 4y2 = 4cos2θ + 4sin2θ = 4
(a) √

Q31. If x = cosecθ – sinθ and y = secθ – cosθ, then the
(c) 1 (d) 0
value of
Ans: (a) sin (2x – 20°) = cos (2y + 20°)
x2y2 (x2 + y2 + 3) is
sin (2x – 20°)
(a) 0 (b) 1
= sin (90° – 2y – 20°)
(c) 2 (d) 3
= sin (70° – 2y)
Ans: (b) x2y2 (x2 + y2 + 3)
2x – 20° = 70° – 2y = (cosecθ – sinθ)2 (secθ – cosθ)2
2 (x + y) = 90° { (cosecθ – sinθ)2 + (secθ – cosθ)2 + 3}
x + y = 45°
∴ sec (x + y) = sec 45° = √ ( ) ( )

Q28. If tan (x + y) tan (x – y) = 1, then the value of tan 2( ) ( ) 3


( )is
. / . /
(a)
√ √
(c) √ (d) 1
{. / . / }
Ans: (a) tan (x + y). tan (x – y) = 1
tan (x + y) = cot (x – y)
= tan (90°– x + y) . / . /
x + y = 90° – x + y
2x = 90° {( ) ( ) } = cos2θ × sin2θ
∴ tan = tan 30° =

. /
Q29. The least value of (4sec2θ + 9 cosec2θ) is = cos6θ + sin6θ + 3cos2θ sin2θ
(a) 1 (b) 19 { }+ 3cos2θ.sin2θ
(c) 25 (d) 7 = (cos θ + sin θ) – 3cos θ.sin2θ
2 2 3 2

Ans: (c) 4 sec2θ + 9coses2θ (cos2θ + sin2θ) + 3cos2θ.sin2θ


= 4 (1 + tan2θ) + 9 (1 + cot2θ) = 1 – 3cos2θ.sin2θ + 3cos2θ.sin2θ = 1
= 4 + 4 tan2θ + 9 + 9cot2θ
Page 139 of 516
3cos10θ + 3cos8θ + cos6θ – 1 is
https://telegram.me/aedahamlibra
Q32. If sinθ + sin2θ = 1, then the value of cos 12
θ + (a) 0 (b)

(a) 0 (b) 1 (c) (d) 1
2 2
(c) –1 (d) 2 Ans: (b) sec θ – tan θ = 1
Ans: (a) sinθ + sin2θ = 1 (sec θ + tan θ) (sec θ – tan θ) = 1
sinθ = 1 – Sin2θ

sinθ = cos2θ √
∴ cos12θ + 3cos10θ + 3cos8θ + cos6θ – 1 ... (1)
= (cos4θ + cos2θ)3 – 1 √ (Given) ... (2)
= (sin2θ + cos2θ)3 – 1 = 1 – 1 = 0 Adding eqn. (1) and (2)


Q33. When the angle of elevation of the sum increases √ √ √
from 30° to 60°, the shadow of a post is diminished by √
5 metres. then the height of the post is ∴ [ ]
√ √
(a) Therefore, √
(c)
√ √ √
Ans: (a)

Q35. If 2y cosθ = x sinθ and 2x secθ – y cosecθ = 3,


then the relation between x and y is
(a) 2x2 + y2 = 2 (b) x2 + 4y2 = 4
2 2
(c) x + 4y = 1 (d) 4x2 + y2 = 4
Ans: (b) 2y cos θ = x sin θ

AB = Pole = h metre
BD = x metre And 2x sec θ – y cosec θ = 3
From ∆ABC,


√ ... (i) Now sin2 θ + cos2 θ = 1
From ∆ABD,


Q36. In a triangle ABC, AB = AC, BA is produced to
D in such a manner that AC = AD. The circular

measure of ∠BCD is

(a)

√ (c)
√ Ans: (d) AB = AC
√ ∴ ∠ABC = ∠ACB ... (1)

metre

Q34. If √ , then the positive value


of sinθ is

Page 140 of 516


https://telegram.me/aedahamlibra
(c) (d) 1

Ans: (d)
cos4 α sin2 β + sin4 α cos2 β = cos2 β sin2 β
cos4 α (1 – cos2 β) + cos2 β (1 – cos2 α)2 = cos2 β (1 –
cos2 β)
[opposite angle of equal sides are equal]
cos4 α – cos4 α cos2 β + cos2 β – 2 cos2 α
AC = AD
cos2 β + cos4 α cos2 β = cos2 β – cos4 β
∴ ∠ACD = ∠ADC ... (2)
cos4 α – 2 cos2 α cos2 β + cos4 β = 0
In a triangle,
(cos2 α – cos2 β)2 = 0
∠ABC + ∠ADC + ∠DCB = 180°
cos2 α = cos2 β
∠ABC + ∠ADC + ∠ACB + ∠ACD = 180°
sin2 α = sin2 β
2∠ACB + 2∠AOC = 180°
[From eqn. (1) & (2) ] Then,
∴∠BCD = 90° or π / 2

Q37. The radian measure of is cos2 β + sin2 β = 1


(a) ( ) ( )
Q40. If sin 2α = cos3α, then the value of (cot6α – cot2α)
(c) ( ) ( ) is
(a) 1 (b) 0
Ans: (a) ( ) [1 minute = 60 seconds]
(c) – 1 (d) 2
[ ] [ ] Ans: (a) If
... (1)
[1 degree = 60 minutes] Now consider, cot6 α – cot2 α
( ) radian Since
Substituting for tan 2α with cos α from (1) above equation
( )
will be

Q38. (where )is equal to


(a)
(c) Q41. The angles of elevation of the top of a tower
standing on a horizontal plane fro m two points on a
Ans: (a)
line passing through the foot of the tower at a distance
Dividing Numerator and Denominator by cos θ 9 ft and 16 ft respectively are complementary angles.
Then the height of the tower is
(a) 9 ft (b) 12 ft
(c) 16 ft (d) 144 ft
Ans: (b) In ∆ABC
... (1)
[ ]
In ∆ABD

Q39. If , then the value of

is
(a) 4 (b) 0
Page 141 of 516
https://telegram.me/aedahamlibrax=

Q45. Minimum value of 4tan2θ + 9cot2θ is :


(a) 12 (b) 1
(c) 6 (d) 13
Ans: (a) 4 tan2 θ + 9 cot2 θ
(2tan θ)2 + (3 cot θ)2
α + β = 90° (given)
(2tan θ)2 + (3cot θ)2 – 12 + 12 = (2 tan θ – 3cot θ)2 + 12
β = 90 – α
∴ Minimum value = 12 because (2tanθ – 3cotθ)2 ≥ 0
since
Q46. 2 cosec 2 23° cot 2
67° – sin2 23° – sin2 67°– cot2
67° is equal to
... (2) (a) 0 (b) 1
From eqn. (1) and (2) (c) sec2 23° (d) tan2 23°
feet. Ans: (c)

Q42. The value of tan 1° tan 2° tan 3° ... tan 89° is: =
(a) 1 (b) 2 =
(c) undefined (d) 0 =
Ans: (a) tan 1° tan 2° tan 3° ... tan 89°
= tan 1° tan 2°... tan45° ... tan (90 – 2) tan (90 – 1)
Q47. If cosec θ – cotθ = , the value of cosecθ is :
= tan 1° tan 2° ... 1 ... cot 2° cot 1°
= (tan 1° cot 1°) (tan 2° cot 2°) ... 1 = 1 (a)
(c)
Q43. The simplified value of
(1 + tanθ + secθ) (1 + cotθ – cosecθ) is Ans: (c) cosec θ – cot θ = ... (i)
(a) – 2 (b) 2 cosec2 θ – cot2 θ = 1
(c) 1 (d) – 1 (cosec θ + cot θ) (cosec θ – cot θ) = 1
Ans: (b) (1 + tan θ + sec θ) (1 + cot θ – cosec θ) cosec θ + cot θ
( )( ) = ... (ii)
On adding both equations.
( )( ) 2 cosec θ =

cosecθ =

Q48. If 5 tan θ = 4, then is


Q44. If sin θ – cosθ = the value of sin θ + cos θ is : (a)
(a) – 2 (b) ± 2 (c)

(c) (d) 2
Ans: (d) =
Ans: (c) sin θ – cos θ =
sin θ + cos θ = x.
On squaring and adding. Q49. The length of the shadow of a vertical tower on
2 2 level ground increases by 10 metres when the altitude
2 (sin θ + cos θ) =
of the sun changes from 45° to 30°. Then the height of
x2 = the tower is
(a) √ √
Page 142 of 516
(c) (√
Ans: (d)
) (√ https://telegram.me/aedahamlibra
) (c) (d) 1
Ans: (b) cos 90° = 0
∴ cos 1°, cos 2° .... cos 179° = 0

Q53. The angle of elevation of a tower from a distance


100m from its foot is 30°. Height of the tower is :
(a) √

(c) √

From ∆ABC, tan45° = Ans: (b)
In ∆ABD, tan30° =


h = (√ )

Q50. If 2 (cos 2 θ – sin2 θ) = 1 (θ is a positive acute


angle), then cot is equal to
(a) √ √ AB = h metre
(c) (d) 1 ∠ACB = 30°;

Ans: (a) 2 cos2θ = 1 BC = 100 metre
cos2θ = ∴ tan 30° =
θ = 30°

cot 30° = √ h= metre

Q51. If x sin θ + y cos θ = √ and


Q54. The degree measure of 1 radian (taking π = ) is
then the correct relation is
(a) 57°22' 16° (approx.) (b) 57° 61' 22'' (approx.)
(a) (c) 57° 16' 22'' (approx.) (d) 57° 22' 16'' (approx.)
Ans: (c) π = radian = 180°
(c)
∴ 1 radian =
Ans: (d) x sinθ + y cos θ = √
Put x = sin θ =
y = cos θ in the above equation, we have =

=
1=1
x = sinθ & y = cosθ is the solution of above equation.
Q55. The value of (sin2 25° + sin2 65°) is :
Now, on using x = sinθ & y = cosθ in

(a)

(c) 1 (d) 0
Ans: (c) sin2 25° + sin2 65°
= sin2 25° + sin2 (90° – 25°)
= sin2 25° + cos2 25° = 1

Q52. The value of Q56. If sec θ + tan θ = 2 + √ , then the value of sin θ +
cos 1° cos 2° cos 3°............ cos 177° cos 178° cos 179° cos θ is:
is: (a)
√ √
(a) (b) 0
√ (c) √

Page 143 of 516


Ans: 58 (b) https://telegram.me/aedahamlibra
(a) 2 (b) √
(c) √ (d) 1
9
Q57. If sinθ + cosec θ = 2, then the value of sin θ+ Ans: (b) Maximum value of a sin θ + b cosθ
cosec9θ is :
=√
(a) 1 (b) 3
∴ Maximum value of 2 sin θ + 3 cos θ
(c) 2 (d) 4
=√ √
Ans: (c) sinθ + cosecθ = 2
n
if x + = 2 then x + =2
Q62. One of the four angles of a rhombus is 60°. If the
∴ sinθ + =2 length of each side of the rhombus is, 8 cm, then the
length of the longer diagonal is
∴ sin9θ + =2
(a) √ cm (b) 8 cm
(c) √ cm (d) cm
Q58. If sin (A – B) = and cos (A + B) = where A > B √
Ans: (a)
> 0 and A + B is an acute angle, then the value B is
(a)
(c)
Ans: (b) sin (A – B) = = sin 30°
A – B = 30°
cos (A + B) =
= cos 60°
∠BAD = 60°
A + B = 60°
∴∠BAO = 30°
∴ A + B + A – B = 30° + 60° = 90°
∠ABO = 60°
2A = 90°
A = 45° ∴ sin 60° =
∴ A – B = 30° √
= OA
B = A – 30° = 45° – 30° = 15°
∴ OA = √
= radian
∴ AC = √ metre

Q59. Evaluate : tan 1° tan 2° tan 3° ........... tan 89°. Q63. If (1 + sinα) (1 + sinβ) (1 + sinγ) = (1 – sinα) (1 –
(a) 0 (b) 1 sinβ) (1 – sinγ), then each side is equal to
(c) –1 (d) 2 (a)
Ans: (b) tan 89° = tan (90° – 1°) = cot 1° (c)
tan 88° = tan (90° – 2°) Ans: (a) (1 + sin α) (1 + sin β) (1 + sin γ) = (1 – sin α) (1
= cot 2° – sin β) (1 – sin γ) = x
∴ Expression = tan 1°, cot 1°, tan 2°, ∴ x.x = (1 + sin α) (1 – sin α) (1 + sin β) (1 – sin β) (1 +
cot2°.... tan 45° = 1 sin γ) (1 + sin γ)
[ ] = (1 – sin2 α) (1 – sin2 β) (1 – sin2 γ)
= cos2 α. cos2β.cos2 γ
Q60. The value of = x = ± cosα . cos β . cosγ
152 (sin 30° + 2 cos 2 45° + 3 sin 30° + 4 cos 2 45° + .... +
17 sin 30° + 18 cos2 45°) is
Q64. The value of is
(a) an integer but not a perfect square
(b) a rational number but not an integer (a) 2 (b)
(c) a perfect square of an integer
(c) (d) 1
(d) irrational
Ans: (c) Ans: (d) Expression

Q61. Maximum value of (2 sinθ + 3 cosθ) is


= cos2θ + sin2θ = 1
Page 144 of 516
https://telegram.me/aedahamlibra
Ans: (b) cos θ + sin θ = √ cos θ
Q65. If tanθ = and θ is acute, then cosecθ On squaring both sides,
cos2θ + sin2θ + 2cos θ. sin θ
(a)
= 2 cos2 θ
(c) cos2θ – sin2θ = 2cosθ. cos θ
Ans: (d) (cos θ + sin θ) (cos θ – sin θ)
= 2 sin θ. cos θ
√ cos θ (cosθ – sinθ)
= sin θ. cos θ
cos θ – sin θ


√ √
( ) √ √

Q69. The value of is


2
Q66. If tan α = n tan β, and sin α = m sin β, then cos α (a) cosec θ (b) tan θ
is (c) 1 (d) cot θ
(a) Ans: (d) Expression

(c)
Ans: (a) tan α = n tan β
tan β tan α
= cosec θ – cot θ – cosec θ = cot θ
cot β and
[ ]
sin α = m sin β sin β sin α
coses β
Q70. The value of is
∴ cosec2β – cot2β = 1
(a) 2 (b) l
(c) 3 (d) 0
Ans: (b)

m2 – n2 cos2 α = sin2α
= 1 – cos2α
m2 – 1 = n2 cos2 α – cos2α
[
= (n2 – 1) cos2α

cos (90º – θ) = sin θ and cot (90º – θ)


= tan θ]
Q67. The tops of two poles of height 24 m and 36 m
are connected by a wire. If the wire makes an angle of Q71. If cos4θ – sin4θ = , then the value of 1 – 2 sin2θ is
60° with the horizontal, then the length of the wire is (a)
(a) 8 m (b) √
(c) (d) 0
(c) 6 m (d) √
4 4
Ans: (d) Ans: (a) cos θ – sin θ

Q68. If cosθ + sinθ = √ , then cosθ – sinθ is


(a) √ √
(c) √ √
Page 145 of 516
https://telegram.me/aedahamlibra
Q76. From 125 metre high tow ers, the angle of
depression of a car is 45°. Then how far the car is
from the tower ?
(a) 125 metre (b) 60 metre
(c) 75 metre (d) 95 metre
Q72. The value of Ans: (a)
is
7
Q77. If tan α + cot α = 2, then the value of tan α +
(a)
√ cot7α is

(c) (a) 2 (b) 16

Ans: 74 (d) (c) 64 (d) 128
Ans: (a) tan α + cot α = 2
tan α + tan2 α + 1 = 2 tan α
tan2 α – 2 tan α + 1 = 0
tan2 α – tan α – tan α + 1 = 0
√ √ √ tan α (tan α – 1) – 1 (tan α – 1) = 0
√ √ (tan α – 1) (tan α – 1) = 0
∴tan α = 1
Q73. The value of is Now, tan7 α + cot7 α (tan α)7
(a) – 1 (b) √ + 2
(c) – (2 + √ ) (d) √ – 2 Q78. Evaluate : 3 cos 80° cosec 10° + 2 cos 59° cosec
√ 31°
√ √
Ans: (c) √ (a) 1 (b) 3
√ √
(c) 2 (d) 5
( √ ) √ √
Ans: (d) 3 cos 80°. cosec 10° + 2 cos 59°. cosec 31°
(√ )
= 3 cos (90° – 10°). cosec 10° + 2 cos (90° – 31°). cosec
( √ )
( √ ) 31°
= 3 sin 10°. cosec 10° + 2 sin 31°. cosec 31°
Q74. If x = a cos θ – b sin θ, y = b cos θ + a sin θ, then =3+2=5
find the value of x2 + y2. [
(a) a2 (b) b2
(c) cos (90° – θ) = sinθ; sinθ.cosecθ = 1]
(d) a2 + b2
Ans: (d) Q79. 1f the angles of elevation of a balloon from two
consecutive kilometre–stones along a road are 30° and
Q75. In a triangle, the angles are in the ratio 2 : 5 : 3. 60" respectively, then the height of the balloon above
What is the value of the least angle in the radian ? the ground will be
(a) √
(a) km (b) km
(c)
(c) km (d) √ km
Ans: (d) Let angles are 2x, 5x and 3x. √

2x + 5x + 3x = 180º Ans: (a)


(sum of interior angle of triangles is 180º)
10x = 18º Q80. If tan θ + cot θ = 2, then the value of tan 2θ +
x = 18º cot2θ is
(a) 2 (b) 1
∴ Least angle in degree = 2x = 2 × 18 = 36º
(c) √ (d) 0
In radian =
Ans: (a) tanθ + cot θ = 2
On squaring both sides,
(tan θ + cot θ)2 = 4
tan2θ + cot2 θ + 2tan θ cot θ = 4
Page 146 of 516
https://telegram.me/aedahamlibra
tan2θ + cot2θ = 4 – 2 = 2 [tan θ. cotθ = 1]
Q86. A kite is flying at a height of 50 metre. If the
Q81. ( ) radians is equal to length of string is 100 metre then the inclination of
string to the horizontal ground in degree measure is
(a) 100° (b) 120°
(a) 90 (b) 60
(c) 108° (d) 180°
(c) 45 (d) 30
Ans: (c) radian = 180°
Ans: (d)
∴ radian = = 108°

Q82. sin2θ – 3 sin θ + 2 = 0 will be true if


(a) 0 ≤ θ < 90 (b) 0 < θ < 90
(c) θ = 0° (d) θ = 90°
Ans: (d) sin2 θ – 3 sin θ + 2 = 0 sinθ =
sin2 θ – 2 sin θ – sin θ + 2 = 0
θ = 30°
sin θ (sinθ – 2) – 1 (sin θ – 2) = 0
(sin θ – 1) (sin θ – 2) = 0
Q87. The simplest value of
sin θ = 1 = sin 90°
sin2x + 2 tan2x – 2 sec2 x + cos2 x is
θ = 90° and sin θ 2
(a) 1 (b) 0
(c) –1 (d) 2
Q83. The eliminant of θ from xcos θ – y sinθ = 2 and x
Ans: (c) sin2x + cos2x – 2 (sec2x –tan2x)
sin θ + y cos θ = 4 will give
1 – 2 (1) = –18
(a) x2 + y2 = 20 (b) 3x2 + y2 = 20
2 2
(c) x – y = 20 (d) 3x2 – y2 = 10
Q88. If x = a sin θ and y = b tan θ then prove that
Ans: (a) x cos θ – y sin θ = 2
x sin θ + y cos θ = 4 is
On squaring both the equations and adding (a) 1 (b) 2
x2cos2 θ + y2 sin2 θ – 2xy (c) 3 (d) 4 The following pie -
sin θ. cos θ + x2sin2 θ + y2 cos2θ + 2xy sinθ . cosθ chart shows the monthly expenditure of a family on food,
= 4 + 16 house rent, clothing, education, fuel and miscellaneous.
x2 (cos2 θ + sin2 θ) + y2 (sin2 θ + cos2 θ) = 20 Study the pie-chart and answer questions nos. 91 to 95.
x2 + y2 = 20

Q84. If and , then θ is


(a) radian (b) radian
(c) radian (d) radian
2 2
Ans: (b) sec θ + tan θ = 7
1 + tan2θ + tan2θ = 7
tan2θ =
for √
Ans: (a)
cosec2θ – cot2θ = 1
Q85. The value of
Q89. From the top of a light -house at a height 20
0 1 metres above sea -level, the angle of depression of a
ship is 30°. The distance of the ship from the foot of
(sec2 A – cosec2 A)
the light-house is
(a) l (b) 3
(a) 20 m (b) √ m
(c) 2 (d) 4
Ans: (c) (c) 30 m (d) √ m

Page 147 of 516


Ans: (b) https://telegram.me/aedahamlibra

tan 30° = , =

x= √

Q90. If θ is a positive acute angle and 4 cos2 θ – 4 cos θ


+ 1 = 0, then the value of tan (θ – 15°) is equal to ED = BC = 10 m
(a) 0 (b) 1 In ∆ABE, tan60° =
(c) √
√ √ √ ... (1)
Ans: (b) 4 cos2θ – 4 cos θ + 1 = 0
In ∆EBC, tan 30° =
(2 cos θ – 1)2 = 0
or,2 cos θ = 1 √

Putting value of EB in (1)
θ = 60°
AB = √ ( √ )
Hence, the value of tan (θ – 15°) = tan (60° – 15°)
AC = AB + BC = 40 m
= tan 45° = 1

Q91. The value of Q93. If ( √ ) + (r si n θ – 1)2 = 0, then the


is value of is equal to
(a) – 1 (b) 0
(a) √
(c) 1 (d) 2
Ans: (a) (c) √
= Ans: (a)

Q92. A vertical pole and a vertical tower are standing Q94. The value of + ( √ ) is equal to
on the same level ground. Height of the pole is 10 (a) – 1 (b) 1
metres. Form the top of the pole is the angle of (c) 2 (d) 0
elevation of the top of the tower and angle of
depression of the foot of the tower are 60° and 30° Ans: (c)
respectively. The height of the tower is +
(a) 20 m (b) 30 m
= 2 cos2 θ + 2sin2 θ – sin θ cos θ + sinθ cosθ = 2
(c) 40 m (d) 50 m
Ans: (c) 2
Q95. The value of sin 1° + sin 22° + sin 23° + .... +
sin289° is
(a) 22 (b) 44
(c)
Ans: (d) (sin2 1° + sin2 89°) + (sin2 2° + sin2 88°) + ...
+ (sin2 44° + sin2 48°) + sin2 45°
= (sin2 1° + cos 2 1°) + (sin 2 2° + cos 2 2°) + ... + (sin 2 44°
+ cos2 44°) + sin2 45°
= 1 + 1 + .... + 1 (44 times) + =

Page 148 of 516


https://telegram.me/aedahamlibra
Q99. If θ is a positive acute angle and cosec θ + cot θ =
Q96. If sin 17° =

then sec 17° – sin 73° is equal , then the value of cosec θ is

to (a) √

(a) (c) √ (d) 1
( √ ) ( √ )
Ans: (c) cosecθ + cotθ = √
(c) √
( √ ) + =√
Ans: (d) sin17° =
=√

cos 17° = √ =
=√
sec 17° – sin 73° = sec 17° – cos 17°
√ =√
= = =
√ √ √
= ; = 30°; θ = 60°

Q97. The shadow of a tower standing on a level plane cosec θ = cosec 60° =

is found to be 30 m longer when the Sun's altitude
changes from 60° to 45°. The height of the tower is Q100. The length of the shadow of a vertical tower on
(a) 15(√ )m (b) 15(√ )m level ground increases by 10 metres when the altitude
(c) 15( √ )m (d) m of the sun changes from 45° to 30°. T hen the height of
the tower is
Ans: (a)
(a) (√ ) metres (b) √ metres
(c) metres (d) 30( √ ) metres

Ans: (a)

Q101. If sin θ + cos θ = √ cos θ, then the value of


cot θ is
(a) √ √
In ∆ABC, tan60° = (c) √ (√ )
x= ... (1) Ans: (a) sinθ + cos θ = √ cos θ

sinθ = (√ ) cos θ
In ∆ ABD, tan 45° =
cotθ =

1= or h = 30 + x

Putting value of x from (1)
cotθ = √
√ √

h=
√ Q102. If is an acute angle and 2sin + 15cos 2 = 7,
(√ )
or = 30 h = 15 ( √ )m then the value of cot is:

(a)
Q98. If cos α + sec α = √ , then the value of cos 3
α+
sec3 α is (c)
(a) 2 (b) 1 Ans: (d) 2 sinα + 15 cos2α = 7where α is acute angle
(c) 0 (d) 4 2 sinα + 15 (1– sin2α) = 7
Ans: (c) cosα + sec α = √ 2 sinα + 15 – 15 sin2α – 7 = 0
taking cube both sides – 15 sin2α + 2 sinα + 8 = 0
cos3α + sec3 α + 3 cosα secα (cosα + secα) = √ 15 sin2α – 2 sinα – 8 = 0
Let sinα = t
cos3α + sec3 α + √ = √
So 15t2 – 2t – 8 = 0
cos3α + sec3 α = 0
15t2 – 12t + 10t – 8 = 0
3t (5t – 4) + 2 (5t – 4) = 0
Page 149 of 516
5t – 4 = 0; 3t + 2 = 0 https://telegram.me/aedahamlibra
Ans: (d) Let height of the pillar is h m
In ∆ACD
t = or t = (–ve value not possible)
tanα =
∴ sin α =
In ∆ABD

cos α =
tan (90 – α) =
∴ cot α =

Q103. The shadow of a tower standing on a level plane


is found to be 40m longer when the sun‘s al titude is
45° than when it is 60°. The height of the tower is:
(a) 40( √ )m (b) 10( √ )m
(c) 20( √ )m (d) m
Ans: (d)

cot α =
=
=
h=√

Q105. If sinθ + sin2 θ = 1, then the value of cos 2


θ+
cos4 is:
(a) 2 (b) 0
Let S 1 and S 2 be the two different positions of sun and (c) 1 (d) –1
AB is the tower Ans: (c) sinθ + sin2θ = 1
In ∆ABC sinθ = 1 – sin2θ
tan 45° = ( sinθ = cos2θ
Squaring both sides
tan 45° = 1) sin2θ = cos4θ
h=x 1 – cos2θ = cos4θ
In ∆ABD tan 60° = cos2θ + cos4θ = 1

√ = Q106. Value of the expression :


(√ ) = √ √
√ (√ ) (a) 0 (b) 2
√ √
x= (c) √
√ √
= √ (√ ) = 20( √ ) Ans: (c)

Q104. From two points on the ground and lying on a √


Q107. The value of x in the equation is :
straight line through the foot of a pillar, the two

angles of elevation of the top of the pillar are (a)

complementary to each other. If the distances of the
(c)
two points from the foot of the pillar are 12 metres √
and 27 metres and the two points lie on the same side Ans: (a)
of the pillar, then the height (in metres) of the pillar
; ; ; ;
is: √ √

(a) 16 (b) 12 √
(c) 15 (d) 18
Page 150 of 516
So, (1) 2 – ( ) = x ( ) ( ) (√ )
√ √
https://telegram.me/aedahamlibra
(c)
2
(d) None
√ Ans: (a) Given sinθ + sin θ = 1
1 – sin2θ = sinθ

x= = Then, cos2θ + cos4θ cos2θ + (cos2θ)2

(1– sin2θ) + (1– sin2θ)2
√ sinθ + sin2θ = 1
Q108. If sin then the value of sin 3θ is equal to :
(take (0° ≤ θ ≤ 90°) Q113. Find the value of
(a) 0 (b) tan 4° tan 43° tan 47° tan 86°
(c) 1 (d) (a) 1 (b)


(c) 2 (d)
Ans: (c) sin 2θ = = sin 60° Ans: (a) tan 4° tan 43° tan47° tan 86°
2θ = 60°, tan (90° – 86°) × tan (90° – 47°) × tan 47° × tan 86°
θ = 30° cot 86° × cot 47° × tan 47° × tan 86°
sin 3θ = sin 3 (30°) = 90° 1
= sin 90° = 1
Q114. If x cos θ – sin θ = 1, then
Q109. If = 3 then the value of sin2θ is : x2 + (1 + x2) sin θ equals
(a) 1 (b) –1
(a)
(c) 0 (d) 2
√ Ans: (a)
(c)
Ans: (a) =3
Q115. The numerical value of + –
By componendo and dividendo
= =2 – is
So, tan θ = 2 (a)
sin2θ = (1– cos2θ) = ( ) (c) √
= = = = Ans: (a)
( ) ( ) ( ) ( )
√ √
Q110. The maximum value of sin4θ + cos4θ is √ (√ )
( ) ( )

(a) 1 (b) 2
(c) 3 (d)
Ans: (a) The maximum value of sin4θ + cos4θ is 1.

Q111. If α + β = 90° then the expression is equal to :


(a) sec2β (b) tan2β
Q116. The minimum value of 2 sin2Θ + 3 cos2Θ is
(c) sec2α (d) tan2α
(a) 1 (b) 3
Ans: (c) + = 90° = 90° –
(c) 2 (d) 4
= + sin + sin2
2
Ans: (c) Given that
= 2sin2θ + 3cos2θ
= 2 (1 – cos2θ) + 3cos2θ = 2 – 2cos2θ + 3 cos2θ
= = = = 2 + cos2θ
For minimum value cosθ = 0
Q112. If = 1 then is equal to Minimum value of 2sin2θ + 3 cos2θ = 2

(a) 1 (b) Q117. A 10 m long ladder is placed against a wall. It is


inclined at an angle of 30° to the ground. The distance
Page 151 of 516
https://telegram.me/aedahamlibra
(in m) of the foot of the ladder from the wall is (Given b=√ =√
= 1.732) ,
(a) 7.32 (b) 8.26
(c) 8.66 (d) 8.16 Now,
Ans: (c)
Q121. If θ be acute angle and tan (4θ – 50°) = cot (50°
– θ), then the value of θ in degrees is :
(a) 30 (b) 40
(c) 20 (d) 50
Ans: (a)
tan (4θ – 50°) = tan (90 – (50° –θ))
4θ – 50° = 90 – 50° + θ
3θ = 90°
cos 30° = θ = 30°.
AB = AC × cos 30°
√ Q122. If sec θ + tan θ = p, (p ≠ 0) then secθ s equal to
= = 8.66 m
(a) ( ) (b) 2( )
2 2 2
Q118. The value of sin 22° + sin 68° + cot 30° is (c) ( )
(a) (b) 4 Ans: (b) Secθ + tanθ = p... (i)
(c) (d) 3 We know that sec2θ – tan2θ = 1
(secθ – tanθ) (secθ + tanθ) = 1
Ans: (b)
Now, Put the value of secθ + tanθ = p
=
= secθ – tanθ = ... (ii)

= (√ ) = 1 + 3 = 4 Now, solving eqn. (i) and (ii)


( )
Q119. If the sum and difference of two angles are
radian and 36° respectively, then the value of smaller Q123. A kite is flying at the height of 75 m from the
angle in degree taking the value of π as is : ground. The string makes an angle θ (where cot θ =
(a) 60° (b) 48° ( ) ) with the level ground. Assuming that
(c) 52° (d) 56°
there is no slack in the string, the length of the string
Ans: (c) Let angles are x and y rad.
is equal to :
x – y = 36°... (i) (a) 75 m (b) 85 m
x+y= (c) 40 m (d) 65 m
x+y= ... (ii) Ans: (b)

Now, on solving each (i) and (ii), we get


x = 68°, and y = 52°
So, smaller angle = 52°

Q120. If 5sinθ = 3, the numerical value of

(a)
(c)
BC = 40
Ans: (c) 5 sinθ = 3
AC = √
sinθ =
Page 152 of 516
AC = √ =√ https://telegram.me/aedahamlibra
Ans: (b) In ∆ A B C
=√ = 85 cm. , A C = 50m

Q124. If sin 31° = The value of sec 31° – sin59°



is
(a)
√ √

(c) (d) – √


Ans: (a) sin31° = cos31 = In ∆ A B D
sec31° – sin59 √ = 50 + x
sec31° – cos31°
√ – 50 = x
√ x = (√ )

, Q128. The value of
√ √
cot17° (cot73° cos222° + is
Q125. The circular measure of the i ncluded angle (a) 0 (b) 1
formed by the hour hand and minute hand of a clock (c) 2 (d) √3
at 3 PM will be Ans: (a) ( )
(a) π/4 (b) π/3
cot 17° (tan 17° sin268° + tan 17° cos268°)
(c) 5π/12 (d) π/2
cot 17° tan17° (sin268° + cos268°)
Ans: (d) Hour hand covered in 12 hr = 360°
(1) (1) = 1
3 hr = = 90° or
Q129. The upper part of a tree broke at a certain
Q126. If θ > 0, be an acute angle, then the value of θ in height makes an angle of 60° with the ground at a
degrees satisfying is distance of 10 m. from its feet. The original height of
(a) 90° (b) 30° the tree was
(a) 20√3 m. (b) 10√3 m.
(c) 45° (d) 60°
(c) 10 (2 +√ (3) m. (d) 10 (2–√ (3) m.
Ans: (d) Ans: (c)

Q130. A pilot in an aeroplane at an altitude of 200 m


[ ] [ ] observes two points lying on either side of a river. If
the angles of depression of the two points be 45° and
60°, then the width of the river is
0 1
(a) ( ) ( )
√ √
(2 – cos θ) = (1 + cos θ)
(c) 400√3 m (d)
2 cos θ = 1
cos θ = Ans: (a) Let BC be the width of river in ∆ A B C
θ = 60° = tan 45°

Q127. A tower is 50 meters high.Its shadow is x


metres shorter when the sun‘s altitude is 45° than BC = 200
when it is 30°. The value of x in metres is In ∆ ABD
(a) 50(√ ) (b) 50(√ )
(c) (d) 50

Page 153 of 516


https://telegram.me/aedahamlibra
BC = 50
BD = √ = 50( √ )
= 50 (2.73) = 136.5 m

Q134. If 0 < A < 90o, then the value of

BD = (a) 2 (b) 0

DC = (c) 1 (d)

Ans: (c)

Q131. If tan (5x 10°) = cot (5y + 20°), then the value of
Q135. If m and where
x + y is
√ , then value of m2 – n2 is
(a) 15° (b) 16°
(a) 2 (m2 + n2) (b)
(c) ( ) (d) 24°
√ (c) 4 mn (d)
Ans: (b) tan (5x – 10°) = cot (5y + 20°)
* +
tan (5x – 10°) = tan (90° – 5y – 20°)
5x – 10° = –5y + 70° Ans: (b)
5x + 5y = 80°
x + y = 16 Q136. From the top and bottom of a straight hill, the
angle of depression and elevation of the top of a pillar
o
Q132. Value of (cos 53° – sin 37°) is of 10 m. height are observed to be 60 and 30 o
(a) 0 (b) 1 respectively. The height (metres) of the hill is
(c) 2 sin 37° (d) 2 cos 53° (a) 40 (b) 30
Ans: (a) cos 53° – sin 37° cos 53° – sin (90° – 53°) (c) 80 (d) 60
cos53° – cos53° = 0 Ans: (a)

Q133. Two men are on opposite sides of a tower. They Q137. The value of
measure the angles of elevation of the top of the towe r sin22o + sin24o + sin26o +..........+ sin290o is
as 30° and 45° respectively. If the height of the tower (a) 0 (b) 22
is 50 m, the distance between the two men is (Take √3 (c) 23 (d) 44
= 1.7) Ans: (c) sin2 2° + sin 2 4° + sin 2 6° + ... + sin 2 86° + sin 2
(a) 136.5 m (b) 50 √3 m 88° + sin2 90°
(c) 100 √3 m (d) 135.5 m sin2 2° + sin2 40° + sin2 6° ... sin2 (90° – 4°) + sin2 (90°
Ans: (a) In ∆ ADC – 2°) + sin2 90°
AC = 50 m sin2 2° + sin2 4° + sin2 6° + ... + cos2 4° + cos2 2° +
sin2 90°
(sin2 2° + cos2 2°) + (sin2 4° + cos2 4°) + (sin2 6° +
cos2 6°) + sin2 90°
√ (1 + 1 + .... 22 terms) + sin2 90°
DC= √ 22 + 1 = 23

Q138. What is the simplified value of ( ) ?


(a) tan B cot A (b) tan A cot B
(c) tan A tan B (d) cot A cot B

Ans: (a)

in ∆ ABC

Page 154 of 516


https://telegram.me/aedahamlibra
(c) cot A
Ans: (c) cosec 2A + cot 2A
(d) cot2A
Q139. If then the value of is
(a)
(c)
Ans: (d) tan θ + sec θ = 2 (Given) ... (i)
tan2 θ – sec2 θ = –1 (Identity) cotA
(tan θ – sec θ) (tan θ +sec θ) = 1
tan θ – sec θ = ... (ii) Q145. What is the simplified value of 1 + tan A tan
Adding eqn. (i) and (ii) (A/2)?
2 tan (a) sin A/2 (b) cos A
(c) sec A (d) sin A
Ans: (c) ( ) ?
( )
Q140. If cos 2 θ – sinθ = 1/4, then what is the value of
( )
sin θ?
( ) ( )
(a) –1 (b) 1/2
( ) ( )
(c) 1 (d) 3/2
Ans: (b) ( )

= = sec A
Q141. What is the simplified value of ?
(a) sec A + tan A (b) (1 – cos A) / (1 + cos
Q146. What is the least value of tan 2θ + cot2θ + sin2θ +
A)
cos2θ + sec2θ + cosec2θ?
(c) (1 – cos A) / (1 + cosec A)
(a) 1 (b) 3
(d) sin A
(c) 5 (d) 7
Ans: (c) ( ) Ans: (d) tan2θ + cot2θ + sin2θ + cos2θ + sec2θ + cosec2θ
sin2θ + cos2θ + tan2θ + 1 + tan2θ + cot2θ + 1 + cot2θ
( ) 1 + 1 + 1 + 2 tan2θ + 2cot2θ
3 + 2 (tan2θ + cot2θ)
3+2×2 3+4=7

Q147. If A = 30°, B = 60° and C = 135°, then what is


the value of sin 3A + cos 3B + tan 3C – 3sin A cos B tan
Q142. What is the simplified value of √ ?? C?
2 2 (a) 0 (b) 1
(a) 1 – cos A (b) 2sin A
(c) (c) 8 (d) 9
sec2 A (d) cosec2A
Ans: (a) Here,
Ans: (a)
A = 30°, B = 60° and C = 135°
then,
Q143. What is the simplified value of ( ) ? sin3A + cos3B + tan3C – 3sinA cos B tan C
(a) tan A (b) cot A sin330° + cos360° + tan3135° – 3 sin 30° cos60° tan 135°
(c) sin A (d) cos A
( ) ( )
Ans: (a) ?
tanA

Q144. What is the simplified value of cosec 2A + cot


2A? Q148. What is the simplified value of
(a) sec A (b) sec (A/2) (a) cosec A – cot A (b) sec A – tan A
Page 155 of 516
(c) sec2A https://telegram.me/aedahamlibra
(d) sec A cosec A (a) cosec A/ (1 + sin A)
(c) cos A (1 + sin A)
(b) cos A/ (1 + sin A)
(d) cosec A (1 + sin A)
Ans: (a) √

Ans: (b)
then,

cosec A – cot A.

Q149. What is the simplified value of cosec 6A – cot6A


– 3 cosec2A cot2A?
(a) – 2 (b) – 1
(c) 0 (d) 1 Q154. If sec θ+ cosec √ = θsec (90 – θ) then what is
Ans: (d) cosec6A – cot6A – 3 cosec2 A cot2A the value of cot √ ?
(cosec2A)3 – (cot2A)3 – 3 cot2A. cosec2A
(a) √ (b) 2
[ (cosec2A – cot2A) ( (cosec2A)2 + (cot2A)2
(c) √ (d) θ
+ cosec2A cot2A)] (–3 cot2A. cosec2A)
Ans: (d)
[1 (cosec2A)2 + (cot2A)2 – 2 cosec2A cot2A
+ 2 cosec2A cot2A+ cosec2A cot2A)] – 3
Q155. What is the simplified value of θ?
cot2A cosec2A
(a) cosecθ+ tanθ (b) sinθ+ cosθ
[ (cosec2A – cot2A)2 + 3 cosec2A cot2A]
(c) (1 – cosθ) / (1 + cosθ) (d) (1 – sin θ) / (1 + sinθ)
– 3 cosec2A cot2A
Ans: (c)
(cosec2A – cot2A)2
= (1) 2 = 1.
Q156. If and √ then what is the value of
Q150. If tan (A/2) = x, then the value of x is x?
(a) sin A/ (1 – cosA) (b) sin A/ (1 + cosA) √
(a) (b) 1/2
(c) [sin A/ (1 + cosA)] (d) [sin A/ (1 – cosA)]
(c) θ (d) 5
Ans: (b) ( ) Ans: (d)


Q157. If + = πθ and cos , then what is the

Q151. If tan A = 1/2 and tan B = 1/3, then what is the value of sin ?

value of tan (2A + B)? (a) 0 (b) 1/2 (c)
(a) 1 (b) 3 (d) 1
(c) 5 (d) 9 √ √
Ans: (d) cos θ
Ans: (b)
∴cosθ = cos30°
∴θ = 30°
Q152. If sec √ = x, then value of x is
Now,
(a) √ √
(c) – 1 (d) ( )
ϕ = 120° – 30° = 90°
Ans: (b) ( ) ∴sin ϕ = sin 90° = 1.
( )
= – sec (180° + 45°) = – sec 45° = √ Geometry

Q1. ABCD is a quadrilater al in which diagonal BD =
Q153. If 2sec A – (1 + sinA)/cos A = x, then the value 64 cm,AL ⊥ BD and CM ⊥ BD, such that AL = 13.2
of x is

Page 156 of 516


https://telegram.me/aedahamlibra
cm and CM = 16.8 cm. The area of the quadrilateral
ABCD in square centimetres is
(a) 537.6 (b) 960.0
(c) 422.4 (d) 690.0
Ans: (b)

In ∆ABC,
∠A + ∠B + ∠C = 180°
∠A + 60° + 40° = 180°
∠A = 180° – 60° – 40° = 80°
AD bisects ∠BAC
∴ ∠A = ∠BAD + ∠DAC
Given : ∠BAD = ∠DAC = 40°
BD = 64 cm Now, In ∆ABE
AL = 13.2 cm ∠B + ∠E + ∠BAE = 180°
CM = 16.8 cm 60° + 90° + ∠ BAE = 180°
So, Area (ABCD) = Area (∆ABD) + Area (∆BCD) ∠BAE = 30°
∴ ∠EAD = ∠BAD – ∠BAE = 40° – 30° = 10°
= × AL × BD + × CM × BD
= × BD × (AL + CM) = (13.2 + 16.8) Q4. G is the centroid of ∆ABC. If AG = BC, then
= 32 × 30 = 960 cm2
∠BGC is
(a) 60° (b) 120°
Q2. In the figure below, if AB || CD and CE ⊥ ED, (c) 90° (d) 30°
then the value of x is Ans: (c)

(a) 37 (b) 45 AG = BC (Given)


(c) 53 (d) 63 BD = DC (given) AD is median
Ans: (c) ∠AEC = ∠ECD (Alternate interior angles as AB So, GD = BD = DC
|| CD) ∆BCD & ∆GCD are both isosceles ∆.
In ∆CED, Then ∠BGC = 90°
∠ECD + ∠CEd + x° = 180°
(Sum of angles of ∆s are 180°) Q5. PA and PB are two tangents drawn from an
37° + 90° + x° = 180° external pointP to a circle with centre O where the
x° = 180° – 37° – 90° points A and B are the points of contact. The
x° = 53° quadrilateral OAPB must be
(a) a square (b) concylic
Q3. In ∆ABC, ∠B = 60°, ∠C = 40°. If AD bisects (c) a rectangle (d) a rhombus
∠BAC andAE ⊥ BC, then ∠EAD is Ans: (b)
(a) 40° (b) 80°
(c) 10° (d) 20°
Ans: (c)

Page 157 of 516


https://telegram.me/aedahamlibra
Q8. If ∆ABC is an isosceles triangle with ∠C = 90º and
AC = 5 cm, then AB is :
(a) 5 cm (b) 10 cm (c)

OAPB is concyclic because ∠A + ∠B = 180°


& ∠O + ∠P = 180°

Q6. If in ∆ABC, ∠A = 90°, BC = a, AC = b and AB = c,


then the value of tan B + tan C is
cm (d) 2.5 cm
(a) Ans: (c) ∆ABC is an isocoles triangle.
(c) Therefore, AC = BC = 5 cm

Ans: (b)
In right angled ∆ABC,
tan B =
tan C =
Now, AB2 = AC2 + BC2
tan B + tan C =
AB2 = 52 + 52 √ √ cm
[ ]
Q9. ABC is a right angled triangle, right angled at C
and p is the length of the per pendicular from C on
Q7. In the following figure, if OA = 10 and AC = 16,
AB. If a, b and c are the lengths of the sides BC, CA
then OB must be
and AB respectively, then
(a)
(a) 3 (b) 4 (c) √
(c) 5 (d) 6
Ans: (b) Here,
Ans: (d)
∠ACB = 90°
∠ADC = 90°
∠BDC = 90°

Triangles ACB, ADC and BDC are right angle triangles.


In OAB, Here, Area of ∆ABC = Area of ∆ADC + Area of ∆BDC
OA2 = OB2 + AB2
[∴ AB = AC, because line drawn from centre to a chord ab = p (AD + DB)
bisect & perpendicular to it] ab = pc c= … (1)
(10)2 = (OB)2 + (8) 2
Now, In ∆ABC,
100 – 64 = OB2
OB2 = 36 C2 = a2 + b2( )
OB = 6

Page 158 of 516


https://telegram.me/aedahamlibra
OY are the internal and external bisectors of
respectively, then ∠BOY is
∠AOC

(a) 70° (b) 80°


Q10. The length of radius of a circumcircle of a (c) 72° (d) 68°
triangle having sides 3 cm, 4 cm and 5 cm is: Ans: (a)
(a) 2 cm (b) 2.5 cm
(c) 3 cm (d) 1.5 cm
Ans: (b) Circumradius of a triangle
=

=

= cm OX is the bisector of ∠AOC.



∴ ∠AOC = 2 ∠COX
Q11. The length of the two sides forming the right OY is the bisector of ∠BOC.
angle of a right -angled triangle are 6 cm and 8 cm. ∴∠BOC = 2 ∠COY
The length of its circum-radius is : ∴∠AOC + ∠BOC
(a) 5 cm (b) 7 cm = 2∠COY + 2∠COX = 180°
(c) 6 cm (d) 10 cm 2 (∠COX + ∠YOC) = 180°
Ans: (a) In a right angled ∆, the length of circumradius is ∠XOY = 90°
half the length of hypotenuse. ∴∠AOX + ∠XOY + ∠BOY = 180°
∴H2 = 62 + 82 ∴∠BOY = 180° – 90° – 20° = 70°
H2 = 36 + 64 100
H = 10 cm Q14. Two circles of radii 4 cm and 9 cm respectively
Circumradius = 5 cm touch each other externally at a point and a common
tangent touches them at the points P and Q
Q12. In the following figure, O is the centre of the respectively. They the area of a square with one side
circle and XO is perpendicular to OY. If the area of PQ, is
the triangle XOY is 32, then the area of the circle is (a) 97 sq. cm (b) 194 sq. cm
(c) 72 sq. cm (d) 144 sq. cm
Ans: (d)
(a) 64 π (b) 256 π
(c) 16 π (d) 32 π
Ans: (a)

r1 + r2 = 13 cm
r2 – r1 = 9 – 4 = 5 cm
∠XOY = 90°;OX = OY = radices (r) PQ
∴ ∆ XOY is a right angled triangle. √
√ cm
2
r = 2 × 32 = 64 ∴ Area of square = 12 × 12 = 144 sq. cm.

Q15. The side BC of ∆ ABC is produced to D. If
∴ Area of circle = πr2
= 64 π sq. units ∠ACD = 108° and ∠B = ∠A then ∠A is
(a) 36° (b) 72°
Q13. A, O, B are three points on a line segment and C (c) 108° (d) 59°
is a point not lying on AOB. If ∠AOC = 40° and OX, Ans: (b)

Page 159 of 516


https://telegram.me/aedahamlibra
(a) (b) 45°
(c) (d) 30°
Ans: (c) Angle traced by hour hand in an hour = 30°
Angle traced In 2 i.e. hours

∠ACD = ∠ABC + ∠BAC Angle traced by minute hand in 60 minutes = 360°


∠ ∴ Angle traced in 15 minutes

∴ Required angle

Q19. If the length of th e side PQ of the rhombus
Q16. If each int etior angle is double of each exterior PQRS is 6 cm and ∠PQR = 120°, then the length of
angle of a regular polygon with n sides, then the value QS, in cm, is
of n is (a) 4 (b) 6
(a) 8 (b) 10 (c) 3 (d) 5
(c) 5 (d) 6 Ans: (b)
Ans: (d) Let exterior ∠be = x
interior ∠be = 2x
x + 2x = 180
3x = 180
x = 60°
no. of side n
∠PQO
Q17. Two tangents are drawn from a point P to a From ∆POQ,
circle at A and B. O is the centre of the circle. If ∠OPQ = 180° – 90° – 60° = 30°
∠AOP = 60°, then ∠APB is
sin OPQ
(a) 120° (b) 90°
(c) 60° (d) 30° OQ = PQ sin 30°
Ans: (c) ∴ QS = 2 × 3 = 6 cm

Q20. In ∆ABC, AD is the median and AD = (


√ ) BC. If ∠BAD = 30°, then measure of
∠ACB is
(a) 90° (b) 45°
In right ∆s OAP and OPB, (c) 30° (d) 60°
AP = PB, OA = OB Ans: (d)
OP = OP

∴ ∠AOP = ∠POB and ∠APO


= ∠OPB
From ∆ AOP,
∠APO = 180° – 90° – 60° = 30°
∴ ∠APB = 2 × 30 = 60°
BD = DC = AD
Q18. The angle formed by the hour –hand and the ∠BAD = 30°
minute–hand of a clock at 2 : 15 p.m. is From ∆ ABD,
Page 160 of 516
∠BAD = 30°
∴ ∠ABD = ∠BAD = 30°
https://telegram.me/aedahamlibra ( √ )

∴ ∠ADB = 180° – 2 × 30° = 120° ( √ ) sq.units


∴ ∠ADC = 180° – 120° = 60°
∴ AD = DC Q24. By decreasing 15° of each angle of a triangle, the
∠DAC = ∠ACD = 60° ratios of their angles are 2 : 3 : 5. The radian measure
of greatest angle is:
Q21. Two sides of a triangle are of length 4 cm and 10 (a) 11π/24 (b) π/12
cm. If the length of the third side is 'a' cm. then (c) π/24 (d) 5π/24
(a) a > 5 (b) 6 ≤ a ≤ 12 Ans: (a) 2x + 3x + 5x = 180° – 45° = 135°
(c) a < 5 (d) 6 < a < 14 10x = 135°
Ans: (d) The sum of any two sides of a triangle is greater
than third side and their difference is less than third side.
10 – 4 <a < 10 + 4 ∴ Largest angle
6 < a < 14
( )
Q22. ∆ABC and ∆DEF are similar and their areas be
respectively 64 cm 2 and 121 cm 2. If EF = 15.4 cm, BC
is: radian
(a) 12.3 cm (b) 11.2 cm RADIAN
(c) 12.1 cm (d) 11.0 cm
Ans: (b) Q25. If G is the centroid of ∆ABC and AG = BC, then
∠BGC is:
(a) 75° (b) 45°
(c) 90° (d) 60°
Ans: (c) In ∆ ABC
Given AG = BC
Q23. The perimeter of an isosceles, right -angled
triangle is 2p unit. The area of the same triangle is:
(a) ( √ ) unit (b) ( √ ) ̇
(c) ( √ ) unit (d) PED ABC
Ans: (a)

AG = BC
i.e., GD = BD = DC
In ∆ BGD
BD = DG ∴ ∠GBD = ∠DGB… (i)
In ∆ CGD
AB = BC = x GD = DC, ∴ ∠GCD = ∆DGC… (ii)
∠GBD + ∠DGB + ∠DGC + ∠DCG = 180
∴ √ √
2 (∠BGD + ∠CGD) = 180
√ units
∠BGC =

( √ )
Q26. D and E are the mid -points of AB and AC of
( √ ) ∆ABC; BC is produced to any point P; DE, DP and
√ ( √ )( √ ) EP are joined. Then,
( √ ) (a) (b) ∆PED = ∆BEC
( √ )
(c) ΓBDE = ΓBEC (d) √
∴ Area of triangle = Ans: (a)
Page 161 of 516
https://telegram.me/aedahamlibra
Ans: (b)

DE | | BC
DE BC
∴ ∆ BDE = ∆ DEP 15 20 25
2∆ BDE = ∆ BEC 3 : 4 : 5 and forming at ∆
∴ ∆ ADE = ∆ BDE
or
∴ ∆ ABC = 4 ∆ ADE
12 = x
..PED ABC
chord = 2 × 12 = 24 cm

Q27. O is the circum centre of t he triangle ABC with Q30. P is a point outside a circle and is 13 cm away
circumradius 13 cm. Let BC = 24 cm and OD is from its centre. A secant drawn from the point P
perpendicular to BC. Then the length of OD is: intersect the circle at points A and B in such a way
(a) 7 cm (b) 3 cm that PA = 9 cm and AB = 7 cm. The radius of the
(c) 4 cm (d) 5 cm circle is:
Ans: (d) (a) 5.5 cm (b) 5 cm
(c) 4 cm (d) 4.5 cm
Ans: (b)

cm OC ⊥ AB
OB = 13 cm AC = BC = 3.5 cm OP = 13 cm
From ∆ OBD, PC = 9 + 3.5 = 12.5 cm
√ √
√ √ √ cm √ √
OA √ √
Q28. AB is a diameter of a circle with centre O. CD is
√ √
a chord equal to the radius of the circle. AC and BD
are produced to meet at P. Then the measure of ∠APB Q31. R and r are the radius of two circles (R > r). If
is: the distance between the centre of the two circles be d,
(a) 120° (b) 30°
then length of common tangent of two circles is:
(c) 60° (d) 90°
(a) √ √
Ans: (c)
(c) √ (d)
Q29. The length of the common chord of two circles of
radii 15 cm and 20 cm whose centres are 25 cm apart
is (in cm):
(a) 20 (b) 24
(c) 25 (d) 15
Page 162 of 516
https://telegram.me/aedahamlibra
Q36. AC and BC are two equal chords of a circle. BA
is produced to any point P and CP, when joined cuts
the circle at T. Then
(a) CT : TP = AB : CA (b) CT : TP = CA : AB
(c) CT : CB = CA : CP (d) CT : CB = CP : CA
Ans: (c) It is based on fundamental concept.

Ans: (b) Length of common tangent Q37. If S is the circumcentre of ∆ABC and ∠A = 50°,
√ then the value of ∠BCS is
(a) 20° (b) 40°
Q32. In an ob tuse-angled triangle ABC, ∠A is the (c) 60° (d) 80°
obtuse angle and O is the orthocenter. If ∠BOC = 54°, Ans: (b)
then ∠BAC is
(a) 108° (b) 126°
(c) 136° (d) 116°
Ans: (b) ∠BAC = 180° – ∠BOC = 180° – 54° = 126°

Q33. The perimeters of two similar triangle ∆ABC


and ∆PQR are 36 cm and 24 cm respectively. If PQ =
10 cm, then AB is:
(a) 25 cm (b) 10 cm
(c) 15 cm (d) 20 cm ∠BAC = 50°
∴ ∠BSC = 100°
Ans: (c)
BS = SC = radius
∴ ∠BCS

Q38. √
cm BC is the chord of a circle with centre O. A is a point
on major arc BC as shown in the above figure. What
is the value of ∠BAC + ∠OBC ?
Q34. Let BE and CF the two medians of a ∆ABC and
(a) 120° (b) 60°
G be their intersection. Also let EF cut AG at O. Then
(c) 90° (d) 180°
AO: OG is
Ans: (c)
(a) 1 : 1 (b) 1 : 2
(c) 2 : 1 (d) 3 : 1
Ans: (c) AF = FB
AE = EC
∴ FE||BC BC
∴ AO : OG = 2 : 1

Q35. If the ratio of areas of two similar triangles is9 :


16, then the ratio of their corresponding sides is
(a) 3 : 5 (b) 3 : 4 ∠BOC = 2∠BAC
(c) 4 : 5 (d) 4 : 3 OB = OC
Ans: (b) Ratio of corresponding sides ∴ ∠OBC = ∠OCB



= 90° – ∠BAC
∴ ∠BAC + ∠OBC
= 90° – ∠ΒAC + ∠BAC = 90°

Page 163 of 516


https://telegram.me/aedahamlibra
Q39. PQ is a direct common tangent of two circles of
radii r 1 and r 2 touching each other externally at A.
Ans: (a) ∠BPC = 90° –
= 90° – 40° = 50°
Then the value of P
Q2 is
Q44. The radius of the circumcircle of the triangle
(a) r1r2 (b) 2r1r2
made by x-axis, y-axis and 4x + 3y = 12 is
(c) 3r1r2 (d) 4r1r2
(a) 2 unit (b) 2.5 unit
Ans: (d) PQ2 = (r1 + r2)2 – (r1 – r2)2 = 4r1r2
(c) 3 unit (d) 4 unit
Ans: (b) Putting x = 0 in 4x + 3y = 12 we get y = 4
Q40. If I is the In -centre of ∆ABC and ∠A = 60°, then
Putting y = 0 in 4x + 3y = 12 we get x = 3
the value of ∠BIC is The triangle so formed is right angle triangle with points
(a) 100° (b) 120° (0, 0) (4, 0) (0, 3)
(c) 150° (d) 110°
So diameter is the hypotenus of triangle √ =5
Ans: (b) ∠BIC = 90° + unit
= 90° + 30° = 120° Radius = 2.5 unit

Q41. Two circles with radii 5 cm and 8 cm touch each Q45. A rail road curve is to be laid out on a circle.
other externally at a point A. If a straight line through What radius should be used if the track is to change
the point A cuts the circles at points P and Q direction by 25° in a distance of 40 metres?
respectively, then AP : AQ is (a) 91.64 metres (b) 90.46 metres
(a) 8 : 5 (b) 5 : 8 (c) 89.64 metres (d) 93.64 metres
(c) 3 : 4 (d) 4 : 5 Ans: (a) θ = 25° radians
Ans: (b)
radians

= 91.64 metre
∴ AP : AQ = 5 : 8
Q46. If D is the mid -point of the side BC of ∆ ABC
Q42. When a pendulum of length 50 cm oscillates, it and the area of ∆ ABD is 16 cm2, then the area of ∆
produces an arc of 16 cm. The angle so formed in ABC is
degree measure is (approx) (a) 16 cm2 (b) 24 cm2
2
(a) 18°25′ (b) 18°35′ (c) 32 cm (d) 48 cm2
(c) 18°20′ (d) 18°08′ Ans: (c) Area of ∆ABD = 16 cm2
Ans: (c) s = 16 cm Area of ∆ABC = 2 × Area of ∆ABD [
r = 50 cm
radian In triangle, the midpoint of the opposite side, divides
it into two congruent triangles. So their areas are equal
and each is half the area of the original triangle]
32 cm2

Q47. The length of the circum -radius of a triangle


( ) having sides of lengths 12 cm, 16 cm and 20 cm is
(a) 15 cm (b) 10 cm
Q43. The external bisectors of ∠B and ∠C of ∆ABC (c) 18 cm (d) 16 cm
meet at point P. If ∠BAC = 80°, then ∠BPC is Ans: (b) Circum Radius
(a) 50° (b) 40°
[where a, b and c are sides of triangle]
(c) 80° (d) 100°
Area of Triangle = √
Page 164 of 516
[∴ ] https://telegram.me/aedahamlibra
Area of Triangle √ = 8 × 3 × 4 cm2
cm

Q48. If P, R, T are the area of a parallelogram, a


In ∆AOM
rhombus and a triangle standing on the same base
r2 = AM2 + x2
and between the same parallels, which of the following
AM2 = r2 – x2 ... (1)
is true?
In ∆AMO'
(a) R < P < T (b) P > R > T
r2 = (r – x)2 + AM2
(c) R = P = T (d) R = P = 2T
AM2 = r2 – (r – x)2 ... (2)
Ans: (d) Parallelogram Area = l × b
From eqn. (1) & (2)
Rhombus Area = l × b
r2 – x2 = r2 – (r – x)2
Triangle Area 2rx = r2
Therefore R = P = 2T.

Q49. ABC is a triangle. The medians CD and BE From eq. (1)


intersect each other at O. Then ∆ ODE : ∆ ABC is √
( ) AM =
(a) 1 : 3 (b) 1 : 4

(c) 1 : 6 (d) 1 : 12 Length of chord AB = 2AM = √
Ans: (d) Area of
Q51. AB is a diameter of the circumcircle of ∆ APB; N
( ) is the foot of the perpendicular drawn from the point
P on AB. If AP = 8 cm and BP = 6 cm, then the length
of BN is
(a) 3.6 cm (b) 3 cm
(c) 3.4 cm (d) 3.5 cm
Ans: (a) Since AB is a diameter. Then ∠APB = 90°
(angle in the semicircle)
∆BPN ~ ∆APB
So, BN = BP2 / AB
BN =
[ ]
Q52. Two circles intersect each other at P and Q. PA
[ ( )]
and PB are two diameters. Then ∠AQB is
* += area of ∆ABC = 1 : 12 (a) 120° (b) 135°
(c) 160° (d) 180°
Q50. Two circles with same radius r intersect each Ans: (d)
other and one passes through the centre of the other.
Then the length of the common chord is Q53. The bisector of ∠A of ∆ABC cuts BC at D and
√ the circumcircle of the triangle at E. Then
(a) r (b) (a) AB : AC = BD : DC (b) AD : AC = AE : AB
(c) √ (c) AB : AD = AC : AE (d) AB : AD = AE : AC
Ans: (b) Ans: (d)

Q54. A and B are centres of the two circles whose


radii are 5 cm and 2 cm respectively. The direct
common tangents to the circles meet AB extended at
P. Then P divides AB.

Page 165 of 516


:5
https://telegram.me/aedahamlibra
(a) externally in the ratio 5 : 2 (b) internally in the ratio 2
Q57. A wheel rotates 3.5 times in one second. What
(c) internally in the ratio 5 : 2 (d) externally in the ratio 7 time (in seconds) does the wheel take to rotate 55
:2 radian of angle?
Ans: (a) Externally in the ratio of 5 : 2 (a) 1.5 (b) 2.5
(c) 3.5 (d) 4.5
Q55. O is the centre of the circle passing through the Ans: (b) Radian covered in one second
points A, B and C such that ∠BAO = 30°, ∠BCO = 40°
Time required to covered 55 radian
and ∠AOC = x°. What is the value of x ?
(a) 70° (b) 140°
(c) 210° (d) 280° Q58. In the following figure. AB be diameter of a
Ans: (b) In ∆AOB circle whose centre is O. If ∠AOE = 150°. ∠DAO = 51°
AO = BO (radii of circles) then the measure of ∠CBE is:
∴ ∠ABO = ∠BAO = 30º
In ∆BOC
(a) 115° (b) 110°
(c) 105° (d) 120°
Ans: (c)

BO = CO (radii of circles)
∴ ∠BCO = ∠OBC = 40º
∠ABC = ∠ABO + ∠OBC
∠ABC = 30° + 40° = 70°
2 × ∠ABC = ∠AOC x° = 140 ∠AOE = 150°
∠DAO = 51°
Q56. If area of an equilateral triangle is a and height ∠EOB = 180° – 150° = 30°
b, then value of is: OE = OB
(a) 3 (b) √ ∴ ∠OEB = ∠OBE = = 75°
(c) (d) ∴ ∠CBE = 180° – 75° = 105°

Q59. Triangle PQ R circumscribes a circle with centre


O and radius rcm such that ∠PQR = 90°. If PQ = 3
cm, QR = 4 cm, then the value of r is :
(a) 2 (b) 1.5
(c) 2.5 (d) 1
Ans: (d) PR2 = PQ2 + PR2 = 32 + 42 = 25
∴ PR = √ = 5 cm
Ans: (c) Let side of triangle = x

∴ = a... (i)

and =b
x= ... (ii)

Putting x in equation (i)

( )

r=

Page 166 of 516


=
https://telegram.me/aedahamlibra
Q60. In a triangle ABC, BC is produced to D so that
CD = AC. If ∠BAD = 111° and ∠ACB = 80°, then the ∠A + ∠B = 145°
measure of ∠ABC is: ∠C + 180° – 145° = 35°
(a) 31° (b) 33° ∠C + 2∠B = 180°
(c) 35° (d) 29° 2∠B = 180° – 35° = 145°
Ans: (d) ∠ACB = 80°
∠B = = 72.5 ° = ∠A
∠ACD = 180° – 80° = 100°
∠B > ∠C
∴ ∠CAD = ∠CDA
∴ CA > AB

Q64. 360 sq. cm and 250 sq. cm are the areas of two
similar triangles. If the length of one of the sides of the
first triangle be 8 cm, then the length of the
corresponding side of the second triangle is
(a) 6 cm (b) cm
=
(c) cm (d) (√ ) cm
∠BAC = 111° – 40°
= 71° Ans: (d) ( ) ,( ) ( )
∠ABC = 180° – 71° – 80° = 29°
x= cm

Q61. The areas of two similar triangles ABC and DEF


are 20 cm2 and 45 cm2 respectively. If AB = 5 cm. then Q65. A chord 12 cm long is drawn in a circle of
DE is equal to : diameter 20 cm. The distance of the chord from the
(a) 6.5 cm (b) 7.5 cm centre is
(c) 8.5 cm (d) 5.5 cm (a) 16 cm (b) 8 cm
(c) 6 cm (d) 10 cm
Ans: (c) Ans: (b) Given, AB = 12 cm; CD = 20 cm
OE = ?
DE2 =
∴ DE = = 7.5 cm

Q62. From a point P, two tangents PA and PB are


drawn to a circle with centre O. If OP is equal to
diameter of the circle, then ∠APB is
(a) 60° (b) 45°
(c) 90° (d) 30° Now, AE = EB = 6cm (The line drawn from centre of
Ans: (a) circle to the chord bisect the chord)
In ∆OAE, By phythagoras theorem
Q63. In ∆ABC. ∠A + ∠B = 145° and ∠C + 2∠B = 180°. (OA)2 = (OE)2 + (AE)2 (10)2 = (OE)2 + (6) 2
State which one of the following relations is true? 100 – 36 = (OE)2 = 64 = OE2 OE = 8cm
(a) CA = AB (b) CA < AB
(c) BC > AB (d) CA > AB Q66. The perpendiculars, drawn from the vertices to
Ans: (d) the opposite sides of a triangle, meet at the point
whose name is
(a) orthocentre (b) incentre
(c) circumcentre (d) centroid
Ans: (a)
Page 167 of 516
https://telegram.me/aedahamlibra
Q67. If in ∆ ABC, ∠ABC = 5∠ACB and ∠ BAC = 3
= 17 cm

∠ACB, then ∠ABC = Q71. In a triangle ABC, AB = AC, ∠BAC = 40°. Then
(a) 120° (b) 130° the external angle at B is :
(c) 80° (d) 100° (a) 80° (b) 90°
Ans: (d) ∠ ∠ ∠ (c) 70° (d) 110°
∠ ∠ ∠ Ans: (d)


Q68. A chord AB of a circle C 1 of radius (√ )cm


touches a circle C 2 which is concentric to C 1. If t he
radius of C2 is √ cm, the length of AB is:
(a) √ cm (b) √ cm ∠ABC = ∠ACB
∠BAC = 40°
(c) √ cm (d) cm
∴ ∠ABC + ∠ACB = 140°
Ans: (d)
∴ ∠ABC = 70°
∴ ∠ABD = 180° – 70° = 110°
Q69. If ∆ABC is similar to ∆DEF such that BC = 3
cm, EF = 4 cm and area of ∆ABC = 54 cm 2, then the
Q72. P and Q are two points on a circle with centre at
area of ∆DEF is :
O. R is a point on the minor arc of the circle, between
(a) 54 cm2 (b) 66 cm2
2 the points P and Q. The tangents to the circle at the
(c) 78 cm (d) 96 cm2
points P and Q meet eahc other at the point S. If
Ans: (d)
∠PSQ = 20°, ∠PRQ = ?
(a) 100° (b) 80°
(c) 200° (d) 160°
Ans: (a)

Q73. If ABCD be a rectangle and P, Q, R, S be the


∆ABC ~ ∆DEF
mid points of and respectively, then the area of

the quadrilateral PQRS is equal to:
∆DEF = = 96 sq. cm2. (a) area (ABCD) (b) area (ABCD)
(c) area (ABCD) (d) √ area (ABCD)
Q70. A chord of length 30 cm is at a distance of 8 cm
from the centre of a circle. The radius of the circle is : Ans: (a)
(a) 19 (b) 17
(c) 23 (d) 21
Ans: (b)

ar ∆ PSR = APRD.... (i)


as ∆ PQR = PBCR.... (ii)
Adding Both eq.
ar ∆ PSRQ = ABCD
AD = 15 cm
OD = 8 cm
OA = √ =√ √

Page 168 of 516


https://telegram.me/aedahamlibra
Q74. ABC is an isosceles triangle such that AB = AC
and ∠B = 35°. AD is the median to the base BC. Then Q78. In a circle of radius 17 cm, two parallel chords of
∠BAD is: lengths 30 c m and 16 cm are drawn. If both the
(a) 55° (b) 70° chords are on the same side of the centre, then the
(c) 35° (d) 110° distance between the chords is
Ans: (a) AB = AC (a) 9 cm (b) 7 cm
∴ ∠ABC = ∠ACB = 35° (c) 23 cm (d) 11 cm
∴ ∠ADB = 90° Ans: (b)
∴ ∠BAD = 55°

Q75. AB and CD are two parallel chords of a circle


such that AB = 10 cm and CD = 24 cm. If th e chords
are on the opposite sides of the centre and distance
between them is 17 cm, then the radius of the circle is:
(a) 10 cm (b) 11 cm
(c) 12 cm (d) 13 cm
Ans: (d)
AE = 15 cm
OA = 17 cm
∴ OE = √
=√ =√ = 8 cm
CF = 8 cm
OC = 17 cm
∴ OF = √
=√ =√
AB = 10 cm, AE = 5 cm = 15 cm
OE = x = 15 –8 = 7 cm
CD = 24 cm, DF = 12 cm
OF = 17 – x Q79. AB is the chord of a circle with centre O and
OA = OD DOC is a line segment originating from a point D on
52 + x2 = 122 + (17 – x)2 the circle and intersecting, AB produced at C such
25 + x2 = 144 + 289 – 34x + x2 that BC = OD. If ∠BCD = 20°, then ∠AOD = ?
34x = 408 (a) 20° (b) 30°
(c) 40° (d) 60°
x= = 12
Ans: (c)
∴ OA = √ = 13 cm

Q76. ABC is a triangle. The bisectors of the internal


angle ∠B and external angle ∠C intersect at D. If
∠BDC = 50°, then ∠A is
(a) 100° (b) 90°
(c) 120° (d) 60°
Ans: (a)

Q77. ABCD is a cyclic trapezium with AB || DC and


BC = DO = OA
AB = and diameter of the circle. If ∠CAB = 30° then
∠DAB = 90°
∠ADC is
∠DOA = 2 × DCA = 40°
(a) 60° (b) 120°
(c) 150° (d) 30°
Ans: (b)

Page 169 of 516


https://telegram.me/aedahamlibra
Q80. In a triangle ABC, the side BC is extended up to
D. Such that CD = AC, if ∠BAD = 109° and ∠ACB =
(a) √ √
(c) √
72° then the value of ∠ABC is
Ans: (d)
(a) 35° (b) 60°
(c) 40° (d) 45°
Ans: (a)

∠ACD = 180° – ∠ACB


= 180° – 72° = 108° O′A = 3 cm
OA = 2 cm
∠CAD = ∠ADC = = 36°
CA = 6 cm
∴ ∠ABC = 180° – 109° – 36° = 35° O′D = 3 cm
O′B = 1 cm
Q81. ABC is a right angled triangle, B being the right
BD = √ √
angle. Mid -points of BC and AC are respectively B′
DE = √ cm
and A′. The ratio of the area of the quadrilateral AA′
B′B to the area of the triangle ABC is
(a) 1 : 2 (b) 2 : 3 Q84. If the arcs of square length in two circles
(c) 3 : 4 (d) None of the above subtend angles of 60° and 75° at their centres, the
Ans: (c) ratio of their radii is
(a) 3 : 4 (b) 4 : 5
(c) 5 : 4 (d) 3 : 5
Q82. ABCD is a cyclic quadrilateral AB and DC are
produced to meet at P. If ∠ADC = 70° and ∠DAB = Ans: (c) [When θ = 2π]
60°, then the ∠PBC+ ∠PCB is s = rθ
(a) 130° (b) 150° s = r1θ1 = r2θ2
(c) 155° (d) 180°
Ans: (a)

Q85. From a point P which is at a distance of 13 cm


from center O of a circle of radius 5 cm, in the same
plane, a pair of tangents PQ and PR are drawn to the
circle. Area of quadrilateral PQOR is
(a) 65 cm2 (b) 60 cm2
2
(c) 30 cm (d) 90 cm2
Ans: (b)
∠ADC = 70°
∠ABC = 180° – 70° = 110°
∠PBC = 70°
∠BCD = 180° – 60° = 120°
∠PCB = 60°
∴ ∠PBC + ∠PCB = 70° + 60° = 130°

Q83. Two circles touch each other internally. Their


radii are 2 cm and 3 cm. The biggest chord of the ∠OQP = ∠ORP = 90°
greater circle which is outside the inner circle of PQ = √
length. =√ = 12
Page 170 of 516
∴ https://telegram.me/aedahamlibra 52 – (49 – 14x + x2) = 144 – (49 + 14x + x2)
52 – 49 + 14x – x2 = 144 – 49 – 14x – x2
PQOR = 2 × ∆OPQ
28x = 144 – 52 = 92
=
= 60 sq. cm2
∴ BN = 7 + x
Q86. In a triangle ABC, ∠A = 90°, ∠C = 55°, . cm
What is the value of ∠BAD?
(a) 45° (b) 55° Q88. The diagonals AC and BD of a cyclic
(c) 35° (d) 60° quadrilateral ABCD intersect each other at the point
Ans: (b) P. Then, it is always true that
(a) AP . BP = CP . DP (b) AP . CD = AB . CP
(c) BP. AB = CD. CP (d) AP . CP = BP . DP
Ans: (d)

∠A = 90°, ∠C = 55°
∴ ∠B = 90° – 55° = 35°
∠ADB = 90°
∴ ∠BAD = 90° – 35° = 55° Here, AC and BD are chords of the circle.
∴ AP . CP = BP . DP
Q87. N is the foot of the perpendicular from a point P
of a circle with radius 7 cm, on a diameter AB of the Q89. If G is the centroid of ∆ABC and ∆ABC = 48cm2,
circle. If the length of the chord PB is 12 cm, the then the area of ∆BGC is
distance of the point N from the point B is (a) 16 cm2 (b) 24 cm2
2
(a) cm (b) (c) 32 cm (d) 8 cm2
Ans: (a)
(c) cm (d) ⊥
Ans: (b)

AB = 14 cm, PB = 12 cm ∆BGC
∠APB = 90°
AP √
√ √ √
Q90. A vertical stick 12 cm long casts a shadow 8 cm
ON = x ∴ AN = 7 – x ; BN = 7 + x long on the ground. At the same time, a tower casts a
∴ From ∆ PAN, PN2 = AP2 – AN2 shadow 40 m long on the ground. The height of the
∴ 52 – (7 – x)2 tower is
∴ From ∆ PNB (a) 65 m (b) 70 m
PN2 = (12)2 – (7 + x)2 (c) 72 m (d) 60 m
∴ 52 – (7 – x)2 = 144 – (7 + x)2

Page 171 of 516


Ans: (d) https://telegram.me/aedahamlibra
metre

Q91. If O be the circumcentre of a triangle PQR and


∠QOR = 110°, ∠OPR = 25°, then the measure of ∠ACB + ∠ACD = 180º (linear pair)
∠PRQ is ∴ ∠ACB = 180º – 140º = 40º
(a) 55° (b) 60° In ∆ ABC
(c) 65° (d) 50°
∠ BAC + ∠ ABC + ∠ ACB = 180º
Ans: (b)
∠ BAC + 3 ∠ BAC + 40º = 180º
4 ∠BAC = 180º – 40º
Q92. In a triangle, if three altitudes are equal, then
∠ BAC =
the triangle is
(a) Right (b) Isoceles
(c) Obtuse (d) Equilateral Q95. A, B, P are three points on a circle having centre
Ans: (d) Triangle will be equilateral. O. If ∠OAP = 25° and ∠OBP = 35°, then the measure
of ∠AOB is
Q93. A, B, C, D are f our points on a circle. AC and (a) 120° (b) 60°
BD intersect at a point E such that ∠BEC = 130° and (c) 75° (d) 150°
∠ECD = 20°. ∠BAC = 130° and ∠ECD = 20°. ∠BAC is Ans: (a) In ∆OBP.
(a) 100° (b) 110° OB =
(c) 120° (d) 90°
Ans: (b)

∴ ∠ OBP = ∠ OPB = 35º


∴ ∠BEC = 130° In ∆D AOP
∴ ∠DEC = 180° – 130° = 50° OA =
∴ ∠EDC = 180° – 50° – 20° = 110° ∴ ∠ OAP = ∠ OPA = 25º
∴ ∠BAC = ∠EDC = 110° Now, ∠ APB = ∠ OPA + ∠OPB
(Angles on the same arc) = 25º + 35º = 60º
Hence, ∠AOB = 2∠APB
Q94. Side √ of ∆ABC is produced to D. If ∠ACD = (Angle be substended by are at centre is twice)
140° and ∠ABC = 3∠BAC, then find ∠A. = 2 × 60º = 120º
(a) 55° (b) 45°
(c) 40° (d) 35° Q96. ABCD is a cyclic quadrilateral, AB is a diameter
Ans: (d) of the circle. If ∠ACD = 50°, the value of ∠BAD is
(a) 30° (b) 40°
(c) 50° (d) 60°
Ans: (b) In ∆ ABC, ∠ ACB = 90º

Page 172 of 516


https://telegram.me/aedahamlibra
contact Q and R respectively. The relation of TQ and
TR is
(a) TQ < TR (b) TQ > TR
(c) TQ = 2TR (d) TQ = TR
Ans: (d) The relation of TQ and TR is TQ = TR.

∴ ∠ACB + ∠ACD Q100. If the inradius of a triangle with perimeter 32


90º + 50º = 140º cm is 6 cm, then the area of the triangle in sq. cm is
As angle mode by triangle (a) 48 (b) 100
in semicircle is equal to 90º. (c) 64 (d) 96
∴ In quad. ABCD ∠BAD + ∠BCD = 180º Ans: (d) Area of triangle = Inradius × Semi-perimeter
angle of (opp. pair of quad is equal to 180º) = 6 ×16 = 96 sq. cm.
∠ BAD = 180º – 140º = 40º
Q101. D and E are the mid -points of AB and AC of
Q97. The length of tangent (upto the point of contact) ∆ABC.If ∠A = 80°, ∠C = 35°, then ∠EDB is equal to
drawn from an external point P to a circle of radius 5 (a) 100° (b) 115°
cm is 12 cm. The distance of P from the centr e of the (c) 120° (d) 125°
circle is Ans: (b) DE is parallel to BC
(a) 11 cm (b) 12 cm So ∠ AED = ∠C = 35º
(c) 13 cm (d) 14 cm
Ans: (c)

AP is a tangent and OA is a radius. Since ∠A = 80º


Therefore, OA is ⊥ at AP. Then ∠ ADE = 65º
So, In ∆ OAP ∠ EDB is supplement to ∠ADE.
OP2 = 52 + 122 So, ∠ EDB = 180º – ∠ ADE
OP2 = 25 + 144 = 169 = 180º – 65º = 115º
OP = 13 cm
Q102. In ∆ABC, ∠A + ∠B = 65°, ∠B + ∠C = 140°, then
Q98. When two circles touch externally, the number find ∠B.
of common tangents are (a) 40° (b) 25°
(a) 4 (b) 3 (c) 35° (d) 20°
(c) 2 (d) 1 Ans: (b) ∠A + ∠B = 65°
Ans: (b) ∴ ∠C = 180° – 65° = 115°
∠B + ∠C = 140°
∴ ∠B = 140° – 115° = 25°

Q103. The sum of three altitudes of a triangle is


(a) equal to the sum of three sides
(b) less than the sum of sides
(c) greater than the sum of sides
Q99. Two circles of equal radii touch externally at a (d) twice the sum af sides
point P. From a point T on the tangent at P, tangents Ans: 102 (b)
TQ and TR are drawn to the circles with points of

Page 173 of 516


https://telegram.me/aedahamlibra
OA = 5, OB = 4
∴ AB = √
=√ √ = 3 cm

Q106. Two parallel chords of a circle, of diameter 20


cm lying on the opposite sides of the centre are of
lengths 12 cm and 16 cm. The distance between the
AP ∠AB chords is
BQ ∠ BC (a) 16cm (b) 24cm
CR ∠AC (c) 14 cm (d) 20 cm
∴ AP + BQ + CR < AB + BC + AC Ans: (c)

Q104. A cyclic quadrilateral ABCD is such that AB =


BC, AD = DC, AC ⊥ BD, ∠CAD = θ. Then the angle
∠ABC =
(a) θ (b) √
(c) 2 θ (d) 3 θ
Ans: (c)

OA = OC = 10 cm
AB = 12 cm
AP = PB = 6 cm
CD = 16 cm
CQ = QD = 8 cm
From ∆OCQ,
OQ = √ =√ = 6 cm
∠B + ∠D = 180° From ∆OAP
∠A + ∠C = 180° OP = √ =√ = 8 cm
∠BAC + ∠BCA ∴ PQ = 6 + 8 = 14 cm
∠DAC = ∠DCA
∴ ∠DAB = ∠DCB = 90° Q107. The height of an eq uilateral triangle is 15 cm.
∠DAC = θ The area of the triangle is
∴ ∠ADE = 90° – θ = ∠CDE (a) √ sq. cm. (b) √ sq. cm.
∴ ∠ABC = 180° – 2 (90° – θ) = 2θ (c) √ sq. cm. (d) sq. cm.
Ans: (c)
Q105. The length of the tangent drawn to a circle of
radius 4 cm from a point 5 cm away from the centre
of the circle is
(a) 3 cm (b) √ cm
(c) √ cm (d) cm
Ans: (a)

AB = BC = CA = 2a cm,
AD ⊥ BC
AD = √
=√ √ ∴√
a= √
∠OBA = 90°
Page 174 of 516
∴ 2a = side = √ cm
∴ Area of triangle
https://telegram.me/aedahamlibra
(a) equal to ( )
(b) greater than ( )

= ( √ ) = √ sq. cm. (c) less than (d) None of these
Ans: (b)
Q108. A, B and C are the three points on a circle such
that the an gles subtended by the chords AB and AC
at the centre O are 90° and 110° respectively. ∠BAC is
equal to
(a) 70° (b) 80°
(c) 90° (d) 100°
Ans: (b)
Let ABC be the triangle and D, E and F are midpoints of
BC, CA and AB respectively.
Hence, in ∆ ABD, AD is median
AB + AC > 2 AD Similarly, we get
BC + AC > 2 CF
BC + AB > 2 BE
On adding the above inequations, we get
(AB + AC + BC + AC + BC + AB) > 2 (AD + BE + CF)
2 (AB + AC + BC) > 2 (AD + BE + CF)
∴ AB + BC + BC > AD + BE + CF
∠AOB = 90° ; OA = OB = r
Thus, the perimeter of triangle is greater than the sum of
∴ ∠BAO = ∠ABO = 45°
the medians.
∴ ∠AOC = 110°; OA = OC = r
∴ ∠OAC = ∠OCA = = 35° Q111. In a ∆ ABC, ∠, ∠ B = 70° and ∠C = 50°, then
∴ ∠BAC = 45° + 35° = 80° ( )BAD = ?
(a) 60° (b) 20°
Q109. In ∆ABC, DE || AC. D and E are two points on (c) 30° (d) 50°
AB and CB r espectively. If AB = 10 cm and AD 2 4 Ans: (c)
cm, then BE: CE is
(a) 2 : 3 (b) 2 : 5 Q112. Two circles with radii 25 cm and 9 cm touch
(c) 5 : 2 (d) 3 : 2 each other externally. The length of the direct
Ans: (d) common tangent is
(a) 34 cm (b) 30 cm
(c) 36 cm (d) 32 cm
Ans: (b)

Q113. In a ∆ ABC, , and (


) ( ) are three medians. Then the
ratio is
DE | | AC
(a) equal to (b) less than
∆ABC – ∆BDE ∴
(c) greater than (d) equal to ⊥
Ans: (c)

Q114. The sum of the interior angles of a polygon is


1444°. The number of sides of the polygon is
(a) 6 (b) 9
Q110. In a ∆ ABC, AD, BE and CF are three medians.
(c) 10 (d) 12
The perimeter of ∆ABC is always
Page 175 of 516
https://telegram.me/aedahamlibra
Ans: (c) Sum of interior angles of polygon = (n – 2) ×
180°
(n – 2) × 180° = 1440
n–2=
n = 10
Hence, the number of sides is 10. en is equal to
(a)
Q115. If AB = 5 cm, AC = 12 and then the
(c)
radius of the circumcircle of ∆ABC is
(a) 6.5 cm (b) 6 cm Ans: (d)
(c) 5 cm (d) 7 cm
Ans: (a) In ∆ ABC,
BC2 = AB2 + AC2
BC2 = (5) 2 + (12)2
BC2 = 25 + 144
BC2 = 169
BC = √ = 13 cm
Since DE is parallel to BC

Radius of triangle = cm

Q116. The perimeters of two similar triangles ∆ABC


Q118. Two circles in tersect each other at the points A
and ∆PQR are 36 cm and 24 cm respectively. If PQ =
and B. A straight line parallel to AB intersects the
10 cm, the AB is
circles at C, D, E and F. If CD = 4.5 cm, then the
(a) 15 cm (b) 12 cm
measure of EF is
(c) 14 cm (d) 26 cm
(a) 1.50 cm (b) 2.25 cm
Ans: (a) ∆ABC ~ ∆PQR (given)
(c) 4.50 cm (d) 9.00 cm
Ans: (c)

(Corresponding sides are proportional)


Q119. If the sides of a right angled tri angle are three
consecutive integers, then the length of the smallest
side is
(a) 3 units (b) 2 units
cm (c) 4 units (d) 5 units
Ans: (a) Consecutive integer = 3, 4 and 5
Q117. In ∆ABC, D and E are two points on the sides
AB and AC respectively so that DE||BC and Q120. In ∆ABC, E and D are points on sides AB and
.T AC respectively such that ∠ABC = ∠ADE. If AE = 3
cm, AD = 2 cm and EB = 2 cm, then length of DC is
(a) 4 cm (b) 4.5 cm
(c) 5.0 cm (d) 5.5 cm
Ans: (d) In ∆ ADE and ∆ ABC
∠ ∠

Page 176 of 516


https://telegram.me/aedahamlibra
B + D + C2 = 180°; B + C2 = 180° – 90° = 90°
A + C1 = B + C2
C1 – C2 = B – A

∠ ∠ Q123. In a circle with centre O, AB is a chord, and AP


By AA is a tangent to the circle. If ∠AOB = 140°, then the
measure of ∠PAB is
(a) 35° (b) 55°
(c) 70° (d) 75°
Ans: (c)

15 = 4 + 2 DC
11 = 2 DC
5.5 = DC

Q121. In a quadrilateral ABCD, the bisectors of ∠A


and ∠B meet at O. If ∠C = 70° and ∠D = 130°, then
measure of ∠AOB is
(a) 40° (b) 60° In ∆ AOB, ∠A + ∠B + ∠O = 180°
(c) 80° (d) 100° ∠A + ∠ B = 180 – 140° = 40°
Ans: (d) ∠A = ∠B = 20°{AO = BO}
∠PAO = 90°
∠PAB + ∠BAO = 90°
∠PAB = 90° – 20° = 70°

Q124. Two parallel chords of a circle of diameter 20


cm are 12 cm and 16 cm long. If the chords are in the
same side of the centre, then the distance between
A + B + C + D = 360 them is
A + B = 360 – (130 + 70) = 160° (a) 28 cm (b) 2 cm
= 80°... (1) (c) 4 cm (d) 8 cm
In ∆ AOB, Ans: (b)
= 180°
0 = 180° – 80° = 100°

Q122. In ∆ABC, ∠A < ∠B. The altitude to the base


divides vertex angle C into two parts C 1 and C 2, with
C2 adjacent to BC. Then
(a) C1 + C2 = A + B (b) C1 – C2 = A – B
(c) C1 – C2 = B – A (d) C1 + C2 = B – A
Ans: (c) In ∆ADO,
OD = √ =√ = 6 cm
In ∆BCO,
OC = √
=√ = 8 cm
distance between chords = OC – OD
In ∆ ADC, = 2CM
A + D + C1 = 180°; A + C1 = 180° – 90° = 90°
In ∆ BDC,

Page 177 of 516


then the measure of ∠BAC is
https://telegram.me/aedahamlibra
Q125. If O is the in -centre of ∆ABC; if ∠BOC = 120°,
Q128. Two circles of radii 10 cm and 8 cm intersect
(a) 30° (b) 60° and the length of the common chord is 12 cm. Then
(c) 150° (d) 75° the distance between their centres is:
Ans: (b) ∠ (a) 15 cm (b) 10 cm
(c) 8 cm (d) 13.3 cm
∠ Ans: (d)

Line joining the centre is ⊥ bisector of common chord


While ∠ is in circle then
∴ QM = MR = cm
∠ = ∠
In ∆OMQ, ∠OMQ = 90°
OQ2 = OM2 + MQ2 (Pythagorus theorem)
Q126. If two circles of radii 9 cm and 4 cm touch
102 = OM2 + 62
externally, then the length of a common tangent is
OM2 = 100 – 36 = 64
(a) 5 cm (b) 7 cm
OM = 8cm
(c) 8 cm (d) 12 cm
In ∆ QMP ∠QMP = 90°
Ans: (d)
QP2 = QM2 + PM2 (Pythagorus theorem)
82 = 62 + PM2
PM = 64 – 36 = √ √
OP = OM + MP = √
So distance between centres O and P
= √ cm

Q129. If in a triangle ABC, BE and CF are two


In figure, AC = AO – CO medians perpendicular to each other and if AB =
= 9 cm – 4 cm = 5 cm{CO = BO'} 19cm and AC = 22cm then the length of BC is :
Also, CB = OO′ = 13 cm (a) 20.5cm (b) 19.5cm
In ∆ ABC (c) 13cm (d) 26cm
Ans: (c)
AB = √
=√ Q130. The perimetres of two similar triangles are 30
= 12 cm cm and 20cm respectively. If one side of the first
triangle is 9cm. Determine the corresponding side of
Q127. The interior angle of a r egular polygon is 140°. the second triangle:
The number of sides of that polygon is (a) 15 cm (b) 5 cm
(a) 9 (b) 8 (c) 6 cm (d) 13.5 cm
(c) 7 (d) 6 Ans: (c)
Ans: (a) Let n be the number of sides.
(n – 2) × 180° = 140° × n
180n – 360 = 140 n
40n = 360
n= =9
Page 178 of 516
https://telegram.me/aedahamlibra
∠EAD = 10°

Q133. The diagonal of a quadrilateral shaped field is


24m and the perpendiculars dropped on it from the
remaining opposite vertices are 8m and 13m. The area
= of the field is:
(a) 252 m2 (b) 1152 m2
2
(c) 96 m (d) 156 m2
PQ = = 6 cm Ans: (a) In ∆ADC
Area of ∆ ADC =
Q131. Two isosceles triangles have equal vertical
angles and their areas are in the ratio 9:16. Then the
ratio of their corresponding heights is:
(a) 4.5 : 8 (b) 8 : 4.5
(c) 3 : 4 (d) 4 : 3
Ans: (c)

Area of ∆ADC = = 96 m2
Area of ∆BAC = = 156 m2
Area of Quadrilateral = 96 + 156 = 252 m2

Q134. The distance between centres of two circles of


= radii 3 cm and 8 cm is 13 cm. If the points of contact
of a direct common tangent to the circles are P and Q,
=√ then the length of the lien segment PQ is :
(a) 11.9 cm (b) 11.5 cm
Q132. In ∆ABC, ∠B = 60°, and ∠C = 40°; AD and AE (c) 12 cm (d) 11.58 cm
are respectively the bisector of ∠A and perpendicular Ans: (c)
on BC. The measure of ∠EAD is :
(a) 9° (b) 11° Q135. ABCD is a square. Draw a triangle QBC on
(c) 12° (d) 10° side BC considering BC as base and draw a triangle
Ans: (d) PAC on AC as its base such that ∆QBC ~ ∆PAC.
Then is equal to:
(a)
(c) √
Ans: (c)

In ∆ ABC
∠Α = 180°− (60° + 40°) = 80°
∠BAD = ∠DAC = 40° (AD is bisector of ∠A)
ABCD is a square.
In ∠AEC
QBC ~ PAC (Given)
∠EAC = 180° – (90° + 40°) = 50°
So, ∠EAD = ∠EAC – ∠DAC
= 50° – 40°
Page 179 of 516
If BC = 1 then AC = √ https://telegram.me/aedahamlibra
Required ratio = =

Q136. Two circles of radii 5 cm and 3 cm touch


externally, then the ratio in which the direct common
tangent to the circles divides externally the line AB2 + OA2 = OB2
joining the centres of the circles is : AB2 = (10)2 – (6) 2 = 100 – 36 = 64
(a) 2.5 : 1.5 (b) 1.5 : 2.5 AB = 8cm
(c) 3 : 5 (d) 5 : 3
Ans: (d) Using property of direct common tangent Q139. In ∆ABC, a line through A cuts the side BC at
Required ratio = Ratio of radii = 5 : 3 D such that BD : DC = 4 : 5. If the area of ∆ABD = 60
cm2 , then the area of ∆ADC is
(a) 50 cm2 (b) 60 cm2
Q137. In ∆ABC, AB = BC = K, AC = √ K, then 2
(c) 75 cm (d) 90 cm2
∆ABC is a :
Ans: (c)
(a) Isosceles triangle (b) Right angled triangle
(c) Equilateral triangle (d) Right isosceles triangle
Ans: (d) In ∆ABC

Area of ∆ABD =
60 = ... (1)
Area of ∆ADC =
AC = √ Area of ∆ADC = ... (2)
AC2 = 2K2 Dividing eqn. (1) and (2)
AC2 = AB2 + BC2
So ABC is right angled triangle
So, in ABC
= = Area of ∆ ADC =
√ √
So cosθ =

θ = 45° Q140. The measure of an angle whose supplement is
So, ABC, ∠B = 90°; ∠C = 45°; ∠A = 45° three times as large as its complement, is
So, ABC is right isoscles triangle. (a) 30° (b) 45°
(c) 60° (d) 75°
Q138. A tangent is drawn to a circle of radius 6cm Ans: (b) Let ‗x‘ be the measure of an angle.According to
from a point situated at a distance of 10 cm from the question
centre of the circle. The length of the tangent will be 3x + x = 180°
(a) 4 cm (b) 5 cm 4x = 180°
(c) 8 cm (d) 7 cm
Ans: (c)

Q141. Two poles of height 7 m and 12 m stand on a


plane ground. If the distance between their feet is 12
m, the distance between their top will be
(a) 13 m (b) 19 m
(c) 17 m (d) 15 m

Page 180 of 516


Ans: (a) https://telegram.me/aedahamlibra
Ans: (d) Angles are = (x + 15°),
( ) ( )
Q142. Two chords of length a unit and b unit of a
circle make angles 60° and 90° at the centre of a circle We know that
respectively, then the correct relation is Sum of the angles of a triange is 180°.
(a) b = √ a (b) b = 2a
(c) b = (d) b =
Ans: (a)

43x = 129 × 15
Q143. The sides of a triangle having area 7776 sq. cm
x = 45°
are in the ratio 3:4:5. The perimeter of the triangle is
(a) 400 cm (b) 412 cm Then angle are = (45+ 15°), ( ) and (
(c) 424 cm (d) 432 cm )
Ans: (d) Let sides of ∆ be 3x, 4x, 5x
= 60°, 60°, 60°
= 6x So this is an equilateral triangle.
Area of √
√ Q147. Let C 1 and C 2 be the inscribed and
7776 = 6x2 circumscribed circles of a triangle with sides 3 cm, 4
∴ x = 36 cm and 5 cm then area of C1 to area of C2 is
Sides of ∆ will be 108, 144 and 180 (a)
Perimeter of ∆ is 108 +144 + 180 = 432 cm (c) ( ) ( )
Ans: (c)
Q144. If a clock started at noon, then the angle turned
by hour hand at 3.45 PM is
Q148. If the altitude of an equilateral triangle is √
(a)
cm, then its area would be :
(c) (a) 12 cm2 (b) 72 cm2
Ans: (c) Clock started at 12 pm (c) √ cm 2
(d) ∠ cm2
Angle turned by hour hand in one hour = = 30° Ans: (d)

Angle turned by hour hand in one minute Q149. If the number of vertices, edges and faces of a
Angle turned by hour hand in 3 hour 45 minutes rectangular parallelopiped are denoted by v, e and f
= 3 × 30° + 45 × respectively, the value of (v – e + f) is
(a) 4 (b) 2
(c) 1 (d) 0
Q145. In a parallelogram PQRS, angle P is four times
Ans: (b) The value of = v – e + f
of angle Q, then the measure of ∠R is
= 8 – 12 + 6 = 2.
(a) 36° (b) 72°
(c) 130° (d) 144°
Q150. If the measure of three angles of a triangle are
Ans: (d) P = 4Q
in the ratio 2 : 3 : 5, then the triangle is :
P + Q = 180°
(a) equilateral (b) isocsceles
4Q + Q = 180°
(c) Obtuse angled (d) right angled
Ans: (d) Sum of the angle of a triangle = 180°
So, R = 180° – 36° = 144° 2x° + 3x° + 5x° = 180°
10x° = 180°
Q146. If the three angles of a triangle are : x° = 18°
Angle are = 36°, 54°, 90° So, this is right angles triangle.
√ then the triangle is :
(a) scalene (b) isosceles
(c) right angled (d) equilateral
Page 181 of 516
Q151. Internal bisectors of https://telegram.me/aedahamlibra
∠Q and ∠R of ∆PQR
intersect at O. If ∠ROQ = 96° then the value of ∠RPQ
Here AD and BE Intersect at O (AD ⊥ BE)
∴∠AOB = 90°
is : AO = cm
(a) 12° (b) 6°
(c) 36° (d) 24° OB = cm

Ans: (a) ∠QPR = AB = √

96° =
∠ =√
=√
= 10 cm

Q154. O is the orthocentre of ΔABC , and if ∠BOC =


110° then ∠BAC will be
(a) 110° (b) 70°
∠ (c) 100° (d) 90°
Ans: (b) For Orthocentre ∠BAC = 180 – ∠BOC
∴∠P = 12°
= 180 – 110 = 70°

Q152. If a person travels from a point L towards east


Q155. If D, E and F are the mid points of BC, CA and
for 12 km and then travels 5 km towards north and
AB respectively of the ∆ABC then the ratio of area of
reaches a point M, then shortest distance from L to M
the parallelogram DEFB and area of the trapezium
is :
CAFD is :
(a) 14 (b) 12
(a) 1 : 3 (b) 1 : 2
(c) 17 (d) 13
(c) 3 : 4 (d) 2 : 3
Ans: (d)
Ans: (d)

LM = √ =√
=√
Area of = BD × ED
Q153. G is the centroid of ∆ABC. The medians AD Area of trapezium
and BE intersect at right angles. If the lengths of AD Here AC = 2 AE = 2FD
and BE are 9 cm and 12 cm respectively; then the
length of AB (in cm) is ?
(a) 10 (b) 10.5 Now =
(c) 9.5 (d) 11
Ans: (a) Given AD = 9 cm = =
BE = 12 cm
Q156. In a ∆ ABC, BC is extended upto D :
∠ ∠ ∠ Then √ is
(a) 60° (b) 75°
(c) 80° (d) 90°
Ans: (c) ∠ A + ∠ B = ∠ ACD

Page 182 of 516


https://telegram.me/aedahamlibra
XY = 4
BC = 8
Hence, BC – XY = 8 – 4 = 4

Q159. O is the centre of a circle and AB is the tangent


to it touching at B . If OB = 3 cm. and OA = 5 cm,
then the measure of AB in cm is
(a) √ (b) 2
∠ ∠ (c) 8 (d) 4
∠ Ans: (d) O A2 = OB2 + BA2
A B2 = 52 – 32
∠ A = 80°

Q157. BE and CF are two altitudes of a triangle ABC.


If AB = 6 cm , AC = 5 cm and CF = 4 cm , then the
length of BE is
(a) 4.8 cm (b) 7.5 cm
(c) 3.33 cm (d) 5.5 cm
= 25 – 9 = 16
Ans: (a) A B = 6 cm, AC = 5 cm, CF = 4 B E = ?
AB=4
ans A B C
Q160. The length of the radius of a circle with centre
O is 5 cm and the length of the chord AB is 8 cm. The
distance of the chord AB from the point O is
(a) 2 cm (b) 3 cm
(c) 4 cm (d) 15 cm
Ans: (b) AB = 8, AD = 4, OA = 5 cm
OD2 = 52 – 42 = 32
AB × F C = A C × B E OD = 3
6×4=5×x
=x
4.8 = x

Q158. X and Y are the mid points of sides AB and AC


of a triangle ABC. If BC + XY = 12 units, then BC –
XY is Q161. In ΔPQR, L and M are two points on the sides
(a) 8 units (b) 4 units PQ and PR respectively such that LM II QR. If PL =
(c) 6 units (d) 2 units 2cm; LQ = 6cm and PM = 1.5 cm, then MR in cm is
Ans: (b) (a) 0.5 (b) 4.5
(c) 9 (d) 8
Ans: (b) By Thales theorem

BC 2XY = BC
BC + XY = 12
2XY + XY = 12
3XY = 12
Page 183 of 516
https://telegram.me/aedahamlibra
∴ ∠ OPB = ∠ PBO
2 ∠ PBO = 120°, ∠ PBO = 60°
x = 4.5 cm
Q165. Which of the set of three sides ca n't form a
Q162. If PA and P B are two tangents to a cirlce with triangle?
centre O such that ∠APB = 80°. Then, ∠AOP = ? (a) 5 cm, 6 cm, 7 cm (b) 5 cm, 8 cm, 15 cm
(a) 40° (b) 50° (c) 8 cm, 15 cm, 18 cm (d) 6 cm, 7 cm, 11 cm
(c) 60° (d) 70° Ans: (b) 5 + 8 < 15
Ans: (b) ∠ A P B = 80° ∴ 5, 8, 15 cannot Form a ∆
∠ A O B = 180 – 80 = 100°
∠AOP= = 50° Q166. Which of the following ratios can be the ratio of
the sides of a right angled triangle?
(a) 9 : 6 : 3 (b) 13 : 12 : 5
(c) 7 : 6 : 5 (d) 5 : 3 : 2
Ans: (b) As only 13, 12 and 5 follows Pythagorous
theorem

Q167. An arc of 30° in one circle is double an arc in a


second circle, the radius of which is three times the
Q163. In a triangle ABC, if ∠A + ∠C = 140° and ∠A + radius of the first. Then the angles subtended by the
3∠B = 180°, then ∠A is equal to arc of the second circle at its centre is
(a) 80° (b) 40° (a) 3° (b) 4°
(c) 60° (d) 20° (c) 5° (d) 6°
Ans: (c) In ∆ A B C Ans: (c) = 30° =
∠ A + ∠ B + ∠ C = 180 Arc 1 = 22 =
∠ B = 180 – [∠ A + ∠ C) r1 = rr2 = 3r
= 180 – 140 = 40° Arc length =
∠ A + 3∠ B = 180
∠ A = 180 – 3 (40)° =
= 180 – 120
= 60°

=
Q164. AB is the diameter of a circle with centre O and
P b e a point on its circumference, If ∠POA = 120°,
then the value of ∠PBO is: Q168. Two circles touch each other internally. T he
(a) 30° (b) 60° radius of the smaller circle is 6 cm and the distance
(c) 50° (d) 40° between the centre of two circles is 3 cm. The radius
Ans: (b) ∠ P O A = 120° of the larger circle is
∠ P O A = ∠ OPB + ∠ PBO (a) 7.5 cm (b) 9 cm
(c) 8 cm (d) 10 cm
Ans: (b) O′O = 3
OA = 6
O′A = 6 + 3 = 9 cm

OP = OB
Page 184 of 516
three non-colinear points is
https://telegram.me/aedahamlibra
Q169. Number of circles that can be drawn through √
(a) 40, 46, 94 (b) 40, 50, 90
(a) exactly one (b) two (c) 46, 54, 80 (d) 50, 40, 90
(c) three (d) more than three Ans: (a) ∠MRP = ∠PQR = 46°
Ans: (a) ∴∠y = 46°
∠NRQ = m ∠QPR = 40°
Q170. In the triangle ABC, ∠BAC = 50° and the ∴∠x = 40°
bisectors of ∠ABC and ∠ACB meets at P. What is the ∠x + ∠z + 46° = 180°
value (in degrees) of ∠BPC? 40° + ∠z + 46° = 180°
(a) 100 (b) 105 ∴∠z = 94°
(c) 115 (d) 125 ∴Value of x, y and z = 40°, 46° and 94°
Ans: (c)
Q173. Two circles of s ame radius intersect each other
at P and Q. If the length of the common chord is 30
cm and distance between the centres of the two circles
is 40 cm, then what is the radius (in cm) of the circles?
(a) 25 (b) √
(c) 50 (d)

∠BAC = 50°


∠BPC =

Ans: (a)
= 115°
∆POR = right angle triangle
Q171. PQR is an equilateral triangle. MN is drawn
parallel to QR such that M is on PQ and N is on PR. If
PN = 6 cm, then the length of MN is
(a) 3 cm (b) 6 cm
(c) 12 cm (d) 4.5 cm
Ans: (b) In Given equilateral ∆, MN || QR
PR = 15 cm
RO = 20 cm
PN = MN ∴ √

(PR = QR)

MN = 6 cm
√ cm

PO = radius of circle
∴Radius of circle = 25 cm.

Q174. The perimeter of an isosceles triangle is 32 cm


and each of the equal sides is 5/6 times of the base.
What is the area (in cm2) of the triangle?
Q172. In the given figure, ∠QRN = 40°, ∠PQR = 46° (a) 39 (b) 48
and MN is a tangent at R. What is the value (in (c) 57 (d) 64
degrees) of x, y and z respectively? Ans: (b)
Page 185 of 516
https://telegram.me/aedahamlibra
Q175. In ∆PQR, ∠R = 54°, the perpendicular bisector
Ans: (b) According to question,
Required ratio
of PQ at S meets QR at T. If ∠TPR = 46°, then what is
the value (in degrees) of ∠PQR?
(a) 25 (b) 40
(c) 50 (d) 60
Ans: (b)

Q176. In the given figure, ABC is a triangle. The


bisectors of internal ∠B and external ∠C intersect at
D. If ∠BDC = 48°, then what is the value (in degrees)
of ∠A ? 3 : 15
√ 2 : 1.
(a) 48 (b) 96
(c) 100 (d) 114 Q179. In the given figure, O is the centre of the circle
Ans: (b) According to question, and ∠DCE = 45°. If CD =
∠A = 2 × ∠BDC = 2 × 48° = 96°

Q177. If length of each side of a rhombus PQRS is 8


cm and ∠PQR = 120°, then what is the length (in cm)
of QS?
(a)
cm, then what is the
length (in cm) of AC. (CB = BD):

(b) 6
(c) 8 (d) 12 (a) 14 (b) 15.5
Ans: (c) ∴ ∠QRS = 180° – 120° = 60° (c) 18.5 (d) 20
Ans: (c)

Q180. In triangle PQR, A is the point of intersection


of all the altitudes and B is the point of intersection of
all the angle bisectors of the triangle. If ∠PBR = 105°,
then what is the value of ∠PAR (in degrees)?
(a) 60 (b) 100
∠ ∠ (c) 105 (d) 115
∴∆RQS is a equlateral triangle. Ans: (a)
∴RQ = QS = 8 cm
Q181. PQR is a right angled triangle in which ∠R =
Q178. In triangle ABC, a line is drawn from the 90°. If RS ⊥ PQ, PR = 3 cm and RQ = 4 cm, then what
vertex A to a point D on BC. If BC = 9 cm and DC = 3 is the value of RS (in cm)?
cm, then what is the ratio of the areas of triangle ABD (a) 12/5 (b) 36/5
and triangle ADC respectively? (c) 5 (d) 2.5
(a) 1 : 1 (b) 2 : 1 Ans: (a)
(c) 3 : 1 (d) 4 : 1
Page 186 of 516
PR = 3 cm and
RQ = 4 cm
https://telegram.me/aedahamlibra
then,

(a) 9 (b) 17
(c) 21 (d) 25
Ans: (b) ∠PSO is a right angle (angle of semicircle)

√ cm
Now,

∴ Again when OS is perpendicular on chord PR and OS


passes through the centre of circle PQR, then it must
bisect the chord PR at S.
Q182. In ∆ABC, ∠BAC = 90° and AD is drawn
∴PS = RS = 17 cm.
perpendicular to BC. If BD = 7 cm and CD = 28 cm,
then what is the length (in cm) of AD?
Q185. A chord of length 60 cm is at a distance of 16
(a) 3.5 (b) 7
cm from the centre of a circle. What i s the radius (in
(c) 10.5 (d) 14
cm) of the circle?
Ans: (d) ∆ABC
(a) 17 (b) 34
(c) 51 (d) 68
∠BAC = 90°
Ans: (b) According to question,

(AD)2 = BD × DC
AC = 30 cm
OC = 16 cm
∴AO2 = OC2 + AC2
∴ √

AD2 = 7 × 28
∴ √
∴AD = 14 cm.

Q183. If there are four lines in a plane, then what


cannot be the number of points of intersection of these √ √ = 34 cm
lines? ∴radius of circle = 34 cm.
(a) 0 (b) 5
(c) 4 (d) 7 Q186. Find the sum of interior angles of a dodecagon?
Ans: (d) (a) 1620° (b) 1800°
(c) 1440° (d) 1260°
Q184. In the given figure, a smaller circle touches a Ans: (b)
larger circle at P and passes through its centre O. PR
is a chord of length 34 cm, then what is the length (in Q187. In the given figure, ABC is a triangle in which,
cm) of PS? AB = 10 cm, AC = 6 cm and altitude AE = 4 cm. If AD

Page 187 of 516


https://telegram.me/aedahamlibra
is the diameter of the circum -circle, then what is the
length (in cm) of circum-radius?
∠ ∠ ∠
(a) 3 (b) 7.5
(c) 12 (d) 15
Ans: (b)

SR, then what is the


Q188. In the given figure, O is the centre of the circle,
area (in cm2) of the quadrilateral PTRS?
∠ ?. What is the value (in degrees) of

(a) 80 (b) 64
(c) 124 (d) 72
(a) 5 (b) 10 Ans: (b)
(c) 15 (d) 20
Ans: (b) Q191. If ∠ is right angled at E, DE = 15 and

Q189. In ∠ ∠ ?A line parallel to QR is = 60 √ , then what is the value of EF?


drawn which touches PQ and PR at A and B (a) || (b) 5
respectively. What is the value of ∠ (c) 15 (d) 30
∠ Ans: (a) According to question,
(a) 60 (b) 30
(c) 24 (d) 36
Ans: (a)

DE = 15, EF = ?
∠DFE = 60°
∠DEF = 90°

AB || QR

Ratio of ∠P, ∠Q and ∠R = 2 : 2 : 5
√ √
∴∠P = 40°, ∠Q = 40°, ∠R = 100° ∴ √
√ √
∠ ∠
- (corresponding angle)
∠ ∠ Q192. In the given figure , BD passes through centre
∴Difference ∠PBA and ∠PAB = (100 – 40) = 60°
O, AB = 12 and AC = 8. What is the radius of the
circle?
Q190. In the given figure, area of isosceles triangle

PQT is 128 cm2 and QT = PQ and PQ = 4 PS, PT
(a) √ √
(c) √ (d)
Ans: (c)

Page 188 of 516


https://telegram.me/aedahamlibra
∴ cm
∆ABD is a right angle because ∠B = 90°
By Pythagorean theorem,
BD2 + AB2 = AD2
BD2 + AB2 = AD2
According to question, ∆BD2 = (18)2 – (12)2
AB = 12 = 324 – 144
AC = 8 = 180
AD = 8 + x ∴ √ cm
As, we know that √
∴Radius √ cm
(AB)2 = AC × CD
(12)2 = 8 × x

Page 189 of 516


https://telegram.me/aedahamlibra
REASONING ABILITY
Ans: (a) FHEG →
Analogy JLIK
PROQ → TVSU

DIRECTIONS: In each of the following questions, DIRECTIONS: In each of the following questions,
select the related parts from the given alternatives. select the related parts from the given alternatives.

Q1. JLNP : OMKI : : SUWY : ? Q8. Find out a set of numbers amongst the four sets of
(a) MLKJ (b) PLHD numbers given in the alternatives, which is the most
(c) XVTR (d) FGHI similar to the numbers given in the question.
Ans: (c) JLNP → OMKI Given : (6, 30, 90)
(a) 6, 42, 86 (b) 7, 42, 218
SUWY → XVTR
(c) 6, 24, 70 (d) 8, 48, 192
Ans: (d) 6 × 5 = 30, 30 × 3 = 90
Q2. 5 : 27 : : 9 : ?
8 × 6 = 48, 48 × 4 = 192
(a) 83 (b) 81
(c) 36 (d) 18
Q9. Fox : Cunning : : Rabbit : ?
Ans: (a) As, 5 × 5 + 2 = 27 Similarly, 9 × 9 + 2 = 83
(a) Courageous (b) Dangerous
(c) Timid (d) Ferocious
Q3. 6 : 11 : : 11 : ?
Ans: (c) Here, animal -behaviour relationship has been
(a) 6 (b) 17
shown. Fox is characterised by its cunningness.
(c) 21 (d) 30
Similarly, rabbit is considered as timid.
Ans: (c) As, 6 × 2 – 1 = 11 Similarly, 11 × 2 – 1 = 21

Q10. Flexible : Rigid : : Confidence : ?


Q4. ABE : 8 : : KLO : ?
(a) Diffidence (b) Indifference
(a) 37 (b) 39
(c) Cowardice (d) Scare
(c) 38 (d) 36
Ans: (a) Flexible is antonym of Rigid. Similarly,
Ans: (c) As, A + B + E 1 + 2 + 5 = 8 Similarly, K + L
Confidence is antonym of Diffidence.
+ O 11 + 12 + 15 = 38
Q11. SHI : RIJ : : QJK : ?
Q5. Sty : Pig : : Byre : ?
(a) TDE (b) PKL
(a) Eagle (b) Cow
(c) UGH (d) VPQ
(c) Tiger (d) Hem

Ans: (b) The resting place of pig is called Sty. Similarly , Ans: (b) SHI → RIJ
the resting place of cow is called Byre. –
QJK → PKL
Q6. Patrol : Security : : Insurance : ?
Q12. YAWC : UESG : : QIOK : ?
(a) Money (b) Policy
(a) MMKO (b) KOME
(c) Savings (d) Risk
(c) MINC (d) MIKE
Ans: (d) In order to ensure security, police or defence
– –
personnel patrol the area. Similarly, to cover risk, Ans: (a) YAWC → UESG
insurance is done. – –
QIOK → MMKO
Q7. FHEG : JLIK : : PROQ : ?
(a) TVSU (b) VTUS Q13. ?
(c) TVUS (d) SUVT (a)
Page 190 of 516
(c) https://telegram.me/aedahamlibra
Q21. 17 : 60 : : 20 : ?
(a) 57 (b) 69
Ans: (a)
(c) 81 (d) 93
Ans: (b) As, 17 × 3 + 9 = 51 + 9 = 60 Similarly, 20 × 3 +
Q14. 1 : 8 : : 27 : ?
9 = 60 + 9 = 69
(a) 37 (b) 47
(c) 57 (d) 64
Q22. 6 : 64 : : 11 : ?
Ans: (d) As, (1) 3 = 1; (2) 3 = 8
(a) 144 (b) 169
(3) 3 = 27; (4) 3 = 64
(c) 121 (d) 124
Ans: (b) As, (6 + 2) 2 = (8) 2 = 64 Similarly, (11 + 2) 2 =
Q15. 24 : 126 : : 48 : ?
(13) 2 = 169
(a) 433 (b) 192
(c) 240 (d) 344
DIRECTIONS: Select the related letter/word/ number
Ans: (d) 52 – 1 = 24; 53 + 1 = 126
from the given alternatives.
72 – 1 = 48; 73 + 1 = 344

Q23. 123 : 4: : 726 : ?


Q16. 987 : IHG : : 654 : ?
(a) 23 (b) 26
(a) FDE (b) FED
(c) 14 (d) 12
(c) EFD (d) DEF
Ans: (d)
Ans: (b)

Q24. Country : President : : State : ?


DIRECTIONS: In each of the following questions,
(a) Chief Minister (b) Prime Minister
select the related parts from the given alternatives.
(c) Speaker (d) Governor
Ans: (d) President is the he ad of Union Executive of a
Q17. From a mongst the given alternatives, select the
India. Similarly, Governor is the head of State Executive.
one in which the set of numbers is most like the set of
numbers given below :
Q25. Mirage : Desert : : ?
(6, 14, 30)
(a) Sky : Illusion (b) Rainbow : Sky
(a) 4, 16, 28 (b) 7, 12, 22
(c) Rain : Rainbow (d) Image : Mirror
(c) 6, 12, 22 (d) 5, 12, 20
Ans: (c) Mirage is an illusion caused by hot air
Ans: (b) 6 + 8 = 14; 14 + (8 × 2) = 30
conditions makin g one see something that is not there,
7 + 5 = 12; 12 + (5 × 2) = 22
especially the appearance of a sheet of water on a hot
road or in a desert. Similarly, Rainbow is an arch of
Q18. Fish : Scales : : Bear : ?
seven colours formed in the sky when the sun shines
(a) Feathers (b) Leaves
through rain.
(c) Fur (d) Skin
Ans: (c) The body of fish remains covered with scales
Q26. TEKCAR : RACKET : : TCEJBO : ?
externally. Similarly, the body of bear remains covered
(a) TCEOBJ (b) OBJECT
with fur.
(c) CEJBOT (d) REJECT
Ans: (b) The letters have been written in reverse order.
Q19. Writer : Pen : : ?
TEKCAR RACKET Similarly, TCEJBO OBJECT
(a) Needle :Tailor (b) Artist : Brush
(c) Painter : Canvas (d) Teacher : Class
Q27. JIHK : PONQ : : WVUX : ?
Ans: (c) Here, Writer uses pen for writting. Similarly,
(a) KNML (b) RSTU
painter works on canvas.
(c) HIGJ (d) MLKN
Q20. FGHI : OPQR : BCDE : ? Ans: (d) J→ I→ H→ K
(a) KLMJ (b) KLMN
(c) IUVW (d) STUW Q28. UUWX : WWYZ : : OOQR : ?
Ans: (b) Successive Change of +1 (a) OOPG (b) MMPQ
(c) XXYZ (d) QQST

Page 191 of 516


Ans: (d) OOQR → QQST https://telegram.me/aedahamlibra
Ans: (c) As, rock is a hard substance. Similarly, sponge
is a soft substance.
Q29. BIMN : CKPR : : CURD : ?
(a) DWUH (b) WUHC Q36. CEGI : DHLP : : KSEA : ?
(c) UHDW (d) HUVN (a) LVJH (b) LSGF
(c) LTGI (d) LUIH
Ans: (a) BIMN → CKPR
Ans: (a) CEGI → DHLP
CURD → DWUH
KSEA → LVJH
Q30. ? : 63 : : 08 : 26
(a) 12 (b) 9 Q37. ABDE : HIKL : : NOQR : ?
(c) 18 (d) 15 (a) SUVX (b) UVXY
Ans: (d) 42 – 1 = 15 (c) STUV (d) UWYZ
43 – 1 = 63 Ans: (b) ABDE → HIKL
32 – 1 = 8 NOQR → UVXY
33 – 1 = 26
DIRECTIONS: Select the related parts from the given
Q31. 64 : ? : : 72 : 53 alternatives.
(a) 44 (b) 54
(c) 52 (d) 70
Q38. Among the four sets, fin d out the set, which is
Ans: (b) 7 + 2 = 9;5 + 3 = 8 like the given set.
9–8=1 Given set: (12, 72, 216)
6 + 4 = 10;5 + 4 = 9 (a) (4, 24, 48) (b) (7, 42, 252)
10 – 9 = 1 (c) (11, 60, 30) (d) (5, 30, 90)
Ans: (d)
DIRECTIONS: Select the related word/letters from
the given alternatives. Q39. Symphony : Composer : : Painter : ?
(a) Fresco (b) Colours
Q32. Find out the questioned number. (c) Art (d) Leonardo
6:5::8:? Ans: (a) Fresco is an art of painting that is done on
(a) 2 (b) 4 freshly spread moist lime plaster.
(c) 6 (d) 10
Ans: (c) As, 6 = 3 × 2Similarly, 8 = 4 × 2 Q40. AB : L : : BC : ?
3 + 2 = 54 + 2 = 6. (a) X (b) V
(c) Y (d) W
Q33. Blue whale : Sea : : ? : Land Ans: (d) Solve by Place values
(a) Turtle (b) Fish
(c) Elephant (d) Green whale Q41. BDGJ : OQTW : : AFIM :
Ans: (c) As, Blu e whale is heaviest animal in the sea. (a) MUSY (b) MRUY
Similarly, Elephant is heaviest animal on the land. (c) NSVZ (d) NRZV

Q34. Anemia : Blood : : Anarchy : ? Ans: (c) BDGJ → OQTW


(a) Disorder (b) Monarchy AFIM → NSVZ
(c) Government (d) Lawlessness
Ans: (c) Anaemia is the lack of blood. Similarly, DIRECTIONS: Select the related letters/word
Anarchy is the lack of government. number from the given alternatives.

Q35. Hard : Rock : : Soft : ? Q42. 8 : 62 : : 9 : ?


(a) Lead (b) Paper (a) 64 (b) 79
(c) Sponge (d) Glass (c) 18 (d) 81
Ans: (b) 82 – 2 = 62
Page 192 of 516
92 – 2 = 79 https://telegram.me/aedahamlibraAns: (a)

Q43. 8 : 12 : : 6 : ? Q51. Zoology : Animal :: Psychology : _?_


(a) 8 (b) 11 (a) Animal (b) Human-being
(c) 5 (d) 7 (c) Animal and human-being (d) Plant
Ans: (a) The relationship is x : (2x – 4) Ans: (d) Zoology is the scientific study of animals.
8 × 2 – 4 = 16 – 4 = 12 Similarly, 6 × 2 – 4 = 12 – 4 = 8 Similarly, Psychology is study of the mind and how it
functions. It is the study of human a nd animals
Q44. 13 : 19 : : 21 : ? behaviour.
(a) 41 (b) 81
(c) 141 (d) 14 Q52. HKNQ : GDAX : : SVYB : ?
Ans: (a) (1) 2 = 1 ; (3) 2 = 9 19 Similarly, (2) 2 = 4 ; (1) (a) TQMK (b) ROLI
2
=1 (c) ZVTQ (d) ADGJ
41 Ans: (b) HKNQ → GDAX

Q45. Eagle : Swoops : : Duck : ? SVYB → ROLI


(a) waddles (b) floats
(c) swims (d) flits Q53. L × M : 12 × 13 :: U × W : _?_
Ans: (a) The movement of eagle is like swooping. (a) 21 × 23 (b) 24 × 26
Similarly, the movement of duck is called waddle. (c) 9 × 11 (d) 12 × 23
Ans: (a) L × M = 12 × 13
Q46. APPLE : 50 : : ORANGE : ? Position Number in the English alphabet series.
(a) 60 (b) 69 Similarly, U × W =
(c) 61 (d) 63
Ans: (a) , Q54. Man : Mammal : : _?_
Add the place values of alphabets (a) Liberty : Literate (b) Hail : Snow
(c) Native : Inhabitant (d) Offspring : Family
Q47. Accommodation : Rent : : Journey : ? Ans: (d) Man is a mammal. Man belongs to the class
(a) Freight (b) Octroi Mammal. Similarly, offspring is a part of family.
(c) Fare (d) Expense
Ans: (c) We pay rent for accommodation. Similarly, we Q55. 6 : 2 :: 8 : _?_
pay fare for journey. (a) 5 (b) 1
(c) 3 (d) 7
Q48. Fire : Smoke : : ? Ans: (c) As,
(a) Children : School (b) Cloud : Rain Similarly, ;
(c) Moon : Sky (d) Shoe : Polish
Ans: (b) Fire causes smoke. Smoke comes out when Q56. AFKP : ZUPK : : BGLQ : ?
something is burnt in fire. Similarly, cloud causes rain. (a) YUQM (b) XURO
(c) YXWV (d) YTOJ
Q49. Grenade : Gun : : ?
(a) Sister : Brother (b) Father : Mother Ans: (d) AFKP → ZUPK
(c) Man : Woman (d) Head : Brain BGLQ → YTOJ
Ans: (d) Grenade and gun are fire arms. Similarly, head
and brain are sensitive organs. Q57. 1 : 8 :: 4 : _?_
(a) 32 (b) 64
DIRECTIONS: Select the related parts from the given (c) 512 (d) 128
alternatives: Ans: (a) As, 1 × 8 = 8 Similarly, 4 × 8 = 32

Q50. AEZ : FPY : : BGX : ? Q58. Life starts : Embryo :: Life ends : _?_
(a) HWW (b) IYY (a) Death (b) Old age
(c) HTX (d) HYW (c) Dead body (d) Illness
Page 193 of 516
life ends dead body is left.
https://telegram.me/aedahamlibra
Ans: (c) Life starts with the formation of embryo. When Ans: (b) BDFH → JLNP
CEGI → KMOQ
DIRECTIONS: In each of the following questions,
select the related parts from the given alternatives. Q66. food : man :: fuel:?
(a) wood (b) fire
Q59. EVFU : TGSH :: IRJQ : _?_ (c) heat (d) smoke
(a) PKLO (b) KWLX Ans: (b) Food is necessary for man. Similarly, fuel is
(c) PKOL (d) OLPK necessary for fire.
Ans: (a) Pairs of Opp Letters
Q67. Spiritual : Belief :: Orchestral: ?
Q60. 9 : 80 : : 100 : ? (a) Theatre (b) Situation
(a) 901 (b) 1009 (c) Music (d) Direction
(c) 9889 (d) 9999 Ans: (c) Spiritual and Belief are inter – related terms.
Ans: (d) The relationship between the numbers is : Similarly, Orchestral and music are related.
x : (x2 – 1)
(9) 2 – 1 = 81 – 1 = 80 DIRECTIONS: Select the related parts from the given
(100) 2 – 1 = 10000 – 1 = 9999 alternatives.

Q61. 25 : 125 : : 36 : ? Q68. Latter : Former :: Hazardous : ?


(a) 180 (b) 206 (a) Risky (b) Comfortable
(c) 216 (d) 318 (c) Safe (d) Harmful
Ans: (c) The relationship between the numbers is : Ans: (c) Latter and Former are antonymous to each other.
x2 : x3 Similarly, Hazardous and Safe are antonymous to each
(5) 2 = 25; (5) 3 = 125 other.
(6) 2 = 36; (6) 3 = 216
Q69. Length : Metre :: Power : ?
Q62. 335 : 216 : 987 : ? (a) Calories (b) Degree
(a) 868 (b) 867 (c) Watt (d) Kilogram
(c) 872 (d) 888 Ans: (c) Metre is a unit of length likewis e watt is a unit
Ans: (a) The relationship between the numbers is : of power.
x : x – 119
335 – 119 = 216 Q70. Square : Cube :: Circle : ?
987 – 119 = 868 (a) Ellipse (b) Parabola
(c) Cone (d) Sphere
Q63. BEGK : ADFJ : : PSVY : ? Ans: (d) As, cube is 3 -D of square. Similarly, sphere is
(a) ROUX (b) LQUT 3-D of circle.
(c) LOQT (d) ORUX
– – – – Q71. Paper : Tree :: Glass : ?
Ans: (d) BEGK → ADFJ (a) Window (b) Sand
– – – – (c) Stone (d) Mirror
PSVY → ORUX
Ans: (b) As, paper is product of Tree. Similarly, glass is
a product of sand.
Q64. AZBY : CXDW :: EVFU : ?
(a) GTHS (b) GHTS
Q72. (a) ZXVT (b) YWUS
(c) GSTH (d) TGSH
(c) PNLJ (d) IHGF
Ans: (a) Pairs of consecutive opposite letters are given :
AZ, BY; CX, DW; EV, FU; GT, HS Ans: (d) Normal Pattern Z → X → V → T
Exception Pattern I → H → G → F
Q65. BDFH : JLNP : : ? : KMOQ
(a) HIJK (b) CEGI Q73. (a) ACDF (b) TUOP
(c) EGJL (d) CFGI
Page 194 of 516
(c) HIVW https://telegram.me/aedahamlibra
(d) FGKL
DIRECTIONS: Select the related parts from the given
Ans: (a) Normal Pattern T → U, O → P
alternatives.
Exception Pattern A → C → D → F
Q82. HAND : JBPE :: PALM: ?
Q74. 10 : 91 :: 9 : ? (a) RBNM (b) RBMN
(a) 69 (b) 72 (c) QBNN (d) RBNN
(c) 89 (d) 97
Ans: (b) The relationship is x : (x2 – 9) . Ans: (d) HAND → JBPE
PALM → RBNN
Q75. 7 : 56 :: 9 : ?
(a) 63 (b) 81 Q83. Astronomy : Stars : : Geology :?
(c) 90 (d) 99 (a) Sky (b) Geometry
Ans: (c) The relationship is x : x (x + 1) (c) Science (d) Earth
Ans: (d) Astronomy is the branch of science which deals
DIRECTIONS: In each of the following questions, with celestial objects such as moons, planets, stars,
select the related parts from the given alternatives. galaxies etc. While geology is the science which deals
with the physical substance of the earth, their history, and
Q76. 20 : 50 :: 100 : ? the processes which act on them.
(a) 150 (b) 250
(c) 200 (d) 156 DIRECTIONS: Select the related parts from the given
Ans: (b) 20×2.5 = 50 alternatives.
100×2.5 = 250
Q84. Up : Down : : Back : ?
Q77. (a) ADHM (b) JNRX (a) Left (b) Deep
(c) EINT (d) FJOU (c) Front (d) Right
Ans: (b) Normal Pattern A → D → H → M Ans: (c) Second is the opposite of first.

Exception Pattern J → N → R → X Q85. 36 : 144 :: 49 : ?


(a) 196 (b) 154
Q78. 7 :133 :: 9 :? (c) 81 (d) 149
(a) 147 (b) 99 Ans: (a) 36 = 62 144 = 122 49 = 72 196 = 142
(c) 171 (d) 158
Ans: (c) 7 × 19 = 133 Similarly, 9 × 19 = 171 Q86. 76: 42:: 66: ?
(a) 36 (b) 63
Q79. 36 : 216 :: 81: ? (c) 12 (d) 35
(a) 729 (b) 629 Ans: (a) 7 × 6 = 42
(c) 319 (d) 826 6 × 6 = 36
Ans: (a) (6) 2 = 36 : (6) 3 = 216 Similarly, (9) 2 = 81; (9) 3
= 729 Q87. Picture : See :: Book: ?
(a) Listen (b) Library
Q80. Victory: Joy:: ? : Sorrow (c) Buy (d) Read
(a) Defeat (b) Depression Ans: (d) Picture is seen. Similarly, book is read.
(c) Melancholy (d) Cry
Ans: (a) Victory is joyful. Similarly, defeat is sorrowful. Q88. Dark : Light:: ?
(a) Ill : Diseased (b) Hot : Cold
Q81. Body: Stomach :: Library: ? (c) Polluted : Contaminated (d) Accrued: Accumulated
(a) Cash (b) Book Ans: (b) Both are antonyms.
(c) Headmaster (d) School
Ans: (b) Stomach is a part of body. Similarly, library has Q89. Swimming : River : : Hiking : ?
different kinds of books. (a) Mountain (b) Sea
Page 195 of 516
(c) Road https://telegram.me/aedahamlibra
(d) Pond
Ans: (a) Swimming is done in the river.
CXDW → XCWD

Hiking is done from mountain.


Q96. AB : NO : : LM : ?
(a) OL (b) KP
DIRECTIONS: In the following questions, select the (c) PK (d) YZ
related letter/ word from the given alternatives.
Ans: (d) AB → NO
Q90. Find out the set among the four sets which is like LM → YZ
the given set.
(23: 30: 37) Q97. AG : IO : : EK : ?
(a) (21 : 28 : 34) (b) (12 :19 : 25) (a) LR (b) MS
(c) (6 : 13 : 20) (d) (7 : 15 : 22) (c) PV (d) SY
Ans: (c) The given set is (23 : 30 : 37)
Ans: (b) AG → IO
= 23 + 7 = 30 + 7 = 37 Similarly, (6 : 13 : 20)
= 6 + 7 = 13 + 7 = 20 EK → MS

Q91. Horse : Neigh : : Bells : ? DIRECTIONS: In the questions , select the related
(a) Beat (b) Chime parts from the given alternatives.
(c) Rustle (d) Roar
Ans: (b) The sound made by horse is called Neigh. Q98. 25 : 175 :: 32 : ?
Similarly, the sound made by a bell is called chime. (a) 150 (b) 170
(c) 162 (d) 160
DIRECTIONS: In each of the following questions, Ans: (d) As,
select the related parts from the given alternatives. 25 25 × (2 + 5) = 175 Similarly, 32 32 × (3 + 2) =
160
Q92. A - E : R - V : : B - F : F - J
(a) CTDH (b) BSCG Q99. F : 216 :: L : ?
(c) FUGK (d) CSCG (a) 1700 (b) 1600
Ans: (a) As, (c) 1723 (d) 1728
Ans: (d) F 6 ; (6) 3 = 216 Similarly, L 12 ; (12) 3 =
1728

Q100. MOUSE : KPSTC : : LIGHT : ?


(a) MJHIU (b) MGHFU
Q93. Chisel : Sculptor :: Harrow? (c) JGEFR (d) JJEIR
(a) Gardener (b) Mason – – –
(c) Blacksmith (d) Guard Ans: (d) MOUSE → KPSTC
– – –
Ans: (a) Here, Tool –Worker relationship has been LIGHT → JJEIR
shown. Chisel is the tool of Sculptor. Si milarly, Harrow
is the tool of Gardener. Q101. Tanning : Leather :: Pyrotechnics : ?
(a) Fireworks (b) Wool
Q94. Moon : Satellite :: Earth? (c) Machinery (d) Bombs
(a) Sun (b) Planet Ans: (a) Tanning is the art of manufacturing leather.
(c) Solar system (d) Round Similarly, Pyrotechnics is the art of manufa cturing
Ans: (b) Moon is a Satellite. Earth is a Planet. fireworks.

Q95. BJCI : JBIC : : CXDW : ? DIRECTIONS: Select the related words/numbers


(a) JCDU (b) BCJU from the given alternatives.
(c) EVFU (d) XCWD
Ans: (d) BJCI → JBIC Q102. King : Throne :: Rider ?
(a) Horse (b) Seat
Page 196 of 516
(c) Saddle https://telegram.me/aedahamlibra
(d) Chair
Ans: (c) A king sits in a throne. Similarly, a rider sits on
synonymous to one another. Deduce means 'to infer', 'to
arrive at facts'. Infer means 'to arrive at a conclusion'.
a saddle on the back of a horse.
Q109. Fish : Gills : : Human : ?
Q103. A + B + Y + Z = 54 (a) Heart (b) Lungs
C+D+W+X=? (c) Nose (d) Mouth
(a) 45 (b) 54 Ans: (c) 'Gill' is the opening on the side of a fish's head
(c) 56 (d) 52 through which it breathes.
Ans: (b) A + B + Y + Z = 1 + 2 + 25 + 26 = 54 'Nose' is the part of the face above the mouth, used for
∴ C + D + W + X = 3 + 4 + 24 + 23 = 54 breathing and smelling.

Q104. Petrology : Rocks : : Palaeontology : ? Q110. A H O P : C K S U : : B J M F : ?


(a) Birds (b) Animals (a) D Q K M (b) C J W M
(c) Fossils (d) Soil (c) E Z U Q (d) D M Q K
Hint: 117 . (c) Petrology is the branch of geology that Ans: (d) A H O P → CKSU
deals with the origin, composition, structure and
alteration of rocks. BJMF→ DMQK
Therefore, palacontology is the study of fossils to
determine the structue and evolution of extinct animals Q111. Cup : Coffee : Bowl : ?
and plants and the age and the conditions of deposition of (a) Dish (b) Soup
the rock strata in which they are found. (c) Milk (d) Food
Ans: (b) Coffee is served in a cup and soup is served in a
Q105. Melt : Liquid : : Freeze : ? bowl.
(a) Crystal (b) Solid
(c) Ice (d) Evaporate Q112. BCDZ : CDEV : : DEFT : ?
Ans: (c) First is the process of formation of the second. (a) FFGQ (b) EGFR
(c) EFGP (d) EFGS

DIRECTIONS: In each of the followin g questions, Ans: (c) BCDZ → CDEV
select the related number/word/letter from given –
alternatives. DEFT → EFGP

Q106. 8 : 27 : : ? : 81 Q113. 57913 : EGIM : 35816 : ?


(a) 24 (b) 34 (a) CDIR (b) BCHP
(c) 44 (d) 54 (c) CEHQ (d) CEHP
Ans: (a) 27 × 3 = 81 ∴ 8 × 3 = 24 Ans: (d) Place value of litters in English Alphabet.

Q107. 9 : 7 : : 80 : ?
(a) 48 (b) 50
(c) 78 (d) 82
Ans: (c) (3) 2 = 9; (3) 2 – 2 = 7 Similarly, (3) 4 –1 = 81 –
1 = 80
(3) 4 – 3 = 81 – 3 = 78 Q114. 8 : 39 : : 72 : ?
In simple way, 9 – 2 = 7 and 80 – 2 = 78 (a) 64 (b) 312
(c) 351 (d) 300
Q108. Defer : Put off: : Deduce : ? Ans: (c) 8 × 9 = 72
(a) Infer (b) Protect Then, 39 × 9 = 351
(c) Delate (d) Oppose
Ans: (a) Defer means 'to delay something until a later DIRECTIONS: In each questions below, select the
time', 'to postpone'. Put off means 'to put something to a related parts from the given alternatives.
later time or date'. ' to delay'. Thus, Defer and Put off are
Q115. 16 : 36 : : 64 : ?
Page 197 of 516
(a) 4
(c) 8
https://telegram.me/aedahamlibra
(b) 6
(d) 100
Ans: (d) 42 = 16 62 = 36 82 = 64 102 = 100
Q123. Kalidas : Meghdoot : : Kautilya : ?
Q116. Saint : Meditation : : Scientist : ? (a) Ramayana (b) Arthashastra
(a) Research (b) Knowledge (c) Kamayani (d) Kadambari
(c) Spiritual (d) Rational Ans: (b) Meghdoot has been written by Kalidas.
Ans: (a) A saint practices meditation. Similarly, A Similarly,
scientist does research. Arthashastra has been written by kautilya.

Q117. King : Palace :: Eskimo : ? Q124. Water : Ocean : : Sand : ?


(a) Caravan (b) Asylum (a) Island (b) Waves
(c) Monastery (d) Igloo (c) River (d) Desert
Ans: (d) A palace is the official home of a King. Ans: (d) Ocean is the mass of water. Similarly, Desert is
Similarly, An igloo is a small round house of an Eskimo. the mass of sand.

Q118. (a) BFJNRV (b) CGKOSW DIRECTIONS: Select the related word/letter/ number
(c) DHLPTX (d) EIMPSW from the given alternatives.
Ans: (d) Normal Pattern B → F → J → N → R→ V
Q125. CUT : BDTVSU : : TIP : ?
Exception Pattern E → I → M → P → S → W (a) UVHJOQ (b) SUHJOQ
(c) USJHQO (d) SUJHOQ
Q119. (a) KJML (b) GFIH Ans: (b) Two letters, one preceding and second
(c) TSVU (d) ZABY following, are given for each letter.
Ans: (d) Normal Pattern K → J → M → L – – –
CUT → BDTVSU
Exception Pattern Z → A → B → Y – – –
TIP → SUHJOQ
Q120. 18 : 5 :: 12 : ?
Q126. Voyage : Sea sickness : : Heights : ?
(a) 4 (b) 10
(a) Ship (b) Travel
(c) 3 (d) 6
(c) Giddiness (d) Motion
Ans: (c) 18/3 – 1 = 5; 12/3 – 1 = 3
Ans: (c) Sea sickness is to feel ill when you are travelling
on a ship or boat. Similarly, Giddiness is to feel that you
DIRECTIONS: elect the related parts from the given
are going to fall.
alternatives.

Q127. Waitress : Restaurant : : ?


Q121. 12 : 20 :: ?
(a) Doctor : Nurse (b) Driver : Truck
(a) 15 : 37 (b) 16 : 64
(c) Teacher : School (d) Actor : Role
(c) 27 : 48 (d) 30 : 42
Ans: (c) Waitress is a person whose job is to serve
Ans: (d)
customers in a restaurant. Similarly, A teacher teaches
students in a school.

Q128. UNDATED : ATEDUND :: CORRECT : ?


Q122. 3 : 7 : : 15 : ? (a) PRECTOC (b) RECTROC
(a) 30 (b) 35 (c) ECTRORC (d) RECTCOR
(c) 45 (d) 49 Ans: (d) UND ATED → ATEDUND
Ans: (b)
Q129. METAPHOR : EMATHPRO ::
NORMAL : ?
3x = 15 × 7
(a) ORMLAN (b) ONRMLA

Page 198 of 516


(c) ONMRLA https://telegram.me/aedahamlibra
(d) ONMRAL
DIRECTIONS: In the following Nine questions, select
Ans: (c) ME TA PH OR → EMATHPRO
the related parts from the given alternatives.
Q130. 8 : 256 : : ?
Q137. Poem : Poet : : ? : Book
(a) 7 : 343 (b) 9 : 243
(a) Editor (b) Author
(c) 10 : 500 (d) 5 : 75
(c) Writer (d) Publisher
Ans: (c)
Ans: (b) A poet is a writer of a poem. Similarly, An
author is a writer of a book.

Q138. Optimist : Cheerful : : Pessimist : __?__


DIRECTIONS: Select the related parts from the given
(a) Petty (b) Helpful
alternatives.
(c) Gloomy (d) Mean
Ans: (c) An optimist person books upon the bright side
Q131. 21 : 3 : : 574 : ?
of things, hence remai ns cheerful always. Similarly, a
(a) 23 (b) 82
pessimist person books upon the dark side of things,
(c) 97 (d) 113
hence remains gloomy always.
Ans: (b) 21/3 = 7 Similarly, 574/x = 7, x = 574/7 = 82

Q139. EGIK : LJHF : : SUWY : ?


Q132. AGMS : CIOU: : DJPV : ?
(a) ZXVT (b) LNPR
(a) BHNY (b) STUV
(c) MOQS (d) TVXZ
(c) FLRX (d) MNOP
Ans: (a) EGIK → LJHF
Ans: (c) AGMS → CIOU
SUWY → ZXVT
DJPV → FLRX

Q140. 8 : 64 : : 27 : __?__
Q133. CAT: DDY : : BIG: ?
(a) 212 (b) 125
(a) CLL (b) CLM
(c) 64 (d) 81
(c) CML (d) CEP
Ans: (b) (2) ∧ 3 = 8 & (2+2) ∧ 3 = > (4) ↑ 3 = 64
Ans: (a) CAT→ DDY Similarly, (3) ∧ 3 = 27 & (3+2) ∧ 3 = > (5) ↑ 3 = 125
BIG→ CLL
Q141. 4 : 20 : : 6 : __?__
Q134. KMF : LLH : : RMS : ? (a) 8 (b) 42
(a) SLR (b) SLU (c) 36 (d) 12
(c) SSU (d) SUS Ans: (b) (4) 2 + 4 = 20
– (6) 2 + 6 = 42
Ans: (b) KMF → LLH

RMS → SLU Q142. 27 : 9 : : 64 : __?__
(a) 16 (b) 20
Q135. ACE : HIL : : MOQ : ? (c) 8 (d) 12
(a) TVX (b) TUX Ans: (a) 3 × 3 × 3 = 27; 3 × 3 = 9 Similarly, 4 × 4 × 4 =
(c) XVT (d) SUW 64; 4 × 4 = 16

Ans: (b) ACE → HIL


Q143. Yard : Inch : : Quart : __?__
MOQ → TUX (a) Milk (b) Liquid
(c) Gallon (d) Ounce
Q136. Lamp : Oil : : Electric Bulb : ? Ans: (d) A Yard is a larger measure than an inch. A quart
(a) Current (b) Bright is a larger measure than an ounce.
(c) Holder (d) Switch
Ans: (a) Oil is the fuel of lamp. Similarly, current is the DIRECTIONS: In each questions below, select the
fuel of electric bulb. related parts from the given alternatives.
Page 199 of 516
https://telegram.me/aedahamlibra
Q144. Pesticide : Crop : : Antiseptic : __?__
(a) USA
(c) Australia
(b) Russia
(d) China
(a) Bandage (b) Bleeding Ans: (a) Hockey is national game of India in the same
(c) Wound (d) Clothing way baseball is national game of USA.
Ans: (c) A perticide is a chemical preparation for
protection of crop from insects. Simi larly, an antiseptic is Q152. Ant : Fly : Bee :: Hamster : Squirrel : ?
a chemical preparation for protection of wound from (a) Rodent (b) Mouse
germs . (c) Cat (d) Spider
Ans: (b) Ant, fly and bee are insects w hile hamster,
Q145. 9 : 24 :: ? : 6 squirrel are rodents. Mouse is also a rodent.
(a) 3 (b) 2
(c) 1 (d) 5 Q153. 144 : 13 : : 49 : ?
Ans: (a) 9 × 3 – 3 = 24; 3 × 3 – 3 = 6 (a) 8 (b) 10
(c) 11 (d) 9
Q146. hive : bee :: eyrie : ? Ans: (a) (12) 2 : (12 + 1) Similarly (7) 2 : (7+1) = 8
(a) Pigeon (b) Sparrow
(c) Parrot (d) Eagle Q154. (a) DFGH (b) KMNO
Ans: (d) A hive is a shelter for bees (c) ABCD (d) RTUV
A eyrie is a large nest of an eagle. Ans: (c) Normal Pattern D → F → G → H

Q147. STAR : RATS :: WARD : ? Exception Pattern A → B → C → D


(a) MICE (b) SHINE
(c) FAME (d) DRAW Q155. (a) EGJK (b) PQST
Ans: (d) Letters are arranged in reverse order. (c) IKMN (d) MOWX
STARRATS Similarly, Ans: (b) Normal Pattern E → G → J → K
WARDDRAW Exception Pattern P → Q → S → T

Q148. AZBZ : CYDY :: EXFX : ? Q156. ABCD : WXYZ : : EFGH : ?


(a) FWFV (b) GWHW (a) STUV (b) STOU
(c) IVJW (d) HWIW (c) STUE (d) TSUV
– –
Ans: (b) AZBZ → CYDY
– –
Ans: (a) ABCD → WXYZ
EXFX → GWHW
Q157. Brick : Wall : : Cell :?
DIRECTIONS: Select the related parts from the given (a) Chlorophyll (b) Organs
alternatives. (c) DNA (d) Tissue
Ans: (d) Brick is a single unit of wall In the same way,
Q149. Choose the similar group of numbers on the cell is single unit of tissue.
basis of certain common properties they possess
(31, 19, 41) DIRECTIONS: select the related word/ letters/
(a) (23,33,51) (b) (16,27,11) numbers from the given alternatives.
(c) (27,29,47) (d) (29,17,37)
Ans: (d) All numbers are prime. Q158. 21 : 3 :: 574 : ?
(a) 23 (b) 82
Q150. 6 : 42 :: 5 : ? (c) 113 (d) 97
(a) 40 (b) 30 Ans: (b)
(c) 35 (d) 45
Ans: (c) 6 × 7 = 42; 5 × 7 = 35 Q159. BCDZ : CDEV : : DEFT : ?
So, 6 : 42 : : 5 : 35 (a) FFGQ (b) EGFR
(c) EFGP (d) EFGS
Q151. Hockey : India :: Baseball :?
Page 200 of 516
Ans: (c) BCDZ →
– https://telegram.me/aedahamlibra
CDEV
(c) (63,70,86) (d) (54,57,78)
Ans: (d) (72, 66, 96) all three numbers are multiples of 3.

DEFT → EFGP Similarly (54, 57, 78) are also multiples of 3.

Q160. Concord : Agreement : : Discord : ___? DIRECTIONS: In the following Six Questions, select
(a) Comparison (b) Conflict the related parts from the given alternatives.
(c) Association (d) Blended
Ans: (b) Concord means agreement. Similarly, Q168. √ √ √ √ ?__
Discord means disagreement or conflict. (a) √ √
(c) √ √
Q161. Female : Feminine: : Male :___?
Ans: (d)
(a) Man (b) Masculine
√ √
(c) Macho (d) Manager
Ans: (b) Feminine means qualities or appearance of a √ √
woman Similarly, masculine means qualities or √ √
appearance of a man. √ √

Q162. 42 : 20 : : 64 :____? √
(a) 33 (b) 34
(c) 31 (d) 32 Q169. Haematology : Blood : : Phycology : ?
Ans: (c) 20 × 2 + 2 = 42 Similarly 31 × 2 + 2 = 64 (a) Fungi (b) Fishes
(c) Algae (d) Diseases
Q163. PS : VY : : FI : ___? Ans: (c) Haematology is the branch of medicine
(a) VZ (b) WZ concerned with the study and prevention of diseases
(c) LO (d) UX related to the blood. Similarly, phycology is the scientific
Ans: (c) study of algae.
→ , →
Q170. Pride of Lions : : ________ of cats
Q164. Carpentry : Skill ___? :Talent (a) Herd (b) School
(a) Singing (b) Plumbing (c) Clowder (d) Bunch
(c) Driving (d) Masonry Ans: (c) A group of Lions is called a pride.
Ans: (a) Carpentry is a type of skill. Similarly, singing i s A group of Cats is called a clowder.
type of talent.
Q171. MAN : PDQ : : WAN : ?
Q165. : ____? (a) ZDQ (b) NAW
(c) YQD (d) YDQ
(a) (b) 1 Ans: (a)
(c)
Ans: (d) Q172. 2 : 32 : : 3 : ?
(a) 243 (b) 293
,
(c) 183 (d) 143
Ans: (a) 25 = 32
Q166. KcaC : CacK : : XgmF : ____? 35 = 243
(a) FmgX (b) Gmef
(c) EmgF (d) EgmX DIRECTIONS: In questions, select the related
Ans: (a) XgmF reverse order FmgX word/letters number from the given alternatives.

Q167. Choose the similar group of numbers on the Q173. D × H : 4 × 8 as M × Q: ?


basis of certain common properties they possess: (a) 12 × 17 (b) 12 × 16
(72, 66, 96) (c) 13 × 17 (d) 14 × 18
(a) (24,69,58) (b) (55,66,77)
Page 201 of 516
Ans: (c) https://telegram.me/aedahamlibra
DIRECTONS:: Select the related word/ letters/ numbers
from the given atternatives :
Q174. 12 : 72 : : 8 : ? Ans: (b) 01 : 36 : : 02 : ____
(a) 36 (b) 32 (1 + 5) 2 = 36 (2 + 5) 2 = 49
(c) 38 (d) 40
Ans: (b) ∴ Q181. Length : Meter :: Power : ?
(a) Calories (b) Degree
∴ (c) Watt (d) Kilogram
Ans: (c) As length is measured in meter so power in watt.
Q175. 6 : 42 : : 7 : ?
(a) 48 (b) 40 Q182. DHLPTX : BFJNRV : : CGKOSW : ?
(c) 56 (d) 52 (a) IJKLMN (b) UMQEIA
Ans: (c) 6 × 6 + 6 = 42 (c) AEIMQU (d) AIOMUQ
7 × 7 + 7 = 56 – – – – – –
Ans: (c) DHLPTX → BFJNRV
– – – – – –
Q176. Garbage : House : : Gangue : ? CGKOSW → AEIMQU
(a) Factory (b) Drugs
(c) Office (d) Ore DIRECTIONS: Select the related word/ letters/
Ans: (d) The waste of the house is called garbage. numbers from the given alternatives:
Similarly, the impurities in the ore are called gangue.
Q183. 42 : 20 :: 64 : ?
DIRECTIONS : Select the related word/ letters/ (a) 31 (b) 32
numbers from the alternatives. (c) 33 (d) 34
Ans: (a) 42 : 20 : : 64 : 31
Q177. DBU : EEZ : : CJH : ? 20 × 2 + 2 = 42
(a) DMN (b) DNM ∴31 × 2 + 2 = 64
(c) DNN (d) DMM
Ans: (d) Q184. Cytology : Cells :: ? : Birds
(a) Odontology (b) Mycology
similarly,
(c) Etymology (d) Ornithology
Ans: (d) Cytology is study of Cell similarly Ornithology
Q178. Pig : Sty : : Dog : ? is study of bird.
(a) Burrow (b) Hives
(c) Shed (d) Kennel Q185. FEDCBA : RQPONM : : LKJIHG :?
Ans: (d) Pig lives in Sty In the same way, Dog lives in (a) ABCDEF (b) MNOPQR
Kennel. (c) STUVWX (d) XWVUTS
Ans: (d) FEDCBA → RQPONM
Q179. Select the related letters from the given LKJIHG → XWVUTS
alternatives :
ACXZ : BDWY : : CEVX : ? DIRECTIONS: Select the related word/ letters/
(a) DFUZ (b) DFUW numbers from the given alternatives:
(c) DGUW (d) DFUV
– –
Ans: (b) ACXZ → BDWY Q186. 42 : 56 :: 110 : ?
– – (a) 18 (b) 132
CEVX → DFUW (c) 136 (d) 140
Ans: (b) 6 × 7 = 42; 7 × 8 = 56, : : 10 × 11 = 110, 11 ×
Q180. 01 : 36 : : 02 : ? 12 = 132.
(a) 48 (b) 49
(c) 69 (d) 70 Q187. Medicine : Patient : : Education : ?
(a) Teacher (b) School
Page 202 of 516
(c) Student https://telegram.me/aedahamlibra
(d) Tuition
Ans: (c) Medicine is given to patient. Similarly,
Ans: (d) As, Power is measured by Watt.
Work is measured by Joule.
Similarly,

Education is given to student.


Q195. Select the related letters from the given
Q188. Select the related letters from the given alternatives.
alternatives : HAPPY : ibqqz : : FRESH : ?
LNPR : TVXZ :: ? : NPRT (a) gsfti (b) gsdti
(a) FHJL (b) FLJH (c) gsftj (d) esftg
(c) HFLJ (d) JLHF Ans: (a) HAPPY → ibqqz
Ans: (a) LNPR → TVXZ FRESH → gsfti
– – – –
NPRT → FHJL
DIRECTIONS: In the following questions, select the
DIRECTIONS: In the following six questions, select related word/number from the given alternatives.
the related parts from the given alternatives.
Q196. 101 : 10201 : : 107 : ?
Q189. 13 : 20 : : 17 : ? (a) 10707 (b) 10749
(a) 25 (b) 26 (c) 11449 (d) 11407
(c) 27 (d) 28 Ans: (c) As, (101) 2 = 10201 Similarly, (107) 2 = 11449
Ans: (d) As,20 = 13 × 2 – 6
∴28 = 17 × 2 – 6 Q197. Car : Road : : Ship : ?
(a) Water (b) Air
Q190. Modesty : Arrogance : : ? (c) Road (d) Both Air and Water
(a) Debility : Strength (b) Obese : Fat Ans: (a) As the means of transport in 'Road' is 'Car',
(c) Ship : Fleet (d) Speak : Lecture Similarly the means of transport on 'Water' is 'Ship'.
Ans: (a) Modesty is the opposite of arrogance. Simil arly,
Debility is the opposite of strength. Q198. QDXM : SFYN : : UIOZ : ?
(a) PAQM (b) LPWA
Q191. 43 : 57 : : 111 : ? (c) QNLA (d) WKPA
(a) 135 (b) 133 Ans: (d) QDXM → SFYN
(c) 134 (d) 136
UIOZ → WKPA
Ans: (b) As, 62 + 7 = 43; (72) + 8 = 57 Similarly, (10) 2 +
11 = 111; (11) 2 + 12 = 133
DIRECTIONS: In the following questions, select the
related word pair from the given alternatives.
Q192. AZBY : CXDW : : EVFU : ?
(a) GTHS (b) FUVE
Q199. 5 : 124 : 6 : ?
(c) ZYEU (d) BXWD
(a) 215 (b) 216
Ans: (a)
(c) 217 (d) 220
Ans: (a) As, (5) 3 – 1 = 124, Similarly, (6) 3 – 1 = 215
DIRECTIONS: In the following questions, select the
related word pair from the given alternatives.
Q200. Kilometre : Metre : : Tonne : ?
(a) Litre (b) Kilogram
Q193. 23 : 0827 : : 45 : ?
(c) Hours (d) Weight
(a) 12125 (b) 64125
Ans: (b) As, Metre is a smaller unit of kilometre.
(c) 12025 (d) 08125
Similarly, Kilogram is a smaller unit of Tonne.
Ans: (b)
Q201. Select the related letters from the given
Q194. Power : Watt : : ? : ?
alternatives:
(a) Pressure : Newton (b) Force : Pascal
DCEF : QPRS :: XWYZ : ?
(c) Resistance : Mho (d) Work : Joule
(a) SRQP (b) NMOP

Page 203 of 516


(c) STOV https://telegram.me/aedahamlibra
(d) IMGF
Ans: (c) ACEG → ZXVT
Ans: (b) DCEF → QPRS
XWYZ → NMOP DIRECTIONS (232 to 235) : Select the related word/
letters/ number from the given alternatives.
DIRECTIONS: In the following question, select
related word pair/number from the given alternatives. Q208. 12 : 60 : : 28 : ?
(a) 160 (b) 150
Q202. 9143 : 9963 : : 6731 : ? (c) 145 (d) 140
(a) 1368 (b) 5666 Ans: (d) 12 × 5 = 60
(c) 8964 (d) 9694 28 × 5 = 140.
Ans: (c) As,
9143 9 + 1 + 4 + 3 = 17 Q209. Book : Literature :: ? : ?
9963 9 + 9 + 6 + 3 = 27 Similarly, 6731 6+7+3+ (a) Man : Beast (b) Dancer : Musician
1 = 17 (c) Song : Music (d) Species : Science
8964 8 + 9 + 6 + 4 = 27 Ans: (c) As, Book is a part of literature. Similarly, Song
is a part of music.
Q203. Fire : Burn : : ? : ?
(a) Water : Drink (b) Wood : Tress Q210. 21 : 3 :: 574 : ?
(c) Ice : Freeze (d) Flower : Rose (a) 97 (b) 23
Ans: (c) As, Fire will burn Similarly, Ice will freeze. (c) 82 (d) 113
Ans: (c) As, 21/3 = 7 Similarly, 574/82 = 7
Q204. Select the related letters from the given
alternatives : DIRECTIONS: In the following question, select the
EGIK : FILO :: FHJL : ? related word from the given alternatives.
(a) JGMP (b) JGPM
(c) GJMP (d) GMJP Q211. Giant : Dwarf :: Genius : ?
Ans: (c) EGIK → FILO (a) Tiny (b) Gentle
(c) Idiot (d) Wicked
FHJL → GJMP
Ans: (c) As, opposite word of 'giant' is 'dwarf' Similarly,
opposite word of 'idiot' is 'genius'.
DIRECTIONS: Choose the related word/letters
number from the given alternatives.
Q212. Influenza: Virus :: Ringworm: ?
(a) Bacteria (b) Fungi
Q205. 3 : 27 : : 4 : ?
(c) parasite (d) Protozoa
(a) 63 (b) 64
Ans: (b) As, Infuenza is caused by virus. Similarly, Ring
(c) 65 (d) 15
worm is caused by Fungi.
Ans: (b) As, (3) 3 = 27 Similarly, (4) 3 = 64.
Q213. (a) A B D G (b) C D F I
Q206. Horse : Neigh : : Elephant : ?
(c) E F H K (d) G H J K
(a) Quack (b) Trumpet
(c) Mew (d) Grunt Ans: (d) Normal Pattern A → B → D → G
Ans: (b) Sound of horse is neigh. Similarly, Sound of Exception Pattern G → H → J → K
elephant is trumpet.
Q214. (a) N M L (b) O P Q
Q207. Select the related letters from the given (c) X W V (d) H G E
alternatives :
Ans: (d) Normal Pattern N → M → L
ACEG : ZXVT :: IKMO : ?
(a) MNOP (b) PQRS Exception Pattern H → G → E
(c) RPNL (d) LNPR
Q215. 11 : 120 :: 13 : ?
(a) 165 (b) 168
Page 204 of 516
(c) 170
Ans: (b) As, (11) 2 – 1
https://telegram.me/aedahamlibra
(d) 169
121 – 1 = 120 Similarly, (13) 2
Ans: (b) As, Plants is related to garden. Similarly, Pages
is related to book.
– 1 169 – 1 = 168

DIRECTIONS: In the following question, select the


Classification
related from the given alternatives.

Q216. 103 : 10609 : : 106 : ? DIRECTIONS: In each of the following questions,


(a) 10606 (b) 10306 select the one which is different from the other three
(c) 11236 (d) 13636 responses.
Ans: (c) As, (103) 2 = 10609 Similarly, (106) 2 = 11236
Q1. Doctor : Patient : : Teacher : ?
Q217. 11 : 121 : : 12 : ? (a) Teach (b) Student
(a) 144 (b) 169 (c) School (d) Old
(c) 196 (d) 154 Ans: (b) As, Patient is cared by doctor Similarly, Student
2
Ans: (a) According to pattern, (11) = 121 Similarly, is taught by teacher.
(12) 2 = 144
Q2. (a) Heat (b) Light
Q218. 15 : 256 : : 14 : ? (c) Bulb (d) Electricity
(a) 225 (b) 144 Ans: (c) Bulb is an item while all others are phenomena.
(c) 25 (d) 95
Ans: (a) As, (15 + 1) 2 = (16) 2 = 256 Similarly, (14 + 1) Q3. (a) Wave (b) Current
2
= (15) 2 = 225. (c) Tide (d) Storm
Ans: (c) Except Tide, all other terms are related to both
Q219. 108 : 11664 : : 107 : ? air and water. But tide is a regular rise and fall in the
(a) 11449 (b) 10449 level of sea, caused by the attraction of the moon and sun.
(c) 10849 (d) 11749
Ans: (a) As, Q4. (a) X (b) Y
(108) 2 = 11664 Similarly, (107) 2 = 11449. (c) H (d) D
Ans: (b) The position number of Y in the English
Q220. (a) BFJNRV (b) CGKOSW alphabet is an odd number.
(c) DHLPTX (d) EIMPSW X 24, Y 25 H 8, D 4
Ans: (d) Normal Pattern B → F → J → N → R→ V
DIRECTIONS : In each of the following questions
Exception Pattern E → I → M → P → S → W
find the odd parts from the given responses.
Q221. (a) KJML (b) GFIH
Q5. (a) ABJNM (b) QRTUZ
(c) TSVU (d) ZABY
(c) IXYOQ (d) WFGOP
Ans: (d) Normal Pattern K → J → M → L Ans: (c) Except in letter group IXYOQ, in all others
Exception Pattern Z → A → B → Y there is only one vowel. In the letter group IXYOQ, there
are two vowels.
Q222. Rain : Clouds : : Heat : ?
(a) Distance (b) Sun Q6. (a) Room (b) Chamber
(c) Night (d) Day (c) Veranda (d) Cabin
Ans: (b) As, Rain is produced by clouds Similarly, Heat Ans: (c) Except Veranda, all others are surrounded by
is produced by Sun. four walls.

Q223. Garden : Plants : : Book : ? Q7. (a) A (b) I


(a) Words (b) Pages (c) D (d) E
(c) Writing (d) White Ans: (c) Except D, all others are Vowels.

Page 205 of 516


Q8. (a) RNMP
https://telegram.me/aedahamlibra
(b) JFEH Q16. (a) Softball (b) Baseball
(c) RPOQ (d) HDCF (c) Cricket (d) Basketball
Ans: (c) Normal Pattern R → N → M → P Ans: (d) Basketball is different from others. In Softball,
Baseball and Cricket, bats and balls are used, Basketball
Exception Pattern R → P → O → Q involves only ball.

Q9. (a) AbcdE (b) IfghO Q17. (a) PRSU (b) SUWY
(c) ApqrL (d) UlmnE (c) HJLN (d) CEGI
Ans: (c) Except in ApqrL, in all others the first and the
last letters given in capital are vowels. Ans: (a) Normal Pattern S → U → W → Y
Exception Pattern P → R → S → U
Q10. (a) 6243 (b) 2643
(c) 8465 (d) 4867 Q18. (a) Tomato (b) Potato
Ans: (d) 6 + 2 = 8, 4 + 3 = 7 8 – 7 = 1 (c) Carrot (d) Onion
2 + 6 = 8, 4 + 3 = 7 8 – 7 = 1 Ans: (a) Except Tomato all others are grown
8 + 4 = 12, 6 + 5 = 11 12 – 11 = 1 underground.
4 + 8 = 12, 6 + 7 = 13 12 – 13 = –1
Q19. (a) 125 (b) 789
DIRECTIONS : In each of the following questions, (c) 236 (d) 347
find the odd number / letters/ word from the given Ans: (a) The number 125 is a perfect cube. 5 × 5 × 5 =
alternatives. 125

Q11. (a) 49 – 33 (b) 62 – 46 DIRECTIONS: In questions, find the odd number/


(c) 83 – 67 (d) 70 – 55 letters/ word/ number pair from the given
Ans: (d) 49 – 33 = 16, 62 – 46 = 16 alternatives.
83 – 67 = 16 But, 70 – 55 = 15
Q20. Find out the pair of numbers that does not
Q12. (a) Flute (b) Violin belong to the group for lack of common property.
(c) Guitar (d) Sitar (a) 16 – 18 (b) 56 – 63
Ans: (a) Except Flute, all others are stringed musical (c) 96 – 108 (d) 86 – 99
instruments. Ans: (d) As 2 × 8 = 16;2 × 9 = 18
7 × 8 = 56;7 × 9 = 63
Q13. (a) JKOP (b) MNST 12 × 8 = 96;12 × 9 = 108
(c) CABD (d) OPWX 86/8 = 10.75; 99/9 = 11
Ans: (b) Except MNST, all others are having a vowel. Also, only (d) is coprime.

Q14. (a) 156 (b) 201 Q21. (a) Major (b) Colonel
(c) 273 (d) 345 (c) Brigadier (d) Admiral
Ans: (a) Except 156, all other numbers are odd numbers. Ans: (d) Major, Colonel and Brigadier are different ranks
in the Indian Army. Admiral is the topmost rank in the
DIRECTIONS: In question, find the odd Indian Navy.
number/letters/ number pair from the given
alternatives. Q22. (a) Life Insurance Corporation.
(b) New India Assurance Company Ltd.
Q15. Find out the pair of numbers that does not (c) United India Insurance Company Ltd.
belong to the group for lack of common property. (d) National Insurance Company Ltd.
(a) 11–115 (b) 10–90 Ans: (a) Except Life Insurance Corporation, all others
(c) 9–72 (d) 8–56 are insurance companies for general insurance, i.e., for
Ans: (a) vehicles, property etc.
;
Page 206 of 516
Q23. (a) Socrates
(c) Mozart
https://telegram.me/aedahamlibra
(b) Beethoven
(d) Bach
Q31. (a) VEIIDD
(c) VEENS
(b) IVEF
(d) EINN
Ans: (a) Socrates was a Greek philosopher. Ludwing Ans: (a) IVEF FIVE
Van Beethoven was a G erman Composer and Musician. VEENS SEVEN
Bach was also a German Composer. WA Mozart was a EINN NINE
Austrian Compose. VEIIDD DIVIDE

Q24. (a) (132, 5) (b) (125, 8) Q32. (a) Arunachal Pradesh (b) Jammu and Kashmir
(c) (124, 7) (d) (112, 4) (c) Haryana (d) Himachal Pradesh
Ans: (d) 112 is completely divisible by 4. Ans: (a) Jammu and Kashmir, Haryana and Himachal
Pradesh are northern States of India. Arunachal Prade sh
is estern-most States of India.

DIRECTIONS: In each of the following questions,


DIRECTIONS: In questions, find the odd find the odd word/number/ parts pair from the given
number/letters/number pair from the given alternatives.
alternatives.
Q33. (a) Board (b) Commission
Q25. (a) 6246 – 6296 (b) 7137 – 7267 (c) Team (d) Agenda
(c) 4344 – 4684 (d) 5235 – 5465 Ans: (d) Except Agenda, all other denotes a group of
Ans: (a) The difference between the two numbers in the persons.
number pair 6246 – 6296 is least.
6296 – 6246 = 50 Q34. (a) ECDBA (b) OMNLK
7267 – 7137 = 130 (c) WUVTS (d) SRTQP
4684 – 4344 = 340
Ans: (d) Normal Pattern E → C → D → B A
5465 – 5235 = 230
Exception Pattern S → R → T → Q P
Q26. (a) (125, 27) (b) (64, 216)
(c) (216, 02) (d) (343, 01) Q35. (a) BE (b) GJ
Ans: (c) Except in (216, 02) in all others both the (c) NP (d) QR
numbers are perfect cubes. Ans: (a) Only BE has a vowel

Q27. (a) Division (b) Addition Q36. (a) 400 (b) 484
(c) Subtract (d) Multiplication (c) 625 (d) 729
Ans: (c) Except Subtract, all others are Nouns. The Noun Ans: (d) Except the number 729, all others are only
for subtract (Verb) is Subtraction. perfect squares.

Q28. (a) 47 (b) 17 Q37. (a) 1000 (b) 1725


(c) 27 (d) 37 (c) 2744 (d) 4096
Ans: (c) 27 is a perfect cube. All other numbers are Ans: (b) Except the number 1725, all other numbers are
Prime Numbers. completely divisible by 4. The number 1725 is
completely divisible by 5.
Q29. (a) 5270 – 2936 (b) 186 – 69 ;
(c) 168 – 570 (d) 1001 – 100
But,
Ans: (b) 1 + 8 + 6 = 15 and 6 + 9 = 15

Q30. (a) 8 5 3 6 2 (b) 6 3 8 5 2 Q38. (a) 12–16 (b) 60–80


(c) 5 2 6 3 8 (d) 2 8 7 5 1 (c) 30–50 (d) 36–48
Ans: (d) Except in the number 28751, in all others the Ans: (c) Except the number pair 30 – 50, all other
digits are the same. numbers pairs has ratio = 3/4

Page 207 of 516


Q39. (a) CX
(c) JQ
https://telegram.me/aedahamlibra
(b) DW
(d) LR DIRECTIONS: In questi on, find the odd word/
Ans: (d) Except LR, all others are pairs of opposite number/letters from the given alternatives.
letters.
Q48. (a) 704, 11 (b) 256, 4
Q40. (a) Cyclotron (b) Basic (c) 832, 13 (d) 310, 5
(c) Pascal (d) Fortran Ans: (d) 704 ÷ 11 = 64, 256 ÷ 4 = 64
Ans: (a) Except Cyclotron, all others are languages of 832 ÷ 13 = 6, 310 ÷ 5 = 62
computer. Cyclotron is an apparatus in which charged
atomic and subatomic particles are accelerated by an Q49. (a) Sparrow (b) Kingfisher
alternating electric field while following an outward (c) Nightingale (d) Bat
spiral or circular path in a magnetic field. Ans: (d) Except (d) , All (a) , (b) and (c) are synonyms
of bird.
Q41. (a) Rooster (b) Buck
(c) Gander (d) Peahen Q50. (a) 206 (b) 125
Ans: (b) Rooster, Gander and Peahen are birds. Buck is (c) 27 (d) 8
an animal. Ans: (a) Except (a) , other can be written in form of x3.

DIRECTIONS: Find the odd number / letters / Q51. (a) GALLOP (b) SINK
number pair from the given alternatives. (c) ALBINO (d) CRAB
Ans: (c) The first and last are vowels.
Q42. (a) Booklet (b) Pamphlet
(c) Voucher (d) Brochure Q52. (a) Crew (b) Company
Ans: (c) Except Voucher, all others are small book (c) Army (d) Ants
containing information on a specific subject. Voucher is a Ans: (d) Options (a) , (b) and (c) are related to
small printed piece of paper which can be exchanged for human beings but option (d) represents insects.
certain goods and services.
Q53. (a) eMnT (b) iRsL
Q43. (a) Konark (b) Madurai (c) oKsZ (d) vGtN
(c) Dilwara (d) Ellora Ans: (d) One small letter is followed by capital letter but
Ans: (d) All except Ellora are famous for temples, while in option (d) , the firs t letter is not vowel while in others
Ellora is famous for caves. first letter is vowel.

Q44. (a) RTW (b) QOM DIRECTIONS: Select the one which is different from
(c) IKG (d) IKM the other three responses.
Ans: (a) Consonants and vowel are used in option (b) ,
(c) and (d) . Only consonants are Q54. 1, 2, 4, 5
used in option (a) . 2.6, 7, 14, 15
3.3, 4, 8, 9
Q45. (a) DH (b) FJ 4.4, 5, 10, 12
(c) HK (d) PR (a) 1 (b) 3
Ans: (c) Pair of odd and even number (c) 4 (d) 2
Ans: (c) Except (c) , all othe r options represents pairs of
Q46. (a) 24 (b) 49 consecutive numbers.
(c) 80 (d) 15
Ans: (b) 24 = 52 – 180 = 92 – 1 Q55. (a) steering wheel (b) engine
49 = 72 – 015 = 42 – 1 (c) car (d) tyre
Ans: (c) All are parts of car.
Q47. (a) 121 (b) 324
(c) 523 (d) 729 Q56. (a) 325 (b) 360
Ans: (c) All are squares except (c) . (c) 230 (d) 256
Page 208 of 516
https://telegram.me/aedahamlibra
Ans: (d) All are divisible by 5 except 256. Ans: (c) Normal Pattern D → F → G → H

Q57. (a) uranus (b) pluto Exception Pattern A → B → C → D


(c) jupiter (d) sun
Ans: (d) All are planets except sun. Q65. (a) (1, 0) (b) (2, 3)
(c) (3, 8) (d) (4, 27)
DIRECTIONS: In question, find the odd word/ Ans: (b) In the number pair (2, 3) both the numbers are
number from the given alternatives. prime numbers.

Q58. (a) LRO (b) CJG DIRECTIONS: In question, find the odd
(c) SZW (d) PWT word/number/number pair from the given
Ans: (a) alternatives.

Q59. (a) Franc (b) Pound (sterling) Q66. (a) (96, 24) (b) (39, 18)
(c) Deutsche Mark (d) Yen (c) (81, 54) (d) (82, 64)
Ans: (d) Yen is the currency of Japan, an Asian county. Ans: (d) Except the number pair (82, 64) , in all other
Pound (Sterling) , Deutsche Mark and Franc are number pairs both the numbers are multiples of 3.
currencies of European countries. 96 = 32 × 3; 24 = 8 × 3
39 = 13 × 3; 18 = 6 × 3
Q60. (a) Yellow Green (b) Yellow Orange 81 = 27 × 3; 54 = 18 × 3
(c) Red Yellow (d) Red Orange
Ans: (b) Red, Green and Blue are called primary colours Q67. (a) Rickets (b) Night-blindness
or basics colours. (c) Influenza (d) Scurvy
Red + Green Yellow colour Except in option (b) , in Ans: (c) Except Influenza, all others are diseases caused
all others there is one primary colour. by deficiency of vitamins while, influenza is caused by
virus.
DIRECTIONS: Select the one which is different from
the other three responses. Q68. (a) Shower (b) Sleet
(c) Raisin (d) Rain
Q61. 27, 125, 216, 343 Ans: (c) Except Raisin, all others are different forms of
(a) 27 (b) 125 precipitation.
(c) 216 (d) 343
Ans: (c) 27 = 3 × 3 × 3 Q69. (a) 71 – 55 (b) 517 – 523
125 = 5 × 5 × 5 (c) 248 – 231 (d) 547 – 563
216 = 6 × 6 × 6 Ans: (c) 248 – 231 = 17
343 = 7 × 7 × 7 563 – 547 = 16
The number 216 is a perfect cube of an even number. 71 – 55 = 16
523 – 517 = 06
Q62. (a) Shimla (b) Darjeeling Except in the option (c) , in all other number pairs, the
(c) Ooty (d) Agra difference between the two numbers is an even number.
Ans: (d) Except Agra, all others are hill stations.
Q70. 10, 13, 234, 681, 997
Q63. (a) Foal (b) Hen (a) 234 (b) 13
(c) Lamb (d) Leveret (c) 681 (d) 10
Ans: (b) Foal is the young one of a horse or of a related Ans: (c) 10 1 + 0 = 1
animal. Lamb is the young one of the sheep. 13 1 + 3 = 4
Leveret is the young one of a hare. 234 2 + 3 + 4 = 9
Hen is a female bird especially of a fowl. 681 6 + 8 + 1 = 15
997 9 + 9 + 7 = 25
Q64. (a) DFGH (b) KMNO Consecutive square numbers are given.
(c) ABCD (d) RTUV
Page 209 of 516
https://telegram.me/aedahamlibra
DIRECTIONS: Find the odd words/letters/number
pair from the given alternatives. Q78. (a) Nephew (b) Niece
(c) Uncle (d) Father
Q71. (a) Heed (b) Led Ans: (b) Except (b) all others are males.
(c) Heap (d) Head
Ans: (d) Except in the word Led, in all other words there Q79. (a) AIUE (b) PIRE
are two vowels. (c) VALT (d) TORE
Ans: (a) All are vowels in 'AIUE'
Q72. (a) Fervent (b) Enthusiastic
(c) Apathetic (d) Ardent Q80. (a) 51 (b) 171
Ans: (c) Fervent: having or displaying a passionate (c) 291 (d) 322
intensity. Ans: (d) Only 322 is an even number.
Enthusiastic: having or showing intense and eager
enoyment. DIRECTIONS: In the following questions, select the
Ardent: very enthusiastic or passionate one which is different from the other three responses.
Apathetic: showing or feeling no interest, enthusiasm or
concern. Q81. 48, 80, 120, 144, 192, 208
Hence, apathetic is odd one out. (a) 80 (b) 120
(c) 144 (d) 192
Q73. (a) FAA (b) OFF Ans: (c) Rest are not perfect square.
(c) ATT (d) EPP
Ans: (a) In all other groups, a vowel is followed by a Q82. (a) Teaching (b) Counselling
consonant repeated twice. (c) Instruction (d) Guidance
Ans: (a) Teaching is different from the other three words.
Q74. (a) 117 – 143 (b) 142 – 156
(c) 64 – 78 (d) 103 – 169 Q83. (a) (25, 49) (b) (121, 169)
Ans: (a) Only 117-143 is divisible by 13. Theref ore, it is (c) (7, 169) (d) (9, 25)
odd one out. Ans: (c) Except in the number pair (7, 169) in all other
number pairs both the numbers are perfect squares.
DIRECTIONS : In each of the following questions, (25, 49) [ (5) 2, (7) 2]
select the one which is different, from other three (121, 169) [ (11) 2, (13) 2]
responses. (9, 25) [ (3) 2, (5) 2]

Q75. (a) Radio (b) Television Q84. (a) HEAT (b) MEAT
(c) Transistor (d) Tube (c) MEET (d) BEAT
Ans: (d) Radio, Transistor and television are the way of Ans: (c) In the word MEET, the second and the third
broadcasting. Hence, tube is odd one out. letters are the same.

Q76. (a) Soldier – Barrack (b) Principal – School Q85. (a) 8465 (b) 2643
(c) Artist – Troupe (d) Singer – Chorous (c) 4867 (d) 6243
Ans: (b) Barrack is a large building or group of buildings Ans: (b) 8465 84 – 65 = 19
where soldiers live. Principal is the head of school. 2643 43 – 26 = 17
Troupe is a group of artists. Chorous is a large group of 4867 67 – 48 = 19
singers. 6243 62 – 43 = 19

Q77. (a) 74, 7 (b) 97, 9 Q86. (a) ADHM (b) JNRX
(c) 41, 4 (d) 63, 6 (c) EINT (d) FJOU
Ans: (b) The second number in each number is the first
Ans: (b) Normal Pattern A → D → H → M
digit of the first number.
The difference between the two digits of the first number Exception Pattern J → N → R → X
is 3 in all the number pairs except 97, 9.
Page 210 of 516
https://telegram.me/aedahamlibra
DIRECTIONS : Find the odd num ber/word from the
given alternatives.
(c) CABD (d) OPWX
Ans: (c) Normal Pattern J → K → O → P

Q87. (a) Sitar (b) Violin Exception Pattern C → A → B → D


(c) Tanpura (d) Veena
Ans: (b) Violin is a musical instrument with four strings Q97. (a) 46-54 (b) 71-17
of treble pitch played with a bow. (c) 31-13 (d) 79-97
Ans: (a) Except (a) in all other pairs second term is
Q88. (a) 5 : 25 (b) 8 : 64 reverse order of first term.
(c) 7 : 49 (d) 6 : 30
Ans: (d) Second number is a square of first number. i.e., Q98. (a) T14 (b) T18
5: (5) 2, 8: (8) 2, 7: (7) 2. Therefore, 6:30 is odd one out. (c) T12 (d) T21
Ans: (d) Except (d) all oter have even numbers.
Q89. (a) 245 (b) 443
(c) 633 (d) 821 DIRECTIONS: Find the odd word/number/
Ans: (c) Sum of all digits of given numbers are 11. But letters/number pair from the given alternatives.
option (c) shows the sum ‗12‘. Therefore, it is odd one
out. Q99. (a) ARBT (b) KVLW
(c) PXQY (d) EPFQ
DIRECTIONS: In questions, find the odd words Ans: (a)
/letters / number pair from the given alternatives.
Q100. (a) Obstetrician (b) Podiatrist
Q90. (a) Green gram (b) Millet (c) Pulmonologist (d) Prosthetist
(c) Barley (d) Wheat Ans: (d) An Obsterician is a medical doctor who
Ans: (a) specializes in the management of pregnancy, labor and
birth.
Q91. (a) Poet (b) Publisher A Podiatrist is a doctor who diagonose and treat
(c) Author (d) Novelist conditions of the foot, ankle, and related structures of the
Ans: (b) Except (b) others are creators. leg. A Pulmonologist is a physician who possesses
specilised knowledge and skill in the diagonosis and
Q92. (a) KHEB (b) WTQN treatment of lung conditions and disease. A Prosthetist is
(c) MKHE (d) MIDA a device designed to replace a missing part of the body or
Ans: (b) Except (b) , vowel is present. to make a part of body work better. Hence (d) is odd one
out.
Q93. (a) 41, 52 (b) 64, 73
(c) 46, 57 (d) 38, 49 Q101. (a) Century (b) Decadent
Ans: (b) (c) Year (d) Month
Ans: (b) Except (b) , all other are related to a period.
DIRECTIONS: Find the odd parts pair from the While decadent is a person who has interest only in
given alternatives. pleasure and enjoyment.

Q94. (a) Hexagon : Angle (b) Square : Line Q102. (a) FEDC (b) STUV
(c) Circle: Arc (d) Line : Dot (c) LKJI (d) RQPO
Ans: (a) Hexagon not made from angle. Ans: (b) Except (b) all others have consecutive letters in
reverse order.
Q95. (a) Cricket (b) Football While (b) has consecutive letters
(c) Polo (d) Hockey in forward order.
Ans: (b) Only football doesn't require the use of
handheld apparatus. Q103. (a) LPXOY (b) RQST
(c) FBDLX (d) MPONL
Q96. (a) JKOP (b) MNST Ans: (b) Except (b) all others have five letters.
Page 211 of 516
Q104. (a) 14 - 16
https://telegram.me/aedahamlibra
(b) 56 - 64 Q113. (a) 50 (b) 120
(c) 77 - 88 (d) 80 - 93 (c) 37 (d) 145
Ans: (d) Except (d) in both number in all others pairs a re Ans: (c) All others except (c) are the multiples of 5.
divided by same number.
Q114. (a) BS (b) EV
DIRECTIONS: In the following Six Questions, find (c) DX (d) IZ
the old word/ number/ letters / number pair from the Ans: (b) Only in option (b) first letter is 4th from the left
given alternatives. end and second letter is 4th from the right end. Others
have no sequence
Q105. (a) 13 - 21 (b) 19 - 27
(c) 15 - 23 (d) 16 - 24 Q115. (a) OKHA (b) PERU
Ans: (d) Except (d) All others have odd numbers. (c) ERKU (d) ANDI
Ans: (b) All except ‗PERU‘ have vowel in both ends.
Q106. (a) Morning (b) Noon
(c) Evening (d) Night DIRECTIONS: In questions below, find the odd parts
Ans: (d) Except (d) , all others belong to the period of pair/figure from the given alternatives.
before sunset.
Q116. (a) 200 (b) 191
Q107. (a) Liberty (b) Society (c) 808 (d) 1331
(c) Equality (d) Fraternity Ans: (d) 1331 is the cube of 11
Ans: (b) Except (b) , all others are principles of society.
Q117. (a) Tennis (b) Cricket
Q108. (a) EGKQ (b) CEIO (c) Volleyball (d) Football
(c) LNQW (d) PRVB Ans: (b)
Ans: (c) Normal Pattern E → G → K → Q
Q118. (a) 68 (b) 85
Exception Pattern L → N → Q → W (c) 153 (d) 174
Ans: (d) Except (d) all others are divided by 17.
Q109. (a) 4025 (b) 7202
(c) 6023 (d) 5061 DIRECTIONS: In questions, select the one which is
Ans: (d) 4 + 0 + 2 + 5 = 11 different from the other three alternatives.
6 + 0 + 2 + 3 = 11
7 + 2 + 0 + 2 = 11 Q119. (a) 7 : 98 (b) 9 : 162
5 + 0 + 6 + 1 = 12 (c) 12 : 288 (d) 17 : 572
Hence, (d) is the odd one out. Ans: (d) Except (d) in all others second number is
divisible by the first one.
DIRECTIONS: In questions, find the odd word/letters
/numbers from the given alternatives. Q120. (a) 42 – 49 (b) 35 – 62
(c) 63 – 70 (d) 28 – 21
Q110. (a) 96 : 80 (b) 64 : 48 Ans: (b) Except (b) all others are divisible by 7.
(c) 80 : 60 (d) 104 : 78
Ans: (a) DIRECTIONS: In Question, find the odd
word/number letters/number pair from the gi ven
Q111. (a) Lawyer (b) Legislator alternatives.
(c) Mayor (d) Governor
Ans: (a) All others except (a) have official rankings Q121. (a) Engineer (b) School
(c) Lawyer (d) Doctor
Q112. (a) Acute (b) Parallel Ans: (c) Except (b) others are connected with a job that
(c) Right (d) Obtuse needs special skill, while school is an organisation.
Ans: (b) All other except (b) are forms of angles
Page 212 of 516
Q122. (a) BDGK
(c) FGJK
https://telegram.me/aedahamlibra
(b) PRUY
(d) KMPT
(c) 9100
Ans: (d) (a) 8 + 1 + 1 + 0 = 10
(d) 1189

Ans: (c) Normal Pattern B → D → G → K (b) 1 + 2 + 3 + 4 = 10


(c) 9 + 1 + 0 + 0 = 10
Exception Pattern F → G → J → K (d) 1 +1 + 8 + 9 = 19
Hence, (d) is the odd one out.
Q123. (a) MSWCH (b) NSWAH
(c) GMRVY (d) UZEIL Q132. (a) Brass (b) Tin
Ans: (c) Normal Pattern M → S → W → 0 C H (c) Bronze (d) Steel
Ans: (b) Tin is element but rest are the alloys.
Exception Pattern G → M → R → V Y

DIRECTIONS: In question, find the odd parts pairs


Q124. (a) BADC (b) JILK
from the given alternatives.
(c) NMPO (d) VUWX
Ans: (d) Normal Pattern B → A → D → C Q133. (a) TEAP (b) QIEM
Exception Pattern V → U → W → X (c) HIUL (d) ZOIV
Ans: (c) HIUL
Q125. (a) D C E B (b) P N Q S T
(c) V K H G M (d) W P Z L H Q134. (a) Tea : Beverages (b) Legumes : Nodules
Ans: (a) Except (a) all others have five letters. (c) Beans : Pulses (d) Rice : Cereals
Ans: (b) Tea is a type of beverage, rice is type of cereal
Q126. (a) 512 (b) 625 and beans is a type of pulse but nodule is source of
(c) 1296 (d) 2401 nitrogen for legume plant.
Ans: (a) Except (a) all others are squares.
252 = 625 Q135. (a) JKOP (b) MNST
362 = 1296 (c) CABD (d) OPWX
492 = 2401 Ans: (c) Normal Pattern J → K → O → P
Exception Pattern C → A → B → D
Q127. (a) Poland (b) Korea
(c) Spain (d) Greece
Q136. (a) CAFD (b) TSWV
Ans: (b) Except (b) all others are countries in Europe
(c) IGLJ (d) OMRP
while Korea is situated in East Asia.
Ans: (a) Normal Pattern T → S → W → V
DIRECTIONS: In questions below, find the odd Exception Pattern C → A → F → D
number/letters/number pair form the given
alternatives. Q137. (a) DHLP (b) TXBF
(c) JNRV (d) YBEM
Q128. (a) Polaris (b) Nike
(c) Crux (d) Phoenix Ans: (d) Normal Pattern D → H → L → P
Ans: (b) Except (b) all others mean the central point. Exception Pattern Y → B → E → H

Q129. (a) (47,49) (b) (5,7) Q138. (a) EBD (b) IFH
(c) (29,31) (d) (11,13) (c) QNO (d) YVX
Ans: (a) All others except (a) are prime numbers. Ans: (c) Normal Pattern E → B → D

Q130. (a) Marigold (b) Lotus Exception Pattern Q → N → O


(c) Tulip (d) Rose
Ans: (b) Lotus grows in the water but rest grow on the Q139. (a) (36,27) (b) (23,14)
land. (c) (82,29) (d) (45,18)
Ans: (c) 36, 27 Sum of digits in both is 9
Q131. (a) 8110 (b) 1234 23, 14 Sum of digits in both is 5
Page 213 of 516
45, 18 Sum of digits in both is 9 https://telegram.me/aedahamlibra
But, in (82, 29) sum of digits is different. DIRECTIONS : Find the Odd Word/ letters/ numbers
from the given alternatives
Q140. (a) Cone (b) Rectangle
(c) Circle (d) Triangle Q149. (a) 100 (b) 125
Ans: (a) Rectangles, circle and triangle are two - (c) 343 (d) 216
dimensional of figures while cone is a 3-d figure. Ans: (a) 125, 343, 216 are cubes while 100 is not.

Q141. (a) Knock (b) Wrong Q150. (a) House (b) Stable
(c) Psychology (d) Fast (c) Hut (d) Cottage
Ans: (d) In all other except (d) first letter is silent. Ans: (b) Stable is the place where animal (Horse) lives
where as all other are Home of people
DIRECTIONS: Find the odd parts pair from the
given alternatives. Q151. (a) BDGK (b) JLOS
(c) NPSW (d) MORU
Q142. (a) 3249 (b) 2709 Ans: (d) Only MORU does not follow the pattern of +2,
(c) 8314 (d) 2518 +3, +4 after every Alphabet
Ans: (c) In all number except 8314, the sum of first three
digits is equal to the unit's digit. DIRECTIONS: Find the odd words/letter/numbers
from the given alternatives:
Q143. (a) GLOVES (b) SWEATER
(c) SHAWL (d) UMBRELLA Q152. (a) 484 (b) 529
Ans: (d) Gloves, Sweater, shawl are worn in winter (c) 625 (d) 566
season while umbrella is used in rainy season. Ans: (d) Only 566 i s not a complete square from the
given options
Q144. (a) PORTRAIT (b) DRAW
(c) PAINT (d) SKETCH Q153. (a) A (b) U
Ans: (a) We used to draw, sketch or paint a portrait. (c) T (d) O
Ans: (c) Only T is consonant else other are vowels.
Q145. (a) HAND (b) NOSE
(c) MOUTH (d) EYES Q154. (a) January (b) April
Ans: (a) Nose, Eyes and mouth are in upper portion of (c) July (d) May
body while hands are in lower portion. Ans: (b) April contains 30 days else all contains 31 days.

Q146. (a) 6 : 22 (b) 8 : 25 DIRECTIONS: Find the odd word/ letters/ numbers
(c) 13 : 40 (d) 15 : 46 from the given alternatives:
Ans: (a) 8 × 3 + 1 = 25 So 8 : 25
13 ×3 + 1 = 40 So 13 : 40 Q155. (a) 28-21 (b) 31-38
15 × 3 + 1 = 46 So 15 : 46 (c) 45-52 (d) 64-71
But 6 × 3 + 1 = 19 ≠ 22 Ans: (a) 28 – 21 as 28 > 21 else 1st number is higher
So 6 : 22 is odd than other.

Q147. (a) 21 (b) 81 Q156. (a) Rupee (b) Pound


(c) 71 (d) 51 (c) Yen (d) Currency
Ans: (c) 21, 81 and 51 are composite number while 71 is Ans: (d) Rupee, Pound, Yen are currencies of country.
prime number.
Q157. (a) DAEH (b) KIMP
Q148. (a) HNOP (b) VUTS (c) HEIL (d) FCGJ
(c) RQPO (d) HGFE Ans: (b) (a) D – 3 = A, A + 4 = E, E + 3 = H
Ans: (a) RQPO, VUTS and HGFE are set of 4 (b) K – 2 = I, I + 4 = M, M + 3 = P
consecutive letters in reverse order but HNOP is not. (c) H – 3 = E, E + 4 = I, I + 3 = L
Page 214 of 516
https://telegram.me/aedahamlibra
(d) F – 3 = C, C + 4 = G, G + 3 = J DIRECTIONS: In the following questions, find the
odd parts pair from the given alternatives.
DIRECTIONS: Find the odd word/let ter/ number
form the given alternative. Q167. (a) Square (b) Triangle
(c) Sphere (d) Circle
Q158. (a) 43-6 (b) 28-4 Ans: (c) Except (c) all others are two dimensional shapes
(c) 50-7 (d) 36-5 while sphere is three-dimensional.
Ans: (b) 43 – 6 = 37, 28 – 4 = 24, 50 – 7 = 43, 36 – 5 =
31 Here except 24 all are prime no. Q168. (a) BCIK (b) KLRT
(c) VWCE (d) PRWY
Q159. (a) Rival (b) Opponent Ans: (d) Normal Pattern B → C → I → K
(c) Foe (d) Ally
Ans: (d) Ally is Antonym of Remaining three. Exception Pattern P → R → W → Y

Q160. (a) POCG (b) KLIZ Q169. (a) cfed (b) ilkj
(c) BUDX (d) FQMV (c) FGHI (d) SVUT
Ans: (d) Only FQMV is without any Vowel Ans: (c) Normal Pattern c → f → e → d
Exception Pattern F → G → H → I
DIRECTIONS: In the following questions, find the od
word /letters/number pair from the given alternatives.
Q170. (a) 286 (b) 166
(c) 495 (d) 583
Q161. (a) 36 - 48 (b) 56 - 44
Ans: (b) As, 2 8 6 = 2 + 6 = 8, 4 9 5 = 4 + 5 = 9 and 5 8 3
(c) 78 - 66 (d) 33 - 64
=5+3=8
Ans: (d) Difference between 36 – 48, 56 – 44 and
but, 1 6 6 = 1 + 6 = 7 ≠ 6
78 – 64 is 12 where as 33 – 64 is different.
Q171. (a) Wing Commander (b) Air Marshal
Q162. (a) Beak (b) Rudder
(c) Captain (d) Group Captain
(c) Wing (d) Fin
Ans: (c) Captain is odd one out.
Ans: (a) The wing, fin and rudder are all parts of an
airplane.
DIRECTIONS: In the following quest ions, select the
odd word from the given alternatives.
Q163. (a) LCUF (b) JHMX
(c) AKWE (d) PNTO
Q172. (a) 243 - 132 (b) 183 - 54
Ans: (b) The group of letters 'JHMX' has no vowel but
(c) 108 - 97 (d) 99 - 63
rest has vowels.
Ans: (d) 99-63 is odd one out.
Q164. (a) 99, 99 (b) 36, 63
Q173. (a) Lion (b) Leopard
(c) 28, 82 (d) 24, 44
(c) Snake (d) Tiger
Ans: (d) Except (d) in all other pairs, second term is
Ans: (c) Except, snake, all others are mammals.
reverse of first term.
Q174. (a) NPR (b) TVW
Q165. (a) FGBC (b) IJOP
(c) FHJ (d) KMO
(c) ACEG (d) LMHI
Ans: (b) Except TVW, there is a gap of one letter in a
Ans: (c) Normal Pattern F → G → B → C group.
Exception Pattern A → C → E → G
Q175. (a) 69 (b) 59
Q166. (a) 79, 63 (b) 167, 150 (c) 61 (d) 53
(c) 84, 67 (d) 112, 95 Ans: (a) Except 69, all others are prime numbers.
Ans: (a)

Page 215 of 516


https://telegram.me/aedahamlibra
DIRECTIONS: In the following question, select the
odd word from the given alternatives.
Ans: (b) 1919 1 + 9 + 1 + 9 = 20
5656 5 + 6 + 5 + 6 = 22
6761 6 + 7 + 6 + 1 = 20
Q176. (a) 11 – 120 (b) 17 – 290 7760 7 + 7 + 6 + 0 = 20
(c) 21 – 442 (d) 12 – 145 Except 5656, Sum of all digit of number is equal to 20.
Ans: (a) (11) 2 – 1 = 120
(17) 2 + 1 = 290 DIRECTIONS: Find the odd number/ letters/ word
(21) 2 + 1 = 442 from the given alternatives.
(12) 2 + 1 = 145
So, (11 – 120) is an odd number pair. Q185. (a) 2890 (b) 3375
(c) 1728 (d) 1331
Q177. (a) Chennai (b) Daman Ans: (a) Except 2890, All are cube of a number.
(c) Raipur (d) Shimla (15) 3 = 3375, (12) 3 = 1728, (11) 3 = 1331.
Ans: (b) Except Daman, all others are capital.
Q186. (a) ACDF (b) TUOP
Q178. (a) B (b) N (c) HIVW (d) FGKL
(c) P (d) W
Ans: (a) Normal Pattern T → U, O → P
Ans: (d) Except 'W' all others are placed at even number
position according to alphabet. Exception Pattern A → C → D → F

Q179. (a) 313 (b) 426 Q187. (a) Cricket (b) Chess
(c) 925 (d) 1034 (c) Football (d) Hockey
Ans: (a) Except 313, all others are composite number. Ans: (b) Except chess, all others are played at stadium.

Q180. In the following question, select the odd word DIRECTIONS: In the following question, select the
from the given alternatives. odd word from the given alternataives.
(a) Ludo (b) Chess
(c) Polo (d) Carrom Q188. (a) 23 (b) 17
Ans: (c) Except polo, All others are played by sitting. (c) 13 (d) 63
Ans: (d) Except 63, all others are prime number.
DIRECTIONS: Choose the odd word/letters
number/number pair from the given alternatives. Q189. (a) Newspaper (b) Journal
(c) Novel (d) Magazine
Q181. In the following question, select the odd Ans: (c) Only Novel is a type of book.
number from the given alternatives.
(a) 31 (b) 37 Q190. (a) Definite (b) Specific
(c) 43 (d) 49 (c) Doubtful (d) Distinct
Ans: (d) Except 49, All are prime numbers Ans: (c) Except Doubt ful, all others are similar word.

Q182. (a) Sirius (b) Proximacentauri Q191. BEHK : PSVY : : ADGJ : ?


(c) Deimos (d) Alpha centauri (a) KMOR (b) DHLP
Ans: (c) Except Deimos (It is a satellite) , all others are (c) PRTV (d) MPSV
star systems. Ans: (d) BEHK → PSVY
ADGJ → MPSV
Q183. (a) PON (b) SRQ
(c) XYZ (d) VUT
Q192. (a) 12 - 48 (b) 7 - 28
Ans: (c) Except, xyz, all others are opposite alphabetical
(c) 5 - 20 (d) 11 - 55
sequence.
Ans: (d) Here,
Q184. (a) 1919 (b) 5656
(c) 6761 (d) 7760
Page 216 of 516
But https://telegram.me/aedahamlibra
Q199. (a) Brown - Colour
(c) Fan - Air
(b) Rice - Grain
(d) Iron - Metal
So, (11 – 55) is odd one out.
Ans: (c) According to question,
Brown is a colour,
DIRECTIONS: In the following question, select the
Rice is a type of Grain
old number pair from the given alternatives.
Iron is a metal
But,fan is not a part of air but it is a electronic device.
Q193. (a) 12 - 28 (b) 20 - 36
(c) 42 - 58 (d) 72 - 90
Ans: (d) Here, Series
28 – 12 = 16
36 – 20 = 16
58 – 42 = 16
But, 90 – 72 = 18 DIRECTIONS: In each of the following questions, a
So, (90 – 72) is odd one out. series is given with one term missing. Choose the
correct alternative from the given ones that will
Q194. (a) 7 - 50 (b) 11 - 122 complete the series.
(c) 15 - 226 (d) 13 - 168
Ans: (d) According to question, Q1. Which one number is wrong in the given series ?
(7) 2 + 1 = 50 126, 98, 70, 41, 14
(11) 2 + 1 = 122 (a) 98 (b) 70
(15) 2 + 1 = 226 (c) 126 (d) 41
But (13) 2 + 1 ≠ 168 = 170 Ans: (d) Successive Change of -28

Q195. (a) 7 - 56 (b) 10 - 110 Q2. 3, 4, 7, 11, 18, 29, ?


(c) 9 - 90 (d) 8 - 96 (a) 31 (b) 39
Ans: (d) According to question, (c) 43 (d) 47
The pattern is : Ans: (d) 3 + 1 = 4 ;3 + 4 = 7 ;
n × (n – 1) 4 + 7 = 11 ;7 + 11 = 18
8 × 7 = 56 11 + 18 = 29 ;18 + 29 = 47
11 × 10 = 110
10 × 9 = 90 Q3. 975, 864, 753, 642, ?
But, 9 × 8 ≠ 96 = 72 (a) 431 (b) 314
(c) 531 (d) 532
Q196. JTIS : HRGQ :: FPEO : ? Ans: (c) Change of -111
(a) DNCM (b) DCNQ
(c) CNDM (d) CNDQ ) Q4. 8, 24, 12, ? 18, 54
– – – – (a) 28 (b) 36
Ans: (a) JTIS → HRGQ (c) 46 (d) 38
– – – –
FPEO → DNCM Ans: (b) First Multiply by 2 than divide by 2 and so on

Q197. BAD : CBE :: ? : IVSU DIRECTIONS: In each of the following questions,


(a) GOOD (b) HSPR find the missing number/ letters/ figure from the given
(c) HALT (d) HURT ) responses :

Ans: (d) BAD → CBE


Q5. Which set of letters when sequentially placed at
IVSU → HURT the gaps in the given letter series shall complete it ?
__a__aaaba__ __ ba__ ab __ (a) abaaaa
Q198. (a) Sward (b) Spear (b) abaaba
(c) Gun (d) Dagger (c) aababa (d) ababaa
Ans: (c) Only Gun has bullet. Ans: (a) a a b a/ aaba/ a a ba / a ab a

Page 217 of 516


Q6. CGK, FJN, IMQ, ?
https://telegram.me/aedahamlibra
(c) efgh (d) wxyz
Ans: (b) Successive change of -2 & +2
(a) SPL (b) NIH
(c) LPT (d) QLP Q13. 4, 196, 16, 169, ?, 144, 64
Ans: (c) C → G → K (a) 21 (b) 81
(c) 36 (d) 32
F→ J→ N Ans: (c) The pattern is as follows :
I→ M→ Q 4 = (2) 2; 16 = (4) 2; 36 = (6) 2;
64 = (8) 2; 196 = (14) 2; 169 = (13) 2;
Q7. 0, 4, 18, 48, ? 180 144 = (12) 2;
(a) 58 (b) 68
(c) 84 (d) 100 Q14. 8, 15, 36, 99, 288, ?
Ans: (d) Check Successive increase on 3 levels (a) 368 (b) 676
(c) 855 (d) 908
Q8. 7, 9, 13, 21, 37, ? Ans: (c) Get Successive change. Resulted Changes are
(a) 58 (b) 63 successive multiples
(c) 69 (d) 72
Ans: (c) Successive increase of 2 4 8 16 32… Q15. Find the wrong number in the series.
6, 9, 15, 22, 51, 99
Q9. Select the number which does NOT belong to the (a) 99 (b) 51
given series : (c) 22 (d) 15
232, 343, 454, 564, 676 Ans: (c) 6 + 3 = 9, 9 + 6 = 15 15 + 12 = 27, 27 + 24 = 51,
(a) 676 (b) 454 51 + 48 = 99
(c) 343 (d) 564
Ans: (d) 232 + 111 = 343 DIRECTIONS: a series is given, with one/two term
343 + 111 = 454 missing. Choose the correct alternative from the given
454 + 111 = 565 ones that will complete the series.
565 + 111 = 676
The number 564 does not belong to the series. Q16. Which one set of letters when sequentially placed
at the gaps in the given letter series shall complete it?
DIRECTIONS: In each of the following questions, a __ab __b __aba __ __ abab
series is given, with one/two terms (s) missing. Choose (a) a bb aa (b) bb aa b
the correct alternative from the given ones that will (c) ab aa b (d) a aa ba
complete the series. Ans: (d) a a b a b / a a b a b/a ab ab

Q10. Which one -set of letters when sequentially Q17. 14, 19, 29, 49, 89, ?
placed at the gaps in the given letter series shall (a) 139 (b) 149
complete it? (c) 159 (d) 169
__cb__cab__baca__cba__ab Ans: (d) Successive addition of 5 10 20 40 80…
(a) cabcb (b) abccb
(c) bacbc (d) bcaba Q18. BMRG, DLTF, FKVE, HJXD, ? .
(a) JIZC (b) JZIB
Ans: (c) (c) GIFB (d) MOLC
Ans: (a) Change of +2 -1 +2 -1 successively in each
Q11. XYZCBAUVWFE ? ? letter.
(a) DR (b) RS
(c) DS (d) MN Q19. 121, 144, 289, 324, 529, 576, ? .
Ans: (a) XYZUVW R ST CBAFE D (a) 961 (b) 841
(c) 900 (d) 729
Q12. reoc, pgme, nikg, lkii ? Ans: (b) Squaring of 11 12 13 ….
(a) acef (b) jmgk

Page 218 of 516


https://telegram.me/aedahamlibra
DIRECTIONS: In questions, a series is given, with
one term missing. Choose the correct alternative from
33 + 3 = 35
36 + 1 = 37
the given ones that will complete the series. 37 – 1 = 36
Therefore, the number 35 is wrong in the series.
Q20. 5, 21, 69, 213, 645, ? .
(a) 1670 (b) 1941 Q26. 5, 13, 29, 61, 120, 253
(c) 720 (d) 1320 (a) 120 (b) 253
Ans: (b) The pattern is as follows : (c) 61 (d) 29
21 – 5 = 16 16 × 3 = 48 Ans: (a) 5 + 8 = 13
69 – 21 = 48 48 × 3 = 144 13 + 16 = 29
213 – 69 = 144 144 × 3 432 29 + 32 = 61
645 – 213 432 432 × 3 = 1296 61 + 64 = 120
and 125 + 128 = 253
1941 – 645 = 1296 Therefore, the number 120 is wrong in the series.

Q21. DA, HE, LI, ? , TQ DIRECTIONS: In questions, a Series is given, with


(a) PJ (b) PT one term missing. Choose the correct alternative from
(c) PM (d) PK the given ones that will complete the series.
Ans: (c) Change of +4 successively
Q27. 0, 7, 28, 63, 124, 215
Q22. FNC, HQG, JTK, ? . (a) 28 (b) 215
(a) LXO (b) LMO (c) 7 (d) 63
(c) KMT (d) LWO Ans: (a) The pattern is:
Ans: (d) Change of +2 +3 +4 successively in each letter. (23– 1) , (33–1) , (43– 1) , (53–1) .
Therefore, the number 28 is wrong in the series.
DIRECTIONS: In questions, identify the wrong
number in the series. Q28. KDW, MGT, OJQ, ?
(a) MNQ (b) QNM
Q23. Which one set of letters when sequentially placed (c) NMQ (d) QMN
at the gaps in the given letter series shall complete it? ()
_c_bd_cbcda_a_db_a Ans: (d) Change of +2 +3 -3
(a) daabbc (b) bdbcba successively
(c) adabcd (d) cdbbca
Q29. FNC, HQG, JTK, ? .
Ans: (c) (a) LXO (b) LMO
(c) KMT (d) LWO
Q24. 9, 19, 40, 83, 170, 340 ()
(a) 83 (b) 40 Ans: (d) Change of +2 +3 +4
(c) 340 (d) 170 successively in each letter.
Ans: (c) 9 × 2 + 1 = 18 + 1 = 19
19 × 2 + 2 = 38 + 2 = 40
Q30. a_ba_c_aad_aa_ea
40 × 2 + 3 = 80 + 3 = 83 (a) babbb (b) babbd
83 × 2 + 4 = 166 + 4 = 170 (c) babbc (d) bacde
170 × 2 + 5 = 340 + 5 = 340
Therefore, the number 340 is wrong in the series. Ans: (d)

Q25. 21, 28, 33, 35, 37, 36 DIRECTIONS: In each of the following
(a) 21 (b) 36 questions,which one set of letters when sequentially
(c) 33 (d) 35 placed at the gaps in the given letter series shall
Ans: (d) 21 + 7 = 28 complete it?
28 + 5 = 33

Page 219 of 516


https://telegram.me/aedahamlibra
Q31. Find the wrong number in the series from the
given alternatives.
(c) a b c b c (d) b c a b c
Ans: (a) a b c b c a ca b / a b c bc a c ab.
17, 36, 53, 68, 83, 92
(a) 92 (b) 53 Q37. BCFG, JKNO, RSVW, ?
(c) 68 (d) 83 (a) ZADE (b) HIKL
Ans: (d) As, 17 + 19 = 36 (c) STUX (d) MNPQ
36 + 17 = 53 Ans: (a) B C F G  2, 3, 6, 7
53 + 15 = 68 J K N O  10, 11, 14, 15
68 + 13 = 8183 R S V W  18, 19, 22, 23
81 + 11 = 92 Next sequence = 26, 27, 30, 31 = 26, 1, 4, 5 = Z A D E
(subtract the excess value by 26)
Q32. c_ab_ca_bc_a
(a) b c a b (b) a b c b Q38. CIM, HNR, MSW, ?
(c) b a c b (d) c b a c (a) SXA (b) UYB
Ans: (c) (c) RXB (d) ZEH
Ans: (c) Increase of 5 → → and so on.
DIRECTIONS: In each of the following q uestions,
select the missing number from the given responses. Q39. 2, 3, 6, 7, 14, 15, ?
(a) 16 (b) 30
Q33. _ba _ bab_babb_b (c) 31 (d) 32
(a) b a a a (b) a b b b Ans: (b) Successive addition of 1 and multiply by 2
(c) b a b b
(c) a b a b Q40. 3120, ? , 122, 23, 4
(a) 488 (b) 621
Ans: (b)
(c) 610 (d) 732
Ans: (b) 4th term 23 = 5 × 4 + 3 = 23
Q34. 500, 484, 459, ? , 374
3rd term 122 = 5 × 23 + 7 = 122
(a) 384 (b) 432
2nd term ? = 5 × 122 + 11 = 621
(c) 418 (d) 423
1st term 3120 = 5 × 621 + 15 = 3120
Ans: (d) 500 – (4) 2 = 484
484 – (5) 2 = 459
DIRECTIONS: In questions, a series is given, with
459 – (6) 2 = 423
one term missing. Choose the correct alternative from
423 – (7) 2 = 374
the given ones that will complete the series.

Q35. 60, 69, 85, ? , 146


Q41. 0, 5, 60, 615,?
(a) 110 (b) 117
(a) 6030 (b) 6170
(c) 109 (d) 120
(c) 6130 (d) 6000
Ans: (a) 60 + (3) 2 = 69
Ans: (b)
69 + (4) 2 = 85
85 + (5) 2 = 110 → → → →
110 + (6) 2 = 146
Q42. AB, DC, EF, HG, IJ, LK. ?
DIRECTIONS: In the following questions a series is (a) LM (b) NM
given, with one term missing. Choos e the correct (c) MN (d) ML
alternative from the given ones that will complete the Ans: (c) A → B D → C E → F H → G I → J L → K
series.
Q43. CGK, FJN, IMQ, ?
Q36. Which one set of letters when sequentially placed (a) SPL (b) NIH
at the gaps in the given letter series shall complete it? (c) LPT (d) QLP
a _ cbc _ ca _ ab _ bca _ ab
Ans: (c) C → G → K F → J → N I → M → Q
(a) b a b c c (b) b c a b b
Page 220 of 516
Q44. PBA, QDC, RFE, ?
https://telegram.me/aedahamlibra
Q50. Which number is wrong in the series?
5, 11, 23, 47, 96
(a) SHG (b) OAB (a) 47 (b) 23
(c) TJI (d) ULK (c) 96 (d) 11
Ans: (a) P → B→ AQ→ D→ CR→ F→ E Ans: (c) Multiply by 2 and add 1

Q45. Find the wrong term in the following series : Q51. BFH, ZDF, JNP, T, ?
49, 49, 50, 54, 60, 79, 104 (a) WX (b) XY
(a) 60 (b) 49 (c) ZA (d) XZ
(c) 104 (d) 54 Ans: (d) B → F → H
Ans: (a) The pattern is: Z→ D→ F
49 + (0) 2 = 49, 49 + (1) 2 = 50, 50 + (2) 2 = 54
54 + (3) 2 = 63, 63 + (4) 2 = 79, 79 + (5) 2 = 104 J→ N→ P

DIRECTIONS: A series is gi ven, with one Q52. 13, 10, ___?___, 100, 1003, 1000, 10003.
number/letter missing. Choose the correct alternative (a) 130 (b) 1030
from the given ones that will complete the series. (c) 1130 (d) 103
Ans: (d) 13 → 10
Q46. Which one of the following numbers lacks the 3 has been replaced with zero in the next term.
common property in the series? 103 → 100
81, 36, 25, 9, 5, 16 1003 → 1000
(a) 5 (b) 9 10003.....
(c) 36 (d) 25
Ans: (a) Except 5, all numbers are perfect square Q53. 5, 13, 29, 61, 125, ___?___.
numbers. (a) 145 (b) 253
(c) 196 (d) 245
Q47. 3, 11, 38, 102, ______ , 443 Ans: (b) The patterns is as follows :
(a) 227 (b) 237 5 × 2 + 3 = 13
(c) 247 (d) 217 13 × 2 + 3 = 29
Ans: (a) Add successively cubes of 2, 3,4,5 and 6 29 × 2 + 3 = 61
61 × 2 + 3 = 125
Q48. BD, FH, JL, NP, ? 125 × 2 + 3 = 253
(a) PQ (b) RS
(c) SU (d) RT Q54. DGJ, FIL, HKN, JMP, ?
Ans: (d) Successive addition of 2 (a) LON (b) LOR
(c) LOQ (d) LOM
Q49. Which one of the letters when sequentially Ans: (b) D → G → J F → I → L so on
placed at the gaps in the given letter series shall
complete it? Q55. YXW, UTS, QPO, ML __, __HG
m_ _ l m _ l _ m m _ l (a) LM (b) JI
(a) mllml (b) mlmll (c) KI (d) IH
(c) llmlm (d) mmlml
Ans: (c) Y → X → W
Ans: (b) m m l l
mmll U→ T→ S
mmll Q→ P→ O

DIRECTIONS: In question, a series is gi ven, with one Q56. CEG, IKM, OQS, ?
term missing. Choose the correct alternative from the (a) TVX (b) UWY
given ones that will complete the series. (c) UWZ (d) TVW
Ans: (b) C → E → G
Page 221 of 516
I→ K→ M https://telegram.me/aedahamlibra
(a) SPL
(c) LPT
(b) NIH
(d) QLP
O→ Q→ S
Ans: (c) C → G → K
Q57. A, B, D, G, ? F→ J→ N
(a) I (b) J I→ M→ Q
(c) K (d) L
Ans: (c) A → B → D → G Q64. 7, 2 = 59; 5, 3 = 28; 9, 1 = 810; 2, 1 = 13; 5, 4 = ?
(a) 19 (b) 9
Q58. BDF, JLN, PRT, ? (c) 20 (d) 239
(a) WXY (b) VXY Ans: (a) 7 – 2 = 5; 7 + 2 = 9
(c) UWY (d) VWX 7, 2 = 59
Ans: (c) B → D → F 5 – 3 = 2; 5 + 3 = 8
5, 3 = 28
J→ L→ N 9 – 1 = 8; 9 + 1 = 10
P→ R→ T 9, 1 = 810
2 – 1 = 1; 2 + 1 = 3
Q59. 19 18 34 32 44 41 2, 1 = 13
24? Therefore, 5 – 4 = 1; 5 + 4 = 9
(a) 3 (b) 6 5, 4 = 19
(c) 9 (d) 4
Ans: (b) 19 – 18 = 1; 1 × 2 = 2 DIRECTIONS: In que stion, which one set of letters
34 – 32 = 2; 2 × 2 = 4 when sequentially placed at the gaps in the given
44 – 41 = 3; 3 × 2 = 6 letter series shall complete it?

Q60. 5, 11, 24, 51, 106, ___?___. Q65. 120, 440, 960, 1680, ?
(a) 115 (b) 122 (a) 2600 (b) 3240
(c) 217 (d) 221 (c) 3040 (d) 2400 Hint: Get
Ans: (c) 5 × 2 + 1 = 11 Successive Differences
11 × 2 + 2 = 24
24 × 2 + 3 = 51 Q66. b_ab_b_aab_b
51 × 2 + 4 = 106 (a) abba (b) baaa
106 × 2 + 5 = 217 (c) aabb (d) abbb

Ans: (d)
DIRECTIONS: In each of the following questions, a
series is given with one term missing. Choose the
Q67. 5, 16, 51, 158, ?
correct alternative from the given ones that will
(a) 483 (b) 481
complete the series.
(c) 1454 (d) 1452
Ans: (b) 5 × 3 + 1 = 16
Q61. 3917, 3526, ___?___, 2857
16 × 3 + 3 = 51
(a) 3082 (b) 3174
51 × 3 + 5 = 158
(c) 3389 (d) 2682
158 × 3 + 7 = 481
Ans: (b)

Q68. 9 * 2 : 9 * 9 :: 9 × 5 : ?
Q62. AB, DC, EF, HG, IJ, LK. ?
(a) 9 × 6 (b) 9 × 7
(a) LM (b) NM
(c) 9 × 8 (d) 9 × 4
(c) MN (d) ML
Ans: (a)
Ans: (c) A → B D → C E → F H → G I → J L → K
Q69. AFI, JOR, MRU, ?
Q63. CGK, FJN, IMQ, ? (a) HMP (b) PMO
Page 222 of 516
(c) RJL
Ans: (a)
https://telegram.me/aedahamlibra
(d) GJN DIRECTIONS: In each of the following questions,
which one set of letters when sequentially placed at
the gaps in the given letter series shall complete it ?
Q70. MRS, LTU, KVW, ?
(a) TQR (b) MOP Q77. Find the number which does not fit in the
(c) JXY (d) CDE following series:
Ans: (c) 49, 81, 123, 156, 221
(a) 81 (b) 123
DIRECTIONS: Select the missing number from the (c) 156 (d) 49
given responses. Ans: (c)

Q71. B, G, K, N, ? Q78. a _ _ b abba _ abb a _ ba


(a) P (b) O (a) abab (b) abba
(c) H (d) L (c) aabb (d) aaab
Ans: (a) Successive reduction of 5, 4, 3, 2, 1 Ans: (a) a a b b a b b a / a a bb a b ba

Q72. 1944, 108, ?, 6, 3 Q79. AZB–, AZ – Y, A – BY, – ZBY


(a) 16 (b) 18 (a) YBZA (b) BYAZ
(c) 11 (d) 12 (c) BZYA (d) AZBY
Ans: (b) 6 × 3 = 18 Ans: (a) A Z B Y / A Z B Y / A Z BY / A ZBY
18 × 6 = 108
108 × 18 = 1944 Q80. Find out the wrong number in the sequence
Hence, 18 is the missing number in the sequence. 102, 101. 98, 93, 86, 74, 66, 53
(a) 101 (b) 66
Q73. 251 (12) 107 (c) 74 (d) 93
381 (?) 125 Ans: (c) 102 – 1 = 101; 101 – 3 = 98; 98 – 5 = 93;93 – 7
(a) 14 (b) 24 = 86; 86 – 9 = 77; 77 – 11 = 66; 66 – 13 = 53
(c) 11 (d) 16 Therefore, the number 74 is wrong in the sequence.
Ans: (d) 251 – 107 = 144 = (12) 2
∴381 – 125 = 256 = (16) 2 Q81. XYZ, ABC, UVW, DEF, RST,
Hence, 16 is the missing number in the given question. GHI, ? .
(a) UVW (b) JKL
Q74. 354 (110) 526 (c) OPQ (d) NOP
297 (?) 631 Ans: (c) X → Y → Z
(a) 128 (b) 116
(c) 135 (d) 143 A → B → C so on
Ans: (b) 354 + 526 = 880/8 = 110 .
∴297 +631 = 928/8 = 116
Q82. 4, 18, 48, ? , 180
Q75. 0, 7, 26, 63, ? (a) 80 (b) 100
(a) 125 (b) 126 (c) 105 (d) 125
(c) 217 (d) 124 Ans: (b) Check Successive Differences on 3 levels
Ans: (d) Succsive (cubes-1)
Q83. FGHJKLNOPRS ?
Q76. Which one set of letters when sequentially placed (a) T (b) M
at the gaps in the given letter series shall complete it? (c) Q (d) U
--baabaa-aa- Ans: (a)
(a) b b a a (b) b a a b
(c) a a b b (d) a b a b DIRECTIONS: In each of the following questions,
Ans: (c) a a b/ a a b/ aa b/ aa b select the missing number from the given responses:

Page 223 of 516


https://telegram.me/aedahamlibra
Q84. b – 0,y – 3, c – 8, x – 15, d – 24, ? ,
(a) e – 48 (b) w – 35
(c) 64 (d) 72
Ans: (d) Check Successive Differences on 2 levels
(c) w – 39 (d) v – 30
Ans: (b) Check Successive Differences on 2 levels Q92. 1, 3, 7, 15, 31, ?
(a) 73 (b) 63
Q85. 15 12 44 28 64 53 (c) 37 (d) 36
38? Ans: (b) 1 + 1 × 2 = 3
(a) 30 (b) 13 3+2×2=7
(c) 70 (d) 118 7 + 4 × 2 = 15
Ans: (b) 15 + 29 = 44 15 + 8 × 2 = 31
44 + (29 – 9) = 64 31 + 16 × 2 = 63
12 + 16 = 28
28 + (16 + 9) = 53 DIRECTIONS : In questions, Series is given, with one
3 + 5 = 8 ; 8 + 5 = 13 number/letter missing. Choose the correct alternative
from the given ones that will complete the series.
Q86. ASY, BRX, CQW, __?__
(a) DVP (b) DPV Q93. Which one set of letters when sequentially placed
(c) PDV (d) PQD at the gaps in the given letter series shall complete it?
Ans: (b) Change of +1 -1 -1 successively in each letter. aa_aabb_b_aa_aabb_bb
(a) abbab (b) bbbaa
Q87. 198, 202, 211, 227, ? (c) babba (d) aabbb
(a) 236 (b) 252 Ans: (a) a a a a a |b b b b b |a a a a a |b b b b b .
(c) 275 (d) 245
Ans: (b) 198 + (2) 2 = 202 Q94. 112, 124, 156, 208, _?
202 + (3) 2 = 211 (a) 316 (b) 280
211 + (4) 2 = 227 (c) 292 (d) 304
227 + (5) 2 = 252 Ans: (b) Check Successive Differences on 2 levels

Q88. BDF, KMO, TVX, ? Q95. Y V S P M ? .


(a) CEG (b) UVW (a) J (b) K
(c) XYZ (d) ACE (c) L (d) I
Ans: (a) B → D → F Ans: (a) Y → V → S → P M .
K→ M→ O
Q96. AEI, BFJ, CGK, ?
T→ V→ X (a) DHL (c) DLH
(b) EIM (d) LPT
Q89. ZXV, TRP, NLJ ?
Ans: (a) A → E → I
(a) IGF (b) HDF
(c) HGE (d) HFD B→ F→ J
Ans: (d) Z → X → V C→ G→ K
T→ R→ P
Q97. 3, 6, 9, 15, 24, 39, 63, ?
N→ L→ J (a) 100 (b) 87
(c) 102 (d) 99
Q90. 15, 23, 31, 39, ? , 55, 63, Ans: (c) 3 + 3 = 6
(a) 45 (b) 47 6+3=9
(c) 46 (d) 44 9 + 6 = 15
Ans: (b) Check Successive Differences 15 + 9 = 24
24 + 15 = 39
Q91. 2, 8, 18, 32, 50, ? 39 + 24 = 63
(a) 70 (b) 68
Page 224 of 516
63 + 39 = 102 https://telegram.me/aedahamlibra
4+5=9
7+2=9
DIRECTIONS: In question, a series is given, with one
term missing. Choose the correct alternative from the DIRECTIONS: A series is given, with one term
given ones that will complete the series. missing. Choose the correct alternative from the given
ones that will complete the series.
Q98. –1, 0, ?, 8, 15, 24
(a) 4 (b) 3 Q105. Which one set of letters when sequenti ally
(c) 2 (d) 1 placed at the gaps in the given letter series shall
Ans: (b) Check Successive Differences on 2 levels complete it ?
ac__c_cb_acbcacbca_bc
Q99. 7, 14, 23, 34, ? (a) abbb (b) bacc
(a) 46 (b) 47 (c) babc (d) bbcc
(c) 44 (d) 45 Ans: (b) acbc/acbc/acbc/acbc/acbc
Ans: (b) Check Successive Differences on 2 levels
Q106. AZ, CX, FU, ?
DIRECTIONS: In Question, a series is given, with one (a) IR (b) IV
term missing. Choose th e correct alternative from the (c) JQ (d) KP
given ones that will complete the series. Ans: (c)

Q100. AE, FJ, KO, ? UY Q107. 1, 2, 6, 24, ? ,720


(a) QN (b) TQ (a) 3 (b) 5
(c) NP (d) PT (c) 120 (d) 8
Ans: (d) Change of +5 Ans: (c)

Q101. 68, 81, 96, ? 132


(a) 105 (b) 110
(c) 113 (d) 130
Ans: (c) Check Successive Differences on 2 levels

Q102. Y, S, U, O, Q, ? ? DIRECTIONS: Choose the correct alternatives from


(a) N O (b) L M the given ones which will complete the series.
(c) J K (d) H I
Ans: (b) Y → S → U → O Q Q108. 156, 506, ?, 1806
(a) 1056 (b) 856
Q103. EAC, GCE, IEG, ? (c) 1456 (d) 1506
(a) JHI (b) KGI Ans: (a) 156, 506, ?, 1806
(c) JGI (d) KIJ 156+350 = 506
506+550 = 1056
Ans: (b) E → A → C 1056+750 = 1806
G→ C→ E
I→ E→ G Q109. 8, 18, 32, 50, 72, ?
(a) 76 (b) 98
(c) 80 (d) 70
Q104. Find the wrong number in the series :
Ans: (b) Check Successive Differences on 2 levels
30, 27, 36, 45, 72
(a) 30 (b) 27
(c) 36 (d) 72 Q110. B D Z X F H V T J ? ? ?
(a) L R P (b) L P R
Ans: (a) 3 + 0 = 3
(c) L R Q (d) K R P
2+7=9
Ans: (a) Change of +2 and -2
3+6=9

Page 225 of 516


https://telegram.me/aedahamlibra
Q111. In the following question, a series
is given with one or more alphabet
missing. Choose the correct
alternative from the given options. Q116. Y, T, P, ? , K
ACF, BEI, CGL, DIO, ? (a) L (b) O
(a) EGJ (b) EKQ (c) N (d) M
(c) EHL (d) EKR Ans: (d) Change of -1 successively

Ans: (d) A → C → F
Q117. 4, 11, 17,22, ? , 29, 31, 32
B→ E→ I (a) 26 (b) 27
C→ G→ L (c) 23 (d) 24
Ans: (a) Check Successive Differences on 2 levels
D→ I→ O
Q118. √ √ √ √
DIRECTIONS: In questions below, which one set of
?__
letters when sequentialy placed at the gaps in the
(a) √ √
given letter series shall complete it?
(c) √ √
Q112. 1331, 729, 343, 125, ? Ans: (d)
(a) 27 (b) 64 √ √
(c) 216 (d) 512 √ √
Ans: (a) The sequence is : √ √
(11) 3, (9) 3, (7) 3, (5) 3 √ √ √

Q113. LU_TUPLUBTU_LUBT_P_UBTUP
(a) LBPU (b) BPUL
Q119. a d h m s ?
(c) PBUL (d) BUPL
(a) T (b) W
Ans: (b) Words LUB and TUP are in consecutive order.
(c) X (d) Z
LUBTUPLUBTUPLUBTUPLUBTUP
Ans: (d)
Q114. 24, 35, 20, 31, 16, 27, __, __
Q120. –1, 0, 3, 8, 15 , ?
(a) 9, 9 (b) 5, 30
(a) 23 (b) 26
(c) 8, 25 (d) 12, 23
(c) 24 (d) 25
Ans: (d) There are two numbers series: with change of -4
Ans: (c)
DIRECTION: In questions, a series is given, with one
term missing. Choose the correct alternativ e from the
Q121. ccbab _ caa _ bccc _ a _
given ones that will complete the series.
(a) babb (b) bbba
(c) baab (d) babc
Q115. Ans: (a) c c b a/b b c a/a a b c/c c b a/b
(a) 35 (b)
DIRECTIONS: In the following two Questions, a
(c)
series is given, with one term missing. Choose the
Ans: (b) correct alternative from the given ones that will
; complete the series.

Q122. a_ _ dba_ _bcad__ _da__ _cd


(a) bccdbcab (b) abcddcba
(c) cbcddcba (d) aabbccdd
Ans: (a) a b c d/b a c d/b c a d/b c d a/a b c d.

Q123. 4, 6, 10, 16, 24, ?


Page 226 of 516
(a) 28
(c) 34
https://telegram.me/aedahamlibra
(b) 30
(d) 40
(a) 48
(c) 29
(b) 36
(d) 21
Ans: (c) Successive increase of 2, 4, 6, 8, 10 Ans: (b) First Multiply by 3 than divide by 2

Q124. SH _ ELAS _ EELA _ HEELA SHEE _ A Q132. bb_aab_caab_ca_


(a) HHSS (b) EEHS (a) cbba (b) acab
(c) ELHA (d) EHSL (c) abbc (d) bcab
Ans: (d) S H E E L A / S H E E L A / S H E E L A / S H Ans: (a) b b c a a |
EELA bbcaa|
bbcaa.
Q125. 12_ 41 _ 34123 _ _ 234
(a) 3212 (b) 2134 DIRECTIONS: A series is given with one (or more)
(c) 3241 (d) 1432 term missing. Choose the correct alternative from the
Ans: (c) 1 2 3 4 / 1 2 3 4 / 1 2 3 4 / 1 2 3 4 given ones that will complete the series :

Q126. _ _ aba _ _ ba _ ab Q133. _cdb_ddb_db_ _cd


(a) abbbb (b) baabb (a) bbcbb (b) ccbcc
(c) bbaba (d) abbab (c) bbbcc (d) cccbc
Ans: (d) a b a b a b a b a b a b Ans: (a) b c d b b d d |
: A series is given,with one term missing. Choose bcdbb|
amongst the given responses choose the meaningful bcd.
one.
Q134. 720, 180, 176, 44, 40, 10, ? /
Q127. Find the missing number ? (a) 6, 4 (b) 8, 6
2, 5, 10, 17, 26, ? (c) 6, 1.5 (d) 4, 2
(a) 36 (b) 49 Ans: (c) 720 / 4 = 180
(c) 37 (d) 47 180 – 4 = 176
Ans: (c) Successive increase of 3 5 7 11 so on 176 / 4 = 44
44 – 4 = 40
Q128. CUS, DVT, EWU, ____ 40 / 4 = 10
(a) FXV (b) VXF 10 – 4 = 6, 6/4 = 1.5
(c) XFV (d) XVF The answer is 6, 1.5.
Ans: (a) Change of +1
Q135. 5, 10, 20, 40, 80, ?___
Q129. 206, 221, 251, 296, ?, 431 (a) 150 (b) 160
(a) 326 (b) 356 (c) 120 (d) 140
(c) 311 (d) 341 Ans: (b)
Ans: (b) Check Successive Differences on 2 levels
Q136. C4X, F9U, I16R, ?___
Q130. A series is given, with one term missing. Choose (a) L27P (b) K25P
the correct a lternative from the given ones that will (c) L25P (d) L25O
complete the series: Ans: (c)
ALZ,CJV,EHR,GFN,?
(a) JDJ (b) IEK Q137. 15, 21, 57, ?___,221
(c) IDJ (d) JEK (a) 121 (b) 126
Ans: (c) Change of +2 -2 -4 (c) 96 (d) 108
Ans: (d)
Q131. A series is given, with one term missing. Choose
the correct alternative from the given ones that will DIRECTIONS: A series is given, with one term
complete the series: missing. Choose the correct alternative from the given
8, 24, 12, ?, 18, 54 options that will complete the series.
Page 227 of 516
https://telegram.me/aedahamlibra
Q138. Which one set of letters when sequentially
11 × 2 + 1 = 23
23 × 2 + 1 = 47
placed at the gaps in the letter series shall complete it? So, The missing number is = 47.
_qpx_rq_xxr_pxx_qp_x
(a) rxpqrx (b) pxrqxr Q145. 8, 11, 14, 17, 20, ?
(c) xrprqx (d) rspprx (a) 21 (b) 22
Ans: (a) r q p x x / r q p x x / r q p x x / r q p x x. (c) 23 (d) 27
Ans: (c)
Q139. 3, 17, 45, 87, ?
(a) 143 (b) 153
(c) 183 (d) 123
Alphabet Test
Ans: (a) Check Successive Differences

Q140. Double, Triple, Quadruple, ? DIRECTIONS: Which one of the given responses
(a) Quintuple (b) Nonuple would be a meaningful descending order of the
(c) Sextuple (d) Octuple following?
Ans: (a) The next term is quintuple because quintuple
means five times. Q1. A group of alphabets are given with each being
assigned a numerical code. These have to be
Q141. AC, EG, ?, MO unscrambled into a meaningful word and the correct
(a) IK (b) IJ code so obtained may be indicated from the given
(c) IL (d) IM responses ?
Ans: (a) The common difference between first and RAHKS
second letter is one letter according to alphabetical 12345
sequence. So, answer will be IK. (a) 5 1 2 3 4 (b) 5 4 2 1 3
(c) 5 3 2 1 4 (d) 5 3 1 2 4
DIRECTIONS: A series is given with one term Ans: (c)
missing. Select the correct alternativ e from the given
ones that will complete the series. Q2. 1.Major
2.Captain
Q142. BKS, DJT, FLU, HHV, ? 3.Colonel
(a) IGU (b) IGX 4.Brigadier
(c) JGW (d) IJX 5.Lt. General
Ans: (c) Change of +2 -1 +1 (a) 5, 4, 3, 1, 2 (b) 5, 1, 4, 2, 3
(c) 4, 5, 1, 3, 2 (d) 3, 4, 2, 5, 1
Q143. 3, 10, 31, 94, ? Ans: (a) Descending Order
(a) 197 (b) 127 5.Lt. General  4.Brigadier  3.Colonel  1.Major 
(c) 283 (d) 317 2.Captain
Ans: (c) The pattern is as follows :
3 × 3 + 1 = 10
Q3. 1.Dissident
10 × 3 + 1 = 31
2.Dissolve
31 × 3 + 1 = 94
3.Dissent
94 × 3 + 1 = 283
4.Dissolute
So, the missing number is 283.
5.Dissolution
(a) 3, 1, 4, 5, 2 (b) 3, 2, 1, 4, 5
Q144. 2, 5, 11, 23, ? (c) 3, 1, 4, 2, 5 (d) 3, 2, 4, 5, 1
(a) 47 (b) 44
Ans: (a) Dissent  1.Dissident  4.Dissolute 
(c) 44 (d) 43
5.Dissolution  2.Dissolve
Ans: (a) The pattern is :
2×2+1=5
5 × 2 + 1 = 11
Page 228 of 516
https://telegram.me/aedahamlibra
Q4. From the given alternative words, select the word
which cannot be formed using the letters of the given
Ans: (d) There is no ‗A‘ letter in the given word.

word: DIRECTIONS: Which one of the given responses


'CONCENTRATION' would be a meaningful order of the following?
(a) CONCERN (b) NATION
(c) TRAIN (d) CENTRE Q9. From the given alternative words, select the word
Ans: (d) There is only one ‗E‘ in the given word. which can be formed using the letters of the given
word:
Q5. From the given alternative words, select the word STRANGULATION
which can be formed using the letters of the given (a) TRIANGLE (b) GARLAND
word: (c) ROASTING (d) TRAUMA
'DETERMINATION' Ans: (c) STRANGULATION
(a) DECLARATION (b) NATIONAL
(c) TERMINATED (d) DEVIATION Q10. 1.curd
Ans: (c) There is no ‗C‘ letter in the given word. (a) 2.grass
option There is no ‗L‘ letter in the given word. 3.butter
(b) option There is no ‗V‘ letter in the given word. 4.milk
(d) option So, Terminated can be formed. 5.cow
(a) 5, 2, 3, 4, 1 (b) 4, 2, 5, 3, 1
Q6. Which one of the given responses would be a (c) 2, 5, 4, 3, 1 (d) 5, 2, 4, 1, 3
meaningful order of the following? Ans: (d) The correct order is :
1.Sentence Cow → Grass → Milk → Curd → Butter
2.Word
3.Chapter Q11. TEACHER
4.Phrase (a) REACH (b) EATER
5.Paragraph (c) EARTH (d) TRACTOR
(a) 4, 3, 1, 2, 5 (b) 2, 3, 5, 4, 1 Ans: (d) TRACTOR is the word which cannot formed
(c) 3, 5, 1, 4, 2 (d) 1, 3, 2, 4, 5 from the given word as T does not comes twice in the
Ans: (c) Meaningful order of the words : original word.
3.Chapter  5.Paragraph  1.Sentence  4.Phrase 
2.Word Q12. REASONABLE
(a) NOBLE (b) BONES
Q7. Arrange the following words as per order in the (c) BRAIN (d) ARSON
dictionary: Ans: (c) ‗I‘ is not appearing in the word
1.Inhabit ‗REASONABLE‘. Hence BRAIN cannot formed from
2.Ingenious the given word.
3.Inherit
4.Influence Q13. Identify the hidden name of the geometrical
5.Infatuation figure in the given word from among the answer
(a) 1, 2, 3, 4, 5 (b) 5, 4, 1, 2, 3 words whose letters are given in jumbled form.
(c) 4, 5, 2, 1, 3 (d) 5, 4, 2, 1, 3 SPIRITUAL
Ans: (d) Arrangement of the words as per dictionary (a) RISPAL (b) RITUAL
5.Infatuation  4.Influence  2.Ingenious  1.Inhabit (c) SPIRIT (d) RIPITU
 3.Inherit Ans: (a) RISPAL → SPIRAL

Q8. From the given alternative words, select the word Q14. Which will appear third in the dictionary?
which cannot be formed using the letters of the given (a) Monarchy (b) Monastic
word: (c) Monetary (d) Moneyed
MISFORTUNE Ans: (c) The correct order is : Monarchy, Monastic,
(a) FORT (b) TURN Monetary, Moneyed.
(c) SOFT (d) ROAM
Page 229 of 516
https://telegram.me/aedahamlibra
Q15. Arrange the following words according to
dictionary order:
(c) 18, 5, 8, 1, 3, 5, 20
Ans: (a)
(d) 20, 5, 8, 1, 3, 5, 18

1.Banquet
2.Bangle Q21. Which word will appear last in the dictionary?
3.Bandage (a) laugh (b) latch
4.Bantam (c) laurels (d) latitude
5.Bank Ans: (c) Arrangement of words as per dictionary:
(a) 3, 2, 4, 5, 1 (b) 3, 5, 2, 1, 4 2.Latch  4.Latitude  1.Laugh  3.Laurels
(c) 3, 2, 1, 5, 4 (d) 3, 2, 5, 1, 4
Ans: (d) According to dictionary, order is : Bandage, Q22. Arrange the given words in a meaningful order:
Bangle, Bank, Banquet, Bantam. 1.INFANT
2.ADOLESCENT
Q16. In this question, from the given alternatives 3.CHILD
select the word which cannot be formed by using the 4.OLD
letters of the given word. 5.ADULT
APPROPRIATE (a) 3, 1, 2, 4, 5 (b) 1, 3, 2, 5, 4
(a) PIRATE (b) APPROVE (c) 3, 2, 4, 5, 1 (d) 5, 4, 3, 2, 1
(c) PROPER (d) RAPPORT Ans: (b) Meaningful order of words :
Ans: (b) There is no 'V' letter in the given word. 1.INFANT  3.CHILD  2.ADOLESCENT 
5.ADULT  4.OLD
DIRECTIONS: In question below from the given
alternatives select the word which cannot be formed Q23. Arrange the following words in their ascending
using the letters of the given word. order, as in a dictionary:
1.Pick2. Pith
Q17. In the following letter series how many times do 3.Pile
PQR occur in such away that Q is in the middle of P 4.Perk
and R. 5.Pour
Q M P N P Q RR O P Q N O P P Q R P M Q R O P Q (a) 4, 1, 2, 3, 5 (b) 4, 1, 3, 2, 5
RPPRRPQRP (c) 4, 3, 2, 1, 5 (d) 5, 4, 3, 2, 1
(a) 5 (b) 6 Ans: (b) Perk > Pick > Pile > Pith > Pour
(c) 4 (d) 3
Ans: (c) Q24. Arrange the following words in their ascending
order:
Q18. ‗CONSTITUTIONAL‘ 1.Millenium
(a) CONSULT (b) LOCATION 2.Diamond Jubilee
(c) TUTION (d) TALENT 3.Silver Jubilee
Ans: (d) There is no 'E' letter in the given word. 4.Centenary
5.Golden Jubilee
Q19. In the following list of English alphabets, one (a) 2, 3, 5, 4, 1 (b) 2, 5, 3, 1, 4
alphabet has not been used. Identify the same. (c) 3, 5, 2, 4, 1 (d) 2, 3, 5, 1, 4
XNFAPSRWLTMDEXMGB Ans: (c) Silver jublee - 25 yr.
CXQJLOPVRCQJZOHSG Golden jublee - 50 yr.
ODIPTSMRABEFGNUNE Diamond jublee - 75 yr.
(a) V (b) I Centenary - 100 yr
(c) K (d) J Millennium - 1000 yr.
Ans: (c) DIRECTION: From the given alternatives select the
word which cannot be formed using the letters of the
Q20. If Alphabets are serially numberd, one of the given word.
answers given below has not a meaningful word
hidden in it. Identify the answer. Q25. Arrange the following words in their descending
(a) 5, 18, 5, 8, 1, 3, 5, 20 (b) 5, 1, 3, 5, 20, 8, 18 order:
Page 230 of 516
1.Weekly
2.Bi-annual
https://telegram.me/aedahamlibra Q30. Arrange the following words as per order in the
3.Fortnightly dictionary.
4.Monthly 1.Obscure
5.Annual 2.Objective
(a) 1, 3, 4, 2, 5 (b) 2, 5, 4, 1, 3 3.Objection
(c) 4, 1, 2, 3, 5 (d) 5, 2, 4, 3, 1 4.Obligation
Ans: (d) Annual - 1 yearMonthly - 30 days 5.Oblivion
Weekly - 7 daysBiannual - 6 month (a) 3, 2, 4, 5, 1 (b) 3, 2, 5, 4, 1
Fortnightly - 15 days. (c) 3, 2, 5, 1, 4 (d) 5, 2, 1, 3, 4
Ans: (a) The correct order is:
Q26. CONTENTION Objection > Objective > Obligation > Oblivion >
(a) TONIC (b) NOTE Obscure
(c) NATION (d) NOTION
Ans: (c) NATION as a letter is not there in word Q31. From the given alternative words, select the
CONTENTION. word which cannot be formed usi ng the letters of the
given word:
Q27. Arrange the following words as per order in the DISINTEGRATION
dictionary. (a) SIGN (b) SIGNATURE
(i) Forge (c) INTERN (d) SINGER
(ii) Forget Ans: (b) SIGNATURE cannot be formed as reference
(iii) Forgo word doesnot have ‗U‘ alphabet.
(iv) Forgive
(v) Format Q32. In the following letter series, how many times
(a) (v) , (ii) , (iv) , (iii) , (i) (b) (i) , (iv) , (iii) , (ii) , (v) does 'P' occur in such a w ay that after 'P', N' O
(c) (iii) , (iv) , (v) , (ii) , (i) (d) (i) , (ii) , (iv) , (iii) , (v) should occur?
Ans: (d) Arrangement of words as per dictionary: APNQPNOSPTOZPNOYMPNO
(i) Forge  (ii) Forget  (iv) Forgive  (iii) Forgo  (a) 2 (b) 5
(iv) Format (c) 4 (d) 3
Ans: (d) APNQ PNO SPTOZ PNO YM PNO
Q28. Which one of the given responses would be a
meaningful order of the following words? . Q33. From the given alternatives select the word
(i) Police which cannot be formed using the letters of tne given
(ii) Punishment word.
(iii) Crime SIGNATURE
(iv) Judge (a) SIGHT (b) GAIN
(v) Judgement (c) NATURE (d) GATE
(a) (i) , (ii) , (iv) , (iii) , (v) (b) (v) , (iv) , (iii) , (ii) , (i) Ans: (a) There in no 'H' in the word SIGNATURE
(c) (iii) , (i) , (iv) , (v) . (ii) (d) (iii) , (i) , (ii) , (iv) , (v)
Ans: (c) Meaningful order of words: Q34. From the given alternatives select the word
(iii) Crime  (i) Police  (iv) Judge  (v) Judgement which can be formed using the letters of the given
 (ii) Punishment word.
DICTIONARY
Q29. From the given alternatives select the word (a) BINARY (b) DAIRY
which cannot be formed using the letters of the given (c) NATION (d) ADDITION
word. Ans: (b) (a) BINARY can not be formed, there is no 'B'
Encouragement in the word DICTIONARY (c) NATION can not be
(a) game (b) tear formed, as there is only one 'N' in the word
(c) neck· (d) meat DICTIONARY (d) ADDITION can not be formed, as
Ans: (c) There is no ‗K‘ letter in the given word. So, that there is only one 'D' in the word DICTIONARY
word ‗Neck‘ cannot be formed.
Page 231 of 516
Q35. DISAPPOINTMENT
(a) POINTER
https://telegram.me/aedahamlibra
(b) OINTMENT
Ans: (d) There is only one ‗E‘ in the given word but
there are two Es in the word TEASE.
(c) DISPOINT (d) NOTPAD
Ans: (a) There is no ‗R‘ letter in the given word. Q41. RESURRECTIONIST
Therefore, the word POINTER cannot be formed. (a) TOURIST (b) NOISE
(c) SORCERER (d) TENDER
Q36. DECOMPOSITION Ans: (d) There is no ‗D‘ letter in the given word.
(a) COMPOSE (b) ECONOMIST Therefore, the word TENDER cannot be formed.
(c) POSITION (d) DOCTOR
Ans: (d) There is no ‗R‘ letter in the given word. Q42. COMMERCIALISM
Therefore, the word DOCTOR cannot be formed. (a) CROME (b) LANCER
(c) MISER (d) OSCAR
Q37. INTELLIGENCE Ans: (b) There is no ‗N‘ letter in given word. Therefore,
(a) NEGLECT (b) GENTLE the word LANCER cannot be formed.
(c) INCITE (d) CANCEL
Ans: (d) There is no ‗A‘ letter in the given word. DIRECTIONS: In questions, from the given
Therefore, the word CANCEL cannot be formed. alternatives select the word which cannot be formed
using the letters of the given word.
Q38. Which one of the given resp onses would be a
meaningful order of the following words in ascending Q43. Which one of the given responses would be a
order? meaningful order of the following?
1.Accommodation 1.Implementation
2.Perception 2.Conceptual Modelling
3.Scheme formation 3.Requirements Analysis
4.Assimilation 4.Logical Modelling
5.Sensation 5.Physical Model
(a) 1, 2, 3, 5, 4 (b) 5, 2, 3, 4, 1 6.Schema Refinement
(c) 5, 1, 4, 2, 3 (d) 5, 2, 4, 3, 1 (a) 1, 3, 2, 6, 5, 4 (b) 3, 2, 5, 4, 6, 1
Ans: (d) Meaningful order of t he words in ascending (c) 3, 2, 1, 4, 6, 5 (d) 3, 2, 4, 6, 5, 1
order : Ans: (d) Meaningful order :
5.Sensation  2.Perception  4.Assimilation  3. Requirements Analysis  2. Conceptual Modelling 
3.Scheme Formation  1.Accommodation 4. Logical Modelling  6. Schema Refinement  5.
DIRECTION: In question, from the given alternative Physical Model  1. Implementation
words, select the word which cannot be formed using
the letters of the given words. Q44. CONSTITUTIONAL
(a) TALENT (b) LOCATION
Q39. Which of the given responses would be a (c) CONSULT (d) TUITION
meaningful order of the following starting from the Ans: (a) ‗TALENT‘ can not be formed from the given
inner layer? word CONSTITUTIONAL as there is no ‗E‘ letter in the
1.Hydrosphere2. Atmosphere given word.
3.Biosphere4. Lithosphere
(a) 4, 1, 3, 2 (b) 2, 4, 3, 1 Q45. MANUSCRIPT
(c) 3, 1, 2, 4 (d) 1, 2, 3, 4 (a) SMART (b) RUSTIC
Ans: (a) Meaningful order of the words in ascending (c) MASTER (d) PRIMUS
order : 4.Lithosphere  1.Hydrosphere  3.Biosphere Ans: (c) ‗MASTER‘ can not be formed as there is no ‗E‘
 2.Atmosphere letter in the given word.

Q40. DISTANCE Q46. SEGREGATION


(a) DANCE (b) STAND (a) GREAT (d) GREETINGS
(c) SANE (d) TEASE (c) SEATING (d) NATION

Page 232 of 516


https://telegram.me/aedahamlibra
Ans: (d) ‗NATION‘ can not be formed as there is only
one ‗N‘ in the given word SEGREGATION.
Infant → Childhood → Adolescent → Adult → Old age
(5) (4) (3) (1) (2)

Q47. INVESTIGATION Q53. Son


(a) INSTIGATION (b) GESTATION 2.Husband
(c) VEST (d) STIGMA 3.Grandfather
Ans: (d) STIGMA cannot be formed using word 4.Brother
'INVESTIGATION' because letter M is not in the given 5.Father
reference word. (a) 1, 5, 2, 4, 3 (b) 1, 2, 3, 4, 5
(c) 1, 4, 2, 5, 3 (d) 1, 2, 4, 5, 3
Q48. SANCTION Ans: (c) Meaningful order is :
(a) ACTION (b) NATION Son → Brother → Husband → Father → Grandfather (1)
(c) NOTION (d) NION (4) (2) (5) (3)
Ans: (c) NOTION cannot be formed because two Os are
not in the reference word. DIRECTIONS: In questions, from the given
alternatives select the word which cannot formed
DIRECTIONS (61-62) : In each of the following using the letters of the given word.
questions, from the given alternative words, select th e
word which cannot be formed using the letters of the Q54. Arrange the following words as per order in the
given word. dictionary :
1.Command
Q49. RESEARCH 2.Commit
(a) SEARCH (b) REACH 3.Connect
(c) HEAR (d) READ 4.Conceive
Ans: (d) READ cannot be fomed as letter D is not in the 5.Conduct
reference word. 6.Commerce
(a) 6 2 1 5 4 3 (b) 6 1 2 4 5 3
Q50. HETEROGENOUS (c) 1 6 2 4 5 3 (d) 1 2 6 5 3 4
(a) REGENT (b) GENERATE Ans: (c) Dictionary order :
(c) STORE (d) ROUTE Command → Commerce → Commit → Conceive→
Ans: (b) There is no 'A' letter in the given word. Conduct → Connect
Therefore, the word GENERATE cannot be formed.
Q55. MISAPPREHENSION
DIRECTIONS: In questions, which one of the given (a) RESPONSE (b) HAPPINESS
responses would be a meaningful order of the (c) PERSON (d) SENSES
following. Ans: (d) 'SENSES' can not be formed using the letters of
the given word MISAPPREHENSION, as there are only
Q51. UNEQUIVOCAL two 'S' in the word.
(a) EQUAL (b) VOCAL
(c) QUAINT (d) NAIVE Q56. CONCEPTUALIZATION
Ans: (c) There is no 'T' letter in the given word. (a) ACTUAL (b) PETAL
Therefore, the word QUAINT cannot be formed. (c) PETROL (d) TOTAL
Ans: (c) 'PETROL' can not be formed as there is no 'R' in
Q52. Adult the given word 'CONCEPTUALIZATION'.
2.Old age
3.Adolescent Q57. From the given alternatives select the word
4.Childhood which can be formed using the letters given in th e
5.Infant word.
(a) 5, 4, 3, 1, 2 (b) 1, 5, 4, 3, 2 ULTRANATIONALISM
(c) 1, 2, 3, 4, 5 (d) 3, 4, 5, 1, 2 (a) ULTRAMONTANE (b) ULTRAMODERN
Ans: (a) Meaningful order is : (c) ULTRAIST (d) ULULATE
Page 233 of 516
Ans: (c) By options, https://telegram.me/aedahamlibra
(a) can not be formed as there is no ‗E‘ in the given word.
Q62. Reading
2.Listening
(b) can not be formed as there is no ‗D‘ in the given 3.Writing
word. 4.Speaking
(d) can not be formed as there is no ‗E‘ and only ‗U‘ in (a) 4, 2, 1, 3 (b) 2, 4, 3, 1
the given word. (c) 2, 4, 1, 3 (d) 4, 3, 2, 1
Ans: (c) Listening→ Speaking → Reading → Writing
Q58. From the given alternatives select the word (2) (4) (1) (3)
which cannot be formed using the letters of the given
word. Q63. Adulthood
LEGALIZATION 2.Infancy
(a) ALERT (b) ALEGATION 3.Childhood
(c) GALLANT (d) NATAL 4.Adolescence
Ans: (a) ALERT can not be formed as there is no ‗R‘ in (a) 1, 3, 4, 2 (b) 2, 3, 4, 1
the word LEGALIZATION. Hence, (a) is the correct (c) 2, 4, 3, 1 (d) 1, 2, 3, 4
choice. Ans: (b) Infancy→ Childhood → Adolescence →
Adulthood (2) (3) (4) (1)
Q59. From the given alternatives select the word
which cannot be formed using the letters of the given Q64. From the given alternatives select the word
word. which cannot be formed using the letters of the given
DEPARTMENT word :
(a) PEN (b) PARK INFLATIONARY
(c) PART (d) DEAR (a) FLAIR (b) FAULTY
Ans: (b) There is no 'K' in the word DEPARTMENT. (c) NATIONAL (d) RATION
Ans: (b) There is no ‗u‘ in the word INFLATIONARY.
DIRECTIONS: In Question, which one of the given
responses would be a meaningful order of the Q65. How many meaningful English words can be
following ? made with the letters ' OEHM ' using each letter only
once in each word ?
Q60. Which one of the given responses would be (a) FOUR (b) THREE
meaningful order of the following in ascending order? (c) TWO (d) ONE
1.Phrase Ans: (d) Home, only one meaningful word is formed.
2.Alphabet
3.Sentence Q66. From the given alternative words, select the
4.Word word which cannot be formed using the letters of the
(a) 2, 1, 4, 3 (b) 1, 2, 3, 4 given word:
(c) 2, 4, 1, 3 (d) 2, 4, 3, 1 CUMBERSOME
Ans: (c) Alphabet → Word → Phrase → Sentence (a) MOUSE (b) SOBER
(2) (4) (1) (3) (c) ROME (d) MERCY
Ans: (d) MERCY cannot be formed from the given word
Q61. Curd CUMBERSOME, as there is no ‗Y‘ in the word.
2.Milk
3.Butter milk Q67. Name a single letter, which can be prefixed to
4.Cow the following words in order to obtain entirely new
5.Ghee words ?
6.Butter TILL TABLE PILE TAB PRING
(a) 2, 5, 6, 4, 1, 3 (b) 4, 6, 2, 1, 3, 5 (a) S (b) B
(c) 4, 2, 1, 3, 6, 5 (d) 2, 6, 4, 5, 3, 1 (c) H (d) C
Ans: (c) Cow → Milk → Curd → Butter Milk → Butter Ans: (a) Only ‗S‘ can be prefixed to the given words.
→ Ghee (4) (2) (1) (3) (6) (5) New words are: still, stable, spile, stab, spring

Page 234 of 516


https://telegram.me/aedahamlibra
DIRECTIONS (Q. Nos. 86 -87) : Which one of the
given responses would be a meaningful order of the
Q73. Which one of the given responses would be a
meaningful order of the following ?
following? 1. Orange 2. Indigo 3. Red 4. Blue 5. Green 6. Yellow
7. Violet
Q68. Which one of the given responses would be a (a) 7, 2, 4, 5, 6, 1, 3 (b) 7, 2, 4, 6, 5, 1, 3
meaningful order of the following ? (c) 7, 2, 6, 4, 5, 1, 3 (d) 7, 2, 6, 4, 1, 5, 3
1.Elephant Ans: (a) These all are the colours of the rainbow. Hence
2.Cat meaningful order is V I B G Y O R.
3.Mosquito
4.Tiger DIRECTIONS: In question arrange the following
5.Whale words as per order in the dictionary.
(a) 5, 3, 1, 2, 4 (b) 1, 3, 5, 4, 2
(c) 3, 2, 4, 1, 5 (d) 2, 5, 1, 4, 3 Q74. Arrange the following words as per order in the
Ans: (c) Meaningful Order: dictionary.
Mosquito → Cat → Tiger → Elephant → Whale 1. Forecast 2. Forget 3. Foreign 4. Forsook 5. Force
(3) (2) (4) (1) (5) (a) 3, 5, 1, 2, 4 (b) 5, 1, 3, 2, 4
(c) 5, 1, 3, 4, 2 (d) 5, 1, 2, 3, 4
Q69. (a) Classroom Ans: (b) Force → Forecast → Foreign → Forget →
(b) Desk Forsook
(c) School
(d) Community Q75. Necessary
(5) Drawer 2.Navigate
(a) (4) , (2) , (3) , (1) , (5) (b) (3) , (2) , (4) , (5) , (1) 3.Nautical
(c) (4) , (3) , (1) , (2) , (5) (d) (1) , (2) , (3) , (4) , (5) 4.Naval
Ans: (c) Meaningful order is : (a) 3,4,2,1 (b) 3,2,4,1
Community → School → Classroom → Desk → Drawer (c) 2,4,3,1 (d) 4,3,2,1
(4) (3) (1) (2) (5) Ans: (a) Words as per order in dictionary
Nautical > Naval > Navigate > Necessary
Q70. (a) Gold So, correct order is 3, 4, 2, 1
(b) Ornaments
(c) Ore Q76. Range
(d) Mine 2.Rain
(5) Process 3.Rein
(a) (4) , (3) , (5) , (1) , (2) (b) (4) , (3) , (5) , (2) , (1) 4.Ranger
(c) (3) , (4) , (5) , (1) , (2) (d) (2) , (1) , (5) , (4) , (3) (a) 2,3,4,1 (b) 2,4,3,1
Ans: (a) Meaningful order is : (c) 2,1,4,3 (d) 2,4,3,1
Mine → Ore → Process → Gold → Ornaments Ans: (c) Rain > Range > Ranger > Rein
(4) (3) (5) (1) (2) So correct order is 2, 1, 4, 3

Q71. ADMINISTRATION Q77. Unscramble the following letters to frame a


(a) Station (b) Mind meaningful word. Then find out the correct numerical
(c) Ration (d) Minister position of the letters.
Ans: (d) Minister cannot be formed as there is no 'e' in OTysrhi
the word 'ADMINISTRATION'. 1234567
(a) 6241375 (b) 6452173
Q72. CHARACTER (c) 6347125 (d) 6742153
(a) Tracer (b) Hearty Ans: (d) H I S T O R Y
(c) Crate (d) Charter 6742153
Ans: (b) Hearty cannot be formed as th ey is no 'y' in the
word 'CHARACTER.

Page 235 of 516


https://telegram.me/aedahamlibra
Q78. If it is possible to form a word with the first,
fourth, seventh and eleventh letters in the word Q83. Voracious
'SUPERFLUOUS, write the first letters of that word. 2.Voucher
(a) O (b) E 3.Vortex
(c) S (d) L 4.Voluntary
Ans: (d) S U P E R F L U O U S (a) 2, 4, 1, 3 (b) 4, 1, 3, 2
1st 4th 7th 11th (c) 1, 4, 2, 3 (d) 3, 1, 4, 2
So, possible word with S, E, L, S is L E S S Ans: (b) The words can be arranged in following order in
dictionary :
Q79. Which one of the given responses would be a Voluntary Voracious  Vortex  Voucher
meaningful order of the following? (4) (1) (3) (2)
1.India So, correct order is 4,1,3,2
2.Bangalore
3.Asia Q84. Absolute
4.Karnataka 2.Abrasive
(a) 3, 1, 4, 2 (b) 3, 1, 2, 4 3.Absorption
(c) 1, 2, 3, 4 (d) 3, 4, 2, 1 4.Abundance
Ans: (a) Asia → India → Karnatka → Bangalore 5.Abiogenesis
(a) 2, 5, 1, 3,, 4 (b) 3, 4, 5, 2, 1
Q80. From the given alternative words, select the (c) 5, 2, 3, 1, 4 (d) 5, 2, 1, 3, 4
word which cannot be formed using the letters of the Ans: (d) Abiogenesis  Abrasive  Absolute 
given word. Absorption  Abundance So, correct order is 5,2,1,3,4
HARBINGER
(a) GARBAGE (b) RANGER Q85. From the given alternative words, select the
(c) BARRING (d) GARNER word which cannot be formed using the letters of the
Ans: (a) Word GARBAGE cannot be formed as requires given word :
two As. Given : IMPASSIONABLE
(a) IMPASSABLE (b) IMPOSSIBLE
Q81. From the given alternative words, select the (c) IMPASSIVE (d) IMPASSION
word which be formed using the letters of the given Ans: (c) There is no ‗V‘ letter in the given word.
word.
ENDEARMENT Q86. Only one meaningful word can be formed by
(a) TEMPER (b) MEANS rearranging the letter of the given jumbled word.
(c) TENDER (d) TENT Find out that word.
Ans: (c) ENDREARMENT, Word TENDER can be Given: MUSPOPAPOTIH
formed (a) METAMORPHIC (b) PHILANTHROPIST
(c) HIPPOCAMPUS (d) HIPPOPOTAMUS
DIRECTIONS: Arrange the following wo rds as per Ans: (d) Meaningful word HIPPOPOTAMUS
order in the dictionary.
Q87. If the words are organised in reverse order of
Q82. What is the best way to order the progression in what they appear in dictionary, which word will come
Hardware ? in the third place.
1.Silicon chips (a) Occulist (b) Odium
2.Transistors (c) Odious (d) Ordeum
3.Vacuum tube Ans: (c) Words in reverse order :
4.Integrated circuits. Ordeum → Odium → Odious → Occulist
(a) 3, 4, 1, 2 (b) 4, 2, 3, 1 (1) (2) (3) (4)
(c) 4, 1, 3, 2 (d) 3, 2, 4, 1
Ans: (d) Order of progresseon in hardware is Q88. Which of the following jumbled words is not an
Vacuum tubeTransistorIntegrated circuits  Silicon animal ?
Chips. (a) LATHPEEN (b) TICRECK
Page 236 of 516
(c) FEFEOC
Ans: (c) LATHPEEN →ELEPHANT
https://telegram.me/aedahamlibra
(d) TAR Ans: (b) Zeal, Zebra, Zest, Zinc, Zygote.

TICRECK → CRICKET Q94. From the given alternative words, select the
FEFEOC → COFFEE word which cannot be formed using the letters of the
TAR → RAT given word:
Hence, coffee is not animal. CONCENTRATION
(a) CONCERN (b) CONTAINER
Q89. Arrange the foll owing words as per order in the (c) CONCERT (d) CENTRAL
dictionary Ans: (d) There is no L in concentration ∴ CENTRAL
(a) Hale will not be Formed
(b) Hake
(c) Halt Q95. Arrange the following words as per order in the
(d) Hamlet dictionary
(5) Ham 1.Direction
(a) 1, 3, 5, 2, 4 (b) 2, 1, 3, 5, 4 2.Directed
(c) 2, 1, 4, 3, 5 (d) 2, 3, 5, 4, 1 3.Director
Ans: (b) HAKE > HALE > HALT > HAM > HAMLOT 4.Directing
(a) 1, 4, 3, 2 (b) 2, 4, 1, 3
Q90. From the given alternative words, select t he (c) 4, 2, 3, 1 (d) 4, 1, 2, 3
word which cannot be formed using the letters of the Ans: (b) Words arranged as per order in the Dictionary ,
given word. Directed, Directing, Direction, Director.
'BOOKBINDING'
(a) BINDING (b) DOING Q96. From the given alternative words, select the
(c) DIGGING (d) DINING word which cannot be formed using the letters of the
Ans: (c) DIGGING cannot be formed from the given given word:
word. ADMISSION
(a) MISSION (b) DISMISS
Q91. Amongst the given responses choose the (c) MASONS (d) NOMADS
meaningful logical series. Ans: (b) DISMISS cannot be make from ADMISSION
1.College
2.Infant Q97. Arrange the following words as per order in the
3.Child dictionary
4.School 1.Ambitions
5.Youth 2.Ambiguous
(a) 2,4,3,1,5 (b) 2,3,4,1,5 3.Ambiguity
(c) 2,4,3,5,1 (d) 2,3,4,5,1 4.Animation
Ans: (d) Infant becomes Child and goes to School than 5.Animal
grows up to Youth and goes to College ie, 2, 3, 4, 5, 1 (a) 3 ,2 , 4 , 1 , 5 (b) 3 , 2 , 5 , 4 , 1
(c) 3 , 2 , 1 , 5 , 4 (d) 3 , 2 , 4 , 5 , 1
Q92. Select the word which cannot be formed using Ans: (c) Arrangement as per Dictionary rules is 3, 2, 1, 5,
the letters of the given word REJUVENATION 4i.e Ambiguity, Ambiguous, Ambitions, Animal
(a) JUTE (b) NATION Animation
(c) REVISION (d) NATIVE
Ans: (c) Revision is the word which cannot be formed Q98. From the given alternative words , select the
from letters of REJUVENATION as, S is not in it. word which cannot be formed using the letters of the
given word:
Q93. Arrange the following words as per order in the INTELLIGENCE
dictionary: (a) CANCEL (b) NEGLECT
1. Zeal 2.Zebra 3.Zygote 4.Zinc 5.Zest (c) GENTLE (d) INCITE
(a) 1,2,5,3,4 (b) 1,2,5,4,3 Ans: (a) CANCEL cannot be Form from
(c) 1,5,2,4,3 (d) 1,5,2,3,4 INTELLIGENCE
Page 237 of 516
https://telegram.me/aedahamlibra
Q99. Arrange the following words according to the
Q103. Which one of the given responses would be a
meaningful order of the following?
dictionary order : 1.estimate
a.extol 2.contract
b.exinct 3.concept
c.extra 4.plan
d.extort 5.execute
(a) d, c, a, b (b) a, d, b, c (a) 1, 4, 3, 5, 2 (b) 5, 2, 4, 3, 1
(c) a, b, c, d (d) b, a, d, c (c) 3, 4, 1, 2, 5 (d) 3, 1, 2, 4, 5
Ans: (d) Words' Dictionary Order : Ans: (c) Concept → 4. Plan → 1. Estimate → 2. Contract
(1) (2) (3) (4) → 5. Execute
Extinct → Extol → Extort → Extra
(b) (a) (d) (c) Q104. PERAMBULATOR
(a) MARBLE (b) LABOUR
Q100. Which will appear fourth in the dictionary ? (c) RAMBLE (d) RAMPANT
(a) deterioration (b) determination Ans: (d) 'Rampant' cannot be formed by using given
(c) degrade (d) density word.
Ans: (b) Dictionary Order :
Degrade → Density → Deterioration → Determination Q105. ORGANISATION
(1) (2) (3) (4) (a) NATION (b) ORGAN
(c) ORGANISE (d) ORATION
Q101. If the following words are arranged in a Ans: (d) 'Organise' cannot be formed.
dictionary order, which will be 3rd word ?
Amphibian, Amorphous, Ambiguous, Ambidextrous, Q106. Grouping of words are given. Choose the next
Ambivalent words in the series from the given options.
(a) Ambiguous (b) Ambivalent AFGHAN, INDEFINITLY, SYNOPSIS,
(c) Amphibian (d) Amorphous STUPENDOUS, BURST
Ans: (b) Arrangement in a Dictionary Order : (a) RESCRIPT (b) COMPANION
Ambidextrous → Ambiguous → Ambivalent (c) HIJACKER (d) GLACIAL
(1) (2) (3) Ans: (c) AFGHAN, INDEFINITLY, SYNOPSIS,
→ Amorphous → Amphibian STUPENDOUS, BURST, HIJACKER.
(4) (5)
Q107. Arrange the given words in the sequence in
Q102. From the given alternative words, select the which they occur in the dictionary.
word which can be formed using the letters of the (a) Ropped
given word. (b) Roster
Measurement (c) Roasted
(a) Summit (b) Assure (d) Road
(c) Master (d) Mantle (5) Roller
Ans: (c) (a) Summit can not be formed as there is no 'i' in (a) 3 5 4 1 2 (b) 4 5 3 1 2
the word Measurement. (c) 3 4 5 1 2 (d) 4 3 5 1 2
(b) Assure can not be formed as there is only 's' in the Ans: (d)
word Measurement.
(d) Mantle can not be formed as there is no 'l' in the word Q108. In the following question, from the given
Measurement. alternative words, select the word which cannot be
formed using the letters of the given word.
DIRECTIONS: In the following questions, from the Suspensefulness
given alternative words, select the word which cannot (a) Sense (b) Fuels
be formed using the letters of the given word. (c) Useful (d) Fullness
Ans: (d) Fullness as a lettter is not there in word
suspense fulness.
Page 238 of 516
https://telegram.me/aedahamlibra
Q109. Arrange the given words in the sequence in
(c) 54132
Ans: (c)
(d) 53412

which they occur in the dictionary.


1.Pragmatic Q114. In the following question, from the given
2.Protect alternative words, select the word which cannot be
3.Pastel formed using the letters of the given word.
4.Postal Biological
5.Pebble (a) Logic (b) Globe
(a) 43521 (b) 35412 (c) Bail (d) Bill
(c) 34512 (d) 43512 Ans: (b) 'E' is not appearing in the word 'Biological'.
Ans: (b) Arranging the words according to dictionary. Hence Globe cannot formed from the given word.
Pastel → Pebble → Postal → Pragmatic → Protect.
So answer is 35412. Q115. Arrange the given words in the sequence in
which they occur in the dictionary.
Q110. In the following question, from the given (i) Treadmill
alternative words, select the word which cannot be (ii) Treason
formed using the letters of the given word. (iii) Treacherous
Herringbone (iv) Tread
(a) Biner (b) None (a) (ii) , (iii) , (iv) , (i) (b) (iii) , (iv) , (ii) , (i)
(c) Bane (d) Hinge (c) (iii) , (iv) , (i) , (ii) (d) (i) , (ii) , (iii) , (iv)
Ans: (c) Bane cannot be formed using Herringbone as it Ans: (c) According to dictionary order, Treacherous 
does not contain letter 'A'. Tread  Treadmill  Treason.
(1) (2) (3) (4)
Q111. Arrange the given words in the sequence in
which they occur in the dictionary. Q116. Which one of the given respons es would be a
(a) Ball meaningful order of the following?
(b) Balanced 1.Cutting
(c) Balls 2.Dish
(d) Balance 3.Vegetable
(5) Balancing 4.Market
(a) 24135 (b) 42135 5.Cooking.
(c) 42513 (d) 54213 (a) 1, 2, 4, 5, 3 (b) 5, 3, 2, 1, 4
Ans: (c) (c) 3, 2, 5, 1, 4 (d) 4, 3, 1, 5, 2
Ans: (d)
Q112. In the following question, from the given
alternative words, select the word which cannot be Q117. Out of the four words given, choose the word
formed using the letters of the given word. which cannot be formed using the letters of the
UNIFORMITY following word.
(a) ANNUITY (b) FORUM TRANSLATION
(c) MINT (d) UNIFORM (a) RATIO (b) NATION
Ans: (a) ANNUITY is the word which cannot be formed (c) TRANSMIT (d) TRANSIT
from letters of 'UNIFORMITY' as, A is not in it. Ans: (c)

Q113. Arrange the given words in the sequence in DIRECTIONS: Arrange the given words in the
which they occur in the dictionary. sequence in which they occur in the dictionary.
1.Storm
2.Strap Q118. Ale
3.Strangle 2.Align
4.Stamped 3.Amend
5.Satire 4.Anatomy
(a) 51432 (b) 51342 5.Alpine
Page 239 of 516
(a) 51342
(c) 12354
https://telegram.me/aedahamlibra
(b) 35412
(d) 12534
Ans: (a) ,
Change of +1
Ans: (d)
Q2. In a certain code SISTER is written as RHRSDQ.
Q119. Den How is UNCLE written in that code ?
2.Dragon (a) TMBKD (b) TBMKD
3.Drop (c) TVBOD (d) TMKBD
4.Desk Ans: (a) As,
5.Doom ,
(a) 54312 (b) 14523 Change of -1
(c) 31245 (d) 13245
Ans: (b) Q3. If in a certain code HYDROGEN is written as
JCJZYSSD, then how can ANTIMONY be written in
Q120. In the following question, select the word which that code?
cannot be formed using the letters of the given word. (a) CPVKOQPA (b) CRZQWABO
Instructor (c) ERXMQSRC (d) G'I'ZOSU'I'E
(a) Rust (b) Tort Ans: (b) Difference is +2, + 4, +6, +8, +10, +12, +14,
(c) Stop (d) Stint +16
Ans: (c) Stop cannot be formed as there is no 'P' in the
word 'Instructor'. Q4. If PEAR is written as GFDN, how is REAP
written in this code?
Q121. Which word will appear second when arranged (a) FDNG (b) NFDG
in order of dictionary? (c) DNGF (d) NDFG
(a) Boost (b) Bonkers Ans: (b)
(c) Bonjour (d) Bannet
Ans: (b) According to dictionary. Q5. If FLATTER is coded as 7238859 and MOTHER
Bonkers will a appear second position. is coded as 468159, then how is MAMMOTH coded?
(a) 4344681 (b) 4344651
Q122. Which word will appear last when arranged in (c) 4146481 (d) 4346481
order of dictionary? Ans: (a)
(a) Heritage (b) Hermitage
(c) Healing (d) Heap Q6. In a certain code, ―CERTAIN‖ is coded as
Ans: (b) According to dictionary, ―XVIGZRM‖, ―SEQUENCE‖ is coded as
Hermitage will appear last position. ―HVJFVMXV‖. How would ―REQUIRED‖ be
coded?
Q123. DISTRIBUTION (a) FJIVWVIR (b) VJIFWTRV
(a) BUTTON (b) DISTURB (c) WVJRIFVI (d) IVJFRIVW
(c) BRITAIN (d) BURNT Ans: (d) Pairs of Opp Letters
Ans: (c) Britain cannot be formed as there is no 'A' in the
word ' DISTRIBUTION'. Q7. If LUXOR is coded as 30, then GUILDS will be
coded as?
(a) 40 (b) 36
(c) 38 (d) 24
Coding Decoding Ans: (d) Add place values

Q8. In certain co de, RAGHAVAN is written as


Q1. If SPARK is coded as TQBSL, what will be the GARVAHNA. In that code which word will be written
code for FLAME ? as MATHAVAN?
(a) GMBNF (b) GNBNF (a) TAMVAHNA (b) TAMVAHAN
(c) GMCND (d) GMBMF (c) TAMHAVNA (d) MATVAHNA
Ans: (a) ,
Page 240 of 516
Reverse order https://telegram.me/aedahamlibra
first by its row and next column number. E.g. ‗F‘ can
be repre sented by 14, 21 etc. ‗T‘ can be represented
Q9. If ‗FRIEND‘ is coded as ‗IULHQG‘, how will you by 59, 78, etc. Similarly identify the set for the word
code ‗ENEMY‘? POSE.
(a) HQHPB (b) HQHPA Matrix I Matrix II
(c) HQEMY (d) HQHPG
Ans: (a) As,
, Increase of +3 in each letter

Q10. WAYIN is written as TXVFX. How LBUK can


be written in that code?
(a) IYRH (b) KATJ
(c) JZSI (d) NDWM
Ans: (a) As, Change of -3 (a) 87, 55, 89, 43 (b) 68, 98, 58, 21
(c) 75, 86, 67, 14 (d) 56, 67, 77, 01
Q11. In a certain code language, if the word Ans: (d) P → 56, 6 8, 75, 87, 99
PARTNER i s coded as OZQSMDQ, then what is the O → 55, 67, 79, 86, 98
code for the word SEGMENT? S → 58, 65, 77, 89, 96
(a) TFHNFOU (b) RDFLDMS E → 01, 13, 20, 32, 44
(c) RDELDMS (d) RDFEDNS POSE →56, 67, 77, 01
Ans: (b) Change of -1
Q16. If PALE is written as RCNG, how can LEAP be
Q12. If DEGI is equal to 25, what is FEHD equal to? written in that code?
(a) 32 (b) 25 (a) NGCR (b) RCGN
(c) 52 (d) 23 (c) CRNG (d) NCRG
Ans: (d) The place value of, F E H D 6+5+8+4 = Ans: (a) Change of +2 successively
23
Q17. If ‗POST‘ is coded as ‗KLHG‘, how is ‗NURS‘
Q13. If SISTER is coded as 20, 10, 20, 21, 6, 19, then coded as?
the code for BROTHER is (a) MGJH (b) MGJH
(a) 2, 15, 16, 21, 9, 5, 18 (b) 3, 19, 16, 21, 9, 6, 19 (c) MFIH (d) MFIG
(c) 4, 20, 15, 18, 8, 7, 9 (d) 3, 18, 16, 20, 9, 7, 19 Ans: (c)
Ans: (b) Code: Place value +1
Q18. If GARMENT is written as 202691422137, how
Q14. If Ame rica is called Greenland, Greenland is is INDULGE written in that code?
called Africa, Africa is called Russia, Russia is called (a) 9144211275 (b) 914211275
India and India is called Pakistan; Delhi is called the (c) 1813326152022 (d) 1813236152022
capital of which country? Ans: (d) Place values
(a) Russia (b) India
(c) Pakistan (d) Greenland Q19. If A = 1, ACE = 9, then ART = ?
Ans: (c) Delhi is the capita l of India but India is called (a) 29 (b) 38
Pakistan. Therefore, Delhi is called the capital of (c) 10 (d) 39
Pakistan. Ans: (d) A = 1,A + C + E = 1 + 3 + 5 = 9
A + R + T = 1 + 18 + 20 = 39
Q15. A word is represented by only one set of
numbers as given in any of the alternatives. The sets Q20. If PARK is coded as 5394, SHIRT is coded as
of numbers given in the alternatives are represented 17698 and PANDIT is coded as 532068, how woul d
by two cl asses of alphabets as in the 2 matrices given you code NISHAR in that code language?
below. The columns and rows of matrix I are (a) 201739 (b) 261739
numbered from 0 to 4 and that of matrix II from 5 to (c) 266734 (d) 231954
9. A letter from these matrices can be represented
Page 241 of 516
Ans: (b) https://telegram.me/aedahamlibra
Q21. If 'SYNDICATE‘ is written as ‗SYTENDCAI
then how can ‗PSYCHOTIC‘ be written ?
(a) PSICYOCTH (b) PSICYCOTH
(c) PSYICTCOH (d) PSYCOHTCI
Ans: (b)

Q22. Some letters are given below in the first line and (a) 44, 62, 65, 51 (b) 53, 42, 65, 36
numbers are given below them in the second line. (c) 53, 54, 51, 31 (d) 44, 54, 65, 24
Numbers are the codes for the alphabets and vice - Ans: (a) By matching code
versa. Choose the correct letter -code for the given set 44, 62, 65, 51 Letters Resembles to CALM in the
of numbers. MATRI X.
EMKBZWQUDJ
5916482073 Q26. If DIVINE is coded as AFSFKB, then
429753 POWERFUL is coded as
(a) Z Q M D E J (b) Z Q E D M J (a) XLHOJVIM (b) MLTBDCRI
(c) Z Q M J D E (d) Z Q M E D J (c) MLWBOCRI (d) HLTBNCRI
Ans: (a) Ans: (*) As,Change of -3

Q23. If DELHI is coded as 73541 and CALCUTTA as Q27. If NOTE is written as PQVG, then TIME is
82589662, then how would CALICUT be coded in that written as
code? (a) VQOG (b) VKOG
(a) 5978213 (b) 8251896 (c) VOKG
(c) 8543691 (d) 5279431 (c) VGKO
Ans: (b) Ans: (b) As,
Change of +2
Q24. If MOB ILE is written as ZAMSUM, how
TUMOR can be written in that code ? Q28. If SMART is coded as UKCPV, then WONDER
(a) BRAIN (b) HGYAD is coded as
(c) GGXYA (d) IHZBE (a) YMPPRT (b) YMPBGP
Ans: (c) (c) YMPBFP (d) YMBPPG
Ans: (b) As,
Q25. A word is represented by only one set of Change of +2 -2 alternatively
numbers as given in any one of the alternatives. The
sets of numbers given in the alternativ es are Q29. A word is represented by only one set of
represented by two classes of alphabets as in the numbers as given in any one of the alternatives. The
matrix given below. The columns and rows of Matrix sets of numbers given in the alternatives are
are numbered from 0 to 6. A letter from the matrix represented by two classes of alphabets as in two
can be represented first by its row and next by its matrices given below. Two columns and rows of
column, e.g.'A' can be represented by 42, 62, etc. and Matrix I are numbered-from 0 to 4 and that of Matrix
'P' can be represented by 15, 43, etc. Similarly, you II are numbered from 5 to 9. A letter f rom these
have to identify the set for the word 'CALM'. matrices can be represented first by its row and next
Matrix by its column, e.g., ‗P‘ can be represented by 67, 75
etc. Similarly, you have to identify the set for the word
‗CARD‘.
MATRIX I MATRIX II

Page 242 of 516


https://telegram.me/aedahamlibra
Q32. A word is represented by only one set of
numbers as given in any one of the alternatives. The
sets of numbers given in the alternatives are
represented by two classes of alphabets as in two
matrices given below. Th e columns and rows of
Matrix I are numbered from 0 to 4 and that of Matrix
II are numbered from 5 to 9. A letter from these
(a) 32, 00, 56, 10 (b) 40, 21, 68, 44
matrices can be represented first by its row and next
(c) 11, 33, 57, 22 (d) 02, 42, 77, 20
by its column, e.g. ‗D‘ can be represented by 02, 14,
Ans: (c) C 02, 11, 23, 32, 40
etc., and ‗R‘ can be represented by 57, 76 etc.
A 00, 13, 21, 33, 42
Similarly, you have to identify the set for the word
R 57, 68, 77, 88, 99
―BEST‖.
D 03, 10, 22, 30, 41
MATRIX- I MATRIX -II

(a) 24, 21, 77, 97 (b) 24, 22, 77, 97


Q30. If each of the letters in the English alphabet is (c) 24, 22, 77, 96 (d) 24, 22, 76, 97
assigned an even numerical value by giving A = 2, B = Ans: (b) B 00, 12, 24, 31, 43
4 and so on, what would be the total value of the E 03, 10, 22, 34, 41
letters for the word LADY when similarly coded? S 58, 65, 77, 89, 96
(a) 72 (b) 84 T 59, 66, 78, 85, 97
(c) 82 (d) 74
Ans: (b) L 12 × 2 = 24 Q33. If each of the letter in the English alphabet is
A 01 × 2 = 02 assigned odd numerical value beginning coding A = 1,
D 04 × 2 = 08 B = 3 & so on, what will be the total value of the letter
Y 25 × 2 = 50 of the word ‗SNAKE‘?
Total = 84 (a) 95 (b) 105
(c) 115 (d) 113
Q31. If the word LEADER is coded as 20 -13-9-12-13- Ans: (a) Add Place values
26, how would you write LIGHT?
(a) 20-15-16-18-23 (b) 20-17-15-16-28 Q34. If DFIN is coded as WURM, then HJMO can be
(c) 20-16-15-17-22 (d) 20-16-17-15-27 coded as
Ans: (b) L 12 + 8 = 20 (a) RPNO (b) SQNP
E 5 + 8 = 13 (c) SQNL (d) TRPO
A 1+8=9 Ans: (c)
D 4 + 8 = 12
E 5 + 8 = 13 Q35. If RUMOUR can be written as QSJKPL, then
R 18 + 8 = 26 how HERMIT can be written?
Therefore, (a) GEPKHR (b) GCOIDN
L 12 + 8 = 20 (c) GCPIDM (d) GCPIEN
I 9 + 8 = 17 Ans: (b)
G 7 + 8 = 15
H 8 + 8 = 16 Q36. In a certain code, LONDON is coded as
T 20 + 8 = 28 24 – 30 – 28 – 8 – 30 – 28. How will FRANCE be
coded?

Page 243 of 516


10
https://telegram.me/aedahamlibra
(a) 10 – 24 – 6 – 28 – 6 – 12 (b) 12 – 26 – 6 – 28 – 8 – matrix are numbered from 1 to 6. A letter from these
matrices can be represented first by its row and next
(c) 12 – 36 – 2 – 28 – 6 – 10 (d) 12 – 26 – 2 – 28 – 8 – by its column number. e.g., 'A' can be represented by
10 42. 'S' can be represented by 21, etc. Similarly. you
Ans: (c) L 12; 12 × 2 = 24 have to identify the set for the word 'PLAY'.
O 15; 15 × 2 = 30
N 14; 14 × 2 = 28
D 04; 04 × 2 = 08
O 15; 15 × 2 = 30
N 14; 14 × 2 = 28
Therefore,
F 06; 06 × 2 = 12
R 18; 18 × 2 = 36
A 01; 01 × 2 = 02
N 14; 14 × 2 = 28 (a) 43, 36, 42, 23 (b) 43, 32, 33, 33
C 03; 03 × 2 = 06 (c) 15, 12, 42, 45 (d) 43, 65, 62, 45
E 05; 05 × 2 = 10 Ans: (d) P 15, 43;
L 36, 65;
Q37. If each of the letters in the English alphabet is A 42, 46, 62 :
assigned odd Y 45
numerical value beginning A = 1, B = 3 and so on,
what will the total value of the letters for the word
'HOTEL' ?
(a) 95 (b) 115
(c) 125 (d) 105
Ans: (b) A 1 × 2 – 1 = 1
B 2×2–1=3
Therefore,
H 8 × 2 – 1 = 15
O 15 × 2 – 1 = 29
T 20 × 2 – 1 = 39
Q40. In a certain code, DOWNBEAT is written a
E 5 × 2 – 1 = 09
TABEWNDO. How will the woprd PROSPECT be
L 12 × 2 – 1 = 23/115
written in that code?
Total Value = 115
(a) TCEPSORP (b) TCPEOSPR
(c) TCPESOPR (d) TCPEOSRP
Q38. In a certain code, MAARK is written as
Ans: (b) Reverse Order
KRAAM. How PAS- SI can be written in that code?
(a) ISSAP (b) ISSPA
Q41. If SENSATIONAL can be written as 1 2 3 1 4 5 6
(c) SSIPA (d) ASS1P
7 3 4 8, how will STATION be written in that code?
Ans: (a) The letters have been written in the reverse
(a) 1 4 5 5 6 7 3 (b) 1 5 4 5 7 6 3
order.
(c) 1 5 5 4 6 7 3 (d) 1 5 4 5 6 7 3
MAARK KRAAM
Ans: (d)
Therefore,
PASSI ISSAP
Q42. If MILITARY can be written as 1, 2, 3, 2, 4, 5, 6,
7, how can LIMIT be written in that code?
Q39. In the following question, a word is represented
(a) 3 2 1 2 4 (b) 4 2 1 2 3
by a set of numbers as given in any one of t he
(c) 1 2 3 2 4 (d) 4 2 1 2 5
alternatives. The sets of numbers given in the
Ans: (a)
alternatives are represented by alphabets as in the
matrices given below. The columns and rows of

Page 244 of 516


https://telegram.me/aedahamlibra
Q43. If LPPHGLDWH means IMMEDIATE, what
does GRPDLQ stand for?
(c) TNATSSIA (d) TNATSISAS
Ans: (a) Reverse order of MANAGER = REGANAM
(a) MATTER (b) DOMAIN Reverse order of ASSISTANT = TNATSISSA
(c) ORANGE (d) DANGER
Ans: (b) Q48. Number of letters skipped in between adjacent
letters in the series increased by one. Which of the
Q44. Aurangabad, September 19, 2009 following series observe the rule ?
(a) Aurangabad, Septemb er 19 2009 (a) KORYBGJ (b) LMEYTPK
(b) Aurangabad, 19 September, 2009 (c) KMPTYEL (d) KPTYELM
(c) Aurangabad, September 19, 2009 Ans: (c)
(d) Aurangabad, September, 19, 2009
Ans: (c) Q49. In a certain code DEPUTATION is written as
ONTADEPUTI. How is DERIVATION written in
Q45. If the word PRINCIPAL is written as that code ?
LAPICNIRP, how ADOLESCENCE can be written in (a) ONVADERITI (b) ONDEVARITI
that code ? (c) ONVAEDIRTI (d) ONVADEIRIT
(a) ECNCESELODA (b) ECNECSLEODA Ans: (a)
(c) ECNSCEELODA (d) ECNECSELODA
Ans: (d) Reversing the order Q50. If MADRAS is coded as 517916 and TENANT is
coded as 432124, how would you encode RMATSN?
Q46. A word is represented by only one set of (a) 851353 (b) 951363
numbers as given in any one of the alternatives. The (c) 951462 (d) 941562
sets of numbers given in the alternatives are Ans: (c)
represented by two classes of alphabets as i n two
matrices given below. The columns and rows of Q51. A word is represented by one set of numbers as
Matrix -I are numbered from 0 to 4 and that of given in any one of the alternatives. The sets of
Matrix - II are numbered from 5 to 9. A letter from numbers given in the alternatives are represented by
these matrices can be represented first by its row and two classes of alphabets as in two matrices given
next by its column, e.g., A can be represented by 01, below. The columns and rows of Matrix I are
20, 42 etc. and H can be represented by 65, 57, 98 etc. numbered from 0 to 4 and that of Matrix II are
Similarly, you have to identify the set for the word numbered from 5 to 9. A letter from these matrices
given in the question. can be represented first by its row and next by its
FAITH column e.g., 'A' can be represented by 40, 01, 13, 32,
Matrix -I Matrix -II and 'N' can be represented by 56, 68, 89 etc. Similarly,
you have to identify the set for the word given below :
SIX-KIDS
Matrix I Matrix II

(a) 24, 31, 10, 59, 57 (b) 12, 20, 40, 68, 65
(c) 31, 34, 23, 76, 79 (d) 43, 42, 41, 78, 89
Ans: (c)
(a) 86, 87, 99 — 40, 41, 86, 64
Q47. In a coded language, MANAGER is written as (b) 98, 96, 85 — 42, 78, 88, 77
REGANAM. How will ASSISTANT be written in that (c) 77, 69, 76 — 22, 95, 28, 31
code ? (d) 65, 55, 67 — 05, 25, 91, 40
(a) TNATSISSA (b) TNATISSSA Ans: (b)

Page 245 of 516


https://telegram.me/aedahamlibra
Q52. If CASUAL is coded as SACLAU, then what Q59. In a certain code, '253' means 'books are old';
would be the code of MATRIC ? '546' means 'man is old' and '378' means 'buy good
(a) CIRTAM (b) TMAICR books'. What stands for "are" in that code?
(c) TAMCIR (d) ATMCIR (a) 2 (b) 4
Ans: (c) (c) 5 (d) 6
Ans: (a)
Q53. In a language FIFTY is written as CACTY, CAR
as POL, TAR as TOL, how can TARIFF be written in Q60. A word is represented by only one set of
that language? numbers as given in any one of the alternatives. The
(a) TOEFEL (b) TOEFDD sets of numbers given in the alternatiives are
(c) TOLADD (d) TOLACC represented by two classes of alphabets as in two
Ans: (d) matrices given below. The columns and rows of
Matrix I are numbered from 0 to 4 and that of Matrix
Q54. If GOODNESS is coded as HNPCODTR, how II are numbered from 5 to 9. A letter from these
can GREATNESS be written in that code? matrices can be represented first by its row and next
(a) HQFZSMFRT (b) HQFZUFRTM by its column, e.g.. 'A' can be represented by 01, 14
(c) HQFZUODTR (d) HQFZUMFRT etc. and 'O' can be represented by 59, 67 etc.
Ans: (d) similarly, you have to identify the set for the word
'PEARL'.
Q55. If LISTEN is coded as 593417 then SILENT is Matrix – I & II
code as :
(a) 591734 (b) 391754
(c) 591743 (d) 395174
Ans: (d)

Q56. If FADE is coded as 3854 then how can GAGE


be coded?
(a) 2834 (b) 2824 (a) 00, 55, 22, 11, 96 (b) 00, 66, 14, 32, 56
(c) 2814 (d) 1824 (c) 13, 77, 30, 14, 88 (d) 12, 88, 43, 32, 89
Ans: (b) Ans: (a)

Q57. Name a single letter, which can be prefixed to Q61. In a certa in coding system APPLE stands for
the following words in order to obtain entirely new ETTPI. What is the code for 'DELHI'?
words ? (a) CQMND (b) ZAHDE
TILL TABLE PILE TAB PRING (c) HIPLM (d) CQPLM
(a) S (b) B Ans: (c) ,
(c) H (d) C Change of +4 in each letetr
Ans: (a) Only ‗S‘ can be prefixed to the given words.
New words are: Q62. Name a single letter, which can be deleted from
still, stable, spile, stab, spring the body of the following words to form entirely n ew
words?
Q58. In a certain code "MOUSE" is written as HOST POST COST LOST STOP
"PRUQC". How is "SHIFT" written in that code? (a) T (b) P
(a) VJIDR (b) VIKRD (c) S (d) O
(c) RKIVD (d) VKIDR Ans: (c) ‗S‘ can be deleted from the body of the
Ans: (d) MOUSE following words to form entirely new words.
↓ +3↓ +3↓ +0↓ –2↓ –2 New words are :
Coded as:PRUQC Similarly, SHIFT HOT, POT, COT, LOT, TOP
↓ +3↓ +3↓ +0↓ –2↓ –2
Coded as:VKIDR
Page 246 of 516
https://telegram.me/aedahamlibra
Q63. If CUSTOM is written as UCTSMO then how
PARENT will be written in the same code?
(a) 20
(c) 26
(b) 25
(d) 27
(a) TNERAP (b) RAPTNE Ans: (d) D A S H 4 + 1 + 19 + 8 = 32
(c) ERAPTN (d) APERTN D A N C E 4 + 1 + 14 + 3 + 5 = 27
Ans: (d)
Q68. In the following question, number of letters
Q64. If 'air' is called 'green', green is called 'blue', skipped in between adja cent letters of the series
'blue' is called 'sky', 'sky' is called 'yellow', 'yellow' is starting from behind increased by one. Which of the
called 'water' and water is called 'Pink' then what is following observes the rule?
the colour of clear sky ? (a) OIGDC (b) OMKIG
(a) Yellow (b) Water (c) ONLKJ (d) OMJFA
(c) Sky (d) Blue Ans: (d)
Ans: (c) The color of sky is blue. But blue is called sky.
Hence, option (c) is correct choice. Q69. If Blue means Pink, Pink means Green, Green
means Yellow, yellow means Red and Red means
Q65. If in a certain language TEACHER is coded as White, then what is the colour of turmeric ?
QBXZEBO, then how is STUDENT coded in the sa me (a) Pink (b) Yellow
language? (c) Red (d) Green
(a) PQRBAQK (b) PQRABKQ Ans: (c) Originally the colour of turmeric is yellow, here,
(c) PQRKBAQ (d) PRKQBAQ yellow means red. So the colour of turmeric is red.
Ans: (b) Each Alphabet of QBXZEBO is 3 less than
TEACHER Q70. If DANGER is coded as 11 - 8 - 21 - 14 - 12 - 25,
∴ STUDENT will be written as PQRABKQ then how will be coded the word MACHINE ?
(a) 10 - 21 - 15 - 14 - 26 - 17 - 18
Q66. In the question, a word is represented by only (b) 20 - 8 - 10 - 16 - 17 - 22 - 13
one set of numbers as given in any one of the (c) 20 - 10 - 8 - 12 - 15 - 16 - 7
alternatives. The sets of numbers given in the (d) 20 - 8 - 10 - 15 -16 - 21 - 12
alternatives are represented by two classes of Ans: (d)
alphabets as in two matric es given below. The
columns and rows of Matrix I are numbered from 0 to Q71. A word is represented by only one set of
4 and that of Matrix II are numbered from 5 to 9. A numbers as given i n any one of the alternatives. The
letter from these matrices can be represented first by sets of numbers given in the alternatives are
its row and next by its column, e.g., 'P' can be represented by two classes of alphabets as in two
represented by 02, 13, etc., and 'A' can be represented matrices given below. The columns and rows of
by 57, 68, etc. Similarly you have to identify the set for matrix I are numbered from 0 to 3 and that of Matrix
the word 'GUNS' II are numbered from 4 to 7. A letter from these
matrices can be represented first by its now and next
by its column, e.g., 'D' can be represented by 01 and
'R' can be represented by 44, Similarly, you have to
identify the set for the word 'TALE'
Matrix - IMatrix - I

(a) 88, 23, 59, 33 (b) 66, 40, 67, 11


(c) 55, 34, 77, 44 (d) 99, 12, 86, 22
Ans: (d) 99, 12, 86, 22 are the codes matched from two
Matrices, for GUNS.

(a) 64, 00, 31, 32 (b) 46, 13, 00 23


Q67. If C is coded 3 , DASH is coded as 32 , then
(c) 00, 31, 64, 32 (d) 30, 76, 23, 32
DANCE will be coded as
Ans: (a)
Page 247 of 516
https://telegram.me/aedahamlibra
Q72. In a certain code language "NIGHT" is written
Q76. If 'LONDON is coded as MPOEPO. What code
is needed for 'DELHI'?
as "ODDGM" and "DARK" is written as "GOYC". (a) DEHLI (b) HLDEI
How is "GREEN" written in that code language? (c) EFIMJ (d) EFMIJ
(a) IABPF (b) MCBNB Ans: (d)
(c) OGHVL (d) FPBAI
Ans: (a) Reversing Positions Q77. In a certain code language, ''SURGEON" is
written as "QLHDURV" and "CORNER" is written
Q73. In a certain code language, "TIRED" is written as "OHKULF". How is "SHADOW" written in that
as "56" and "BRAIN" is written as "44". How is code language?
'LAZY" written in that code language? (a) DRTERS (b) TRADEV
(a) 64 (b) 61 (c) UQBCFU (d) TFBCPX
(c) 58 (d) 43 Ans: (b) Reversing Positions
Ans: (a) As,
TIRED = 20 + 9 + 18 + 5 + 4 = 56 Q78. In a certain code language, "SUN is written as
BRAIN = 2 + 18 + 1 + 9 + 14 = 44 Similarly, LAZY = 12 "54" and "PUT" is written as "57". How is "CAT"
+ 1 + 26 + 25 = 64. Written in that code language?
(a) 28 (b) 24
Q74. In a certain code language, "RIVER" is written (c) 52 (d) 36
as "12351" and "RED" is written as "156". How is Ans: (b) A = 1, B = 2,.......... Z = 26
"DRIVER" written in that code language? As,
(a) 612311 (b) 612531 SUN (19 + 21 + 14) = 54
(c) 621351 (d) 612351 PUT (16 + 21 + 20) = 57 Similarly, CAT (3 + 1 +
Ans: (d) 20) = 24

Q75. A word is represented by only one set of Q79. A word is represented by only one set of
numbers as given in any one of the alternatives. The numbers as given in any one of the alternatives. The
sets of numbers given in the alternatives are sets of numbers given in the alternatives are
represented by two classes of alphabets as shown in represented by two classes of al phabets as shown in
the given two matrices. The columns and rows of the given two matrices. The columns and rows of
Matrix – I are numbered from 0 to 4 and that of Matrix-I are numbered from 0 to 4 and that of
Matrix – II are numbered from 5 to 9. A letter from Matrix-II are numbered from 5 to 9. A letter from
these matrices can be represented first by its row and these matrices can be represented first by its row and
next by its column, for example, 'E' can be next by its column, for example 'Q' c an be
represented by 02, 11, etc. and 'G' can be represented represented by 12, 43, etc.., and 'M' can be
by 65, 56 etc. Similarly, you have to identify the set for represented by 67, 99, etc. Similarly, you have to
the word 'EAGER;'. identify the set for the word "PRICE".
Matrix – I Matrix – II Matrix – I Matrix – II

(a) 02, 10, 65, 11, 68 (b) 02, 10, 65, 87, 85 (a) 23, 03, 55, 66, 99 (b) 42, 24, 88, 56, 65
(c) 02, 10, 65, 11, 85 (d) 02, 10, 65, 59, 85 (c) 11, 10, 96, 97, 85 (d) 04, 41, 69, 75, 57
Ans: (c) Ans: (d) 'P' can be represented by 04
'R' can be represented by 41

Page 248 of 516


'I' can be represented by 69
'C' can be represented by 75
https://telegram.me/aedahamlibra
Ans: (b) Seema > Sohan > Seeta... (i) Deepti > Sweta >
Seema... (ii)
'E' can be represented by 57 Combining (i) and (ii) we get
Deepti > Sweta > Seema > Sohan > Seeta
Q80. In a certain code language, "POTTER" is
written as "ONSUFS". "WALKER" written in that Q5. A man showed a boy next to him and said - ―He is
code language? the son of my wife‘s sister -in-law, but I am the only
(a) VZKLFS (b) VZLKFS child of my parents.‖ How is my son related to him?
(c) VZKLFT (d) WALLFS (a) Brother (b) Uncle
Ans: (a) (c) Nephew (d) Cousin

Blood Relationship
Ans: (d)
Q1. Nithya is Sam‘s Sister. Mogan is Sam‘s Father.
Selvan is Rajan‘s Son. Rajan is Mogan‘s Brother. Q6. Sunil is the son of Kesav. Simran, Kesav‘s sister,
How is Nithya related to Selvan? has a son Maruti and daughter Sita. Prem is the
(a) Daughter (b) Sister maternal uncle of Maruti. How is Sunil related to
(c) Cousin (d) Wife Maruti?
Ans: (c) Nithya is Sam‘s Sister and Mogan is Sam‘s (a) Uncle (b) Brother
Father (c) Nephew (d) Cousin
Nithya is Mogan‘s Daughter.
Selvan is Rajan‘s Son and Rajan is Mogan‘s Brother
Selvan is Mogan‘s Nephew.
So, Nithya is Selvan‘s Cousin. Ans: (d)
Hence, Sunil is the cousin of Maruti.
Q2. A man said to a lady ―Your mother‘s husband‘s
sister is my aunt‖. How is the lady related to man? Q7. Showing a man on the stage , Rita said , " He is
(a) Sister (b) Mother the brother of the daughter of the wife of my husband.
(c) Daughter (d) Granddaughter How is the man on stage related to Rita ?
Ans: (a) Mother‘s husband Father (a) Son (b) Husband
Father‘s sister Aunt (c) Cousin (d) Nephew
So, man‘s and lady‘s aunt is same position both are
brother and sister.
Ans: (a)
Q3. Arun said, ―This girl is the wife of the grandson of
That man is son of Rita
my mother‖. Who is Arun to the girl ?
(a) Grandfather (b) Husband
Q8. A boy introduced a girl as the daughter of the son
(c) Father-in-law (d) Father
of the father of his uncle. How is the girl related to the
Ans: (c) Grandson of Arun‘s mother means either son or
boy ?
nephew of Arun. Therefore, Arun is the father -in-law of
(a) Aunt (b) Grand-daughter
that girl.
(c) Niece (d) Sister

Q4. Seema‘s younger brother Sohan is older than


Seeta. Sweta is younger than Deepti but elder than
Seema. Who is the eldest ?
(a) Seeta (b) Deepti
(c) Seema (d) Sweta

Page 249 of 516


https://telegram.me/aedahamlibra
(a) Mother in law
(c) Aunt
(b) Sister in law
(d) Sister

Ans: (c)
A is Aunt of E.
Ans: (c)
There is no option of cousin sister.
Q13. Chanda is the wife of Bharat. Mohan is the s on
of Chanda. Ashish is the brother of Bharat and father
Q9. A family consisted of a man, his wif e, his three
of Dhruv. How is Mohan related to Dhruv?
sons, their wives and three children in each son's
(a) Sister (b) Cousin
family. How many members are there in the family ?
(c) Brother (d) Mother
(a) 12 (b) 13
(c) 15 (d) 17
Ans: (d) A man + his wife = 1 + 1 = 2
His three sons + their wives = 3 + 3 = 6
Ans: (b)
Three children in each one‘s family = 3 × 3 = 9
Hence, Mohan is cousin of Dhruv.
Total members = 2 + 6 + 9 = 17
Q14. Introducing Rita, Monica said, "She is the only
Q10. Suket has three daughters and each daughter
daughter of m y father's only daughter." How is
has a brother. How many male members are there in
Monica related to Rita?
the family ?
(a) Aunt (b) Niece
(a) 4 (b) 2
(c) Cousin (d) Mother
(c) 3 (d) 1
Ans: (b) Suket has three daughters and each daughter has
a brother.

Ans: (d)
Monica is the mother of Reeta.

Hence, there are 2 male members in a family. Q15. X is sister of Y. Y is brother of Z. Z is husband
of P. O is father of Y. How is P related to O?
Q11. Introducing a man, a woman said "His mother is (a) Sister (b) Daughter
the only daughter of my father'. How is the man (c) Uncle (d) Daughter - in - law
related to the woman?
(a) Son (b) Father
(c) Brother (d) Uncle

Ans: (d)
So, P is daughter -in-law of O.

Q16. A's mother is sister of B and has a daughter C.


Ans: (a) How can A be related to B from among the following
Hence, man is son of the woman. ?
(a) Daughter (b) Father
Q12. If A is the mother of B and K, D is the husband (c) Niece (d) Uncle
of A. E is the son of D‘s brother. What is the relation
of A with E
Page 250 of 516
https://telegram.me/aedahamlibra
Q21. Introducing a boy Ankit said, "He is the son of
daughter of my grandfather's son". How is that boy
related to Ankit?
(a) Cousin (b) Brother
(c) Father – in – law (d) Nephew
Ans: (d) Nephew.
Ans: (c)
A is the Niece of B Q22. Pointing to a lady, Rohit said "She is the sister of
the daughter of my father's wife's son". How is the
Q17. If 'P 3 Q' means 'P is daughter of Q', 'P 5 Q' lady related to Rohit?
means 'P is father of Q', 'P 7 Q' means 'P is mother of (a) Daughter (b) Sister
Q' and 'P 9 Q' means ' P is sister of Q', then how is J (c) Niece (d) Daughter or Niece
related to K in J 3 L 9 N 3 0 5 K? Ans: (d)
(a) Mother (b) Wife
(c) Niece (d) Daughter
Ans: (c) J 3 L → J is daughter of L
L 9 N → L is sister of N
Direction And Distance
N 3 O → N is daughter of O
O 5 K → O is father of K.
Hence J is niece of K. Q1. A chi ld is looking for his father, he went 90
metres in the east before turning to his right. He went
Q18. P and Q are brothers, P is the father of S, R is 20 metres before turning to his right again to look for
the only son of Q and is married to U. How is U his father at his uncle's place 30 metres from this
related to S? point. His father was not there. From here he went
(a) Sister – in – law (b) Mother – in – law 100 metres to his north before meeting his father in a
(c) Sister (d) Mother street. How far did the son meet his father from the
starting point ?
(a) 80 m (b) 100 m
(c) 260 m (d) 140 m

Ans: (a)
Therefore, U is sister – in – law of S.

Q19. Punit said to lady, "The sister of your father's


wife is my aunt". How is the lady related to Punit? Ans: (b)
(a) Daughter (b) Grand Daughter ∴Required distance = AF
(c) Niece (d) Cousin sister

Ans: (d) The lady is the cousin sister of Punit.
√ √
Q20. Pointing to a lady, Diwakar said, "Her mother's
only grandson is my son". How is that lady related to Q2. K is a place which is located 2 km away in the
Diwakar? north-west direction from the capital P, R is another
(a) Aunty (b) Sister place that is located 2 km away in the south -west
(c) Mother (d) Wife direction from K. M is another place and that is
Ans: (d) According to blood relation analysis: A lady located 2 km away in north -west direction from R. T
related to Diwakar as a sister. is yet another place that is located 2 km away in the

Page 251 of 516


https://telegram.me/aedahamlibra
south-west direction from M. In which direction is T
located in relation to P?
(a) South-west (b) North-west
(c) West (d) North

Ans: (c) Ans: (d)


It is clear that T is located to the West of P. Person is in south west direction & 5 km from the starting
point.
Q3. Ganesh cycles towards South–West a distance of 8
m, then he moves towards East a distance of 20 m. Q6. Raghu starts from his house in his car and travels
From there he moves towards North –East a distance 8 km towards the North, then 6 km towards East then
of 8 m, then he moves towards West a distance of 6 m. 10 m towards his right, 4 km towards his left, 10 km
From there he moves towards North –East a distance towards North and finally 4 km towards his right. In
of 2 m. Then he moves towards West a distance of 4 m which
and then towards South – West 2 m and stops at that Directions is he now with reference to the starting
point. How far is be from the starting point? point?
(a) 12 m (b) 10 m (a) South (b) North East
(c) 8 m (d) 6 m (c) South East (d) North

Ans: (b)
AH = 20 – (4 + 6) = 10m

Q4. Vivek and Ashok start from a fixed point. Vivek Ans: (b)
moves 3 km north and turns right and then covers 4
km. Ashok moves 5 km west and turns right and Q7. Satish start from A and walks 2 km east upto B
walks 3 km. Now how far are they apart? and turns southwards and walks 1 km upto C. At C
(a) 10 km (b) 9 km he turns to east and walks 2 km upto D. He then turns
(c) 8 km (d) 6 km northwards and walks 4 km to E. How far is he from
Ans: (b) The direction diagram is as follows : his starting point ?
(a) 5 km (b) 6 km
(c) 3 km (d) 4 km

Here, C‘B = B‘A = 5 km


So, C‘B + BC = 5 + 4 = 9 km
Therefore, they are 9 km far apart each other.

Q5. A person walks 9 km to the South. From there he


walks 5 km to the North . After this he walks 3 km to Ans: (a)
the West. In which direction and how far is he now ∴ Required distance AE,
from the starting point? √
(a) 4 km South (b) 4 km North √
(c) 5 km North West (d) 5 km South West √ √
Page 252 of 516
https://telegram.me/aedahamlibra
Q8. Ram and Sham start walking in opposite
Q11. Seema walks 30 m North. Then she turns right
and walks 30 m then she turns rig ht and walks 55 m.
directions. Ram covers 6 kms and Sham 8 kms. Then Then she turns left and walks 20 m. Then she again
Ram turns right and walks 8 kms and Sham turns left turns left and walks 25m. How many metres away is
and walks 6 kms. How far each is from the starting she from her Original position?
point? (a) 45 m (b) 50 m
(a) 11 kms (b) 8 kms (c) 66 m (d) 55 m
(c) 9 kms (d) 10 kms Ans: (b)

Q12. A cyclist rides 40 kms to the east, turns north


and rides 20 kms. again turns left any rides 20 kms.
How far is he from the starting point ?
(a) 0 km (b) 10 kms
(c) 20 kms (d) 30 kms
Ans: (d)
OB √

√ √

Q9. Sherly starting from a fixed point goes 15 m


toward North and then after turning to his right he Ans: (d)
goes 15 m. Then he goes 10, 15 and 15 metres after
AD = √
turning to his left each time . How far is he from his
starting point?
Q13. Dinesh and Ramesh start together from a certain
(a) 15 metres (d) 5 metres
point in the opposite direction on motorcycles. The
(c) 10 metres (d) 20 metres
speed of Dinesh is 60 km per hour and Ramesh 44 km
Ans: (c)
per hour. What will be the distance between them
after 15 minutes ?
(a) 20 km. (b) 24 km
(c) 26 km (d) 30 km
Ans: (c) Relative speed of Dinesh and Ramesh's
motorcycles = (60 + 44) = 10 4 km/hr
Distance travelled by them = Relative speed × Time
covered,
∴ AF = AE – EF = 25 – 15 = 10 metres 104km/hr ( )

Q10. A man started from a place walked towards Q14. Four players P, Q, R and S are standing a play
North for 5 km then turned 90° to his right and field in such a way that Q is to East of P, R is to the
walked another 5 km. then he turned 45° to his right South of P and S is to the North of P. In which
and walked 2 km and turned 45° to his left. What is direction of Q is S Standing ?
his direction now? (a) North (b) South
(a) South (b) South East (c) North-West (d) South-East
(c) East (d) South East Ans: (c)

Q15. Ravi is walking in the East direction. After


covering a distance of one kilometer, he turns 45° left
and then 90° right. In which direction is he now ?
(a) North (b) West
Ans: (c) (c) North West (d) North East

Page 253 of 516


https://telegram.me/aedahamlibra
Therefore, today is Tuesday.
Day after tomorrow will be Thursday.
Thursday + 3 = Sunday

Q19. If the day after tomorrow is Sunday, what day


was tomorrow's day before yesterday?
Ans: (d) (a) Friday (b) Thursday
After walking towards 90° right, he is now in north east (c) Monday (d) Tuesday
direction. Ans: (b) The day after tomorrow is Sunday.
Therefore, today is Friday.
Q16. After going 80 m, from his house towards east, a The day on tomorrow‘s day before yesterday = Friday – 1
person turns left and goes 20 m then turns right and = Thursday
moves 100 m, then turns left and goes 60m, then turns
right and goes 120 m to reach the park. What is the Q20. Suresh is 7 ranks ahead of Ashok in the class of
distance between his house and the park? 39 students. If Ashok's rank is 17th from the last,
(a) 120 m (b) 20 m what is Suresh's rank from the start ?
(c) 100 m (d) 80 m (a) 16th (b) 23th
Ans: (*) (c) 24th (d) 15th
Ans: (a)

Q21. After 9'O clock at what time between 9 p.m. and


10 p.m. will the hour and minute hands, of a clock
point in opposite direction?
(a) 15 min. past 9 (b) 16 min. past 9
So, distance of house from park (c) min. past 9 (d) min. past 9
=
Ans: (c) At 9 O‘clock, the minute hand is 9 × 5 = 45
√ minute – spaces behind the hour hand. Therefore, the
=√ √ minute hand will have to gain 45 – 30 = 15 minute space
= 310.5m over the hour hand.
Note: None of the answers is correct.
Correct answer is 310.5 m Gain of 55 minute spaces equal 60 minutes.
∴ Gain of 15 minute spaces equals
Sequence & Ranking Test =
Therefore, hour and minute hands of a clock point in
opposite direction after 9 O‘clock at
Q17. In A Row Of Boys, Srinath Is 7th From The Left
And Venkat Is 12th From The Right. If They
Interchange Their Positions, Srinath Becomes 22nd minutes past 9.
From The Left. How Many Boys Are Th ere In The
Row ? Q22. In a row of girls, Kamala is 9th from the l eft and
(a) 19 (b) 31 Veena is 16th from the right. If they interchange their
(c) 33 (d) 34 positions, Kamla becomes 25th from the left. How
Ans: (c) many girls are there in the row?
(a) 34 (b) 36
Q18. If the day before yesterday was Sunday, what (c) 40 (d) 41
day will it be three days after the day after tomorrow Ans: (c)
?
(a) Sunday (b) Monday Q23. Suresh was born on 4th October 1999.
(c) Wednesday (d) Saturday Shashikanth was born 6 days before Suresh. The
Ans: (a) Day before yesterday was Sunday.

Page 254 of 516


https://telegram.me/aedahamlibra
Independence Day of that year fell on Sunday. Which
day was Shashikanth born?
(a) K
(c) I
(b) O
(d) M
(a) Tuesday (b) Wednesday Ans: (d)
(c) Monday (d) Sunday
Ans: (b) Shashikant was born on 29th September 1999. Q28. Among 5 boys, Vasant is taller than Manohar,
15th August, 1999 was Sunday. but not as tall as Raju. Jayant is taller than Dutta but
Days upto 29th September from 15 August shorter than Manohar. Who is the tallest in the
16 + 29 = 45 days = 6 weeks 3 odd days group?
Sunday + 3 = Wednesday. (a) Manohar (b) Vasant
(c) Jayant (d) Raju
Q24. In a row of trees, a tree is 7th from left end and Ans: (d) According to given condition; the correct order
14th from the right end. How many trees are there in is :
the row? Raju > Vasant > Manohar > Jayant > Dutta
(a) 18 (b) 19 ∴ Raju is the tallest among them.
(c) 20 (d) 21
Ans: (c) Total number of trees in the row = 14 + 7 – 1 = Q29. If the 5 th date of a month is Tuesday, what date
20 will be 3 days after the 3rd Friday in the month?
(a) 17 (b) 22
Q25. If John celebrated his victory day on Tuesday, (c) 19 (d) 18
5th January 1965, when will he celebrate his next Ans: (d) 5th date of a month is Tuesday.
victory day on the same day? Friday will be on = 5 + 3
(a) 5th January 1970 (b) 5th January 1971 = 8th of a month
th
(c) 5 January 1973 (d) 5th January 1974 1st Friday is on 1st of a month
Ans: (b) 5 January 1965 Tuesday 2nd Friday is on 8th of a month
5 January 1966 Wednesday 3rd Friday will be on 15th of a month
5 January 1967 Thursday 3 days after 15th = 15 + 3 = 18
5 January 1968 Friday
5 January 1969 Sunday Q30. Sohan ranks seventh from the top and twenty -
Since, 1968 is a leap year. sixth from the bottom in a class. How many students
5 January 1970 Monday are there in the class ?
5 January 1971 Tuesday (a) 33 (b) 34
(c) 31 (d) 32
Q26. Sita is elder than Swapna. Lavanya is elder th an Ans: (d) Clearly, number of students in the class
Swapna but younger than Sita. Suvarna is younger (26 + 7 – 1) = 32
than both Hari and Swapna, Swapna is elder than
Hari. Who is the youngest ? Q31. Roshan is taller than Hardik who is shorter than
(a) Sita (b) Lavanya Susheel. Niza is taller than Harry but shorter than
(c) Suvarna (d) Hari Hardik. Susheel is shorter than Roshan. Who is t he
Ans: (c) Sita > Swapna............................ (i) tallest ?
S Sw (a) Roshan (b) Susheel
S > Lavanya > Sw ........................... (ii) (c) Hardik (d) Harry
L Ans: (a) Roshan, Susheel > Hardik
Hari, Sw > Suvarna.......................... (iii) Hardik > Niza > Harry
H Su Roshan > Susheel
Sw > H .......................... (iv) Roshan > Susheel > Hardik > Niza > Harry
From all the statements : Therefore, Roshan is the tallest.
S > L > Sw > H > Su
Q32. G is fatter than H but not as fat as M. Q i s also
Q27. In the alphabetical order A, B, C, D, .... Z, which not as fat as M. Who is the most lean person in the
letter is 11 th to the left of the letter which is 4 th to the group?
right of T? (a) Q (b) H
Page 255 of 516
(c) G
Ans: (b) M > G > H
https://telegram.me/aedahamlibra
(d) M (a) 30
(c) 33
(b) 32
(d) 31
M>Q Ans: (d) Total number of students in the class = 45.
H is the most lean person in the group Neha‘s rank from first = 15th
So number of students from the last = 45 – (1 + 14) = 30
Q33. Priti scored more than Rahul. Yamuna scored as So, Neha‘s rank from the last is 31st.
much as Divya. Lokita scored less than Manju. R ahul
scored more than Yamuna. Manju scored less than Q38. If Ramya's rank is 22 nd out of 46 students. What
Divya. Who scored the lowest? is her rank from the last?
(a) Yamuna (b) Lokita (a) 29 (b) 32
(c) Rahul (d) Manju (c) 25 (d) 26
Ans: (b) Priti > Rahul Ans: (c) Total number of students in the class = 46
Rahul > Yamuna = Divya Ramya‘s rank from start = 22nd
Manju > Lokita So Ramya‘s rank from last = (46 – 22 + 1) = 25th
Divya > Manju
Now, Priti > Rahul > Yamuna = Divya > Manju > Lokita Q39. In a row of men, Manoj is 30 th from the right
th
Therefore, Lokita scored the lowest. and Kiran is 20 from the left. When they
interchange their position, Manoj becomes 35 th from
Q34. The digits are given as below: the right. What is the total number of men in the row?
562, 871, 438, 753 (a) 45 (b) 44
If the position of the first and the third digits of each (c) 54 (d) 34
of the numbers are interchanged, which of the Ans: (c)
following will be the sum of the first and the second
digits of the third highest number? Q40. Kathir is senior of Ganesh. Ganesh is senior of
(a) 9 (b) 7 Apparu. Apparu is junior of Raju. Raju is junior of
(c) 6 (d) 8 Ganesh. Who is the most senior?
Ans: (d) Arranging digits according to question. (a) Ganesh (b) Raju
265, 178, 834, 357 (c) Kathir (d) Apparu
Hence, third highest would be 265 and 8 be the sum of Ans: (c) Kathir > Ganesh > Raju > Apparu
first and second digits of the number. Kathir is the senior most

Q35. Veni is an year older than Smith. Smith is two Q41. In a row of students, if John , who is 16th from
years older than Salim. Raju is an year older than the left, and Johnson, who is 8th from the right,
Salim. Who is the youngest of all ? . interchange their positions, John becomes 33rd from
(a) Raju (b) Salim left. How many students are there in a row?
(c) Veni (d) Smith (a) 38 (b) 39
Ans: (b) Suppose the age of Salim is x years (c) 40 (d) 41
Age of Raju = x + 1 year Ans: (c) When John interchange his position, new
Age of Smith = x + 2 years Position will be 33 from left and 8 from Right
Age of Veni = x + 3 years Total = 33 + 8 – 1 = 40
Therefore, Salim is the youngest of all.
Q42. X is elder than Z, Y is younger than Z, Z is elder
Q36. In a line, Naresh is 17 th from the left & 22nd from than W. W is younger than X, who is the eldest ?
the right. How many students are there in the line ? (a) W (b) Z
(a) 40 (b) 38 (c) X (d) Y
(c) 39 (d) 37 Ans: (c) X > Z – (i)
Ans: (b) Naresh is 17th from left and 22nd from the right. Z > Y– (ii)
So, total number of students in the line = 17 + 22 – 1 = 38 Z > W– (iii)
X > W– (iv)
Q37. In a class of 45, Neha's rank is 15th from first, Order will be
what is her rank from the last ? X>Z>W>Y
Page 256 of 516
So, X is the eldest https://telegram.me/aedahamlibra
Ans: (a) Total number of students who have failed
= (40 + 1) – (13 + 17)
Q43. In a class the students are standing in a row. A (41 – 30) = 11
student is 11th from the right end and 9th from the
left end. How many students are standing in that row?
(a) 19 (b) 20
(c) 21 (d) 18
Number Puzzle
Ans: (a)

Q44. A is elder to B while C and D are elder to E, who Q1. The following equations follow a common
lies between A and C. If C is elder to B, who is the property. Find out the correct value to complete D :
youngest? A = 51 (714) 14 :
(a) A (b) C B = 61 (915) 15 :
(c) B (d) D C = 71 (1136) 16 :
Ans: (c) C and D > E > A > B D = 81 (?) 17
So, B is the youngest. (a) (1377) (b) (1378)
(c) (1356) (d) (1346)
Q45. There are 45 trees in a row. The lemon tree is Ans: (a) As, A = 51 × 14 = 714
20th from right end. What is the rank of lemon tree B = 61 × 15 = 915
from left end? C = 71 × 16 = 1136
(a) 26 (b) 24 ∴ D = 81 × 17 = 1377
(c) 25 (d) 27
Ans: (a) Required rank of lemon tree = (45 + 1) – 20 DIRECTIONS: In each of the following questions.
= 26. Select the missing number from the given responses.

Q46. Priyank ranks 6 th from the bottom and 28 th from Q2. Find the missing number from the given
the top in a class. How many students are there in the responses:
class?
(a) 31 (b) 32
(c) 33 (d) 34
Ans: (c) Priyank is 6th from the bottom 28th from top.
So, total number of students = 6 + 28 – 1 = 33

Q47. In a row of people Manu is 7 th from bottom end (a) 4 (b) 5


of row. Shrey is 10 ranks above Manu. If shrey is 8 th
(c) 3 (d) 6
from top, then how many people are there in this Ans: (d) 5 + 4 = 9 and 9 × 2 = 18
row? 6 + 3 = 9 and 9 × 3 = 27
(a) 25 (b) 26
12 + 4 = 16 and ?
(c) 24 (d) 23
= 96/16 = 6
Ans: (c) Manu's rank from bottom end = 7th
Shrey's rank from bottom end = (7 + 10) = 17th
DIRECTIONS: Select the missing number from the
Shrey's rank from top end = 8th
given responses :
∴ Total number of people = (17 + 8 – 1) = 24
Q3.
Q48. In a class, there are 40 students. Some of them
passed the examination and others failed. Raman's
rank among the student who have passed is 13 th from
top and 17 th from bottom. H ow many students have
failed?
(a) 11 (b) 10
(c) 9 (d) Cannot be determined (a) 60 (b) 68
Page 257 of 516
(c) 55 https://telegram.me/aedahamlibra
(d) 65

Ans: (d)
(a) 20 (b) 25
Q4. (c) 10 (d) 15
Ans: (d) The product of two numbers in a sector is equal
to the central number in the previous sector.
3 × 5 = 15
8 × 3 = 24
7 × 2 = 14
(a) 58 (b) 51
5 × 3 = 15
(c) 65 (d) 64
8 × 4 = 32
Ans: (c) Column wise
9×1=9
First Column,
9 × 2 = 18
(7) 2 + (4) 2 + (2) 2 = 49 + 16 + 4 = 69
7 × 4 = 28
Second Column,
(3) 2 + (9) 2 + (1) 2 = 9 + 81 + 1 = 91 Similarly, In third
Q7.
column,
(2) 2 + (6) 2 + (5) 2 = 4 + 36 + 25 = 65

DIRECTIONS: Select the missing number from the


given responses.

Q5.
(a) 21 (b) 20
(c) 23 (d) 22
Ans: (c) First Column
10 + 12 + 4 + 10 = 36
36/2 = 18
Second Column
(a) 343 (b) 512 11 + 12 + 12 + 5 = 40
(c) 729 (d) 1000 40/2 = 20
Third Column
15 + 8 + 10 + 13 = 46 = 46/2 = 23

DIRECTIONS: Select the missing number from the


given responses.
Ans: (b)
Q8.
Q6.

=?
(a) 14400 (b) 15600
(c) 23040 (d) 17400
Ans: (a) 2 × 3 × 5 × 4 = 120
120 × 120 = 14400

Page 258 of 516


Q9. https://telegram.me/aedahamlibra
5+8×2 5 + 16 = 21

Q12.

(a) 2 (b) 4
(a) 6 (b) 2 (c) 3 (d) 5
(c) 3 (d) 4 Ans: (c) As, 3 × 7 = 21, 11 × 7 = 77
Ans: (b) The sum of numbers of the left vertically half 4 × 9 = 36, 12 × 9 = 108
part is equal to the sum of the numbers on the right. Therefore,14 × 8 = 112
1 + 7 + 9 + 5 + 1 + 9 + 6 + 4 = 42 ? × 8 = 24
Therefore, ?=3
? + 7 + 8 + 4 + 2 + 8 + 8 + 3 = 42
? = 42 – 40 = 2 DIRECTIONS: Select the missing number from the
given responses.
DIRECTIONS: In the following questions, select the
missing number from the given responses. Q13.

Q10.

(a) 9 (b) – 4
(c) 4 (d) – 8
(a) 5 (b) 2 Ans: (b) 12 × 16 = 192 + 5 = 197
(c) 3 (d) 4 16 × 16 = 256 + 7 = 263
Ans: (b) First column (7 + 2) × 4 = 36 18 × 20 = 360-4 = 356
Second Column (6 + 8) × 3 = 42
Third Column (9 + 4) × ? = 26 Q14.
13 × ? = 26

DIRECTIONS: Find the missing number.


(a) 66 (b) 87
Q11. (c) 78 (d) 76
Ans: (b) 8 × 8 × 88 = 5632
7 × 7 × 77 = 3773 Similarly, 6 × 6 × ? = 3132
∴?=

DIRECTIONS: Select the missing number/letter from


(a) 42 (b) 21 the given responses.
(c) 11 (d) 18
Ans: (b) First Column Q15. 12 (288) 2; 5 (50) 2, 19 (?) 2
3 + 6 × 9 = 3 + 54 = 57 (a) 722 (b) 324
Second Column (c) 776 (d) 684
4 + 7 × 1 = 4 + 7 = 11 Ans: (a) (12 × 12) × 2 = 144 × 2 = 288
Third Column (5 × 5) × 2 = 50 Similarly, (19 × 19) × 2 = 361 × 2 = 722
Page 259 of 516
https://telegram.me/aedahamlibra
DIRECTIONS: In the following questions, select the
missing number from the given responses.
Q19.
Q16. Select the missing number from the given
responses.

(a) 135 (b) 235


(c) 347 (d) 407
Ans: (d) First Column
24 × 2 + 5 = 48 + 5 = 53
Second Column
(a) 56 (b) 49 51 × 4 + 7 = 204 + 7 = 211
(c) 45 (d) 64 Third Column
Ans: (d) 23 = 8; 33 = 27 67 × 6 + 5 = 402 + 5 = 407
∴ 43 = 64
DIRECTIONS: Select the missing number from the
Q17. given responses.

Q20. Find the missing number.

(a) 230 (b) 140


(c) 120 (d) 410
Ans: (b) First Column (a) 37 (b) 45
√ √ √ (c) 47 (d) 57
9 × 2 × 6 = 108 Ans: (c) First Figure
Second Column 3 + 5 + 2 + 4 = 14
√ √ √ 14 + 13 = 27
8 × 3 × 4 = 96 Second Figure
Third column √ √ √ 6 + 2 + 3 + 5 = 16
4 × 7 × 5 = 140 16 + 21 = 37
Third Figure
Q18. 2 + 2 + 9 + 5 = 18
18 + 29 = 47

Q21.

(a) 5 (b) 4
(c) 6 (d) 7
Ans: (d) First number in each ro w is the product of the
second and third numbers. (a) 122 (b) 128
First Row (c) 124 (d) 126
25 = 5 × 5 Ans: (c) First Column (3 × 4 × 5) – 2 = 58
Second Row Second Column (5 × 6 × 2) – 2 = 58
30 = 5 × 6 Third Column (8 × 4 × 2) – 2 = 62
Third Row Forth Column (7 × 6 × 3) – 2 = 124
35 = ? × 5
∴?= Q22.
Page 260 of 516
https://telegram.me/aedahamlibra
54 × 2 + 2 = 110
So, missing number = 110 × 2 + 2 = 222

DIRECTIONS: In questions below, select the missing


(a) 20 (b) 50
number from the given responses
(c) 54 (d) 30

Ans: (b) Q25. Find the missing number from the given
= Second number in each column. responses.
First Column
√ √
Second Column
√ √
Third column

2 × ? = 100 (a) 12 (b) 10
(c) 9 (d) 8

Ans: (a)

Q23.

Q26.

(a) 330 (b) 336


(c) 428 (d) 420
Ans: (b) 6 + 18 = 24 (a) 17 (b) 23
24 + 36 = 60 (c) 47 (d) 73
60 + 60 = 120 Ans: (b) 7 + 22 + 5 = 16
120 + 90 = 210 9 + 42 + 7 = 32
210 + 126 = 336 8 + 32 + 6 = 23

DIRECTIONS: Select the missing number from the DIRECTIONS: S elect the missing number from the
given responses. given responses.

Q24. Select the missing number from the given Q27.


responses:

(a) 1 (b) 8
(c) 6 (d) 16
Ans: (c) 3 + 9 – 5 = 7, 2 + 8 – 6 = 4
4+7–5=6
(a) 132 (b) 122
(c) 222 (d) 212 Q28.
Ans: (c) Moving clockwise, the terms are :
5 × 2 + 2 = 12
12 × 2 + 2 = 26
26 × 2 + 2 = 54
Page 261 of 516
https://telegram.me/aedahamlibra
Q31.

(a) 444 (b) 515


(c) 343 (d) 373 (a) 12 (b) 17
Ans: (c) 81 × 9 = 729, 64 × 8 = 512 (c) 18 (d) 16
49 × 7 = 343 Ans: (c) (36) – (14 + 12 + 7) = 3
(54) – (9 + 11 + 16) = 18
DIRECTIONS: Select the missing number from the
given responses. Q32.

Q29.

(a) 30 (b) 11
(c) 0 (d) 1
Ans: (c) 3 × 4 × 2 = 24
2 × –2 × –1 = 4
6×0×5=0

Q33.

(a) 16 (b) 12
(c) 90 (d) 48
(a) 9 (b) 40
Ans: (c) 1 × 3 × 4 × 5 = 60
(c) 7 (d) 36
6 × 7 × 1 × 2 = 84
Ans: (b) The pattern is
therefore, 3 × 10 × 3 × 1 = 90
(18 + 17) + (9 – 6) = 38
(11 + 11) + (19 – 9) = 32
Q34.
(15 + 6) + (26 – 3) = 44
(12 + 8) + (39 –19) = 40

Q30.

(a) 1332 (b) 1321


(c) 1231 (d) 1331

(a) 97 (b) 907


(c) 1097 (d) 9107
Ans: (c) 8 × 2 + 1 = 17
8 × 17 + 1 = 137
8 × 137 + 1 = 1097
Ans: (d)
Page 262 of 516
https://telegram.me/aedahamlibra
DIRECTIONS: In questions below, Select the missing
number from the given responses.
(a) 79 (b) 73
Q35. (c) 75 (d) 77
Ans: (c) 9 × 5 = 458 × 4 = 32
45 + 32 = 77
9 × 6 = 54
3 × 7 = 2154 + 21 = 75

(a) 535 (b) 451 Q40.


(c) 702 (d) 154
Ans: (d) Figure follows the Rule
(0 + 6 + 4 + 3 + 1 + 5) × 7 = 19 × 7 = 133
(8 + 2 + 5 + 3 + 6 + 4) × 7 = 28 × 7 = 196
(2 + 1 + 5 + 7 + 3 + 4) × 7 = 22 × 7 = 154

Q36.
(a) 58 (b) 45
(c) 54 (d) 42

(a) 24 (b) 12
(c) 18 (d) 19
Ans: (d) 3 + 4 + 3 = 10, 4 + 6 + 4 = 14
8 + 6 + 6 = 20, 8 + 3 +8 = 19
Ans: (c)
Q37.
Q41.

(a) 20 (b) 5
(c) 4 (d) 21
Ans: (b) 225 = (15) 2 (8 + 7) 2
49 = (7) 2 (3 + 4) 2 (a) 60 (b) 65
121 = (11) 2 (6 + 5) 2 (c) 68 (d) 55
So missing number is 5.

Q38.

Ans: (b)
(a) 45 (b) 80
(c) 30 (d) 32 Q42. Select the missing number from the given
Ans: (a) 4-, 4×2 = 8, 8×2 = 16, 16×2 = 32 responses:
5-, 5×3 = 15, 15×3 = 45, 45×3 = 135
6-, 6×4 = 24, 24×4 = 96, 96×4 = 384

Q39.
Page 263 of 516
https://telegram.me/aedahamlibra
(a) 8 (b) 31 (a) 75 (b) 45
(c) 32 (d) 36 (c) 20 (d) 30

Ans: (a) Ans: (d)


5 × 4 + 6 = 26 (7 – 5) × 5 = 10
7 × 4 + 4 = 32 (16 – 8) × 5 = 40
9 × 4 + x = 44 (15 – 9) × 5 = 30
x = 44 – 36
x=8 Q46.

Q43. Select the missing number from the given


alternatives:

(a) 11 (b) 12
(c) 22 (d) 33
Ans: (c) = 198, 198 ÷ 3 = 66, 66 ÷ 3 = 22

(a) 9 (b) 8 Q47.


(c) 7 (d) 6
Ans: (b) Here R1 × R2 + R3  R4
: :8 × x + 6 = 70
8 x = 64 x = 8
(a) 15 (b) 20
Q44. Select the missing numbers from the given
(c) 11 (d) 10
responses
Ans: (c) 7 + 9 – 6 = 10
5 + 8 – 3 = 10
9 + 6 – 4 = 11

DIRECTIONS: In the following questions, select the


missing number from the given responses.
(a) 36 (b) 46
(c) 40 (d) 26
Q48.
Ans: (a) Each coloumn sum upto 99
∴99 – (35 + 28) = 36

DIRECTIONS: In the questions, select the missing


number from the given responses.

Q45. Find the missing number from the given


alternatives
(a) 20 (b) 15
(c) 40 (d) 10
Ans: (c)

Page 264 of 516


https://telegram.me/aedahamlibra
Q49. In the following question, select the number
which can be placed at the sign of question non
marking the given alternatives.

(a) 35 (b) 42
(c) 45 (d) 95
(a) 43 (b) 49 Ans: (a) The pattern is :
(c) 59 (d) 71 As,
Ans: (d) As, 3 × 10 × 6 + 6 = 186
3 × 1 × 5 × 2 + 1 = 31 and 9 × 5 × 3 + 3 = 138
4 × 6 × 3 × 2 + 1 = 145 Similarly, 2 × 7 × 5 × 1 + 1 = 71 5 × 7 × 1 + 1 = 36 Similarly, 3 × 2 × 5 + 5 = 35.

Q50. In the following question, select the number Q53. In the following question, select the missing
which can be placed the sign of question mark (?) number from the given series.
from the given alternatives.

(a) 37 (b) 47
(c) 48 (d) 25
(a) 14 (b) 15 Ans: (b)
(c) 16 (d) 18 √ √ √
Ans: (b) Here,
√ √ √
7 × 2 × 8 + 3 = 115
So, Answer is 47.
6 × 5 × 9 + 3 = 273
∴3 × 1 × 4 + 3 = 15
Q54. In the following question, select the number
So, answer is 15.
which can be placed at the sign of question mark (?)
from the given alternatives.
Q51. In the following question, select the number
which can be at the sign of question mark (?) from the
given alternatives.

(a) 38 (b) 40
(c) 42 (d) 44
Ans: (d) The pattern is as follows :
(a) 53 (b) 56
(7 × 6) + (6 × 5) = 42 + 30 = 72
(c) 59 (d) 66
(3 × 11) + (11 × 1) = 33 + 11 = 44
Ans: (b) The pattern is :
(2 × 5) + (5 × 8) = 10 + 40 = 50
1512 ÷ 3 = 504
504 ÷ 3 = 168
Q55. In the following question, select the number
168 ÷ 3 = 56.
which can be placed at the sign of question mark (?)
So, answer is 56.
from the given alternatives.
Q52. In the fol lowing question, select the number
which can be placed at the sign of question mark (?)
from the given alternatives.
Page 265 of 516
https://telegram.me/aedahamlibra
11 + 12 = 23 and 11 × 12 = 132

Venn Diagram
(a) 9 (b) 10
(c) 11 (d) 12
Ans: (c) The pattern is as follows : Q1. Find out which of the diagrams given in the
(1 + 8 + 9 + 4) = 22 alternatives correctly represents the relationship
(7 + 6 + 2 + 7) = 22 stated in the question.
(2 + 4 + 5 + ?) = 22 Sharks, Whales, Turtles
? + 11 = 22
? = 22 – 11 = 11

Q56. In the following question, select the letter which (a) (b)
can be placed at the sign of question mark (?) from
the given alternatives.

(c) (d)
Ans: (c)

Q2. Which one of the following diagrams represents


(a) X (b) Y the correct relationship among 'Judge', 'Thief' and
(c) Z (d) A 'Criminal'?
Ans: (c)

Q57.
(a) (b)

(a) 5 (b) 6
(c) 7 (d) 8 (c) (d)
Ans: (a) According to question, Ans: (c)
As,
(5 + 7 + 6) × 2 = 36 Q3. Which number space indicated Indian teachers
(9 + 11 + 8) × 2 = 56 Similarly, (10 + ? + 17) × 2 = 64 who are also advocates?
(10 + ? + 17) 64/2 = 32
27 + ? = 32
? = 32 – 27 = 5

Q58.

(a) 2 (b) 3 (c) 4


(d) 6
Ans: (b)
(a) 10 (b) 12
(c) 14 (d) 16 Q4. Which of the answer figure indicates the best
Ans: (b) The pattern is : relationship between milk, goat, cow, hen ?
5 + 6 = 11 and 5 × 6 = 30 Answer figures :
8 + 9 = 17 and 8 × 9 = 72
Page 266 of 516
https://telegram.me/aedahamlibra
(a) (b)

(c) (d)
Ans: (c)

Q5. Identify the figure which best represents the


relationship among Tree, Plant, and House.
In the above figure, the circle stands for employed, the
Answer figures :
square s tands for social worker, the triangle stands
for illiterate and the rectangle stands for truthful.
(a) (b) Employed, truthful and illiterate social workers are
indicated by which region?
(a) 5 (b) 4
(c) (d) (c) 2 (d) 1
Ans: (c) Ans: (d) Employed, truthful and illiterate social w orkers
would be indicated by the region common to all the four
Q6. In the given diagram, Circle represents strong geometrical figures. Such region is marked '1‘.
men, Square represents short men and Triangle
represents military officers. Which region represents Q9.
military officers who are short but not strong?

In the above diagram, parallelogram represents


women, triangle represent the sub -inspectors of police
and circle represents the graduates . Which numbered
area represents women graduate sub -inspectors of
police?
(a) 2 (b) 3 (a) 5 (b) 3
(c) 4 (d) 1 (c) 8 (d) 13
Ans: (a) Ans: (b) Women graduate sub –inspectors of police can
be represented by the region common to all the three
Q7. Which figure represents the relationship among geometrical figures. Such region is marked '3'.
Sun, Moon, Molecule ?
Q10. Which one of the following diagrams best depicts
the relationship among Earth, Sea, Sun?
(a) (b)

(c) (d) (a) (b)


Ans: (d)

Q8. (c) (d)


Ans: (c)

Q11. In the given diagram, circle represents


professionals, square represents dancers, triangle
Page 267 of 516
represents musicians and rectahttps://telegram.me/aedahamlibra
ngle represents
Europeans. Different regions in the diagram are
Ans: (c)

numbered 1 to 11. Who among the following is neither Q15. The diagram represents Teachers, Singers and
a dancer nor a musician but is professional and not a Players. Study the diagram and find out how many
European? teachers are also singers.

(a) 11 (b) 1
(c) 10 (d) 8
(a) 4 (b) 5
Ans: (c) The required region should be prese nt only in
(c) 9 (d) 13
circle. Such region is marked ‗10‘.
Ans: (c)

Q12. Which one of the following diagrams represents


Q16. The figure represents three classes of youth in a
the correct relationship among day, week, year?
village. How many educated youth are poor?

(a) (b)

(c) (d)
Ans: (a)

Q13. In the given figure, circles represent students


studying three diff erent subjects. How many students
study all the three subjects?

(a) 14 (b) 9
(c) 6 (d) 19
Ans: (a)

Q17. Identify the diagram that best represents the


relationship among the classes given below :
(a) 2 (b) 3 Doctors, Engineers, Lawyers
(c) 4 (d) 1
Ans: (b)

Q14. Identify the diagram that best represents the


relationship among the classes given below :
Liquids, Milk, River water

Ans: (d)

(a) (b) Q18. In a group of persons, 11 persons speak


Kannada, 20 persons speak Tamil and 11 persons
speak Telugu. In that group, if two persons speak two
languages and one person speak all the languages,
(c) (d) then how many persons are there in thegroup ?

Page 268 of 516


https://telegram.me/aedahamlibra
DIRECTIONS: Identify the diagram that best
represents the relationship among classes given below
:

Q22.

(a) 40 (b) 41
(c) 42 (d) 43
Ans: (c)

DIRECTIONS: In Question Nos. 34 and 35 which one In the fig.


of the following diagrams represents the correct Q represents all quadrilaterals
relationship among: K represents all Kites
R represents all Rhombus
Q19. In the following figure, the boys who are P represents all Parallelogram
cricketer and sober are indicated by which number ? The statement 'Rhombus is also a Kite' can be
described as
(a) P and K is nothing but R (b) P or K is nothing but R
(c) P and R is nothing but K (d) P or R is nothing but K
Ans: (a)

Q23. Christians, Catholics, Pope

(a) 6 (b) 5 (a) (b)


(c) 4 (d) 2
Ans: (d)

Q20. Lion, Fox and Carnivorous


(c) (d)
Ans: (b)

(a) (b) Q24. Identify the diagram that best represents the
relationship among classes given below:
Universe,Stars,Sun
(c) (d)
Ans: (c)

Q21.
(a) (b)

Which number indicates doctors who are not married (c) (d)
? Ans: (b)
(a) 6 (b) 4
(c) 2 (d) 1 Q25. Identify the diagram that best represents the
Ans: (d) relationship among classes given below
Delhi, Sri Lanka, Asia

Page 269 of 516


https://telegram.me/aedahamlibra
Ans: (b)

Q29. In the given figure, how many are musical toys?

(a) (b)

(c) (d)
Ans: (b) (a) 53 (b) 61
(c) 42 (d) 45
Q26. If the number indicates the number of persons,
Ans: (c)
then how many youth graduates are there ?
Q30. Identify the diagram that best represents the
relationship among the given classes.
Earth, Saturn, Planet, Star

(a) (b)
(a) 20 (b) 30
(c) 40 (d) 50
Ans: (a)
(c) (d)
Q27. In the given figure, how many pens are blue? Ans: (c)

Q31. Choose the correct Venn diagram which b est


illustrates the relationship among Hockey, Cricket,
Games.

(a) (b)

(a) 23 (b) 19
(c) 12 (d) 15
Ans: (a) (c) (d)
Ans: (b)
Q28. In the given figure, how many people study 2
subjects? Q32. In the given figure, which letter represents
carnivorous plants which are not green?

(a) 11 (b) 23 (a) d (b) g


(c) 12 (d) 40 (c) e (d) f
Page 270 of 516
Ans: (a) https://telegram.me/aedahamlibra
Q1. In a classroom, there are 5 rows, and 5 children
A, B, C, D and E are seated one behind the other in 5
Q33. In the given figure, ho w many yellow birds are seperate rows as follows :
there? A is sitting behind C, but in front of B.
C is sitting behind E, D is sitting in front of E.
The order in which they are sitting from the first row
to the last is
(a) DECAB (b) BACED
(c) ACBDE (d) ABEDC
Ans: (a) 1st Row D
2nd Row E
(a) 25 (b) 32 3rd Row C
(c) 18 (d) 20 4th Row A
Ans: (b) 5th Row B

Q34. In the given figure, How many water are either Q2. A group of friends are sitting in an arrangement
tap or shower? one each at the corner of an octagon. All are facing
the centre. Mahima is sitting diagonally opposite
Rama, who is on Sushma's right. Ravi is next to
Sushma and opposite Girdhar, who is on Chandra's
left. Savitri is not on mahima's right but opposite
Shalini. Who is on Shalini's right?
(a) Ravi (b) Mahima
(c) Girdhar (d) Rama
Ans: (a) Ravi is to the right of Shalini.

Q3. Five boys A, B, C, D. E are sitting in a park in a


(a) 168 (b) 111 circle. A is facing South -West, D is facing South -East,
(c) 125 (d) 108 B and E are right opposite A and D respectively and C
Ans: (b) is equidistant between D and B. Which direction is C
facing'?
Q35. In the given figure, how many players are quick (a) West (b) South
and fat? (c) North (d) East

Ans: (d)
So, C is facing towards East.

Q4. Four students ABCD are sitting one each of the


(a) 31 (b) 30 four corners of a square all facing the centre of the
(c) 32 (d) 33 square. The student E sitting at the centre is facing
Ans: (b) only C and the student A is sitting facing the back of
E. If D is sitting on the right of E, where B will be
sitting to E?
Sitting Arrangement (a) B is sitting on the left of E (b) B is to the back of E
(c) A is facing B and E (d) B is on the right of E

Page 271 of 516


https://telegram.me/aedahamlibra
(c) J
Ans: (b)
(d) L

Q10. Five policemen are standing in a row facing


south. Shekhar is to the immediate right of Dhanush.
Ans: (a) Bala is between Basha and Dhanush. David is at the
extreme right end of the row. Who is standing in the
Q5. There are five buses M, N, O, P, Q in a row on a middle of the row?
road. Bus M is standing at the front and Q is standing (a) Bala (b) Basha
at the back end. Bus N stands between M and O. Bus (c) Shekhar (d) Dhanush
P stands between O and Q. Which bus is in the middle Ans: (d) Standing arrangement : (facing south)
of the five?
(a) M (b) P
(c) N (d) Q
Ans: (*)
Hence, Dhanush is standing at the middle of the row.

Bus O is in the middle of the five. Q11. Seven persons A, B, C, D, E, F and G are
standing in a straight line.
Q6. Five coaches P, L, R, M, O are in a row. R is to D is to the right of G.
the right of M and left of P. L is to the right of P and C is between A and B.
left of O. Which coach is in the middle? E is between F and D.
(a) P (b) L There are three persons between G and B. Who is on
(c) R (d) O the extreme left?
(a) A (b) B
Ans: (a)
(c) D (d) G
Hence, P coach is in the middle of the five coaches.
Ans: (d) Standing Arrangement:

Q7. Five boys A, B, C, D and E are standing in a row.


D is on the right of E. B is on the left of E, but on the
right of A. D is on the left of C, who is standing on the
extreme right. Who is standing in the middle ?
(a) D (b) E G is standing on the extreme left.
(b) B (d) C
Q12. A,B,C,D,E,F are sitting on the round table with
equal distances. F is sitting opposite to E and between
Ans: (b)
A and D. C is sitting right side of E and opposite to A.
Who are the neighbours of A?
Q8. (i) A and B can speak Tamil and Malayalam.
(a) F and D (b) E and F
(ii) C and D can speak English and Hindi.
(c) E and C (d) B and F
(iii) B and D can speak Malayalam and Hindi.
(iv) A and C can speak Tamil and English.
One who speaks English, Hindi and Malayalam is
(a) A (b) B
(c) C (d) D
Ans: (d)
Ans: (d)
D can speak English, Hindi, and Malayalam.
B and F are neighours of A

Q9. Six persons are sitting in a circle. 'J' is between


Q13. 4 friends ABCD are sitting in a coffee shop. A
'N' and 'O'; 'N' is opposite 'M'; and 'L' is not in either
and B are sitting face to face. D is not sitting next to A
of the neighbouring seats of 'N'. Who is opposite to
but can see the facial expression of A clearly. B is
'K'?
(a) M (b) O
Page 272 of 516
sitting together?
https://telegram.me/aedahamlibra
talking to C who is sitting opposite to him. Who are Assumptions :
I.People vote to make politicians rich.
(a) D & C (b) A & B II.Politicians become rich by their virtue.
(c) A & C (d) A & D (a) Only I is implicit (b) Only II is implicit
Ans: (c) Sitting Arrangement: (c) Both I and II are implicit (d) Both I and II are not
implicit
Ans: (d) The statement implies that politicians win
elections by the votes of people. Therefore, neither of the
assumptions is implicit in the statement.

Q2. A statement is given followed by two assumptions,


So, the correct answer is (c) .
(a) and (b) . You have to consider
the statement to be true, even if it seems to be at
Q14. Five boys A, B, C, D and E are standing in a line.
variance from commonly known facts. You are to
A is taller than E but shorter than D. B is shorter than
decide which of the given assumptions can definitely
E and C is the tallest. Who is in the middle?
be dr awn from the given statement. Indicate your
(a) A (b) C
answer.
(c) D (d) E
Statement: Theoretical education does not bring in
Ans: (a) C > D > A > E > B
economic advancement and it lends to a steady loss of
So, A is in the middle.
confidence and money in the country.
Assumptions :
Q15. A, B, C, D and E are sitting around a ci rcular
(a) There is close relationship between development of
table. A don‘t have B and E as his neighbours. E is not
confidence and economic development
sitting with D. When seen in clockwise direction,
(b) Theoretical education makes priceless
which of the order of sitting is currect?
contribution for development of confidence.
(a) ABDEC (b) ADBEC
(a) Only 1 is implicit (b) Only 2 is implicit
(c) DBCEA (d) BCEDA
(c) Both 1 and 2 are implicit (d) Both 1 and 2 are not
implicit
Ans: (d) Neither (a) nor (b) is implicit in the
statement. The statement does not indicate that
confidence and economic development are related.

Q3. Two statements are given followed by four


Ans: (b) conclusions, I, II, III and IV. You have to consider the
So, ADBEC of order of sitting is correct. statements to be tru e, even if they seem to be at
variance from commonly known facts. You are to
decide which of the given conclusions can definitely be
Arguments & Syllogism drawn from the given statements. Indicate your
answer.
Statements :
(A) No cow is a chair
DIRECTIONS: In the following question, two (B) All chairs are tables.
statements P and Q are given followed by four Conclusions :
conclulions I, II, III and IV. You have to consider the I.Some tables are chairs.
two statements to be true even if they seem to be at II.Some tables are cows
variance from commonly known facts. You have to III.Some chairs are cows
decide which of the given conclusions, if any, follow IV.No table is a cow
the given statements. (a) Either II or III follow (b) Either II or IV follow
(c) Only I follows (d) All conclusions follow
Q1. Statements: Politicians become rich by the votes Ans: (*)
of the people.
Page 273 of 516
https://telegram.me/aedahamlibra
II. Some women are aged.
(a) Both conclusion I and II follow
(b) Only conclusion I follows
(c) Only conclusio n II follows
(d) Neither conclusion I nor II follows

Ans: (c)
So, Conclusion - I and either II or III follow. Case I : False
Case II : True
Q4. Statements :
1.Temple is a place of worship. Q7. Statement :
2.Church is also a place of worship. To keep myself updated, I always listen to 9 O‘clock
Conclusions : news on radio.
I.Hindus and Christians use the same place for Conclusions:
worship. I.The persons does not read newspaper.
II.All churches are temples. II.Recent news is available only on radio.
(a) Neither conclusion I nor II follows (a) Only conclusion I is implicit
(b) Both conclusion I nor II follows (b) Only conclusion II is implicit
(c) Only conclusion I follows (d) Only conclusion II (c) Neither conclusion I nor II is implicit
follows (d) Both conclusions I and II are implicit
Ans: (a) Temple and Church are places of worship. It Ans: (c) Neither Conclusion I nor Conclusion II follows.
does not imply that Hindus and Christians use the same The person always list ens to 9 O‘clock news on radio to
place for worship. Church is different temple. Therefore, keep himself updated. This does not imply that he does
neither Conclusion I nor II follows. not read newspaper. The use of term ‗only‘ in the
Conclusion II makes it unsuitable.
Q5. Statement :
The human organism grows and develops through Q8. Statements :
stimulation and action. (a) Due to contamination of water, large number of
Conclusions : people were admitted to hospital.
I.Inert human organism cannot grow and develop. (b) The symptoms were of Typhoid.
II.Human organisms do not react to stimulation and Conclusions :
action. I.Contamination of water may lead to Typhoid.
(a) Neither concl usion I nor II follows II.Typhoid is a contagious disease.
(b) Both conclusion I nor II follows (a) Only conclusion I is true (b) Only conclusion II is
(c) Only conclusion I follows (d) Only conclusion II true
follows (c) Both conclusions I and II are true
Ans: (a) Growth and development of human organism is (d) Both conclusions I and II are false
a continuous process. Some changes take place in human Ans: (a)
body now and then. Theref ore, neither Conclusion I nor
II follows. DIRECTIONS: Two statements are given followed by
two conclusions I and II. You have to consider the
Q6. Statement: statements to be true even if they seem to be at
I. All teachers are aged. variance from commonly known facts. You have to
II.Some women are teachers. decide whic h of the given conclusions, if any, follow
Conclusion: from the given statements.
I. All aged are women

Page 274 of 516


Q9. Statements : https://telegram.me/aedahamlibra
(a) 60% of the government employees went on strike.
You have to decide which of the given conclusions, if
any, follow from the given statement.
(b) Mr. Gopal is a government employee.
Conclusions : Q12. Statements:
I.Mr. Gopal went on strike. No children are voters.
II. Mr. Gopal did not participate in the strike. Conclusions:
(a) Only conclusion I follows (b) Only conclusion II (I) No adults are voters.
follows (II) No voters are children.
(c) Both conclusions I and II follow (a) Only conclusion (II) follows
(d) Either conclusion I and II follows (b) Both conclusions (I) and (II) follow
Ans: (d) (c) Neither conclusion (I) nor (II) follows
(d) Only conclusion (I) follows
Q10. Statements:
(a) Best performance in Olympics fetches a gold Ans: (a)
medal. Conclusion :
(b) Player ‗X‘ got gold medal but later was found to I : False
be using a prohibited drug. II : True
Conclusions:
(I) ‗X‘ should be allowed to keep the to gold medal. Q13. Statements:
(II) Gold medal should be withdrawn and given to the 1.Education is a process of lighting.
next person. 2.Mind requires light to enlighten the core of cognitive
(a) Only conclusion (II) follows aspect.
(b) Neither conclusion (I) nor (II) follows Conclusions :
(c) Both conclusions (I) and (II) follow I.Education is a light which removes the darkness of
(d) Only conclusion (I) follows Hint: (a) mind.
II.Education is a static process for mind.
Q11. Statements: (a) Both conclusions I and II follow
(a) All books are novels. (b) Neither conclusion I nor II follows
(b) Some novels are poems. (c) Only conclusion I follows (d) Only conclusion II
Conclusions: follows
(I) Some books are poems. Ans: (c) Clearly, I follows directly from the given
(II) Some poems are novels. statement. However, II is slightly different to the given
(a) Only conclus ion (II) follows statements and so does not follow.
(b) Neither conclusion (I) nor (II) follows
(c) Both conclusions (I) and (II) follow Q14. Statement: Songs always have singers to sing
(d) Only conclusion (I) follows them. Conclusions:
I.Singers make a song.
II.There is no un-sung song.
(a) Only conclusion II follows
(b) Both conclusions I and II follow
(c) Neither conclusion I nor II follows
(d) Only conclusion I follows
Ans: (a) Ans: (d) Any written piece is recognised as song when it
I.False is sung by a singer. Therefore, only Conclusion 1
II.True follows.

DIRECTIONS: In the following questions one DIRECTIONS: In questions one/two statement (s) are
statement is given followed by two conclusions. Y ou given, followed by two/three conclusions I, II and III.
have to consider the statement to be true even if it You have to consider th e statement (s) to be true even
seems to be at variance from commonly known facts. if they seem, to be at variance from commonly known
facts. You have to decide which of the given
Page 275 of 516
.
https://telegram.me/aedahamlibra
conclusions, if any, follow from the given statement (s) DIRECTIONS: In question two/four statements are
given followed by two/four conclusions I, II, III and
IV. You have to consider the statements to be true
Q15. Statements : even it they seem to be at variance from commonly
I.Some cats are dogs. known facts. You have to decide which of the given
II.No dog is a toy. conclusions, if any, follow from the given statements.
Conclusions :
I.Some dogs are cats. Q18. Statement: (I) All cities are towns.
II.Some toys are cats. (II) Some cities are villages.
III.Some cats are not toys. Conclusions: (I) All villages are towns
IV:All toys are cats. (II) No village is a town
(a) Only Conclusions I and III follow (III) Some villages are towns.
(b) Only Conclusions II and III follow (a) Only conclusion (III) follows
(c) Only Conclusions I and II follow (b) Only conclusion (I) follows
(d) Only Conclusion I follows (c) Only conclusion (II) follows
(d) None of these

Ans: (a)
OR
Ans: (a)
Conclusion III follows.

Q19. Statements:
I. All apples are bananas.
Conclusion II can follow, but it has not been given in
II.All bananas are sweet.
options. Hence only conclusions I and III follow.
Conclusions:
I.Some apples are sweet.
Q16. Statement :
II.Some bananas are apples.
To be happy in life, you should solve your problems
(a) Conclusion I follows. (b) Conclusion II follows.
than try to avoid them.
(c) Either conclusion I or II follows.
Conclusion :
(d) Both conclusions I and II follow.
I.Life will be dull if you don't have problems.
II.To avoid problems you should have solutions ready.
(a) Only conclusion I follows (b) Only conclusion II
follows
(c) Neither conclusion I nor II follows
(d) Both conclusions I and II follows
Ans: (c) Ans: (d)
Conclusions :
Q17. Statement: Sun is the source of light. I.Some apples are sweet. (True)
Conclusions: (I) Moon is not the source of light. II.Some bananas are apples. (True)
(II) Light has only one source. So, both I and II follow.
(a) Only conclusion (I) follows
(b) Only conclusion (II) follows Q20. Statement:
(c) Both conclusions (I) and (II) follow Sick people need medicine
(d) Neither conclusions (I) nor (II) follows Conclusions:
Ans: (d) Neither Conclusion I nor II follows. Sun is the I.Healthy people do not need medicine
source of light . It does not imply that light has only one II.People keep medicine in their home.
source. (a) Only conclu sion II follows
(b) Both conclusions I and II follow

Page 276 of 516


(d) Only conclusion I follows
https://telegram.me/aedahamlibra
(c) Neither conclusion I nor II follows II.Some houses are towers.
(a) Only conclusion I follows.
Ans: (d) It is mentioned that medicines are needed for (b) Only conclusion II follows.
sick people. So, healthy people do not need medicines. (c) Both conclusions I & II follow.
(d) Neither conclusion I nor II follows.
DIRECTIONS In the fo llowing questions two/three
statements are given followed by two/three
conclusions respectively. You have to consider the
statements to be true even if they seems to be at
variance from commonly known facts. You have to
decide which of the given conclusion s, if any, follow
from the given statements.

Q21. Statement:
1.Some years are decades. Ans: (b)
2.All centuries are decades. By looking at above venn diagram, we can concluded
Conclusions: that only conclusions II follow.
I.Some centuries are years.
II.Some decades are years. DIRECTIONS: In the questions, Three statements are
III.No century is a year. given followed by two/four conclusions I, II, III, & IV.
(a) Only conclusion I and III follow You have to consider the three statements to be true
(b) Only conclusion I follows even if they seem to be at variance from commonly
(c) Only conclusion II follows known facts. You have to decide which of t he given
(d) Only conclusion I and II follow conclusions if any follow from the given statements.
Ans: (c) First Premise is Particular Affirmative
(I-type) . Q23. Statements :
Some ladies are beautiful.
Some beautifuls are honest.
All honest are sensitives.
Conclusion 1: Some sensitives are beautifuls.
Conclusion 2: Some honest are ladies.
Conclusion 3: Some sensitives are ladies.
(a) None of the Conclusion follows.
(b) Only Conclusion 1 follows.
(c) Only Conclusions 1 and 2 follow.
(d) All Conclusions follow.

Conclusions I - False
Conclusions II - True
Conclusions III - False
Conclusions I and III form Complem entary Pair.
Therefore, either I or III follows. but according to given Ans: (b)
option only conclusion - II follows.
Q24. Statements :
Q22. Statements: 1.All clerks are superintendents.
I.Some towers are windows. 2.All superintendents are managers.
II.All windows are houses. 3.All managers are supervisors.
III.Some houses are temples. Conclusions :
Conclusions: I.All supervisors are clerks.
I.Some towers are temples. II.Some clerks are supervisors.

Page 277 of 516


(a) Only conclusion I.
https://telegram.me/aedahamlibra
III.All superintendents are clerks.
(b) Only conclusion II.
(c) Only conclusion III. (d) Only conclusion IV
Ans: (b)
Statement 2:
Q25. Statements :
I.Meena is older than Roma.
II.Rashmi is older than Meena.
III.Roma is older then Rashmi.
If 1st two statements are true then 3rd statement is
DIRECTIONS: In the following Questions, one
(a) false (b) uncertain
statement is given followed by two conclusions, I and
(c) vague (d) true
II. You have to consider the statement to be true, even
Ans: (a) I. Meena > Roma
if it seems to be at variance from commonly known
II. Rashmi > Meena
facts. You are to decide which of the given conclusions
III. Roma > Rashmi
can definitely be drawn from the given statement.
If 1st two statements are ture
Indicate your answer.
3rd statement is false.
Rashmi > Meena > Roma
Q29. Statement:It is desirable to put the child in
school at the age of 5 or so.
Q26. While having an argument with someone, if you
Assumptions: I.At that age the child reaches
fail
appropriate level of development and is ready to
(a) wait for the next opportunity to beat your opponents
learn.
(b) agree to disagree with a part of the argument
II.The schools do not admit children after 6 years of
(c) feel totally shaken (d) analyse the re asons for
age.
the falure
(a) Only assumption I is implicit
Ans: (b)
(b) Only assumption II is implicit
(c) Neither assumption I nor II is implicit
Q27. One statement is given followed by two
(d) Both assumption I and II are implicit
Conclusions I and II. You have to consider the
Ans: (a)
statement to be true, even if it seems to be at variance
from commonly known facts. You are to decide which
Q30. Statement: Every school should promote
of the given conclusions c an definitely be drawn from
partnerships that will increase parental involvement
the given statement. Indicate your answer.
and participation for promoting the growth of
Statement: The rich must live more simply. All poor
children.
people are simple.
Conclusions I: For the growth of the children, parents
Conclusions :
should be involved in various school activities.
I.Rich people waste money.
Conclusions II. Involvement of parents in school
II.Poor people save money.
activities has no influence on the growth of the
(a) Only I follows. (b) Only II follows.
children.
(c) Neither I nor II follow. (d) Both I and II follow.
(a) Only I follows (b) Only II follows
Ans: (c) Neither I nor II
(c) Neither I nor II follows (d) Both I and II follow
Ans: (a) Parental involvement and participation is
Q28. (a) All roses in Sita's garden are red.
necessary for promoting the growth of children.
(b) All marigold flowers in Sita's garden are orange.
(c) All flowers in Sita's garden are either red or
Q31. Statements: (a) Some schools are Laboratories
orange.
(b) All Laboratories are theaters
If 1st two statements are true, the third is
Conclusions: I. Some theatres are schools
(a) True (b) False
II.No school is a theater
(c) Uncertain (d) Vague
(a) Neither conclusion I nor II follow
Ans: (c) Uncertain
(b) Conclusion I follows
Statement 1:
Page 278 of 516
(c) Conclusion II follows
follows
https://telegram.me/aedahamlibra
(d) Conclusion I & II
Q34. Which conclusion is true with respect to the
given statements?
Statements:
(i) Roy studies History and Political Science
Ans: (b) (ii) Roy studies in Oxford University.
Conclusions: Conclusion:
(I) True (a) Roy does not study Political Science.
(II) False (b) Roy studies History and Political Science in Oxford
University.
Q32. Statement: Cactus plants are fleshy but n eed (c) Roy studies Social Science.
least water (d) Roy does not study History.
Conclusions:I.All fleshy plants are cactus Ans: (b) From the statements (I) and (II) we can
II.Cactus plants need least water conclude that Roy studies History and Political Science
(a) Neither I nor II follows (b) Only II follows in Oxford University.
(c) Both I and II follows (d) Only I follows
Ans: (b) Conclusions: Q35. Statement :
(I) False Travelling by metro in Delhi is more convenient and
(II) True economical.
Assumptions :
Q33. Two statements are given, each followed by two I. Other modes of transport are not available.
conclusion/assumption, I and II. You have to consider II.Metro services are reasonably good.
the statement to be true even if they seem to be at (a) Only assumption I is implicit
variance from commonly known facts. You have to (b) Neither I nor II are implicit.
decide which of the given conclusion/assumptions, if (c) Both I and II are implicit (d) Only assumption II is
any, follows from the given statements. implicit.
Statements: Ans: (d) Travelling by metro in Delhi is more convenient
(i) Some papers are pens. and economical. So we can assume that metro services
(ii) All the pencils are pens. are reasonably good. So assumption II is implicit.
Conclusion:
I.Some pens are pencils Q36. In each of the following question below are given
II.Some pens are papers some statements followed by some conclusions.
(a) Either I or II follows (b) Both I and II follow Taking the given statements to be true even if they
(c) Only I conclusion follows (d) Only II conclusion seem to be at variance from commonly known facts,
follows read all the conclusions and then decide which of the
given conclusion logically follows the given
statements.
Statements :
I.Some pens are pencils.
II.All pencils are erasers.
Ans: (b)
Conclusions :
From the Venn diagram it follows.
I.Some pens are erasers.
So, some pens are pencils and some pens are papers.
II.No pens are erasers.
So, both (I) and (II) follow.
III.Some erasers are pencils.
(a) Only conclusion (II) follows.
DIRECTIONS: One/ two statement are given, each
(b) Only conclusion (I) and (II) follow.
followed by two conclusion/assumption, I and II. You
(c) Only conclusion (I) and (III) follow.
have to consider the statement to be true even if they
(d) No conclusion follows.
seem to be at variance from commonly known facts.
You have to decide which of the given conclusion/
assumptions, if any, follows from the given statements

Page 279 of 516


https://telegram.me/aedahamlibra
Ans: (c)
I. = True
II. = False Ans: (c)
III. = True So, conclusion I and II are follow.
So, Only Conclusion I and III follow.
Q39. One or two statements are given followed by two
Q37. In the following question below are given some Conclusions/Assumptions, I and II. You have to
statements followed by some conclusions. Taking the consider the statements to be true, even if it seems to
given statements to be true even if they seem to be at be at variance from com monly known facts. You are
variance from commonly known facts , read all the to decide which of the given conclusions/assumptions
conclusions and then decide which of the given can definitely be drawn from the given statement.
conclusion logically follows the given statements. Indicate your answer.
Statements : Statement:
I.Some pens are pencils. I.All players are Singers.
II.All pencils are erasers. II.All Dancers are Singers.
Conclusions : Conclusion:
I.Some pencils are not pens. I.Some Singers are Dancers.
II.Some erasers are not pens. II.Some Dancers are Players.
(a) Only conclusion (I) follows. (a) Only conclusion II follows
(b) Only conclusion (II) follows. (b) Both conclusion I & II follow.
(c) Neither conclusion (I) nor conclusion (II) follows. (c) Only conclusion I follows (d) Neither conclusion I
(d) Both conclusions follow. nor II follows.
Ans: (c) According to question,

Ans: (c)
So, neither conclusion I nor II follows.

Q38. In each of the following question below are give n


some statements followed by some conclusions.
Taking the given statements to be true even if they So, Only conclusion I follows.
seem to be at variance from commonly known facts,
read all the conclusions and then decide which of the Q40. In each of the following question below are given
given conclusion logically follows the given some statements followed by some conclusions.
statements. Taking the given statements to be true even if they
Statements : seem to be at variance from commonly known facts,
I.All cups are vegetable. read all the conclusions and then decide which of the
II.All vegetable are pens. given conclusion l ogically follows the given
Conclusions : statements.
I.some pens are vegetable. Statements :
II.Some pens are cups. I.All women are hardworking.
(a) Only conclusion (I) follows II.All intelligent are advocate.
(b) Only conclusion (II) follows III.Some intelligent are women.
(c) Both conclusion follow. (d) Neither conclusion (I) Conclusions :
nor conclusion (II) follows I.Some advocates are women.
II.Some hardworking are women.
III.Some women are advocate.
IV.Some hardworking are intelligent.

Page 280 of 516


(a) Only conclusion https://telegram.me/aedahamlibra
(II) , (III) and (IV) follow
(b) Only conclusion (I) , (II) , and (III) follow
(c) Only conclusion (I) , (III) , and (IV) follow (a) (b)
(d) All conclusions follow

(c) (d)
Ans: (a)
Ans: (d)
DIRECTIONS: From the given answer f igures, select
So, all conclusions are follow.
the one in which the question figure is hidden/
embedded.
Completion & Embedded
Q3. Question Figure:
Figures

Q1. From the given answer figures, select the one in


which the question figure is hidden/embedded in the Answer Figures:
same direction.
Question Figure:

Ans: (c)

Answer Figures: Q4. Question Figure

Answer Figures

Ans: (d)

DIRECTIONS: From the given answer figures, select


the one in which the question figure is
hidden/embedded.
Ans: (c)
Q2. From the given answer figures, select the one in
which the question figure is hidden/embedded. Q5. Which answer figure completes the form in
Questions Figure : question figure ?
Question Figures :

Answer Figures :

Answer figures :
Page 281 of 516
https://telegram.me/aedahamlibra

Ans: (c)
Ans: (b)
DIRECTIONS: Which answer figure completes the
Q6. Question Figure pattern given in the question figure?

Q9. From the given answer figures, select the o ne in


which the question figure is hidden / embedded.
Question Figure

Answer Figures

Answer Figures

Ans: (d)

DIRECTIONS: In the following questions, which


answer figure will complete the question figure? Ans: (b)

Q7. From the given answer figures, select the one in Q10. Question Figure :
which the question figure is hidden/embedded
Question Figure

Answer Figures :
Answer Figures

Ans: (c)
Ans: (b)
DIRECTIONS: In question nos. 27 and 28, which
Q8. Question Figure answer figure will complete the pattern in the
question figure?

Q11. Question Figure :

Answer Figure

Page 282 of 516


https://telegram.me/aedahamlibra

Answer Figures :
Answer Figures:

Ans: (b)
Ans: (c)
DIRECTIONS: Which answer figure w ill complete
Q12. Question figure
the pattern in the question figure?

Q15. Which one of the answer figures is hidden in the


following question figure?
Question Figure :
Answer figures

Answer Figures.
Ans: (b)

DIRECTIONS: which answer figure will complete the


pattern in the question figure?

Q13. From the given answer figures, select the one in


Ans: (c)
which the question figure is hidden/embedded.
Question Figure
Q16. Question figure :

Answer figures :
Answer Figures

Ans: (d)
Ans: (c)
Q17. From the given answer figures, select the figure
Q14. Which one of the answer figures shall complete which is hidden/embedded in the question figure.
the given question figure ? Question figure :
Question Figure:
Page 283 of 516
https://telegram.me/aedahamlibra
Answer figures :

Ans: (a)

Q21. Which of the answer figures is embedded in the


Ans: (d)
question figure ?
Question Figure :
Q18. Which answer figure will complete the pattern in
the question figure ?
Question Figure:

Answer figures :

Answer Figure:

Ans: (c)
Ans: (a)

Q19. From the given answer figures, select the one in


Figure Counting &
which the question figure is hidden/ embedded.
Question Figure:
Analysis

Q1. Question Figure:

Answer Figures:

Answer Figures:

Ans: (b)

Q20. Which answer figure will complete the question


figure ?
Question Figure :
Ans: (c)

Q2. How many rectangles are there in the given


diagram?

Answer Figures :
Page 284 of 516
https://telegram.me/aedahamlibra
Q6. Question Figure :

(a) 4 (b) 7
(c) 9 (d) 18
Ans: (d)

Q3. How many triangles are there in the figure Answer Figures :
ABCDEF?

Ans: (a)
(a) 24 (b) 26
(c) 28 (d) 30 Q7. Identify th e response figure from which the
Ans: (c) question figure‘s pieces have been cut.
Question figure
DIRECTIONS: : Among the four answer figures,
which figure can be formed from the cut -pieces given
below in the question figure?

Q4. How many triangles are there in this figure?


Question figure : Answer Figures.

(a) 24 (b) 26
Ans: (d)
(c) 28 (d) 20
Ans: (c)
Q8. There is a ball and a rectangular jar. Four
positions are shown below to keep them balanced.
Q5. Question Figure :
Which of the following will not get balanced easily?
Question figures:

Answer Figures :
Answer figures:

Ans: (c) Ans: (c)

Page 285 of 516


https://telegram.me/aedahamlibra
Q9. Which of the answer figures include the separate
components found in the question figure?
Question figure:

Answer Figures.

Answer figure:

Ans: (b)

Q13. How many rectangles are there in the question


figure ?
Ans: (c) Question figure :

Q10. How many triangles are there in the given


figure?

(a) 6 (b) 7
(c) 8 (d) 9
Ans: (d)
(a) 10 (b) 12
Q14. Among the for answer figures, which figure can
(c) 14 (d) 11
be formed from t he cut - pieces given below in the
Ans: (c)
question figure ?
Question figure :
Q11. Find out which of the answer figures will exactly
make up the question figure?
Question Figure:

Answer figures :

Answer Figures:

Ans: (d)

Q15. Find out which answer figure will exactly make


up the question figure.
Ans: (a) Question figure :

Q12. Find out which of the following answer figures


will exactly make up the question figure ?
Question Figure :

Answer figures :
Page 286 of 516
https://telegram.me/aedahamlibra
(a) 32
(c) 37
(b) 34
(d) 40
Ans: (c)

Q20. How many quadrilaterals are there in the given


Ans: (c) figure?

Q16. How many triangles are there in the give figure ?

(a) 48 (b) 60
(a) 2 (b) 3
(c) 56 (d) 52
(c) 4 (d) 5
Ans: (c)
Ans: (a)
Q17. Find the number of triangles in the following
Q21. How many triangles are there in the gi ven figure
figure :
?

(a) 8 (b) 14
(c) 10 (d) 12 (a) 24 (b) 26
Ans: (b) (c) 28 (d) 30
Ans: (b)
Q18. How many triangles are there in the given
figure?
(a) 20 (b) 22
Paper Cutting And
(c) 28 Folding

Q1. A piece of paper is folded and cut as shown below


in the question figures. From the given answer
figures, indicate how it will appear when opened.
(d) 32 Questions Figures :
Ans: (b)

Q19. How many triangles are there in the given


figure?

Answer Figures :

Page 287 of 516


https://telegram.me/aedahamlibra

Ans: (c) Answer figures

Q2. A piece of paper is folded and cut as shown below


in the question figures. From the given answer
figures, indicate how it will appear when opened.
Question figure

Ans: (a)

Q5. In the following question, a piece of paper is


folded and cut as shown below i n the question figures.
From the given answer figures, indicate how it will
Answer figures
appear when opened.
Question Figures:

Ans: (c)
Answer figures:
Q3. A square sheet of paper has been folded and
punched as shown below in the question figures. You
have to figure out from amongst the four answer
figures, how it will appear when opened?
Question Figures :
Ans: (d)

Q6. In the following question a piece of paper is folded


and punched as shown below in the question figure.
From the given answer figures, indicate how it will
appear when opened?
Answer Figures : Question figure

Answer figures

Ans: (c)

Q4. A square sheet of paper has been folded and


punched as shown below. You have to figure out from
amongst the four response figures, how it will appear
when opened? Ans: (b)
Question figures :

Page 288 of 516


https://telegram.me/aedahamlibra
Q7. A piece of paper is folded and punched as shown
below in the question figures. From the given answer
Q10. A piece of paper is folded and cut as shown
below in the question figures. From the given answer
figures, indicate how it will appear when opened. figures, indicate how it will appear when opened.
Question figures : Question figures :

Answer Figures
Answer figures :

Ans: (d)
Ans: (d)
Q8. A piece of paper is folded and punched as shown
below in the question figures. From the given answer Q11. A piece of paper is folded and cut as shown
figures, indicate how it will appear when opened. below in the question figures. From the giv en answer
Question Figures : figures, indicate how it will appear when opened.
Question figures :

Answer Figures.
Answer figures :

Ans: (a)
Ans: (c)
Q9. A piece of paper is folded and cut as shown below
in the question figures. From the given answer
Q12. A piece of paper is folded and cut as shown
figures, indicate how it will appear when opened.
below in the question figures. From the given answer
Question figure:
figures indicate how it will appear when opened.
Question figure :

Answer figures:
Answer figures :

(a) (b)

Ans: (b)

Page 289 of 516


https://telegram.me/aedahamlibra
(a) (b)
(c) (d)
Ans: (c)
(c) (d)
Q13. A piece of paper is folded and cut. From the Ans: (c)
figures given, indicate how it will appear when opened
Q16. A piece of paper is folded and punched as shown
below in the question figures. From the given answer
figures, indicate how it will appear when opened?

(a) (b)

(a) (b)
(c) (d)
Ans: (d)

Q14. A piece of paper is folded and punched as shown (c) (d)


below in the question figure. From the given answer Ans: (b)
figures, indicates how it will appear when opened.
Question Figures : Q17. A piece of paper is folded and punched as shown
below in the question figures. From the given answer
figures, indicate how it will appear when opened?

Answer Figures:

(a) (b)

Ans: (b)

Q15. A piece of paper is folded and punched own (c) (d)


below in the question figures. F rom the given answer Ans: (c)
figures, indicates how it will appear when opened.
Q18. A piece of paper is folded and punched as shown
below in the question figures. From the given answer
figures, indicate how it will appear when opened?

Page 290 of 516


https://telegram.me/aedahamlibra

Ans: (b)

Q3. Which of the answer figure is exactly the mirror


(a) (b) image of the question fi gure if a mirror is placed on
the line MN?
Question Figure :

(c) (d)
Ans: (c)

Mirror & Water Images


Answer Figures :

Q1. Which answer figure is the exact mirror image of


the given question figure when the mirror is held from
the right at PQ?
Question Figure:
Ans: (d)

Q4. In the question if a mirror is placed on the line


MN, then which of the answer figures is the right
image of the given figure?
Question figure

Answer Figures:

Answer figures

Ans: (c)

Q2. Question Figure :


Ans: (c)

DIRECTIONS: A mirror is placed on line MN. Then


which of the answer figures is the correct image of the
given figure?
Answer Figures :
Q5. From the answer figures, find out the figure
which is the exact mirror image of the question figure,
when the mirror is placed on the line MN.
Page 291 of 516
Question Figure : https://telegram.me/aedahamlibra

Ans: (b)

Answer Figures : Q8. If a mirror is placed on the line MN, then which
of the answer figures is the right image of the given
figure ?
Question Figure :

Ans: (d)

Q6. Question Figure :


Answer Figures :

Ans: (c)
Answer Figures :
Q9. If a mirror is plac ed on the line MN, then which
of the answer figures is the right image of the given
figure ?
Question Figure :
(a) (b)

(c) (d)
Ans: (c)

Q7. If a mirror is placed on the line MN, then which Answer Figures :
of the answer figures is the right image of the given
figure ?
Question Figure :

Ans: (c)

Q10. Which one of the following is water image of


"COMMISSION"?
(a) (b)
Answer Figures :
(c)
(d)
Page 292 of 516
Ans: (c) https://telegram.me/aedahamlibra
Q11. Mirror is placed on the line MN, then which of
the answer figures is the right image?

Answer Figure:

(a) (b)

(c) (d)
Ans: (b)

(a) (b) Q14. If a mirror is placed on the line AB, then which
of the answer figures is the right image of the given
figure?

(c) (d)
Ans: (b)

Q12. If a mirror is place on the line MN , then which


of the answer figures is the right image of the figure?

(a) (b)

(c) (d)
Ans: (c)

Q15. If a mirror is on the line AB, then which of the


answer figures the right image of the given figure?

(a) (b)

(c) (d)
Ans: (a)
(a) (b)

Q13. Question Figure:

(c) (d)
Ans: (b)

Page 293 of 516


https://telegram.me/aedahamlibra
Q16. If a mirror placed on the line AB, then which of
the answer is the right image of the given figure?

(c) (d)
Ans: (d)

Q19. If a mirror is placed on the line AB, then which


of the answer figures is the right image of the given
figure?
(a) (b)

(c) (d)

Q17. If a mirror is placed on the line MN, then which


of the answer figure is the right image of the given
figure?

(a) (b)

(c) (d)
Ans: (b)
(a) (b)
Q20. If a mirror is placed on the line AB, then which
of the answer figures is the right image of the given
figure?
(c) (d)
Ans: (c)

Q18. If a mirror is placed on the MN, then which of


the answer figures is the right imag e of the given
figure?
Question Figure:

(a) (b)

(c) (d)
Ans: (c)
Answer Figure :

(a) (b)

Page 294 of 516


https://telegram.me/aedahamlibra
Visual & Non Verbal Q4. Which of the following cubes can be created by
folding the given figure?
Reasoning Question Figure.

Q1. A dice is thrown four times and its four different


positions are given be low. Find the number on the
face opposite the face showing 2. Answer Figures.

(a) 4 (b) 5
(c) 6 (d) 3
Ans: (b) Ans: (b) When folded in the form of a cube, then 'F'
appears opposite 'B', 'E' appears opposite 'C' and 'A'
Q2. Four different positions of dice are as shown appears opposite ‗D‘.
below. What number is opposite to face 3 ? In option (a) 'F' is adjacent to 'B'
In option (c) 'E' is adjacent to ‗C‘.
In option (d) 'E' is adjacent to ‗A‘.

Q5. Two positions of a dice are given. Which number


would be at the top when bottom is 2?
(a) 4 (b) 3
(c) 2 (d) 6
Ans: (a) The numbers 1, 2, 5 and 6 are on the adjacent
faces of the number 3. So, the number 4 lies opposite 3.

Q3. Three views of the same cube are given. All the
faces of the cube are numbered from 1 to 6. Select one (a) 4 (b) 1
figure which will result when the cube is unfolded. (c) 5 (d) 6
Question Figure : Ans: (d) The numbers 1, 2, 5 and 6 are on the adjacent
faces of number 3. Therefore. the number 4 lies opposite
3.
The numbers 3, 4 and 6 can not be on the faces opposite
to 1. Therefore, 5 lies opposite 1.
Now, 2 lies opposite 6.
Answer Figures :
Q6. Two positions of a dice are shown below. If 1 is at
the bottom, which number will be on top?

Ans: (a) If we fold the option (b) the number 2 will lie (a) 4 (b) 3
opposite 5. (c) 8 (d) 5
If we fold the option (c) the number 1 will lie opposite 3. Ans: (b) By looking, the dice position, we can say that 2,
If we fold the option (d) the number 2 will lie opposite 5. 4, 5 and 6 are adjacent faces of 3. therefore, if 1 number
Therefore, answer figure (d) is correct. is at the bottom then 3 will be on the top.

Page 295 of 516


https://telegram.me/aedahamlibra
Q7. From the given block s when 10 is at the bottom,
which number will be at the top ? Ans: (a)
From above, it is clear that 3 is opposite to 4.
Therefore, 1 is opposite 6.

Q10. Find the missing figure.


Question Figures :
(a) 8 (b) 12
(c) 6 (d) 4
Ans: (b) From the two views of blocks it is clear that
when 10 is at the bottom, number 12 will be at the top.
Answer Figures :
Q8. Which of the four cubes pictured below are
correct view when the six squares are folded (in
clockwise direction) into a cube?
Question Figure:

Ans: (a) The middle element adjecnts to the right side


line after rotating 90° anticlock wise. The bottom element
goes up on the top and becomes enlarge.
The top element becomes the inner figure of bottom
element.
Answer Figures:

Q11. Select the figure which is different from the rest.

Ans: (a) 3 lies opposite


.
Ans: (a)
1 lies opposite W.
1 lies opposite
Q12. Choose the cube that will be formed by folding
. the sheet of paper shown in the problem figure.
In option (b) 3 is on the adjacent face of dot. Question Figure :
In option (c) 1 is on the adjacent face of W.
In option (d) is on the top and hence the positions of
dot (
) and W are not correct.

Q9. Four positions of a dice are given below, Identify Answer Figures :
the number at the bottom then top is 6.
Question Figures :

Ans: (d) Option (d) is correct.


(a) 1 (b) 3
(c) 4 (d) 5 Q13.

Page 296 of 516


https://telegram.me/aedahamlibra
Q17. Three positions of a cube are shown below. What
will come opposite to face containing (I) ?

The solid so formed by joining unit cubes is rotated to


obtain different positions, which of these cannot be
the shape after it has turned?

(a) VI (b) IV
(c) II (d) V
(a) (b) Ans: (a) Therefore, the number II, IV, III and V are on
the adjacent faces of number I.
Therefore, the number IV liles opposite I.

Q18. Three positions of a cube are shown below. What


(c) (d) will come opposite to face containing '$'?
Ans: (a) Option (a) is correct.

Q14. Two position of dice are shown below. When


three is at the top what number will be at the bottom.
(a) % (b) &
(c) ^ (d) +
Ans: (b) By looking, the dice position, we can say that
%, *, ^ and + are adjacent faces of $.
therefore,
(a) 5 (b) 1 ∴& will come opposite of $.
(c) 4 (d) 2
Ans: (a) When 3 is at the top 5 will be at the bottom. Q19. Three creation of a cube are shown below. what
will opposite to face containing '5'?
Q15. In t he given cubes, which colour is opposite to
purple?

(a) 6 (b) 1
(c) 3 (d) 2
Ans: (b) The numbers 2, 3, 4 and 6 are on the adjacent
(a) Violet (b) Red faces of the number 1.
(c) Yellow (d) Blue So, the number 5 lies opposite 1.
Ans: (c) Purple → Red, Blue, Yellow, Green, Orange
Green, Orange, Red, Blue are adjacent to purple Q20. Three positions of a cube are shown below. What
will come opposite to face containing 'Q'?
Q16. The following figure is folded to form a block.
Which symbol will appear on the opposite of ?

(a) P (b) R
(c) S (d) Z
Ans: (c) The letters P, X , R and Z are on the adjacent
(a) (b) faces of the letter Q. So, the letter S lies opposite Q.
(c) (d)
Ans: (c)
Page 297 of 516
https://telegram.me/aedahamlibra

Page 298 of 516


https://telegram.me/aedahamlibra
GENERAL AWARENESS
(a) The study of coins
Ancient India (b) The study of inscriptions
(c) The study of epics
(d) The study of geography
Q1. The home of Gargi, Maitrey, and Kapila was at
Ans: (b)
(a) Vidisha (b) Ujjain
Q11. The Harappan Civilisation was discovered in the
(c) Pataliputra (d) Mithila
year :
Ans: (d)
(a) 1935 (b) 1942
Q2. The university which became famous in the post -
(c) 1901 (d) 1922
Gupta Era was :
Ans: (d)
(a) Kanchi (b) Taxila
Q12. How was Burma (now Myanmar) known to
(c) Nalanda (d) Vallabhi
ancient Indians ?
Ans: (c)
(a) Malayamandalam
Q3. The First Tirthankara of the Jains was :
(b) Yavadwipa
(a) Arishtanemi (b) Parshvanath
(c) Suvarnabhumi
(c) Ajitanath (d) Rishabha
(d) Suvarnadwipa
Ans: (d)
Ans: (c)
Q4. Yavanika or curtain was introduced in Indian
Q13. Who is hailed as the ―God of Medicine‖ by the
theatre by which of the following?
practitioners of Ayurveda ?
(a) Shakas (b) Parthians
(a) Susruta (b) Chyavana
(c) Greeks (d) Kushans
(c) Dhanwantari (d) Charaka
Ans: (c)
Ans: (c)
Q5. Which among the following is the oldest dynasty ?
Q14. Prince Ellara conquered Sri Lanka in the second
(a) Maurya (b) Gupta
century BC.
(c) Kushan (d) Kanva
With which of the following dynasties of Dravida ruler
Ans: (a)
was he associated ?
Q6. Where is Brihadeshwar Temple situated ?
(a) Chera (b) Chola
(a) Kanchi (b) Madurai
(c) Pandya (d) Pallava
(c) Shri Shailan (d) Tanjore
Ans: (b)
Ans: (d)
Q15. The greatest development in the Kushana period
Q7. Ganhadra school of art came into existence in
was in the field of
(a) Hinayana sect
(a) religion (b) art
(b) Mahayana sect
(c) literature (d) architecture
(c) Vaishnava sect
Ans: (b)
(d) Shaiva sect
Q16. The tutor of Alexander, the Great was
Ans: (b)
(a) Darius (b) Cyrus
Q8. Which rulers built the Ellora temples?
(c) Socrates (d) Aristotle
(a) Chalukya (b) Sunga
Ans: (d)
(c) Rashtrakuta (d) Pallava
Q17. ‗Charak‘ was the famous court physician of
Ans: (c)
(a) Harsha
Q9. Mohammed-bin-Qasim conquered Sind in the
(b) Chandra Gupta Maurya
year
(c) Ashoka
(a) 712 A.D. (b) 812 A.D.
(d) Kanishka
(c) 912 A.D. (d) 1012 A.D.
Ans: (d)
Ans: (a)
Q18. Most of the chola temples were dedicated to
Q10. Epigraphy means
(a)Ganesh (b) Shiva
Page 299 of 516
(c) Durga https://telegram.me/aedahamlibra
(d) Vishnu
Ans: (b)
(c) Megasthenes
(d) Justin
Q19. Lothal is a site where dockyards of which of the Ans: (c)
following civilization were found ? Q28. Worship of Mother Goddess was associated with
(a) Indus Valley (a) Aryan Civilization
(b) Mesoptamian (b) Mediterranean Civilization
(c) Egyptian (c) Indus Valley Civilization
(d) Persian (d) Later Vedic Civilization
Ans: (a) Ans: (c)
Q20. The essential feature of the Indus Valley Q29. The striking feature of the Indus Valley
Civilisation was Civilization was
(a) worship of forces of nature (a) Urban Civilization
(b) organised city life (b) Agrarian Civilization
(c) pastoral farming (c) Mesolithic Civilization
(d) caste society (d) Paleolithic Civilization
Ans: (b) Ans: (a)
Q21. Arabs were defeated in 738 A.D. by Q30. The caste system of India was created for :
(a) Pratiharas (b) Rashtrakutas (a) immobility of labour
(c) Palas (d) Chalukyas (b) recognition of the dignity of labour
Ans: (d) (c) economic uplift
Q22. Name the clan Buddha belonged to (d) occupational division of labour
(a) Gnathrika (b) Maurya Ans: (d)
(c) Sakya (d) Kuru Q31. ―Monolithic Rathas‖ of the Pallavas are found at
Ans: (c) (a) Kanchipuram
Q23. The people of the Indus Valley Civilization (b) Puri
usually built their houses of (c) Mahabalipuram
(a) Pucca bricks (d) Agra
(b) Stone Ans: (c)
(c) Wood Q32. Which was the oldest University?
(d) All of the above (a) Gandhara (b) Kanauj
Ans: (a) (c) Nalanda (d) Vaishali
Q24. The monk who influenced Ashoka to embrace Ans: (c)
Buddhism was Q33. Bindusara sent Asoka to quell the rebellion in—
(a) Vishnu Gupta (a) Swarnagiri (b) Taxila
(b) Upagupta (c) Ujjain (d) Tosali
(c) Brahma Gupta Ans: (c)
(d) Brihadratha Q34. Which language was mostly used for the
Ans: (b) propagation of Buddhism?
Q25. Which of the following is not one of the animals (a) Sanskrit (b) Prakrit
carved on the Sarnath Pillar ? (c) Pali (d) Sauraseni
(a) Humped Bull (b)Deer Ans: (c)
(c) Elephant (d) Horse Q35. Who was the court poet of Harsha?
Ans: (b) (a) Bhani (b) Ravi Kirti
Q26. The headquarters of the Ghadar Party was at (c) Banabhatta (d) Vishnu Sharma
(a) Karachi (b)Moscow Ans: (c)
(c) Berlin (d) San Francisco Q36. Buddha gave his first religious message at
Ans: (d) (a) Rajagriha (b) Pataliputra
Q27. The Greek ambassador sent to Chandragupta (c) Gaya (d) Sarnath
Maurya's Court was : Ans: (d)
(a) Kautilya Q37. Which Chola king founded the city of Puhar?
(b) Seleucus Nicator (a) Rajendra Chola
Page 300 of 516
(b) Ellara
(c) Senguttavan
https://telegram.me/aedahamlibra
(d) Kumaragupta
Ans: (c)
(d) Karikala Q47. Which of the following Gupta kings stopped the
Ans: (d) Huns from invading India?
Q38. The Virupaksha Temple was built by the (a) Kumaragupta
(a) Chalukyas (b) Pallavas (b) Samudragupta
(c) Vakatakas (d) Satavahanas (c) Skandagupta
Ans: (a) (d) Chandragupta
Q39. The art style which combines Indian and Greek Ans: (c)
features is called Q48. Who amidst the following was a wife of emperor
(a) Sikhara (b) Verna Ashoka who influenced him?
(c) Nagara (d) Gandhara (a) Chandalika (b) Charulata
Ans: (d) (c) Gautami (d) Karuwaki
Q40. Chalukya king Pulakesin-Il was defeated by Ans: (d)
(a) Mahendra Varman-I Q49. ‗Tripitaka‘ is the religious book of
(b) Narasimha Varman-I (a) Jains (b) Buddhists
(c) Parameswara Varman-I (c) Sikhs (d) Hindus
(d) Jatila Parantaka Ans: (b)
Ans: (a) Q50. During whose reign did the Gandhara School of
Q41. The crop which was not known to Vedic people Art blossom ?
is (a) Harsha
(a) barley (b) wheat (b) Ashok
(c) rice (d) tobacco (c) Kanishka
Ans: (a) (d) Chandragupta II
Q42. The last Buddhist king who was a great Sanskrit Ans: (c)
scholar and a writer was Q51. Vaishakha Poornima has a great significance
(a) Kanishka (b) Ashoka because it was on this day
(c) Bimbisara (a) Buddha was born
(d) Harshavardhana (b) Buddha got enlightened
Ans: (d) (c) Buddha died
Q43. Carving in the famous Ajanta caves was first (d) All of the above
started during the reign of the Ans: (d)
(a) Kadambas (b) Satavahanas Q52. Which one of the following Chola Kings
(c) Rashtrakutas (d) Marathas conquered Ceylon (Singhal) first ?
Ans: (b) (a) Aditya-I (b) Rajaraja-I
Q44. ―Harsha Charita‖ was written by (c) Rajendra (d) Vijayalya
(a) Kalidasa (b) Banabhatta Ans: (b)
(c) Valmiki (d) Vyasa Q53. Buddha preached his first sermon at–
Ans: (b) (a) Gaya (b) Sarnath
Q45. Buddhism in Nepal was introduced during the (c) Pataliputra (d) Vaishali
reign of Ans: (b)
(a) Samudragupta Q54. Harappa is situated on the bank of the river :
(b) Ashoka (a) Ganga (b) Ravi
(c) Chandragupta (c) Yamuna (d) Sindhu
(d) Harshavardhana Ans: (b)
Ans: (b) Q55. The Aryans successded in their conflicts with the
Q46. The Gupta king who assumed the title of pre-Aryans because
‗Vikramaditya‘ was (a) they used elephants on a large scale
(a) Skandagupta (b) they were taller and stronger
(b) Samudragupta (c) they were from an advanced urban culture
(c) Chandragupta-II (d) they used chariots driven by horses
Page 301 of 516
https://telegram.me/aedahamlibra
Ans: (d)
Q56. Which of the following are beliefs of Buddhism ?
(b) Bimbisara
(c) Ajatshatru
(a) The world is full of sorrows. (d) Mahapadma Nanda
(b) People suffer on account of desires. Ans: (b)
(c) If desires are conquered, nirvana will be attained. Q65. What is ‗Milindapanho‘ ? -
(d) The existence of God and Soul must be recognised. (a) A Buddhist place
(a) (a), (b), (c) and (d) (b) One of the names of Buddha
(b) (b) and (c) (c) A Buddhist Specimen of Art
(c) (a), (b) and (c) (d) A Buddhist text
(d) (b), (c) and (d) Ans: (d)
Ans: (c) Q66. Buddha, Dhamma and Sangha together are
Q57. Who was the mother of Mahavira? known as
(a) Yashoda (b) Trishala (a) Triratna (b) Trivarga
(c) Jameli (d) Mahamaya (c) Trisarga (d) Trimurti
Ans: (b) Ans: (a)
Q58. ‗Prince of Pilgrims‘ was the name attributed to Q67. Which one of the following is not included in the
(a) Fa-Hien (c) Hiuen Tsang ‗Eight Fold Path‘ of Buddhism ?
(b) I-tsing (d) Megasthenes (a) Right Speech
Ans: (b) (b) Right Contemplation
Q59. The year of accession of Kanishka to throne was (c) Right Desire
: (d) Right Conduct
(a) 108 AD (b) 78 AD Ans: (c)
(c) 58 AD (d) 128 AD Q68. Pulakesin II was the greatest ruler of the
Ans: (b) (a) Cholas of Tamil Nadu
Q60. The famous poet Kalidasa lived in the court of (b) Chalukyas of Badami
(a) Chandra Gupta I (c) Chalukyas of Kalyani
(b) Samudra Gupta (d) Pallavas of Kanchi
(c) Kumara Gupta Ans: (b)
(d) Chandra Gupta II Q69. The Gandhara art flourished under :
Ans: (d) (a) the Kushanas
Q61. The seashore temple at Mahabalipuram was (b) the Satavahanas
built by (c) the Guptas
(a) Mahendra Varman I (d) the Mauryas
(b) Narasimha Varman I Ans: (a)
(c) Nandi Varman II Q70. The script of the Indus Valley Civilization is
(d) Dandi Varman (a) Kharosthi (b) Undeciphered
Ans: (b) (c) Brahmi (d) Tamil
Q62. The Nalanda University was founded by Ans: (b)
(a) Harsha Vardhana Q71. Which one of the following tribal assemblies was
(b) Kumara Gupta normally involved in the election of the tribal chief ?
(c) Samudra Gupta (a) Samiti (b) Sabha
(d) Chandra Gupta (c) Gana (d) Vidata
Ans: (b) Ans: (a)
Q63. Which inscription mentions about the village Q72. The Sage who is said to have Aryanised South
administration under the Cholas ? India, was
(a) Junagarh (b) Uttaramerur (a) Yagnavalkya (b) Vashistha
(c) Aihole (d)Nasik (c) Agastya (d) Vishwamitra
Ans: (b) Ans: (c)
Q64. Which of the following ruler was a Q73. The Earliest Settlements of Aryan tribes were at
contemporary of Buddha ? (a) Uttar Pradesh
(a) Udayin (b) Bengal
Page 302 of 516
(c) Sapta Sindhu
(d) Delhi
https://telegram.me/aedahamlibra
(b) Social aspects
(c) Economic doctrines
Ans: (c) (d) Military aspects
Q74. Buddha means Ans: (a)
(a) Great Conqueror Q82. Who built the famous Shiva temple at Ellora ?
(b) Great Saint (a) Rashtrakuta Ruler Krishna I
(c) Wise one (b) Mauryan Emperor Ashoka
(d) Enlightened one (c) Gupta King Samudra Gupta
Ans: (d) (d) Chalukyan King Pulikeshi II
Q75. Who was the contemporary South Indian ruler Ans: (a)
of Harshavardhana ? Q83. Find the odd one :
(a) Krishnadevaraya (a) Samveda (b) Yajurveda
(b) Pulakeshin II (c) Vishnu Purana (d) Rigveda
(c) Mayuravarma Ans: (c)
(d) Chikkadevaraja Wodeyar Q84. In which of the following mudra did Gautam
Ans: (b) Buddha delivered his first sermon at Sarnath?
Q76. From which among the following rulers has the (a) Abhaya Mudra
Government of India borrowed and adopted its (b) Dhyana Mudra
symbols ? (c) Dharmachakra Mudra
(a) Ashoka (d) Bhumisparsa Mudra
(b) Krishnadevaraya Ans: (c)
(c) Pulakesin Q85. The people of the Indus valley civilisation
(d) Kanishka worshipped
Ans: (a) (a) Vishnu (b) Pashupati
Q77. Who compiled the tales of ―The Panchatantra‖ ? (c) Indra (d) Brahma
(a) Valmiki Ans: (b)
(b) VedaVyasa Q86. With which religion is Kaivalya associated?
(c) Vishnu Sharma (a) Buddhism (b) Jainism
(d) Tulsidas (c) Hinduism (d) Sikhism
Ans: (c) Ans: (b)
Q78. Which one of the following was the book written Q87. Match the following :
by Amoghvarsha the Rashtrakuta King ? A. Mohenjodaro 1. Statue of a priest
(a) Adipurana B. Harappa 2. Port
(b) Ganitasara Samgraha C. Kalibangan 3. Plough marks
(c) Saktayana D. Lothal 4. The Great Bath
(d) Kavirajamarga (a) A–4, B–1, C–3, D–2
Ans: (d) (b) A–3, B–2, C–4, D–1
Q79. The word ‗Buddha‘ means (c) A–2, B–3, C–1, D–4
(a) A Conqueror (d) A–1, B–4, C–2, D–3
(b) A Liberator Ans: (*)
(c) A Enlibhtened one Q88. Who were the patrons of Sangama Literature?
(d) A Wanderer (a) Nayakas (b) Chandellas
Ans: (c) (c) Pandyas (d) Solankis
Q80. Who among the following Mughal rulers has Ans: (c)
been called the ‗Prince of Builders‘? Q89. Who among the following was the court
(a) Akbar (b) Jahangir physician of Kanishka?
(c) Shah Jahan (d)Babur (a) Vasumitra (b) Nagarjuna
Ans: (c) (c) Charaka (d) Patanjali
Q81. Kautilya‘s ―Arthashastra‖ mainly deals with Ans: (c)
____ : Q90. Who among the following was the first
(a) Political state craft grammarian of the Sanskrit language?
Page 303 of 516
(a) Kalhana
(c) Kalidasa
https://telegram.me/aedahamlibra
(b) Maitreyi
(d) Panini
(a) Kalinjar
(c) Asirgarh
(b) Ajaygarh
(d) Gulbarga
Ans: (d) Ans: (c)
Q91. Harsha moved his capital from _____ to _____ . Q7. The battle that led to the foundation of Muslim
(a) Thanesar, Kannauj power in India was
(b) Delhi, Deogiri (a) The first battle of Tarain
(c) Kamboj, Kannauj (b) The second battle of Tarain
(d) Valabhi, Delhi (c) The first battle of Panipat
Ans: (a) (d) The second battle of Panipat
Q92. Ashoka was a king of which dynasty? Ans: (b)
(a) Pradyota (b) Haryanka Q8. Which of the following aspects is not common to
(c) Maurya (d) Nanda both Bhakti movement and Sufi movement?
Ans: (c) (a) Personal love for God
(b) Worship of idols
(c) Mysticism
Medieval India (d) Visit to holy shrines
Ans: (b)
Q1. The capital of the Yadava rulers was : Q9. Mughal presence in the Red Ford ceased with the
(a) Dwarasamudra fall of
(b) Warangal (a) Aurangzeb
(c) Kalyani (d) Devagiri (b) Muhammad Shah
Ans: (d) (c) Shah Alam
Q2. Which of the following is in the World Heritage (d) Bahadur Shah ‗Zafar‘
list ? Ans: (d)
(a) Khajuraho Q10. The foreign traveller who visited India during
(b) Nalanda ruins the Mughal period and who left us an expert‘s
(c) Hampi ruins description of the Peacock Throne, was
(d) Tajmahal (a) Geronimo Verroneo
Ans: (c) (b) ‗Omrah‘ Danishmand Khan
Q3. Who got the monumental ‗Rayagopurams‘ in (c) Travernier
front of the temples at Hampi, Tiruv -annamalai, (d) Austin of Bordeaux
Chidambaram, Srirangam, Tirupati, etc., Ans: (c)
constructed? Q11. Amir Khusrau was a musician and
(a) Vidyaranya (a) Sufi saint
(b) Krishnadevaraya (b) Persian and Hindi writer, and scholar
(c) Harihara (c) historian
(d) Rajaraja (d) All of the above
Ans: (b) Ans: (d)
Q4. The writer of Ram Charit Manas, Tulsidas, was Q12. The famous Kohinoor diamond was produced
related to which ruler ? from one of the mines in
(a) Chandragupta Maurya (a) Orissa (b) Chhota Nagpur
(b) Nawab Vajid Ali Sah (c) Bijapur (d) Golconda
(c) Harsha (d) Akbar Ans: (d)
Ans: (d) Q13. Who of the following was sent as an ambassador
Q5. Shah Jahan built the Moti Masjid at to the royal court of Jahangir by James 1, the then
(a) Delhi (b) Jaipur king of England?
(c) Agra (d) Amarkot (a) John Hawkins
Ans: (c) (b) William Todd
Q6. Which among the following fort was known as the (c) Sir Thomas Roe
‗Key of Deccan‘? (d) Sir Walter Raleigh
Ans: (a)
Page 304 of 516
ruler
https://telegram.me/aedahamlibra
Q14. The Qutub Minar was completed by the famous Q22. The reputed musician duo, Tansen and Baiju
Bawra, flourished during the reign of—
(a) Qutub-ud-din Aibak (a) Jahangir
(b) Iltutmish (b) Bahadur Shah Zafar
(c) Firoz Shah Tughlaq (c) Akbar
(d) Alauddin Khilji (d) Shah Jahan
Ans: (b) Ans: (c)
Q15. Which of the undermentioned facts about Taj Q23. Which Sultan of Delhi established an
Mahal in not correct? employment bureau, a charity bureau and a
(a) It is a magnificent mausoleum charitable hospital?
(b) It was built by Shah Jahan (a) Firoz Tughlaq
(c) It is situated outside Agra Fort (b) Mohammad Tughlaq
(d) The names of artisans who built it are engraved on it? (c) Alauddin Khilji
Ans: (d) (d) Balban
Q16. The Lodi dynasty was founded by Ans: (a)
(a) Ibrahim Lodi Q24. Who among the following Mughal emperors,
(b) Sikandar Lodi brought about the fall of Sayyid Brothers?
(c) Bahlol Lodi (a) Bahadur Shah I
(d) Khizr Khan (b) Rafi-ud-daulah
Ans: (c) (c) Shah Jahan II
Q17. After th e death of Rajaram in 1700 A.D., (d) Muhammad Shah
Marathas continued the war against the Mughals Ans: (d)
under his brave wife Q25. Which Rajput ruler drew up a set of tables
(a) Tarabai (b) Lakshmibai called Zij Muhammadshuhi to enable the people to
(c) Ramabai (d) Jijabai make astronomical observations?
Ans: (a) (a) Ajit Singh
Q18. The Muslim adventurer who destroyed the (b) Raja Sawai Jai Singh
Nalanda University was (c) Bhara Mal
(a) Alla-ud-din Khilji (d) Man Singh
(b) Muhammad-bin-Tughlak Ans: (b)
(c) Muhammad-bin-Bhaktiyar Q26. Who was called the ‗Second founder of the
(d) Muhammad-bin-Quasim Maratha Kingdom‘?
Ans: (c) (a) Raja Ram
Q19. The famous Peacock Throne of Shah Jahan was (b) Balaji Viswanath
taken away in 1739 by (c) Baji Rao I
(a) Afghan invader Ahmed Shah Abdali (d) Balaji Baji Rao
(b) Persian invader Nadir Shah Ans: (c)
(c) Mongol invader Chengiz Khan Q27. Who among the following Sultans of Delhi has
(d) British East India Company been described by the historians as the ‗mixture of
Ans: (b) opposites‘?
Q20. The French East India Company was founded in (a) Balban
(a) 1600 (b) 1620 (b) Alauddin Khilji
(c) 1664 (d) 1604 (c) Muhammad Bin Tughlaq
Ans: (c) (d) Ibrahim Lodi
Q21. The Sikh military sect ‗the Khalsa‘ was Ans: (c)
introduced by— Q28. The temple built in 1 100 A. D.
(a) Har Rai and dominating all other temples in Bhubaneshwar is
(b) Harkishan (a) Raja Rani temple
(c) Gobind Singh (b) Kandariya Mahadev
(d) Tegh Bahadur (c) Tribhuvaneswara Lingaraja
Ans: (c) (d) Mukhteswara
Page 305 of 516
https://telegram.me/aedahamlibra
Ans: (c)
Q29. Tansen, a great musici an of his times, was in the
Ans: (c)
Q38. Which one is not situated at Fatehpur Sikri ?
Court of (a) The Panch Mahal
(a) Jehangir (b) Akbar (b) Moti Masjid
(c) Shah Jahan (d)Bahadur Shah (c) Tomb of Salim Chishti
Ans: (b) (d) The Mariam Palace
Q30. The capital of the Bahamani Kings was Ans: (b)
(a) Gulbarga (b) Bijapur Q39. Where is the Bada Imambara located?
(c) Belgaum (d) Raichur (a) Agra (b) Lucknow
Ans: (a) (c) Patna (d) Allahabad
Q31. The city of Dhillika (Delhi) was founded by Ans: (b)
(a) Chauhans (b) Tomars Q40. Who amongst the following was the Last Guru
(c) Pawars (d) Pratiharas of the Sikhs ?
Ans: (b) (a) Guru Arjun Dev
Q32. Krishnadevaraya main -tained freindly relations (b) Guru Teg Bahadur
with the (c) Guru Gobind Singh
(a) French (b) British (d) Guru Angad Dav
(c) Portuguese (d) Dutch Ans: (c)
Ans: (c) Q41. Qutab Minar, as we find at present, was finally
Q33. Which of the following was built by Akbar to re-built by
commemorate his conquest of Khandesh in Gujarat? (a) Balban
(a) Bada Imambara (b) Ala-ud-din Khilji
(b) Buland Darwaza (c) Sikandar Lodi
(c) Jama Masjid (d) Firoz Tughluq
(d) Siddi Bashir Ans: (d)
Ans: (b) Q42. In which of the following towns is ―Moti Masjid‖
Q34. The remains of the Great Vijayanagar Empire situated ?
can be found in (a) Agra (b) Jaipur
(a) Bijapur (b) Golconda (c) Lahore (d) Ahmedabad
(c) Hampi (d) Baroda Ans: (a)
Ans: (c) Q43. Find out the correct match of the following :
Q35. The famous city of Bhopal was founded by the (a) Asaf Khan — Akbar
Rajput ruler (b) Adam Khan — Akbar
(a) Prithviraj Chauhan (c) Bairam Khan — Akbar
(b) Dharmapala (d) Shaista Khan — Akbar
(c) Raja Bhoja Ans: (c)
(d) Jaichand Q44. Which of the following Mu ghal emperors is
Ans: (c) credited with composition of Hindi songs ?
Q36. Razia Sultan, the first woman to sit on the (a) Babar (b) Akbar
throne of Delhi, was the daughter of Sultan (c) Jahangir (d) Shahjehan
(a) Mohammed Ghori Ans: (c)
(b) Mohammed of Ghazni Q45. Humayun had to run away from India after he
(c) Iltutmish was defeated in the battle of
(d) Alauddin Khilji (a) Panipat (b) Gogra
Ans: (c) (c) Khanwa (d) Kannauj
Q37. The innovator of the Revenue settlement during Ans: (d)
the rule of Akbar was Q46. Babur was succeeded to the Mughal throne by :
(a) Raja Mansingh (a) Sher Shah
(b) Raja Bhagwan Das (b) Akbar
(c) Raja Todarmal (c) Humayun
(d) Raja Birbal (d) Bahadur Shah
Page 306 of 516
https://telegram.me/aedahamlibra
Ans: (c)
Q47. Which one of the following monuments in Delhi
(a) 1106 A.D
(c) 1306 A.D
(b) 1206 A.D
(d) 1406 A.D
is not included as a World Heritage Site? Ans: (b)
(a) Red Fort Q56. Which is the holy book of the Sikh religion ?
(b) Humayun‘s Tomb (a) Bhagwad Gita
(c) Qutab Minar (b) Baani
(d) Jantar Mantar (c) Gurmukhi
Ans: (d) (d) Guru Granth Sahib
Q48. The ‗Sun Temple‘ of Konark is in the State of Ans: (d)
(a) Odisha Q57. Multan was named by the Arabs as
(b) Jharkhand (a) City of beauty
(c) Andhra Pradesh (b) City of wealth
(d) Madhya Pradesh (c) City of gold
Ans: (a) (d) Pink city
Q49. Bibi-Ka-Maqbara is located in India .at Ans: (c)
(a) Fatehpur Sikri Q58. Tulsidas wrote Ramcharitamanas during the
(b) Aurangabad reign of
(c) Hyderabad (a) Krishnadeva Raya
(d) Jaunpur (b) Akbar
Ans: (b) (c) Rama Raya
Q50. Who among the following introduced the (d) Jehangir
Mansabdari system? Ans: (b)
(a) Shah Jashan (b) Sher Shah Q59. Who was the founder of Lodhi dynasty ?
(c) Akbar (d) Jahangir (a) Daulat Khan Lodhi
Ans: (c) (b) Sikandar Lodhi
Q51. Who among the following was known as the (c) Bahlol Lodhi
'Parrot of India? (d) Ibrahim Lodhi
(a) Hussain Shah Ans: (c)
(b) Amir Khusrau Q60. The greatness of Shershah lies in his :
(c) BarbakShah (a) Secular attitude
(d) Nanak (b) Victories against Humayun
Ans: (b) (c) Superior generalship
Q52. The saviour of the Delhi Sultanate was (d) Administrative reforms
(a) Qutub-ud-din Aibak Ans: (d)
(b) Minas-us-Siraj Q61. Which of the following was the founder of the
(c) Iltutmish house of Peshawar?
(d) Ghias-ud-din Balban (a) Ramachandra Pant
Ans: (d) (b) Balaji Vishwanath
Q53. The famous Sun Temple at Konark was built by (c) Balaji Baji Rao
(a) Prataparudra (d) Parsuram Triamsuk
(b) Anantavarman Ans: (b)
(c) Narasimha–I Q62. In which of the Round Table Conference
(d) Narasimha–II Mahatma Gandhi participated?
Ans: (c) (a) First Round Table Conference, 1930
Q54. Who succeeded Guru Nanak ? (b) Second Round Table Conference, 1931
(a) Guru Angad (c) Third Round Table Conference, 1932
(b) Guru Ramdas (d) All of the above
(c) Guru Arjan Ans: (b)
(d) Guru Hargobind Q63. Who built ‗Adhai Din Ka Jhopra‘ or ‗A hut of
Ans: (a) two and a half days‘ at Ajmer?
Q55. When the rule of the Delhi Sultan began ? (a) Qutbuddin Aibak
Page 307 of 516
(b) Balban
(c) Alauddin Khalji
https://telegram.me/aedahamlibra
(d) Masulipatnam
Ans: (a)
(d) Muhammad-bin-Tughlaq Q72. What is Gol Gumbadh ?
Ans: (a) (a) Mausoleum of Hyder Ali
Q64. Amir Khusro was a (b) Mausoleum of Aurangazeb
(a) poet (b) play writer (c) Mausoleum of Chand Bibi
(c) painter (d) architect (d) Mausoleum of Mohammed Adil Shah
Ans: (a) Ans: (d)
Q65. Who was the elder broth er of Shah Shuja and Q73. Who among the following was the first to make
Murad Baksh ? use of artillery in warfare in medieval India?
(a) Aurangzeb (a) Babur
(b) Azam Shah (b) Ibrahim Lodi
(c) Muhammad Kam Baksh (c) Sher Shah Suri
(d) Dara Shikoh (d) Akbar
Ans: (d) Ans: (a)
Q66. Which one of the following painters of Q74. Bahadur Shah (First) was born in the year
Jahangir‘s reign was conferred the title of Nadir -ul- _____.
Asra‘? (a) 1543 (b) 1643
(a) Bishandas (b) Mansur (c) 1743 (d) 1843
(c) Manohar (d) Daulat Ans: (b)
Ans: (b)
Q67. The Grand Trunk Road built by Sher Shah Suri
connects:
Modern India
(a) Agra–Punjab
(b) Punjab–East Bengal Q75. Who was the leader of the Bardoli Satyagraha ?
(c) Lahore–East Bengal (a) Dr. Rajendra Prasad
(d) Multan–Agra (b) Pandit Jawaharlal Nehru
Ans: (c) (c) Sardar Vallabh Bhai Patel
Q68. Abdul Fazal was the son of which Sufi saint? (d) Acharya J. B. Kripalani
(a) Sheikh Mubarak Ans: (c)
(b) Hazarat Khwaja Q76. What was the basis of transfer of power to India
(c) Nasiruddin Chirag on 15th August by ?
(d) Baba Qutubuddin Bakhtiyar Kaki (a) On this day the Indian National Congress had
Ans: (a) demanded ―Poorna Swaraj‖.
Q69. Which of the following writers has called (b) On this day Mahatma Gandhi started ‗Quit India
Akbar‘s Din-i-Ilahi as a monument of his folly, not of Movement‘.
wisdom? (c) Anniversary of formation of Interim Government
(a) Badayuni (d) Anniversary of the surrender of Japanese army before
(b) Vincent Smith Admiral Mountbatten
(c) Barni Ans: (d)
(d) W. Haig Q77. Mahatma Gandhi got his inspiration for Civil
Ans: (b) Disobedience from :
Q70. Which dynasty was started by Khizr Khan ? (a) Tuoreau (b) Ruskin
(a) The Sayyids (b) The Lodhis (c) Confucius (d) Tolstoy
(c) The Rajputs (d) The Khiljis Ans: (a)
Ans: (a) Q78. What was the ultimate goal of Mahatma
Q71. Where in India was the first French factory Gandhi‘s Salt Satyagraha?
established? (a) repeal of Salt Satyagraha
(a) Surat (b) curtailment of the Government‘s power
(b) Pondicherry (c) economic relief to the common people
(c) Chandannagore
Page 308 of 516
(d) ‗Purna Swaraj‘ for India https://telegram.me/aedahamlibra
Ans: (d)
(d) Haripura Congress Conference, 1938
Ans: (c)
Q79. One time associate of Mahatma Gandhi, broke Q87. Who among the following controlled maximum
off from him and launched a radical movement called trade in the western coastal region during 17th
‗self-respect movement‘. Who was he ? century ?
(a) P. Thyagaraja Shetti (a) Portuguese
(b) Chhatrapati Maharaj (b) Dutch
(c) E.V. Ramaswamy Naicker (c) The house of Jagat Seth
(d) Jyotirao Govindrao Phule (d) Mulla Abdul Gaffar
Ans: (c) Ans: (a)
Q80. Who led the Salt Satyagraha Movement with Q88. The ‗Doctrine of Lapse‘ was first applied to the
Gandhi? Princely State of
(a) Annie Besant (a) Satara (b) Jhansi
(b) Mridula Sarabhai (c) Oudh (d) Jaunpur
(c) Muthu Lakshmi Ans: (a)
(d) Sarojini Naidu Q89. Apart from the Quit India Movement which
Ans: (d) started on 9th August 1942, what other sensational
Q81. When was the first train steamed off in India ? activity of the freedom fighters was done on 9t h
(a) 1848 (b) 1853 August?
(c) 1875 (d) 1880 (a) Salt Satyagraha
Ans: (b) (b) Boycott of Simon Commission
Q82. When was first telegraph line started in India ? (c) Champaran Satyagraha
(a) 1851 (b) 1875 (d) Kakori Mail train ―robbery‖
(c) 1884 (d) 1900 Ans: (d)
Ans: (a) Q90. The Muslim League advocated a separate
Q83. Who was the Chairman of the Union Powers Muslim State
Committee of the Constituent Assembly of India? (a) At its birth in 1906
(a) Sardar Vallabhbahi Patel (b) During the Khilafat Movement
(b) Dr. B.R. Ambedkar (c) In 1930, when it opposed the Civil Disobedience
(c) Sir Alladi Krishnaswami Ayyar Movement
(d) Pt. Jawaharlal Nehru (d) At the Lahore Session of 1940
Ans: (d) Ans: (d)
Q84. The original name of Swami Dayananda Q91. ‗Do or Die‘ is the famous slogan given by
Saraswati was (a) Mahatma Gandhi
(a) Abhi Shankar (b) Vallabhbhai Patel
(b) Gowri Shankar (c) Jawaharlal Nehru
(c) Daya Shankar (d) Rajiv Gandhi
(d) Mula Shankar Ans: (a)
Ans: (d) Q92. Who spoke : ―At the stroke of midnight, when
Q85. Who introduced the perma -nent settlement in the world sleeps, India awakes to life and freedom‖ ?
Bengal ? (a) Netaji Subhas Chandra Bose
(a) Lord Cornwallis (b) Mahatma Gandhi
(b) Lord Dalhousie (c) Jawaharlal Nehru
(c) William Bentinck (d) C. Rajagopalachari
(d) Lord Curzon Ans: (c)
Ans: (a) Q93. Which town/city in India has got a tower
Q86. In which session of Indian National Congress the (minaar) named after Muhammad Ali Jinnah ?
tricolour flag was unfurled for the first time ? (a) Mumbai (b) Aligarh
(a) Calcutta Session, 1920 (c) Calicut (d) Guntur
(b) Annual Session of Congress at Nagpur, 1920 Ans: (d)
(c) Lahore Congress, 1929
Page 309 of 516
answer :
https://telegram.me/aedahamlibra
Q94. Match List -I with List -II and select the correct Q100. The Simon Commission which came to Indi a in
February 1928 was boycotted because
List-I (a) all its members were Englishmen
A. Lord Clive (b) at that time a large number of nationalist leaders were
B. Lord Wellesley in prison
C. Lord Dalhousie (c) the chairman, Sir John Simon was very unpopular
D. Lord Curzon List-II (d) the Montague-Chelmsford Reforms had failed
1. Subsidiary Alliance Ans: (a)
2. Indian Universities Act Q101. Who is generally acknowledged as the pioneer
3. Doctrine of Lapse of local self-government in modern India ?
4. Dual Government in Bengal (a) Ripon (b) Mayo
(a) A-2, B-3, C-4, D-1 (c) Lytton (d) Curzon
(b) A-4, B-1, C-3, D-2 Ans: (a)
(c) A-4, B-3, C-2, D-1 Q102. The ‗Mohammadan Anglo-Oriental College‘
(d) A-1, B-4, C-2, D-3 later became the
Ans: (b) (a) Osamania University
Q95. Who is called the ‗Father of the Indian National (b) Jamia-Milia Muslim University
Congress‘? (c) Baraktullah University
(a) Mahatma Gandhi (d) Aligarh Muslim University
(b) A.O. Hume Ans: (d)
(c) Lokmanya Tilak Q103. The immortal national song Bande Mataram
(d) Surendra Nath Banerjee has been written by
Ans: (b) (a) Rabindranath Tagore
Q96. Match the following (b) Sarat Chandra Chattopadhyaya
A. Brahmo Samaj 1. Bombay (c) Bankim Chandra Chattopadhyaya
B. Veda Samaj 2. Bengal (d) Surendranath Bandopadhyaya
C. Arya Samaj 3. Madras Ans: (c)
D. Prarthana 4. North Samaj India A B C D Q104. Gandhiji considered Khadi as a symbol of
(a) 1 3 2 4 (a) industrialisation
(b) 3 2 4 1 (b) economic independence
(c) 2 4 1 3 (c) economic growth
(d) 2 3 4 1 (d) moral purity
Ans: (d) Ans: (b)
Q97. The Governor-General of India who initiated the Q105. Sardar Vallabhbhai Patel was equated with
introduction of English in India was— (a) Mazzini (b) Cavour
(a) Lord Curzon (c) Garibaldi (d) Bismarck
(b) Lord Macaulay Ans: (d)
(c) Lord Bentinck Q106. The Marathas were defeated at Panipat
(d) Lord Hastings because
Ans: (c) (a) The Marathas did not fight bravely
Q98. Cabinet Mission came to India in the year (b) The Marathas were not equal to Afghans in strength
(a) 1946 (b) 1945 (c) The Martha army was short of food supplies
(c) 1942 (d) 1940 (d) The Marathas were considered alien by the local
Ans: (a) population
Q99. From which of the following Upanishads the Ans: (b)
words ‗Satyameva Jayate‘ inscribed in Devanagari Q107. With which ‗Movement‘, the following
Script below the abacus of the State Emblem are ? were/are associated?
(a) Prashna (b) Mundaka List-I
(c) Mandukya (d) Ishavasya a. Vinoba Bhave
Ans: (b) b. Medha Patkar
c. Sunderlal Bahuguna
Page 310 of 516
d. Jaya Prakash Narayan List-II
1. ‗Chipko‘
https://telegram.me/aedahamlibra
(d) Gandhiji
Ans: (b)
2. ‗Sampurna Kranti‘ Q115. Who attended the Congress of Oppressed
3. ‗Narmada Bachao‘ Nationalists at Brussels in 1927, on behalf of the
4. ‗Bhoodan‘ National Congress ?
(a) a – 4, c – 1, b – 2, d – 3 (a) Jawaharlal Nehru
(b) a – 4, b – 3, c – 1, d – 2 (b) Mahatma Gandhi
(c) b – 3, c – 1, a – 2, d – 4 (c) Dr. Ansari
(d) d – 2, a – 4, b – 1, c – 2 (d) Motilal Nehru
Ans: (b) Ans: (a)
Q108. The Home Rule League was started by Q116. The ―Arya Samaj‖ was founded by
(a) M.K. Gandhi (b) B.G. Tilak (a) Swami Dayananda Saraswati
(c) Ranade (d) K.T. Telang (b) Swami Vivekananda
Ans: (b) (c) Keshav Chandra Sen
Q109. The Revolt of 1857 was started by (d) Ishwar Chandra Vidya-sagar
(a) the Sepoys Ans: (a)
(b) the Zamindars Q117. Which of the following reform movements was
(c) the Peasants the first to be started in the 19th century?
(d) the Plantation Workers (a) Prarthana Samaj
Ans: (a) (b) Brahmo Samaj
Q110. Which of the following was established by B.R. (c) Arya Samaj
Ambedkar ? (d) Rama Krishna Mission
(a) Swaraj Party Ans: (b)
(b) Samaj Samata Party Q118. Provincial Autonomy was one of the important
(c) All India Scheduled Castes Federation features of the Act of
(d) The Independent Labour Party (a) 1935 (b) 1919
Ans: (c) (c) 1909 (d) 1858
Q111. In which state was the first Non- Ans: (a)
Congress Government set up in independent India? Q119. The Cabinet Mission came to India in
(a) Punjab (b) Bihar (a) 1943 (b) 1944
(c) Maharashtra (d) Kerala (c) 1945 (d) 1946
Ans: (d) Ans: (d)
Q112. ―Long years ago we made a tryst with destiny, Q120. The issue on which the Civil Disobedience
and now the time comes when we shall redeem our Movement of 1930 was launched was
pledge.....‖ This was stated on the night of August 14, (a) equal employment opportunities for Indians
1947 by (b) the proposed execution of Bhagat Singh
(a) Dr. B.R. Ambedkar (c) salt monopoly exercised by the British Government
(b) C. Rajagopalachari (d) complete freedom
(c) Jawaharlal Nehru Ans: (c)
(d) Dr. Rajendra Prasad Q121. Who declared as his ultimate aim the wiping of
Ans: (c) every tear from every eye?
Q113. ‗Dyarchy‘ was introduced in the Government (a) Jawaharlal Nehru
of India Act of (b) Gandhiji
(a) 1909 (b)1919 (c) Bal Gangadhar Tilak
(c) 1935 (d)None of these (d) Sardar Patel
Ans: (b) Ans: (b)
Q114. Who propounded the theory of ‗Economic Q122. Bhulabhai Desai‘s most memorable
Drain of India‘ during British imperialism? achievement was his defence of the Indian National
(a) W.C. Bannerji Army (I.N.A.) personnel at the Red Fort Trial
(b) Dadabhai Naoroji towards the end of
(c) Gopalkrishna Gokhale (a) 1943 (b) 1944
Page 311 of 516
(c) 1945 https://telegram.me/aedahamlibra
(d) 1946
Ans: (c)
(b) Lord Dalhousie
(c) Lord Bentinck
Q123. Mahatma Gandhi‘s remark, ―A post-dated (d) Lord Curzon
cheque on a crumbling bank‖ is regarding the Ans: (b)
proposals of Q131. The decline of Indian Handicrafts industry in
(a) Simon Commission the 19th century was attributed to
(b) Cripps Mission (a) compe tition from British manufacturing industries
(c) Cabinet Mission only
(d) Wavel Plan (b) disappearance of Indian Princely Courts only
Ans: (b) (c) establishment of alien rule only
Q124. During whose Veceroyship did the High Courts (d) All of the above
come into existence at the three presidential cities of Ans: (d)
Calcutta, Madras and Bombay ? Q132. Name the important French possession in
(a) Warren Hastings India.
(b) Lord Cornwallis (a) Goa (b) Pondicherry
(c) John Lawrence (c) Daman (d) Cochin
(d) Lord Dalhousie Ans: (b)
Ans: (c) Q133. According to Gandhiji, which of the following
Q125. Who was the founder -editor of the famous are the major means of Satyagraha ?
newspaper ‗Kesari‘ during the National Struggle ? (a) Non-cooperation
(a) Mahatma Gandhi (b) Strike
(b) Jawaharlal Nehru (c) Demonstration
(c) Lokmanya Tilak (d) Civil disobedience
(d) Muhammad Iqbal (a) a and b are correct
Ans: (c) (b) a and d are correct
Q126. Which of the following, according to Mahatma (c) b and d are correct
Gandhi, is the strongest force in the world? (d) c and d are correct
(a) Non-violence of the brave Ans: (b)
(b) Non-violence of the weak Q134. On imprisonment in 1908 by the Brities, Bal
(c) Non-violence of the coward Gangadhar Tilak was sent to
(d) Non-violence of the downtrodden (a) Andaman and Nicobar
Ans: (a) (b) Rangoon
Q127. According to Dadabhai Naoroji ‗Swaraj‘ meant (c) Singapore
(a) Complete independence (d) Mandalay
(b) Self government Ans: (d)
(c) Economic independence Q135. The System of Dyarchy was introduced in India
(d) Political independence in
Ans: (b) (a) 1909 (b) 1935
Q128. The Quit India Movement was launched in (c) 1919 (d) 1945
1942 in the month of Ans: (c)
(a) January (b) March Q136. Simon Commission was boycotted by the
(c) August (d) December nationalist leaders of India because :
Ans: (c) (a) they felt that it was only an eyewash
Q129. The pledge for ―Poorna Swaraj‖ was taken at (b) all the members of the Commission were English
the Congress Session of (c) the members of the Commission were biased against
(a) Calcutta (b) Lahore India
(c) Allahabad (d) Madras (d) it did not meet the demands of the Indians
Ans: (b) Ans: (b)
Q130. Which Governor General is associated with Q137. Through which principle/device did Mahatma
Doctrine of Lapse? Gandhi strive to bridge economic inequalities?
(a) Lord Ripon (a) Abolition of machinery
Page 312 of 516
(c) Adoption of non-violence
https://telegram.me/aedahamlibra
(b) Establishment of village industries (c) Spencer
(d) Lord Warren Hastings
(d) Trusteeship theory Ans: (d)
Ans: (d) Q146. Swami Dayanand Saraswati established the
Q138. The Governor -General who abolished the first Arya Samaj in 1875 at
practice of Sati was (a) Bombay (b) Lahore
(a) Dalhousie (c) Nagpur (d) Ahmadnagar
(b) Ripon Ans: (a)
(c) William Bentinck Q147. What was not true about Sir Syed Ahmad
(d) Curzon Khan?
Ans: (c) (a) He founded the Mohamma dan Anglo Oriental
Q139. Who declared ―Swaraj is my birth right and I College at Aligarh
shall have it‖? (b) He was a leader of the Muslim League
(a) Gopal Krishna Gokhale (c) He was a leader of the Aligarh Movement
(b) Bal Gangadhara Tilak (d) He opposed the Indian National Congress
(c) Lala Lajpat Rai Ans: (b)
(d) K.T. Telang Q148. The first newspaper which was published in
Ans: (b) India was
Q140. The Indian National Army (Azad Hind Fauj) (a) The Calculatta Gazette
fought in the Second World War against– (b) The Calculatta Gazette
(a) Germany (b) Japan (c) The Oriental Magazine of Calcutta
(c) Italy (d) Great Britain (d) The Bengal Gazette
Ans: (d) Ans: (d)
Q141. Tricolour was adopted as the Nation al Flag Q149. Who created the Madras Presidency as it
in— existed till Indian Independence?
(a) Lahore Congress (a) Sir Thomas Munro
(b) Belgaum Congress (b) Lord Hastings
(c) Allahabad Congress (c) Lord cornwallis
(d) Haripura Congress (d) Lord Welliesley
Ans: (a) Ans: (a)
Q142. Who was the Congress President during 1940 - Q150. Who led the armed raid on the government
46? armoury at Chittagong in 1930?
(a) Jawaharlal Nehru (a) Chandra Shekhar Azad
(b) Sardar Vallabhbhai Patel (b) Bhagat Singh
(c) Maulana Azad (c) Surya Sen (d) Sukhdev
(d) Rajendra Prasad Ans: (c)
Ans: (c) Q151. The slogan of Quit India Movement was given
Q143. The joint session of the Congress and Muslim by
League was held in 1916 at— (a) Sardar Patel
(a) Delhi (b) Kanpur (b) Mahatma Gandhi
(c) Lucknow (d) Madras (c) Subhash Chandra Bose
Ans: (c) (d) Pt. Jawaharlal Nehru
Q144. Who among the following, is acknowledged as Ans: (b)
the pioneer of Local Self-Government in India? Q152. Gandhiji opposed the Communal Award
(a) Rippon (b) Mayo because he thought this would bring
(c) Lytton (d) Curzon (a) communal disunity
Ans: (a) (b) division in the Hindu Society
Q145. Who, among the following, abolished the Dual (c) economic miseries to India
System of Government in Bengal? (d) destruction to handi-crafts
(a) Lord Clive Ans: (b)
(b) Lord Cornwallis
Page 313 of 516
https://telegram.me/aedahamlibra
Q153. Which of the following movements was NOT
led by Mahatma Gandhi?
(c) Sardar Vallabhbhai Patel
(d) Jamnalal Bajaj
(a) Champaran Satyagraha Ans: (c)
(b) Wahabi Movement Q162. The man behind the first railway line in India
(c) Non-Cooperation Movement was:
(d) Civil Disobedience Movement (a) William Dudley
Ans: (b) (b) Roger Smith
Q154. The Arya Samaj is against (c) George Clark
(a) existence of God (d) Warren Hastings
(b) rituals and idol-worship Ans: (c)
(c) Hinduism Q163. The Communal Awared of 1932, gave separate
(d) Islam representation to
Ans: (b) (a) Harijans (b) Muslims
Q155. When and where did the Theosophical Society (c) Sikhs (d) Christians
establish its headquarters in India? Ans: (a)
(a) 1882-Adyar (b) 1885-Belur Q164. Subhash Chandra Bose set up the provisional
(c) 1890-Avadi (d) 1895-Vellore Government of Free India in
Ans: (a) (a) Burma (b) Singapore
Q156. Gandhi started the ‗Dandi March‘ from (c) Thailand (d) Indonesia
(a) Ahmedabad (b) Allahabad Ans: (b)
(c) Dandi (d) Calcutta Q165. Who gave the solgan – ‗Jai Hind‘?
Ans: (a) (a) Subhash Chandra Bose
Q157. ―India House‖ is located in (b) Jawahar Lal Nehru
(a) New Delhi (b) Kolkata (c) Moti Lal Nehru
(c) London (d) New York (d) Mahatma Gandhi
Ans: (c) Ans: (a)
Q158. Find the correct match: Q166. Who was the pioneer of the western system of
I II eduction in India ?
A. Abdul Gaffar Khan 1. Mahatma (a) Dayanand Saraswati
B. Dadabhai NaoroJi 2. Frontier Gandhi (b) Lala Lajpat Rai
C. Mohandas Karam - 3. Grand Old chand Gandhi dia (c) Raja Ram Mohan Roy
Man of ln (d) Surendranath Bannerjee
D. Rabindra Nath Tagore 4. Gurudev Ans: (c)
(a) A-3, B-1, C-4, D-2 Q167. Who is associated with the term ‗Loknayak‘ in
(b) A-2, B-3, C-1, D-4 India ?
(c) A-4, B-1, C-2, D-3 (a) Lala Lajpat Rai
(d) A-2, B-1, C-3, D-4 (b) Bal Gangadhar Tilak
Ans: (b) (c) Jay Prakash Narayan
Q159. Where did Mahatma Gandhi first apply his (d) Madan Mohan Malviya
technique of Satyagraha? Ans: (c)
(a) Dandi (b) Champaran Q168. The Swarajya Party was formed following the
(c) England (d) South Africa failure of
Ans: (d) (a) Non-Cooperation Movement
Q160. The Indian Natio nal Congress had its first (b) Civil Disobedience Movement
meeting in the city of (c) Quit India Movement
(a) Calcutta (b) Bombay (d) Champaran Satyagraha
(c) Ahmedabad (d) Allahabad Ans: (a)
Ans: (b) Q169. Gandhiji‘s movement of boycotting the foreign
Q161. Bardoli Satyagraha is associated with goods aimed at
(a) Rajendra Prasad (a) full independence
(b) Acharya Vinoba Bhave (b) creating anti-British sentiment
Page 314 of 516
(c) promotion of welfare state https://telegram.me/aedahamlibra
(d) promotion of cottage industries
Q177. Which Party was established by Subhash
Chandra Bose after he came out of Indian National
Ans: (d) Congress ?
Q170. According to Ma hatma Gandhi, which of the (a) Indian National Army
following is correct? (b) Republican Party
(a) Religion is to be separated from politics. (c) Forward Block
(b) Politics and religion must go hand in hand. (d) Socialist Party
(c) Religion should be completely banned. Ans: (c)
(d) Politics must be completely eliminated from socio - Q178. Which movement got the support both from
economic life. Hindus and Muslims?
Ans: (b) (a) Non Cooperation Movement
Q171. The Governor General of India impeached by (b) Quit India Movement
the House of Commons in England was: (c) Champaran Satyagraha
(a) Waren Hastings (d) Anti-Partition Movement
(b) Cornwallis Ans: (a)
(c) Wellesley Q179. Who was the first Muslim President of Indian
(d) William Bentinct National Congress ?
Ans: (a) (a) Badruddin Tyabji
Q172. The last Governor -General of the East India (b) Maulana Abul Kalam Azad
Company ,and the first Viceroy under the crown was (c) Hassan Imam
(a) Lord Elgin (d) M.A.Ansari
(b) Lord Canning Ans: (a)
(c) Lord Warren Hastings Q180. Who are the three, among the following who
(d) Lord Dalhousie fought against British in 1857 Revolt ?
Ans: (b) a. Kunwar Singh
Q173. Who said that ―India‘s soul lives in villages‖ ? b. Tantia Tope
(a) Vinoba Bhave c. Nana Saheb
(b) Jayaprakash Narayan d. Maulavi Ahmudullah
(c) Jawahar Lal Nehru (a) a, b and c (b) a, c and d
(d) Mahatma Gandhi (c) a, b and d (d) b, c and d
Ans: (d) Ans: (*)
Q174. Who among the following established the Q181. The Portuguese G overnor who abolished Sati
Ghadar Party ? in Goa was
(a) V.D. Savarkar (a) Albuquerque (b) Cabral
(b) Bhagat Singh (c) Almeida (d) De Braganza
(c) Lala Hardayal Ans: (a)
(d) Chandrashekhar Azad Q182. Who was the Nawab when Dalhousie annexed
Ans: (c) Awadh in 1856 ?
Q175. The ‗Poorna Swaraj‘ resolution was adopted in (a) Nasiruddin Mahmud Shah
the annual session of the Indian National Congress (b) Wajid Ali Shah
held at (c) Shujaud Daula
(a) Bombay (b) Lahore (d) Alivardi Khan
(c) Calcutta (d) Madras Ans: (b)
Ans: (b) Q183. Who termed Cripps‘ proposals as ‗a post dated
Q176. Who among the following was the founder of cheque in a crashing bank‘ ?
the Dravida Kazhagam ? (a) Ambedkar (b) Annie Besant
(a) Periyar E.V. Ramaswamy Naicker (c) Patel (d) Gandhiji
(b) Acharya Vinoba Bhave Ans: (d)
(c) C. Rajagopalachari Q184. Who said about Mahatma Gandhi that he is a
(d) Mahatma Gandhi ―half naked fakir‖ ?
Ans: (a) (a) Winston Churchill
Page 315 of 516
(b) Lord Mountbatten
(c) Lord Wavell
https://telegram.me/aedahamlibra
(c) Sardar Bagat Singh
(d) Rukmani Lakshmipath
(d) Lord Linlithgow Ans: (c)
Ans: (a) Q193. In 1937, an education al conference endorsing
Q185. Who gave the title of ‗Sardar‘ to Vallabhbhai Gandhi‘s proposals for ‗basic education‘ through the
Patel ? vernacular medium was held at
(a) Rajaji (b) Gandhiji (a) Surat (b) Bombay
(c) Nehru (d) M.A. Jinnah (c) Ahmedabad (d) Wardha
Ans: (b) Ans: (d)
Q186. Who among the following is known as the Q194. The idea of federation was first proposed in
―Father of the Indian Renaissance‖ ? (a) Indian Councils Act of 1892
(a) Raja Ram Mohan Roy (b) Morley-Minto Reforms
(b) Rabindra Nath Tagore (c) Montague-Chelmsford Reforms
(c) Swami Dayanand Saraswati (d) Nehru Report of 1928
(d) Swami Vivekanand Ans: (d)
Ans: (a) Q195. Gandhi‘s concept of Trusteeship:
Q187. In which year did the Indian National Congress (a) Recognises right of private ownership of property
split between moderates and extremists ? (b) Transforms the capitalistic society into an egalitarian
(a) 1907 (b) 1908 one
(c) 1909 (d) 1910 (c) Excludes legi slative regulation of the ownership and
Ans: (a) use of wealth
Q188. Who f ounded the Indian National Party in (d) Does not fix minimum or maximum income
Berlin during 1914? Ans: (b)
(a) Subhash Chandra Bose Q196. Which one of the following wars decided the
(b) W.C. Banerjee fate of the French in India ?
(c) Surendranath Banerjee (a) Battle of Wandiwash
(d) Champakaraman Pillai (b) First Carnatic War
Ans: (d) (c) Battle of Buxar
Q189. What Satyagraha was held at Nagpur in 1923? (d) Battle of Plassey
(a) Salt Satyagraha Ans: (a)
(b) Individual Satyagraha Q197. Which one of the following novels was a source
(c) Ryots Satyagraha of inspiration for the freedom fighters in India?
(d) Flag Satyagraha (a) Pariksha Guru
Ans: (d) (b) Anandmath
Q190. Who gave the title of ―Mahamana‖ to Madan (c) Rangbhoomi
Mohan Malviya ? (d) Padmarag
(a) Dada Bhai Naurozi Ans: (b)
(b) Gopal Krishna Gokhale Q198. Who among the following organised the ―All
(c) Mahatma Gandhi India D epressed Classes Association.‖ in colonial
(d) Bal Gangadhar Tilak India ?
Ans: (c) (a) M.K.Gandhi
Q191. Rowlatt Act 1919 was enacted during t he (b) Jyotiba Phule
period of (c) Pandita Ramabai
(a) Lord Chelmsford (d) B.R. Ambedkar
(b) Lord William Ans: (d)
(c) Lord Minto Q199. The Ramakrishna Mission was founded by
(d) Lord Bentinck (a) Dayanand Saraswati
Ans: (a) (b) Keshab Chandra
Q192. Who founded the Naujawan Bharat Sabha ? (c) Ram Mohari Roy
(a) B.C.Pal (d) Vivekananda
(b) G. Subramania Iyer Ans: (d)
Page 316 of 516
―Azad Hind Fauj‖ ?
https://telegram.me/aedahamlibra
Q200. Who among the following is the founder of the Q208. Raja Ram Mohan Roy was the founder of :
(a) Arya Samaj
(a) Lala Har Dayal (b) Ram Krishna Mission
(b) Subash Chandra Bose (c) Brahmo Samaj
(c) Vir Savarkar (d) Prathna Samaj
(d) Chandrashekhar Azad Ans: (c)
Ans: (b) Q209. Gandhiji‘s famous Quit India Movement call to
Q201. Find the incorrect match among the the British was given in –
(a) Sir Syed Murtza Khan – Aligarh Muslim University (a) 1943 (b) 1941
(b) Dr. B.R. Ambedkar – Ambedkar University (c) 1942 (d) 1940
(c) Pt. Jawaharlal Nehru – Jawaharlal Nehru University Ans: (c)
(d) Madan Mohan Malviya – Banaras Hindu University Q210. The only AICC session Gandhiji presided was
Ans: (a) held at :
Q202. Which Charter Act brought to an end, the East (a) Calcutta (b) Madras
India Company‘s monopoly in India‘s foreign trade ? (c) Belgaum (d) Lahore
(a) Charter Act of 1853 Ans: (c)
(b) Charter Act of 1793 Q211. Who was the founder of Swatantra Party ?
(c) Charter Act of 1813 (a) B.G.Tilak
(d) Charter Act of 1833 (b) Dadabhai Naoroji
Ans: (c) (c) Gopal Krishna Gokhale
Q203. The first woman President of Indian National (d) C. Rajagopalachari
congress was Ans: (d)
(a) Kamala Devi Chattopadhyaya Q212. Dadabhai Naroji has described his theory of
(b) Sarojini Naidu ‗Drain of Wealth‘ in the book.
(c) Annie Besant (a) Poverty and Un-British Rule in India
(d) Rajkumari Amrit Kaur (b) British Rule and its Consequences
Ans: (c) (c) Exploitative Nature of British Rule in India
Q204. The British Government intervened in the (d) Nature of British Colonial Rule
affairs of the Company and passed an Act in 1773 Ans: (a)
A.D., known as the Q213. The battle of Plassey was fought between
(a) Regulating Act (a) Sirajudduala and Robert Clive.
(b) Pitt‘s India Act (b) None of the options
(c) Charter Act (c) Mir Kasim and Robert Clive.
(d) Company Act (d) Mir Jafar and Robert Clive.
Ans: (a) Ans: (a)
Q205. Cripps Mission came to India in ______ Q214. Where did the Black-Hole tragedy took place ?
(a) 1946 (b) 1945 (a) Monghyr (b) Calcutta
(c) 1942 (d) 1940 (c) Murshidabad (d) Dacca
Ans: (c) Ans: (b)
Q206. Under whose leadership was the Chittagong Q215. Who was referred to as the ―Ambassador of
Armoury Raid organised ? Hindu-Muslim Unity‖ by Sarojini Naidu ?
(a) Sukhdev (b)Bhagat Singh (a) Muhammad Ali Jinnah
(c) Surya Sen (d) Rajguru (b) Abdul Ghaffar Khan
Ans: (c) (c) Shaukat Ali
Q207. In 1939 Subhash Chandra Bose was elected as (d) Maulana Azad
President of the Congress Party defeating Ans: (a)
(a) Pattabhi Sitharamayya Q216. The Panchsheel agreement was between India
(b) Jawaharlal Nehru and:
(c) Maulana Abul Kalam Azad (a) Russia (b) China
(d) VB Patel (c) Brazil (d) Sri Lanka
Ans: (a) Ans: (b)
Page 317 of 516
https://telegram.me/aedahamlibra
Q217. The India Independence Bill was first
presented in the House of Commons in London on: Q225. Who was the founder of Arya Samaj?
Ans: (b)

(a) August 10,1947 (a) Acharya Narendra Dev


(b) August 1,1947 (b) Dayanand Saraswati
(c) July 14,1947 (c) Raja Ram Mohan Roy
(d) July 4,1947 (d) Acharya Vinoba Bhave
Ans: (d) Ans: (b)
Q218. In which year was the Morley Minto reform Q226. Who introduced Western education?
passed? (a) Raja Ram Mohan Roy
(a) 1917 (b) 1902 (b) Lal Bahadur Shastri
(c) 1909 (d) 1912 (c) Netaji Subhash Chandra Bose
Ans: (c) (d) Sardar Vallabhbhai Patel
Q219. ―Earth provides enough to satisfy everyman‘s Ans: (a)
need but not everyman‘s greed‖. Who said this? Q227. The Swadeshi Boycott Movement is related to
(a) Guru Nanak Dev (a) Partition of Bengal in 1947
(b) Mahatma Gandhi (b) Partition of Bengal in 1905
(c) Pope Paul VI (c) Non cooperation movement in 1921
(d) Smt. Indira Gandhi (d) Partition of Punjab in 1947
Ans: (b) Ans: (b)
Q220. The ‗Blue Water Policy‘ was introduced by the Q228. The first Mu slim to be elected President of
Portugese leader ‗Indian National Congress‘ was?
(a) Franscisco-de-Almeida (a) Maulana Azad
(b) Alfonso de Albuquerque (b) Mohammed Ali
(c) Francis Caron (c) Badruddin Tyabji
(d) Francis Martin (d) Shah Waliullah
Ans: (a) Ans: (c)
Q221. Which one of the following party was founded Q229. Who was the first Indian to be elected as the
by Subhash Chandra Bose? leader of the Communist International?
(a) Abhinav Bharat (a) Jawaharlal Nehru
(b) Azad Hind Sena (b) P.C. Joshi
(c) Revolutionary Army (c) Sardar Vallabhai Patel
(d) Forward Block (d) M.N Roy
Ans: (d) Ans: (d)
Q222. Indian National Congress split for the first time Q230. British achieved political power in India after
in its session at which of the following ?
(a) Allahabad (b) Calcutta (a) Battle of Plassey
(c) Surat (d) Lahore (b) Battle of Panipat
Ans: (c) (c) Battle of Buxar
Q223. Which of the following was published by (d) Battle of Wandiwash
Gandhiji during his stay in South Africa? Ans: (a)
(a) Young India Q231. What was the Wood‘s Despatch about ?
(b) Indian Opinion (a) Industry (b) Army
(c) Nav Jivan (c) Education (d) Agriculture
(d) None of these Ans: (c)
Ans: (b) Q232. Motilal Nehru and C.R. Das were the founders
Q224. What was the immediate cause of the Mutiny of of a party known as the :
1857? (a) Gadar Party
(a) The discontentment of the soldiers (b) Forward Bloc
(b) The use of greased cartridges in the new Enfield Rifle (c) Swarajya Party
(c) The social conditions of India (d) Socialist Congress
(d) The introduction of railways and telegraphs Ans: (c)
Page 318 of 516
the Gadar party?
https://telegram.me/aedahamlibra
Q233. One of the following was not associated with Ans: (c)
Q241. Who discovered sea route to India via the
(a) Lala Hardayal ‗Cape of Good Hope‘?
(b) Baba Gurdit Singh (a) Vasco da Gama
(c) Mohammad Barkatullah (b) Amundsen
(d) Sohan Singh Bhakna (c) Christopher Columbus
Ans: (b) (d) John Cabot
Q234. The Bandung Conference was a major Ans: (a)
milestone in the history of : Q242. Name the Pakistan Prime -Minister who signed
(a) The Non-aligned movement the Simla Agreement in 1972.
(b) Indo-Chinese relationship (a) Ayub Khan
(c) U.S.-Vietnam War (b) Yahya Khan
(d) Creation of ASEAN (c) Z.A. Bhutto
Ans: (a) (d) Benazir Bhutto
Q235. Who was the architect of North and South Ans: (c)
Blocks of the Central Secretariat in Delhi ? Q243. The first woman to preside over the UN
(a) Sir Edward Lutyens General Assembly:
(b) Herbert Bakers (a) Rajkumari Amrit Kaur
(c) Robert Tor Tussell (b) Vijaylakshmi Pandit
(d) Antonin Raymond (c) Kamla Nehru
Ans: (a) (d) Indira Gandhi
Q236. Who was the town-planner of Chandigarh ? Ans: (b)
(a) Edward Lutyens Q244. Who built Shantiniketan?
(b) Charles Correa (a) Guru Ramdas
(c) Le Corbusier (b) Maharaja Pratap Singh
(d) Christopher Wren (c) Rabindra Nath Tagore
Ans: (c) (d) British Govt.
Q237. The first woman to become a Chief Minister of Ans: (c)
any State in India was
(a) Nandini Satpathy
(b) Dr. J. Jayalalitha
Indian Art & Culture
(c) Sucheta Kripalani
(d) Ms. Mayawati Q1. Where is ―Pushkar Fair‖ held ?
Ans: (c) (a) Jodhpur (b) Ajmer
Q238. The methods of democratic socialism are (c) Jaipur (d) Udaipur
(a) Revolution Ans: (b)
(b) General strike Q2. Which language is spoken by the people of
(c) Sabotage Lakshadweep?
(d) Persuasion and propaganda (a) Malayalam (b) Kannada
Ans: (d) (c) Tamil (d) Telugu
Q239. Which country offered its good office to resolve Ans: (a)
Indo-Pak conflict peacefully, in the year 1966? Q3. The most popular festival in Tamil Nadu is :
(a) U.S.A (b) Britain (a) Gudipadwa (b) Onam
(c) U.S.S.R (d) Japan (c) Bihu (d) Pongal
Ans: (c) Ans: (d)
Q240. Identify the UNESCO approved World Q4. Who amongst the following is renowned in the
Heritage site from the following. field of painting?
(a) Somnath Temple (a) Parveen Sultana
(b) Salarjung Museum (b) Prof. T.N. Krishnan
(c) Khajuraho group of temples (c) Ram Kinkar
(d) Chaar Bagh Railway station
Page 319 of 516
(d) Raja Ravi Varma https://telegram.me/aedahamlibra
Ans: (d)
(d) Mohiniattam
Ans: (b)
Q5. What do the paintings of Ajanta depict? Q13. Who amongst the following is renowned in
(a) Ramayana (b) Mahabharata Hindustani classical music (vocal)?
(c) Jatakas (d) Panchatantra (a) Shovana Narayan
Ans: (c) (b) M. S. Subbalakshmi
Q6. The Gandhara Art flourished during the period (c) Pt. Jasraj
of (d) M. S. Gopalakrishnan
(a)Guptas (b) Mauryas Ans: (b)
(c) Satavahanas (d) Kushans Q14. Match the artists with their art form.
Ans: (d) Artist Art form
Q7. Who painted the master piece ―Hamsa (a) Jatin Das 1. Sitar
Damayanti‖ ? (b) Parveen 2. Painting Sultana
(a) Anjolie Ela Menon (c) Pradosh 3. Hindustani Dasgupta music (Vocal)
(b) Abanindranath Tagore (d) Ustad 4. Sculpture Vilayat Khan
(c) Amrita Shergill (a) (b) (c) (d)
(d) Raja Ravi Varma (a) 1 2 3 4
Ans: (d) (b) 2 3 4 1
Q8. Who, amidst the following is a distinguished (c) 3 4 2 1
painter? (d) 4 1 3 2
(a) Amrita Shergill Ans: (b)
(b) N. Rajam Q15. Match the artists and their artform :
(c) Kamala Dasgupta Artists Artform
(d) U. Srinivas (a) Ustad (a) Bharat-
Ans: (a) Allauddin Natyam Khan
Q9. The birth place of an iconic personality, Amrita (b) Hema (b) Sarod Malini
Sher–Gil, is (c) Birju (c) Santoor Maharaj
(a) Hungary (b) India (d) Pt. Shiv (d) Kathaka Kumar dance
(c) Austria (d) Poland Sharma
Ans: (a) (a) (b) (c) (d)
Q10. The famous Dilwara temples of Mount Abu are (a) 1 3 2 4
a sacred pilgrimage place for the (b) 2 4 3 1
(a) Buddhists (b) Jains (c) 3 2 4 1
(c) Sikhs (d) Parsis (d) 2 1 4 3
Ans: (b) Ans: (d)
Q11. Kalamkari painting refers to Q16. Which in the following is a stringed instrument ?
(a) A hand painted cotton textile in SouthIndia (a) Mridangam (b) Tabla
(b) A handmade drawing on bamboo handicrafts in (c) Shehnai (d) Santoor
North- Ans: (d)
East India Q17. Guru Kelucharan Mohapatra was a maestro of
(c) A block painted woollen cloth in Western Himalayan which of the following dance forms ?
region of India (a) Kathak (b)Odissi
(d) A hand painted decorative silk cloth in North Western (c) Kuchipudi (d) Bharatnatyam
India Ans: (b)
Ans: (a) Q18. Who amongst the following is renowned in
Q12. In which of the following styles of dance the Hindustani Classical Music (vocal) ?
story/theme is always taken from Mahabharata and (a) Shovana Narayan
Ramayana ? (b) M.S. Subbalakshmi
(a) Odissi (c) Pt. Jasraj
(b) Bharatnatyam (d) M.S. Gopalakrishnan
(c) Kuchipudi Ans: (c)
Page 320 of 516
https://telegram.me/aedahamlibra
Q19. Which is the most ancient musical instrument of
India ?
(d) Hindustani (Classical-Vocal)
Ans: (c)
(a) Flute (b) Tabla Q27. Where do the striptease artists work ?
(c) Veena (d) Sitar (a) Cabarets (b) Films
Ans: (c) (c) Circus (d) Plays
Q20. In which of the following dance forms Birju Ans: (a)
Maharaj attained prominence ? Q28. Which one of the following is not a form of
(a) Bharatnatyam (b) Kuchipudi Carnatic music ?
(c) Kathak (d) Odissi (a) Kriti (b) Thillana
Ans: (c) (c) Slokam (d) Tappa
Q21. Match List -I with List -II and select the correct Ans: (d)
answer from the code given below the Lists : Q29. Who of the following is a famous Tabla player ?
List-I (Musician) (a) Zakir Hussain
a. Asad Ali Khan (b) Vikku Vinayakram
b. Mallikarjun (c) Pt. V.G. Jog
c. Vismillah Khan (d) Palghat Mani Iyer
d. T. N. Krishnan List-II (Excellence in) Ans: (a)
1. Shahnai 2. Violin Q30. What is the name of the famous folk drama of
3. Classical music 4. Vina Code : Bhils?
abcd (a) Gavari (b) Swang
(a) 2 4 3 1 (c) Tamasha (d) Rammat
(b) 4 3 1 2 Ans: (a)
(c) 3 4 2 1 Q31. ―Bhangra‖ is a dance of
(b) 1 2 3 4 (a) Haryana (b) Rajasthan
Ans: (b) (c) Delhi (d) Punjab
Q22. The ‗National School of Drama‘ is situated in Ans: (d)
which of the following cities ? Q32. In which one of the following films did Fran act
(a) Mumbai (b) New Delhi as a character artist and not in the role of a villain ?
(c) Bhopal (d) Kolkata (a) Ram aur Shyam
Ans: (b) (b) Zanjeer
Q23. Which of the following folk/tribal dances is (c) Madhumati
associated with Karnataka? (d) Himalaya Ki Godh Mein
(a) Yakshagana (b) Veedhi Ans: (b)
(c) Jatra (d) Jhora Q33. Indian first talkie film produced in 1931 was
Ans: (a) (a) Shakuntala (b) Indra Sabha
Q24. Of which of the following States is Nautanki, a (c) Neel Kamal (d) Alamara
folk dance? Ans: (d)
(a) Haryana (b) Uttar Pradesh Q34. Who directed the film on the dacoit queen
(c) Rajasthan (d) Gujarat Phoolan Devi ?
Ans: (b) (a) Abbas Mastan
Q25. ‗Tabal Chongli‘ is a form of fold dance (b) Shashi Kapoor
associated with the State of : (c) Shekhar Kapoor
(a) West Bengal (d) Anil Kapoor
(b) Assam Ans: (c)
(c) Andhra Pradesh Q35. Which of the following is responsible for
(d) Maharashtra fostering the development of dance, dra ma and music
Ans: (b) in India?
Q26. Which is the field in which Ustad Bismillah (a) Lalit Kala Akademi
Khan has distinguished himself? (b) Sangeet Natak Akademi
(a) Sitar (b) Guitar (c) National School of Drama
(c) Shenai (d) Sahitya Akademi
Page 321 of 516
https://telegram.me/aedahamlibra
Q36. Pandit Lachhu Maharaj, who passed away
Ans: (b)
Indian Polity &
recently, was associated with
(a) Flute Constitution
(b) Tabla
(c) Kathak
Q1. The system of judicial review originated in
(d) Bharatnatyam
(a) India (b) Gemany
Ans: (b)
(c) Russia (d) U.S.A.
Q37. Ebraham Alkazi is an eminent personality in
Ans: (d)
which one of the following area?
Q2. Who among the following was not a member of
(a) Heart surgery
the Consti-tuent Assembly established in July 1946 ?
(b) Civil aviation
(a) Dr. Rajendra Prasad
(c) Theatre
(b) K M Munshi
(d) Painting
(c) Mahatma Gandhi
Ans: (c)
(d) Abul Kalam Azad
Q38. Birju Maharaj is a well known exponent of
Ans: (c)
(a) Manipuri dance
Q3. Which of the following countries has introduced
(b) Kathak
―direct democracy‖?
(c) Odissi
(a) Russia (b) India
(d) Kathakali
(c) France (d) Switzerland
Ans: (b)
Ans: (d)
Q39. Ten thaats or scales, belong to which system of
Q4. Which of the following is a feature to both the
music?
Indian Federation and the American Federation ?
(a) Hindustani Music
(a) A single citizenship
(b) Folk Music
(b) Dual judiciary
(c) Chinese Music
(c) Three Lists in the Constitution
(d) Western Music
(d) A Federal Supreme Court to interpret the Constitution
Ans: (a)
Ans: (d)
Q40. Correlate the field of activites of the following
Q5. Indian Penal Code came into operation in
eminent persons and their forte :
(a) 1858 (b) 1860
Eminent person
(c) 1859 (d) 1862
(A) Thomas L. Friedman
Ans: (d)
(B) Zubin Mehta
Q6. The Constitution of India came into force on
(C) Ismat Chugtai
(a) 26 January, 1950
(D) Jamini Roy Activity
(b) 26 January, 1952
1. Literature 2. Journalism
(c) 15 August, 1948
3. Painting 4. Music A B C D
(d) 26 November, 1949
(a) 2 4 1 3
Ans: (a)
(b) 4 2 3 1
Q7. The Constitution of India contains
(c) 2 4 3 1
(a) 340 Articles
(d) 1 3 4 2
(b) 395 Articles
Ans: (a)
(c) 400 Articles
Q41. Charles Correa is a renowned Indian
(d) 404 Articles
(a) Freedom fighter
Ans: (b)
(b) Painter
Q8. Who was the President of the Constituent
(c) Architect, urban planner
Assembly?
(d) Award winning doctor
(a) Pt. Jawahar Lal Nehru
Ans: (c)
(b) Sardar Patel
(c) Dr. Rajendra Prasad
(d) Dr. B.R. Ambedkar

Page 322 of 516


https://telegram.me/aedahamlibra
Q9. The concept of Constitution first originated in
Ans: (c) (d) B.R. Ambedkar
Ans: (c)
(a) Switzerland (b) Britain Q17. Where do we find the ideals of Indian
(c) U.S.A. (d) Japan democracy in the Constitution ?
Ans: (b) (a) The Preamble (b) Part III
Q10. ―Persons may change but rules should not (c) Part IV (d) Part I
change‖ is the principle of Ans: (a)
(a) Absolute Monarchy Q18. The concept of ―Rule of Law‖ is a special feature
(b) Constitutional Government of constitutional system of
(c) Unwritten Constitution (a) Britain (b) U.S.A.
(d) Republic (c) France (d) Switzerland
Ans: (b) Ans: (a)
Q11. Direct Democracy is a system of Government in Q19. The phrase ―equality before law‖ used in
which Article-14 of Indian Constitution has been borrowed
(a) People choose the Civil Servants from _____
(b) People directly elect their Representatives. (a) U.S.A. (b) Germany
(c) People take part directly in the policy making and (c) Britain (d) Greece
administration of the country. Ans: (c)
(d) Government officials consult people on various Q20. Who among the following was not a member of
appointments. the Drafting Committee of Indian Constitution ?
Ans: (c) (a) B. R. Ambedkar
Q12. What is meant when the Constitution declares (b) Alladi Krishnaswamy
India a ―Secular State‖ ? (c) Rajendra Prasad
(a) Religious worship is not allowed (d) Gopalachari Ayyangar
(b) Religions are patronised by the State Ans: (c)
(c) The state regards reli gions as a private affairs of the Q21. The idea of parliamentary form of government is
citizen and does not discriminate on this basis adapted from
(d) None of these (a) US (b) UK
Ans: (c) (c) Ireland (d) USSR
Q13. Who was the Chairman of the Drafting Ans: (b)
Committee of the Constituent Assembly ? Q22. In which of the following Acts the territorial
(a) Dr. B. R. Ambedkar division of governance in India was done ?
(b) C. Rajagopalachari (a) Government of India Act, 1858
(c) Dr. Rajendra Prasad (b) Indian Councils Act, 1861
(d) Jawaharlal Nehru (c) Indian Councils Act, 1892
Ans: (a) (d) Government of India Act, 1861
Q14. When was the Indian Constitution adopted ? Ans: (a)
(a) 15th August, 1947 Q23. The Constitution __________.
(b) 26th November, 1949 (a) is silent on the President‘s re-election to the office.
(c) 26th January, 1950 (b) allows re-election of a person to the President‘s post
(d) 2nd October,1952 (c) restricts a person to remain President for only two
Ans: (b) terms.
Q15. Indian Constitution is : (d) has been amended to allow a person only one term as
(a) Federal (b) Quasi Federal President.
(c) Unitary (d) Presidential Ans: (b)
Ans: (b) Q24. Indian Constitution came into force on
Q16. Who described the Government of India Act, (a) 15th August, 1947
1935 as a new charter of bondage ? (b) 26th January, 1950
(a) Mahatma Gandhi (c) 26th November, 1948
(b) Rajendra Prasad (d) 6th November, 1948
(c) Pt. Jawaharlal Nehru Ans: (b)
Page 323 of 516
https://telegram.me/aedahamlibra
Q25. What was the exact constitutional position of the
Indian Republic when the Constitution was brought
(b) Supreme Court alone
(c) Both Supreme Court and High Courts
into force with effect from 26th January, 1950? (d) All Courts down to the District Courts
(a) A Democratic Republic Ans: (c)
(b) A Sovereign Democratic Republic Q33. A writ iss ued by the Supreme Court compelling
(c) A Sovereign Secular Democratic Republic a quasijudicial/ public authority to perform its
(d) A Sovereign Secular Socialist Democratic Republic mandatory duty is
Ans: (b) (a) Quo warranto
Q26. Which of these is NOT included as a (b) Mandamus
Fundamental Right in the Indian Constitution ? (c) Certiorari
(a) Right to Freedom of Speech (d) Prohibition
(b) Right to Equality before the Law Ans: (b)
(c) Right to Constitutional Remedies Q34. Which of the following expressions does not
(d) Right to equal wages for equal work figure in the Preamble to the Indian Constitution?
Ans: (d) (a) Sovereign Democratic Republic
Q27. The term ‗secular‘ was added in the Preamble to (b) Socialist
the Indian Constitution by (c) Secular
(a) 41st Amendment (d) Federal
(b) 42nd Amendment Ans: (d)
(c) 43rd Amendment Q35. How many types of writs can be issued by the
(d) 44th Amendment Supreme Court?
Ans: (b) (a) Two (b) Three
Q28. Of the following words in the Preamble of the (c) Five (d) Six
Constitution of India, which was not inserted through Ans: (c)
the Constitution (Forty Second Amendment) Act, Q36. How many Fundamental Duties are in the
1976? Indian Constitution ?
(a) Socialist (b) Secular (a) Eleven (b) Nine
(c) Dignity (d) Integrity (c) Twenty (d) Twelve
Ans: (c) Ans: (a)
Q29. Fundamental Rights in India are guaranteed by Q37. Which one of the following is the guardian of
it through Fundamental Rights?
(a) The Right to Equality (a) Legislature
(b) Right Against Exploitation (b) Executive
(c) Right to Constitutional Remedies (c) Political parties
(d) Educational and Cultural Rights (d) Judiciary
Ans: (c) Ans: (d)
Q30. Which one of the following is not en umerated as Q38. The Preamble to our Constitution makes no
a right in the Constitution of India ? mention of
(a) Political and social right (a) Justice
(b) Educational right (b) Fraternity
(c) Economic right (c) Equality of status
(d) Right to religion (d) Adult franchise
Ans: (c) Ans: (d)
Q31. How many Fundamental Duties are included in Q39. Freedom of the press is implied in the right to
Indian Constitution ? (a) equal protection of the laws
(a) Nine (b) Ten (b) freedom of speech
(c) Eleven (d) Twelve (c) freedom of association
Ans: (c) (d) work and material security
Q32. Under the Constitution, the power to issue a writ Ans: (b)
of Habeas Corpus is vested in
(a) High Courts alone
Page 324 of 516
https://telegram.me/aedahamlibra
Q40. Which one of the following fundamental rights
was described by Dr. B.R. Ambedkar as the heart and
(c) Citizen (d)Civil Society
Ans: (c)
soul of the Constitution? Q48. Directive Principles of State Policy in the Indian
(a) Right to religion Constitution were taken from the Constitution of
(b) Right to constitutional remedies (a) Britain (b) Ireland
(c) Right to property (c) USA (d) Canada
(d) Right to education Ans: (b)
Ans: (b) Q49. Directive Principles of State Policy in the Indian
Q41. No person can be employed in factories or mines Constitution were taken from the Constitution of
unless he is above the age of (a) Britain (b) Ireland
(a) 12 years (b) 14 years (c) USA (d) Canada
(c) 18 years (d) 20 years Ans: (b)
Ans: (b) Q50. According to Preamble, the ultimate power lies
Q42. Under which one of the following writs an in the hands of :
official can be prevented from taking an action which (a) Parliament (b) Constitution
he is officially not entitled? (c) President (d) People
(a) Mandamus Ans: (d)
(b) Quo Warranto Q51. In Indian Constitution, fundamental rights were
(c) Certiorari taken from the Constitution of
(d) Habeas Corpus (a) USA (b) UK
Ans: (a) (c) USSR (d) None of these
Q43. Right to vote is a Ans: (a)
(a) Political right Q52. In which part of the Indian Constitution, the
(b) Civil right fundamental duties are enshrined ?
(c) Economic right (a) IV A (b) IV B
(d) Legal right (c) V (d) IV
Ans: (a) Ans: (a)
Q44. What is the chief source of political power in Q53. Which one of the following is issued by the court
India ? in case of an illegal detention of a person?
(a) The people (a) Habeas Corpus
(b) The Constitution (b) Mandamus
(c) The Parliament (c) Certiorari
(d) The Parliament and the State Legislatures (d) Quo Warranto
Ans: (a) Ans: (a)
Q45. Which of the following rights is not granted by Q54. How many Fundamental Rights were granted
the Constitution of India at present as a fundamental initially?
right ? (a) Six (b) Seven
(a) Right to equality (c) Four (d) Five
(b) Right to freedom Ans: (b)
(c) Right to property Q55. Political liberty implies
(d) Right against exploitation (a) people‘s control over the government
Ans: (c) (b) free political activities of the people
Q46. ‗The Right to Public Office‘ is a (c) co-operation between politics and democracy
(a) Civil right (d) voters can make and unmake their government
(b) Economic right Ans: (b)
(c) Moral right Q56. The concept of Directive Principles of State
(d) Political right Policy in Indian Constitution is adopted from that of
Ans: (a) (a) Ireland and Spain
Q47. The essential feature of democracy is giving (b) US and UK
prominence to the (c) USSR and China
(a) Executive (b) Judiciary (d) Japan and Korea
Page 325 of 516
https://telegram.me/aedahamlibra
Ans: (b)
Q57. The Directive Principles of State Policy has be en Q64. From which Constitution of the World, the
Ans: (c)

adopted from which Constitution? Indian Constitution has adopted the concept of
(a) U.S. Constitution ―Directive Principles of State Policy‖?
(b) British Constitution (a) Ireland (b) U.S.A.
(c) Irish Constitution (c) Canada
(d) French Constitution (d) Australiads, tanks and wells
Ans: (c) Ans: (a)
Q58. Which of the following is not guaranteed by Q65. Who among the following pr esides over the Lok
Indian Constitution? Sabha in the absence of Speaker or Deputy Speaker of
(a) Right to Equality the Lok Sabha ?
(b) Right of religious freedom (a) A member appointed by the President.
(c) Right to Constitutional remedies (b) A member nominated by the members of the
(d) Right to free education for all parliament present in the House.
Ans: (d) (c) A person from the panel formed by the Speake r of the
Q59. The term ‗Fraternity‘ in the Preamble of the Lok Sabha
Indian Constitution means a sense of (d) A senior most member of the House.
(a) friendliness Ans: (c)
(b) statehood Q66. What is the maximum time interval permitted
(c) love and affection between two sessions of Parliament ?
(d) brotherhood (a) 4 months (b) 6 months
Ans: (d) (c) 8 months (d) 9 months
Q60. In Indian Republic, the real executive authority Ans: (b)
rests with the : Q67. The President‘s Rule is imposed on a State in
(a) Prime Minister India, when
(b) President (a) The State Cabinet of Ministers resigns
(c) Bureaucrats (b) The Governor of the State dies
(d) Council of Ministers (c) The elections are announced
Ans: (d) (d) There is a Constitutional breakdown
Q61. What is the period within which a proclamation Ans: (d)
of national emergency made by the President is to be Q68. The authority to prorogue the two Houses of the
placed before each house of the Parliament for Parliament rests with the
approval? (a) President
(a) within one month (b) Vice-President
(b) within two months (c) Prime Minister
(c) within four months (d) Speaker
(d) within six months Ans: (a)
Ans: (a) Q69. The maximum number of members of Lok
Q62. Who among the following decides if a particular Sabha and Rajya Sabha has been fixed by the
Bill is a Money Bill or not? Constitution of India respectively as
(a) President (a) 500 and 250
(b) Speaker of Lok Sabha (b) 525 and 270
(c) Chairman of Rajya Sabha (c) 537 and 275
(d) Finance Minister (d) 545 and 250
Ans: (b) Ans: (d)
Q63. Rajya Sabha enjoys more powers than the Lok Q70. Which of the following taxes is exclusively and
Sabha in the case of — totally assigned to the Central Government by the
(a) Money Bills Constitution ?
(b) Non-money bills (a) Estate Duty
(c) Setting up of new All-India Services (b) Sales Tax
(d) Amendment of the Constitution (c) Taxes on Railway Fares and Freights
Page 326 of 516
(d) Corporation Tax https://telegram.me/aedahamlibra
Ans: (d)
Q78. Who certifies a Bill to be a Money Bill in India ?
(a) Finance Minister
Q71. The Electoral College that elects the President of (b) President
India consists of (c) Speaker of the Lok Sabha
(a) Elected members of all the State Legislatures (d) Prime Minister
(b) Members of the Lok Sabha and State Assemblies Ans: (c)
(c) Elected members of the Lok Sabha, Rajya Sabha and Q79. The ‗Speaker‘s vote‘ in the Lok Sabha is called
State Assemblies (a) casting vote (b) sound vote
(d) Elected members of the Rajy a Sabha and State (c) direct vote (d) indirect vote
Legislatures Ans: (a)
Ans: (c) Q80. How many times the President of India can seek
Q72. Which organ is the custodian of the National re-election to his post ?
Purse ? (a) Once
(a) Executive (b) Judiciary (b) 2 times
(c) Legislature (d) Civil Servants (c) 3 times
Ans: (c) (d) Any number of times
Q73. Which of the following is not a tool of legislative Ans: (d)
control over administration in India? Q81. What is the tenure of the Prime Minister of
(a) Dissolution of House India?
(b) Resolutions (a) Conterminous with the tenure of the Lok Sabha
(c) Questions (b) Conterminous with the tenure of the President
(d) No Confidence Motion (c) As long as he enjoys the support of a majority in the
Ans: (a) Lok Sabha
Q74. The President of India is elected by (d) Five years
(a) members of both houses of Parliament Ans: (c)
(b) members of both houses of Parliament and of State Q82. The Parliamentary Committee which scrutinises
legislatures the report of the Comptroller and Auditor General of
(c) me mbers of both houses of Parliament and of State India is
legislative assemblies (a) Estimates Committee
(d) elected members of both houses of Parliament and (b) Select Committee
elected members of State legislative assemblies (c) Public Accounts Committee
Ans: (d) (d) None of these
Q75. In our Constitution, Economic Planning is Ans: (c)
included in Q83. A Presidential Ordinance can remain in force
(a) Union list (a) For three months
(b) State list (b) For six months
(c) Concurrent list (c) For nine months
(d) Not any specified list (d) Indefinitely
Ans: (c) Ans: (b)
Q76. How many readings does a nonmoney bill have Q84. The declaration of Constitutional Emergency in
in each House of the Parliament? an Indian State has to be approved by the Parliament
(a) Two (b) Three within a period of
(c) Four (d) One (a) 2 months (b) 4 months
Ans: (b) (c) 6 months (d) 12 months
Q77. The interval between two sessions of either Ans: (a)
House of Parliament should not exceed Q85. How many times was the term of the Lok Sabha
(a) three months extended upto 6 years ?
(b) six months (a) Once (b) Twice
(c) nine months (c) Thrice (d) Never
(d) twelve months Ans: (a)
Ans: (b)
Page 327 of 516
https://telegram.me/aedahamlibra
Q86. For the election of President of India, a citizen
should have completed the age of
(b) After each general election to the House of the People
(c) Both 1 and 2
(a) 25 years (b) 30 years (d) Neither 1 nor 2
(c) 35 years (d) 18 years Ans: (c)
Ans: (c) Q94. Under which of the following situations can
Q87. Indian Parliament Means Rajya Sabha be dissolved?
(a) Rajya Sabha - Lok Sabha (a) When financial emergency is declared in the country.
(b) Rajya Sabha - Lok Sabha - (b) When emergency is declared due to failure of
Prime Minister Constitutional machinery.
(c) President of India - Rajya Sabha - Lok Sabha (c) Both (a) and (b)
(d) President of India - Vice- (d) None of these
President of India - Lok Sabha - Rajya Sabha Ans: (d)
Ans: (c) Q95. What is the correct order of succession
Q88. The Parliament can legislate on subjects given in (earlier to later)among the following Presidents of India?
the Union List only in consultation with the State a. Dr. Zakir Hussain
Government for the State of : b. Dr. S. Radhakrishnan
(a) Assam c. Fakhruddin Ali Ahmed
(b) Rajasthan d. V.V. Giri
(c) Jammu & Kashmir (a) a-c-d-a (b) c-b-d-a
(d) Kerala (c) d-a-b-c (d) b-a-d-c
Ans: (c) Ans: (d)
Q89. The president of I ndia can nominate how many Q96. When the Vice -President officiates as President
members from the Anglo -Indian community to the of India, he draws the salary of
Lok Sabha? (a) Chairman of Rajya Sabha
(a) Two (b) Five (b) President
(c) Ten (d) Twelve (c) Member of Parliament
Ans: (a) (d) Both (a) and (b)
Q90. Rajya Sabha being a permanent house Ans: (b)
(a) one-third of its members retire every two years Q97. The power to create or abolish the Upper House
(b) one-half of its members retire every three years of the State Legislature belongs to the
(c) one-fifth of its members retire every year (a) Governor
(d) one-sixth of its members retire every year (b) Parliament
Ans: (a) (c) High Court
Q91. Which item out of the following is contained in (d) State Legislature
the concurrent list? Ans: (b)
(a) Trade Unions Q98. The committee on subordinate legislation deals
(b) Agriculture with
(c) Tolls (a) Delegated legislation
(d) Markets and Fairs (b) Financial legislation
Ans: (a) (c) Municipal legislation
Q92. The Vice -President of India can be removed by (d) State legislation
the Ans: (a)
(a) President on the advice of Council of Ministers Q99. Delimitation of the Lok Sabha constituencies
(b) Lok Sabha with the consent of the President was last done in the year
(c) Rajya Sabha with the concurrence of Lok Sabha (a) 1970 (b) 1973
(d) Rajya Sabha with the concurrence of the President (c) 1976 (d) 1977
Ans: (c) Ans: (*)
Q93. The president of India addresses both Houses of Q100. Who nominates persons for various
Parliament assembled together at the commencement parliamentary delegation to foreign countries?
of the first session (a) The Speaker of Lok Sabha
(a) of each year (b) The Prime Minister
Page 328 of 516
(c) The President
(d) The Chairman of Rajya Sabha
https://telegram.me/aedahamlibra
(a) 1954 (8) 1964
(c) 1974 (d) 1984
Ans: (a) Ans: (a)
Q101. The President of the India is : Q109. Zero hour is at the discretion of
(a) The head of the ‗State‘ (a) Prime Minister
(b) The head of the Government (b) Speaker
(c) The head of the State as well as Government (c) Opposition leader
(d) None of these (d) President
Ans: (c) Ans: (b)
Q102. What is the term of a Member of the Rajya Q110. Which of the following motion is related with
Sabha ? the Union Budget?
(a) Three years (b) Four years (a) Adjournment
(c) Five years (d) Six years (b) Censure
Ans: (d) (c) Cut
Q103. Who decides whether a bill is a Money Bill or (d) None of the above
not ? Ans: (c)
(a) President Q111. Which one of the following committees is NOT
(b) Prime Minister a Standing Committee of the Parliament?
(c) Finance Minister (a) Public Accounts Committee
(d) Speaker of the Lok Sabha (b) Estimates Committee
Ans: (d) (c) Committee on Public Undertakings
Q104. What is the maximum age limit prescribed for (d) Consultative Committee to the Ministry of Finance
the post of the President of India ? Ans: (d)
(a) 58 years Q112. In which of the following list docs the subject
(b) 60 years ‗planning‘ figure ?
(c) 62 years (a) State List
(d) There is no maximum age limit (b) Concurrent List
Ans: (d) (c) Residuary List
Q105. What should be the gap between first No - (d) Union List
Confidence motion and second No-confidence motion Ans: (b)
? Q113. The Speaker of Lok Sabha is elected by
(a) 2 months (b) 3 months (a) all the members of Parliament
(c) 6 months (d) 9 months (b) the people directly
Ans: (c) (c) all the members of Lok Sabha
Q106. In which of the Parliamentary Financial (d) the members of the majority party in the Lok Sabha
Committees is the Rajya Sabha not represented ? Ans: (a)
(a) Public Accounts Committee Q114. Who appoints Secretary General of the Lok
(b) Estimates Committee Sabha ?
(c) Committee on Public Undertakings (a) Deputy Speaker
(d) Expenditure Committee (b) Speaker
Ans: (b) (c) President
Q107. One of the important attributes of (d) Leader of Ruling Party
Parliamentary form of government is Ans: (b)
(a) Fixed tenure for the executive Q115. Which one of the following is not an All India
(b) Executive is answerable to the people Service ?
(c) Executive is separate from the legislature (a) Indian Foreign Service
(d) Collective responsibility of Council of Ministers to (b) Indian Forest Service
the Parliament (c) Indian Administrative Service
Ans: (d) (d) Indian Police Service
Q108. In which year was the ―House of the People‖ Ans: (a)
named as ―Lok Sabha‖ ?
Page 329 of 516
https://telegram.me/aedahamlibra
Q116. Who of the following enjoys the rank of
Cabinet Minister in Union Cabinet ? Q125. The name of the upper house of the Indian
Ans: (b)

(a) Judge of Supreme Court Parliament is :


(b) Secretary to Government of India (a) Senate
(c) Political Advisor to PM (b) Rajya Sabha
(d) Deputy Chairman of Planning Commission (c) House of Lords
Ans: (d) (d) Legislative Assembly
Q117. The concept of Concurrent List in Indian Ans: (b)
Constitution is borrowed from the Constitution of Q126. The President of India can be removed from his
(a) Japan (b) Canada office by the
(c) Australia (d) U.S.A. (a) Chief Justice of India
Ans: (c) (b) Parliament
Q118. How many members are nominated by the (c) Lok Sabha
President to Rajya Sabha ? (d) Prime Minister
(a) 2 (b) 12 Ans: (b)
(c) 15 (d) 20 Q127. Who was the First Speaker of the Lok Sabha ?
Ans: (b) (a) Neelam Sanjeeva Reddy
Q119. Name the two non -permanent Houses in the (b) Hukum Singh
Parliamentary setup of India. (c) Ganesh Vasudev Mavlankar
(a) The Rajya Sabha and Vidhan Sabha (d) K.S. Hegde
(b) The Lok Sabha and Vidhan Parishad Ans: (c)
(c) The Rajya Sabha and Vidhan Parishad Q128. The term of office of the Vice-
(d) The Lok Sabha and Vidhan Sabha President of India is
Ans: (d) (a) 4 years (b) 5 years
Q120. A money bill can be introduced in (c) 2 years (d) 6 years
(a) either house of the Parliament Ans: (b)
(b) only Council of States (Rajya Sabha) Q129. Which of the following statements about a
(c) only House of the People (Lok Sabha) Money Bill is not correct?
(d) None of the above (a) A Money Bill can be tabled in either House of
Ans: (c) Parliament.
Q121. If the President of India wants to resign, he has (b) The Speaker of Lok Sabha is the final authority to
to address the letter of resignation to the decide whether a Bill is a Money Bill or not.
(a) Chief Justice of India (c) The Rajya Sabha must return a Money Bill passed by
(b) Prime Minister the Lok Sabha and send it for consideration within 14
(c) Vice-President days.
(d) Speaker (d) The President cannot return a Money Bill to the Lok
Ans: (c) Sabha for reconsideration.
Q122. How many items are there in the Union List ? Ans: (a)
(a) 52 (b) 66 Q130. In the Union Government, the Council of
(c) 97 (d) 99 Ministers is collectively responsible for the :
Ans: (c) (a) President
Q123. Who admits a new State to the Union of India ? (b) Prime Minister
(a) President (c) Lok Sabha
(b) Supreme Court (d) Parliament
(c) Prime Minister Ans: (c)
(d) Parliament Q131. The Residuary powers of legislation under
Ans: (d) Indian Constitution rests with
Q124. In India, how many times has the President (a) President
declared Financial Emergency ? (b) Prime Minister
(a) Once (b) Never (c) Parliament
(c) Thrice (d) Twice (d) States
Page 330 of 516
https://telegram.me/aedahamlibra
Q132. Which one of the following Committees is
Ans: (c) Q138. How many members of the State Legislative
Council are elected by the Assembly ?
described as the ‗twin sister‘ of the Estimates (a) 1 6 of the members
Committee? (b) 1 3 of the members
(a) Public Accounts Committee (c) 1 12 of the members
(b) Committee on Public Undertakings (d) 5 6 of the members
(c) Departmental Standing Committee Ans: (b)
(d) Privilege Committee Q139. Who is the chief advisor to the Governor ?
Ans: (a) (a) Chief Justice of Indian Supreme Court
Q133. Which of the following statements is correct (b) Chief Minister
about the President of India ? (c) Speaker of the Lok Sabha
(a) Addresses first session of Parliament after each (d) President
General Election Ans: (b)
(b) Addresses first session of Parliament at the beginning Q140. The States in India are demanding greater
of each year autonomy from the centre in the _____ field.
(c) Addresses every session of Parliament (a) Legislative
(d) Never addresses Parliament (b) Administrative
Ans: (a) (c) Financial
Q134. If the name of a tribe has to be excluded from (d) All the above
the list of Scheduled Tribe, who has the power for Ans: (d)
such exclusion? Q141. The members of the Legislative Assembly are
(a) President (a) indirectly elected
(b) Parliament (b) directly elected by the people
(c) National Commission for Scheduled Tribes (c) partly elected and partly nominated by the Governor
(d) Union Council of Ministers (d) mainly nominated
Ans: (b) Ans: (b)
Q135. Who decides disputes regarding Q142. In case no party enjoys absolute majority in the
disqualification of members of Parliament? Legislative Assembly of a state, the Governor will go
(a) The Supreme Court by :
(b) The Election Commission (a) the advice of former Chief Minister
(c) The Prime Minister in Consultation with the Election (b) the advice of the Prime Minister
Commission (c) the advice of the President of India
(d) The President in Consultation with the Election (d) his own discretion
Commission Ans: (d)
Ans: (d) Q143. Who is the highest Law Officer of a State ?
Q136. The President can advance money to meet (a) Solicitor General
unforeseen expenses from the (b) Secretary General, Law Department
(a) Consolidated Fund of India (c) Attorney General
(b) Grants of the Central Government (d) Advocate General
(c) Aid from the Union Government Ans: (d)
(d) Contingency Fund Q144. In relation to the State Government, local
Ans: (d) government exercises :
Q137. A motion moved by Member of Parliament (a) Co-ordinate Authority
when he feels a minister has committed a breach of (b) Delegated Authority
privilege of the House by withholding facts of a case is (c) Superior Authority
called (d) Independent Authority
(a) No confidence motion Ans: (b)
(b) Censure motion Q145. Which among the following state has Vidhan
(c) Privilege motion Parishad ?
(d) Cut motion (a) Karnataka
Ans: (c) (b) Tamil Nadu
Page 331 of 516
(c) Kerala
(d) Andhra Pradesh
https://telegram.me/aedahamlibra
(b) President on a unani -mous recommendation from the
Union Council of Ministers
Ans: (*) (c) President on the basis of an inquiry and report by the
Q146. The discretionary powers of a Governor is Supreme Court
limited in (d) President on recommendation from Central
(a) Appointment of Chief Minister Administrative Tribunal
(b) Dismissal of the Ministry Ans: (c)
(c) Dissolution of the Legislative Assembly Q153. What is the period of appointme nt of the
(d) Assent to Bills Comptroller and Auditor - General of India ?
Ans: (d) (a) 6 years
Q147. Who was the first woman Chief Minister of (b) Up to 65 years of age
Uttar Pradesh? (c) 6 years or 65 years of age whichever is earlier
(a) Annie Besant (d) Up to 64 years of age
(b) Mayawati Ans: (c)
(c) Vijayalakshmi Pandit Q154. A member of the Union Public Service
(d) Sucheta Kriplani Commission can be removed by the
Ans: (d) (a) President
Q148. The term of a Governor is : (b) Prime Minister
(a) 4 Years (b) 5 Years (c) Chief Justice of the Supreme Court
(c) 6 Years (d) 3 Years (d) Chairman of the UPSC
Ans: (b) Ans: (a)
Q149. What amidst the following is not true of the Q155. Who is the highest civil servant of the Union
general electoral roll prepared through the agency of Government ?
the Election Commission? It is to be used for elect ions (a) Attorney–General
to the (b) Cabinet Secretary
(a) Lok Sabha (c) Home Secretary
(b) Panchayatiraj and Nagarpalika institutions (d) Principal Secretary to the Prime Minister
(c) Legislative Assemblies of the states Ans: (b)
(d) Legislative Councils of the states where these exist Q156. When does Lok Sabha or a Vidhan Sabha
Ans: (d) election candidate forfeit his security deposit ?
Q150. Who finally approves the draft Five -Year-Plan (a) When he fails to win the election
? (b) When he fails to secure even 1 4 of total votes polled
(a) Planning Commission (c) When he fails to secure even 1 5 of total votes polled
(b) President (d) When he fails to secure even 1 6 of total votes polled
(c) National Development Council Ans: (d)
(d) Parl iament and State Legislatures Q157. The annual report of the UPSC is submitted to
Ans: (c) (a) The President
Q151. Which authority recommends the principles (b) The Supreme Court
gov erning the grantsin- (c) The Prime Minister
aid of the revenues of the States out of the Consolidated (d) The Chairman of the Union Public Service
Fund of India ? Commission
(a) Public Accounts Committee Ans: (a)
(b) Union Ministry of Finance Q158. Which of the follwoing is an extra –
(c) Finance Commission constitutional and nonstatutory body?
(d) Inter-State Council (a) Finance Commission
Ans: (c) (b) Planning Commission
Q152. Members of the Union Public Service (c) Union Public Service Commission
Commission can be removed by the (d) Election Commission
(a) Parliament after a resolution adopted with 2/ 3rds Ans: (b)
majority Q159. The Attorney General of India has the right to
audience in
Page 332 of 516
(a) any Sessions Court
(b) High Court
https://telegram.me/aedahamlibra
(c) Parliament
(d) Prime Minister
(c) Supreme Court Ans: (b)
(d) any court of law within the territory of India Q167. Who of the following enjoy constitutional
Ans: (d) position ?
Q160. The National Development Council includes : (A) Finance Commission
(a) all central Cabinet Ministers (B) National Development Council
(b) Chief Ministers of all the States (C) Election Commission
(c) Cabinet Ministers of all the States and the Centre. (D) Planning Commission
(d) Members of the Estimates Committee of the (a) A, C (b) A, B
Parliament (c) B, D (d) B, C
Ans: (b) Ans: (a)
Q161. Who is the Chairman of the Planning Q168. The Second Chief Election Commissioner of
Commission ? India was
(a) President of India (a) Sukumar Sen
(b) Finance Minister (b) S. P. Sen Verma
(c) Prime Minister (c) K. V. K Sundaram
(d) Minister for Planning (d) T. Swaminathan
Ans: (c) Ans: (c)
Q162. A person cannot contest election from Q169. The Attorney General of India has the right of
(a) more than one constituency audience in
(b) more than two constituencies (a) the Supreme Court
(c) more than three constituencies (b) any High Court
(d) more than four constituencies (c) any Sessions Court
Ans: (b) (d) any Court of Law within India
Q163. The Chairman of the Finance Commission Ans: (d)
must be Q170. The power to decide an Election Petition for the
(a) A person of Finance and Banking field State is vested in the
(b) An Economist of high calibre (a) Parliament
(c) An expert from Judiciary — level of High Court (b) Supreme Court
Judge (c) High Courts
(d) A person having experience in Public Affairs (d) Election Commission
Ans: (d) Ans: (c)
Q164. Who was the first Chief Election Commissioner Q171. Which committee recommended for three
of India ? language formula ?
(a) G.V. Mavlankar (a) Raj Committee
(b) T. Swaminathan (b) Kothari Committee
(c) K.V.K. Sundaram (c) Rajamannar Committee
(d) Sukumar Sen (d) Dutt Committee
Ans: (d) Ans: (b)
Q165. Who is the Chairman of Planning Commission Q172. The members of the All India Services serve the
? (a) Central Government only
(a) President (b) State Government only
(b) Prime Minister (c) Union Territories only
(c) Finance Minister (d) both the Union and State Governments
(d) Vice-President Ans: (d)
Ans: (b) Q173. If a budget is defeated in the legislature of a
Q166. Who appoints the Chief Election Commissioner state then
of India ? (a) The Finance Minister alone has to resign
(a) Chief Justice of India (b) The Finance Minister concerned has to be suspended
(b) President
Page 333 of 516
Minister has to resign
https://telegram.me/aedahamlibra
(c) The council of Ministers along with the Chief (a) Pondicherry
(b) Nagaland
(d) Reelection have to be ordered (d) Daman and Diu
Ans: (c) (d) Lakshadweep
Q174. The Comptroller and Auditor General is Ans: (b)
closely connected with which of the following Q182. The main feature of the Federal State is
Committees of Parliament? (a) Decentralisation
(a) The Estimates Committee (b) Centralisation
(b) The Committee on Public Undertakings (c) Theory of separation of powers
(c) The Public Accounts Committee (d) Sovereignty
(d) All of these Ans: (a)
Ans: (c) Q183. How many states are there in the Indian
Q175. What is the minimum age to qualify for Lok Union?
Sabha Elections? (a) 27 (b) 28
(a) 25 years (b) 30 years (c) 30 (d) 29
(c) 21 years (d) 18 years Ans: (d)
Ans: (a) Q184. The‘ Narmada Water Dispute Tribunal‘ was
Q176. Which of the following states is not included in constituted to resolve the water sharing between
Capital Region Planning Council ? (a) Gujarat and Rajasthan
(a) Rajasthan (b) Gujarat and Maharashtra
(b) Haryana (c) Gujarat, Maharashtra, Rajasthan and Madhya Pradesh
(c) Uttar Pradesh (d) Gujarat and Daman and Diu
(d) Madhya Pradesh Ans: (c)
Ans: (d) Q185. To whome can a case of dispute in the election
Q177. Article 370 of the Constitution is applicable to of the President of India be referred to ?
the State of : (a) Election Commission
(a) Nagaland (b) Parliament
(b) Mizoram (c) Supreme Court of India
(c) Manipur (d) Cabinet
(d) Jammu and Kashmir Ans: (c)
Ans: (d) Q186. Who is empowered to transfer a Judge from
Q178. Article 370 of the Constitution of India one High Court to another High Court?
provides for (a) Chief Justice of India
(a) temporary provisions for Jammu & Kashmir (b) President of India
(b) special provisions in respect of Nagaland (c) Law Minister of India
(c) special provisions in respect of Nagaland (d) The Union Cabinet
(d) provisions in respect of the financial emergency Ans: (b)
Ans: (a) Q187. Where the High Courts in India first set up ?
Q179. Sarkaria Commission was constituted to give (a) Delhi and Calcutta
its report on (b) Bombay, Madras, Calcutta
(a) New pay scales for the government servants (c) Bombay, Del0hi, Calcutta
(b) Centre-State relations (d) Madras and Bombay
(c) Ram Janmabhomi –Babri Masjid dispute Ans: (b)
(d) Kaveri dispute Q188. The Supreme Court of India enjoys
Ans: (b) (a) original jurisdictions.
Q180. Sikkim was granted statehood in the year (b) advisory jurisdictions.
(a) 1973 (b) 1974 (c) appellate and advisory jurisdictions.
(c) 1975 (d) 1976 (d) original, appellate and advisory jurisdictions.
Ans: (c) Ans: (d)
Q181. Which of the following is not a Union Territory Q189. Judicial review in the Indian Constitution is
? based on :
Page 334 of 516
(a) Rule of Law
(b) Due process of Law
https://telegram.me/aedahamlibra
Q197. Which one of the following statements about
Ans: (a)

(c) Procedure established by Law the Chief Justice of India (CJI) is not correct ?
(d) Precedents and Conventions (a) He appoints the Chief Justice of all High Courts.
Ans: (c) (b) The CJI administers the oath of office to the President
Q190. The Supreme Court is empowered to settle (c) When both the offic es of the President and Vice -
election disputes of President and Vice- President fall vacant simultaneously, the CJI discharges
President. This is its the duties of the President.
(a) Original Jurisdiction (d) The CJI can hold his office till he attains the age of 65
(b) Appellate Jurisdiction years.
(c) Advisory Jurisdiction Ans: (a)
(d) Miscellaneous Jurisdiction Q198. How many judges are there in Supreme Court
Ans: (a) ?
Q191. Of the following, who held the offices of Judge (a) 25 (b) 26
of the Supreme Court and the Speaker of the Lok (c) 30 (d) 31
Sabha Ans: (d)
(a) M. Hidayatullah Q199. What is ordinary Law ?
(b) K.S. Hegde (a) Laws made by the High Court
(c) Subba Rao (b) Laws made by the Supreme Court
(d) P.N. Bhagwati (c) Laws made and enforced by the Government
Ans: (b) (d) Laws made by the common people.
Q192. To whom can a case of dispute in the election of Ans: (c)
the President be referred ? Q200. Which one of the following is related to
(a) Chief Election Commissioner Advisory Jurisdiction of the Supreme Court?
(b) Parliament (a) Speaker of the Parliament seeking opinion from the
(c) Supreme Court of India Supreme Court
(d) None of these (b) Election Commission seeking opinion from the
Ans: (c) Supreme Court
Q193. The Supreme Court of India acts as a Federal (c) States seeking opinion from the Supereme Court
Court when it deals with (d) President of India seeks opinion on law or facts
(a) Civil cases Ans: (d)
(b) Inter-State disputes Q201. Subordinate courts are supervised by _______
(c) Appeals from lower courts (a) Supreme Court
(d) Election petitions (b) District Court
Ans: (b) (c) High Court
Q194. Who interprets the Constitution? (d) Parliament
(a) Legislature (b) Executive Ans: (c)
(c) Judiciary (d) President Q202. Which of the following is not an administrative
Ans: (c) function of a Village Panchayat ?
Q195. The Pension of a High Court Judge is charged (a) Providing sanitation and drainage
on the (b) Providing burial and cremation grounds
(a) Public Accounts of India (c) Providing college education
(b) Consolidated Fund of the State (d) Maintenance of roa
(c) Public Accounts of the State Ans: (c)
(d) Consolidated Fund of India Q203. Which of the following Indian States first
Ans: (b) adopted the 3-tier Panchayati Raj system ?
Q196. Supreme Court is the guardian for (a) Bihar
(a) Fundamental Rights (b) Uttar Pradesh
(b) Directive Principles (c) Rajasthan
(c) Preamble (d) Madhya Pradesh
(d) Centre and State disputes Ans: (c)
Page 335 of 516
introduced in India ?
https://telegram.me/aedahamlibra
Q204. When was the Panchayati Raj System Q212. Of the va rious grounds below, which is the one
criterion on which discriminaion by the State is not
(a) 1950 A.D. (b) 1945 A.D. prohibited in Article 15 of the Constitution ?
(c) 1947 A.D. (d) 1962 A.D. (a) Place of birth (b) Race
Ans: (*) (c) Language (d) Caste
Q205. The Panchayat Raj system in India was Ans: (c)
introduced under the direction of w hich article of the Q213. Which article of the Indian Constitution
constitution ? provides for the institution of Panchayati Raj ?
(a) 32 (b) 40 (a) Article. 36 (b) Article. 39
(c) 45 (d) 51 (c) Article 40 (d) Article. 48
Ans: (b) Ans: (c)
Q206. Panchayati Raj System was implemented first Q214. The articles 17 and 18 of constitution provide
in the pair of states (a) social equality
(a) Andhra Pradesh and Rajasthan (b) economic equality
(b) Assam and Bihar (c) political equality
(c) Arunachal Pradesh and Uttar Pradesh (d) religious equality
(d) Punjab and Chandigarh Ans: (a)
Ans: (a) Q215. Article 370 of the Indian Constitution upholds
Q207. The ―Recall Provision‖ to remove the elected (a) land reforms legislation in India
office bearers from the local Self Government (b) diplomatic privileges and immunities
institution has been executed in : (c) special status of Jammu and Kashmir State
(a) Bihar (d) duties and rights of Lokpal
(b) Kerala Ans: (c)
(c) Haryana Q216. By which Amendment were ‗Fundamental
(d) Madhya Pradesh Duties‘ added to the Constitution ?
Ans: (d) (a) 40th Amendment
Q208. The following article of the Indian Con stitution (b) 42nd Amendment
abolished the practice of untouchability. (c) 44th Amendment
(a) Article 14 (b) Article 18 (d) 45th Amendment
(c) Article 17 (d) Article 19 Ans: (b)
Ans: (c) Q217. In the Eighth Schedule of the Constitution of
Q209. Where is the objective of ―social justice‖ India, which languages were added subsequently ?
articulated in the Constitution of India ? (a) English, Sindhi, Marathi, Sanskrit
(a) Article 14 (b) Article 15 (b) Sanskrit, Sindhi, Konkani, Manipuri
(c) Article 16 (d) Preamble (c) Sindhi, Konkani, Manipuri, Nepali
Ans: (d) (d) Marathi, Oriya, Konkani, Nepali
Q210. Which among the following Articles of the Ans: (c)
Indian Constitution provides for State emergency and Q218. A proceeding under Article 226 in case of
suspends constitutional arrangement in a State ? detention of a person is a
(a) Article-352 (b) Article-356 (a) Civil proceeding
(c) Article-389 (d) Article-392 (b) Criminal proceeding
Ans: (b) (c) Judicial proceeding
Q211. What provision in the Constitution enabled the (d) Statutory proceeding
Central Government to impose the service tax and to Ans: (b)
expand its span ? Q219. The two words that were inserted by the 42nd
(a) List I, Schedule VII Amendment to the Preamble of the Constitution are
(b) List III, Schedule VII (a) Secular, Democratic
(c) Residuary Powers under Article 248 (b) Sovereign, Democratic
(d) Emergency Powers (c) Socialist, Secular
Ans: (c) (d) Secular, Republic
Ans: (c)
Page 336 of 516
defections were banned?
https://telegram.me/aedahamlibra
Q220. By which constitutional amendment political Q229. Which Article of the Const itution deals with
fundamental duties ?
(a) The Fiftieth amendment of 1984 (a) Aritcle 30 A
(b) The Fifty-third amendment of 1986 (b) Aritcle 50
(c) The Fifty-fourth amendment of 1986 (c) Aritcle 51 A
(d) The Fifty-second amendment of 1985 (d) Aritcle 25
Ans: (d) Ans: (c)
Q221. By which Constitutional Amendment Bill, did Q230. Which Article of the Indian Constitution
the Parliament lower the voting age from 21 to 18 provides free legal aid and equal justice?
years? (a) 30 (b) 25
(a) 42nd (b) 44th (c) 39-A (d) 33-B
(c) 61st (d) 73rd Ans: (c)
Ans: (c) Q231. Under which of the following amendment,
Q222. By which Constitution Amendment Act, Right education was made a fundamental right?
to Property ceased to remain a fundamental right? (a) 83rd amendment in 2003
(a) 44th (b) 42nd (b) 83rd amendment in 2002
(c) 43rd (d) 45th (c) 86rd amendment in 2002
Ans: (a) (d) 87rd amendment in 2003
Q223. Which one of the following languages is not Ans: (c)
specified in the Eighth schedule of the Indian Q232. Article 324 –329 of the Indian Constitution
Constitution ? deals with which of the following?
(a) Urdu (b) English (a) Tribunals (b) Elections
(c) Sindhi (d) Sanskrit (c) Cast System
Ans: (b) (d) Panchayti system
Q224. Which Article empowers the President to Ans: (b)
impose Financial Emergency ? Q233. In the 42nd Constitutional Amendment 1976,
(a) Article 356 (b) Article 364 which word was added to the Preamble?
(c) Article 352 (d) Article 360 (a) Democratic (b) Equality
Ans: (d) (c) Secular (d) Socialist
Q225. Article 1 of the Constitution of India declares Ans: (*)
India to be a Q234. The word ―Secular‖ was added to the Preamble
(a) Union of States of the Constitution of India by which Constitutional
(b) Federal State Amendment?
(c) Quasi-Federal State (a) 41st Constitutional Amendment
(d) Unitary State (b) 42nd Constitutional Amendment
Ans: (a) (c) 43rd Constitutional Amendment
Q226. Which Article of the Constitution enjoins the (d) 44th Constitutional Amendment
State to eastablish Village Panchayat? Ans: (b)
(a) Article 32 (b) Article 40 Q235. Governance through trade union organisations
(c) Article 44 (d) Article 57 is known as :
Ans: (b) (a) Guild Socialism
Q227. Which Article of the Indian Constitution deals (b) Fabian Socialism
withe Election Commission ? (c) Syndicalism
(a) Article 352 (b) Article 356 (d) Liberalism
(c) Article 360 (d) Article 324 Ans: (c)
Ans: (d) Q236. The concept of political sovereignty was
Q228. Which Article of the Constitution deals with advocated by
removal/ dismissal of a Civil Servant? (a) Plato (b) John Locke
(a) Article 25 (b) Article 256 (c) Rousseau (d) Austin
(c) Article 311 (d) Article 377 Ans: (c)
Ans: (c)
Page 337 of 516
https://telegram.me/aedahamlibra
Q237. Which of the following is the biggest head of
non-plan expenditure of Government of India ?
(d) Political Parties
Ans: (c)
(a) Interest payments Q246. The term ―Fourth Estate‖ is used for
(b) Subsidies (a) The Press and Newspaper
(c) Defence (b) Parliament
(d) Salaries and Wages (c) Judiciary
Ans: (a) (d) The Executive
Q238. How many spokes are there in the Ashoka Ans: (a)
Chakra depicted on the National Flag of India ? Q247. ―From each according to his capacity, to each
(a) 16 (b) 20 according to his needs‖ is the principle of
(c) 24 (d) 32 (a) Democracy (b) Fascism
Ans: (c) (c) Communism (d) Dictatorship
Q239. Who was the First Speaker of Independent Ans: (c)
India‘s Lok Sabha? Q248. The Creamy Layer, concept refers to
(a) Hukam Singh (a) the grouping based on social status
(b) Bali Ram Bhagat (b) the grouping based on castes
(c) Rabi Ray (c) the grouping based on economic status
(d) G.V. Mavalankar (d) the grouping based on milk consumption
Ans: (d) Ans: (c)
Q240. Name the President of India who was elected Q249. Out of the follo wing, which political party has
unopposed not been recognised as a National Political Party?
(a) Dr. Shankar Dayal Sharma (a) Indian National Congress
(b) Dr. Neelam Sanjiva Reddy (b) Bharatiya Janata Party
(c) Dr. Fakhruddin Ali Ahmed (c) Bahujan Samaj Party
(d) Dr. Zakir Husain (d) Trinamul Congress
Ans: (b) Ans: (d)
Q241. Which is the national flower of India? Q250. The National Integration Council
(a) Rose (b) Lotus (NIC) is chaired by the :
(c) Lily (d) Sunflower (a) Prime Minister
Ans: (b) (b) Finance Minister
Q242. The English Crown is an example of (c) Home Minister
(a) Real executive (d) President of India
(b) Quasi-real executive Ans: (a)
(c) Nominal executive Q251. Which of the following is the correct
(d) Nominated executive chronological order of the Prime Ministers in India ?
Ans: (c) I. Indira Gandhi
Q243. Despotism is possible in a II. Jawahar Lal Nehru
(a) One party state III. Morarji Desai
(b) Two party state IV. Charan Singh
(c) Multi Party state (a) I, II, III, IV (b) II, III, I, IV
(d) Two and multi party state (c) II, I, III, IV (d) III, II, IV, I
Ans: (a) Ans: (c)
Q244. Who said that ―Oh Disrespectable democracy I Q252. The idea of Lokpal is taken from
love you ‖ ? (a) Britain
(a) G.B. Shaw (b) Carpenter (b) America
(c) Lord Bryce (d) Appa Dorai (c) Scandinavian Countries
Ans: (a) (d) France
Q245. Fourth Estate is referred to Ans: (c)
(a) Public Opinion Q253. Who founded the Republican Party of India ?
(b) Chamber of Commerce (a) Namboo Dripad
(c) The Newspaper (b) Mulji Vaishya
Page 338 of 516
(c) Dr.B.R.Ambedkar
(d) Sripad Dange
https://telegram.me/aedahamlibra
(a) State enhances the ideal of individualism
(b) Nation state is unquestionably sovereign
Ans: (c) (c) State is the instrument of exploitation in the hands of
Q254. Who was the first to use the term ‗State‘? few.
(a) Hobbes (b) Plato (d) State is under the control of a king
(c) Aristotle (d) Machiavelli Ans: (b)
Ans: (d) Q262. The first General Election in India and first
Q255. Who had played key role in the formation of Amendment to the Constitution was held in:
Lokpal bill in India ? (a) 1949 (b) 1950
(a) Vipin Hazarika (c) 1951 (d) 1952
(b) Anna Hajare Ans: (c)
(c) Baba Amte
(d) Medha Patekar
Ans: (b)
Physical Geography
Q256. How many assembly segments are there as per
Vidhan Sabha Elections 2013 in Delhi ? Q1. The light from the Sun reaches the Earth in
(a) 70 (b) 72 about—
(c) 66 (b) 68 (a) 8 seconds (b) 8 minutes
Ans: (a) (c) 10 seconds (d) 10 minutes
Q257. Consumer Protection Act 1986, was a mended Ans: (b)
in : Q2. Brightest planet in our solar system is
(a) 1992 (b) 1993 (a) Venus (b) Mercury
(c) 1994 (d) 1991 (c) Mars (d) Jupiter
Ans: (*) Ans: (a)
Q258. Which among the following is not a preventive Q3. The largest planet in our solar system is
detention act? (a) Mars (b) Jupiter
(a) Terrorist and Disruptive Activities (c) Saturn (d) Mercury
(Prevention) Act Ans: (b)
(TADA) Q4. Which is the coldest among the following ?
(b) Prevention of Terrorism Act (a) Mars (b) Earth
(POTA) (c) Pluto (d) Mercury
(c) Conservation of Foreign Exchange and Prevention of Ans: (c)
Smuggling Activities Act Q5. Biggest planet of solar system is
(COFEPOSA) (a) Earth (b) Mars
(d) Foreign Exchange Regulation Act (FERA) (c) Saturn (d) Jupiter
Ans: (d) Ans: (d)
Q259. In India, woman had never been a Chief Q6. The planet revolving east to west is
Minister in the State of (a) Venus (b) Uranus
(a) Tamil Nadu (c) Neptune (d) Pluto
(b) Rajasthan Ans: (a)
(c) Uttar Pradesh Q7. Which planet orbits closest to the earth?
(d) Maharashtra (a) Mars (b) Jupiter
Ans: (d) (c) Venus (d) Mercury
Q260. Which committee was established on Criminal - Ans: (c)
Politician and Bureaucratic nexus ? Q8. Isohels are the isopleths of
(a) Vohra Committee (a) sunshine
(b) Indrajit Gupta Committee (b) flowering time
(c) Tarkunde Committee (c) rain (d) clouds
(d) Santhanam Committee Ans: (a)
Ans: (a)
Q261. What is the fascist view of state ?
Page 339 of 516
https://telegram.me/aedahamlibra
Q9. The planet that takes 88 days to make one
revolution of the sun is :
(b) Isostatic adjustment
(c) Collision of earth plates
(a) Mercury (b) Saturn (d) Volcanic eruption
(c) Jupiter (d) Mars Ans: (c)
Ans: (a) Q18. Basaltic lava is found in the
Q10. Which planet in our solar system is nearly as big (a) Deccan Trap
as the earth ? (b) Himalayas
(a) Mercury (b) Mars (c) Indo-Gangetic Plain
(c) Venus (d) Pluto (d) North-Eastern Hills
Ans: (c) Ans: (a)
Q11. A spinning neutron star is known as Q19. Where are the hot deserts generally found ?
(a) White dwarf (b) Black hole (a) On the eastern margins of the Tropics
(c) Pulsar (d)Quasar (b) On the western margins of the Tropics
Ans: (c) (c) Nearer the Equator
Q12. In a solar or lunar eclipse , the region of earth‘s (d) In the middle of the Continents
shadow is divided into Ans: (b)
(a) Five parts (b) Four parts Q20. Metamorphic rocks originate from—
(c) Two parts (d) Three parts (a) igneous rocks
Ans: (d) (b) sedimentary rocks
Q13. Which of the following is an alternative theory to (c) both igneous and sedimentary rocks
the Big Bang theory developed in 1948, stating that (d) None of these
the universe does not change e ven though it is Ans: (c)
expanding over time ? Q21. A geyser is a spring which
(a) Oscillating Universe (a) throws water continuously
(b) Steady State Universe (b) throws water intermittently
(c) Mirror Universe (c) throws water and steam at regular intervals
(d) Eternal Inflation (d) throws only steam
Ans: (b) Ans: (c)
Q14. The outermost layer of sun is called Q22. The topography of plateau is ideal for
(a) Lithosphere (a) cultivation (b) forestry
(b) Chromosphere (c) mining
(c) Photosphere (d) generation of hydro power
(d) Corona Ans: (d)
Ans: (d) Q23. The Mohorovicic (Moho) Discontinuity
Q15. If a star is bigger than Sun, but not more than separates
twice as big, it will turn into a _______. (a) Outer core and Mantle
(a) Pulsar (b) Maxima (b) Inner and Outer core
(c) Avenger (d) Discover (c) Sima and Nife
Ans: (a) (d) Crust and Mantle
Q16. Extensive deserts occur in the western tropical Ans: (d)
regions of continents because : Q24. The highest grade and best quality coal is
(a) of easterly trade winds. (a) Lignite (b) Peat
(b) cold ocean currents flow along the western coasts. (c) Bituminous (d) Anthracite
(c) of the effect of both the offshore easterly trade winds Ans: (d)
and cold ocean currents. Q25. What is the scale used for measuring the
(d) the rate of evaporation is greater along the western intensity of the earthquake?
margin areas. (a) Richter scale
Ans: (c) (b) Metric scale
Q17. Most of the devastating earthquakes are usually (c) Centigrade scale
caused by (d) Newton‘s scale
(a) Eustatic movement Ans: (*)
Page 340 of 516
estuary ?
https://telegram.me/aedahamlibra
Q26. Which one of the following rivers forms an Q35. Which type of lake is formed by volcanic
activities ?
(a) Ganga (b) Tapti (a) Caldera lake
(c) Godavari (d) Mahanadi (b) Karst lake
Ans: (b) (c) Lagoon
Q27. Desertification can be checked by (d) Fresh water lake
(a) plugging gullies Ans: (a)
(b) checking over-grazing Q36. The rate of deforestation is high in
(c) contour ploughing (a) Tropical zone
(d) forming shelter belts (b) Desert areas
Ans: (b) (c) Temperate zone
Q28. The latitude of a place is expressed by its (d) Boreal forest
angular distance in relation to— Ans: (c)
(a) Equator Q37. Sandstone is metamorphosed to:
(b) South Pole (a) Shale (b) Slate
(c) Axis of the Earth (c) Quartzite (d) Marble
(d) North Pole Ans: (c)
Ans: (a) Q38. Which of the following is an example of Plutonic
Q29. The scale that measures the intensity of Igneous Rock?
earthquakes is called as (a) Basalt (b) Granite
(a) Vernier scale (c) Slate (d) Dolomite
(b) Richter scale Ans: (b)
(c) Beaufort scale Q39. Obsidian, Andesite, Gabbro and Perodite are :
(d) Diagonal scale (a) Metamorphic rocks
Ans: (*) (b) Intrusive rocks
Q30. Natural embankments found on upper valley (c) Sedimentary rocks
sides are-known as (d) Extrusive rocks
(a) high banks (b) levees Ans: (*)
(c) bunds (d) ridge Q40. Spot the odd item in the following :
Ans: (b) (a) Basalt (b) Ruby
Q31. The inner-most layer of the earth is known as (c) Emerald (d) Sapphire
(a) Lithosphere Ans: (a)
(b) Mesosphere Q41. Solar eclipse occurs when
(c) Asthenosphere (a) the Moon does not lie on the line joining the Sun and
(d) Barysphere the Earth
Ans: (c) (b) the Moon comes between the Sun and the Earth
Q32. Prime Meridian passes through (c) the Sun comes between the Moon and the Earth
(a) Greenwich (b) Ujjain (d) the Earth comes between the Moon and the Sun
(c) New York (d) Tokyo Ans: (b)
Ans: (a) Q42. Nappe is a kind of
Q33. In the context of ‗time‘, GMT means (a) fluvial feature
(a) General Meridian Time (b) folded structure
(b) Greenwich Mean Time (c) erosional plain
(c) Global Mean Time (d) delta region
(d) None of these Ans: (b)
Ans: (b) Q43. An earthquake is also known as
Q34. The natural gaps across the mountains which (a) Teacher (b) Tremor
provide routes are called (c) Temper (d) None of these
(a) Peaks (b) Dunes Ans: (b)
(c) Plateaus (d) Passes Q44. The Equator does not pass through which of the
Ans: (d) following countries ?
Page 341 of 516
(a) Kenya
(c) Indonesia
https://telegram.me/aedahamlibra
(b) Mexico
(d) Brazil
Q53. What is the position of the Earth when it is at the
greatest distance from the sun ?
Ans: (b) (a) Aphelion (b) Antipode
Q45. Sink hole is a pheno menon of _______ (c) Perihelion (d) Aldiate
topography. Ans: (a)
(a) Plain (b) Desert Q54. Speed of wind is measured by
(c) Tundra (d) Karst (a) barometer (b) hygrometer
Ans: (d) (c) thermometer (d)anemometer
Q46. In mountainous areas a natural cause for road Ans: (d)
blocks is often : Q55. The line on a map connecting points of equal
(a) road repairs and construction temperature at a given time is
(b) land slides (a) Isohyet (b) Isobar
(c) tree felling (c) Isthumus (d) Isotherm
(d) traffic congestion Ans: (d)
Ans: (b) Q56. Daily weather c hanges in the atmosphere are
Q47. Equinox occurs when the sun is vertically above associated with
_____ (a) Troposphere (b) Mesosphere
(a) Tropic of Capricorn (c) Ionosphere (d) Stratosphere
(b) Tropic of Cancer Ans: (a)
(c) Poles Q57. Trade winds blow from the
(d) Equator (a) equatorial low pressure
Ans: (d) (b) polar high pressure
Q48. The minimum short-term natural hazard is (c) subtropical high pressure
(a) blizzard (b) earthquake (d) subpolar low pressure
(c) volcanic eruption Ans: (c)
(d) bolt of lightning Q58. Which of the following winds is called anti -trade
Ans: (d) wind ?
Q49. When Granite rocks get metamorphosed, they (a) Chinook (b) Cyclone
form (c) Typhoon (d) Westerlies
(a) Quartzite (b) Gneiss Ans: (a)
(c) Marble (d) Slate Q59. If there is no carbon dioxide in the earth‘s
Ans: (b) atmosphere, the temperature of earth‘s surface would
Q50. Magnetic Meridian is a be
(a) Line parallel to the equator of the Earth (a) dependent on the amount of oxygen in the atmosphere
(b) Latitude (b) higher than the present
(c) Line joining the geographic north and geographic (c) less than the present
south of Earth (d) the same
(d) Plane passing through the magnetic north and Ans: (c)
magnetic south of Earth Q60. Storms of gases are visible in the chromosphere
Ans: (d) of the Sun during
Q51. The land forms formed by wave erosion is : (a) Cyclones
(a) Spit (b) Beach (b) Anticyclones
(c) Cave (d) Barrier island (c) Lunar eclipse
Ans: (c) (d) Solar eclipse
Q52. A broad, low embankment built up along the Ans: (d)
banks of a river channel during floods is called Q61. Match List -I with List -II and select the correct
(a) Delta (b) Levee answer using the code given below the Lists :
(c) Flood Plain List - I
(d) Dune a. Australia
Ans: (b) b. China
c. France
Page 342 of 516
d. West Indies List - II
1. Hurricane
https://telegram.me/aedahamlibra
Q68. Subtropical high pressure belts are otherwise
called
2. Willy-willy (a) Furious fifties
2. Typhoon (b) Screeching sixties
4. Mistral Code : (c) Horse latitudes
abcd (d) Roaring forties
(a) 2 1 4 3 Ans: (c)
(b) 1 2 3 4 Q69. The temperature increases rapidly above
(c) 1 3 2 4 (a) Ionosphere (b) Exosphere
(d) 4 1 2 3 (c) Stratosphere (d) Troposphere
Ans: (a) Ans: (b)
Q62. Which one of the following is the highest cloud ? Q70. Which of the following is FALSE with respect to
(a) Cirrus rain water harvesting ?
(b) Stratocumulus (a) It is a device of water conservation.
(c) Nimbostratus (b) It helps raising water table.
(d) Cumulus (c) It helps meet rising water demand.
Ans: (a) (d) It increases run-off losses.
Q63. Which one of the following is called as ―Roaring Ans: (d)
Forty‖? Q71. Choose the corr ect option which represents the
(a) Winds blowing in southern hem isphere between 400– arrangement of atmospheric layers.
600 S (a) Ionosphere, Ecosphere, Mesosphere, Stratosphere,
(b) Winds blowing in northern hemisphere between 400 – Troposphere
600 N (b) Ecosphere, Troposphere, Ionosphere, Mesosphere,
(c) Very cold winds which blow in winters Stratosphere
(d) Very hot and fast blowing summer winds (c) Mesosphere, Ionosphere. Ecosphere, Troposphere,
Ans: (a) Stratosphare
Q64. Which of the following asborbs part of the (d) Troposphare, Ionosphere, Ecosphere
insolation and preserves earth‘s radiated heat? Ans: (d)
(a) Oxygen Q72. Most of the ozone in the earth‘s atmosphere is
(b) Nitrogen found in the :
(c) Water vapour (a) Troposphere
(d) Carbon-di-oxide (b) Stratosphere
Ans: (c) (c) Mesophere
Q65. In which region does rainfall occur throughout (d) Thermosphere
the year ? Ans: (b)
(a) Mediterranean Q73. A level of atmosphere which is composed partly
(b) Equatorial of electrons and positive ions is called
(c) Tropical (a) Troposphere
(d) Temperate (b) Ionosphere
Ans: (b) (c) Stratosphere
Q66. Smog is a combination of (d) Mesosphere
(a) air and water vapour Ans: (b)
(b) water and smoke Q74. Rainfall caused by intense evaporation in
(c) fire and water equatorial areas is called _______
(d) smoke and fog (a) Orographic rainfall
Ans: (d) (b) Cyclonic rainfall
Q67. The layer of the atmosphere in which Radio (c) Frontal rainfall
Waves are reflected back is called ; (d) Convectional rainfall
(a) Ionosphere (b) Troposphere Ans: (d)
(c) Stratosphere (d) Exosphere Q75. Which of the following is/are ‗rain cloud‘?
Ans: (a) 1. Cirrus
Page 343 of 516
2. Nimbostratus
3. Cumulonimbus
https://telegram.me/aedahamlibra
(b) Pacific Ocean
(c) Indian Ocean
4. Altocumulus (d) Arctic Ocean
(a) 1 and 2 only Ans: (b)
(b) 2 and 3 only Q84. Tides in the sea are caused by
(c) 3 and 4 only (a) effect of Sun
(d) 1 and 4 only (b) effect of Moon
Ans: (b) (c) eombined effect of Moon and Sun
Q76. The sea bed sloping gradually and bor dering the (d) gravitational force of Earth and Sun
continent is known as : Ans: (c)
(a) Coast Q85. The Sargasso sea is situated in the :
(b) Continental Shelf. (a) Atlantic Ocean
(c) Continental Platform. (b) Pacific Ocean
(d) Continental Slope. (c) Indian Ocean
Ans: (b) (d) Arctic Ocean
Q77. A place where there is constant intermingling of Ans: (a)
salt water and fresh water is ____ Q86. The rhythmic rise and fall of ocean water twice
(a) Estuary (b) Delta in a day is called
(c) Gulf (d) Bay (a) Tide (b) Ocean current
Ans: (a) (c) Wave (d)Water cycle
Q78. Echo sounding is the tech-nique applied to— Ans: (a)
(a) measure the depth of the sea Q87. Spring tides occur on
(b) measure the amplitude of sound waves (a) New moon day only
(c) record earthquake waves (b) Full moon day as well as on new moon day
(d) record the density of air in the atmosphere (c) Full moon day only
Ans: (a) (d) The day when the moon‘s position is in its first
Q79. Which current is responsible for i ncrease in the quarter
temperature of the western Europe ? Ans: (b)
(a) Labrador current Q88. The largest reservoir of fresh water is :
(b) Gulf stream (a) Glaciers (b) Ground Water
(c) Canari current (c) Ponds (d) Lakes
(d) North Equatorial current Ans: (a)
Ans: (d) Q89. ‗El Nino‘ that affects our Monsoons, has its
Q80. Sea breeze is formed during origins in
(a) Day time (b) Night time (a) The Indian Ocean
(c) Both (d) Seasonal (b) The Himalayan Plateau
Ans: (a) (c) The Pacific Ocean
Q81. Waves approaching the shore is known as (d) The Arabian Peninsula
(a) Swash (b) Tsunami Ans: (c)
(c) Currents (d) Tide Q90. Ring of Fire is found commonly in _____
Ans: (b) (a) Pacific Ocean
Q82. The term territorial water means (b) Atlantic Ocean
(a) The water found within the country in form of rivers (c) Indian Ocean
and lakes (d) Arctic Ocean
(b) The water secured from other countries Ans: (a)
(c) The water which is supplied to other countries Q91. The tides in the sea are primarily due to
(d) Water of the sea located close to the coast of a (a) the atmospheric effect of the Earth
country (b) the gravitational effect of Venus on the Earth
Ans: (d) (c) the gravitational effect of the Sun on the Earth
Q83. Mariana Trench is found in (d) the gravitational effect of the Moon on the Earth
(a) Atlantic Ocean Ans: (d)
Page 344 of 516
(a) Greenish brown
https://telegram.me/aedahamlibra
Q92. The colour of loamy soil is
Q100. ‗Terra rossa‘ is a Latin word which means
Ans: (b)

(b) Bluish green (a) hot area


(c) Yellowish brown (b) red terrain
(d) Blackish brown (c) lateritic region
Ans: (d) (d) region near the poles
Q93. Which of the following types of soil is best suited Ans: (b)
for cotton cultivation ? Q101. The asha–grey soils of high –latitude coniferous
(a) Black (b) Red forests a known as :
(c) Laterite (d) Mountain (a) Tundra soils
Ans: (a) (b) Podsols
Q94. Which one of the following methods of soil (c) Grey–Brown soils
conservation is most effective in arid areas ? (d) Red and Yellow soils
(a) Mulching Ans: (b)
(b) Shelter belt Q102. Bamboo is classified as
(c) Gully plugging (a) Tree (b) Grass
(d) Terracing (c) Shrub (d) Herb
Ans: (b) Ans: (b)
Q95. Which one of the following practices is adopted Q103. Which one of the following is the indirect use of
for restoring the fertility of soil ? forests ?
(a) Weeding (b) Levelling (a) Medicinal plants
(c) Fallowing (d) Harrowing (b) Checking soil erosion
Ans: (c) (c) Building material
Q96. Black soil is mainly related with the crop of (d) Grazing
(a) cotton (b) sugarcane Ans: (b)
(c) tea (d) coffee Q104. Which one is not a non -conventional energy
Ans: (a) source?
Q97. Alluvial soils are usually (a) Nuclear energy
(i) Rich in nitrogen (b) Solar Energy
(ii) Poor in humus (c) Wind energy
(iii) Highly fertile (d) Tidal power
(iv) Easy to plough Ans: (a)
(a) (i), (ii) and (iv) Q105. Coastal Andhra Pradesh and Orissa often face
(b) (ii), (iii) and (iv) natural disasters due to
(c) (i), (iii) and (iv) (a) Cyclones (b) Earthquakes
(d) (i), (ii) and (iii) (c) Landslides (d) Tornadoes
Ans: (d) Ans: (a)
Q98. Plantation of trees on a large scale to check soil Q106. Which one of the following is an abi otic and
erosion are called: renewable resource?
(a) Shelter belts (a) iron ore (b) livestock
(b) Contour ploughing (c) water (d) forests
(c) Strip cropping Ans: (c)
(d) Afforestation Q107. An ecosystem consists of :
Ans: (d) (a) producers, consumers and decomposers in a particular
Q99. Highly specialized form of agr iculture in which area.
crops like coffee, tea and rubber are cultivated refer (b) all the plants and animals of an area.
to (c) a living community and its environment.
(a) multiple cropping (d) carnivorous and herbivorous of an area
(b) plantation agriculture Ans: (c)
(c) terrace farming
(d) extensive farming
Page 345 of 516
https://telegram.me/aedahamlibra
Q108. After which one of the following tribes of India,
has a large continent of ancient geological history of Q116. With what bio -region is the term ‗‗Steppe‘‘
Ans: (a)

the world been named? associated ?


(a) Santhals (b) Bhils (a) Grasslands
(c) Marias (d) Gonds (b) Tropical forests
Ans: (d) (c) Savanna
Q109. The distribution of characteristic vegetation of (d) Coniferous forests
a region is determined mainly by Ans: (a)
(a) soil, water and temperature Q117. Vergreen type Forests are found in :
(b) water, greenhouse effect and altitude (a) Mediterranean region
(c) wind, water and latitude (b) Monsoon climatic area
(d) sunlight, water and wind (c) Desert region
Ans: (a) (d) Equatorial region
Q110. There is a need to ke ep larger area under Ans: (d)
forests for : Q118. Which of the following phenomena is
(a) absorption of carbon dioxide supposedly associated with global warming ?
(b) protecting wildlife (a) El Nino
(c) raising precipitation (b) La Nina
(d) geological balance (c) El Nino Modoki
Ans: (a) (d) Southern Oscillation
Q111. A natural region has the similarity of Ans: (a)
(a) climate and natural vegetation Q119. The land of maximum biodiversity is
(b) climate and occupation (a) Tropical (b) Temperate
(c) soil and drainage (c) Monsoonal (d) Equatorial
(d) economic base and reces Ans: (a)
Ans: (a) Q120. What is the alternate name for contour lines ?
Q112. Which of the following exhibits unidirectional (a) Isopotential (b) Istherm
flow in an ecosystem ? (c) Isohypse (d) Isohyet
(a) Light (b) Energy Ans: (c)
(c) Water (d) Biomass Q121. What is meant by the term ―midnight sun‖ ?
Ans: (b) (a) Twilight
Q113. The word Biosphere refers to : (b) Rising Sun
(a) Zone of soil, water and a ir around earth capable of (c) Very bright moon
supporting the flora and fauna. (d) Sun shining in the polar circle for long time
(b) Part of earth surface which can support the flora Ans: (d)
(c) Parts of air around earth a where life can exist Q122. The primary goal of planning for disasters is to
(d) Zone of water on earth which can support life. reduce
Ans: (a) (a) Death (b) Damage
Q114. Which of the following p henomena is (c) Risk (d) Vulnerability
supposedly associated with global warming ? Ans: (d)
(a) Southern Oscillations
(b) El Nino
(c) La Nino
Geography Of India
(d) El Nino Modiki
Ans: (d) Q1. Where is Khyber Pass situated?
Q115. Name the condition which influences the (a) Bhutan (b) Bangladesh
development of plants into distinctive forms. (c) India (d) Pakistan
(a) Climatic conditions Ans: (d)
(b) Soil conditions Q2. Which one of the following river flows through a
(c) Environmental conditions rift valley ?
(d) Social conditions
Page 346 of 516
(a) Godavari
(c) Krishna
https://telegram.me/aedahamlibra
(b) Narmada
(d) Mahanadi
(d) Satpura Range
Ans: (c)
Ans: (b) Q12. Which one of the following state has the longest
Q3. Which foreign country is closest to Andaman coastline ?
Islands ? (a) Maharashtra
(a) Sri Lanka (b) Myanmar (b) Tamil Nadu
(c) Indonesia (d) Pakistan (c) Gujarat
Ans: (b) (d) Andhra Pradesh
Q4. Which one of the following is not a part along the Ans: (c)
western coast of India ? Q13. Kanchenjunga is situated in
(a) Nhava Sheva (b) Marmagao (a) Nepal
(c) Tuticorin (d) Kochi (b) Sikkim
Ans: (c) (c) West Bengal
Q5. The standard time of India is (d) Himachal Pradesh
(a) 5 1 2 hours ahead of GMT Ans: (b)
(b) 4 1 2 hours behind of GMT Q14. Where did a major earthquake take place in
(c) 4 hours ahead of GMT India in 1999?
(d) 5 1 2 hours behind of GMT (a) Latur (b) Jabalpur
Ans: (a) (c) Chamoli (d) Uttar Kashi
Q6. Which one among the following is the highest Ans: (c)
peak ? Q15. The Valley of Kashmir is located on a
(a) Kamet (b) Kun Lun (a) nappe (b) fault trough
(c) Nanga Parbat (d) Nanda Devi (c) plateau (d) plain
Ans: (d) Ans: (a)
Q7. The approximate length of the coastline of India Q16. Mansarovar Lake is situated in
is (a) Nepal (b) India
(a) 5,500 km (b) 6,000 km (c) Tibet (d) Bhutan
(c) 6,500 km (d) 7,000 km Ans: (c)
Ans: (d) Q17. An example of ‗horseshoe‘ shaped coral islands
Q8. Which of the following will never get the vertical in the neighbourhood of India is
rays of the sun ? (a) Andaman and Nicobar is lands
(a) Mumbai (b) Maldives
(b) Chennai (c) Lakshadweep
(c) Thiruvanthapuram (d) Sri Lanka
(d) Srinagar Ans: (c)
Ans: (d) Q18. In which of the following States in Jog Falls
Q9. The latitude passing through the northern most located ?
part of India is (a) Orissa
(a) 35° N (b) 36° N (b) Kerala
(c) 37° N (d) 38° N (c) Andhra Pradesh
Ans: (a) (d) Karnataka
Q10. Which Indian State has the largest coastline ? Ans: (d)
(a) Andhra Pradesh Q19. The Zoji-La pass connects :
(b) Maharashtra (a) srinagar and Leh
(c) Orissa (b) Arunachal Pradesh and Tibet
(d) Tamil Nadu (c) Chamba and spiti
Ans: (a) (d) Kalimpong and Lhasa
Q11. Where in India can ‗Mushroom‘ rock be found ? Ans: (a)
(a) Eastern Ghats Q20. The sea which existed in the place of the
(b) Western Ghats Himalayas was
(c) Thar Desert (a) Red Sea (b) Arabian Sea
Page 347 of 516
(c) Tethys Sea https://telegram.me/aedahamlibra
(d) Dead Sea
Ans: (c)
Ans: (a)
Q30. To conserve coral reefs the Government of India
Q21. The mountains which are not a part of the declared one of the following as Marine Park :
Himalayan chain: (a) Andaman Islands
(a) Aravalli (b) Kunlun (b) Gulf of Kutch
(c) Karakoram (d) Hindukush (c) Lakshadweep Islands
Ans: (a) (d) Gulf of Mannar
Q22. Zojiia pass connects : Ans: (b)
(a) Nepal and Tibet Q31. The plateau that has both West and East flowing
(b) Leh and Kargil drainage system is
(c) Leh and Srinagar (a) Malwa
(d) Kashmir and Tibet (b) Chota Nagpur
Ans: (c) (c) Ranchi
Q23. The northern part of the west coast is known as (d) Hazaribagh
(a) Coromandal coast Ans: (a)
(b) Malabar coast Q32. Which of the following rock formations resulted
(c) Konkan coast in Eastern Ghats?
(d) Northern circars (a) Charnockites, bauxite, granite gneiss and quartzite
Ans: (c) rock
Q24. Indira Point is the southern most tip of : (b) Khondalites, bauxite, granite gneiss, metamorphic
(a) Maldives gneisses and quartzite rock
(b) Laccadives (c) Charnockites, khondalites, metamorphic gneisses and
(c) Andaman and Nicobar Islands quartzite rock
(d) Dhanushkodi (d) Charnockites, granite gneiss, quartzite rock,
Ans: (c) khondalites, metamorphic gneisses
Q25. India‘s area i s about.....times larger than Ans: (d)
Pakistan. Q33. Continuous chain of mountains that rise
(a) 3 (b) 4 abruptly more or less parallel to the coastline of India
(c) 6 (d) 9 is
Ans: (b) (a) Aravalli
Q26. Tiny marine animals which constitute limestone (b) Satpura
skeletons are called (c) Eastern ghats
(a) Clamitomonous (d) Western ghats
(b) Foraminifera Ans: (d)
(c) Coral reefs Q34. Which country is separated from India by a
(d) Diatoms narrow channel of sea formed by the Palk Strait and
Ans: (c) the Gulf of Mannar?
Q27. Which of the following is refer red to as ‗Young (a) Bangladesh (b) Myanmar
Fold Mountains‘ ? (c) Sri Lanka (d) Pakistan
(a) Aravallis (b) Nilgiris Ans: (c)
(c) Himalayas (d) Vindhyas Q35. The Andaman is separated from Nicobar by
Ans: (c) which water body?
Q28. What is the total area of India ? (a) 11° channel
(a) 8,511,965 sq. km (b) 10° Channel
(b) 3,897,950 sq. km (c) Palk Strait
(c) 5,926,780 sq. km (d) Gulf of Mannar
(d) 3,287,590 sq. km Ans: (b)
Ans: (d) Q36. Mount Abu is a hill station located in _____
Q29. Niyamgiri hill is located in Kalahandi district ? ranges.
(a) Orissa (b) West Bengal (a) Vindhya (b) Satpuda
(c) Punjab (d) Kerala (c) Aravalli (d) Sahyadri
Page 348 of 516
https://telegram.me/aedahamlibra
Ans: (c)
Q37. Which of the following is a river flowing from
(d) Uttar Pradesh
Ans: (d)
Central India and joining Yamuna/ Ganga? Q43. Which one of the following pairs is correctly
(a) Ghagra (b) Gomti matched ?
(c) Kosi (d) Betwa (a) Tulbul Project – Himachal Pradesh
Ans: (d) (b) Srisailam Project – Tamil Nadu
Q38. Match the dams and the rivers across which (c) Papanasam Project – Karnataka
those have been constructed. (d) Ukai Project – Gujarat
Dam River Ans: (d)
(a) Gandhi Sagar 1. Bhagirathi Q44. Where is Nathpa Jhakri Power Project located ?
(b) Jayakwadi 2. Krishna (a) Uttarakhand
(c) Nagarjuna 3. Godavari (b) Arunachal Pradesh
(d) Tehri 4. Chambal (c) Himachal Pradesh
(a) (b) (c) (d) (d) Andhra Pradesh
(a) 4 3 2 1 Ans: (c)
(b) 3 1 4 2 Q45. In India, the irrigation of agricultural land is
(c) 2 4 3 1 carried out maximum by
(d) 4 2 1 3 (a) Canals (b) Wells
Ans: (a) (c) Tubewells (d) Tanks
Q39. The highest multipurpose dam built on the river Ans: (b)
Ravi is– Q46. The Loktak Lake on which a hydroelectric
(a) Bhakra Nagal project was constructed is situated in the State of
(b) Kahalgaon (a) Madhya Pradesh
(c) Ranjit Sagar dam (b) Manipur
(d) Rihand dam (c) Meghalaya
Ans: (c) (d) Himachal Pradesh
Q40. Match the dams and the States in which they are Ans: (b)
situated : Q47. According to a report in the science journal
Dam ‗Nature Geoscience‘ twenty four out of thirty-three
A. Hirakud B. Mettur deltas in the world are sinking and thus shrinking.
C. Mahanadi D. Almatti State The Indian delta in the ‗Greater Peril‘ category is
1. Chhattisgarh 2. Orissa (a) Brahmani (b) Godavari
3. Karnataka 4. Tamil Nadu (c) Mahanadi (d) Krishna
(A) (B) (C) (D) Ans: (d)
(a) 3 2 4 1 Q48. Which is the longest irrigation canal in India ?
(b) 2 4 1 3 (a) Sirhind Canal
(c) 1 3 2 4 (b) Yamuna Canal
(d) 4 1 3 2 (c) Indira Gandhi Canal
Ans: (b) (d) East Kosi Canal
Q41. Which of the following canals is located in West Ans: (c)
Bengal ? Q49. Surat is situated on the banks of the river
(a) Lower Ganga Canal (a) Tapti (b) Mahanadi
(b) Sarada Canal (c) Bhima (d) Godavari
(c) Eden Canal Ans: (a)
(d) Sirhind Canal Q50. The originating point of river godavri is
Ans: (c) (a) Nasik (b) Pune
Q42. In which part of India, canal irrigation system is (c) Mumbai (d) Sholapur
the most common ? Ans: (a)
(a) Tamil Nadu Q51. The river Brahmaputra is known as _______ as
(b) Maharashtra it enters Arunachal Pradesh.
(c) Sikkim (a) Dibang (b) Dihang
Page 349 of 516
(c) Subansiri https://telegram.me/aedahamlibra
(d) Dhansiri
Ans: (b)
(c) Indus (d) Ravi
Ans: (b)
Q52. The largest estuary in India is at the mouth of Q62. Which one of the following is the origin of the
river Brahmaputra river?
(a) Hooghly (b) Bhagirathi (a) Milam
(c) Godavari (d) Krishna (b) Gangotri
Ans: (a) (c) Yamunotri
Q53. The river which joins Ganga from southern side (d) Chemayungdung
is : Ans: (d)
(a) Betwa (b) Chambal Q63. Which river in India flows in a rift-valley?
(c) Son (d) Ken (a) Cauvery (b) Tapti
Ans: (c) (c) Narmada (d) Krishna
Q54. The first multi -purpose project of independent Ans: (c)
India is Q64. Which of the following water bodies is not a
(a) Bhakra-Nangal freshwater resource ?
(b) Damodar (a) Jaisamand (b) Ganga river
(c) Hirakud (c) Yamuna river (d) Chilika lake
(d) Nagarjunasagar Ans: (c)
Ans: (b) Q65. Which was the first hydel power p roject in India
Q55. The Chukha Power Project was built by India in ?
(a) Nepal (b) Bhutan (a) Pallivasal in Kerala
(c) Myanmar (d) Bangladesh (b) Paikara in Tamil Nadu
Ans: (b) (c) Siva Samudram in Karnataka
Q56. Which is the river on which Indira Sagar Dam is (d) Nizamnagar in Andhra Pradesh
planned to be constructed ? Ans: (c)
(a) Mahanadi (b)Godavari Q66. Where do Bhagirathi and Alakananda join
(c) Krishna (d) Brahmaputra Ganga ?
Ans: (*) (a) Gangotri (b)Karanprayag
Q57. An important river of the Indian desert is (c) Devprayag (d)Rudraprayag
(a) Luni (b) Narmada Ans: (c)
(c) Krishna (d) Beas Q67. On which river is the Tehri dam built?
Ans: (a) (a) Alakananda (b) Bhagirathi
Q58. Nasik is situated on the bank of the river : (c) Ganga (d) Hooghly
(a) Mahanadi (b) Tapti Ans: (b)
(c) Krishna (d) Godavari Q68. (c) Which of the following are alternative names
Ans: (d) for the river ―Brahmaputra‖ ?
Q59. The river on which the reservoir for Indira (a) Yamuna, Yarlung Zangbo and Tsangpo
Gandhi Canal has been built is (b) Yamuna, Megna and Tsangpo Yamuna
(a) Sutlej (b) Ravi (c) Jamuna, Siang, Yarlung Zangbo and Tsangpo
(c) Luni (d) Jhelum (d) Jamuna, Siang, Yarlung Zangbo, Megna and Tsangpo
Ans: (a) Ans: (d)
Q60. Cauvery water sharing is a dispute between Q69. What is Damodar Valley Corporation ?
(a) Karnataka and Andhra Pradesh (a) Statutory body
(b) Tamil Nadu and Andhra Pradesh (b) Municipal Corporation looking after Damodar Valley
(c) Tamil Nadu and Karnataka (c) A private enterprise located in Bihar
(d) Karnataka and Maharashtra (d) A non government organisation
Ans: (c) Ans: (a)
Q61. The world‘s highest rail bridge being Q70. Which of the following is the largest irrigation
constructed in the State of J & K will be on which of plant in India?
the following rivers ? (a) Buckingham Canal
(a) Jhelum (b) Chenab (b) Indira Gandhi Canal
Page 350 of 516
(c) Upper Ganges Canal
(d) Tajewala Canal
https://telegram.me/aedahamlibra
(d) Malwa Plateau
Ans: (c)
Ans: (b) Q79. The driest part of India is
Q71. What is Dakshin Gangotri ? (a) Western Rajasthan
(a) River valley in Andhra Pradesh (b) Jammu and Kashmir
(b) Unmanned station located in Antarctica (c) Gujarat
(c) Second source of River Ganga (d) Madhya Pradesh
(d) Island in the Indian Ocean Ans: (a)
Ans: (b) Q80. Why does the west coast of India receive more
Q72. Name the longest lake in India? rainfall from southwest monsoon than the east coast?
(a) Pangong lake (a) Unlike the east coast this coast is straight
(b) Pulicat Lake (b) The Western Ghats obstruct the winds causing rainfall
(c) Kolleru Lake (c) The east coast is broader than the west coast
(d) Vembanad Lake (d) The Eastern Ghats extend parallel to wind direction
Ans: (d) Ans: (b)
Q73. The only perennial river in Peninsular India is Q81. What should be the proportion of forest cover
_____ . for India to maintain her ecological balance?
(a) Godavari (b) Kaveri (a) 11.1 percent
(c) Krishna (d) Bhima (b) 22.2 percent
Ans: (b) (c) 33.3 percent
Q74. Rajasthan reveives very little rain because— (d) 44.4 percent
(a) it is too hot Ans: (c)
(b) there is no water available and thus the wind s remain Q82. Which is the first national park established in
dry India?
(c) the monsoon fails to reach this area (a) Velvadan National Park
(d) the winds do not come across any barriers to cause the (b) Periyar National Park
necessary uplift to cool the wind (c) Bandipur National Park
Ans: (d) (d) Corbett National Park
Q75. The South West monsoon engulfs the entire Ans: (d)
India by Q83. Where in India can you normally spot the
(a) 5th June (b) 15th June Siberian crane in winter ?
(c) 1st July (d) 15th July (a) Sasangir Sanctuary
Ans: (d) (b) Ranthambore Sanctuary
Q76. The period by which the entire country in India (c) Dachigam National Park
comes under southwest monsoon is (d) Keoladeo Ghana Sanctuary
(a) l st - l0th June Ans: (d)
(b) l0th - 20th June Q84. In terms of area, which one of the following
(c) 20th - 30th June Indian States has the largest coverage of forests ?
(d) lst - 15th July (a) Arunachal Pradesh
Ans: (d) (b) Chhattisgarh
Q77. ‗Summer Monsoons‘ in India bring rainfall to (c) Madhya Pradesh
(a) Southern-eastern tip (d) Orissa
(b) Western Coast Ans: (c)
(c) North-western India Q85. The only sanctuary where Kashmir stag is found
(d) Eastern Coast is
Ans: (b) (a) Kanha (b) Dachigam
Q78. Heavy rainfall during the months of October (c) Gir (d) Mudumalai
and November is received by Ans: (b)
(a) Gharo, Khasi and Jaintia hills Q86. When was the first National Forest Policy issued
(b) Chota Nagpur Plateau by the Government of India ?
(c) Coromandel Coast (a) 1952 (b) 1940
Page 351 of 516
(c) 1942 https://telegram.me/aedahamlibra
(d) 1999
Ans: (a) Q96. Movement of people from a village to a small
Ans: (c)

Q87. Manas Sanctuary in Assam is known for town and later to a city is known as :
(a) elephants (b) bears (a) intra–state migration
(c) tigers (d) wild ass and (b) step–wise migration
Ans: (a) (c) inter–state migration
Q88. Which of the following is correctly matched? (d) forced migration
Wildlife Sancturay State Ans: (b)
(a) Bandipur — Tamil Nadu Q97. Kanha National Park is located in :
(b) Manas — Uttar Pradesh (a) Bihar
(c) Ranthambhore — Rajasthan (b) Madhya Pradesh
(d) Simlipal — Bihar (c) Andhra Pradesh
Ans: (c) (d) Tmil Nadu
Q89. The wild ass is found in Ans: (b)
(a) Kachchh Q98. The largest wildlife sanctuary in India is famous
(b) Assam for which of the following animal ?
(c) Jammu and Kashmir (a) Wild Indian Ass
(d) Tamil Nadu (b) Rhinoceros
Ans: (a) (c) Apes
Q90. Which are the States in which the Agasthyamala (d) Tigers
biosphere is spread? Ans: (a)
(a) Andhra Pradesh – Karnataka Q99. The natural habitat of Rhinoceros in India is
(b) Tamil Nadu – Karnataka (a) Bharatpur (b) Gir forest
(c) Maharashtra–Madhya Pradesh (c) Khaziranga
(d) Kerala – Tamil Nadu (d) Nilgiris
Ans: (d) Ans: (c)
Q91. An area legally reserved for wild life in its Q100. In India, the Temperate Forest Research
natural surroundings is Centre is in which city?
(a) Biosphere Reserve (a) Shimla (b) Ranchi
(b) Sanctuary (c) Dehradun (d) Srinagar
(c) Social Forests Ans: (a)
(d) National Park Q101. Watermelons grow best in
Ans: (b) (a) Alluvial soil (b) Sandy soil
Q92. Which one of the following is a Wild Life (c) Black soil (d) Laterite soil
Sanctuary ? Ans: (b)
(a) Jaldapara (b) Garumara Q102. To which group does the black cotton soil of
(c) Corbett (d) Chapramari India belong ?
Ans: (c) (a) Laterite (b) Podzol
Q93. Mudumalai Sanctuary is famous for (c) Chernozem (d) Alluvial
(a) Tigers (b) Bisons Ans: (c)
(c) Birds (d) Elephants Q103. Red soil is normally found in Ind ia in which
Ans: (a) regions?
Q94. The largest tiger reserve in India is (a) Eastern Region only
(a) Pench (b) Manas (b) Southern Region only
(c) Nagarjun (d) Corbett (c) Eastern & Southern part of the Deccan Plateau
Ans: (c) (d) None of these
Q95. Kanchenzunga National Park is located at Ans: (c)
(a) Uttar Pradesh Q104. The maximum area under crops in India is
(b) West Bengal used for the cultivation of :
(c) Sikkim (a) Wheat (b) Rice
(d) Jammu and Kashmir (c) Sugarcane (d) Cotton
Page 352 of 516
https://telegram.me/aedahamlibra
Ans: (b)
Q105. Which of the following is not a Kharif crop ? Q113. The term ―Green Revolution‖ has been used to
Ans: (a)

(a) Rice (b) Wheat indicate higher production through


(c) Sugarcane (d) Cotton (a) creation of grasslands
Ans: (b) (b) planting more trees
Q106. Which is the home of ―Alphonso mango‖? (c) enhanced agricultural productivity per hectare
(a) Ratnagiri (b) Benares (d) creation of gardens in urban areas
(c) Malda (d) Vijayawada Ans: (c)
Ans: (a) Q114. Match the following and select the correct
Q107. The crops gro wn after the summer monsoon answer from the codes given below :
are called Crops Producing state
(a) Kharif (b) Rabi (a) Tea (a) Himachal Pradesh
(c) Annual (d) Seasonal (b) Sugarcane (b) Assam
Ans: (b) (c) Groundnut (c) Uttar Pradesh
Q108. Which of the following has not been a (d) Apple (d) Gujarat
component of the agricultural strategy that brought (a) a-2, b-4, c-1, d-3
about the Green Revolution ? (b) a-2, b-3, c-4, d-l
(a) Greater intensity of cropping (c) a-3, b-2, c-1, d-4
(b) Guaranteed maximum prices (d) a-4, b-3, c-1, d-2
(c) New agricultural technology Ans: (b)
(d) Package of inputs Q115. Green Revolution was started in
Ans: (b) (a) 1960 (b) 1970
Q109. In which of the following grouping of States of (c) 1980 (d) 1990
India is rubber grown on a commercial scale ? Ans: (a)
(a) Maharashtra-Gujarat- Q116. Crop rotation is being adopted
Madhya Pradesh (a) to increase the productivity of the land
(b) Kerala –Tamil Nadu – Karnataka (b) to increase the crop yield
(c) Sikkim-Arunachal Pradesh- (c) to increase the soil water
Nagaland (d) to increase the crop resistance to pests.
(d) Orissa-Madhya Pradesh- Ans: (a)
Maharashtra Q117. The State which occupies the first place in
Ans: (b) India in the production of Tobacco is
Q110. Which of the following is considered a cash (a) Tamil Nadu
crop in India ? (b) West Bengal
(a) Maize (b) Gram (c) Andhra Pradesh
(c) Onion (d) Wheat (d) Maharashtra
Ans: (b) Ans: (c)
Q111. Rotation of crops means Q118. Fibre crops are
(a) growing of different crops in succession to maintain (a) jute, sugarcane, linseed and rice
soil fertility (b) cotton, maize, tobacco and banana
(b) some crops are grown again and again (c) cotton, hemp, jute and mesta
(c) two or more crops are grown simultaneously to (d) hemp, cotton, maize and saffron
increase productivity Ans: (c)
(d) None of the above Q119. Bulk of natural rubber in India is produced in
Ans: (a) (a) Andhra Pradesh
Q112. Of the gross cropped area in India, the (b) Kerala
foodgrains occupy (c) Karnataka
(a) more than 70% (d) Tamil Nadu
(b) 60% to 70% Ans: (b)
(c) 50% to 60% Q120. High yielding plants can be produced by
(d) less than 50% (a) Crop Rotation
Page 353 of 516
(b) Hybridisation
(c) Inter-cropping
https://telegram.me/aedahamlibra
Q129. Among the following districts of Tamil Nadu,
Ans: (c)

(d) Mixed - cropping which district is unfit for cultivat ion due to increased
Ans: (b) salinity :
Q121. Name the food cro p which gives highest output (a) Coimbatore
in India. (b) Tiruchirapalli
(a) Wheat (b) Jowar (c) Nagapattinam
(c) Maize (d) Rice (d) Ramanathapuram
Ans: (a) Ans: (c)
Q122. Which of the following is not a Rabi crop in Q130. The most ideal region for the cultivation of
India ? cotton in India is
(a) Wheat (b) Barley (a) The Brahmaputra Valley
(c) Rapeseed (d) Jute (b) The Deccan Plateau
Ans: (d) (c) The Indo Gangetic Valley
Q123. Agricultural Commodities are graded with : (d) The Rann of Kutch
(a) ISI (b) Eco-products Ans: (b)
(c) AGMARK (d) Green Product Q131. Which State in India is estimated to have the
Ans: (c) largest coal reserves in India?
Q124. Jhumming is shifting agriculture practised in (a) Andhra Pradesh
(a) North-eastern India (b) Bihar (including the newly created Jharkhand)
(b) South-western India (c) Madhya Pradesh (including the newly created Ch -
(c) South-eastern India hattisgarh)
(d) Northern India (d) Orissa
Ans: (a) Ans: (b)
Q125. Social forestry is Q132. Which one of the following States produces the
(a) growing differe nt types of plants together on private maximum manganese in India ?
land (a) Madhya Pradesh
(b) management of forest by cooperative societies (b) Andhra Pradesh
(c) growing one type of plant in government owned land (c) Uttar Pradesh
(d) growing and management of useful plants on (d) Orissa
government owned land Ans: (d)
Ans: (d) Q133. Which of the following produces maximum
Q126. Which of the following is not a characteristic of crude petroleum in India at present?
Indian agriculture ? (a) Assam
(a) Multiplicity of crops (b) Gujarat
(b) Predominance of large farms (c) Off-shore Bombay High
(c) Overdependence on nature (d) Coastal Tamil Nadu
(d) Low level of productivity Ans: (c)
Ans: (b) Q134. Tummalapalle, where huge reserves of
Q127. Which State called the ‗Rice-Bowl‘ of India ? uranium have been found, as announced by the
(a) Kerala Atomic Energy Commission in July, 2011, lies in the
(b) Karnataka state of
(c) Andhra Pradesh (a) Jharkhand
(d) Tamil Nadu (b) Andhra Pradesh
Ans: (c) (c) Tamil Nadu
Q128. Which State is the largest producer of pulses in (d) Gujarat
India ? Ans: (b)
(a) Bihar Q135. ―Ankleshwar‘ is known for
(b) Rajasthan (a) Gold (b) Coal
(c) Madhya Pradesh (c) Gas (d) Oil
(d) Maharashtra Ans: (d)
Page 354 of 516
https://telegram.me/aedahamlibra
Q136. Which of the following oilfields is situated in
the North-eastern region of India?
Q144. Which group of the industries maximise
savings on transport costs by being located near the
(a) Kalol (b) Ankaleshwar sources of materials?
(c) Mehsana (d) Digboi (a) Iron and steel, aluminium, cement
Ans: (d) (b) Iron and steel, sugar, cotton textiles
Q137. ―Panna‖ is an important place in Madhya (c) Heavy machinery, cement, sugar
Pradesh. It is famous for : (d) Iron and steel, cement, silk
(a) Gold mines Ans: (a)
(b) Silver mines Q145. Ankleshwar and Kalol are two oil fields in
(c) Diamond mines (a) Maharashtra (b) Assam
(d) Iron mines (c) Gujarat (d) Rajasthan
Ans: (c) Ans: (c)
Q138. Tummalapalli in Andhra Pradesh has recently Q146. Which kind of power accounts for the largest
come on the world map for its largest share of power generation in India ?
(a) Uranium deposits (a) hydro-electricity
(b) Tungsten deposits (b) thermal (c) nuclear
(c) Coal deposits (d) solar
(d) Bauxite ore deposits Ans: (b)
Ans: (a) Q147. Which one of the following is used in large
Q139. In India, there are many coal fields found in quantities in Cement Industry ?
(a) Cauvery Valley (a) Gypsum (b) Limestone
(b) Krishna Valley (c) Coal (d) Clay
(c) Ganga Valley Ans: (b)
(d) Damodar Valley Q148. The product used fo r manufacturing of
Ans: (d) Industrial alcohol is
Q140. Which one of the following r egion is most rich (a) Khandasari (b) Bagasse
in coal deposits? (c) Molasses (d) Paper-pulp
(a) Bramhaputra Valley Ans: (c)
(b) Damodar Valley Q149. Which of the following projects suppl ies
(c) Mahanadi Valley electricty to the Rourkela Steel Plant?
(d) Godavari Valley (a) Damodar Valley Project
Ans: (b) (b) Hirakud Dam Project
Q141. Where was the first cotton mill in India (c) Tungabhadra Project
established? (d) Nagarjuna Project
(a) Surat (b) Mumbai Ans: (b)
(c) Ahmedabad (d) Coimbatore Q150. Nepanagar in Madhya Pradesh has a
Ans: (b) (a) sugar mill
Q142. Where was the first Cotton Mill in India (b) newsprint factory
established ? (c) steel plant
(a) Surat (d) heavy engineering plant
(b) Bombay (now Mumbai) Ans: (b)
(c) Ahmedabad Q151. The iron and steel industries at Bhilai,
(d) Coimbatore Durgapur and Rourkela were set up during the
Ans: (b) (a) II Five Year Plan
Q143. Which of the following is correctly matched (b) I Five Year Plan
with regard to thermal power projects ? (c) III Five Year Plan
(a) Korba-Uttar Pradesh (d) IV Five Year Plan
(b) Ramagundam-Tamil Nadu Ans: (b)
(c) Talcher-Andhra Pradesh Q152. It forms the bed -rock of all industrial activities
(d) Kawas-Gujarat and is considered ―mother industry‖.
Ans: (d) (a) Petro-chemical industry
Page 355 of 516
(b) Cement industry
(c) Iron and Steel industry
https://telegram.me/aedahamlibra
(c) Gulf of Kutch
(d) Andaman and Nicobar Islands
(d) Fertilizer industry Ans: (a)
Ans: (c) Q161. Which one of the following pairs of places does
Q153. Where in India is the biggest steel the National highway number 2 join ?
manufacturer of the world (a) Delhi – Amritsar
(L.N. Mittal Group) setting up a steel plant ? (b) Delhi – Mumbai
(a) Jharkhand (b) Bihar (c) Delhi – Kolkata
(c) Orissa (d)Chhattisgarh (d) Delhi – Ahmedabad
Ans: (a) Ans: (c)
Q154. ―Manchester‖ of South India is : Q162. Which one of the following is a major port on
(a) Coimbatore (b) Madurai the East Coast of India ?
(c) Bengaluru (d) Chennai (a) Kandla
Ans: (a) (b) Vishakhapatnam
Q155. Which of the following is correctly matched (c) Karikal
with regard to thermal power project ? (d) Pondicherry (Puducherry)
(a) Talcher – Andhra Pradesh Ans: (b)
(b) Korba – Uttar Pradesh Q163. Which of the following places are connected to
(c) Kawas – Gujarat the highest road of the world ?
(d) Ramagundam – Madhya Pradesh (a) Gangtok and Kathmandu
Ans: (c) (b) Leh and Srinagar
Q156. The first oil refinery in India was set up at (c) Manali and Leh
(a) Barauni (d) Srinagar and Siachen
(b) Vishakhapatnam Ans: (b)
(c) Digboi Q164. Roads that link the important cities of various
(d) Mumbai states are referred to as :
Ans: (c) (a) State Roads
Q157. Which of the following industries is at Koraput (b) National Highways
? (c) State Highways
(a) Ship Building (d) Superways
(b) Aircrafts Building Ans: (b)
(c) Iron and Steel Q165. How many major sea ports are there in India ?
(d) Electric Locomotives (a) 6 (b) 9
Ans: (b) (c) 10 (d) 12
Q158. In India, the Black Revolution is related to the Ans: (*)
manufacturing of which product? Q166. Which two of the following are connected by
(a) Charcoal the North South corridor ?
(b) Crude petroleum (a) Srinagar and Kanyakumari
(c) Diamond (b) Mumbai and Chennai
(d) Black gram (c) Amritsar and Kolkata
Ans: (b) (d) Hyderabad and Bhopal
Q159. SIDO is related to the development of Ans: (a)
(a) Small industries Q167. The Jawaharlal Nehru Port is located at
(b) Steel Industry (a) Paradip (b) Cochin
(c) Soap Industry (c) Mumbai (d) Kolkata
(d) Sugar Industry Ans: (c)
Ans: (a) Q168. India‘s first Railway University will come up at
Q160. The proposed sea -route ―Sethu Samudram‖ is (a) Vadodara, Gujarat
a canal through which of the sea-lanes ? (b) Bengaluru, Karnataka
(a) Gulf of Mannar (c) Hyderabad, Andhra Pradesh
(b) Malacca Strait (d) Lucknow, Uttar Pradesh
Page 356 of 516
https://telegram.me/aedahamlibra
Ans: (a)
Q169. In which of the following States is Dampa T iger
(d) Arunachal Pradesh
Ans: (d)
Reserve situated? Q178. Which of the following border is known as
(a) Assam (b) Karnataka Radcliffe line ?
(c) Mizoram (d) Orissa (a) India and China
Ans: (c) (b) India and Bangladesh
Q170. Which of the following districts is on the (c) India and Pakistan
international border of India? (d) India and Afghanistan
(a) Sirsa (b) Anantnag Ans: (c)
(c) Karimganj (d) Purulia Q179. Which of the following states is called ‗Tiger
Ans: (c) State‘ of India?
Q171. The maximum concentration of scheduled caste (a) Himachal Pradesh
population is in the (b) Gujarat
(a) Indo-Gangetic Plains (c) Madhya Pradesh
(b) North-East India (d) Assam
(c) Western Coast Ans: (c)
(d) Eastern Coast Q180. Where are the ‗Todas‘ found?
Ans: (a) (a) Madhya Pradesh
Q172. Ethnic group Mongoloids are found in India in (b) Rajasthan
: (c) Tamil Nadu
(a) Southern region (d) Arunachal Pradesh
(b) South-central region Ans: (c)
(c) North-western region Q181. According to 1991 census, in which State/Union
(d) North-eastern region Terriroty has the highest percentage of scheduled
Ans: (d) castes population ?
Q173. Which of the following states does not border (a) Pujnab
on Myanmar ? (b) Uttar Pradesh
(a) Mizoram (b) Manipur (c) West Bengal
(c) Nagaland (d) Meghalaya (d) Karnataka
Ans: (d) Ans: (a)
Q174. Which of the following States does not border Q182. Which among the following States has lowest
on Myanmar? rate of literacy according to 2001 census ?
(a) Mizoram (b) Assam (a) Gujarat (b) Rajasthan
(c) Nagaland (d) Manipur (c) Uttar Pradesh (d) Bihar
Ans: (b) Ans: (d)
Q175. Which district of UP is bordered by an Q183. As per 2001 Census, what is the percentage of
international border ? India‘s population to the world population ?
(a) Ballia (b) Bahraich (a) 8 (b) 16
(c) Basti (d) Barabanki (c) 26 (d) 28
Ans: (b) Ans: (b)
Q176. Cities with population from one to five million Q184. According to Census 2001, what is the
are called percentage of Indian population below the age 40 ?
(a) Conurbation (a) 75 (b) 70
(b) Million City (c) 65 (d) 60
(c) Metropolitan Ans: (a)
(d) Cosmopolitan Q185. A high growth rate of population is
Ans: (c) characterised by
Q177. Apatanis are the major tribal group of (a) high birth and high death rates
(a) Jharkhand (b) high birth and low death rates
(b) Nagaland (c) low birth and low death rates
(c) Sikkim (d) low birth and high death rates
Page 357 of 516
https://telegram.me/aedahamlibra
Ans: (b)
Q186. Which State of India has the largest percentage
Q195. The state which has recently overtaken
Karnataka for the top rank in the production of bio -
of poor? fertilisers is :
(a) Bihar (b) Chhattisgarh (a) Tamil Nadu (b) Gujarat
(c) Orissa (d) Jharkhand (c) Maharashtra (d) Punjab
Ans: (a) Ans: (a)
Q187. The most literate union territory in India is Q196. As per the provisional result of the 2011
(a) Delhi (b) Lakshadweep Census, the density of population in India is
(c) Chandigarh (d) Pondicherry (a) 325 (b) 352
Ans: (b) (c) 372 CD 382
Q188. Which is the largest state of India, population - Ans: (d)
wise, according to 2001 census ? Q197. Which of the following registers the highest
(a) Maharashtra density in the country as per Census 2011 ?
(b) Bihar (a) Delhi (b) Chandigarh
(c) Uttar Pradesh (c) Puducherry (d) West Bengal
(d) West Bengal Ans: (a)
Ans: (c) Q198. As per 2011 Census data, the child sex ratio has
Q189. Diu is an island off come down from 927 to :
(a) Daman (b) Goa (a) 904 (b) 920
(c) Gujarat (d) Maharashtra (c) 917 (d) 914
Ans: (c) Ans: (d)
Q190. The least populated State in India is Q199. Which Union Territory is at the bottom of
(a) Arunachal Pradesh child-sex-ratio?
(b) Sikkim (a) Dadra and Nagar Haveli
(c) Mizoram (b) Chandigarh
(d) Uttarakhand (c) Andaman and Nicobar Islands
Ans: (b) (d) Daman and Diu
Q191. Which one of the following is not a desert Ans: (b)
district of Rajasthan? Q200. The study of population is called :
(a) Kota (b) Barmer (a) Cartography (b)Anthropology
(c) Jaisalmer (d) Churu (c) Demography (d) Biography
Ans: (a) Ans: (c)
Q192. The State which has developed wind energy is Q201. Density of population shows:
(a) Gujarat (a) Land capital ratio
(b) Karnataka (b) Land product ratio
(c) Kerala (c) Land labour ratio
(d) Andhra Pradesh (d) Man land ratio
Ans: (a) Ans: (d)
Q193. Which of the following state is most famous for Q202. The perc entage of India‘s total population
its beautiful sea beaches? employed in agriculture is nearly
(a) Gujarat (b) Goa (a) 60% (b) 50%
(c) Tamil Nadu (d) Orissa (c) 70% (d) 80%
Ans: (b) Ans: (a)
Q194. Which State of India has the largest area ? Q203. Which of the following states has the highest
(a) Uttar Pradesh literacy rate?
(b) Madhya Pradesh (a) Goa
(c) Maharashtra (b) Karnataka
(d) Rajasthan (c) Mizoram
Ans: (d) (d) Himachal Pradesh
Ans: (c)

Page 358 of 516


called as seven sister states ?
https://telegram.me/aedahamlibra
Q204. Which o f the following States are together Q212. The first short based integrated steel plant in
the country is
(a) Tripura, Meghalaya, Assam, Arunachal Pradesh, (a) Vijaynagar
Nagaland, Manipur, Mizoram (b) Salem
(b) Sikkim, West Bengal, Assam, Arunachal Pradesh, (c) Vishakhapatnam
Nagaland, Manipur, Mizoram (d) Bhadravati
(c) Sikkim, West Bengal, Meghalay a, Assam, Arunachal Ans: (c)
Pradesh, Nagaland, Tripura Q213. The Wheeler Island has been renamed as
(d) Tripura, Meghalaya, Assam, Orissa, Sikkim, (a) Vikram Sarabhai Island
Manipur, Mizoram (b) Satish Dhawan Island
Ans: (a) (c) Abdul Kalam Island
Q205. Which State of India is leading in solar energy (d) C.V. Raman Island
generation? Ans: (c)
(a) Gujarat (b) Rajasthan
(c) Haryana
(d) Uttar Pradesh
World Geography
Ans: (b)
Q206. Which State has the longest coastline? Q1. Where is Lake Superior, the largest freshwater
(a) Andhra Pradesh lake in the world, located?
(b) Maharashtra (a) USA (b) Brazil
(c) Gujarat (c) Canada (d) Russia
(d) Karnataka Ans: (a)
Ans: (c) Q2. Match the rivers flowing through the cities below
Q207. Raipur is the capital of which Indian State? :
(a) Jharkhand City River
(b) Goa (a) Rotterdan 1. Seine
(c) Himachal Pradesh (b) Paris 2. Potomac
(d) Chhattisgarh (c) Budapest 3. Rhine
Ans: (d) (d) Washington4. Danube
Q208. Which cities were f irst connected by the STD (a) (b) (c) (d)
services? (a) 2 3 1 4
(a) Delhi and Mumbai (b) 1 3 4 2
(b) Delhi and Kolkata (c) 3 1 4 2
(c) Kanpur and Delhi (d) 4 3 2 1
(d) Kanpur and Lucknow Ans: (c)
Ans: (d) Q3. Where is ―Ground Zero‖ ?
Q209. Which of the following is called the ‗shrimp (a) Greenwich (b) New York
capital of India‘ ? (c) Indira Point (d) Shriharikota
(a) Mangalore (b) Nagapatnam Ans: (b)
(c) Kochi (d) Nellore Q4. Which one of the following parts of the world
Ans: (d) does not receive rainfall any time of the year ?
Q210. Which of the following districts lies to the east (a) Central Europe
of the district Farukhabad? (b) Central North America
(a) Hardoi (b) Etawah (c) Polar regions
(c) Badaun (d) Jalaun (d) Sub-Saharan areas
Ans: (a) Ans: (c)
Q211. Which one of the following is known as the Q5. Black Forests are found in
‗Pearl City‘ ? (a) France (b) Germany
(a) Kandla (b) Tuticorin (c) Czechoslovakia
(c) Kochi (d) Hyderabad (d) Rumania
Ans: (d)
Page 359 of 516
https://telegram.me/aedahamlibra
Q6. In which country is the volcano Mount
Ans: (b) d. Lahore 4. Thames a b c d
(a) 2 3 4 1
Gamkonora, the highest peak of Halmahera island, (b) 3 2 1 4
which erupted in July 2007 located? (c) 1 4 3 2
(a) Japan (b) Indonesia (d) 4 1 2 3
(c) Russia (d) France Ans: (a)
Ans: (b) Q15. The Strait that connects Red Sea and
Q7. The Melanesian Island Group lies in the Mediterranean Sea is
(a) Pacific Ocean (a) Panama Strait
(b) Atlantic Ocean (b) Suez Strait
(c) Indian Ocean (c) Palk Strait
(d) Arctic Ocean (d) Bering Strait
Ans: (a) Ans: (b)
Q8. Selvas are Q16. "Yosemite" is a
(a) Huge Canadian forests (a) River (b) Peak
(b) Equatorial rain forests of Brazil (c) Waterfall (d) Dam
(c) Coniferous forests of Siberia Ans: (c)
(d) Evergreen monsoon forests Q17. Match correctly the foll owing deserts and their
Ans: (b) location by choosing the correct response:
Q9. Source of River Nile is Desert
(a) Lake Nassir a. Kalahari b. Atacama
(b) Lake Victoria c. Thar d. Great Victoria Location
(c) Lake Chad (i) South America
(d) Lake Tanganyika (ii) Australia
Ans: (b) (iii) Africa
Q10. Which one of the following is the largest lake in (iv) Asia
the world? (a) a- (ii), b- (iii), c- (i), d- (iv)
(a) Lake Superior (b) a- (iv), b- (iii), c- (ii), d- (i)
(b) Caspian Sea (c) a- (iii), b- (ii), c- (i), d- (iv)
(c) Lake Baikal (d) a- (iii), b- (i), c- (iv), d- (ii)
(d) Lake Victoria Ans: (d)
Ans: (b) Q18. The ―Grand Canyon‖ is on the river
Q11. The Chinese river known as yellow river is (a) Colorado (b) Columbia
(a) Hwang-Ho (b) Sikiang (c) Ohio (d) Mississippi
(c) Yangtse-Kiang (d) Mekong Ans: (a)
Ans: (a) Q19. The region famous for many kinds of wine and
Q12. The world famous ‗Serangeti Wildlife Sanctuary champagne is
is located in (a) Eastern Europe
(a) Kenya (b) Tanzania (b) Western Europe
(c) Zambia (d) Uganda (c) Mediterranean
Ans: (b) (d) Glassland
Q13. The country located between 8 ° N and 37° N Ans: (c)
latitudes is Q20. Sunda Trench is in
(a) Bangladesh (b) India (a) Indian Ocean
(c) China (d) Pakistan (b) Pacific Ocean
Ans: (b) (c) Atlantic Ocean
Q14. Match the following : (d) Gulf of Mexico
Towns Rivers Ans: (a)
a. Jabalpur 1. Ravi Q21. The temperate grasslands of North America are
b. Paris 2. Narmada known as
c. London 3. Siene (a) Pampas (b) Downs
Page 360 of 516
(c) Steppes https://telegram.me/aedahamlibra
(d) Prairies
Ans: (d)
(c) Siberia and Sweden
(d) Canada and Alaska
Q22. Which one of the following is the world‘s largest Ans: (d)
desert ? Q31. The ‗Masai‘ is a primitive tribe of
(a) Arabian (b) Kalahari (a) Angola (b) Botswana
(c) Sahara (d) Thar (c) Nigeria (d) Tanzania
Ans: (c) Ans: (d)
Q23. The longest river in the world is Q32. What do you mean by ‗Density of Population‘?
(a) Ganga (b) Nile (a) Ratio of people living below poverty line to total
(c) Brahmaputra (d) Amazon population
Ans: (b) (b) Number of persons live per square kilometre
Q24. Which of the following is the only volcanic peak (c) Number of persons in a city
in Antarctica? (d) Number of persons living per kilometre
(a) Mt. Blanc (b) Mt. Erebus Ans: (b)
(c) Mt. Elbrus (d) Mt. Cook Q33. The part of Equatorial region which has well
Ans: (b) developed rubber plantations is
Q25. Match List I and List II and mark the correct (a) Amazon basin (b) Indonesia
answer. (c) Malaysia (d) Zaire basin
List I List II Ans: (c)
(Climate type) (Station) Q34. Which one of the following countries is the
a. Taiga 1. Jacobabad largest producer of uranium in the world ?
b. monsoon 2. India (a) Canada (b) South Africa
c. Alpine 3. Veckhoyansk (c) Namibia (d) U.S.A.
d. Desert 4. La Paz Ans: (a)
(a) a = 3, b = 2, c = 4, d = 1 Q35. The largest producer of world‘s mica is
(b) a = 4, b = 2, c = 1, d = 3 (a) U.S.A. (b) U.K.
(c) a = 2, b = 4, c = 3, d = 1 (c) Canada (d) India
(d) a = 3, b = 4, c = 1, d = 2 Ans: (*)
Ans: (a) Q36. Leading producer of rice is
Q26. Geothermal energy is maximum utilised in (a) India (b) Pakistan
(a) Iceland (b) New Zealand (c) China (d) Burma
(c) Russia (d) Japan Ans: (c)
Ans: (a) Q37. Which of the following countries has highest
Q27. Which of the following is called ‗Queen of the percentage of land under cultivation?
Adriatic‘? (a) U.S.A. (b) India
(a) Venice (b) Rome (c) China (d) Canada
(c) Flanders (d) Lisbon Ans: (b)
Ans: (a) Q38. Detroit, USA, is famous for which of the
Q28. Which country is known as ‗Land of Midnight following industries ?
Sun‘? (a) Iron and Steel
(a) Sweden (b) Norway (b) Automobile
(c) Germany (d) Finland (c) Petro-chemical
Ans: (b) (d) Cotton textile
Q29. Suez Canal is located in which of the following Ans: (b)
countries ? Q39. Largest Mica deposits are in
(a) Sudan (b) Saudi Arabia (a) South Africa (b) India
(c) Egypt (d) Jordan (c) USA (d) Australia
Ans: (c) Ans: (b)
Q30. The Eskimos live in Q40. The world‘s largest p roducer of desalinated sea
(a) Finland and Norway water is
(b) Norway and Siberia (a) Kuwait (b) Iran
Page 361 of 516
(c) Saudi Arabia https://telegram.me/aedahamlibra
(d) Iraq
Ans: (c)
Q48. The l ongest railway line in the world connects
_______
Q41. Which one of the following is correctly matched (a) New York and Seattle
? (b) Leningrad and Vladivostok
a. North China : Rice (c) Trivandrum and Guwahati
b. South China : Wheat (d) Perth and Sydney
c. Candy Basin : Coffee Ans: (b)
d. Malaysia : Natural Rubber Q49. New Britain and New Ireland are parts of–
(a) a (b) b (a) USA
(c) c (d) d (b) Canada
Ans: (d) (c) Australia
Q42. Ports of the Baltic Sea remain open for trade (d) Papua New Guinea
even during winter because Ans: (d)
(a) It lies in the tropical belt. Q50. The two countries connected by a tunnel under
(b) North Atlantic Drift, a warm ocean current flows in the sea are
the region. (a) England and France
(c) Local winds keep it warm. (b) France and Denmark
(d) Western disturban ces cause abrupt rise in the (c) Australia and New Zealand
temperature. (d) Spain and Italy
Ans: (b) Ans: (a)
Q43. Which one of the following countries has no Q51. What is the new name of the old colony of
mineral deposits ? Northern Rhodesia?
(a) Switzerland (b) Austria (a) Zambia (b) Zimbabwe
(c) Norway (d) Sri Lanka (c) Uganda (d) Tanzania
Ans: (a) Ans: (a)
Q44. Which of the following is the world‘s largest Q52. What is called as the ‗Roof‘ of the World ?
mining port ? (a) Indira Point (b)Kanchenjunga
(a) Nhavasheva (b) Hamilton (c) Pamir Knot (d) Indira Col
(c) Hedland (d) Pardon Ans: (c)
Ans: (c) Q53. Excluding the East European countries and
Q45. Match the following : Russia, the country with the larg est area under forest,
I II Crops First in the world is
A. Rice 1. Brazil (a) USA (b) Canada
B. Maize 2. Malaysia (c) Australia (d) Brazil
C. Rubber 3. China Ans: (d)
D. Sugarcane 4. U.S.A. Q54. ‗Nippon‘ is the name given to Japan which
(a) A-1, B-3, C-4, D-2 means
(b) A-2, B-1, C-3, D-4 (a) land of the rising Sun
(c) A-3, B-4, C-2, D-1 (b) land of Pagodas
(d) A-4, B-2, C-1, D-3 (c) land of northern lights
Ans: (c) (d) land of volcanoes and earthquakes.
Q46. Cuba is the largest producer of Ans: (a)
(a) Barley (b) Sugar Q55. Male is the capital of
(c) Wheat (d) Rice (a) Mongolia (b) Monaco
Ans: (*) (c) Maldives (d) Mauritius
Q47. One of the leading producers of asbestos in the Ans: (c)
world is Q56. What is called as the ‗Roof‘ of the World ?
(a) Australia (b) Russia (a) Indira Point (b) Kanchenjunga
(c) Canada (d) Armenia (c) Pamir Knot (d) Indira Col
Ans: (b) Ans: (c)

Page 362 of 516


https://telegram.me/aedahamlibra
Q57. Which of the following regions is re ferred to as
Garden of Eden as mentioned in the Bible ? Q6. Which of the following Mahatma Gandhi series of
Ans: (c)

(a) Dead Sea currency notes issued by the RBI has ―ecology‖
(b) Kutch region of South Iraq depicted on it?
(c) Nile Valley (a) Rs. 500 (b) Rs. 100
(d) Cango Valley (c) Rs. 50 (d) Rs. 5
Ans: (b) Ans: (b)
Q58. Which among the following policy of Life Q7. What are ―Open Market Operations‖?
Insurance Company is related to regular old -age (a) Activities of SEBI registered brokers
pension? (b) Selling of currency by the RBI
(a) Jivan Kishore (c) Selling of gilt-edged securities by the Government
(b) Jivan Chhaya (d) Sale of shares by FIIs
(c) Jivan Sanchay Ans: (c)
(d) None of these Q8. The best way, a bank can avoid loss is to
Ans: (d) (a) lend only to individuals known to the bank
(b) accept sound collateral
(c) give only short-term loans
Indian Economy (d) lend only to bank‘s old customers
Ans: (b)
Q1. The Indian economy can be most appropriately Q9. Which of the following is not an objective of the
described as a : monetary policy of the RBI ?
(a) Capitalist economy (a) Boost economic development
(b) Socialist economy (b) Direct credit in desirable direction
(c) Traditional economy (c) Control inflationary pressure
(d) Mixed economy (d) Ensure social justice
Ans: (d) Ans: (d)
Q2. What i s the maximum amount of investment in Q10. Merchant Banking is an institution which
the shares of debentures of notified companies like the provides finances to :
ICICI, the IDBI etc. that will entitle a rebate in (a) domestic whole sale trade
income tax up to 20% of the amount invested? (b) international trade among countries
(a) Rs. 80000 (b) Rs. 60000 (c) domestic retail trade among
(c) Rs. 20000 (d) Rs. 10000 (d) international aid agencies.
Ans: (a) Ans: (b)
Q3. Which of the following PSUs has been privatised? Q11. The Government of India made it obligatory on
(a) HZL (b) CMC the part of all comm ercial banks that they should give
(c) Hotel Corporation of India some cash amount while purchasing Government
(d) NALCO bonds. What would you call this?
Ans: (a) (a) Statutory Liquidity Ratio
Q4. For whom was the first departmental life (b) Cash Reserve Ratio
insurance started ? (c) Minimum Reserve Ratio
(a) Army (d) Floating Reserve Ratio
(b) Civil officers of Central Government Ans: (a)
(c) Employees of postal department Q12. Which amidst the follow ing taxes collected by
(d) Life Insurance Corpor-ation the Union is NOT mandated to be assigned to the
Ans: (c) States?
Q5. The proceeds of income tax go to (a) Terminal taxes on goods or passengers carried by
(a) Central Government railway, sea or air.
(b) State Government (b) Taxes on railway fares and freights.
(c) Centre and States (c) Taxes on consignment of goods.
(d) Corporation authorities (d) Service Tax.
Ans: (d)
Page 363 of 516
Q13. Inflation is caused by :
(a) Increase in supply of goods
https://telegram.me/aedahamlibra
(c) Income tax, Customs duty and House tax
(d) Customs duty, Entertainment tax and Income tax
(b) Increase in cash with the government Ans: (b)
(c) Decrease in money supply Q21. Punjab National Bank has been honoured wi th
(d) Increase in money supply the Golden Peacock Award 2002 for excellence in—
Ans: (d) (a) agricultural finance
Q14. Open market operation refers to (b) rural industrialisation
(a) borrowing by commercial banks from the R.B.I. (c) housing development
(b) lending by scheduled banks to non-scheduled banks (d) corporate excellence
(c) purchase and sale of Government securities by the Ans: (d)
R.B.I. Q22. Which authority decides about the States‘ share
(d) purchase and sale of bonds and securities by the in central taxes?
Central Govt. (a) Finance Commission
Ans: (c) (b) Planning Commission
Q15. What is known as the open market operation of (c) Election Commission
the RBI ? (d) Finance Ministry
(a) Buying and selling of stocks Ans: (a)
(b) Auctioning of foreign exchange Q23. In which plan was self-reliance first emphasised
(c) Trading in securities (a) Second Plan (b) Third Plan
(d) Transactions in gold (c) Fourth Plan (d) Fifth Plan
Ans: (c) Ans: (d)
Q16. The Government resorts to devaluation of its Q24. India‘s biggest nationalised enterprise today
currency in order to promote (a) the Indian Railways
(a) national income (b) the Indian Commercial Banking System
(b) international goodwill (c) the Indian Power Sector
(c) exports (d) the Indian Telecommu-nication System
(d) savings Ans: (a)
Ans: (c) Q25. In the budget figures of the Government of
Q17. Which car has been the best seller in India in India, interest payments, subsidies, pensions, social
2004-2005 ? services and the like are parts of the
(a) Maruti 800 (a) Plan Expenditure
(b) Maruti Suzuki Alto (b) State Government Expenditure
(c) Tata Indica (c) Public Debt in the form of Capital Expenditure
(d) Santro Xing (d) Non-plan Expenditure
Ans: (b) Ans: (d)
Q18. Which of the following is apex bank for Q26. Which one of the following is not correct ?
industrial loans ? (a) First Five Year Plan–1951–56
(a) RBI (b) NABARD (b) Second Five Year Plan1956– 61
(c) ICICI (d) IDBI (c) Third Five Year Plan–1961– 66
Ans: (d) (d) Fourth Five Year Plan–1966–71
Q19. Which of the following taxes is levied by the Ans: (d)
Union and appropriated and planned by the states ? Q27. With the inclusion of Shipping Corporation of
(a) Service tax India recently in the list of Nav Ratna PSEs, their
(b) Stamp duty number now stands at
(c) Property tax (a) 15 (b) 16
(d) Passenger and freight duty (c) 17 (d) 18
Ans: (b) Ans: (b)
Q20. Which of the following sets belong to Central tax Q28. Which is not the objective of Public
? Procurement and Distribution system followed by
(a) Excise duty, Sales tax and Customs duty Indian Government ?
(b) Excise duty, Customs duty and Income tax
Page 364 of 516
stocks
https://telegram.me/aedahamlibra
(a) Maintain price stabi lity through creation of buffer (a) 1998
(c) 1999
(b) 1996
(d) 2000
(b) Protect the interests of both consumers and poor Ans: (d)
farmers Q36. Where is the biggest private sector power
(c) Control the production of food grains project in India located?
(d) Reduce personal and regional inequality in the (a) Rajahmundry in Andhra Pardesh
distribution (b) Neyveli in Tamil Nadu
Ans: (c) (c) Korba in Madhya Pradesh
Q29. Token privatisation or deficit privatisation of (d) Dabhol in Maharashtra
public sector units occur when the government sells Ans: (*)
(a) 5% of shares Q37. Which of the following is not a part of national
(b) 10% of shares income?
(c) 15 % of shares (a) Wages and Salaries
(d) 20% of shares (b) Profits
Ans: (a) (c) Rent
Q30. EXIM Policy, 2002-07, has set a target to achieve (d) Interest on national debt
a share in the global trade by 2007 at Ans: (d)
(a) 0.5 per cent Q38. In which sector of the Indian economy i s
(b) 1.0 per cent productivity the highest ?
(c) 1.5 per cent (a) Manufacturing
(d) 2.0 per cent (b) Transport, Communication and Commerce
Ans: (b) (c) Agriculture
Q31. The term ‗Mixed Economy‘ denotes (d) Other sectors
(a) existence of both rural and urban sectors Ans: (a)
(b) existence of both private and public sectors Q39. The gift edged market in the capital market of
(c) existence of both heavy and small industries India refers to
(d) existence of both developed and underdeveloped (a) long-term private securities
sectors (b) market dealing in existing securities.
Ans: (b) (c) market for corporate securities
Q32. The present Indian monetary system is based on (d) market for Government securities
(a) Gold Reserve System Ans: (b)
(b) Proportional Reserve System Q40. Banks in India were nationalised for the first
(c) Convertible Currency System time in the year –
(d) Minimum Reserve System (a) 1950 (b) 1960
Ans: (d) (c) 1969 (d) 1979
Q33. Wholesale price based inflation rate in India Ans: (c)
reached its highest level in 13 years on 27th July, Q41. The Reserve Bank of India issue s currency notes
2008. It was under
(a) 11.75 per cent (a) fixed fiduciary system
(b) 11.85 per cent (b) maximum fidciuary system
(c) 12.00 per cent (c) minimum reserve system
(d) 12.05 per cent (d) proportional reserve system
Ans: (*) Ans: (c)
Q34. The abbreviation ‗SEBI‘ stands for Q42. In Centre -State financial relations in India,
(a) Savings and Exchange Bank of India Gadgil Formula is used in
(b) Securities and Exchange Bank of India (a) division of tax revenue
(c) Survey of essential business in India (b) formulating the policy for fresh borrowings
(d) Securities and Exch-ange Board of India (c) writing off States‘ indebtedness to the Centre
Ans: (d) (d) allocating Central Plan assistance between States
Q35. The Annapurna Scheme was implemented in the Ans: (a)
year
Page 365 of 516
https://telegram.me/aedahamlibra
Q43. The single largest item of expenditure of the
Central Government in India in recent years is
(d) Sardar Patel
Ans: (b)
(a) Defence Q51. While computing national income estimates,
(b) Subsidies which of the following is required to be observed ?
(c) Interest payment (a) The value of exports to be added and the value of
(d) General services imports to be subtracted
Ans: (c) (b) The value of exports to be subtracted and the value of
Q44. Cochin refineries is in : imports to be added
(a) Public Sector (c) The value of both exports and imports to be added
(b) Joint Sector (d) The value of both exports and imports to be
(c) Private Sector subtracted
(d) Co-operative Sector Ans: (a)
Ans: (b) Q52. The best example of a capital intensive industry
Q45. The first state owned company from India to be in India is
listed on the New York Stock Exchange is (a) Textile Industry
(a) Videsh Sanchar Nigam Ltd. (b) Steel Industry
(b) Mahanagar Telephone Nigam Ltd. (c) Tourism Industry
(c) Tata Iron and Steel Company (d) Sports Goods Industry
(d) Wipro Ans: (b)
Ans: (a) Q53. After ONGC, OIC, NTPC and SAIL, the
Q46. Which amidst the following is a PSU ? ‗Navratna‘ PSU which was awarded ‗Maharatna‘
(a) Bank of Rajasthan status is
(b) ICICI Bank (a) HAL (b) GAIL
(c) Corporation Bank (c) Coal India Ltd (d) BHEL
(d) Citibank Ans: (c)
Ans: (c) Q54. Which of the following is an example of Joint -
Q47. Which from the following is NOT a Navaratna Sector enterprise in India?
PSU ? (a) Maruti Udyog Limited
(a) SAIL (b) BHEL (b) The Indian Oil Corporation
(c) NTPC (c) Hindustan Antibiotics Ltd.
(d) Shipping Corporation of India (d) Bharat Aluminium Ltd.
Ans: (*) Ans: (a)
Q48. The Government of India derives its single Q55. Who amongst the following has never been the
largest source of revenue from Governor of Reserve Bank of India ?
(a) Direct Taxes (a) D. Subbarao
(b) Customs Duties (b) C. Rangarajan
(c) Deficit Financing (c) B.B. Bhattacharya
(d) Union Excise Duties (d) Y.V. Reddy
Ans: (d) Ans: (c)
Q49. What does the open market operation of the RBI Q56. The main difference between Gross Domestic
mean ? Product (GDP) and Gross National Product (GNP is
(a) Buying and selling shares (a) Transfer payments
(b) Auctioning of foreign exchange (b) Net foreign income from abroad
(c) Trading is securities (c) Capital consumption allowance
(d) Transactions in gold (d) Capital gains
Ans: (c) Ans: (b)
Q50. Who estimated the National Income for the first Q57. Which of the following Government of India
time in India ? programmes aims to help, build or upgrade dwelling
(a) Mahalanobis units of below the poverty line rural families ?
(b) Dadabhai Naoroji (a) National Social Assistance Programme
(c) V.K.R.V. Rao (b) Jawahar Rozgar Yojana
Page 366 of 516
(c) Indira Awaas Yojana https://telegram.me/aedahamlibra
(d) Jawaharlal Nehru National Urban Renewal Mission
Ans: (c)
Q66. As an export item of India, which spice occupies
Ans: (c) the top position in value ?
Q58. Who was the head of the 10th Finance (a) Pepper (b) Chillies
Commission ? (c) Turmeric (d) Cardamom
(a) Manmohan Singh Ans: (b)
(b) Vasant Sathe Q67. The second plan gave priority to
(c) Shiv-Shankar (a) Agriculture
(d) K.C Pant (b) Services
Ans: (d) (c) Heavy Industry
Q59. Indian Special Economic Rules amendment (d) Foreign Trade
came in the year Ans: (c)
(a) 2000 (b) 2002 Q68. About how many Indians cannot meet their
(c) 2004 (d) 2006 essential needs as per a report by McKinsey Global
Ans: (d) Institute
Q60. From which of the following banks did Madan (MGI) released on 19th February, 2014 ?
Mohan Malaviya take loans for financing ―The (a) 66% (b) 56%
Hindustan Times‖ ? (c) 46% (d) 36%
(a) Punjab National Bank Ans: (b)
(b) Bank of Maharashtra Q69. The market in which loans of money can be
(c) Bank of Baroda obtained is called
(d) State Bank of India (a) Reserve market
Ans: (a) (b) Institutional market
Q61. Which of the following does not form a part of (c) Money market
the foreign exchange reserves of India ? (d) Exchange market
(a) Gold (b) SDRs Ans: (c)
(c) Foreign currency assets Q70. Which of the following is the Regulator of the
(d) Foreign currency and securities held by the banks and credit rating agencies in India ?
corporate bodies (a) RBI (b) SBI
Ans: (d) (c) SIDBI (d) SEBI
Q62. Imperial Bank was constituted in the year : Ans: (*)
(a) 1930 (b) 1935 Q71. Which of the following method is not used in
(c) 1955 (d) 1921 determining National Income of a country ?
Ans: (d) (a) Income Method
Q63. The Oilseeds Production Programme (b) Output Method
(OPP) was launched in (c) Input Method
(a) 1986 (b) 1987 (d) Investment Method
(c) 1988 (d) 1990 Ans: (d)
Ans: (a) Q72. Government of India has decided to
Q64. Agriculture should serve as an instrument of integrate____with recently launched Pradhan Mantri
income, livelihood and opportunity to the local Krishi Sinchayee Yojana.
community — this statement was given by (a) Mahatma Gandhi National Rural Employment
(a) Dr Madhavan Nair Guarantee Act
(b) Dr Manmohan Singh (b) National Rural Livelihood Mission
(c) Dr Abdul Kalam (c) Haryali
(d) Dr M S Swaminathan (d) Accelerated Irrigation Benefit Programme
Ans: (d) Ans: (a)
Q65. Rashtriya Krishi Bima Yojana was introduced Q73. State Bank of India was previously known as :
in (a) Imperial Bank of India
(a) 1992 (b) 1998 (b) Canara Bank
(c) 1999 (d) 1996 (c) Syndicate Bank
Page 367 of 516
(d) Co-operative Bank of India https://telegram.me/aedahamlibra
Ans: (a)
Ans: (a)
Q83. Fixed Foreign Exchange Rate can be changed by
Q74. Reserve bank of India was nationalised in : (a) RBI (b) SEBI
(a) 1949 (b) 1951 (c) Ministry of Finance
(c) 1947 (d) 1935 (d) FIPB
Ans: (a) Ans: (c)
Q75. The Industrial Development Bank of India was Q84. National Income of India is compiled by
set up in (a) Finance Commission
(a) July, 1968 (b) July, 1966 (b) Indian Statistical Institute
(c) July, 1964 (d) July, 1962 (c) National Development Council
Ans: (c) (d) Central Statistical Organization
Q76. Which bank is limited to the needs of agriculture Ans: (d)
and rural finance ? Q85. NTPC is a Central Public Sector Enterprise in
(a) RBI (b) SBI which sector?
(c) IFC (d) NABARD (a) Education (b) Health
Ans: (d) (c) Power (d) Transport
Q77. Lender of the Last Resort is : Ans: (c)
(a) SBI (b) IDBI Q86. The industry having the largest investment in
(c) NABARD (d) RBI Indian Economy is
Ans: (d) (a) Tea (b) Cement
Q78. What is the name of portal launched by RBI (c) Steel (d) Jute
recently to check illegal money collection? Ans: (c)
(a) Sahyog (b) Sahayata Q87. Securities and Exchange Board of India is a
(c) Sampark (d) Sachet (a) Quasi Judicial body
Ans: (d) (b) Regulatory Body
Q79. When was the Pr adhan Mantri Fasal Bima (c) Advisory Body
Yojana launched ? (d) Consititutional Body
(a) February 2000 Ans: (b)
(b) February 2015 Q88. Project ‗Sankalp‘ started for the purpose
(c) January 2016 ________
(d) February 1995 (a) To eradicate illiteracy
Ans: (c) (b) To eradicate Polio
Q80. Name the Indian State with the highest tax (c) To eliminate AIDS/HIV
revenue. (d) To eliminate unemployment
(a) Assam (b) Sikkim Ans: (c)
(c) Karnataka Q89. The nationalisation of major commercial banks
(d) Maharashtra took place in
Ans: (d) (a) 1947 (b) 1956
Q81. How do you calculate the poverty line? (c) 1969 (d) 1980
(a) Income of an individual under a threshold value Ans: (c)
published by Government of India Q90. The 14th Finance Commission has
(b) Income of any individual less than 50 INR in a day recommended increase in States share in net proceeds
(c) Average income of all the individuals in a country from tax collection from 32% to
(d) Income of a family less than 100 INR in a day (a) 35% (b) 40%
Ans: (a) (c) 42% (d) 45%
Q82. Which one is not the main objective of fiscal Ans: (c)
policy in India? Q91. Which State in India has introduced FAT Tax
(a) To increase liquidity in the economy on junk food
(b) To promote price stability (a) Rajasthan
(c) To minimize the inequalities of income & wealth (b) Kerala
(d) To promote employment opportunity (c) Andhra Pradesh
Page 368 of 516
(d) Bihar https://telegram.me/aedahamlibra
Ans: (b)
Q3. If the price of an inferior good falls, its demand
(a) rises
Q92. ―Himayat‖ is a training cumplacement (b) falls
programme for unemployed youth in the State of (c) remains constant
(a) Haryana (d) can be any of the above
(b) Punjab Ans: (a)
(c) Jammu and Kashmir Q4. Enterpreneurial ability is a special kind of labour
(d) Himachal Pradesh that
Ans: (c) (a) is hired out to firms at high wages
Q93. ‗Pradhan Mantri Jan-Dhan Yojana‘ has been (b) organizes the process of production
launched for : (c) produces new capital goods to earn interest
(a) Promoting financial inclusion in the country (d) manages to avoid losses by continual innovation
(b) Providing loans to poorest people in the country Ans: (b)
(c) Providing financial help to the marginalised Q5. When marginal utility is zero, the total utility is
community (a) Minimum (b) Increasing
(d) Promoting women in backward areas (c) Maximum (d) Decreasing
Ans: (a) Ans: (c)
Q94. As per the 2016 –17 Budget, the largest source of Q6. The ‗break-even point‘ is where
money to the Government of India is : (a) marginal revenue equals marginal cost
(a) Income Tax (b) average revenue equals average cost
(b) Corporation Tax (c) total revenue equals total cost
(c) Nontax revenues (d) None of these
(d) Borrowings and other liabilities Ans: (b)
Ans: (d) Q7. Given the money wages, if the price level in an
Q95. Which one of the following organisations is a economy increases, then the real wages will
financial institution ? (a) increase
(a) KVIC (b) IFCO (b) decrease
(c) SEBI (d) ICICI (c) remain constant
Ans: (c) (d) become flexible
Ans: (b)
Q8. Seawater, fresh air, etc., are regarded in
Economics Economics as
(a) Giffen goods
Q1. If an industry is char acterised by economies of (b) inferior goods
scale then (c) free goods
(a) barriers to entry are not very large (d) normal goods
(b) long run unit costs of production decreases as the Ans: (c)
quantity the firm produces increases Q9. Engel‘s Law states the relationship between
(c) capital requirement are small due to the efficiency of (a) quantity demanded and price of a commodity
the large scale operation (b) quantity demanded and price of substitutes
(d) the costs of entry into the market are likely to be (c) quantity demanded and tastes of the consumers
substantial (d) quantity demanded and income of the consumers
Ans: (b) Ans: (d)
Q2. When there is one buyer and many sellers then Q10. Knowledge, technical skill, education
that situation is called etc. in economics, are regarded as
(a) Monopoly (a) social-overhead capital
(b) Single buyer right (b) human capital
(c) Down right (c) tangible physical capital
(d) Double buyers right (d) working capital
Ans: (b) Ans: (b)
Q11. The main determinant of real wage is
Page 369 of 516
(a) extra earning
(b) nature of work
https://telegram.me/aedahamlibra
Q19. The demand for which of the following
commodity will not rise in spite of a fall in its price?
(c) promotion prospect (a) Television (b) Refrigerator
(d) purchasing power of money (c) Salt (d) Meat
Ans: (d) Ans: (c)
Q12. ―Interest is a reward for parting with liquidity‖ Q20. The situation in which total revenue is equal to
is according to total cost, is known as
(a) Keynes (b) Marshall (a) monopolistic competition
(c) Haberler (d) Ohlin (b) equilibrium level of output
Ans: (a) (c) break-even point
Q13. An exceptional demand curve is one that moves (d) perfect competition
(a) upward to the right Ans: (c)
(b) downward to the right Q21. Why is rent earned by land even in the long run
(c) horizontally ?
(d) vertically (a) Land has original and indestructible power
Ans: (b) (b) Land is a man made factor
Q14. Opportunity cost of production of a commodity (c) Its supply is inelastic in the short run
is (d) Its supply is inelastic in the long run
(a) the cost that the firm could have incurred when a Ans: (d)
different technique was adopted Q22. The four factors of production are
(b) the cost that the firm could have incurred under a (a) land, labour, capital, organisation,
different method of production (b) land, electricity, water, labour
(c) the actual cost incurred (c) labour, capital, land, rainfall,
(d) the next best alternative output (d) labour, climate, land, tools,
Ans: (d) Ans: (a)
Q15. Under which market condition do firms have Q23. If the change in demand for a commodity is at a
excess capacity? faster rate than change in the price of the commodity,
(a) Perfect competition the demand is
(b) Monopolistic competition (a) perfectly inelastic
(c) Duopoly (b) elastic
(d) Oligopoly (c) perlectly elastic
Ans: (b) (d) inelastic
Q16. The excess of price a person is to pay rather than Ans: (c)
forego the consumption of the commodity is called Q24. Different firms constituting the industry,
(a) Price (b) Profit produce homogeneous goods under
(c) Producers‘ surplus (a) monopoly
(d) Consumer‘s surplus (b) monopolistic competition
Ans: (c) (c) oligopoly
Q17. The ‗break-even‘ point is where (d) perfect competition
(a) marginal revenue equals marginal cost Ans: (d)
(b) average revenue equals average cost Q25. A situation of large number of firms producing
(c) total revenue equals total cost similar goods is termed as :
(d) None of the above (a) Perfect competition
Ans: (c) (b) Monopolistic competition
Q18. If the price of Pepsi decreases relative to the (c) Pure competition
price of Coke and 7-Up, the demand for (d) Oligopoly
(a) Coke will decrease Ans: (a)
(b) 7-Up will decrease Q26. Total fixed cost curve is
(c) Coke and 7-Up will increase (a) Vertical
(d) Coke and 7-Up will decrease (b) Horizontal
Ans: (d) (c) Positively Sloping
Page 370 of 516
(d) Negatively sloping https://telegram.me/aedahamlibra
Ans: (b) Q35. Name the curve which shows the quantity of
Ans: (b)

Q27. If the main objective of the government is to products a seller wishes to sell at a given price level.
raise revenue, it should tax commodities with (a) Demand curve
(a) high elasticity of demand (b) Cost curve
(b) low elasticity of supply (c) Supply curve
(c) low elasticity of demand (d) None of these
(d) high income elasticity of demand Ans: (c)
Ans: (c) Q36. Real wage is :
Q28. What is selling cost ? (a) Profit price level (b) Rent price level
(a) Cost incurred on transportation of commodities to (c) Interest price level (d) Money wage price
market level
(b) Cost incurred on promoting the sale of the product Ans: (d)
(c) Cost incurred on commission and salaries personnel Q37. Plant and machinery are
(d) Cost incurred on advertisement (a) Producers‘ goods
Ans: (b) (b) Consumers‘ goods
Q29. Consumer‘s sovereignty me Ans: (a) consumers (c) Distributors‘ goods
are free to spend their income as they like. (d) Free goods
(b) consumers have the power to manage the economy. Ans: (a)
(c) consumer‘s expenditures influence the alloca tion of Q38. Micro-economics is also called :
resources. (a) Income theory
(d) consumer goods are free from government control. (b) Investment theory
Ans: (a) (c) Price theory
Q30. A horizontal demand curve is (d) Expenditure theory
(a) ralatively elastic Ans: (c)
(b) perfectly inelastic Q39. When percentage change in demand for a
(c) perfectly elastic commodity is less than percentage change in its price,
(d) of unitary elasticity then demand is said to be
Ans: (c) (a) Highly elastic
Q31. The degree of monopoly power is to be measured (b) Inelastic
in terms of the firm‘s (c) Relatively elastic
(a) normal profit (d) Perfectly inelastic
(b) supernormal profit Ans: (b)
(c) both normal and supernormal profit Q40. Who developed the innovations theory of profit ?
(d) selling price (a) Walker (b) Clark
Ans: (b) (c) Knight (d) Schumpeter
Q32. Cost of production of the producer is given by: Ans: (d)
(a) sum of wages paid to labourers. Q41. Bilateral monopoly refers to the market
(b) sum of wages and interest paid on capital. situation of
(c) sum of wages, interest, rent and supernormal profit. (a) two sellers, two buyers
(d) sum of wages, interest, rent and normal profit. (b) one seller and two buyers
Ans: (d) (c) two sellers and one buyer
Q33. Which of the following is not a fixed cost ? (d) one seller and one buyer
(a) Salaries of administrative staff Ans: (d)
(b) Rent of factory biilding Q42. The opportunity cost of a factor of production is
(c) Property taxes (a) what it is earning in its present use.
(d) Electricity charges (b) what it can earn in the long period.
Ans: (a) (c) what has to be paid to retain it in its present use.
Q34. For an inferior good, demand falls when (d) what it can earn in some other use.
(a) price rises (b) income rise Ans: (d)
(c) price falls (d) income falls Q43. The basic object of all production is to
Page 371 of 516
(a) satisfy human wants
(b) provide employment
https://telegram.me/aedahamlibra
(d) go far advertising campaign.
Ans: (a)
(c) make profits Q51. If the supply curve is a straight line passing
(d) increase physical output through the origin, then the price elasticity of supply
Ans: (a) will be
Q44. Third stage of Law of Variable Proportion is (a) less than unity
called (b) infinitely large
(a) negative returns (c) greater than unity
(b) positive returns (d) equal to unity
(c) constant returns Ans: (d)
(d) increasing returns Q52. Which of the following occurs when labour
Ans: (a) productivity rises ?
Q45. ―Marginal Cost‖ equals (a) The equilibrium nominal wage falls.
(a) total cost minus total benefit for the last unit produced (b) The equilibrium quantity of labour falls.
(b) total cost divided by total benefit for the last unit (c) Competitive firms will be induced to use more capital
produced (d) The labour demand curve shifts to the right
(c) total cost divided by quantity Ans: (d)
(d) the change in total cost divided by the change in Q53. Which of the following is not an economic
quantity problem ?
Ans: (d) (a) Deciding between paid work and leisure
Q46. Which of the following economists is called the (b) Deciding between expenditure on one good and the
Father of Economics ? other
(a) Malthus (b) Robinson (c) Deciding between alternative methods of personal
(c) Ricardo (d) Adam Smith savings
Ans: (d) (d) Deciding between different ways of spending leisure
Q47. Price and output are determinates in market time
structure other than Ans: (d)
(a) monopoly Q54. Economics classifies the manmade instrument of
(b) perfect competition production as :
(c) oligopoly (a) organization (b) capital
(d) monopsony (c) equipment (d) labour
Ans: (b) Ans: (b)
Q48. Any factor of production can earn economic - Q55. A demand curve, which is parallel to the
rent, when its supply will be horizontal axis, showing quantity, has the price
(a) Perfectly elastic elasticity equal to
(b) Perfectly inelastic (a) Zero (b) One
(c) Elastic in nature (c) Less than one (d) Infinity
(d) All of the above Ans: (d)
Ans: (b) Q56. A ‗Market Economy‘ is one which
Q49. The law of diminishing returns applies to (a) is controlled by the Government
(a) All sectors (b) is free from the Government control
(b) Industrial sector (c) in influenced by international market forces
(c) Agricultural sector (d) All of these
(d) Service sector Ans: (b)
Ans: (a) Q57. Multiplier process in economic theory is
Q50. In short run, if a competitive firm incurs losses, conventionally taken to mean :
it will (a) the manner in which prices increase
(a) stop production. (b) the manner in which banks create credit
(b) continue to produce as long as it can cover its variable (c) income of an economy grows on account of an initial
costs. investment
(c) raise price of its product.
Page 372 of 516
increases
https://telegram.me/aedahamlibra
(d) the manner in which government expenditure (a) David Ricardo
(b) John Stuart Mill
Ans: (c) (c) Thomas Malthus
Q58. Say‘s Law of Market holds that (d) John Maynard Keynes
(a) supply is not equal to demand Ans: (d)
(b) supply creates its own demand Q66. ‗Supply creates its own demand‘.
(c) demand creates its own supply This statement is related to
(d) supply is greater than demand (a) Prof. J.B. Say
Ans: (b) (b) John Robinson
Q59. The method of calculating the national income (c) Adam Smith
by the product method is otherwise known as : (d) J.S. Mill
(a) Income method Ans: (a)
(b) Value added method Q67. Which one of the following is not a method of
(c) Expenditure method estimating National Income ?
(d) Net output method (a) Expenditure method
Ans: (d) (b) Product method
Q60. When aggregate supply exceeds aggregate (c) Matrix method
demand (d) Income method
(a) unemployment falls Ans: (c)
(b) prices rise Q68. An individual‘s actual standard of living can be
(c) inventories accumulate assessed by
(d) unemployment develops (a) Gross National Income
Ans: (c) (b) Net National Income
Q61. Investment and savings are kept equal through a (c) Per Capita Income
change in the level of (d) Disposable Personal Income
(a) Consumption Ans: (c)
(b) Investment Q69. ‗Personal Income‘ equals
(c) Government expenditure (a) The household sector‘s income
(d) Income (b) Private income minus savings of the corporate sector
Ans: (a) minus corporation tax
Q62. While determining income the expenditure on (c) Personal disposable income plus miscellaneous
which of the following items is not considered as receipts of the Goverment
investment ? (d) All of the above
(a) Construction of factory Ans: (c)
(b) Computer Q70. In accounting terms, what constitutes the
(c) Increase in the stock of unsold articles ‗closing stock‘?
(d) Stock and share in joint stock company (a) Net Investment
Ans: (c) (b) Gross Investment-Capital Losses
Q63. The term ‗Green GNP‘ emphasises (c) Opening Stock-Capital Losses
(a) rapid growth of GNP (d) Opening Stock + Net Investment – Capital Losses
(b) increase in per capita income Ans: (d)
(c) economic development Q71. Liquidity Preference means
(d) sustainable development (a) holding assets in the form of bonds and shares
Ans: (d) (b) holding assets in the form of cash
Q64. ―Supply creates its own demand‖ – Who said (c) creation of immovable property
this ? (d) assets in the form of jewellery
(a) J. B. Say (b) J. S. Mill Ans: (b)
(c) J. M. Keynes (d) Senior Q72. Net National Product in National Income
Ans: (a) Accounting refers to
Q65. Who among the following is not a classical (a) Gross Domestic Product— Depreciation
economist? (b) Gross Domestic Product + Subsidies
Page 373 of 516
https://telegram.me/aedahamlibra
(c) Gross National Product— Depreciation
(d) Gross National Product + Subsidies
Q80. A camera in the hands of a professional
photographer is a _______ good.
Ans: (c) (a) Free (b) Intermediary
Q73. While estimating national income which of the (c) Consumer (d) Capital
following is not taken into account? Ans: (b)
(a) Services of a teacher Q81. Which of the following is not an investment
(b) Services of a doctor expenditure in goods and services?
(c) Services of a housewife (a) Expansion of the main plant of a company
(d) Services of a maid servant (b) Purchase of a house
Ans: (c) (c) Purchase of machinery
Q74. Full employment is a situation where (d) An increase in business inventories
(a) there is no involuntary unemployment Ans: (b)
(b) there is involuntary unemployment Q82. Which one of the following is not a method for
(c) there is no voluntary unemployment computing GNP ?
(d) there is voluntary unemployment (a) Income Approach
Ans: (b) (b) Expenditure Approach
Q75. National Income include : (c) Savings Approach
(a) Financial help to earthquake victims (d) Value Added Approach
(b) Pocket money of a child Ans: (a)
(c) Winning of a lottery prize Q83. Which of the following relations always holds
(d) Construction of a new house true ?
Ans: (d) (a) Income = Consumption + Investment
Q76. A rising Per Capita Income will indicate a better (b) Income = Consumption + Saving
welfare if it is accompanied by (c) Saving = Investment
(a) unchanged Income distribution overall. (d) Income = Consumption + Saving + Investment
(b) changed Income distribution in favour of rich. Ans: (b)
(c) changed Income distribution in favour of poor. Q84. The main emphasis of Keynesian economics is on
(d) changed Income disribution in favour of Industrial (a) Expenditure (b) Exchange
Labour. (c) Foreign trade (d) Taxation
Ans: (c) Ans: (a)
Q77. Which of the following would not constitute an Q85. The value of investment multiplier relates to
economic activity in Economics ? (a) change in income due to change in aut onomous
(a) A teacher teaching students in his college investment.
(b) A teacher teaching students in a coaching institute (b) change in autonomous investment due to change in
(c) A teacher teaching his own daughter at home income.
(d) A teacher teaching students under Sarva Shik sha (c) change in income due to change in consumption.
Abbiyan Scheme (d) change in the income due to change in induced
Ans: (c) investment.
Q78. Price mechanism is a feature of Ans: (b)
(a) Capitalist economy Q86. Collective consumption means
(b) Barter economy (a) household consumption
(c) Mixed economy (b) individual consumption
(d) Socialist economy (c) self–consumption
Ans: (a) (d) consumption by the citizens of the country
Q79. Which one of the following is not a dimension of Ans: (d)
human development index ? Q87. If a change in all inputs leads to a proportionate
(a) Life expectancy change in output, it is case of
(b) Knowledge (a) Constant returns to scale
(c) Social status (b) Diminishing returns to scale
(d) Standard of living (c) Increasing returns to scale
Ans: (c) (d) Variable returns to scale
Page 374 of 516
https://telegram.me/aedahamlibra
Q88. One of the features of a free market economy is
Ans: (a) (c) Securities issued by the private sector
(d) Securities issued by the joint venture companies
(a) active state intervention Ans: (b)
(b) public ownership of factors of production Q96. Selling cost have to be incurred in case of
(c) rationing and price control (a) Perfect Competition
(d) consumer‘s sovereignty (b) Monopoly
Ans: (d) (c) Monopolistic Competition
Q89. When income increase, consumption also (d) None of the given options
increases : Ans: (c)
(a) in a lower proportion Q97. The term ‗Macro Economics‘ was used by
(b) in a higher proportion __________ .
(c) in the same proportion (a) J.M. Keynes
(d) None of the options (b) Ragner Frisch
Ans: (a) (c) Ragner Nurkse
Q90. The Ability Principle of Taxation is given by (d) Prof. Knight
(a) Adam Smith Ans: (b)
(b) Edgeworth Q98. The demand of a commodity is a direct demand
(c) Joan Robinson but the demand of a factor of production is called a
(d) J.S.Mill (a) Crossed demand
Ans: (a) (b) Joint demand
Q91. Situation Analysis is useful for: (c) Derived demand
(a) Analysis of Capital Market (d) Independent demand
(b) SWOT Analysis Ans: (c)
(c) Capital Market Q99. The basis of determining dearness allowance to
(d) Analysis of Capital Market and Capital Market employees in India is _______
Ans: (b) (a) National Income
Q92. Which of the following curve descr ibes the (b) Consumer Price Index
variation of household expenditure on a particular (c) Standard of Living
good with respect to household income ? (d) Inflation Rate
(a) Demand curve Ans: (b)
(b) Engel curve Q100. ‖The General Equilibrium Analysis‖ was
(c) Great Gatsby curve developed by
(d) Cost curve (a) Marshall (b) Ricardo
Ans: (b) (c) Walras (d) Adam Smith
Q93. Equilibrium output is determined by: Ans: (c)
(a) the equality between total Var iable cost and Marginal Q101. What is included in the Tetiary sector ?
revenue. (a) Banking
(b) the equality betweem Marginal cost and Marginal (b) Manufacturing
revenue. (c) Forestry
(c) the equality between Average cost and Average (d) Mining
revenue. Ans: (a)
(d) the equality between total cost and total revenue. Q102. Elasticity of demand is the degree of
Ans: (b) responsiveness of demand of a commodity to a
Q94. Pump priming should be resorted to at a time of (a) change in consumers‘ wealth
? (b) change in the price of substitutes
(a) Inflation (b) Deflation (c) change in consumers‘ tastes
(c) Stagflation (d) Reflation (d) change in its price
Ans: (b) Ans: (d)
Q95. What are gilt–edged securities? Q103. The first computer made available for
(a) Securities issued by the multinational companies. commercial use was :
(b) Securities issued by the Government (a) MANIAC (b) ENIAC
Page 375 of 516
(c) UNIVAC https://telegram.me/aedahamlibra
(d) EDSAC
Ans: (c)
(d) Industry, Trade and Transport
Ans: (b)
Q104. The Great Depression occurred during Q112. Interest on public debt is part of
(a) 1914-18 (b) 1929-34 (a) Transfer payments by the enterprises
(c) 1939-45 (d) 1922-26 (b) Transfer payments by the government
Ans: (b) (c) National income
Q105. ‗Take-off stage‘ in an economy means (d) Interest payments by households
(a) Steady growth begins. Ans: (b)
(b) Economy is stagnant. Q113. Which of the following is not viewed as national
(c) Economy is about to collapse. debt ?
(d) All controls are removed. (a) Life Insurance Policies
Ans: (a) (b) Long-term Government Bonds
Q106. The difference in the value of visible exports (c) National Savings Certificates
and visible imports is called : (d) Provident Fund
(a) Balance Sheet of items Ans: (a)
(b) Balance of Payments Q114. A mixed economy works primarily through the
(c) Balance of Trade (d) Balance of Account (a) market mechanism
Ans: (c) (b) central allocative machinery
Q107. Sectoral distribution of GDP in dex (c) market mechanism regulated by Government policy
measures_________ (d) market mechanism guided by Government
(a) Agriculture development of a country participation and planning
(b) Economic development of a country Ans: (d)
(c) Social development of a country Q115. Capital formation in an economy depends on
(d) Socio-Economic development of a Country (a) Total Income
Ans: (b) (b) Total demand
Q108. Indirect tax means : (c) Total savings
(a) there is not direct relationship be tween the tax payer (d) Total production
and the government. Ans: (c)
(b) direct relationship between tax payer and the Q116. Which one of the following items is not
government. included in the current account of India‘s Balance of
(c) tax base is income Payments ?
(d) the incidence and impact are on the same person on (a) Short-term commercial borrowings
whom tax is imposed. (b) Non-monetary gold movements
Ans: (a) (c) Investment income
Q109. What does a Leasing Company provide ? (d) Transfer payments
(a) Machinery and capital equipment on hire Ans: (b)
(b) Legal guidance in establishing an enterprise Q117. The permission given to a bank customer to
(c) Office accommodation on hire draw cheques in excess of his current account balance
(d) Technical consultancy and experts for a fee is called
Ans: (a) (a) a personal loan
Q110. An economy is in equili-brium when (b) an ordinary loan
(a) planned consumption exceeds planned saving (c) discounting a bill of exchange
(b) planned consumption exceeds planned investment (d) an overdraft
(c) intended investment equals intended investment Ans: (d)
(d) intended investment exceeds intended savings Q118. The existence of a parallel economy or Black
Ans: (c) Money
Q111. What are the main components of basic social (a) makes the economy more competitive
infrastructure of an economy ? (b) makes the monetary policies less effective
(a) Education, Industry and Agriculture (c) ensures a better distribution of income and wealth
(b) Education, Health and Civil amenities (d) ensures increasing productive investment
(c) Transport, Health and Banks Ans: (b)
Page 376 of 516
Q119. What is dual pricing? https://telegram.me/aedahamlibra
(a) Wholesale price and Retail pricing
Q127. Which of the following subjects does not figure
in the Concurrent List of our Constitution ?
(b) Pricing by agents and Pricing by retaliers (a) Stock Exchanges and futures markets
(c) Price fixed by Government and Price in open market (b) Protection of wild animals and birds
(d) Daily prices and Weekly prices (c) Forests
Ans: (c) (d) Trade unions
Q120. Beyond a certain point deficit financing will Ans: (a)
certainly lead to Q128. Mixed Economy means :
(a) inflation (a) Promoting both agriculture and industries in the
(b) deflation economy
(c) recession (b) Co-existence of public and private sectors
(d) economic stagnation (c) Co-existence of rich and poor
Ans: (a) (d) Co-existence of small and large industries
Q121. Ad Valorem tax is levied Ans: (b)
(a) according to value added by the Government. Q129. The aim of Differentiated Interest Scheme was
(b) according to value addition to a commodity to provide concessional loans to _______.
(c) according to value given by producers (a) weaker section of the society
(d) according to value added by the finance ministry (b) Public Sector Industries
Ans: (c) (c) Public Limited Companies
Q122. Core Industries are (d) big exports
(a) Basic industries Ans: (a)
(b) Consumer goods industries Q130. When too much money is chasing too few
(c) Capital goods industries goods, the situation is
(d) Government industries (a) deflation (b) inflation
Ans: (a) (c) recession (d) stagflation
Q123. Which one of the following is not a ‗canon of Ans: (b)
taxation‘ according to Adam Smith ? Q131. Devaluation of money means :
(a) Canon of certainty (a) decrease in the internal value of money
(b) Canon of simplicity (b) decrease in the external value of money
(c) Canon of convenience (c) decrease in both internal and external value of money
(d) Canon of economy (d) the government takes back currency notes of any
Ans: (b) denominations
Q124. Which one of the following is not included i n Ans: (b)
current revenue of the Union Government ? Q132. The ratio of a bank‘s cash holdings to its total
(a) Tax revenue deposit liabilities is called the
(b) Non-tax revenue (a) Variable Reserve Ratio
(c) Loans (b) Cash Reserve Ratio
(d) Interest payments (c) Statutory Liquidity Ratio
Ans: (c) (d) Minimum Reserve Ratio
Q125. ―Functional Finance‖ is associated with : Ans: (b)
(a) Adolph Wogner Q133. ―Dear Money‖ means
(b) Adam Smith (a) low rate of interest
(c) Adams (b) high rate of interest
(d) Abba ‗P‘ Lerner (c) depression
Ans: (d) (d) inflation
Q126. Which of the following is an indirect tax ? Ans: (b)
(a) Capital Gains Tax Q134. The share broker who sells shares in the
(b) Excise Duty apprehension of falling prices of shares is called
(c) Wealth Tax (a) Bull (b) Dog
(d) Estate Duty (c) Bear (d) Stag
Ans: (b) Ans: (c)
Page 377 of 516
(a) supply and demand
https://telegram.me/aedahamlibra
Q135. ‗Gresham‘s Law‘ in Economics relates to Q143. Bank money refers to
(a) currency notes
(b) circulation of currency (b) coins
(c) consumption of supply (c) gold bullions
(d) distribution of goods and services (d) cheques
Ans: (b) Ans: (d)
Q136. What does ECS in banking transactions stand Q144. If the Central Bank wants to encourage an
for ? increase in the supply of money and decrease in the
(a) Excess Credit Supervisor cost of borrowing money, it should
(b) Extra Cash Status (a) lower cash reserve ratio
(c) Exchange Clearing Standard (b) raise discount rates
(d) lectronic Clearing Service (c) sell government securities
Ans: (d) (d) All of the above
Q137. When too much money is chasing too few Ans: (a)
goods, the situation is Q145. ‗Money‘ is an example of
(a) Deflation (b) Inflation (a) Sunk capital
(c) Recession (d) Stagflation (b) Floating capital
Ans: (b) (c) Concrete capital
Q138. Green banking means (d) Social capital
(a) development of forestry by banks Ans: (b)
(b) financing of environmental friendly projects by banks Q146. Inflation is caused by
(c) financing of irrigation projects by banks (a) increase in money supply and decrease in production
(d) None of the above (b) increase in money supply
Ans: (b) (c) increase in production
Q139. The term stagflation refers to a situation where (d) decrease in production
(a) growth has no relation with the change in prices Ans: (a)
(b) rate of growth and prices both are decreasing Q147. Pegging up of a currency means, fixing the
(c) rate of growth in faster than the rate of price increase value of a currency
(d) rate of growth is slower than the rate of price increase (a) at a constant level
Ans: (d) (b) at a lower level
Q140. Inflation redistributes income and wealth in (c) at a higher level
favour of : (d) leaving it to market forces
(a) Pensioners (b) Poor Ans: (a)
(c) Middle class (d) Rich Q148. Debenture holders of a company are its
Ans: (d) (a) Shareholders (b) Creditors
Q141. Bank rate is the rate of interest (c) Debtors (d) Directors
(a) at which public borrows money from Commercial Ans: (b)
Bank Q149. The Cash Reserve Ratio is a tool of :
(b) at which public borrows money from RBI (a) Monetary policy
(c) at which Commerical Banks borrow money from RBI (b) Tax policy
(d) at which Commerical Banks borrow money from (c) Agricultural policy
public (d) Fiscal policy
Ans: (c) Ans: (a)
Q142. For channelising the unaccounted money for Q150. MUDRA Bank has been launched to help
productive purposes the Government Introduced th e (a) Small business
scheme of : (b) Marginal farmers
(a) Special Bearer Bonds (c) Poor women
(b) Resurgent India Bonds (d) Rural sector
(c) Provident Funds Ans: (a)
(d) Market Loans Q151. ‖Smart Money‖ term is used for
Ans: (a) (a) Credit Card
Page 378 of 516
(b) Internet Banking
(c) eBanking
https://telegram.me/aedahamlibra
(d) Integration of the various sectors of economy
Ans: (c)
(d) Cash with Public Q159. Dumping is a form of price discrimination at
Ans: (a) (a) within industry
Q152. What is the role of ―Ombudsman‖ in a bank? (b) national level
(a) To provide quality and speedy redressal of grievances (c) international level
of customers. (d) local level
(b) To provide suggestions for innovative schemes in the Ans: (c)
banks. Q160. Which type of foreign investment is considered
(c) To inspect the internal working of the branches. as unsafe?
(d) To monitor the poverty alleviation programmes (a) Foreign Direct Investment
undertaken by or implemented by the bank. (FDI)
Ans: (a) (b) Portfolio Investment
Q153. A closed economy is one which (c) NRI deposits
(a) Does not trade with other countries (d) External commercial borrowing
(b) Does not possess any means of international transport Ans: (b)
(c) Does not have a coatastal line Q161. Which among the following has the least
(d) Is not a member of the U.N.O. possibility of globalisation ?
Ans: (a) (a) selection of labour force
Q154. The difference between visible exports and (b) location of capital works
visible imports is defined as (c) to manage resources for investment
(a) Balance of trade (d) increase in infrastructure
(b) Balance of payment Ans: (b)
(c) Balanced terms of trade Q162. Which natioalised bank of India has a shining
(d) Gains from trade star as its emblem?
Ans: (a) (a) Syndicate Bank
Q155. A favourable Balance of Trade of a country (b) Indian Bank
implies that (c) Bank of India
(a) Imports are greater than Exports (d) Bank of Baroda
(b) Exports are greater than Imports Ans: (c)
(c) Both Imports and Exports are equal Q163. The data collection for national income
(d) Rising Imports and Falling Exports estimation is conducted in India by—
Ans: (b) (a) The Finance Ministry of the Government of India
Q156. Theoretically trade between two countries lakes (b) The RBI
place on account of (c) The NSSO (National Sample Survey Organi-sation)
(a) differences in costs (d) None of these
(b) scarcity of goods Ans: (c)
(c) comparative differences in costs Q164. ‗NABARD‘ is associated with the development
(d) need for exports of
Ans: (c) (a) agricultural sector and rural areas
Q157. Floating Exchange Rate is also referred to as (b) heavy industries
(a) Flexible Exchange Rate (c) banking sector
(b) Fixed Exchange Rate (d) real estates
(c) Real Exchange Rate Ans: (a)
(d) Controlled Exchange Rate Q165. One of the main factors that led to rapid
Ans: (a) expansion of Indian exports is
Q158. Globalisation means (a) Imposition of import duties
(a) Integration of economy (b) Liberalisation of the economy
(b) Integration of financial market (c) Recession in other countries
(c) Inte gration of the domestic economy with the world (d) Diversification of exports
economy Ans: (d)
Page 379 of 516
https://telegram.me/aedahamlibra
Q166. Indian agriculture is typically characterised as
(a) land surplus, labour scarce economy
(a) China
(c) India
(b) Japan
(d) Pakistan
(b) land surplus, labour surplus economy Ans: (a)
(c) land scarce, labour surplus economy Q175. AGMARK is a guarantee of standard :
(d) land scarce, labour scarce economy (a) quality (b) quantity
Ans: (c) (c) weight (d) size
Q167. The monetary policy is India is formulated by Ans: (a)
(a) Central Government Q176. Capital Market Regulator is:
(b) Industrial Financial Corporation of India (a) NSE (b) RBI
(c) Reserve Bank of India (c) SEBI (d) IRDA
(d) Industrial Development Bank of India Ans: (c)
Ans: (c) Q177. The upper limit of investment in plant and
Q168. When was the Minimum Wages Act enacted in machinery for smallscale industries has been fixed
India ? currently at
(a) 1936 (b) 1948 (a) 35 lakhs (b) 45 lakhs
(c) 1951 (d) 1956 (c) 60 lakhs (d) 1 crore
Ans: (b) Ans: (d)
Q169. Poverty in less developed countries is largely Q178. Steel sheets used in the production of furnitures
due to is an example of
(a) voluntary idleness (a) an intermediate good
(b) income inequality (b) a final good
(c) lack of cultural activities (c) an investment good
(d) lack of intelligence of the people (d) a consumption good
Ans: (b) Ans: (a)
Q170. Reserve Bank of India was nationalised in Q179. Economies of a firm are :
(a) 1948 (b) 1947 (a) An increase in its profits
(c) 1949 (d) 1950 (b) A reduction in its selling expenses
Ans: (c) (c) Its dominance of the market
Q171. Distribution of food rains operates under a two (d) Saving in it‘s production costs
tier system with the introduction of Ans: (d)
(a) Targetted Public Distribution System Q180. The time element in price analysis was
(b) The Consumers Cooperatives introduced by :
(c) The Cooperative Marketing Societies (a) J.M. Keynes
(d) The Service Cooperatives (b) Alfred Marshall
Ans: (a) (c) J.S. Mill
Q172. The duties levied on alcoholic liquors, narcotic (d) J.R. Hicks
drugs and opium come under— Ans: (b)
(a) Central Excise Duty Q181. If the fixed costs of a factory producing candles
(b) Land Revenue is Rs 20,000, selling price is Rs 30 per dozen candles
(c) State Excise Duty and variable cost is Rs 1.5 per candle, what is the
(d) General Sales Tax break-even quantity?
Ans: (a) (a) 20000 (b) 10000
Q173. The Imperial Bank of India, after its (c) 15000 (d) 12000
nationalisation came to be known as : Ans: (a)
(a) Reserve Bank of India
(b) State Bank of India
(c) United Bank of India
Physics
(d) Indian Overseas Bank
Ans: (b) Q1. Which among the following waves is used for
Q174. Special Economic Zone (SEZ) concept was first communication by artificial satellites ?
introduced in
Page 380 of 516
(a) Micro waves
(b) Radio waves
https://telegram.me/aedahamlibra
(d) artificial radioactivity
Ans: (a)
(c) A. M. Q10. The source of the Sun‘s energy is the process of
(d) Frequency of 1016 series (a) Photoelectric emission
Ans: (a) (b) Nuclear fission
Q2. Which among the following is the fundamental (c) Nuclear fusion
quantity ? (d) Thermionic emission
(a) Volume (b) Time Ans: (c)
(c) Velocity (d) Force Q11. What is colour of light related to ?
Ans: (b) (a) Amplitude (b) Frequency
Q3. The mass of a star is two times the mass of the (c) Quality (d) Velocity
Sun. How it will come to an end ? Ans: (a)
(a) Neutron Star Q12. What apparatus is used to locate a submerged
(b) Black hole object?
(c) White Dwarf (a) Radar (b) Sonar
(d) Red Giant (c) Quasar (d) Pulsar
Ans: (a) Ans: (b)
Q4. What is the minimum escape velocity of rocket to Q13. The shape of our milky way galaxy is
be launched into space ? (a) circular (b) elliptical
(a) 5 Km/Sec. (b) 6 Km/Sec. (c) spiral
(c) 11 Km/Sec. (d) 15 Km/Sec. (d) None of the above
Ans: (c) Ans: (c)
Q5. A boat will submerge when it displaces water Q14. Wollen clothes keep the body warm because
equal to its own: (a) Wool increases the temperature of the body
(a) volume (b) weight (b) Wool is a bad conductor
(c) surface area (d) density (c) Wool absorbs radiant heat from outer objects
Ans: (b) (d) Wool rejects heat from the outer objects
Q6. Two waves, each of amplitude 1.5 mm and Ans: (b)
frequency 10 Hz, are travelling in opposite direction Q15. When a barometer reading suddenly recedes, it
with a speed of 20 mm/s. T he distance in mm between indicates that climate–
adjacent nodes is : (a) will be very warm
(a) 1.0 (b) 1.2 (b) will be extremely stormy
(c) 1.5 (d) 2.0 (c) will remain cold
Ans: (a) (d) incessant rain for at least 48 hours
Q7. The velocity of sound in moist air is more than in Ans: (b)
dry air because the moist air has Q16. Clear nights are colder than cloudy nights
(a) less density than dry air. because of
(b) less pressure than dry air. (a) conduction
(c) more pressure than dry air. (b) condensation
(d) more density than dry air. (c) radiation
Ans: (a) (d) insolation
Q8. Ultra violet radiations of the Sun do not reach the Ans: (c)
earth because, earth‘s atmosphere is surrounded by Q17. The speed of light with the rise in the
(a) Carbon dioxide temperature of the medium :
(b) Ammonia (a) Increases
(c) Chlorine (d) Ozone (b) Decreases
Ans: (d) (c) Remains unaltered
Q9. Energy is continuously created in the sun due to: (d) Drops suddenly
(a) nuclear fusion Ans: (c)
(b) nuclear fission Q18. When the barometer reading dips suddenly, it is
(c) radioactivity an indication of
Page 381 of 516
(a) Hot weather
(b) Calm weather
https://telegram.me/aedahamlibra
Q26. Decibel is the unit used for
(a) Speed of light
(c) Storm (b) Intensity of heat
(d) Dry weather (c) Intensity of sound
Ans: (c) (d) Radio wave frequency
Q19. The term ‗equinox‘ means Ans: (c)
(a) The path which the Earth takes around the Sun Q27. The mass-energy relation is the outcome of
(b) The axis of the Earth around which it rotates (a) quantum theory
(c) When the day and night are of equal duration (b) general theory of relativity
(d) The time when the Sun seems to be going round and (c) field theory of energy
round in the sky in the Arctic but does not go below the (d) special theory of relativity
horizon Ans: (d)
Ans: (c) Q28. A simple microscope consists of :
Q20. If the velocity -time graph of a part icle is (a) a short focus convex lens
represented by y = mt + c, then the particle is moving (b) a long focus convex lens
with (c) a short focus concave lens
(a) constant speed (d) a long focus concave lens
(b) constant velocity Ans: (a)
(c) constant acceleration Q29. When the main switch of the house is put off it
(d) varying acceleration disconnects the
Ans: (c) (a) live wire only
Q21. Longitudinal waves cannot travel through (b) live wire and the earth wire
(a) Vacuum (b) Solid (c) live wire and the neutral wire
(c) Liquid (d) Gas (d) earth wire and the neutral wire
Ans: (a) Ans: (c)
Q22. Who measured the velocity of light first ? Q30. The sudden fall of atmospheric pressure
(a) Galileo (b) Newton indicates
(c) Romer (d) Einstein (a) fair weather (b) storm
Ans: (c) (c) rain (d) cold weather
Q23. Dynamo is a device for converting Ans: (b)
(a) Heat energy into electrical energy Q31. What is the reason for twinkling of stars ?
(b) Mechanical energy into electrical energy (a) Dispersion of light
(c) Magnetic energy into electrical energy (b) Total internal reflection
(d) Chemical energy into electrical energy (c) Atmospheric reflection
Ans: (b) (d) Atmospheric refraction
Q24. An electron microscope gives higher Ans: (d)
magnification than an optical microscope, because : Q32. A multimeter is used to measure
(a) it uses more powerful lenses (a) current
(b) the velocity of electron is smaller than that of visible (b) voltage
light (c) resistance
(c) the electrons have more energy than the light particles (d) All of the above
(d) the wavelength of electrons is smaller as compared to Ans: (d)
the wavelength of visible light Q33. Which one of the following instruments is used
Ans: (d) to study dispersion of light ?
Q25. For a body moving with non -uniform velocity (a) Microscope
and uniform acceleration (b) Telescope
(a) Displacement – Time graph is linear (c) Spectrometer
(b) Displacement – Time graph is non-linear (d) Photometer
(c) Velocity – Time graph is nonlinear Ans: (c)
(d) Velocity – Time graph is linear Q34. A decibel is
Ans: (b) (a) a musical instrument
Page 382 of 516
(b) the wavelength of noise
(c) a musical note
https://telegram.me/aedahamlibra
(c) a parachute has a larger surface area and air resistance
is more
(d) a measure of sound level (d) None of these
Ans: (d) Ans: (c)
Q35. Energy of Ultraviolet rays is great than Q42. Alternating current is converted into direct
(a) Infra–red rays current by a
(b) Gamma rays (a) transformer (b) dynamo
(c) X–rays (c) oscillator (d) rectifier
(d) Cosmic rays Ans: (d)
Ans: (a) Q43. Why two thin shirts can keep us warmer than a
Q36. A hydrogen balloon floats up because of single thick shirt in winter ?
(a) air pressure decreases with decrease in height (a) Two thin shirts become thicker so prevent
(b) air pressure decreases with decrease in weight transmission of heat
(c) weight of the balloon is less than the weight of air (b) Air layer between two shirts works as good conductor
displaced by it. (c) Air layer between t wo shirts behaves like insulating
(d) the pressure inside the balloon is more than the media
pressure outside it (d) No radiation of heat takes place
Ans: (c) Ans: (c)
Q37. In a Laser (say neon laser) all the atoms emit the Q44. A storm is predicted if atmospheric pressure
light waves of (a) rises suddenly
(a) Same frequency (b) rises gradually
(b) Same amplitude (c) falls suddenly
(c) Same phase (d) falls gradually
(d) All of the above Ans: (c)
Ans: (d) Q45. Smog is a combination of
Q38. What is the full form of ‗AM‘ regarding radio (a) Air and water vapours
broadcasting? (b) Water and smoke
(a) Amplitude Movement (c) Fire and water
(b) Anywhere Movement (d) smoke and fog
(c) Amplitude Matching Ans: (d)
(d) Amplitude Modulation Q46. The process involved in making soap is
Ans: (d) (a) saponification
Q39. The sensation of weightlessness in a spacecraft in (b) hydrolysis
an orbit is due to the (c) condensation
(a) absence of gravity outside (d) polymerisation
(b) acceleration in the orbit which is equal to the Ans: (a)
acceleration due to gravity outside Q47. In severe winter, in cold countries wat er pipes
(c) presence of gravity outside but not inside the burst because
spacecraft (a) water expands on freezing
(d) fact that spacecraft in the orbit has no energy (b) contraction of water pipes
Ans: (b) (c) high atmospheric pressure
Q40. The oldest type of energy known to man is (d) combined effect of all the above three
(a) wind power Ans: (a)
(b) solar power Q48. Velocity of sound in air does not change with the
(c) tidal energy change of
(d) geothermal energy (a) Temperature of air
Ans: (d) (b) Pressure of air
Q41. A parachute descends slowly whereas a stone (c) Moisture content in air
dropped from the same height falls rapidly, because (d) Wind in the direction of propagation of sound
(a) stone is heavier than parachute Ans: (b)
(b) special mechanisms are present in parachute Q49. Why is it difficult to breathe at higher altitudes?
Page 383 of 516
(a) Due to low air pressure
(b) Due to low temperature
https://telegram.me/aedahamlibra
(b) Convection
(c) Absorption
(c) Due to ozone (d) Advection
(d) Due to high humidity Ans: (d)
Ans: (a) Q58. Curie point is the temperature at which
Q50. A colour-blind person cannot (a) Matter becomes radioactive
(a) see distant objects (b) A metal loses magnetic properties.
(b) see black colour (c) A metal loses conductivity
(c) distinguish between certain colours (d) Transmutation of metal occurs.
(d) have persistence of vision Ans: (b)
Ans: (c) Q59. The spokes used in the wheel of a bicycle
Q51. Atomic power plant works on the principle of increase its
(a) fission (a) Moment of inertia
(b) fusion (b) Velocity
(c) thermal combustion (c) Acceleration
(d) combined effect of all the above three (d) Momentum
Ans: (a) Ans: (a)
Q52. The fuse in our domestic electric circuit melts Q60. Two stones of unequal masses are thrown
when there is a high rise in vertically up with the same velocity. Which of the
(a) Inductance (b) Current following will happen?
(c) Resistance (d) Capacitance (a) The heavier mass will reach greater height
Ans: (b) (b) The lighter mass will reach greater height
Q53. A soap bubble shows colours when illu minated (c) Both will reach the same height
with white light. This is due to (d) Any of them may reach greater height
(a) Diffraction ()
(b) Polarisation Ans: (c)
(c) Interference Q61. Cloudy nights are warmer compared nights,
(d) Reflection because clouds mainly
Ans: (c) (a) absorb heat from the atmosphere send it towards earth
Q54. Which of the following principle is used to (b) prevent cold waves from the sky descending on earth
produce ‗low-temperatures‘ ? (c) reflect back the heat given by earth
(a) Super conductivity (d) produce heat and radiate it toward earth
(b) Joule-Kelvin effect Ans: (c)
(c) Thermo-electric effect Q62. The frequency of ultrasound wave is typically
(d) Adiabatic demagneti-sation (a) Above 20 kHz
Ans: (d) (b) Above 20,000 kHz
Q55. The device used to change the speed of an (c) Below 20 kHz
electric fan is (d) Below 02 kHz
(a) Amplifier (b) Regulator Ans: (c)
(c) Switch (d) Rectifier Q63. The phenomenon of light associated with the
Ans: (b) appear-ance of blue colour of the sky is
Q56. The propagation of sound waves in a gas (a) Interference
involves (b) Reflection
(a) adiabatic compression and rarefaction (c) Refraction
(b) isothermal compression and rarefaction (d) Scattering
(c) isochoric compression and rarefaction Ans: (d)
(d) isobaric compression and rarefaction Q64. The function of ball bearings in a wheel is :
Ans: (a) (a) to increase friction
Q57. Heat transfer horizontally within the (b) to convert kinetic friction into rolling friction
atmosphere is called (c) to convert static friction into kinetic friction
(a) Conduction (d) just for convenience
Page 384 of 516
https://telegram.me/aedahamlibra
Q65. One can distinguish a telescope from a
Ans: (b) (c) to avoid skidding and to minimise friction
(d) to go fast and save fuel.
microscope by observing Ans: (c)
(a) length Q73. When hot -water is sprinkled on a hotter glass
(b) colour tumbler, it breaks because
(c) size of the lens (a) glass suddenly expands
(d) length and size of the lens (b) glass suddenly contracts
Ans: (d) (c) water evaporates
Q66. Electrostatic precipitator is used to control – (d) glass reacts chemically with water
(a) Air pollution Ans: (b)
(b) Water pollution Q74. The velocity of sound in air
(c) Solid waste (a) decreases with increase of temperature
(d) Noise pollution (b) increases with decrease of temperature
Ans: (a) (c) does not depend on temperature
Q67. In MRI machine, which one of the following is (d) decreases with decrease of temperature
used? Ans: (d)
(a) Sound wave Q75. Mud houses are cooler in summers and warmer
(b) ‗X‘-ray in winters as compared to brick houses because
(c) Ultra-sound wave (a) mud is a good conductor
(d) Magnetic wave (b) mud is a bad conductor
Ans: (d) (c) mud is a poor insulator
Q68. A spherical ball made of steel when dropped in (d) evaporation of water causes cooling in summers and
mercury container will sunlight coming through holes causes warming in winters
(a) sink in mercury Ans: (b)
(b) will be on the surface of mercury Q76. A person wears spectates with concave lenses. It
(c) will be partly immersed mercury means that normally (when not using glasses), the
(d) will dissolve in mercury image of distant objects is focussed in his eyes
Ans: (b) (a) behind the retina
Q69. Gamma rays have greatest similarity with (b) in front of the retina
( -rays (b) -rays (c) on the retina
(c) X-rays (d) U.V.-rays (d) on the blind spot
Ans: (c) Ans: (b)
Q70. Light houses are places with powerful lights to : Q77. A sphere rolls down on two inclined planes of
(a) guide and resolve traffic jams in crowded metro -cities different angles but same height, it does so
during nights. (a) in the same time
(b) guide and help large crowds at religious gathering (b) with the same speed
during nights. (c) in the same time with the same speed
(c) indicate to the incoming warships, the location of a (d) in the same time with the same kinetic energy
harbour during night. Ans: (b)
(d) guide and warn the ships coming from different Q78. A rubber ball is dropped from a height of 2
directions in the ocean. metres. To what height will it rise if there is no loss of
Ans: (d) energy/velocity after rebounding ?
Q71. The density of the liquid when heated (a) 4 metres (b) 3 metres
(a) decreases (c) 2 metres (d) 1 metre
(b) increases Ans: (c)
(c) does not change Q79. Which one of the following has the highest value
(d) may increase of decrease depending on pressure of specific heat?
Ans: (a) (a) Glass (b) Copper
Q72. Vehicle tyres are inflated properly (c) Lead (d) Water
(a) to ensure smooth running. Ans: (d)
(b) to allow the vehicle to take more load.
Page 385 of 516
electric energy
https://telegram.me/aedahamlibra
Q80. Necessary element of change in solar energy to Q88. In a refrigerator, the cooling system should
always be
(a) Berillium (b) Silicon (a) at the top
(c) Tantelum (d) Pure Copper (b) at the bottom
Ans: (b) (c) at the middle
Q81. Woollen cloth protects the body from cold (d) can be anywhere
because Ans: (b)
(a) it is a good conductor of heat Q89. The sparkling of a diamond is due to
(b) it is a poor conductor of heat (a) total internal reflection of light
(c) external heat rays enter into the body through the (b) interference of light
woollen cloth (c) polarisation of light
(d) it reflects heat (d) refraction of light
Ans: (b) Ans: (a)
Q82. Tape recorder should not be kept near one of the Q90. Waves that are required for long distance
following things : wireless communication are
(a) Clock (b) Magnet (a) Infrared rays
(c) Electrical switchboard (b) Ultraviolet rays
(d) Radio (c) Radio waves
Ans: (b) (d) Microwaves
Q83. Which of the following is a good conductor of Ans: (c)
heat but a bad conductor of electricity ? Q91. It is dangerous to observe Solar eclipse with
(a) Celluloid (b) Rubber naked eyes because
(c) Asbestos (d) Mica (a) Infrared radiations from sun burn our retina
Ans: (d) (b) Ultraviolet radiations from Sun burn our retina
Q84. To measure the speed of an approaching car a (c) All radiations from sun initiate chemical reactions in
police officer shines eyes
(a) light waves on it (d) Cosmic rays reach eyes more during the eclipse
(b) microwaves on it Ans: (b)
(c) radio waves on it Q92. ‗Hydraulic brakes‘ work on the principle of
(d) ultra high frequency waves on it (a) Pascal‘s law
Ans: (c) (b) Thomson‘s law
Q85. Which of the following is a result of Surface (c) Newton‘s law
tension ? (d) Bernoulli‘s theorem
(a) Gravitational pull Ans: (a)
(b) Viscosity Q93. Red is used as an emergency or danger signal as
(c) Capillary action (a) it is a striking colour
(d) Radiation (b) its wavelength is the longest
Ans: (c) (c) it is scattered
Q86. A bird sitting on a high tension electric wire does (d) it has very high energy
not get electrocuted because Ans: (b)
(a) it has high resistance Q94. Small bubbles are perfectly spherical because of
(b) the body is earthed (a) Gravity
(c) it does not form a closed path for the flow of current (b) Pressure
(d) its feet are good insulators (c) Viscosity
Ans: (c) (d) Surface tension
Q87. Radio activity is due to Ans: (d)
(a) unstable nucleus Q95. A light year is a measure of :
(b) stable nucleus (a) Speed (b) Velocity
(c) stable electronic configu-ration (c) Distance (d) Time
(d) stable electronic configu-ration Ans: (c)
Ans: (a) Q96. Speed of sound in air is unaffected by change in :
Page 386 of 516
(a) Pressure (b)Humidity
(c) Temperature
https://telegram.me/aedahamlibra
(d) Volume
Q106. The best material for the core of a transformer
is
Ans: (a) (a) stainless steel
Q97. An optically plane surface reflects a beam of (b) mild steel
light (c) hard steel
(a) as a parallel beam in one direction (d) soft iron
(b) as diffused beams in all directions Ans: (d)
(c) as parallel beams in all directions Q107. The width of depletion layer of a P-N junction
(d) as a diffused beam in one direction (a) decreases with light doping
Ans: (a) (b) increases with heavy doping
Q98. How much mechanic al work must be done to (c) is independent of applied voltage
completely melt 1 gram of ice at 0°C ? (d) is increased under reverse bias
(a) 4.2 J (b) 80 J Ans: (d)
(c) 336 J (d) 2268 J Q108. A bottle of sodalime is grasped by the neck and
Ans: (b) swung briskly in a vertical circle, Near which portion
Q99. Negative feedback in amplifiers of the bottle do the bubbles collect?
(a) increases bandwidth and decreases noise (a) Near the bottom
(b) decreases bandwidth and decreases noise (b) In the middle of the bottom
(c) increases bandwidth and increases noise (c) Near the neck
(d) decreases bandwidth and increases noise (d) Bubbles remain distributed uniformly
Ans: (a) Ans: (c)
Q100. A rear-view mirror for driving is Q109. The best conductor of electricity among the
(a) plain (b) concave following is :
(c) convex (d) inverted (a) Copper
Ans: (c) (b) Iron
Q101. The device used for locating submerged objects (c) Aluminium
under sea is. (d) Silver
(a) sonar (b) radar Ans: (d)
(c) laser (d) maser Q110. Decibel is a term connected with :
Ans: (a) (a) air (b) water
Q102. Tear gas used by the police to disperse the mob (c) sound (d) soil
contains Ans: (c)
(a) carbon dioxide Q111. Water will not be vapourised if
(b) chlorine (c) ammonia (a) the temperature is 0°C
(d) hydrogen sulphide (b) the humidity is 0%
Ans: (b) (c) the humidity is 100%
Q103. Kilowatt -hour is the unit of : (d) the temperature is 100°C
(a) Energy (b) Power Ans: (c)
(c) Force (d) Momentum Q112. Which of the following is a conductor of
Ans: (a) electricity ?
Q104. A man cannot see clearly beyond 10 meters. (a) Rubber (b) Pure water
The disease he suffers from : (c) Salt water (d) Benzene
(a) Far sight Ans: (c)
(b) Myopia Q113. The temperature of boiling water in a steam
(c) Cataract engine may be high because :
(d) Hypermetropia (a) there are dissolved substances in water
Ans: (b) (b) there is low pressure inside the boiler
Q105. Light beam which is highly directional is called: (c) there is high pressure inside the boiler
(a) eraser (b) grazer (d) the fire is at very high temperature
(c) maser (d) laser Ans: (c)
Ans: (d)
Page 387 of 516
https://telegram.me/aedahamlibra
Q114. In the Earth, the weight of a body is maximum
at the
(c) Ductility
(d) Malleability
(a) North Pole (b) South Pole Ans: (a)
(c) Equator (d) Surface Q122. Persons suffering from myopia are advised to
Ans: (d) use
Q115. A man inside an artificial satellite feels (a) convex lens
weightlessness because the force of attraction due to (b) concave lens
earth is (c) plano-convex lens
(a) zero at that place (d) plano-concave lens
(b) is balanced by the force of attraction due to moon Ans: (b)
(c) equal to the centripetal force Q123. A dynamo is used to convert
(d) non-effective due to particular design of the satellite (a) mechanical energy into electrical energy
Ans: (a) (b) electrical energy into mechanical energy
Q116. The primary colours in photography are (c) electrical energy into magnetic energy
(a) Red, Blue, Yellow (d) magnetic energy into mechanical energy
(b) Red, Yellow, Green Ans: (a)
(c) Red, Blue, Green Q124. Least audible sound for most of the human ear
(d) Blue, Yellow, Green is :
Ans: (a) (a) 10.0 μ bar
Q117. Ozone layer above the surface of Earth (b) 0.0002 μ bar
provides a shield against (c) 0.005 μ bar
(a) X-rays (d) 5.0 μ bar
(b) Ultra-violet rays Ans: (b)
(c) Gamma rays Q125. For which of the following substances, the
(d) Infra-red rays resistance decreases with increase in temperature?
Ans: (b) (a) Pure silicon (b) Copper
Q118. A water tank appears shallower when it is (c) Nichrome (d) Platinum
viewed from top due to Ans: (c)
(a) rectilinear propagation of light Q126. The oil in the wick of an oil lamp rises up due
(b) reflection to:
(c) total internal reflection (a) capillary action
(d) refraction (b) low viscosity of oil
Ans: (d) (c) gravitational force
Q119. Hydraulic machines work under the Principle (d) pressure difference
of : Ans: (a)
(a) Newton‘s Law Q127. The wall of a dam is made thicker at the
(b) Joules Law bottom than at the top beca use the pressure exerted
(c) Pascal‘s Law by the water on the wall
(d) Floatation Law (a) depends on the thickness of the wall
Ans: (c) (b) increases with height of the wall
Q120. A water tank appears shallower when it is (c) increases with depth of the wall
viewed from top due to (d) depends on the area of the wall
(a) rectilinear propagation of light Ans: (c)
(b) reflection Q128. Solar cells work on the principle of
(c) total internal reflection (a) photovoltaic effect
(d) refraction (b) photoelectric effect
Ans: (d) (c) photoconductive effect
Q121. Which of the following properties is generally (d) photosynthesis
found in nonmetals? Ans: (a)
(a) Brittleness
(b) Conductivity
Page 388 of 516
https://telegram.me/aedahamlibra
Q129. When two ice cubes are pressed together, they
join to form one cube. Which one of the following
(b) The metal ball suffers greater change in momentum
(c) Both suffer the same change in momentum
helps to hold them together? (d) The initial momentum of the rubber ball is greater
(a) Hydrogen bond formation than that of the metal ball
(b)Vander waals forces Ans: (a)
(c) Covalent attraction Q137. A NOT gate can be implemented by:
(d) Dipole interaction (a) a single diode
Ans: (a) (b) two diodes
Q130. The dynamo converts (c) a single resistor
(a)mechanical energy into electrical energy (d) a single transistor
(b) electrical energy into mechanical energy Ans: (d)
(c) mechanical energy into magnetic energy Q138. Which one of the following is responsible for
(d)magnetic energy into mechanical energy the working of Newton‘s colour disc experiment ?
Ans: (a) (a) Formation of pure spectra
Q131. A wire carrying current is (b) Formation of impure spectra
(a) positively charged (c) Persistence of vision
(b) negatively charged (d) Principle of complementary colour
(c) neutral Ans: (c)
(d) charged with a type depending upon current strength Q139. Steel is more elastic than rubber because it :
Ans: (c) (a) requires larger deforming force
Q132. Water is not suitable as a calorimetric (b) is never deformed
substance because it : (c) is deformed very easily
(a) has high specific heat (d) is harder than rubber
(b) is a good conductor Ans: (c)
(c) has high boiling point Q140. In radio-communication, the signals emitted by
(d) low latent heat of vapourization transmitting antenna are reflected on
Ans: (a) (a) stratosphere (b) ozonosphere
Q133. Which, of the following radiations has the least (c) ionosphere (d) troposphere
wavelength ? Ans: (c)
(a) X-rays (b) a-rays Q141. Sensitivity of the human eye is maximum in the
(c) g-rays (d) b-rays (a)Violet region (b) Green region
Ans: (c) (c) Blue region (d) Red region
Q134. The motion of a body that repeats itself after a Ans: (b)
regular interval of time is Q142. In Astrophysics, what name is given to a
(a) a periodic motion hypothetical hole in outer space from which star s and
(b) a simple harmonic motion energy emerge?
(c) an aperiodic motion (a) Black hole (b) Ozone hole
(d) an oscillatory motion (c) Asteroid belt (d) White hole
Ans: (a) Ans: (d)
Q135. A barometer is kept inside a bell Q143. The device used for measuring the wavelength
jar. Air is slowly pumped out of the jar. of X-rays is
(a) The mercury level starts rising (a) G.M. Counter
(b) The mercury level starts falling (b) Cyclotron
(c) The mercury level does not change (c) Bragg Spectrometer
(d) The bell jar breaks up (d) Mass Spectrometer
Ans: (a) Ans: (c)
Q136. A metal ball and a rubber ball, both having the Q144. Instrument used to study the behaviour of a
same mass, strike a wall normally with the same vibrating string is :
velocity. The rubber ball rebounds and the metal ball (a) Barometer (b) Hydrometer
does not rebound. It can be concluded that (c) Hygrometer (d) Sonometer
(a) The rubber ball suffers greater change in momentum Ans: (d)
Page 389 of 516
from it is separated out due to
https://telegram.me/aedahamlibra
Q145. When the milk is churned vigorously the cream Ans: (c)
Q153. Flywheel is an important part of a steam engine
(c) Centripetal force because it :
(b) Gravitational force (a) accelerates the speed of the engine
(c) Frictional force (b) helps the engine in keeping the speed uniform
(d) Centrifugal force (c) decreases the moment of inertia
Ans: (d) (d) gives strength to the engine
Q146. When a stone is thrown in the calm water of a Ans: (b)
pond the waves produced on the surface of water in Q154. A real gas can act as an ideal gas in
the pond are (a) high pressure and low temperature
(a) longitudinal (b) low pressure and high temperature
(b) transverse (c) high pressure and high temperature
(c) both longitudinal and transverse (d) low pressure and low temperature
(d) waves are not produced Ans: (b)
Ans: (b) Q155. If both the mass and the velocity of a body is
Q147. When two semiconductors of pand n -type are increased to twice of their magnitude, the kinetic
brought in contact, they form p -n junction which acts energy will increase by
like a/an (a) 2 times (b) 4 times
(a) Conductor (b) Oscillator (c) 8 times (d) 16 times
(c) Rectifier (d) Amplifier Ans: (b)
Ans: (c) Q156. The minimum number of forces to keep a
Q148. Centigrade and Fahrenheit temperatures are particle in equilibrium is
the same at (a) 1 (b) 2
(a) 32° (b) 40° (c) 3 (d) 4
(c) –273° (d) –40° Ans: (b)
Ans: (d) Q157. A thin oil film on a water surface appears
Q149. A wheel-barrow is an example of coloured because of
(a) class III lever (a) reflection (b) interference
(b) pulley system (c) diffraction (d) polarization
(c) class I lever Ans: (b)
(d) class II lever Q158. To open a door easily, the handle should be
Ans: (d) fixed
Q150. The splitting of white light into its components (a) near the hinges
is due to: (b) away from mid-point opposite to hinges
(a) reflection (c) in the middle
(b) refraction (d) None of these
(c) transmission Ans: (d)
(d) dispersion Q159. A particle is moving in a uniform circular
Ans: (d) motion with constant speed v along a circle of radius
Q151. A current carrying conductor is associated with r. The acceleration-of the particle is
(a) a magnetic field (a) zero (b) (c)
(b) an electric field
Ans: (d)
(c) an electro-magnetic field
Q160. The device used to convert solar energy into
(d) an electrostatic field
electricity is
Ans: (c)
(a) Photovoltaic cell
Q152. The threshold frequency is the frequency below
(b) Daniell cell
which :
(c) Electrochemical cell
(a) photo current increases with voltage
(d) Galvanic cell
(b) photo current decreases with voltage
Ans: (a)
(c) photo electric emission is not possible
(d) photo current is constant
Page 390 of 516
https://telegram.me/aedahamlibra
Q161. A wire of resistance R is cut into ‗n‘ equal
parts. These parts are then connected in parallel. The
(a) nail is more longer than needle.
(b) nail exerts more pressure than needle on the balloon.
equivalent resistance of the combination will be (c) needle exerts more pressure than nail on the balloon.
( (a) (b) (c) (d) gas is reactive with the needle.
Ans: (c)
Ans: (d)
Q170. Burns caused by steam are much severe than
Q162. If a wire of resistance R is melted and recast to
those caused by boiling water because
half of its length, then the new resistance of the wire
(a) Steam pierces through the pores of body quickly
will be
(b) Steam has latent heat
(a) R/4 (b) R/2
(c) Steam is gas and engulfs the body quickly
(c) R (d) 2R
(d) Temperature of steam is higher
Ans: (a)
Ans: (b)
Q163. Rainbow is formed due to
Q171. A person can jump higher on the moon‘s
(a) refraction and dispersion
surface than on the earth because :
(b) scattering and refraction
(a) the moon‘s surface is rough
(c) diffraction and refraction
(b) the acceleration due to gravity in moon is smaller than
(d) refraction and reflection
that on the earth.
Ans: (d)
(c) the moon is cooler than earth
Q164. Distances of stars are measured in
(d) the moon has no atmosphere
(a) Galactic unit
Ans: (b)
(b) Stellar mile
Q172. The apparent weight of a man in a lift is less the
(c) Cosmic kilometre
real weight when :
(d) Light year
(a) the lift is going up with an acceleration
Ans: (d)
(b) the lift is going down with uniform speed
Q165. 1 Micron is equal to
(c) the lift is going up with uniform speed
(a) 10–9 m (b) 10–12m
–6 (d) the lift is going down with an acceleration
(c) 10 m (d) 10–15m
Ans: (d)
Ans: (c)
Q173. Railway tracks are banked on curves so that :
Q166. The waves used in sonography are
(a) the train may not fall down inwards.
(a) Micro waves
(b) the weight of the train may be reduced.
(b) Infra-red waves
(c) necessary centripetal force may be obtained from the
(c) Sound waves
horizontal component of the normal reaction due to track.
(d) Ultrasonic waves
(d) no frictional force may be produced between the
Ans: (d)
wheels and the track.
Q167. A man in a train moving with a constant
Ans: (c)
velocity drops a ball on the platform. The path of the
Q174. When water freezes its density
ball as seen by an observer standing on the platform is
(a) Decreases
(a) A straight line
(b) Becomes Zero
(b) A circle
(c) Remains constant
(c) A parabola
(d) Increases
(d) None of these
Ans: (a)
Ans: (c)
Q175. When number of turns in a coil is trippled,
Q168. Total internal reflection can not take place
without any change in the length of coil, its self
when light goes from :
inductance becomes?
(a) glass to water
(a) Nine times (b) Three times
(b) water to glass
(c) Six times (d) One–third
(c) water to air
Ans: (a)
(d) glass to air
Q176. Which of the following phenomenon helps to
Ans: (b)
conclude that light is a transverse wave?
Q169. It is easy to burst a gas -filled balloon with a
(a) interference (b) diffraction
neelde than with a nail. It is because
(c) polarisation (d) refraction
Page 391 of 516
Q177. Unit of resistance is :
https://telegram.me/aedahamlibra
Ans: (c) (b) Land covered with fresh snow
(c) Prairie land
(a) volt2× ampere (d) Paddy crop land
(b) volt/ampere Ans: (b)
(c) ampere/volt Q185. Which of the following is an example of simple
(d) volt × ampere harmonic motion?
Ans: (b) (a) Earth spinning on its axis
Q178. Distant objects are visible as a little out of focus (b) Simple pendulum motion
in this condition : (c) Ball bouncing on floor
(a) presbiopia (d) Motion of a ceiling fan
(b) hypermetropia Ans: (b)
(c) astigmatism Q186. At boiling point of liquids, its
(d) myopia (a) Temperature increases
Ans: (d) (b) Atmospheric pressure increases
Q179. To prevent damage from lightning, lightning (c) Temperature remains constant
conductors are used on tall structures. The lightning (d) Vapour pressure decreases
conductor Ans: (c)
(a) should be made of a good conductor but can be of any Q187. The boiling point of water depends upon the
shape (a) atmospheric pressure
(b) should be in the form of a vertical rod (b) volume
(c) can be of any shape (c) density (d) mass
(d) should be made of a good conductor like copper with Ans: (a)
sharp-pointed edges Q188. The washing machine works on the principle of
Ans: 4) (a) Dialysis (b) Diffusion
Q180. Super cooling stands for cooling of a liquid : (c) Reverse osmosis
(a) at freezing point (d) Centrifugation
(b) below freezing point Ans: (d)
(c) at melting point Q189. Heat is transmitted from higher temperature to
(d) above melting point lower temperature through the actual motion of the
Ans: (b) molecules in
Q181. The SI unit of luminous emittance is : (a) Conduction
(a) Lux (b) Candela (b) Convection
(c) Lumen (c) Radiation
(d) Lumen second (d) Both conduction and convection
Ans: (a) Ans: (b)
Q182. For which one of the following, ‗Diodes‘ are Q190. A galvanometer can be converted into a
generally used for? voltmeter by connecting with it a
(a) Rectification (a) high resistance in parallel
(b) Amplification (b) low resistance on parallel
(c) Modulation (c) high resistance on series
(d) Filtration (d) low resistance in series
Ans: (a) Ans: (c)
Q183. The period of revolution of a certain planet in Q191. 0°K is equivalent to
an orbit of radius R is T. Its period of revolution in an (a) 273°C (b) – 273°C
orbit of radius 4R will be: (c) 0°C (d) 100°C
(a) 8 T (b) 4 T Ans: (b)
(c) 6 T (d) 2 T Q192. The absolute zero is a temperature at which
Ans: (a) _______
Q184. Which one of the following reflects back more (a) molecular motion in a gas would cease
sunlight as compared to other three? (b) water freezes
(a) Sand desert (c) all gases become liquid
Page 392 of 516
(d) all gases become solid https://telegram.me/aedahamlibra
Ans: (a)
(d) low resistance
Ans: (c)
Q193. In the absence of the earth‘s atmosphere, sky Q201. Where is a transistor most likely to be found ?
would appear (a) Wrist watch
(a) blue (b) deep red (b) Fuse
(c) white (d) black (c) Hearing aid
Ans: (d) (d) Fluorescent lamp
Q194. In an optical fibre the signal is transmitted Ans: (c)
(a) in a straight line path Q202. Fuse wire is made of an alloy of
(b) In a curved path (a) Tin and Lead
(c) due to total internal reflection (b) Tin and Copper
(d) Due to refraction (c) Lead and Copper
Ans: (c) (d) Copper and Silver
Q195. The temperature of a liquid is 32° Ans: (a)
F. What is its temperature in Celsius scale? Q203. Which one of the following is a good electrical
(a) 32° C (b) 0° C conductor?
(c) 100° C (d) 212° C (a) Graphite (b) Diamond
Ans: (b) (c) Peat (d) Charcoal
Q196. Which of the following is not true about X - Ans: (a)
rays? Q204. The quality or tone of a musical sound
(a) have low penetrating power produced by a stringed instrument depends on
(b) travel with the speed of light (a) frequency of vibration
(c) Can be reflected or refracted (b) length of the strings in the instrument
(d) can affect photographic plates (c) Amplitude of vibration
Ans: (a) (d) wave form of the sound
Q197. Which of the following is the cleanest source of Ans: (d)
energy? Q205. During the motion of a projectile fired from the
(a) Biofuel earth surface, _______
(b) Fossil fuel (a) its kinetic energy remains constant
(c) Nuclear power (b) its momentum remains constant
(d) Wind energy (c) vertical component of its velocity remains constant
Ans: (d) (d) horizontal component of its velocity remains constant
Q198. Sun appears red in colour at sunrise and sunset Ans: (d)
due to Q206. A body moving in a circular path with a
(a) the fact that sun emits only red colour at that time constant speed has a
(b) red light having longer wave length scatters away (a) constant velocity
(c) that sun comes out of the mountains (b) constant acceleration
(d) that all other colours scatter away except red (c) constant kinetic energy
Ans: (d) (d) constant displacement
Q199. In which form is the supplied heat energy Ans: (c)
stored during change in temperature of substance? Q207. Transformer is a device to convert
(a) Heat energy (a) D.C. to A.C.
(b) Kinetic energy (b) Low voltage D.C. into high voltage D.C.
(c) Potential energy (c) Low voltage A.C. into high voltage A.C.
(d) Both kinetic and potential energy (d) Mechanical energy into Electrical energy
Ans: (b) Ans: (c)
Q200. For photoelectric effect to take place, the metal Q208. One nanometer is equal to ______ meters.
used as the cathode should have______ (a) 10 raised to the power (-3)
(a) high melting point (b) 10 raised to the power (-6)
(b) low melting point (c) 10 raised to the power (-9)
(c) low work function (d) 10 raised to the power (-12)
Page 393 of 516
https://telegram.me/aedahamlibra
Ans: (c)
Q209. Hydraulic brakes in automobile work on
(c) Edward Teller
(d) Samuel Cohen
(a) Poiseuille‘s principle Ans: (c)
(b) Pascal‘s principle Q8. By which organic compound all the oils are
(c) Archimedes‘ principle known ?
(d) Bernoulli‘s principle (a) Carbohydrate
Ans: (b) (b) Protein
(c) Hydrocarbon
(d) Ester
Chemistry Ans: (c)
Q9. The acid used in lead storage cells is
Q1. Cement is made hard with (a) phosphoric acid
(a) dehydration (b) hydrochloric acid
(b) hydration and dissociation of water (c) nitric acid
(c) Dissociation of water (d) sulphuric acid
(d) Polymerisation Ans: (d)
Ans: (b) Q10. Soap is prepared by boiling caustic soda with
Q2. ―All the four quantum numbers of two electrons (a) Alcohol (b) Kerosene oil
in an atom are not the same.‖ It is the law of — (c) Glycerine (d) Fats
(a) Hund‘s Ans: (d)
(b) Exclusion Principle of Pauli Q11. Which one of the following metals is used to
(c) Uncertainty Principle of Hiesenberg galvanise iron ?
(d) Avogadro‘s Law (a) Copper (b) Lead
Ans: (b) (c) Zinc (d) Mercury
Q3. The gas used for artificial fr uit ripening of green Ans: (c)
fruit is — Q12. Which among the following elements is found in
(a) Ethylene (b) Acetylene maximum percentage in the human body ?
(c) Ethane (d) Methane (a) Carbon (b) Hydrogen
Ans: (a) (c) Nitrozen (d) Oxyzen
Q4. An atomic pile is used for Ans: (d)
(a) producing X-rays Q13. Bauxite is an alloy of which of the following
(b) conducting nuclear fission metals ?
(c) coducting thermonuclear fusion (a) Aluminium (b) Silver
(d) accelerating atoms (c) Tin (d) Iron
Ans: (b) Ans: (a)
Q5. The elem ent which is commonly used in nuclear Q14. Which of the following is ozone depleting
for producing electricity by nuclear fission is pesticide ?
(a) Radium (b) Plutonium (a) D.D.T.
(c) Uranium (d) Deuterium (b) Benzene
Ans: (c) (c) Methyl bromide
Q6. Which of the following is used to iodise common (d) Ethylene ozonide
salf ? Ans: (c)
(a) Potassium iodide Q15. Which is abundant in Gobar gas?
(b) Iodine (a) Carbon dioxide
(c) Potassium iodate (b) Methane
(d) Sodium iodide (c) Oxygen
Ans: (c) (d) Hydrogen
Q7. Who developed Hydrogen Bomb? Ans: (b)
(a) Wernher Von Braun Q16. The presence of ozone in the stratosphere is
(b) J. Robert Oppen Heimer responsible for

Page 394 of 516


years
https://telegram.me/aedahamlibra
(a) increasing the average global temperature in recent (a) Lead
(c) Zinc
(b) Tin
(d) Mercury
(b) higher rate of photosynthesis Ans: (d)
(c) checking the penetration of ultra -violet rays to the Q26. German silver is an alloy of
earth (a) copper, nickel and silver
(d) supplying oxygen for people travelling in jets (b) silver, copper and aluminium
Ans: (c) (c) zinc, copper and nickel
Q17. The long range potential of nuclear energy in (d) zinc, silver and copper
India depends on its reserves of Ans: (c)
(a) thorium (b) uranium Q27. Metal are good conductors, because
(c) plutonium (d) radium (a) they contain free electrons
Ans: (b) (b) the molecules in them are very close to each other
Q18. Which of the following is a natural dye ? (c) the molecules in them collide very freely
(a) Crystal violet (d) they contain reflecting surface
(b) Aniline blue Ans: (a)
(c) Alizarin Q28. Tear gas is :
(d) Phenolphthalein (a) Chloroprene
Ans: (c) (b) Carbonyl chloride
Q19. Which of the following elements is non - (c) Nitrous oxide
radioactive ? (d) Bleaching powder
(a) Uranium (b) Thorium Ans: (b)
(c) Plutonium (d) Zirconium Q29. A polymeric substance used to make parachute
Ans: (d) is
Q20. What is the chemical name for ‗Baking Soda‘ ? (a) Rayon (b) Viscose
(a) Sodium carbonate (c) Cotton (d) Terylene
(b) Sodium bicarbonate Ans: (b)
(c) Sodium nitrite Q30. Heat resistant variety of glass is—
(d) Sodium nitrate (a) Pyrex glass (b) Hard glass
Ans: (b) (c) Flint glass (d) None of these
Q21. Which of the following vegetable oils does not Ans: (c)
contain essential fatty acids ? Q31. Impure camphor is purified by
(a) Sunflower oil (b) Mustard oil (a) sublimation
(c) Coconut oil (d)Groundnut oil (b) fractional crystallisation
Ans: (a) (c) fractional distillation
Q22. Which of the following fibres catches fire least ? (d) steam distillation
(a) Nylon (b) Cotton Ans: (a)
(c) Rayon (d) Terricot Q32. Bhopal gas tragedy is associated with leakage of
Ans: (b) (a) carbon dioxide
Q23. The polymerisation of which of the following is (b) nitrogen dioxide
used in manufacturing of polythene in industry ? (c) sulphur dioxide
(a) Methane (b) Styrene (d) methyl isocyanate
(c) Acetylene (d) Ethylene Ans: (d)
Ans: (d) Q33. The substance most commonly used as a food
Q24. Cloud is a colloidal dispersion of preservative is
(a) Air in a dispersion medium of water (a) sodium carbonate
(b) Fog in a dispersion medium of water (b) tartaric acid
(c) Mist in a dispersion medium of air (c) acetic acid
(d) Water drops in a dispersion medium of air (d) sodium salt of benzoic acid
Ans: (d) Ans: (d)
Q25. Which of the following metals form an amalgam Q34. The acid used in lead storage cells is
with other metals ? (a) phosphoric acid
Page 395 of 516
(b) hydrochloric acid
(c) nitric acid
https://telegram.me/aedahamlibra Ans: (a)
Q44. Pasteurisation is the process in which milk is
(d) sulphuric acid heated to
Ans: (d) (a) 60°C for 10 minutes
Q35. ‗Milk Sugar‘ is (b) 63° C for 20 minutes
(a) Lactose (b) Maltose (c) 63°C for 30 minutes
(c) Galactose (d) Sucrose (d) 72°C for 10 minutes
Ans: (a) Ans: (c)
Q36. Who discovered the atom bomb? Q45. The most abundant inert gas in the atmosphere
(a) Madam Curie is
(b) Pierre Curie (a) Helium (b) Neon
(c) Otto Hahn (c) Argon (d) Krypton
(d) Albert Einstein Ans: (c)
Ans: (c) Q46. Helium gas is used in gas balloons instead of
Q37. Polythene is polymer of hydrogen gas because it is
(a) Ethylene (b) Propylene (a) lighter than hydrogen
(c) Acetylene (d) Aniline (b) more abundant than hydrogen
Ans: (a) (c) non-combustible
Q38. When conc. H2SO4 is added to dry KNO3, (d) more stable
brown fumes are evolved. These fumes are due to Ans: (c)
(a) SO3 (b) SO2 Q47. Gunpowder consits of a mixture of
(c) NO2 (d) N2O (a) sand and TNT
Ans: (c) (b) TNT and charcoal
Q39. Preparation of ‗Dalda or Vanaspati‘ ghee from (c) nitre, sulphur and charcoal
vegetable oil utilizes the following process : (d) sulphur, sand and charcoal
(a) Hydrolysis (b) Oxidation Ans: (c)
(c) Hydrogenation Q48. The compound to which H2 does not add is
(d) Ozonolysis (a) Biphenyl ethylene
Ans: (c) (b) Tetraphenyl ethylene
Q40. Which gas is used as fire extinguisher ? (c) Tetra – a –naphthylethylene
(a) Carbon dioxide (d) Tetra –9 – phenanthrylethylene
(b) Carbon monoxide Ans: (b)
(c) Carbon suboxide Q49. White coal is
(d) Sulphur dioxide (a) Uranium
Ans: (a) (b) Hydro-electricity
Q41. Cream gets separated out of milk when it is (c) Diamond
churned. This is due to (d) Ice
(a) gravitational force Ans: (*)
(b) centripetal force Q50. Which one of the following is not a radio -active
(c) centrifugal force element ?
(d) frictional force (a) Uranium (b) Thorium
Ans: (c) (c) Radium (d) Cadmium
Q42. Petroleum consists of a mixture of Ans: (d)
(a) Carbohydrates Q51. The important ore of aluminium is
(b) Carbonates (a) bauxite (b) cryolite
(c) Hydrocarbons (c) fluorspar (d) haematite
(d) Carbides Ans: (a)
Ans: (c) Q52. What are the major pollutants of cigarette
Q43. Percentage of carbon in steel ranges from smoke?
(a) 0.1 to 1.5 (b) 1.5 to 3.0 (a) Carbon monoxide and dioxin
(c) 3.0 to 4.0 (d) 4.0 to 6.0 (b) Carbon monoxide and nicotine
Page 396 of 516
(c) Carbon monoxide and benzene
(d) Dioxin and benzene
https://telegram.me/aedahamlibra
Q60. The material used in the manufacture of lead
pencil is
Ans: (b) (a) Graphite (b) Lead
Q53. The main constituent of biogas is (c) Carbon (d) Mica
(a) oxygen (b) methane Ans: (a)
(c) acetic acid (d) methyl alcohol Q61. Detergents are
Ans: (b) (a) Sodium salts of fatty acids
Q54. Wax used for making candle is chemically a (b) Sodium salts of sulphonic acids
mixture of (c) Sodium salt of benzoic acid
(a) aliphatic hydrocarbons (d) None of the above
(b) aromatic hydrocarbons Ans: (b)
(c) cyclic hydrocarbons Q62. The substance that is added to make natural
(d) aliphatic and aromatic hydrocarbons rubber strong and more bouncy is
Ans: (a) (a) Polythene (b) Sponge
Q55. Ultraviolet radiation striking the earth is due to (c) Sulphur (d) Chlorine
the depletion of Ans: (c)
(a) carbon monoxide Q63. Galvanization of iron is carried out using
(b) carbon dioxide (a) Zinc (b) Tin
(c) ozone (c) Copper (d) Chromium
(d) oxygen Ans: (a)
Ans: (c) Q64. Two elements which are used to absorb neutrons
Q56. Match the sourc in Column B with the product to control the chain reaction during nuclear fission
of Column A. are
Column A Column B (a) Boron and Cadmium
(Product) (Source) (b) Boron and Plutonium
(a) Formic acid (a) Lemon (c) Cadmium and Uranium
(b) Citric acid (b) Tamarind (d) Uranium and Boron
(c) Tartaric acid (c) Ants a b c Ans: (a)
(a) 3 2 1 Q65. Supersonic jet causes pollution by thinning of
(b) 3 1 2 (a) O3 layer (b) O2 layer
(c) 2 3 1 (c) SO2 layer (d) CO2 layer
(d) 2 1 3 Ans: (a)
Ans: (b) Q66. Which one of the following minerals is found in
Q57. Which of the following substance is highly Monazite sand ?
plastic ? (a) Potassium (b) Uranium
(a) Quartz (b) Mica (c) Thorium (d) Sodium
(c) Granite (d) Clay Ans: (c)
Ans: (d) Q67. Which of the following could be used as fuel in
Q58. Which one of the following metals does not react propellant or rockets ?
with water to produce Hydrogen ? (a) Liquid Hydrogen + Liquid Nitrogen
(a) Potassium (b) Cadmium (b) Liquid Oxygen + Liquid Argon
(c) Sodium (d) Lithium (c) Liquid Nitrogen + Liquid Oxygen
Ans: (b) (d) Liquid Hydrogen + Liquid Oxygen
Q59. Silver halides are used in photographic plates Ans: (d)
because they are Q68. Cement is formed by strongly heating a mixture
(a) oxidised in air of
(b) soluble in hyposolution (a) limestone and graphite
(c) reduced by light (b) limestone and clay
(d) totally colourless (c) chalk and graphite
Ans: (c) (d) clay and graphite
Ans: (b)
Page 397 of 516
https://telegram.me/aedahamlibra
Q69. A balloon filled with helium rises in air because
(a) air exerts an upward force on the balloon
(c) 75% (d) 60%
Ans: (c)
(b) the balloon is weightless Q78. Which one of the following statements is not
(c) helium is less dense than air correct?
(d) helium pushes down on the air below the balloon Vulcanisation of rubber makes it
Ans: (c) (a) soft and flexible
Q70. Which of the following contains high content of (b) strong and hard
lead? (c) chemical resistant
(a) Coal (d) withstand higher temperatures
(b) Cooking gas Ans: (a)
(c) High octane fuel Q79. The coating of a thin layer of zinc on steel or
(d) Low octane fuel iron objects is known as
Ans: (c) (a) Hot dipping
Q71. Which of the following is used in welding broken (b) Tinning
pieces of iron rails and machine parts? (c) Galvanising
(a) Aluminium sulphate (d) Electroplating
(b) Solder Ans: (c)
(c) Aluminium powder Q80. Which one of the following minerals is found in
(d) None of the above pure form?
Ans: (c) (a) Iron (b) Copper
Q72. Which of the following in automobile exhaust (c) Bauxite (d) Manganese
can cause cancer? Ans: (b)
(a) Oxides of nitrogen Q81. Among the fuels given below, the one with
(b) Carbon monoxide highest calorific value is
(c) Lead (a) Coke (b) Coal
(d) Polyclinic hydrocarbons (c) Wood (d) Natural gas
Ans: (d) Ans: (d)
Q73. The atmospheric gas that is mainly responsible Q82. Coal is formed from
for Green House effect : (a) crude oil
(a) Ozone (b) Nitrogen (b) deposit of animal waste
(c) Oxygen (d) Carbondioxide (c) coke
Ans: (d) (d) compressed and hardened biomass
Q74. The anode of a dry cell is made up of Ans: (d)
(a) Iron (b) Cadmium Q83. Tear gas used by the police to disperse the mob
(c) Zinc (d) Lead contains
Ans: (c) (a) Carbon dioxide
Q75. The chief constituent of vinegar is (b) Chlorine
(a) Formic acid (c) Ammonia
(b) Acetic acid (d) Hydrogen sulphide
(c) Salycylic acid Ans: (b)
(d) Oxalic acid Q84. Brass is an alloy of
Ans: (b) (a) Lead and tin
Q76. The common name for the compound having (b) Zinc and copper
formal NaOH is (c) Antimony, tin and lead
(a) Caustic soda (d) Zinc, tin and copper
(b) Caustic potash Ans: (b)
(c) Soda ash Q85. Which of the following is used to denature
(d) Sodium hydroxide ethanol ?
Ans: (a) (a) Methyl alcohol
Q77. The ratio of pure gold in 18 carat gold is (b) Propyl alcohol
(a) 100% (b) 80% (c) Phenol
Page 398 of 516
(d) Methane https://telegram.me/aedahamlibra
Ans: (a) Q95. Gold dissolves in
Ans: (a)

Q86. The main constituent of natural gas is (a) Hydrochloric acid


(a) Methane (b) Ethane (b) Nitric acid
(c) Butane (d) Propane (c) Aqua-regia
Ans: (a) (d) Acetic acid
Q87. The metal, used to galvanise iron to protect it Ans: (c)
from rusting is Q96. The main source of Naphthalene is
(a) Zinc (b) Chromium (a) Charcoal (b) Camphor
(c) Lead (d) Antimony (c) Coal-tar (d) Diesel
Ans: (a) Ans: (c)
Q88. Hardest allotrope of carbon is Q97. Which of the following is a super-
(a) diamond (b) graphite cooled liquid?
(c) coke (d) lamp black (a) Ice-cream (b) Ammonia
Ans: (a) (c) Glass (d) Wood
Q89. In vulcanisation process, rubber can be Ans: (c)
hardened by adding Q98. The metal extracted from Bauxite is :
(a) Nitrogen (b) Silicon (a) Silver (b) Copper
(c) Sulphur (d) Alcohol (c) Manganese (d) Aluminum
Ans: (c) Ans: (d)
Q90. The basic chemical building block of natural Q99. The coloured discharge tubes for advertisement
rubber obtained from trees is mainly contain
(a) Isoprene (b)Vinyl Chloride (a) Xenon (b) Argon
(c) Acetylene (d)Neoprene (c) Helium (d) Neon
Ans: (a) Ans: (d)
Q91. Submarines move under water. Q100. When quick lime is added to water
They have engines that run on (a) heat is liberated
(a) Petrol and oxygen (b) heat is absorbed
(b) Diesel (c) Batteries (c) temperature decreases
(d) Steam (d) no heat change takes place
Ans: (b) Ans: (a)
Q92. Silver nitrate solution is kept in brown bottles in Q101. Which of the following is a renewable source of
laboratory because energy ?
(a) it reacts with ordinary white bottles (a) Coal
(b) brown bottles stops the passage of light through it (b) Geothermal powder
(c) brown bottles do not react with it (c) Natural gas
(d) brown bottles react with it (d) Uranium
Ans: (b) Ans: (b)
Q93. Name two elements that find wide application in Q102. Which one of the following is a micro-element ?
transistor industry (a) Chlorine (b) Hydrogen
(a) Silicon and Germanium (c) Nitrogen (d) Oxygen
(b) Carbon and Platinum Ans: (a)
(c) Iridium and Germanium Q103. Ozone depletion in the atmosphere is mainly
(d) Tungsten and Platinum caused by
Ans: (a) (a) sulphur dioxide
Q94. Two elements which can form a large number of (b) nitrogen dioxide
compounds are (c) hydrogen sulphide
(a) Carbon and Hydrogen (d) chlorofluorocarbon
(b) Carbon and Nitrogen Ans: (d)
(c) Carbon and Oxygen Q104. The element with highest first ionisation energy
(d) Carbon and Sulphur is
Page 399 of 516
(a) hydrogen
(c) lithium
https://telegram.me/aedahamlibra
(b) helium
(d) sodium
Q114. Which of the following is the major constituent
of LPG?
Ans: (b) (a) Methane (b) Ethane
Q105. The resources which can be used continuously, (c) Propane (d) Butane
year-after -year are called Ans: (d)
(a) Biotic (b) Abiotic Q115. The particle required to continue the chain
(c) Non-renewable process of Uranium fission is :
(d) Renewable (a) Electron (b) Proton
Ans: (d) (c) Neutron (d) Positron
Q106. When Hydrogen starts burning in air, it Ans: (c)
produces ? Q116. Which of the following is an iron ore ?
(a) Ammonia (b) Water (a) Bauxite (b) Magnetite
(c) Methane (d) Carbonic Acid (c) Lignite (d) Nitrite
Ans: (b) Ans: (b)
Q107. The unit of ionic product of water Q117. The Bhopal tragedy was caused by the gas
(Kw) is : (a) phosgene
(a) Mol2 let-1 (b) Mol2 let-2 (b) carbon monoxide
-1 -2
(c) Mol let (d) Mol-1 let-1 (c) methyl isocyanate
Ans: (b) (d) chlorine
Q108. Chemical name of bleaching powder is : Ans: (c)
(a) Calcium chlorate Q118. Gas released during Bhopal tragedy was :
(b) Calcium hypochlorite (a) Sodium isothiocyanate
(c) Calcium chloro hypochlorite (b) Potassium isothiocyanate
(d) Calcium bi chloride (c) Ethyl isothiocyanate
Ans: (b) (d) Methyl isothiocyanate
Q109. Conduction band electron s have more mobility Ans: (d)
than holes because they Q119. Permanent – hardness of water may be
(a) are lighter removed by addition of
(b) experience collision less frequently (a) Alum
(c) have negative charge (b) Sodium carbonate
(d) need less energy to move them (c) Lime
Ans: (b) (d) Potassium Permangante
Q110. The gas that usually causes explosions in coal Ans: (b)
mines is Q120. One property of ammonia is
(a) Hydrogen (a) It is insoluble in water
(b) Carbon monoxide (b) It is a odourless gas
(c) Air (d) Methane (c) It is a yellowish gas
Ans: (d) (d) Its aqueous solution turns red litmus blue
Q111. When water itself combines chemically with Ans: (d)
some element or mineral it is called Q121. Water gas is the mixture of
(a) Carbonation (b) Desilication (a) carbon dioxide and hydrogen
(c) Hydration (d) Oxidation (b) carbon monoxide and nitrogen
Ans: (c) (c) carbon monoxide and hydrogen
Q112. KMnO4 can be used as a/an (d) carbon dioxide and nitrogen
(a) insecticide (b) fertiliser Ans: (c)
(c) pesticide (d) disinfectant Q122. The function of skimming tank in sewage
Ans: (d) treatment is to remove
Q113. Oxygen has (+) oxidation number only in (a) oil and fatty substances
(a) OF2 (b) Cl2O (b) suspended solids
(c) H2O (b) N2O (c) gritty substances
Ans: (a) (d) inorganic substances
Page 400 of 516
https://telegram.me/aedahamlibra
Q123. The chief source of naphthalene is
Ans: (a) (d) hydroquinone
Ans: (a)
(a) Coal-tar (b) Diesel Q133. The ‗Green House effect‘ is produced due to
(c) Charcoal (d) Camphor the presence of excess amount of
Ans: (a) (a) Sulphur dioxide
Q124. Maximum permissible concentration of copper (b) Nitrogen dioxide
in drinking water in mg/L is (c) Carbon monoxide
(a) 0.01 (b) 0.05 (d) Methane
(c) 1.0 (d) 2.0 Ans: (d)
Ans: (c) Q134. Which of the following is caused by the higher
Q125. Asbestos is formed of : concentration of Nitrogen and Phosphorous ?
(a) Calcium and Magnesium (a) Eutrophication
(b) Copper, Zinc and Manganese (b) Hardness
(c) Lead and Iron (c) Alkalinity
(d) Calcium and Zinc (d) Acidity
Ans: (a) Ans: (a)
Q126. Maximum permissible concentration of copper Q135. ‗Aerosol‘ is
in drinking water in mg/L is (a) Suspended solid particles in air
(a) 0.01 (b) 0.05 (b) Pollutant gas, present in air
(c) 1.0 (d) 2.0 (c) Microbes, present in air
Ans: (c) (d) Green house gas present in air
Q127. One carat of diamond is equal to Ans: (a)
(a) 100 mg (b) 150 mg Q136. Maximum permissible concentration of total
(c) 200 mg (d) 250 mg hardness in drinking water is
Ans: (c) (a) 50 (b) 100
Q128. The most common cause of pollution of air is (c) 200 (d) 500
(a) Carbon dioxide Ans: (c)
(b) Carbon monoxide Q137. In silvery paints, the main cons tituent present
(c) Sulphur dioxide is
(d) Smoke (a) Aluminium powder
Ans: (c) (b) Silver powder
Q129. The next higher homologue of C6H14 is (c) Lead powder
(a) C7H14 (b) C7H12 (d) White lead
(c) C6H12 (d) C7H16 Ans: (a)
Ans: (d) Q138. Ozone layer in the upper parts of the
Q130. Charcoal is used in water treatment as a/an atmosphere serves as a protective shield against
(a) absorbent (b) coagulant harmful
(c) solvent (d) adsorbent (a) solar ultraviolet radiation
Ans: (d) (b) solar infrared radiation
Q131. The function of sedimentation tank in sewage (c) CO2 in the air
treatment is to: (d) SO2 in the air
(a) Disinfect the sewage Ans: (a)
(b) Remove water content Q139. Ferrous sulphate is used in water treatment as
(c) Aerate the sewage a/an
(d) Remove suspended solids (a) Oxidizing agent
Ans: (d) (b) Reducing agent
Q132. The chemical used in developing photographic (c) Coagulant aid
films is (d) Adsorbent
(a) silver bromide Ans: (c)
(b) hypo Q140. The enormous amount of energy released in an
(c) sodium sulphate explosion of an atmoic bomb is due to the
Page 401 of 516
https://telegram.me/aedahamlibra
(a) conversion of mass into energy
(b) conversion of chemical energy into heat energy
(c) sol (d) precipitate
Ans: (b)
(c) conversion of mechnical energy into nuclear energy Q150. The chemical formula of the laughing gas is
(d) conversion of neutrons into protons (a) NO (b) N2O
Ans: (d) (c) NO2 (d) N2O3
Q141. When a lead storage battery is discharged, Ans: (b)
(a) SO2 is evolved Q151. Glass is soluble in
(b) Lead sulphate is consumed (a) H2SO4 (b) HClO4
(c) Lead is formed (c) HNO 3
(d) Sulphuric acid is consumed (d) HF
Ans: (d) Ans: (d)
Q142. Radioactive elements emit : Q152. Which of the following metals can deposit
(a) Radiowaves copper from copper sulphate solution ?
(b) Infrared waves (a) Platinum (b) Mercury
(c) Ultraviolet waves (c) Iron (d) Gold
(d) Ans: (c)
Ans: (d) Q153. Exposure to mixtures of chemicals are greater
Q143. Silicon is a than expected on the basis of effects of exposure to
(a) Semiconductor each chemical individually.
(b) Insulator This is known as
(c) Nonconductor (a) Additives (b) Antagonism
(d) Conductor (c) Synergism (d) Independent
Ans: (a) Ans: (c)
Q144. Bayer‘s reagent is Q154. Which among the following is used as a catalyst
(a) Alkaline potassium permanganate in the production of high octane fuels?
(b) Acidified potassium permanganate (a) HNO3 (b)H2SO4
(c) Hydrogen peroxide (c) HCl (d) HF
(d) Bromine water Ans: (b)
Ans: (a) Q155. Which is a natural colloid ?
Q145. If the sewage is fully oxi dized, the nitrogen is in (a) Sodium chloride
the form of (b) Urea
(a) Nitrites (b) Ammonia (c) Cane-sugar
(c) Nitramines (d) Nitrates (d) Blood
Ans: (d) Ans: (d)
Q146. The value of gas constant R in Ergs deg –1 Q156. Ultraviolet rays can be used in water treatment
mole–1 is as
(a) 8.314 × 107 (b) 8.341 × 107 (a) Flocculator (b) Precipitator
(c) 8.413 × 107 (d) 4.183 × 107 (c) Hydrolyser (d) Disinfectant
Ans: (a) Ans: (d)
Q147. Nitrogen is an essential constituent of all : Q157. Gypsum is used for improvement of:
(a) Fats (b) Proteins (a) Alkaline soils (b) Saline soils
(c) Vitamins (d) Carbohydrates (c) Podsols (d) Acidic soils
Ans: (b) Ans: (b)
Q148. The percentage of nitrogen present in Q158. Mark the compound which possesses ionic,
ammonium sulphate is : covalent and coordinate bonds.
(a) 27% (b) 25% (a) NH4Cl (b) SO3
(c) 30.5% (d) 18% (c) SO2 (d) H2O
Ans: (a) Ans: (a)
Q149. A colloidal system in which a liquid is dispersed Q159. Ionisation energy of nitrogen is greater than
in a liquid is called that of oxygen because nitrogen has
(a) gel (b) emulsion (a) high bond dissociation energy
Page 402 of 516
(b) smaller atomic radius https://telegram.me/aedahamlibra
(c) stable half filled 2p sub level
(b) Nitrous oxide, Chloroform
(c) Chloroform, Nitrogen dioxide
(d) high nuclear charge (d) Nitrogen dioxide, ether
Ans: (c) Ans: (b)
Q160. An alloy used in making heating elements for Q169. Magnetite is
electric heating device is (a) Fe2O3 (b) Fe3O4
(a) Solder (b) Alloy Steel (c) FeCO3 (d) 2Fe2O3. 3H2O
(c) Nichrome (d) German Silver Ans: (b)
Ans: (c) Q170. The inert gas which is substituted for nitrogen
Q161. An example of heterocyclic compound is in the air used by deep sea divers for breathing is
(a) Naphthalene (b) Furan (a) Neon (b) Krypton
(c) Benzene (d) Anthracene (c) Argon (d) Helium
Ans: (b) Ans: (d)
Q162. Brass is an alloy of : Q171. For a spontaneous process
(a) Cu, Al and Zn (
(b) Cu and Sn (b) –ve
(c) Cu and Zn (
(d) Al and Mg (d) –ve
Ans: (c) Ans: (b)
Q163. How many bones are there in a newly born Q172. Atoms of different elements have
infant ? (a) same atomic number and same electronic
(a) 206 (b) 230 configuration
(c) 280 (d) 300 (b) different atomic number and same eletronic configu -
Ans: (c) ration
Q164. A thick paste of cement, sand and water is (c) different atomic number and different number of
called valence electrons
(a) Concrete (b) R.C.C. (d) same number of electrons and neutrons
(c) Mortar (d) Kiln Ans: (c)
Ans: (c) Q173. Antacid tablets consist of
Q165. The percentage of litrogen in Air is (a) Hydroxides of sodium, magnesium and aluminium
(a) 0.94 (b) 0.03 (b) Hydroxides of magnesium and aluminium
(c) 78.03 (d) 85.02 (c) Hydroxides of sodium and aluminium
Ans: (c) (d) Hydroxides of magnesium and sodium
Q166. Aquaregia is a mixture of the following in the Ans: (b)
ratio 1 : 3 by volume Q174. Interveinal necrotic spots on leaves appear due
(a) conc. HNO3 and conc. HCl to
(b) conc. HNO3 and conc. (a) HF injury (b) SO2 injury
H2SO4 (c) NO2 injury (d) O3 injury
(c) dil. HCl and dil. HNO3 Ans: (b)
(d) conc. HCl and conc. HNO3 Q175. The process of improving the quality of rubber
Ans: (c) by heating it with sulphur is called.
Q167. Which among the following is a covalent (a) Vulcanization
compound ? (b) Acceleration
(a) Calcium chloride (c) Sulphonation
(b) Magnesium fluoride (d) Galvanization
(c) Sodium chloride Ans: (a)
(d) Carbon tetrachloride Q176. Vulcanized rubber contains sulphur
Ans: (d) (a) 2% (b) 3-5%
Q168. The pair of compounds used as anaesthetic in (c) 7-9% (d) 10-15%
medicines : Ans: (b)
(a) Ether, Ammonia
Page 403 of 516
https://telegram.me/aedahamlibra
Q177. The apparatus used to test acidity of aqueous
solution is
(c) Xenon (d) Carbon
Ans: (b)
(a) pH meter (b) Ammeter Q187. Acid Rain is caused due to pollution of
(c) Hygrometer (d) Acidmeter atmosphere by
Ans: (a) (a) Oxides of nitrogen and phosphorus
Q178. The chemical name of baking soda is (b) Oxides of carbon and nitrogen
(a) Sodium carbonate (c) Oxides of nitrogen and sulphur
(b) Sodium bicarbonate (d) None of these
(c) Sodium chloride Ans: (c)
(d) Sodium nitrate Q188. The term ‗brown air‘ is used for
Ans: (b) (a) Acid fumes
Q179. Which of the following will replace hydrogen (b) Photochemical smog
from acids to form salts ? (c) Sulphurous smog
(a) S (b) Na (d) Industrial smog
(c) Ag (d) P Ans: (b)
Ans: (b) Q189. Cyanide poisoning causes death in seconds
Q180. Carborundum is another name of because
(a) Silicon carbide (a) It denatures enzymes of the heart/muscle
(b) Silicon oxide (b) It breaks the electron transport chain
(c) Calcium carbide (c) It causes Lysis of red blood cells.
(d) Calcium oxide (d) It causes cardiac arrest
Ans: (a) Ans: (d)
Q181. Who developed the model of atomic structure ? Q190. Which of the following partic les has the dual
(a) Bohr and Rutherford nature of particle– wave?
(b) Volta (a) Electron (b) Meson
(c) Alfred Nobel (c) Proton (d) Neutron
(d) Faraday Ans: (a)
Ans: (a) Q191. The green colour seen in firework displays is
Q182. Concentration of a material which is lethal to due to the chloride salt of :
50% animal is called (a) Sodium (b) Strontium
(a) LD50 (b) LC50 (c) Barium (d) Calcium
(c) NOAEL (d) ADI Ans: (c)
Ans: (a) Q192. Acid rain is caused due to pollution of
Q183. In galvanization, iron is coated with atmosphere by
(a) Copper (b) Zinc (a) oxides of carbon and nitrogen
(c) Tin (d) Nickel (b) oxides of nitrogen and sulphur
Ans: (b) (c) oxides of nitrogen and phosphorus
Q184. Which one of the following contains maximum (d) None of these
percentage of carbon ? Ans: (b)
(a) Cast iron Q193. The acid which fails to liberate carbon dioxide
(b) Stainless steel from sodium bicarbonate is :
(c) Wrought iron (a) Acetic acid
(d) High speed steel (b) Formic acid
Ans: (a) (c) Carbonic acid
Q185. The most abundant element is (d) Sulphuric acid
(a) Calcium (b) Silicon Ans: (d)
(c) Oxygen (d) Nitrogen Q194. If there is one million Mg 2+ ions in MgCl2, how
Ans: (c) many chloride ions are there?
Q186. Which element behaves chemically both as (a) Two million (b)One million
metal and nonmetal ? (c) Half a million (d) Ten million
(a) Argon (b) Boron Ans: (a)
Page 404 of 516
https://telegram.me/aedahamlibra
Q195. Soap helps in better cleaning of clothes because
(a) Soap acts like catalyst
(d) Ammonium Nitrate
Ans: (a)
(b) It reduces the surface tension of solution Q204. Limestone is a raw material used by which
(c) It gives strength to solution industry?
(d) It absorbs the dirt (a) Aluminium
Ans: (b) (b) Fertilizers
Q196. The ―King of Metals‖ is : (c) Cement
(a) Gold (b) Silver (d) Petrochemicals
(c) Iron (d) Aluminium Ans: (c)
Ans: (a) Q205. The most abundant metal in the Earth‘s crust
Q197. Chile saltpeter is the common name of: is
(a) Sodium nitrate (a) zinc (b) copper
(b) Potassium nitrite (c) aluminium (d) iron
(c) Potassium nitrate Ans: (c)
(d) Sodium nitrite Q206. Copper substances when exposed to air gains a
Ans: (a) green coating due to the formation of:
Q198. Soldering of two metals is possible because of (a) CuO
the property of : (b) CuCO3 Cu (OH)2
(a) Viscosity (b) Osmosis (c) CuSO4
(c) Cohesion (d) Cu (NO3)2
(d) Surface tension Ans: (b)
Ans: (c) Q207. Which of the following is the correct sequence
Q199. The deterioration of a metal by an of countries in terms of maximum carbon dioxide
electrochemical process is commonly termed as emission?
(a) Erosion (b) Corrosion (a) China, USA, European Union, India
(c) Passivation (d) Abrasion (b) USA, India, China, European Union
Ans: (b) (c) China, India, European Union, USA
Q200. The inert gas which is substituted for nitrogen (d) European Union, China, USA, India
in the air used by deep sea divers for breathing, is Ans: (a)
(a) Xenon (b) Krypton Q208. Helium is added to the oxygen supply of deep
(c) Argon (d) Helium sea divers because it is
Ans: (d) (a) less poisonous than nitrogen
Q201. Which of the following is not a gaseous air (b) Lighter than nitrogen
pollutant? (c) Readily miscible with oxygen
(a) Oxides of nitrogen (d) less soluble in blood than nitrogen at high pressure
(b) Hydrocarbon Ans: (d)
(c) Smoke Q209. L.P.G. is mostly liquefied
(d) Oxides of sulphur (a) hydrogen (b) oxygen
Ans: (b) (c) butane (d) methane
Q202. Which method wil l be employed to test the Ans: (c)
hardness of water? Q210. Which of the following is not soluble in water ?
(a) Boiling (a) Lead Sulphate
(b) Distillation (b) Zinc sulphate
(c) Formation of lather with soap (c) Potassium sulphate
(d) None of these (d) Sodium sulphate
Ans: (c) Ans: (b)
Q203. Which of the following is the main compound Q211. For extinguishing fire, we use
of voter ink? (a) Hydrogen
(a) Silver nitrate (b) Carbon Monoxide
(b) Ammonium Chloride (c) Carbon Dioxide
(c) Silver Chloride (d) Marsh gas
Page 405 of 516
https://telegram.me/aedahamlibra
Q212. Sea water is saltier than rain water because
Ans: (c) (c) Graphite
(d) Wood Charcoal
(a) Sea animals are salt producing Ans: (a)
(b) The air around the sea is saltish Q221. Which one of the following forms of
(c) Rivers wash away salts from earth and pour them into phosphorus is most reactive?
the sea (a) Black phosphorus
(d) Sea beds have salt producing mines (b) White phosphorus
Ans: (c) (c) Violet phosphorus
Q213. Which of the process is known as nitrification? (d) Red phosphorus
(a) Reaction of Nitrogen Monoxide with oxygen to form Ans: (b)
nitric acid Q222. Bee sting contains
(b) Reaction of nitrogen dioxide with water to form nitric (a) An acidic liquid
acid (b) a salt solution
(c) Conversion of ammonia to nitrites (c) an alkaline liquid
(d) Conversion of nitrite to nitric oxide (d) a corrosive liquid
Ans: (c) Ans: (a)
Q214. Besides CO2, the other green house gas is Q223. The manufacturing of iron from ironore
(a) CH4 (b) N2 N2 involves the process of
(c) Ar Ar (d) O2 (a) Oxidation
Ans: (a) (b) Reduction
Q215. Which of the following is not a nitrogenous (c) Electrolysis
fetiliser? (d) Fractional distillation
(a) Ammonium sulphate Ans: (b)
(b) Urea Q224. Synthetic detergents are prepared from
(c) Ammonium nitrate (a) potassium salts of higher fatty-
(d) Superphosphate acids
Ans: (d) (b) sodium salts of higher fattyacids
Q216. Silver gets corroded due to ___ in air. (c) hydrocarbons of petroleum
(a) Oxygen (d) glycerides
(b) Hydrogen Sulphide Ans: (c)
(c) Carbon dioxide Q225. Which of the following gases is known as
(d) Nitrogen ―Laughing Gas‖?
Ans: (b) (a) Nitrous oxide
Q217. Biofertilizers convert nitrogen to _______ . (b) Nitrogen peroxide
(a) nitrates (b) ammonia (c) Nitrogen
(c) nitrogenase (d) amino acids (d) Nitric oxide
Ans: (b) Ans: (a)
Q218. Inert gases are Q226. The compound used in pricklyheat powder to
(a) Miscible with water prevent excessive sweating is
(b) Not stable (a) Boric acid
(c) Chemically unreactive (b) Phosphoric acid
(d) Chemically very active (c) Carbonic acid
Ans: (c) (d) Sulphonic acid
Q219. Which of the following fuels causes minimum Ans: (a)
environmental pollution? Q227. Sour taste of lemon is d ue to the presence of
(a) Diesel (b) Kerosene which of the following ?
(c) Hydrogen (d) Coal (a) Citric acid (b) Acetic acid
Ans: (c) (c) Oxalic acid (d) Formic acid
Q220. The most pure form of Carbon among the Ans: (a)
options is Q228. Which of the following acts as best adsorbent?
(a) Anthracite (b) Lampblack (a) Charcoal
Page 406 of 516
(b) Activated Charcoal
(c) Activated Coconut Charcoal
https://telegram.me/aedahamlibra
Q3. What is the limit of MG/DL of blood sugar in the
normal person at the time of fast ?
(d) Carbon black (a) 40-60 (b) 120-150
Ans: (c) (c) 70-100 (d) 160-200
Q229. Which one of the following takes place during a Ans: (c)
nuclear fusion? Q4. Why radiologists do not take direct X-ray
(a) a heavy nucleus bombarded by neutrons breaks up photographs of intestine ?
(b) a heavy nucleus breaks up spontaneously (a) Intestines are affected by direct exposure to X-rays
(c) two light nuclei combine to form a heavy nucleus (b) X-rays can not cross intestine
(d) a light nucleus breaks up spontaneously (c) X-rays are not able to capture clear picture
Ans: (c) (d) Even short exposure of X -rays causes cancer in
Q230. Which acid is used in Lead storage battery? intestine
(a) Sulphuric acid Ans: (c)
(b) Nitric acid Q5. The pollen grains of flowers pollinated by insects
(c) Hydrochloric acid are :
(d) Acetic acid (a) smooth and dry
Ans: (a) (b) rough and sticky
Q231. Which of the following is also used as a Bio (c) rough and dry
fertilizer? (d) large and showy
(a) Urea (b) Ammonia Ans: (b)
(c) Uric Acid (d) Nitrates Q6. Which of the following is NOT one of the
Ans: (*) important steps in processing tea leaves?
Q232. Which among the following is not a (a) Rolling (b) Drying
characteristic of transition metals? (c) Fermenting
(a) Tendency to gain electrons (d) Withering
(b) Low electronegativity Ans: (c)
(c) Low ionization energy Q7. The age of trees is determined by its :
(d) Malleability (a) girth (b) height
Ans: (a) (c) growth rings
Q233. Malathion is a/an (d) general appearance
(a) insecticide (b) fumigant Ans: (c)
(c) preservative (d) pesticide Q8. Quantity of fresh air required for a man is
Ans: (a) (a) 1000 cubic feet of air for every 20 minutes
(b) 1000 cubic feet of air for every 20 seconds
(c) 1000 cubic feet of air for every 10 minutes
Biology (d) 1000 cubic feet of air for every 10 seconds
Ans: (a)
Q1. Which among the following helps in circulation of Q9. Which one of the following is a female sex
blood ? hormone?
(a) Arithrocytus (a) Estrogen (b) Androgen
(b) Blood platelets (c) Oxytocin (d) Insulin
(c) Monocytes Ans: (a)
(d) Lymphocytes Q10. AIDS virus has :
Ans: (d) (a) single-stranded RNA
Q2. Our bones and teeth are generally made of — (b) double-stranded RNA
(a) Tricalcium phosphate (c) single-stranded DNA
(b) Fluoropetite (d) double-stranded DNA
(c) Chloropetite Ans: (a)
(d) Hydrolith Q11. Typhoid fever is caused by-
Ans: (a) (a) virus (b) bacteria
(c) fungus (d) allergy
Page 407 of 516
https://telegram.me/aedahamlibra
Ans: (b)
Q12. All the progeny obtained from a si ngle plant by
(b) Monocytes
(c) Neutrophils
vegetative propagation are called (d) Lymphoctyes
(a) Clones (b) Pure line Ans: (a)
(c) Indred line (d) Pedigree line Q22. Which organ of the body never rests ?
Ans: (a) (a) Eyes (b) Pancreas
Q13. Which of the following is known as Vitamin B1? (c) Liver (d) Heart
(a) Retinol (b) Thiamin Ans: (d)
(c) Riboflavin (d) Ascorbic Acid Q23. Which of the following is most important for
Ans: (b) growth of children upto the age of 14?
Q14. Who amongst t he following was awarded Nobel (a) Proteins (b) Vitamins
Prize for path breaking contributions to develop (c) Fats (d) Milk
treatment against Parkinson's disease ? Ans: (a)
(a) Arvid Carlsson Q24. A plant with compound leaves is
(b) Walter Kohn (a) Papaya (b) Coconut
(c) Robert B. Laughlin (c) Peepal (d) Hibiscus
(d) Ferid Murad Ans: (b)
Ans: (a) Q25. In a human body, the longest bone is in the
Q15. The element which is the most abundant in the (a) vertebral column
human body is: (b) thigh (c) rib cage
(a) Oxygen (b) Carbon (d) arm
(c) Iron (d) Nitrogen Ans: (b)
Ans: (a) Q26. The total number of vertebrae in a human being
Q16. Fermentation is a process of decomposition of an is
organic compound by : (a) 26 (b) 30
(a) catalysts (b) enzymes (c) 29 (d) 33
(c) carbanions (d) free radicals Ans: (d)
Ans: (b) Q27. Glaucoma is a disease of the
Q17. Amniocentesis is a method for : (a) Skin (b) Lungs
(a) determination of foetal health conditions (c) Liver (d) Eyes
(b) determination of the amino acids sequence Ans: (d)
(c) inducing abortion Q28. Who among the following analysed DNA for the
(d) artificial insemination first time ?
Ans: (a) (a) Arthur Cornberg
Q18. Virus contains (b) Hargobind Khurana
(a) Protein and lipid (c) M. W. Nirenberg
(b) Nucleic acid and protein (d) Watson and Krick
(c) Lipid and carbohydrate Ans: (d)
(d) Carbohydrate and nucleic acid Q29. Which vitamin helps in blood coagulation?
Ans: (a) (a) Vitamin-A (b) Vitamin-C
Q19. Night blindness results from the deficiency of — (c) Vitamin-K (d) Vitamin-E
(a) Glucose (b) Vitamin-E Ans: (c)
(c) Vitamin-B2 (d) Vitamin-A Q30. The source of oxygen generated during
Ans: (d) photosynthesis is :
Q20. Water in plants is trans-ported by (a) water
(a) cambium (b) phloem (b) carbon dioxide
(c) epidermis (d) xylem (c) chlorophyll
Ans: (d) (d) mesophyll cells
Q21. What are the blood corpuscles that help to build Ans: (a)
up resistance against diseases ? Q31. The Vitamin which helps in clotting of blood is :
(a) Leucocytes (a) A (b) D
Page 408 of 516
(c) B https://telegram.me/aedahamlibra
(d) K
Ans: (d)
(a) Pupil
(c) Iris
(b) Cornea
(d) Choroid
Q32. Roundworm is a human parasite found in the Ans: (c)
(a) Small intestine Q42. In which of the following generally hormone is
(b) Liver (c) Stomach not found ?
(d) Large intestine (a) Rat (b) Monkey
Ans: (d) (c) Bacteria (d) Cat
Q33. Out of the following glands which is referred to Ans: (d)
as the master gland? Q43. Which of the following represents a foodchain
(a) Thyroid (b) Adrenal gland involving a producer, a vegetarian and a
(c) Pituitary (d) Pancreas nonvegetarian ?
Ans: (c) (a) Grass – Insect – Elephant
Q34. Process of digestion is helped by (b) Plant – Rabbit – Tiger
(a) Enzyme (b) Hormone (c) Fish – Insect – Whale
(c) Mineral (d) Vitamin (d) Tiger – Rabbit – Owl
Ans: (a) Ans: (b)
Q35. Excretory products of mammalian embryo are Q44. Which of the following is correlated with blood
eliminated out by pressure ?
(a) Placenta (b) Amniotic fluid (a) Liver (b) Testis
(c) Allantois (d) Ureter (c) Pancreas (d) Adrenal
Ans: (a) Ans: (d)
Q36. Milk is not considered a b alanced diet now -a- Q45. Correlate the following :
days because of the absence of : Source
(a) Magnesium and Vitamin D (a) Green leafy vegetables
(b) Iron and Vitamin C (b) Raw cabbage
(c) Calcium and Vitamin C (c) Brewers yeast
(d) Iron and Vitamin A (d) Wheat-germ oil Vitamin
Ans: (b) (a) A (b) B1
Q37. The largest cell in the human body is : (c) C (d) D
(a) Nerve cell (b) Muscle cell (5) E
(c) Liver cell (d) Kidney cell (a) (b) (c) (d)
Ans: (a) (a) (a) (b) (c) (d)
Q38. The animal which uses sounds as its ‗eyes‘ is (b) (a) (c) (b) (5)
(a) Dog (b) Cat (c) (b) ( c)
(c) Snake (d) Bat (d) (5)
Ans: (d) (d) (b)
Q39. Eyes of potato are useful for (d) (5) (a)
(a) Nutrition Ans: (b)
(b) Respiration Q46. Select the biofertilizer in the following :
(c) Reproduction (a) Compost
(d) Vegetative propagation (b) Ammonium sulphate
Ans: (d) (c) Cattle dung
Q40. Who discovered the Cholera-bacillus ? (d) Algae and blue-green algae
(a) Louis Pasteur Ans: (d)
(b) Ronald Ross Q47. Which of the following is effective against
(c) Robert Koch tuberculosis ?
(d) Joseph Lister (a) Penicillin
Ans: (c) (b) Chloromycetin
Q41. A person will have brown eyes, blue eyes or (c) Terramycin
black eyes depending on the particular pigment (d) Streptomycin
present in the : Ans: (d)
Page 409 of 516
https://telegram.me/aedahamlibra
Q48. Match List I with List II and select the correct
answer using the codes given below the lists:
Q55. Which one of the following is a non
snake ?
-poisonous

List-I : Symptom (a) Cobra (b) Dryophis


A. Weak sight (c) Elapes (d) Python
B. Anaemia Ans: (d)
C. Skin problem Q56. The standard audible capacity of a healthy
D. Breaking of bones List-II : Cause human being as pe r World Health Organi -sation is in
1. Deficiency of Iron the range of
2. Deficiency of Vitamin C (a) 45-50 decibels
3. Deficiency of Vitamin A (b) 200-250 decibels
4. Deficiency of Calcium A B C D (c) 5-10 decibels
(a) 2 4 1 3 (d) 2000-2500 decibels
(b) 3 2 1 4 Ans: (c)
(c) 3 1 2 4 Q57. Triple vaccine is administered to a new born
(d) 2 3 4 1 child to immunize it against
Ans: (c) (a) whooping cough, tetanus and measles
Q49. Bleeding of gums and loosening of teeth is (b) whooping cough, tetanus and diphtheria
caused due to deficiency of (c) tetanus, diphtheria, measles and rubella
(a) Vitamin A (b) Vitamin B (d) tetanus, diphtheria, small pox and rubella
(c) Vitamin C (d) Vitamin D Ans: (b)
Ans: (c) Q58. Heart murmur indicates a
Q50. Silk is obtained from— (a) defective valve
(a) Mulberry tree (b) poor oxygenation
(b) Saliva of the silk worm (c) dislocation of the heart
(c) Larvae of silk worm (d) improper development of muscles
(d) Cocoon of silk worm Ans: (a)
Ans: (*) Q59. Raja Rao the famous Indian Novelist who died
Q51. Animals having open circulatory system on 8th July 2006.
possesses : The title of his first novel was
(a) coelenteron (b) spongocoel (a) Kanthapura
(c) pseudocoel (d) haemocoel (b) The Serpent and The Rope
Ans: (d) (c) The Chess Master and His Moves
Q52. Scurvy is a disease which is caused due to the (d) The Cat and Shakespeare
deficiency of vitamin Ans: (a)
(a) A (b) K Q60. The branch of biology dealing with the study of
(c) C (d) B12 cells is known as
Ans: (c) (a) Cytology (b) Histology
Q53. ‗Why excessive heating and repeated use of (c) Psychology (d) Physiology
cooking oil are most undesirable ? Ans: (a)
(a) The oil vapour can cause indoor pollution Q61. The enzyme that is present in the saliva of man
(b) Carcinogenic substance s like benzpyrene are is
produced (a) Pepsin (b) Renin
(c) Nutrient value of food is lost (c) Amylase (d) Trypsin
(d) Loss and wastage of oil Ans: (c)
Ans: (b) Q62. Which one of the following set belongs to the
Q54. Quarantine regulation is concerned with class Mammalia ?
(a) growing of better varieties of plant (a) Lion, Hippopotamus, Penguin, Bat.
(b) prevention of entry of diseased organism (b) Lion, Bat, Whale, Ostrich
(c) spraying of insecticide over diseased plants (c) Hippopotamus, Penguin, Whale, Kangaroo
(d) identification of diseased organism (d) Whale, Bat, Kangaroo, Hippopotamus
Ans: (b) Ans: (d)
Page 410 of 516
(a) Star fish
https://telegram.me/aedahamlibra
Q63. Which of the following has no skeleton at all ?
(b) Sponge
Q72. The vitamin that helps in blood clotting is
(a) Vitamin C (b) Vitamin D
(c) Jelly fish (d) Silver fish (c) Vitamin E (d) Vitamin K
Ans: (d) Ans: (d)
Q64. The disease in which the sugar level of blood Q73. How many bones are there in the human body ?
increases is known as (a) 187 (b) 287
(a) Diabetes mellitus (c) 206 (d) 306
(b) Diabetes insipidus Ans: (c)
(c) Diabetes imperfectus Q74. The normal RBC count in adult male is
(d) Diabetes sugarensis (a) 5.5 million (b) 5.0 million
Ans: (a) (c) 4.5 million (d) 4.0 million
Q65. Which one of the following animals is called Ans: (b)
farmer‘s friend ? Q75. Vitamin E is particularly important for
(a) Ant (b) Earthworm (a) development of teeth
(c) Bee (d) Butterfly (b) carbohydrate metabolism
Ans: (b) (c) normal activity of sex glands
Q66. Study of newly born up to the age of 3 months is (d) general health of epithelial tissues
called Ans: (c)
(a) Chorology (b) Conchology Q76. Maximum harm to a tree is caused by
(c) Nematology (d) Neonatology (a) Loss of half of its leaves
Ans: (d) (b) loss of all leaves
Q67. Which of the following organisms is most likely (c) loss of half of its branches
to produce greenhouse gases such as nitrous oxide and (d) loss of its bark
methane? Ans: (b)
(a) Fungi (b) Earthworm Q77. In which vertebrate oxygenated and
(c) Bacteria (d) Green plants deoxygenated blood gets mixed ?
Ans: (d) (a) Fish (b) Amphibian
Q68. Flat footed camels can walk easily in sandy (c) Bird (d) Mammal
deserts because Ans: (b)
(a) pressu re on the sand is decreased by increasing the Q78. Bee Keeping is known as
area of the surface in contact (a) Sericulture
(b) pressure on the sand is increased by increasing the (b) Apiculture
area of the surface in contact (c) Aquaculture
(c) pressure on the sand is decreased by decreasing the (d) Agriculture
area of the surface in contact Ans: (b)
(d) pressure on the sand is increased by decreasing the Q79. Pick out the viral disease among the following :
area of the surface in contact (a) Hepatitis (b) Meningitis
Ans: (a) (c) Arthritis (d) Nephritis
Q69. Jaundice is a symptom of disease of Ans: (a)
(a) Kidney (b) Liver Q80. For a healthy heart, one needs to take a balanced
(c) Pancreas (d) Thyroid diet, adequate sleep and
Ans: (b) (a) indulge in vigorous mental activities
Q70. Yellow Fever is transmitted by (b) play games like carrom, chess and cards
(a) Aedes (b) Anopheles (c) do right amount of physical exercise
(c) House-fly (d) Culex (d) do sedentary work
Ans: (a) Ans: (c)
Q71. The metal, which is a constituent of vitamin B12 Q81. An ECG shows the functioning of the
is (a) brain (b) heart
(a) Iron (b) Magnesium (c) lungs (d) kidneys
(c) Zinc (d) Cobalt Ans: (b)
Ans: (d)
Page 411 of 516
referred as
https://telegram.me/aedahamlibra
Q82. Insufficient blood supply in human body is Q90. The colour of the eye depends upon the pigment
present in
(a) Ischemia (a) cornea (b) iris
(b) Hyperemia (c) rods (d) cones
(c) Hemostasia Ans: (b)
(d) Hemorrhage Q91. Which one of the following is not a vaccine ?
Ans: (a) (a) BCG
Q83. BCG immunization is for (b) Anti-rabies
(a) Measles (c) Polio vaccine
(b) Tuberculosis (d) Progesterone
(c) Diphtheria Ans: (d)
(d) Leprosy Q92. An instrument for measuring blo od pressure is
Ans: (b) called
Q84. The chemicals released by one species of animals (a) Barometer
in order to attract the other m embers of the same (b) Spirometer
species are (c) Sphygmomanometer
(a) Hormones (d) Haemocytometer
(b) Nucleic acids Ans: (c)
(c) Pheromones Q93. The animal which can tolerate more summer
(d) Steroids heat is
Ans: (c) (a) Buffalo (b) Cow
Q85. The colour of cow‘s milk is slightly yellow due to (c) Goat (d) Donkey
the presence of Ans: (c)
(a) Xanthophyll Q94. The dried flower buds are used as a spice in
(b) Riboflavin (a) Cardamom (b) Cinnamon
(c) Ribulose (c) Cloves (c) Saffron
(d) Carotene Ans: (c)
Ans: (d) Q95. Which of the following are warm -blooded
Q86. Cereals are a rich source of animals ?
(a) Starch (b) Glucose (a) Whales (b)Whale Sharks
(c) Fructose (d) Maltose (c) Alytes (d) Draco
Ans: (a) Ans: (a)
Q87. Plasma membrane in eukaryotic cells is made up Q96. In coriander, the useful parts are
of (a) roots & leaves
(a) Phospholipid (b) leaves & flowers
(b) Lipoprotein (c) leaves & dried fruits
(c) Phospholipo-protein (d) flowers & dried fruits
(d) Phospho-protein Ans: (c)
Ans: (a) Q97. Which amongst the following is the largest
Q88. What happens to a person who receive s the mammal ?
wrong type of blood? (a) Elephant (b) Whale
(a) All the arteries constrict (c) Dinosaur (d) Rhinoceros
(b) All the arteries dialates Ans: (b)
(c) The RBCs agglutinate Q98. Bark of this tree is used as a condiment–
(d) The spleen and lymphnodes deteriorate (a) Cinnamon (b) Clove
Ans: (c) (c) Neem (d) Palm
Q89. Chewing gum is made from Ans: (a)
(a) Resin (b) Tannin Q99. White lung disease is prevalent among the
(c) Latex (d) Gum workers of :
Ans: (c) (a) Paper industry
(b) Cement industry
Page 412 of 516
(c) Cotton industry
(d) Pesticide industry
https://telegram.me/aedahamlibra
(d) Initiation of respiration
Ans: (c)
Ans: (*) Q110. The number of heart beats on an average in an
Q100. Normal fasting blood sugar level per 100 ml. of adult human is in the range of
blood in man is (a) 60-65 (b) 66-70
(a) 30 - 50 mg (c) 71-80 (d) 85-90
(b) 50 - 70 mg Ans: (c)
(c) 80 -100 mg Q111. Some viruses have RNA but no
(d) 120 -140 mg DNA. This would indicate that–
Ans: (c) (a) these viruses cannot replicate
Q101. ‗Cod‘ is a variety of (b) these viruses have no heritable information
(a) Goat (b) Fish (c) RNA transmits the hereditary information in these
(c) Crop (d) Coral viruses
Ans: (b) (d) their nucleic acids can be crystallised
Q102. The total number of bones in our body is : Ans: (b)
(a) 226 (b) 206 Q112. Which of the following is not a par t of the
(c) 256 (d) 236 Darwin‘s theory of evolution?
Ans: (b) (a) Natural selection
Q103. The process of preventing t he birds from flying (b) Struggle for existence
is called (c) Survival of the fittest
(a) brailing (b) debeaking (d) Inheritance of acquired characters
(c) dubbing (d) pecking Ans: (d)
Ans: (a) Q113. Dialysis is used to perform the function of—
Q104. Goitre is caused by the deficiency of (a) Lungs (b) Heart
(a) Iodine (b) Chlorine (c) Liver (d) Kidneys
(c) Sodium (d) Calcium Ans: (d)
Ans: (a) Q114. The deficiency of iron in man result in
Q105. To which group of blood an universal recipient (a) Anaemia (b) Night blindness
belongs? (c) Scurvy (d) Rickets
(a) A group (b) B group Ans: (a)
(c) 0 group (d) AB group Q115. Normal blood pressure of man is
Ans: (d) (a) 80/120 mm Hg
Q106. An insect - catching plant is : (b) 90/140 mm Hg
(a) Australian Acacia (c) 120/160 mm Hg
(b) Smilax (d) 85/120 mm Hg
(c) Nepenthes Ans: (a)
(d) Nerium Q116. Vegetable oils are converted into solid fat
Ans: (c) (ghee) by
Q107. Scurvy is caused due to the deficiency of : (a) Hydrolysis
(a) Vitamin-D (b) Vitamin-K (b) Addition of agar
(c) Vitamin-E (d) Vitamin-C (c) Oxidation using air and a catalyst
Ans: (d) (d) Hydrogenation
Q108. The percentage of water content in the human Ans: (d)
blood plasma normally varies from Q117. Which one of the following four secretions, is
(a) 60-64 (b) 70-75 different from the remaining three in regard to its
(c) 80-82 (d) 91-92 mode of transport from the source gland to the site of
Ans: (d) action?
Q109. The functions of pacemaker is (a) Saliva (b) Sweat
(a) Regulation of urine formation (c) Bile (d) Epinephrine
(b) Regulation of digestion Ans: (c)
(c) Initiation of heart beat
Page 413 of 516
https://telegram.me/aedahamlibra
Q118. Which of the following groups of organisms
digest their food before it actually enters the
(b) hydrolysis
(c) fermentation
organism? (d) oxidation
(a) Bacteria and Protozoa Ans: (c)
(b) Bacteria and Fungi Q127. The floral part that produces pollen grains is
(c) Fungi and Protozoa (a) Sepal (b) Petal
(d) Mucor and Rhizopus (c) Anther (d) Ovary
Ans: (b) Ans: (c)
Q119. Why death of fish is more common during Q128. A person of blood group AB can receive blood
summer than in winter? from a person of
It is because of (a) Only group AB
(a) shortage of food (b) Only group A
(b) concentration of toxins (c) Only group B
(c) depletion of oxygen (d) Any blood group
(d) spread of diseases Ans: (d)
Ans: (c) Q129. Normal blood pressure of man is
Q120. The colour of cow‘s milk is slightly yellow due (a) 80/120 mm Hg
to the presence of (b) 90/140 mm Hg
(a) xanthophyll (b) riboflavin (c) 120/160 mm Hg
(c) ribulose (d) carotin (d) 85/150 mm Hg
Ans: (d) Ans: (a)
Q121. Maximum harm to a tree is caused by Q130. Which one of the following is a hereditary
(a) loss of half of its leaves disease ?
(b) loss of all leaves (a) Haemophilia
(c) loss of half of its branches (b) Myopia
(d) loss of its bark (c) Anaemia
Ans: (b) (d) Leukemia
Q122. Which cells in our body have the least Ans: (a)
regenerative power? Q131. Butter is
(a) Brain cells (a) fat dispersed in milk
(b) Mussle cells (b) water dispersed in fat
(c) Bone cells (c) water dispersed in oil
(d) Liver cells (d) fat dispersed in water
Ans: (a) Ans: (b)
Q123. The total number of ear bones is Q132. Which of the following is a biodegradable waste
(a) 2 (b) 4 ?
(c) 6 (d) 8 (a) Wool
Ans: (c) (b) Polythene bags
Q124. It causes clotting of blood (c) Plastics
(a) Thrombin (d) Nylon
(b) Haemoglobin Ans: (a)
(c) Pectin Q133. Entamoeba histolytica is a parasite found in
(d) All of the above man‘s
Ans: (a) (a) liver (b) intestines
Q125. Which of the following is not a gland ? (c) lungs (d) kidneys
(a) Thyroid (b) Stomach Ans: (b)
(c) Liver (d) Pancreas Q134. Which of the following diseases is caused by a
Ans: (b) virus?
Q126. The method used to obtain alcohol from (a) Influenza
molasses is called (b) Pneumonia
(a) distillation (c) Cholera
Page 414 of 516
(d) Whooping cough https://telegram.me/aedahamlibra
Ans: (a)
(c) Sodium (d) Calcium
Ans: (b)
Q135. Chicken pox is caused by Q144. Genes, the hereditary units, are located in the
(a) Protozoa (b) Bacteria (a) nuclear membrane
(c) Virus (d) Fungus (b) chromosomes
Ans: (c) (c) lysosomes
Q136. Malaria can be cured with a drug extracted (d) cell membrane
from Ans: (b)
(a) Belladonna tree Q145. The blood vessel which carries oxygenated
(b) Cinchona tree blood to the liver is
(c) Oak tree (a) Coronary Artery
(d) Neem tree (b) Pulmonary Artery
Ans: (b) (c) Carotid Artery
Q137. A disease caused by fungus is (d) Hepatic Artery
(a) Round worm Ans: (d)
(b) Ringworm Q146. The DPT vaccine is given toyoung babies to
(c) Tapeworm protect them from
(d) Filaria (a) diphtheria, polio and tetanus
Ans: (b) (b) diphtheria, pneumonia and tuberculosis
Q138. Rabies is caused by (c) diphtheria, smallpox and tetanus
(a) bacteria (b) virus (d) diphtheria, whooping cough and tetanus
(c) fungus (d) algae Ans: (d)
Ans: (b) Q147. Haemoglobin is an important constituent of
Q139. The gas released during photosynthesis : (a) Red blood cells
(a) Carbon dioxide (b) White blood cells
(b) Oxygen (c) Platelets
(c) Carbon monoxide (d) Plasma
(d) Sulphur dioxide Ans: (a)
Ans: (b) Q148. Which part of the nervous system controls the
Q140. Which of the following events occurred first ? activities of internal organs?
(a) DNA structure of genes was discovered by James D (a) Spinal cord
Watson and Francis Crick (b) Cerebrum
(b) Yuri Gagarin became the first man in space (c) Cerebellum
(c) World‘s first h uman heart transplant was performed (d) Medulla Oblongata
by Dr. Christian Barnard Ans: (d)
(d) World‘s first test tube baby Louise Brown was born Q149. ‗Lockjaw‘ is the last phase of which of the
in U.K. following diseases?
Ans: (a) (a) Diptheria (b) Pneumonia
Q141. Which of these is produced by a mould? (c) Syphilis (d) Tetanus
(a) Chloroform (b) Glycerine Ans: (d)
(c) Penicillin (d) Quinine Q150. The following sugar gives energy most readily
Ans: (c) (a) Lactose (b) Cellulose
Q142. The acid present in gastric juice is (c) Maltose (d) Glucose
(a) Hydrochloric Ans: (d)
(b) Nitric Q151. The food conducting tissue of a plant is
(c) Sulphuric (a) Phloem (b) Xylem
(d) Ascorbic (c) Parenchyma
Ans: (a) (d) Collenchyma
Q143. Bone is used as a fertiliser because it contains Ans: (a)
the plant nutrient Q152. The sailva secreted in the mouth digests
(a) Nitrogen (b) Phosphorus (a) Proteins (b) Starch
Page 415 of 516
(c) Fats https://telegram.me/aedahamlibra
(d) Vitamins
Ans: (b) Q163. The red colour of human blood is due to
Ans: (a)

Q153. Which of these produces energy? (a) myoglobin


(a) Carbohydrates (b) hemoglobin
(b) Proteins (c) Vitamins (c) immunoglobulin
(d) Mineral salts (d) haptoglobin
Ans: (a) Ans: (b)
Q154. The human skeleton contains aorund Q164. Which of the following processes does not
(a) 110 bones (b) 210 bones increase the amount of carbon dioxide in air ?
(c) 150 bones (d) 250 bones (a) Breathing
Ans: (*) (b) Photosynthesis
Q155. Which one of the following is not a true fish ? (c) Burning of petrol
(a) Shark (b) Starfish (d) Aerobic decay of vegetation
(c) Eel (d) Sea-horse Ans: (b)
Ans: (b) Q165. How many valves does a human heart have?
Q156. Which are not included in the milk -teeth in a (a) Four (b) Three
child of 3-4 years ? (c) Two (d) One
(a) Incisors (b) Canines Ans: (a)
(c) Molars (d) Premolars Q166. Which one of the following pairs is correctly
Ans: (c) matched?
Q157. The primary source of carbohydrates are (a) Tetanus – BCG
(a) Marine animals (b) Tuberculosis – ATS
(b) Plants (c) Malaria – Chloroquin
(c) Coal-tar (d) Scurvy – Thiamin
(d) Crude oil Ans: (c)
Ans: (b) Q167. Mumps is a viral disease that c inflammation of
Q158. Opium is a plant product obtained from :
(a) dried leaves (b) dried latex (a) Parotid gland
(c) roots (d) stem bark (b) Sublingual gland
Ans: (b) (c) Submaxillary gland
Q159. Which of the following animals has a clitellum (d) Infra-orbital gland
? Ans: (a)
(a) Millipede (b) Centipede Q168. Tobacco smoke is injurious to health because it
(c) Earthworm (d) Ringworm contains :
Ans: (c) (a) Carbon monoxide
Q160. Which of the following metals is present in (b) Nicotine
chlorophyll? (c) Polycyclic aromatic hydrocarbons
(a) Beryllium (b) Magnesium (d) Melathene
(c) Calcium (d) Barium Ans: (a)
Ans: (b) Q169. Tuberculosis infection is by means of :
Q161. Heavy alcohol consuming people generally die (a) Mycobacterium avonin
of (b) Staphylococcus
(a) blood cancer (c) Mycobacterium tuberculosis
(b) cirrhosis (d) Streptococcus
(c) liver or stomach cancer Ans: (c)
(d) weakening of heart muscles leading to cardiac arrest Q170. The element which is required by the plant in
Ans: (b) large quantity :
Q162. Which virus from the following combinations is (a) Calcium (b) Nitrogen
contagious for human beings (c) Phosphorus (d) Sulphur
(a) H5N1 (b) H2N3 Ans: (b)
(c) H4N1 (d) H4N2 Q171. Which of the following causes Malaria ?
Page 416 of 516
(a) Insect
(c) Protozoa
https://telegram.me/aedahamlibra
(b) Bacteria
(d) Virus
(d) Bone cancer
Ans: (b)
Ans: (c) Q180. Which of the following is used for wrapping of
Q172. Which one of the following organs excretes fractured bones?
water, fat and various catabolic wastes ? (a) White cement
(a) Kidney (b) Skin (b) White lead
(c) Spleen (d) Salivary glands (c) Zinc oxide
Ans: (a) (d) Plaster of Paris
Q173. Which of the following branches deals with the Ans: (d)
interactions of same species of living organisms with Q181. Which one of the following is an extinct animal
their non-living environment ? ?
(a) Autecology (a) Passenger pigeon
(b) Synecology (b) Mountain quail
(c) Ecology (c) Pink-headed duck
(d) Palaeontology (d) Ibis
Ans: (c) Ans: (a)
Q174. The contractile proteins in a muscle are Q182. In which one of the following is swim bladder
(a) Actin and Myosin absent ?
(b) Actin and Tropomyosin (a) Cuttlefish
(c) Myosin and Troponin (b) Bony fish
(d) Troponin and Tropomyosin (c) Cartilaginous fish
Ans: (a) (d) Silverfish
Q175. Allosomes are Ans: (c)
(a) cell organelles Q183. The chief raw material used for manufacturing
(b) plant hormones Rayon is :
(c) alleles (a) Nylon
(d) sex chromosomes (b) Cellulose
Ans: (d) (c) Silicon
Q176. Dormancy period of animals during winter (d) Radium and Argon
season is called : Ans: (b)
(a) Aestivation Q184. The coding segment of DNA is called in
(b) Hibernation (a) Codon (b) Muton
(c) Regeneration (c) Intron (d) Exon
(d) Mutation Ans: (a)
Ans: (b) Q185. The colour change in the Chameleon is due to
Q177. IUCN categorized major threatened species the presence of
under : (a) Haemoglobin
(a) seven classes (b) Chromatophore
(b) five classes (c) Chlorophyll
(c) six classes (d) Pneumatophore
(d) four classes Ans: (b)
Ans: (*) Q186. Which of the following correctly explains the
Q178. The total number of bones in the human body phenomenon of ―Test Tube Baby‖ ?
is : (a) When every process of embryo formation is in the test
(a) 206 (b) 260 tube.
(c) 306 (d) 360 (b) When the embryo develops in a test tube.
Ans: (a) (c) When the fertilisation is external and development is
Q179. The radioactive Strontium-90 causes : internal.
(a) Brain cancer (d) When the fertilisation is internal and development is
(b) Skin cancer external.
(c) Lung cancer Ans: (c)
Page 417 of 516
https://telegram.me/aedahamlibra
Q187. Which one of the following animals is
sanguinivorous ?
(c) Protozoa (d) Fungi
Ans: (b)
(a) Fruit-fly (b) Mosquito Q197. A seed can germinate in the absence of
(c) House-fly (d) Snail (a) adequate light
Ans: (b) (b) supply of oxygen
Q188. Which organ is the alternate term for womb? (c) suitable moisture
(a) Uterus (b) Ureter (d) suitable temperature
(c) Vagina (d) Vulva Ans: (a)
Ans: (a) Q198. An invertebrate does not have
Q189. Of which tissue nails, hoofs and horns are made (a) Notochord (b) Body cavity
of (c) Haemocoel (d) Gills
(a) Cutide (b) Chitin Ans: (a)
(c) Keratin (d) Tunicin Q199. Excess amount of absorbed water by plants is
Ans: (c) liberated out by :
Q190. Which one of the following glands is (a) Evaporation
responsible for secretion of sex hormones ? (b) Osmosis
(a) Adrenal gland (c) Diffusion
(b) Thyroid gland (d) Transpiration
(c) Pituitary gland Ans: (d)
(d) Sebaceous gland Q200. Bio - diesel is mostly produced by :
Ans: (c) (a) Myrtaceae (b) Malvaceae
Q191. The chemical which is used in art and craft and (c) Liliaceae
can cause anemia and leukemia is : (d) Euphorbiaceae
(a) Benzene (b) Dioxine Ans: (d)
(c) Phthalate (d) Aldrin Q201. The only plant cells without nuclei among the
Ans: (a) following are :
Q192. In onion the edible part is (a) Cambium cells
(a) Leaf (b) Root (b) Root hairs
(c) Stem (d) Flower (c) Companion cells
Ans: (a) (d) Tracheid cells
Q193. Haemoglobin has highest affinity for Ans: (a)
(a) Oxygen Q202. Which of the following item is not included in
(b) Carbon dioxide Environmental Auditing ?
(c) Carbon monoxide (a) Pollution monitoring schemes
(d) Nitrogen (b) Scrutiny by the government agencies
Ans: (a) (c) Safety provisions for industrial workers
Q194. Mercury poisoning in man causes the disease (d) Storage of toxic chemicals
(a) Black lung Ans: (b)
(b) Arsenicosis Q203. Symptoms of Jaundice occur mainly due to
(c) Minamata disorder and malfunction of :
(d) Tai-Etai (a) Intestine (b) Liver
Ans: (c) (c) Stomach (d) Pancreas
Q195. The main use of salt in the diet is to Ans: (b)
(a) increase the solubility of the food particles in water Q204. The larvae of cockroach is called
(b) produce in small amounts the hydrochloric acid (a) Caterpillar (b) Nymph
required for the digestion of food (c) Maggot (d) Grub
(c) ease the process of cooking Ans: (b)
(d) make the taste of the food better Q205. Which of the following amphibians lacks
Ans: (b) tongue ?
Q196. Blue-green algae are included in the group (a) Sphenodon (b) Salamander
(a) Eubacteria (b) Cyanobacteria (c) Ichthyophis (d) Necturus
Page 418 of 516
https://telegram.me/aedahamlibra
Ans: (c)
Q206. Nutrients are recycled in the atmosphere with
(a) Diesel
(c) Hydrogen
(b) Kerosene
(d) Coal
the help of certain micro-organisms referred to as Ans: (c)
(a) producers Q215. Vegetables and fruits should be a part of our
(b) consumers diet because they stimulate
(c) decomposers (a) Peristalsis (b) Salivation
(d) None of these (c) Excretion (d) Respiration
Ans: (c) Ans: (a)
Q207. A cross between homozygous recessive and Q216. Which protein – sugar pair is present in milk?
heterozygous plant is (a) Casein, Sucrose
(a) Back cross (b) Casein, Lactose
(b) Test cross (c) Ferritin, Maltose
(c) Monohybrid cross (d) Albumin, Glucose
(d) Dihybrid cross Ans: (b)
Ans: (b) Q217. Bryophytes are often called as amphibian plant
Q208. Iodine solution is used to test the presence of because they :
(a) Sugar (b) Proteins (a) appear like frog
(c) Starch (d) Fats (b) are found both in water and on land
Ans: (c) (c) do not have habitat preference
Q209. The largest invertebrate is a/an (d) can eat insects
(a) Sponge (b) Mollusc Ans: (b)
(c) Arthropod (d) Echinoderm Q218. Crude oil is sometimes termed sweet because it
Ans: (b) is :
Q210. Growing more than one crop on a piece of land (a) sweet in taste due to dissolved sugars
during the year is known as (b) mildly sweet due to low sulphur content
(a) Uni cropping (c) less acidic
(b) Multiple cropping (d) less alkaline
(c) Double cropping Ans: (b)
(d) Triple cropping Q219. Contraceptive pills in the market contain
Ans: (b) (a) Inorganic compounds
Q211. Filaria is transmitted by (b) Herbicides (c) Antibiotics
(a) Aedes mosquito (d) Steroid-hormones
(b) Anopheles mosquito Ans: (d)
(c) Culex mosquito Q220. ‗White Revolution‘ is related to
(d) Swamp mosquito (a) Wheat production
Ans: (c) (b) Milk production
Q212. Green manure is obtained from (c) Flood control
(a) Fresh animal excreta (d) Fish production
(b) Decomposing green legume plants Ans: (b)
(c) Domestic vegetable waste Q221. Which of the following atmospheric gases
(d) Oil seed husk cakes constitute greenhouse gases?
Ans: (b) a. Carbon dioxide
Q213. In our body, antibodies are formed against b. Nitrogen
pathogens in c. Nitrous oxide
(a) liver by RBC d. Water vapour Select the correct answer using the codes
(b) blood by platelets given below :
(c) brain by macrophages (a) a, c and d (b) a and d
(d) thymus by lymphocytes (c) a and c (d) a, b and d
Ans: (d) Ans: (a)
Q214. Which of the follow ing fuels causes minimum Q222. Bio-magnification implies
environmental pollution ? (a) Toxic matters are magnified
Page 419 of 516
(b) Living beings are magnified
(c) Light is magnified
https://telegram.me/aedahamlibra
Q232. The animal who can consume more salt among
the following is
(d) Food is magnified (a) Sheep (b) Camel
Ans: (a) (c) Donkey (d) Dog
Q223. The sweet taste of fruits is due to Ans: (b)
(a) Lactose (b) Fructose Q233. DPT vaccine is administered to prevent
(c) Maltose (d) Ribose diseases like:
Ans: (b) (a) Diphtheria, Pertussis and Tetanus
Q224. Kala-azar is transmitted by (b) Dengue, Pertussis and Typhoid
(a) tsetse fly (c) Dengue, Polio and Tetanus
(b) housefly (d) Diphtheria, Pertussis and Typhoid
(c) anopheles mosquito Ans: (a)
(d) sand fly Q234. The natural disaster in which carbon -di-oxide
Ans: (d) suddenly erupts from a deep lake water is known as
Q225. Who among the following have venous heart? _________.
(a) Mammals (b) Reptilians (a) Lacaustrine (b) Fluvial
(c) Fishes (d) Amphibians (c) Glacial (d) Liminic
Ans: (c) Ans: (d)
Q226. Protein part of enzyme is know as Q235. The process of photo synthesis involves
(a) Isoenzyme conversion of
(b) Holoenzyme (a) chemical energy into radiant energy
(c) Apoenzyme (b) chemical energy into mechanical energy
(d) All the above (c) solar energy into chemical energy
Ans: (c) (d) mechanical energy into solar energy
Q227. An example of protein which acts as a hormone Ans: (c)
is : Q236. Of the following man -made disasters, which is
(a) Trypsin (b) Oxytocin socially induced ?
(c) Keratin (d) Casein (a) Debris Avalanche
Ans: (b) (b) Salt Water Intrusion
Q228. Malaria is transmitted from one person to (c) Arson
another by: (d) Ozone depletion
(a) Aedes Mosquito Ans: (c)
(b) Culex Mosquito Q237. The fasting blood glucose level in adults in
(c) Anopheles Mosquito mg/100 ml is
(d) All of the above (a) 200 (b) 160
Ans: (c) (c) 100 (d) 60
Q229. The animal that do not develop hypertension Ans: (c)
inspite of heavy intake of salt is: Q238. Proteins are digested by
(a) Sheep (b) Buffalo (a) Proteases (b) Amylases
(c) Tiger (d) Camel (c) Lipases (d) Nucleases
Ans: (d) Ans: (a)
Q230. Transcription means the synthesis of Q239. Tendons and ligaments are
(a) Lipids (b) Protein (a) Connective tissue
(c) DNA (d) RNA (b) Muscular tissue
Ans: (c) (c) Epithelial tissue
Q231. The outer skin most of the crustaceans are (d) Skeletal tissue
made up of a carbohydrate. This carbohydrate is Ans: (a)
(a) cellulose (b) galactose Q240. A molecule in plants comparable to
(c) chitin (d) starch haemoglobin in animals is
Ans: (c) (a) Cytochrome (b) Cellulose
(c) Chlorophyll (d) Carotene
Page 420 of 516
https://telegram.me/aedahamlibra
Q241. How many chambers does a mammalian heart
Ans: (c) Q251. Lactogenic hormone is secreted by
(a) Mammary glands
have ? (b) Placenta
(a) 4 (b) 1 (c) Ovary
(c) 2 (d) 3 (d) Pituitary
Ans: (a) Ans: (d)
Q242. Respiration is regarded as a Q252. Match correctly the insect vectors in List I with
(a) Synthetic Process the diseases transmitted by them given in List II:
(b) Catabolic Process List I
(c) Anabolic Process a. Anopheles (female)
(d) Reduction Process b. Culex
Ans: (b) c. Sand fly
Q243. Conglobate gland is a leaf like structure found d. Tse-tse fly List II
in 1. Kala-azar
(a) Female Cockroach 2. Sleeping sickness
(b) Male Cockroach 3. Filariasis
(c) Male Ascaris 4. Malaria
(d) Female Ascaris (a) a-1, b-4, c-2, d-3
Ans: (b) (b) a-2, b-l, c-4, d-3
Q244. The type of tail found in Shark is (c) a-3, b-2, c-1, d-4
(a) Protocercal (b) Homocercal (d) a-4, b-3, c-l. d-2
(c) Heterocercal (d) Diphycercal Ans: (d)
Ans: (c) Q253. The non -green heterotrophic plants of plant
Q245. AIDS virus destroys kingdom are
(a) Lymphocytes (a) mosses (b) ferns
(b) Monocytes (c) algae (d) fungi
(c) Neutrophils Ans: (d)
(d) Basophils Q254. In any spreadsheet, the address of the first cell
Ans: (a) is
Q246. Haptens are: (a) 0A (b) 1A
(a) Pseudoantigens (c) A0 (d) Al
(b) Incomplete antigens Ans: (d)
(c) Antibodies Q255. The endangered species are listed in what
(d) Isoantigens colour data book?
Ans: (b) (a) Black (b) Red
Q247. Panda belongs to the same family as that (c) Green (d) Blue
(a) Kangaroo (b) Porcupine Ans: (b)
(c) Whale (d) Bear Q256. Flowers emit fragrance to :
Ans: (d) (a) purify air
Q248. Yeast is an important source of (b) drive away flies
(a) vitamin B (b) invertase (c) attract insects
(c) vitamin C (d) protein (d) perform all the above
Ans: (a) Ans: (c)
Q249. The largest White Blood Corpuscle is Q257. Animals with Chitinous exoskeleton are
(a) Lymphocyte (b) Monocyte (a) Insects (b) Sponges
(c) Thrombocyte (d) Erythrocyte (c) Sea-Urchin (d) Snails
Ans: (b) Ans: (a)
Q250. Rinderpest disease of Cattle is caused by Q258. Bone osification test is conducted to as certain
(a) Insects (b) Bacteria human
(c) Virus (d) Protozoa (a) Brain efficiency
Ans: (c) (b) Tentative age
Page 421 of 516
(c) Tentative height
(d) Drug addiction
https://telegram.me/aedahamlibra
(a) Dicot plants
(b) Monocot plants
Ans: (b) (c) Both the above
Q259. In a poultry unit, the factor most influencing (d) Algae
the cost is the cost of Ans: (a)
(a) feed Q268. Virus in Latin means
(b) transportation (a) Sweet (b) Small
(c) chicks (c) Fluid (d) Poison
(d) medicines Ans: (d)
Ans: (a) Q269. Intercalary meristems are found in
Q260. Clitellum is found in (a) Node
(a) Leech (b) Snail (b) Lateral bud
(c) Nereis (d) Earthworm (c) Terminal bud
Ans: (d) (d) Inter node
Q261. Stamens are fused with e ach other by their Ans: (d)
anthers and also with the petals in : Q270. Green blocks are referred to
(a) Leguminosae (a) Green cover
(b) Liliaceae (b) Green Ministry
(c) Compositae (c) Bio-bricks
(d) Euphorbiaceae (d) Pro-biotic curd
Ans: (c) Ans: (c)
Q262. Crossing-over occurs during Q271. Thalassemia is an example of
(a) Leptotene (b) Zygotene (a) Deletion mutation
(c) Pachytene (d)Diplotene (b) Point mutation
Ans: (c) (c) Silent mutation
Q263. Endoscope is an instrum ent used to detect the (d) Frame shift mutation
ulcers in the stomach has a long narrow tube Ans: (d)
(with a small glowing bulb at one end) which is inserted Q272. The biggest single-celled organism is
in to the stomach through the mouth contains. (a) Yeast
(a) small current carrying wire (b) Acetabularia
(b) a narrow tube containing water (c) Acetobacter
(c) optical fibre (d) Amoeba
(d) a narrow tube containing some chemical solution Ans: (d)
Ans: (c) Q273. One of the best solutions to get rid of non -
Q264. The pancreas secretes biodegradable wastes is
(a) Insulin (a) Burning (b) Dumping
(b) Bile juice (c) Burying (d) Recycling
(c) Peptic juice Ans: (d)
(d) None of these Q274. Leaving agricultural land uncultivated for
Ans: (a) some years known as
Q265. Pinna (external ear) is present in (a) Intensive farming
(a) Amphibian (b) Fish (b) Fallowing
(c) Mammal (d) Reptile (c) Shifting cultivation
Ans: (c) (d) Subsistence farming
Q266. Translocation of water is Ans: (b)
(a) Apoplastic Q275. What is ‗Biodiversity‘ ?
(b) Symplastic (a) Many types of flora & fauna in one forest
(c) Both (a) and (b) (b) Many types of flora and fauna in many forests
(d) None of the above (c) Many population of one species in one forest
Ans: (c) (d) All the above are true.
Q267. The kidney shaped guard cells are present in Ans: (a)
Page 422 of 516
https://telegram.me/aedahamlibra
Q276. The special modified epidermal cells
surrounding stomatal pore are called
(a) Darwin
(c) Mendel (d)Weismann
(b) Lamarck

(a) Epithelial cells Ans: (a)


(b) Guard cells Q286. Typhoid fever is caused by
(c) Subsidiary cells (a) Bacteria (b) Virus
(d) Accessory cells (c) Protozoa (d) Fungi
Ans: (b) Ans: (a)
Q277. Pleura is the covering of Q287. How many neck canal cells are found in the
(a) Lung (b) Liver archegonium of a fern ?
(c) Kidney (d) Heart (a) One (b) Two
Ans: (a) (c) Three (d) Four
Q278. Process through which plants reproduce Ans: (a)
(a) Pollination Q288. HIV often changes its shape due to the presence
(b) Condensation of an enzyme called
(c) Eating (a) Reverse Transcriptase
(d) Evaporation (b) Enterokinase
Ans: (a) (c) Nucleotidase
Q279. Bulbils takes part in (d) Nucleoditase
(a) Sexual reproduction Ans: (a)
(b) Vegetative reproduction Q289. Which of the following listed is not a feature of
(c) Food storage organic farming ?
(d) Respiration (a) The non-use of chemical fertilizers and pesticides
Ans: (b) (b) Soil is nurtured for furture use by maintaining micro -
Q280. Root Hairs arise from organisms
(a) Cortex (b) Pericycle (c) Use of synthetic fertilizers
(c) Epidermis (d) Endodermis (d) Very less energy consumption
Ans: (c) Ans: (c)
Q281. Green manure is obtained from Q290. In human body, ligaments are made up of
(a) Domestic vegetable waste (a) white fibres and some yellow elastic fibres
(b) Oil seed husk cakes (b) white fibres only
(c) Fresh animal excreta (c) yellow fibres only
(d) Decomposing green legume plants (d) yellow fibres and muscle fibres
Ans: (d) Ans: (a)
Q282. From which part ofOpium plant we get Q291. ‗Coralloid root‘ of Cycas helps in
morphine ? (a) Absorption of water
(a) Leaves (b) Stem (b) Absorption of water and fixation of Nitrogen
(c) Bark (d) Fruit coat (c) Anchorage
Ans: (d) (d) Transport of food
Q283. The smallest known prokaryotic organism is Ans: (b)
(a) Microcystis Q292. What does the word ‗amphibian‘ means ?
(b) Mycoplasma (a) Three lives (b) Four lives
(c) Bacteria (c) Two lives (d) One lives
(d) Chlorella Ans: (c)
Ans: (b) Q293. Which of the following is used in the treatment
Q284. The largest cells in mammalian blood are of cancer ?
(a) Erythrocytes (a) Physiotherapy
(b) Monocytes (b) Chemotherapy
(c) Basophils (c) Electrotherapy
(d) Lymphocytes (d) Psychotherapy
Ans: (b) Ans: (b)
Q285. ‗Survival of the fittest‘ was coined by Q294. What does the word ‗amphibian‘ means ?
Page 423 of 516
(a) Three lives
(c) Two lives
https://telegram.me/aedahamlibra
(b) Four lives
(d) One lives
Q303. The best milch breed in the word is :
(a) Chittagong (b) Sindhi
Ans: (c) (c) Deoni
Q295. Which of the following is used in the treatment (d) Holstein–Friesian
of cancer ? Ans: (d)
(a) Physiotherapy Q304. B–Diversity is also known as :
(b) Chemotherapy (a) Within habitat diversity
(c) Electrotherapy (b) Ecosystem diversity
(d) Psychotherapy (c) Global diversity
Ans: (b) (d) Between habitat diversity
Q296. Which of the following fuels causes minimum Ans: (b)
environmental pollution ? Q305. Addition oxcessive amounts of heat to a lake is
(a) Kerosene (b) Diesel referred to as :
(c) Coal (d) Hydrogen (a) Refrigeration effect
Ans: (d) (b) Green House effect
Q297. With which one of the following Jacobsons (c) Thermal pollution
organ is related to ? (d) Heat Bloom
(a) Vision (b) Chewing Ans: (c)
(c) Smelling (d) Hearing Q306. Linseed is obtained from
Ans: (c) (a) Castor (b) Flax
Q298. The maximum fixation of solar energy is done (c) Groundnut (d) Sesame
by Ans: (b)
(a) Protozoa (b) Bacteria Q307. Oxyntic cell is meant for the secretion of
(c) Fungi (d) Green plants (a) Pepsin
Ans: (d) (b) Enterokinase
Q299. Animal protein is called first class protein (c) Hydrochloric acid
because it is (d) Lactic acid
(a) easily digestible Ans: (c)
(b) delicious in taste Q308. Point out the incorrect pair :
(c) cheaper in the market (a) Green Revolution – Agricultural Development
(d) rich in essential amino acids (b) White Revolution – Dairy Development
Ans: (d) (c) Blue Revolution – Development of Fisheries
Q300. Hydrogen peroxide is an effective sterilizing (d) Operation Flood – Irrigation Development
agent. Which one o f the following product results Ans: (d)
when it readily loses active oxygen? Q309. Soilless agriculture refers to
(a) Nascent Hydrogen (a) Hydroponics
(b) Water (b) Hygroponics
(c) Hydrogen (c) Sericulture
(d) Ozone (d) Inter–cropping
Ans: (b) Ans: (a)
Q301. Molybdenum deficiency affects the activity of : Q310. Life originated by chemosynthesis was proved
(a) All of the given options in the laboratory by :
(b) Chlorate reductase (a) Sanger (b) Pasteur
(c) Nitrogenase (c) Miller (d) Aristotle
(d) Nitrate reductase Ans: (c)
Ans: (d) Q311. Excretion in Hemichordates takes place by
Q302. Which of the following plant shows chloroplast (a) Glomerulus (b) Pronephron
dimorphism? (c) Mesonephron (d) Metanephron
(a) Sugar beet (b) Rice Ans: (a)
(c) Wheat (d) Sugarcane Q312. The use of heat treatment of ore that includes
Ans: (d) smelting and roasting is termed as :
Page 424 of 516
(a) Electrometallurgy
(b) Hydrometallurgy
https://telegram.me/aedahamlibra
Q321. Which one of the following substances is
Ans: (c)

(c) Pyrometallurgy normally found in urine?


(d) Cryometallurgy (a) blood proteins
Ans: (c) (b) creatinine
Q313. Christmas factor is involved in : (c) red blood cells
(a) Blood Coagulation (d) white blood cells
(b) Excretion Ans: (b)
(c) Digestion Q322. Intensive cultivation refers to
(d) Respiration (a) production with intensive use of labour
Ans: (a) (b) production with intensive use of fertilizer
Q314. Commercially valued cork is obtained from : (c) raising production by intensive use of existing land
(a) Quercus spp (d) raising production by large scale use of imported
(b) Cedrus Deodara inputs
(c) Ficus Ans: (c)
(d) Cycas Q323. In water treat ment plant, use of chloramines
Ans: (a) ensures _____
Q315. The Ozone layer protects us from: (a) taste and odour control
(a) Cosmic rays (b) weed control in reservoirs
(b) Ultra-Violet rays (c) disinfection
(c) Visible rays (d) removal of permanent hardness
(d) Infrared rays Ans: (c)
Ans: (b) Q324. What are Lipids?
Q316. Chromosome designation of Turner sydrome is (a) Lipids are monosaccharides
: (b) Lipids do not provide energy to cells
(a) 44A+XO (b) 44A+XXX (c) Fruits are a good source of lipids
(c) 44A+XXY (d) 44A+XYY (d) Cholesterol and trans fatty acids are types of Lipids
Ans: (a) Ans: (d)
Q317. Which of the foll owing is true with reference to Q325. The parts of human body affected by Pyria are:
blood platelets? (a) eyes
(a) They have prominent nuclei. (b) small intestine
(b) They are involved in phagocytosis (c) teeth and gums
(c) They have a pigment called haemoglobin (d) large intestine
(d) They are also called thrombocytes. Ans: (c)
Ans: (d) Q326. Which of t he following procedure treats spinal
Q318. Which part of the cinchona yields a drug? stenosis (stricture) ?
(a) Pericarp (b) Bark (a) Laminoplasty
(c) Endosperm (d) Leaf (b) Corpectomy
Ans: (b) (c) Foraminotomy
Q319. What is the famous ‗Chipko‘ movement (d) All of the above
associated with? Ans: (d)
(a) Saving the tigers Q327. Which one of the following is also called as milk
(b) Saving the wetland sugar ?
(c) None of these (a) Glucose (b) Fructose
(d) Trees (c) Maltose (d) Lactose
Ans: (d) Ans: (d)
Q320. The highest concentration of urea is found in Q328. Which cell disorder in our body is responsible
(a) Hepatic portal vein for colour blindness?
(b) Dorsal aorta (a) WBC (b) Cone cell
(c) Hepatic vein (c) Red Cell (d) Neuron
(d) Renal vein Ans: (b)
Page 425 of 516
https://telegram.me/aedahamlibra
Q329. Hepatitis affects which organ of the human
body?
(a) Dermatogen
(c) Protoderm
(b) Calyptrogen
(d) Histogen
(a) Liver (b) Pancreas Ans: (b)
(c) Spleen (d)Small intestine Q338. In bio fortification technique plant breeders use
Ans: (a) breeding to overcome
Q330. Inhibition of photosynthesis in the presence of (a) Loss due to insect pests
O2 in C3 plants is called : (b) Decrease in food production
(a) Hexose monophosphate shunt (c) Deficiencies of micronutrients and vitamins
(b) Pasteur effect (d) Loss due to plant diseases
(c) Decker effect Ans: (c)
(d) Warburg effect Q339. Rio Summit is associated with
Ans: (d) (a) Convention on Biological Diversity
Q331. Which is the largest blood vessel in human (b) Green house gases
body? (c) Ozone depletion
(a) Aorta (d) Wet lands
(b) Anatomises Ans: (a)
(c) Tunica Intima Q340. The substrate of photorespiration is
(d) Atrium (a) Fructose (b) Pyruvic acid
Ans: (a) (c) Glycolate (d) Glucose
Q332. The name of the longest muscle in human body Ans: (c)
is Q341. Kyoto Protocol is associated with
(a) Sartorius muscle (a) Species conservation
(b) Massete muscle (b) Climate change
(c) Stapedius muscle (c) Wetland Conservation
(d) Gluteus Maximus muscle (d) Medicinal plants
Ans: (a) Ans: (b)
Q333. What do you call the study of fungi? Q342. Honey that has high concentration of sugar
(a) Mycology (b) Parasitology does not decay because
(c) Bacteriology (d) Ecology (a) Bacteria cannot survive in an active state as it is
Ans: (a) totally deprived of oxygen
Q334. The most serious air pollutant causing health (b) It contains natural antioxidant that prevents bacterial
hazard is attack
(a) Sulphur dioxide (c) Bacteria cannot survive in an active state in a solution
(b) Carbon monoxide of high osmotic strength as water is drawn out
(c) Ozone (d) None of these
(d) Nitrogen oxide Ans: (c)
Ans: (a) Q343. Gustation refers to the sense of which of the
Q335. Why is Carbon Monoxide a pollutant ? following ?
(a) Reacts with haemoglobin (a) Smell (b) Hearing
(b) Makes nervous system inactive (c) Tactile (d) Taste
(c) It reacts with Oxygen Ans: (d)
(d) It inhibits glycolysis Q344. Potato is a
Ans: (a) (a) Root (b) Stem
Q336. Which lobe of human brain is associated with (c) Bud (d) Fruit
hearing ? Ans: (b)
(a) Frontal lobe Q345. The suicidal bags of the cell are
(b) Parietal lobe (a) Lysosomes
(c) Temporal lobe (b) Ribosomes
(d) Occipital lobe (c) Dictyosomes
Ans: (c) (d) Phagosomes
Q337. Root cap is derived from Ans: (a)
Page 426 of 516
https://telegram.me/aedahamlibra
Q346. Which one of the following events in a botanical
garden is never directly influenced by light ?
Q354. The first protocol to ban the emissions of
choloroflurocarbons in the atmosphere was made in
(a) Flowering (a) Montreal (b) Osaka
(b) Photosynthesis (c) Geneva (d) Florida
(c) Transpiration Ans: (a)
(d) Fertilization Q355. The food in Onion is stored in the form of
Ans: (d) _______
Q347. Which organism is responsible for alcohol (a) Cellulose (b) Protein
fermentation? (c) Starch (d) Sugar
(a) Chlorella (b) Yeast Ans: (d)
(c) Agaricus (d) Puccinia Q356. Which of the f ollowing green house gases has
Ans: (b) the greatest heattrapping ability?
Q348. Which one of the following weeds is effective in (a) Chlorofluoro carbon
controlling water pollution caused by industrial (b) Methane
effluents? (c) Carbon dioxide
(a) Parthenium (d) Nitrous oxide
(b) Elephant grass Ans: (a)
(c) Water hycinth Q357. Which organ of human body secretes Insulin?
(d) Mogar grass (a) Pancreas (b) Kidney
Ans: (c) (c) Gall bladder (d) Liver
Q349. Male mosquitoes take their food from Ans: (a)
(a) human blood Q358. Which of the following is not connective tissue?
(b) standing water (a) Bone (b) Cartilage
(c) sap of plants (c) Blood (d) Skeletal muscle
(d) dung and debris Ans: (d)
Ans: (c) Q359. Which of the following vitamins contain
Q350. What is the main purpose of white blood nitrogen?
corpuscles? (a) Vitamin A (b) Vitamin B
(a) to carry nutrients (c) Vitamin C (d) Vitamin D
(b) to combat infection Ans: (b)
(c) to carry oxygen Q360. Which is used as an Air pollution indicator?
(d) to give strength (a) Algae (b) Fungi
Ans: (b) (c) Bacteria (d) Lichens
Q351. Which of the following is the treatment of Ans: (d)
water pollution? Q361. The elements known as primary nutrients for
(a) Bag house filter plants
(b) Windrow composting (a) Nitrogen, Phosphorus and Potassium
(c) Venturi scrubber (b) Nitrogen, Oxygen and Silicon
(d) Reverse Osmosis (c) Potassium, Boron and Nitrogen
Ans: (d) (d) Nitrogen, Phosphorus and Iron
Q352. The xylem in plants are responsible for : Ans: (a)
(a) transport of water Q362. Which of the following increases the rate of
(b) transport of food heart beat?
(c) transport of amino acids (a) Peripheral nerves
(d) transport of oxygen (b) Sympathetic nerves
Ans: (a) (c) Parasympathetic nerves
Q353. ‖Alfalfa‖ is the name of a (d) Cranial nerves
(a) Mineral (b) Tribe Ans: (b)
(c) Grass (d) Town Q363. Which of the following is a Dominant
Ans: (c) Autosomal Disorder?
(a) Albinism
Page 427 of 516
(b) Cystic Fibrosis
(c) Phenyl Ketorunia
https://telegram.me/aedahamlibra
Computer & IT
(d) Alzheimer‘s Disease
Ans: (d)
Q1. Which of the following is not a role assigned to
Q364. The maximum fixation of solar energy is done
Science and Technology Entrepreneurship Parks ?
by
(a) Training
(a) Bacteria (b) Protozoa
(b) Testing and Calibration
(c) Fungi (d) Green plants
(c) Technology development
Ans: (d)
(d) Raising funds for entre-preneurs
Q365. Among the following which on e lays eggs and
Ans: (d)
does not produce young ones directly?
Q2. What is an organisation‘s introductory web page
(a) Echidna (b) Kangaroo
called?
(c) Porcupine (d) Whale
(a) Portal (b) Vortal
Ans: (a)
(c) Homepage (d) Web site
Q366. Which one of the following is not the excretory
Ans: (d)
organ?
Q3. What is ‗Teletext‘ ?
(a) Kidneys (b) Liver
(a) Flashing of telephone conversation on TV screen.
(c) Lungs (d) Spleen
(b) Printing the text of the message on the telex machine.
Ans: (d)
(c) Connecting the local telephone with outstation
Q367. The presence of air b ubble in blood stream is
telephones without STD codes.
dangerous to life because
(d) Flashing of the text of news and information on the
(a) Air combines with blood forming a complex
TV screen.
(b) The flow of blood increases manifold
Ans: (d)
(c) The flow of blood is obstructed
Q4. Who developed the concept ‗World Wide Web‘ ?
(d) The pressure of blood increases manifold
(a) Tim Berners-Lee
Ans: (c)
(b) Microsoft
Q368. The sigmoid colon is a part of
(c) Arthur Clarke
(a) Small Intestine
(d) AT & T Bell Lab
(b) Large Intestine
Ans: (b)
(c) Pharynx
Q5. Which one of the following companies announced
(d) Rectum
sometime back the launch of India‘s first personal
Ans: (b)
computer with one terabyte hard drive capacity?
Q369. Which of the following gas leaked in the Bhopal
(a) Infosys Technologies
Gas tragedy in December 1984?
(b) Wipro
(a) Methyl isocyanate
(c) HCL Infosystems
(b) Methyl isochlorate
(d) IBM
(c) Methyl phosphate
Ans: (c)
(d) Methyl isopropate
Q6. ‗C‘ language is a
Ans: (a)
(a) Low level language
Q370. Which one of the following is a carbohydrate ?
(b) High level language
(a) Urea (b) Insulin
(c) Machine level language
(c) Glycine (d) Glucose
(d) Assembly level language
Ans: (d)
Ans: (b)
Q371. Riboflavin is a/an
Q7. What is clickjacking ?
(a) Vitamin (b) Hormone
(a) Malicious technique of tricking Web usear into
(c) Fatty acid (d) Enzyme
revealing confidential information
Ans: (a)
(b) A device that sends and receives data in a bit second
(c) A form of Computer Engineering
(d) A digital process that is used to display an image on
monitor

Page 428 of 516


https://telegram.me/aedahamlibra
Ans: (a)
Q8. In which of the following areas, a spreadsheet
(c) 1 MB (d) 1 KB
Ans: (d)
software is more useful ? Q17. The first operational electronic digital computer
(a) Psychology (b) Publishing is
(c) Statistics (a) ENIAC (b) EDVAC
(d) Message sending (c) EDSAC (d) UNIVAC
Ans: (c) Ans: (a)
Q9. WIKILEAKS, a whistleblowers website is an Q18. Fifth generation computers do not have :
international organisation based in (a) Speech recognition
(a) U.S.A. (b) U.K. (b) Artificial intelligence
(c) Sweden (d) Norway (c) Very large scale integration
Ans: (a) (d) Vacuum tubes
Q10. Where are programs and data to be used by the Ans: (d)
computer available? Q19. Most of the computers available today are :
(a) Processing Unit (a) 3rd generation computers
(b) Output (b) 4th generation computers
(c) Storage (c) 5th generation computers
(d) Input (d) 6th generation computers
Ans: (c) Ans: (b)
Q11. Who introduced ―MMX‖ technology? Q20. A ‗bug‘ in a programme is a
(a) IBM (b) Apple (a) Statement
(c) Microsoft (d) Intel (b) Error
Ans: (d) (c) Signature
Q12. If you undergo a CAT scan then you have been (d) both (b) and (c)
subjected to Ans: (b)
(a) a computer aided test Q21. What is the process of defining tables called ?
(b) a computerised axial topography (a) Data definition
(c) a computer aided tomography (b) Data Normalisation
(d) a computerised axial tomography (c) Index definition
Ans: (d) (d) Data administration
Q13. A technique for producing animation in which Ans: (c)
one image changes into another is called Q22. What is the process of defining tables called ?
(a) Embedding (a) Data definition
(b) Linking (b) Data Normalisation
(c) Morphing (d) Scanning (c) Index definition
Ans: (c) (d) Data administration
Q14. The name ―modem‖ is derived from Ans: (c)
(a) modern demarkator Q23. A ‗Bit‘ refers to
(b) modulator demand (a) Binary informatics
(c) modern demodulator (b) Bilingual information
(d) modulator demodulator (c) Binary terminator
Ans: (d) (d) Binary digit
Q15. The period of the Second Generation Computers Ans: (d)
was Q24. The term API refers to
(a) 1946 – 1958 (a) Application Program Interface
(b) 1940 – 1960 (b) Application Program Interaction
(c) 1959 – 1964 (c) Applied Program Interaction
(d) 1965 – 1975 (d) Application Process Interface
Ans: (c) Ans: (a)
Q16. 1024 bytes equals : Q25. Telnet is a :
(a) 1 TB (b) 1 GB (a) browser (b) protocol
Page 429 of 516
(c) gateway https://telegram.me/aedahamlibra
(d) search engine
Ans: (b)
(c) Only Roman system
(d) Hexadecimal system
Q26. Where is India‘s super computer ‗PARAM‘ Ans: (a)
located ? Q36. A col lection of HTML pages makes up
(a) Chennai (b) Pune the_______.
(c) Bengaluru (d) Kolkata (a) Hyperlinks
Ans: (b) (b) Hypertext
Q27. A bug in a programme is a/an (c) World Wide Web
(a) Statement (b) Error (d) Hypermedia
(c) Syntax Ans: (c)
(d) Both (b) and (c) Q37. Identify the LIFO (Last In First Out) structure
Ans: (b) among the following:
Q28. A ____ is an input device, which is used for (a) Stack (b) Queue
converting pictures, maps and drawings into digital (c) De-queue (d) Array
form for storage in computers. Ans: (a)
(a) OCR (b) OMR Q38. The telephone line is connected through the
(c) MICR (d) digitizer ____on the computer.
Ans: (d) (a) USB (b) Modem
Q29. The process of writing on an optical disc is called (c) Ethernet (d) PS2
(a) Ripping (b) Fetching Ans: (b)
(c) Scanning (d) Burning Q39. The waste generated by end life personal
Ans: (d) computers is known as
Q30. An attribute that uniquely identifies the rows of (a) PC-waste
a table is known as the ....... key. (b) Physical waste
(a) primary (b) candidate (c) Computer waste
(c) composite (d) foreign (d) E-waste
Ans: (a) Ans: (d)
Q31. In MS -Excel, ______ provide a visual Q40. A portable, personal computer small enough to
representation of the values in a worksheet. fit on your lap is called a
(a) Charts (b) Formulae (a) Note-book computer
(c) Templates (d) Views (b) PDA
Ans: (a) (c) Mainframe computer
Q32. The most crucial portion of any computer is the (d) Workstation
_____. Ans: (a)
(a) I/O Unit (b) Hard Disk Q41. A collection of pictures that can be inserted into
(c) CPU (d) Memory documents is called :
Ans: (c) (a) Photo Shop
Q33. The alignment of beginning and the end of lines (b) Auto Shapes
is referred to as (c) Word Art
(a) Indentation (b) Justification (d) Clip Art
(c) Superscript (d) Font Ans: (d)
Ans: (b) Q42. One kilobit is equal to_______ bits
Q34. What MS -DOS command is used to create a (a) 512 (b) 1000
subdirectory ? (c) 1024 (d) 1042
(a) DIR/MK (b) MKDIR Ans: (c)
(c) CHDIR (d) RMDIR Q43. A Supercomputer would be used for which one
Ans: (b) of the following applications ?
Q35. What type of information system would be (a) Business Computing
recognised by digital circuits ? (b) Desktop Publishing
(a) Binary system (c) Weather Forecasting
(b) Both hexadecimal and binary system (d) Computer Aided Designing
Page 430 of 516
Q44. The ‗IBM-DOS‘ is a
https://telegram.me/aedahamlibra
Ans: (c) Q53. The father of computer is
(a) Oliver Twist
(a) Single user operating system (b) Charles Dickens
(b) Multiuser operating system (c) Charles Babbage
(c) Batch operating system (d) Love Lice
(d) Time-sharing operating system Ans: (c)
Ans: (a) Q54. A computer executes programs in the sequence
Q45. The term used to describe a person who spends of :
plenty of time online exploring the Internet is (a) Execute, Fetch, Decode
(a) Netaddict (b) Netguru (b) Store, Fetch, Execute
(c) Cybernaut (d) Cyberphile (c) Fetch, Decode, Execute
Ans: (a) (d) Decode, Fetch, Execute
Q46. Which one of the fol lowing converts assembly Ans: (c)
language into machine language ? Q55. After ‗PROM‘ is programmed, we can only ____
(a) Algorithm (b) Interpreter the information.
(c) Compiler (d) Assembler (a) read (b) write
Ans: (d) (c) read and write (d) remove
Q47. The most advanced form Of Read Only Memory Ans: (a)
(ROM) is Q56. Which one of the following is odd?
(a) PROM (a) IMAP (b) POP
(b) RAM (c) SNMP (d) SMTP
(c) Cache Memory Ans: (c)
(d) EEPROM Q57. The Simplest CPU – scheduling algorithm is
Ans: (d) ______.
Q48. The process of combining strings is known as (a) SJF scheduling algorithm
(a) compiling (b) Combining (b) Round robin scheduling algorith
(c) Attaching (d) Concatenation (c) Multilevel scheduling algorithm
Ans: (d) (d) FCFS scheduling algorithm
Q49. What is the full form of ALU ? Ans: (d)
(a) Alternative Logic Unit Q58. In operationg system, Round Robin Scheduling
(b) Arithmetic Logic Unit means :
(c) Arithmetic Least Unit (a) A kind of scheduling
(d) Arithmetic Local Unit (b) Repetition policy
Ans: (b) (c) A memory allocation policy
Q50. The section of the CPU that selects, interprects (d) A process allocation policy
and monitors the execution of program instructions is Ans: (a)
(a) Memory (b) Register Q59. Which of the following computer memories is
(c) Control unit (d) ALU non-volatile ?
Ans: (c) (a) DRAM (b) SRAM
Q51. In the field of internet, www stands for (c) ROM (d) RAM
(a) world wide wrestling Ans: (c)
(b) world wide web Q60. What is the full form of PDA?
(c) world wide wordstar (a) Personal Digital Assistant
(d) world without wrestling (b) Personal Data Addition
Ans: (b) (c) Personal Digital Addition
Q52. DTP stands for (d) Peripheral Digital Assistant
(a) Digital Transmission Protocol Ans: (a)
(b) Desktop Publishing Q61. Which of the following commands is called as
(c) Data Type Programming three-finger salute in computers ?
(d) Document Type Processing (a) Ctrl + Alt + Delete
Ans: (b) (b) Ctrl + Shift + Escape
Page 431 of 516
(c) Ctrl + Shift + Enter
(d) Ctrl + Alt + Arrow key
https://telegram.me/aedahamlibra
(c) Application gateway
(d) Bridge
Ans: (a) Ans: (d)
Q62. A _____ is a computer, appliance, or router that Q70. Find the ―odd one out‖.
sits between the trusted and untrusted systems. (a) Linux (b) Windows 98
(a) bridge (b) switch (c) C++ (d) Windows 7
(c) firewall (d) hub Ans: (c)
Ans: (c) Q71. Programs that duplicate the functionality of one
Q63. Which of the following was used as circuitry for system on another system is known as
first generation of computers? (a) Emulators (b) Simulators
(a) Vacuum tube (c) Evaluators (d) PCB
(b) Transistors Ans: (a)
(c) Integrated circuits Q72. C. P. U. consists of
(d) Microprocessors (a) Arithmetic and Logical unit and Register
Ans: (a) (b) Arithmetic and Logical unit, Register and Control unit
Q64. The first computer made available for (c) System unit and Memory
commercial use was (d) Hard disk and Control unit
(a) MANIAC (b) ENIAC Ans: (b)
(c) UNIVAC (d) EDSAC Q73. Which of the following is the operation executed
Ans: (c) on data stored in registers?
Q65. In IT terminology failure in the kernel is called (a) Bitoperation
as (b) Macrooperation
(a) Crash (b) C r a s h dump (c) Microoperation
(c) Dump (d) Kernel error (d) Byteoperation
Ans: (a) Ans: (c)
Q66. The UNIX operating system is suitable for Q74. When does a Page fault occur?
(a) Multi user (a) There is an error in a specific page
(b) Real-Time Processing (b) A program accesses a page not currently in main
(c) Distributed Processing memory
(d) Single user (c) A program accesses a page of main memory
Ans: (a) (d) A program accesses a page belonging to another
Q67. In the following list of devices which device is program
used in network layer? Ans: (b)
(a) Repeaters Q75. One of the common forms of representing
(b) Router remote sensing data as information is in the form of
(c) Application Gateway (a) Thematic Maps
(d) Switch (b) Political Maps
Ans: (b) (c) Distribution Maps
Q68. The Network is overloaded with enormous data (d) Land use Maps
sent by many computers within the network. Ans: (a)
The inability of the network to deliver the data is termed Q76. Direct access of file is also known as
as __________ . (a) random access
(a) Access control (b) relative access
(b) Congestion (c) file access
(c) Error propagation (d) sequential access
(d) Deadlock Ans: (a)
Ans: (b)
Q69. In the following list of devices, which device is
used in datalink layer?
(a) Repeaters
(b) Routers
Page 432 of 516
Discoveries & https://telegram.me/aedahamlibra
Inventions (b) Alexander Graham Bell
(c) J.L. Baird
(d) Thomas Barrow
Ans: (b)
Q1. Who invented the video-tape?
Q10. Humidity is measured by
(a) Richard James
(a) Lactometer (b) Polarimeter
(b) Charles Ginsberg
(c) Thermometer (d) Hygrometer
(c) P.T. Farnsworth
Ans: (d)
(d) Georges de Mestral
Q11. Who is associated with the invention of Nylon ?
Ans: (b)
(a) Louis Pasteur
Q2. Who invented the ―Voice Mail‖?
(b) J. Nicephore Niepce
(a) Gordon Matthews
(c) John Corbutt
(b) Alexander Graham Bell
(d) Dr. Wallace H. Carothers
(c) J.A. Fleming
Ans: (a)
(d) V. Poulsen
Q12. Who discovered Solar System ?
Ans: (a)
(a) Newton (b) John Hadley
Q3. What is the name given to the outermost
(c) Copernicus (d) Galileo
―planetoid‖ discovered recently in the Solar System ?
Ans: (c)
(a) Quark (b) Xenon
Q13. Who invented aeroplane ?
(c) Sedna (d) Asterix
(a) Orville Wright and Wilbur Wright
Ans: (c)
(b) Sir Frank Whittle
Q4. Who developed the first automatic automobile ?
(c) Michael Faraday
(a) Goatleab Daimler
(d) Christian Huygens
(b) Henry Ford
Ans: (a)
(c) Rudolf Diesel
Q14. Who is the founder of quantum theory of
(d) Karl Benz
radiation ?
Ans: (b)
(a) Einstein (b) Bohr
Q5. Electron was discovered by
(c) Plank (d) S. N. Bose
(a) Ernest Rutherford
Ans: (c)
(b) Max Planck
Q15. Meteorology is the science of:
(c) Joseph Thomson
(a) Weather (b) Mateors
(d) Albert Einstein
(c) Metals (d) Earthquakes
Ans: (c)
Ans: (b)
Q6. Leprosy bacillus was invented by
Q16. Root pressure is measured by
(a) Koch (b) Hansen
(a) Barometer (b) Atmometer
(c) Fleming (d) Harvey
(c) Manometer (d) Auxanometer
Ans: (a)
Ans: (d)
Q7. Bacteria was first discovered by
Q17. Crescograph was invented by
(a) A.V. Leeuwenhoek
(a) S.N. Bose (b) P.C. Roy
(b) Robert Hooke
(c) J.C. Bose
(c) Robert Koch
(d) P.C. Mahalanobis
(d) Louis Pasteur
Ans: (c)
Ans: (a)
Q18. Who invented aeroplane ?
Q8. Who invented the Jet Engine?
(a) Edison (b) Stevenson
(a) Karl Benz
(c) Hoffman (d)Wright Brothers
(b) Sir Frank Whittle
Ans: (d)
(c) Thomas Savery
Q19. ―Linkage‖ was discovered by
(d) Michael Faraday
(a) Blakslee (b) Morgan
Ans: (b)
(c) Muller (d) Bateson
Q9. The telephone was invented by:
Ans: (d)
(a) G. Marconi
Q20. Who discovered the first antibiotic ?
Page 433 of 516
(a) W Fleming
(c) Louis Pasteur
https://telegram.me/aedahamlibra
(b) C Waksman
(d) A Fleming
(a) Study of flies
(b) Study of ants
Ans: (d) (c) Study of bees
Q21. Phycology is the study of (d) Study of spiders
(a) Bacteria (b) Algae Ans: (b)
(c) Fungi (d) Lichens Q28. Temperature of distant luminous bodies can be
Ans: (b) determined by
Q22. Richter Scale is used for measuring : (a) Mercury thermometers
(a) Velocity of wind (b) Gas thermometers
(b) Density of liquid (c) Pyrometers
(c) Intensity of earthquake (d) Colour thermometers
(d) Humidity of air Ans: (c)
Ans: (*) Q29. The Beaufort scale is used to measure :
Q23. Planimeter is used to measure: (a) Atmospheric pressure
(a) Height of a region (b) Altitudes of mountains
(b) Direction (c) Wind velocity
(c) Road Distance (d) Intensity of earthquakes
(d) Areas Ans: (c)
Ans: (d) Q30. ‗Agronomy‘ is the practice of raising ______
Q24. Venturimeter is used to measure: (a) Plants and Animals
(a) rate of flow of liquids (b) Crop plants
(b) liquid pressure (c) Agriculture
(c) surface tension (d) Fruit plants only
(d) liquid density Ans: (b)
Ans: (a) Q31. Science dealing with study of soil is called
Q25. Infrared radiations are detected by (a) Pedology (b) Pedagogy
(a) Pyrometer (b)Nanometer (c) Ecology (d) Pomology
(c) Photometer (d) Spectrometer Ans: (a)
Ans: (c) Q32. The movement inside the Earth‘s crust is studied
Q26. Which of the following statement is false? by which of the following ?
(a) Polystyrene was discovered in 1839 by Edward (a) Geology
Simon (b) Seismology
(b) It is used for making plastics and CD cases (c) Plate tectonics
(c) It is not biodegradable (d) Pantograph
(d) None of these Ans: (b)
Ans: (d)
Q27. What is Myrmecology?

Page 434 of 516


https://telegram.me/aedahamlibra
ENGLISH LANGUAGE
Q10. Having finished at school (a)/ Raghu thought/ of
Error Detection going to Bombay in (b)/ search some job. (c)/ No error
(d)
Ans: (c)
Directions: In the following questions, some parts of the
Q11. When shall we (a)/ arrive (b)/ to our destination ?
sentences have err ors and some are correct. Find out
(c)/ No error. (d)
which part of a sentence has an error. The number of that
Ans: (c)
part is the answer. If a sentence is free from error, your
Q12. Based on the newspaper reports, (a)/ we can
answer is No error.
conclude that (b)/ many accidents caused by reckless
Q1. You must either (a)/ be regular with your studies (b)/
driving. (c)/ No error. (d)
and study for longer peri od before the examination. (c)/
Ans: (c)
No error (d)
Q13. Females (a)/ a re not appointed (b)/ in our college.
Ans: (c)
(c)/ No error. (d)
Q2. The new taxation rates (a)/ announced by the
Ans: (d)
government (b)/ are bound to effect the export sector. (c)/
Q14. The officer (a)/ is angry on the clerk (b)/ for not
No error (d)
attending to the work. (c)/ No error. (d)
Ans: (c)
Ans: (b)
Q3. These days, job opportunities are not as better (a)/ as
Q15. No sooner (a)/ I had spoken, (b) than he left. (c)/ No
they used to be (b)/ in the early 70's. (c)/ No error (d)
error. (d)
Ans: (a)
Ans: (b)
Q4. When viewed with his point of view, the (a) / entire
Q16. Computer education (a)/ in universities and colleges
episode assumes (b)/ a different colour altogether. (c)/ No
(b) leaves much to be desired. (c)/ No error. (d)
error (d)
Ans: (c)
Ans: (a)
Q17. You will be prosecuted (a)/ for bringing seeds (b)/
Q5. On many occasions (a)/ we did helped the poor (b)/
into Australia.
people by way of giving them food to eat and clothes to
( (c)/ No error. (d)
put on. (c)/ No error (d)
Ans: (c)
Ans: (b)
Q18. You must either tell me (a)/ the whole story or, at
Q6. Unless it is accepted to both the parties, an (a)/
least (b)/ the first half of it. (c)/ No error. (d)
arbitrator would be of no (b)/ use to settle this dispute.
Ans: (a)
(c)/ No error (d)
Q19. Our new neighbours (a)/ had been living in Arizona
Ans: (a)
(b)/ since ten years before moving to their present house.
Q7. Although the manager was keen on getting the work
(c)/ No error. (d)
(a)/ done through Sudhir yesterday, (b)/ he tries to avoid
Ans: (c)
it (c)/ No error (d)
Q20. The patient (a)/ was accomp anied (b)/ with his
Ans: (c)
friend. (c)/ No error. (d)
Q8. The various consequences of (a)/ the decision taken
Ans: (c)
by the (b)/ finance ministry was not foreseen by the
Q21. A city dweller finds it difficult (a)/ to pass away the
bureaucrats. (c)/ No error (d)
time (b)/ in a village. (c)/ No error. (d)
Ans: (c)
Ans: (b)
Q9. I never considered him to be a person who would (a)/
Q22. Visitors (a)/ were not permitted (b)/ entering the
go back on his promise and (b)/ then do not even
park (c)/ after dark. No error (d)
apologise. (c)/ No error (d)
Ans: (c)
Ans: (c)
Q23. The fifth and final act (a)/ of Macbeth contain (b)/
the sleepwalking scene. (c)/ No error. (d)
Ans: (b)
Page 435 of 516
are shot dead. (c)/ No error (d)
https://telegram.me/aedahamlibra
Q24. One of the terrorists (a)/ of the Kashmir valley (b)/ Q40. My sister (a)/has read (b)/pages after pages of the
Bible. (c)/ No error. (d)
Ans: (c) Ans: (c)
Q25. Ten kilometres (a)/ is (b)/ a long distance to walk. Q41. Your success in the IAS examinations depends not
(c)/ No error (d) only on (a)/ what papers you have selected (b)/but on
Ans: (d) how you have written them. (c) No error. (d)
Q26. I saw him (a)/a couple of times (b)/ since May (c). Ans: (c)
/No error (d) Q42. Heavy rain (a)/prevented us (b)/ to go to the
Ans: (a) cinema. (c)/No error. (d)
Q27. Tea (a)/which I am drinking (b)/ is hot (c). /No Ans: (c)
error (d) Q43. If majority of the individuals in a State (a)/prosper
Ans: (a) (b)/the State itself would prosper. (c) /No error. (d)
Q28. Although the pol ice officer sympathised with poor Ans: (c)
(a)/ he refused to (b) /take an action against Q44. If motorists do not observe the traffic regulations
the rich man (c). /No error (d) (a)/they will be stopped, ticketed (b)/and have to pay a
Ans: (a) fine. (c)/ No error. (d)
Q29. We were looking forward (a)/ to hear news Ans: (c)
(b)/about the missing fishermen (c)./ No error (d) Q45. He asked (a)/supposing if he fails (b)/ what he
Ans: (b) would do. (c)/ No error. (d)
Q30. The actress (a)/was shocked (b)/by the news of he r Ans: (b)
dog‘s death (c). /No error (d) Q46. We had a lot of difficulty (a)/to find (b)/the house.
Ans: (c) (c)/ No error. (d).
Q31. One of the questions (a)/ he asked me was (b)/ Ans: (b)
―Who did you travel with (c) ?‖ /No error Q47. Patience as well as perseverance (a)/are ne cessary
(d) ) (b)/for success. (c)/ No error/ (d).
Ans: (d) Ans: (b)
Q32. I know (a)/a doctor (b)/you are referring to (c)./No Q48. The passer -by told us (a)/where was the marriage
error (d)) hall (b)/and even led us to it. (c)/ No error/ (d).
Ans: (b) Ans: (b)
Q33. The introduction of j ob-oriented courses (a)/in the Q49. The increase in consumption is directly
self-financing colleges (b)/ attract many students (c)./ No (a)/proportional to the increase (b)/in in come. (c)/No
error (d) error. (d).
Ans: (c) Ans: (d)
Q34. It is better (a)/to keep one‘s head in the face of Q50. In Singapore (a)/my brother -inlaw with his wife
danger than (b)/losing one‘s courage (c). /No error (d) (b)/were present at the function. (c)/No error. (d).
Ans: (c) Ans: (c)
Q35. The short story (a)/should not exceed (b)/more than Q51. Scarcely had (a)/I arrived than (b)/the train left.
two hundred words (c)./No error (d) (c)/No error. (d).
Ans: (c) Ans: (b)
Q36. To die with honour (a)/is better than (b)/live with Q52. The reason why (a)/he was rejected (b)/was because
dishonour. (c)/ No error. (d) he was too young. (c)/No error. (d).
Ans: (d) Ans: (c)
Q37. It is I (a)/who is to blame. (b)/for this bad situation Q53. Teachers of various schools (a)/ met to discuss
(c)./ No error. (d) about (b)/how to improve the standard of English. (c)/No
Ans: (b) error. (d).
Q38. Gowri told me (a)/his name after ( 2)/he left. (c)/ Ans: (b)
No error. (d) Q54. His tradition -bound attitude (a)/ had to be a
Ans: (c) constant source of dissatisfaction (b)/among the younger
Q39. John would have told (a)/you the truth (b)/ if you members of the family. (c)/No error. (d).
had asked him. (c)/ No error. (d) Ans: (d)
Ans: (d)
Page 436 of 516
https://telegram.me/aedahamlibra
Q55. The two first to arrive (a)/were the lucky recipients
(b)/of a surprise gift. (c)/No error. (d).
Ans: (d)
Q71. Even today (a)/ it is incredulous to think (b)/ that
Ans: (a) men have walked on the moon. / (c) No error. (d)
Q56. Two hours have passed (a)/ since (b)/he had fallen Ans: (b)
asleep. (c)/No error (d). Q72. The firemen wore (a)/ inflammable clothing (b)/ for
Ans: (c) protection. / (c) No error. (d)
Q57. Having broken down (a)/the driver sent the car Ans: (b)
(b)/to the garage. (c)/No error (d). Q73. Beside food, (a)/ the pilgrims carried (b)/ some
Ans: (b) medicines. / (c) No error. (d)
Q58. He is one of those writers (a)/ who has won acclaim Ans: (a)
(b)/the world over. (c)/No error (d). Q74. Adults suffering chicken pox (a)/ can develop (b)/
Ans: (b) all kinds of complications (c) No error. (d)
Q59. The mason will not (a)/do the work (b)/except give Ans: (a)
the order. (c)/No error (d). Q75. The we ll-known pianist (a)/ had to practice for
Ans: (c) several hours a day (b)/ even after he rose to fame. (c)/
Q60. When students are ill (a)/they find that they have a No error. (d)
lot of work (b)/to catch up with when they return. (c)/No Ans: (b)
error (d). Q76. The Prime Minister was asked (a)/ to write a
Ans: (c) forward (b)/ to the book. (c)/ No error. (d)
Q61. Scarcely did I reach the airport, (a)/nervous and Ans: (b)
tense, than the plane took off, (b)/leaving me stranded in Q77. I must complement you (a)/ on your good manners
an alien place. (c)/ No error (d). (b)/ and your impeccable behaviour. (c)/ No error. (d)
Ans: (b) Ans: (a)
Q62. The power to (a)/distinguish between differences Q78. The sweets (a)/ were shared (b)/ between the four
(b)/is the basis of science and art. (c)/No error (d). girls. / (c) No error. (d)
Ans: (b) Ans: (c)
Q63. I have (a)/seen her only once (b)/ but I‘m liking her Q79. The long -distance train (a)/ which met with an
a lot. (c)/No error (d). accident (b)/ was carrying some army personal. / (c) No
Ans: (c) error. (d)
Q64. I told him (a)/that we enjoyed very much (b)/at the Ans: (c)
party. (c)/ No error (d). Q80. The young men from Japan (a)/ found the assent of
Ans: (b) the mountain (b)/ hard going. (c)/ No error. (d)
Q65. John is working very hardly (a)/ as the Ans: (b)
examinations (b)/are fast approaching. (c)/No error (d). Q81. If the worst (a)/comes to worst,
Ans: (a) ( 2)/I will have to bid good-bye to my studies and join my
Q66. He ate (a)/ nothing (b)/ since yesterday (c)/. No family business. (c) /No error. (d)
error (d) Ans: (b)
Ans: (a) Q82. The interim report does not (a)/ analyse thoroughly
Q67. An experimental vaccine (a)/ has brought (b)/ the principle causes (b)/of the disaster (c) / No error (d).
glimmer of hope for the malarial researchers (c)/. No Ans: (b)
error (d) Q83. The items I liked most (a)/were the rosewood
Ans: (c) carvings (b)/and the teak -wood furnitures of Dutch
Q68. After making me wait for two agonising hours (a)/ design (c)/No error. (d)
the great man called me in (b)/ and asked me what do I Ans: (c)
want (c)/. No error (d) Q84. A part of the training (a)/they offered was (b)/real
Ans: (c) good (c)/No error. (d)
Q69. The ebb and flow of the tides (a)/ are (b)/ now Ans: (c)
understood (c)/. No error (d) Q85. The soil was moist as (a)/there was little rain (b)/the
Ans: (b) day before (c)/No error. (d)
Q70. The green paint on the wall (a)/ provides a suitable Ans: (b)
contrast (b)/ with the yellow doors (c)/. No error (d)
Page 437 of 516
https://telegram.me/aedahamlibra
Q86. The ma nager wanted to know who had arrived
(a)/early that day (b)/the cashier or the accountant (c)/No
Q100. He denied (a)/ to have (b)/ been there (c)/. No
error (d).
error. (d) Ans: (b)
Ans: (b)
Q87. They are a politically important family; (a)/one of
his sisters is a minister (b)/ and the other is married with
Improvement Of
a minister (c)/No error. (d)
Ans: (c)
Sentences
Q88. Supposing that the information (a)/proves to be
totally reliable, (b)/should we still have to recast the plans Directions —In these questions, a part of the sentence is
? (c)/No error. (d) given in bold. Below are given alternatives to the bold
Ans: (c) part at 1, 2 and 3 which may improve the sentence.
Q89. Though its gloss can (a)/fool few unwary Choose the correct alternative. In case no improvement is
customers, (b)/it wouldn‘t be difficult for the clever ones needed your answer is (d).
to judge its real worth (c)/No error. (d) Q1. On one occasion he persuaded me to accompany him
Ans: (b) on a shooting expedition he was planning.
Q90. She pretends as if she has (a)/ never in her life, (a) has planned
(b)/told a lie. (b) had planned
Isn‘t it ? (c)/No error. (d) (c) planned
Ans: (a) (d) No improvement
Q91. Knowledge of (a)/ at least two languages (b)/ are Ans: (b)
required to pass the examination (c)/. No error (d). Q2. We have no less than a thousand students in our
Ans: (c) College.
Q92. The members of the Opposition Party in the (a) not less
Parliament (a)/ shout upon the minister (b)/ if he makes a (b) no less
wrong statement (c)/. (c) no fewer
No error (d). (d) No improvement
Ans: (b) Ans: (c)
Q93. Everyone of the films (a)/ you suggested (b)/ are Q3. When she regained consciousness she was stretched
not worth seeing (c)/. No error (d). in the ditch out, soaked to the skin.
Ans: (a) (a) out in the ditch stretched
Q94. The Secretary and the Principal of the college (a)/ (b) on the ditch outstretched
are attending (b)/ the District Development Council (c) outstretched in the ditch
Meeting at the Collectorate (c)/. No error (d). (d) No improvement
Ans: (d) Ans: (c)
Q95. No sooner had the hockey match started (a)/ when it Q4. Too little is known by this time about possible side-
began (b)/ to rain (c)/. No error (d). effects of the drug.
Ans: (b) (a) presently
Q96. The Secretariat (a)/ comprises of (b)/ many air - (b) in the end
conditioned rooms (c)/. No error (d). (c) before hand
Ans: (b) (d) No improvement
Q97. It is high time (a)/ he stood on his own (b)/ two legs Ans: (a)
(c)/. No error (d). Q5. If you want to save money you must get rid of
Ans: (c) shopping.
Q98. You should avoid (a)/ to travel (b)/ in the rush hour (a) cut down on
(c)/. No error (d). (b) get on with
Ans: (b) (c) put up with
Q99. There is (a)/ only one of his novels (b)/ that are (d) No improvement
interesting (c)/. Ans: (a)
No error (d).
Ans: (c)
Page 438 of 516
who is trying to concentrate.
https://telegram.me/aedahamlibra
Q6. The loud and incessant chatter worries your father (d) No improvement
Ans: (b)
(a) aggravates Q14. The newspaper provides mor e International News
(b) annoys than domestic News.
(c) irritates (a) National
(d) No improvement (b) alien
Ans: (c) (c) provincial
Q7. Not only they went to see a film, but also had dinner (d) No improvement
out. Ans: (a)
(a) Not only did they go Q15. The teacher was very angry at his student.
(b) They didn't go not only (a) angry with
(c) They not only went (b) angry against
(d) No improvement (c) angry about
Ans: (a) (d) No improvement
Q8. Birds sit on the boughs of trees in my garden and Ans: (a)
with their sweet notes fill the air with music. Q16. He backed out of the agreement.
(a) fill their sweet notes in the air (a) gave his full support
(b) by their sweet notes fill the air (b) reconsidered the point
(c) fill the air by their sweet notes (c) withdrew his support from
(d) No improvement (d) went through the back door
Ans: (c) Ans: (c)
Q9. India will enter the league of major developed Q17. Man is the only animal who can talk.
nations as a space giant within a short time. (a) which
(a) energy (b) whom
(b) force (c) that
(c) power (d) No improvement
(d) No improvement Ans: (d)
Ans: (c) Q18. He did not know the answer, I did not neither.
Q10. Many a man would welcome the opportunity. (a) neither did I
(a) Many man (b) either did I
(b) A many man (c) neither have I
(c) Many a men (d) No improvement
(d) No improvement Ans: (a)
Ans: (d) Q19. When the owner offered tea, the peon denied it.
Q11. The greater the demand, higher the price. (a) disliked
(a) high (b) declined
(b) the high (c) disobeyed
(c) the higher (d) No improvement
(d) No improvement Ans: (b)
Ans: (c) Q20. ―Do you remember his phone number?
Q12. I prefer to ride than to walk. I don't suspect so.‖
(a) riding to walking (a) don't think
(b) ride to walk (b) don't thing
(c) riding than walking (c) may think
(d) No improvement (d) No improvement
Ans: (a) Ans: (a)
Q13. The workers went on a strike asking for better pay Q21. Please report to me when you return back from
and service condition Delhi.
(a) requesting (a) you have returned back from
(b) demanding (b) you return from
(c) needing (c) you returned back from
Page 439 of 516
(d) No improvement https://telegram.me/aedahamlibra
Ans: (b)
(d) No improvement Directions (31–40): In the
following questions, part of the sentence is in bold.
Q22. Ravi has been demanding a lot more marks, doesn't Below are given alternatives to the bold part at 1, 2, and
he ? 3 which may improve the sentence.
(a) hasn't he ? Choose the correct alternative. In case no improvement is
(b) isn't he ? needed, your answer is 4.
(c) isn't it ? Ans: (b)
(d) No improvement Q31. He told me that he was married four years.
Ans: (a) (a) over four years
Q23. Exercise is benevolent to good health. (b) four years ago
(a) beneficial (c) about four years
(b) beneficent (d) No improvement
(c) bounteous Ans: (b)
(d) No improvement Q32. Chennai is by the way a cosmopolitan city.
Ans: (a) (a) by the by
Q24. He parked his vehicle under the shade of a tree. (b) by and large
(a) on (b) in (c) on the large
(c) beneath (d) No improvement (d) No improvement
Ans: (b) Ans: (b)
Q25. In case if it rains, I shall not visit you. Q33. For me, money is only the means to an end.
(a) If so (b)In case of (a) only means
(c) If/In case (d)No improvement (b) only the mean
Ans: (c) (c) only a means
Q26. TV sets have become costlier, thanks to the sudden (d) No improvement
rise in the prices of imported compounds. Ans: (c)
(a) tubes (b) components Q34. The bullet struck a wall and was diverted from its
(c) things (d) No improvement course.
Ans: (b) (a) twisted (b) reflected
Q27. Because the enemy had a reputation for engaging in (c) deflected (d) No improvement
sneak attacks we were frequently on the alert. Ans: (c)
(a) occasionally Q35. You should not boasting of your achievements.
(b) inevitably (a) boast for (b) boast at
(c) constantly (c) boast of (d) No improvement
(d) No improvement Ans: (c)
Ans: (c) Q36. Kannan asked me to go round with them, but I
Q28. Less people came to watch cricket matches this didn't want.
year, so the gate receipts were lesser than last year. (a) don't want
(a) Many (b) Fewer (b) don't want to
(c) Lot of (d) No improvement (c) didn't want to
Ans: (b) (d) No improvement
Q29. God has bestowed man unusual gifts. Ans: (c)
(a) bestowed with man Q37. We eat that we may live.
(b) bestowed for man (a) might (b) shall
(c) bestowed on man (c) should (d) No improvement
(d) No improvement Ans: (d)
Ans: (c) Q38. He had his breakfast, when we visited him.
Q30. Many workers were being held hostages. (a) He had taken his breakfast
(a) held to be hostages (b) He wished to have his breakfast
(b) held as hostages (c) He avoided his breakfast
(c) held like hostages (d) No improvement
Ans: (a)
Page 440 of 516
whom he was eluding.
https://telegram.me/aedahamlibra
Q39. When the Inspector of Police said this, we knew Ans: (c)
Q48. The U.N.O. insists on better understanding between
(a) intending (b) referring to the countries of the world.
(c) hinting (d) No improvement (a) among the countries
Ans: (b) (b) with the countries
Q40. The doctor reassured that the operation was a (c) by the countries
routine one. (d) No improvement
(a) is reassuming Ans: (d)
(b) reassured me Q49. He doesn‘t smoke, nor I did.
(c) was reassuming (a) nor I smoke
(d) No improvement (b) nor I do
Ans: (b) (c) nor do I
Q41. Let us go see her new house, shall we? (d) No improvement
(a) do we ? (b) don‘t we ? Ans: (c)
(c) do they ? (d) No improvement Q50. His reasons can be good, is it not ?
Ans: (d) (a) can they ?
Q42. It took a long time for him to realise, what was (b) isn‘t he ?
truth. (c) can‘t they ?
(a) what is truth. (d) No improvement Directions (51 – 55): In the
(b) what was the truth. following questions, a part of the sentence is bold. Below
(c) what the truth was. are given alternatives to the bold part at (a),
(d) No improvement (b) and (c) which may improve the sentence. Choose the
Ans: (c) correct alternative. In case no improvement is needed,
Q43. My mother is worried about my father‘s health and your answer is (d).
also I. Ans: (c)
(a) I also (b) also me Q51. If I am the P.M. I would ban all processions.
(c) so am I (d) also I am (a) will be (b) were
Ans: (c) (c) am (d) No improvement
Q44. Roads are wet, it must have rained last night. Ans: (b)
(a) must had Q52. Mrs. Madhuri has great respect and unlimited
(b) might have been faith in her Director.
(c) must have been (a) respect and unlimited faith for
(d) No improvement (b) respect for and unlimited faith for
Ans: (d) (c) respect for and unlimited faith in
Q45. He must know them in their most plausible form, (d) respect in and unlimited faith for
isn‘t it ? Ans: (c)
(a) isn‘t he ? (b) mustn‘t he ? Q53. He is addicted to smoke.
(c) can‘t he ? (d) No improvement (a) addicted to smoking
Ans: (b) (b) used to smoke
Q46. English opens the window upon the wor ld‘s (c) addicted of smoking
knowledge. (d) addicted with smoking
(a) the English Ans: (a)
(b) the English language Q54. Write down the address lest you may forget.
(c) the English land (a) you may not forget
(d) No improvement (b) you cannot forget
Ans: (b) (c) you will forget
Q47. If one works hard, you have got the reward. (d) you should forget
(a) one gets Ans: (d)
(b) one has got Q55. We used to ge t up early in the morning, have
(c) one will get breakfast and then went out to play.
(d) No improvement (a) go out to play
Page 441 of 516
(b) gone out to play
(c) had gone out to play
https://telegram.me/aedahamlibra
(a) neither I do nor does my friend
(b) neither do I nor does my friend
(d) played (c) neither I live nor does my friend
Ans: (a) (d) No improvement
Q56. Can this machine be adopted in farm work. Ans: (b)
(a) by (b) into Q66. He is the poorest of the two workers.
(c) for (d) No improvement (a) the poorer of
Ans: (c) (b) the poorest between
Q57. The driver was dazzled by the bright light of (c) poorest of
approaching car. (d) No improvement
(a) twinkled (b) flashed Ans: (a)
(c) glowed (d) No improvement Q67. The help given to us was more adequate for our
Ans: (d) purpose.
Q58. What does it matter most is the quality of the (a) much adequate
goods that we require. (b) enough adequate
(a) What it matters more (c) more than adequate
(b) What does it matter more (d) No improvement
(c) What matters most Ans: (c)
(d) No improvement Q68. Quite the most remarkable article we ever
Ans: (c) remember to have read.
Q59. You called on me when I was not at home, don‘t (a) ever remarkable article we remember to have read.
you ? (b) remarkable article we remember ever to have read.
(a) did you ? (b) didn‘t you ? (c) remarkable ever article we remember to have read.
(c) didn‘t I ? (d) No improvement (d) No improvement
Ans: (b) Ans: (d)
Q60. He paid the caretaker for his board and lodging. Q69. You won‘t tell them what has happened, wouldn‘t
(a) room (b) furniture you ?
(c) boarding (d) No improvement (a) won‘t you ?
Ans: (d) (b) isn‘t it ?
Q61. It‘s high time that you go home. (c) will you ?
(a) have gone (b) should go (d) No improvement
(c) went (d) No improvement Ans: (c)
Ans: (c) Q70. She doesn‘t mind to be disturbed.
Q62. It is proposed to construct a bath for males 99 -feet (a) being disturbed
long. (b) to being disturbed
(a) a 99-feet long bath for males (c) being disturbing
(b) a 99-feet long males bath (d) No improvement Directions (71–80): In the
(c) a males bath 99-feet long following questions, a part of the sentence is in bold.
(d) No improvement Below are given alternatives to the bold part at (a), (b)
Ans: (a) and (c) which may improve the sentence. Choose the
Q63. Corrupt officials have siphoned off more than five correct alternative. In case no improvement is needed,
hundred crore rupees over the last decade from welfare your answer is (d).
funds. Ans: (a)
(a) away (b) up Q71. By this time tomorrow, I will reach my home.
(c) in (d) No improvement (a) will be reaching
Ans: (d) (b) shall have reached
Q64. They are migratory workers from another country. (c) can reach
(a) immigrant (b) itinerant (d) No improvement
(c) emigrant (d) No improvement Ans: (d)
Ans: (c) Q72. He was so afraid th at his knees knocked one
Q65. Neither I nor my friend live here. another.
Page 442 of 516
(a) one against
(b) each other
https://telegram.me/aedahamlibra
(b) enquired of the peasant if he could
(c) repl ied of the peasant whether he will
(c) both (d) No improvement
(d) No improvement Ans: (b)
Ans: (b) Q82. As I was new to the place, I felt like a fish in
Q73. They only work when they have no money. water.
(a) when they have no money, they only work (a) felt like a fish in the water.
(b) when they only work they have no money (b) felt like a fish with water.
(c) they work only when they have no money. (c) felt like a fish out of water.
(d) No improvement. (d) No improvement
Ans: (c) Ans: (c)
Q74. When we saw him last, he ran to catch a bus. Q83. The flood -affected people are looking forward
(a) has run with the visit of the Governor.
(b) was running (a) looking forward to
(c) had run (b) looking forward on
(d) No improvement (c) looking forward for
Ans: (b) (d) No improvement
Q75. He suddenly struck a note of discord in his Ans: (a)
otherwise harmonious presentation. Q84. He is fond of saving money.
(a) unhappiness (a) hoarding
(b) regret (b) not spending
(c) anger (c) spending carefully
(d) No improvement (d) No improvement
Ans: (d) Ans: (d)
Q76. If I dyed my hair green, everybody will laugh at Q85. He comes often to our house.
me. (a) come often
(a) would (b) did (b) often comes
(c) may (d) No improvement (c) often come
Ans: (a) (d) No improvement
Q77. The students often play truant, didn‘t they ? Ans: (b)
(a) can they ? Q86. Most of the participating members at the fair, which
(b) is indeed ? was organised by the students, were Mathematics
(c) don‘t they ? teacher‘s.
(d) No improvement (a) Mathematic teachers
Ans: (c) (b) Mathematics teachers
Q78. He is adequately provided for the necessities of life. (c) Teacher in mathematics
(a) by (b) to (d) No improvement
(c) with (d) No improvement Ans: (b)
Ans: (c) Q87. Americans do not object my calling them by their
Q79. The T.V. news is doctored by nonprofessionals and first names.
whetted by political higher-ups. (a) my calling the
(a) wetted (b) vetted (b) to my calling them
(c) written (d) No improvement (c) been called
Ans: (b) (d) No improvement
Q80. Will you type these letters now ? Ans: (b)
(a) Could (b) Can Q88. The NYC building at New York City is as high as
(c) Shall (d) No improvement every other building in the United States of America.
Ans: (a) (a) high than every other building
Q81. The traveller commanded of the peasant he would (b) as high as every other building
tell him the way to the nearest village. (c) higher than any other one
(a) exclaimed of the peasant if he would (d) No improvement
Page 443 of 516
https://telegram.me/aedahamlibra
Q89. But here again, there are allegations
Ans: (c)
of
Q96. She has slept for eight hours last night.
(a) slept
indiscriminate reclamations and acquisition to the (b) had slept
farmland by the Adanis. (c) has been sleeping
(a) reclamations and also acquisition by the farmland (d) No improvement
(b) reclamation as well as acquisition of farmland Ans: (a)
(c) reclamation and acquisition of farmland Q97. I have seen the film and she also has .
(d) No improvement (a) has also (b) has too
Ans: (c) (c) too has (d) No improvement
Q90. It is a three-years degree course. Ans: (b)
(a) an three-years degree course Q98. He is in want of a reliable servant.
(b) a three-year degree course (a) refused (b) needs
(c) a three years degree course (c) declined (d) No improvement
(d) No improvement Ans: (b)
Ans: (b) Q99. He was rejected because he was too young.
Q91. As soon as winter sets in, the number of tourists (a) so (b) hence
start increasing suddenly. (c) though (d) No improvement
(a) the number of tourists are increased Ans: (d)
(b) the amount of tourists start increasing Q100. I hope that I shall get a First Class.
(c) the number of tourists increases (a) I feel that
(d) No improvement (b) I hope
Ans: (c) (c) I am doing
Q92. Is respect really preferable than money ? (d) No improvement
(a) preferable to money ? Ans: (b)
(b) preferred, or money ?
(c) preferable than money ?
(d) No improvement
Active/Passive Voice
Ans: (a)
Q93. His speech was marked by disagreement and Directions (1-5): In these questions, the sentences have
scorn for his opponent‘s position. been given in Active/ Passive Voice. From the given
(a) is being marked with disagreement and scorn for his alternatives, choose the one which best expresses the
opponent‘s position. given sentence in Passive/Active Voice.
(b) was marked with disagreement and scorn for his Q1. They first sun-dried the garbage for one to three days
opponent‘s position. to bring down the moisture level.
(c) was marked by disagreement and scorn for this (a) The moisture level was brought down by sun -drying
opponents positions. the garbage for one to three days.
(d) No improvement (b) One to three days of sun -drying brought down the
Ans: (b) moisture level of the garbage.
Q94. Changu is as tall if not, taller than Chanchu. (c) The moisture level of the garbage came down when it
(a) as tall, if not, taller than was sun-dried for one to three days.
(b) as tall as, if not taller to (d) The garbage was first sundried for one to three days
(c) as tall as, if not taller than to bring down the moisture level.
(d) No improvement Ans: (d)
Ans: (c) Q2. Women like men to flatter them.
Q95. The bigger dilemma facing these mega stores is (a) Men are liked by women to flatter them.
how to retain customers after the novelty wear thin. (b) Women like to be flattered by men.
(a) wears thin. (c) Women like that men should flatter them.
(b) cools down. (d) Women are liked to be flattered by men.
(c) wears off. Ans: (b)
(d) No improvement Q3. What one must do, one must do properly.
Ans: (a)
Page 444 of 516
https://telegram.me/aedahamlibra
(a) What must be done, must be done properly.
(b) It must be done properly what one must do.
(b) Men, women an d children throng the shops before
festivals making various purchases.
(c) It must be done what one must do properly. (c) Men, women and children make purchases during
(d) One must do properly what has to be done. festivals.
Ans: (a) (d) The shops are througed by people making purchases.
Q4. Look at the poll results–do they inspire hope ? Ans: (b)
(a) Let the poll results be looked – is hope inspired by Q11. The smugglers did not realise that their
them ? conversation was being recorded.
(b) Let the poll results be looked at–has hope been (a) The smugglers did not realise that someone was
inspired by them ? recording their conversation.
(c) Let the poll results be looked at–is hope being (b) Someone did not realise that the smugglers were
inspired by them ? recording their conversation.
(d) Let the poll re sults be looked at–is hope inspired by (c) Conversation was recorded when the smugglers did
them ? not realise.
Ans: (d) (d) The smugglers recorded their conversation without
Q5. It is your duty to make tea at eleven O'clock. realising.
(a) You are asked to make tea at eleven O'clock. Ans: (a)
(b) Your are required to make tea at eleven O'clock. Q12. The principal kept the staff members waiting
(c) You are supposed to make tea at eleven O'clock. (a) The staff members are kept waiting for the principal.
(d) Tea is to be made by you at eleven O'clock. (b) The staff members were kept waiting by the principal.
Ans: (c) (c) The staff members were waiting for the principal.
Q6. He was congratulated by his teacher on his brilliant 4) The staff members were being kept waiting by the
success in the recent examination. principal.
(a) His teacher congratulated him on his brilliant success Ans: (b)
in the recent examination. Q13. Who taught you grammar ?
(b) His teacher c ongratulated him for his success in the (a) By whom you were taught grammar ?
examination. (b) By whom were you taught grammar ?
(c) His teacher congratulated him on his success. (c) By whom was grammar you taught ?
(d) His teacher congratulated him. (d) By whom were grammar taught to you ?
Ans: (a) Ans: (b)
Q7. People speak English all over the world. Q14. The king gave him a reward.
(a) English is spoken all over the world. (a) He was given by the king a reward.
(b) English was spoken all over the world. (b) He was given the reward by a king.
(c) English was spoken by people. (c) He was given a reward by the king.
(d) English is spoken by people. (d) A reward was given by him to the king.
Ans: (a) Ans: (c)
Q8. Who gave you permission to enter? Q15. He teaches us grammar.
(a) By whom were you given permission to enter? (a) Grammar was taught to us by him.
(b) By whom was you given permission to enter? (b) We are taught grammar by him.
(c) By whom you were given permission to enter? (c) We were teached grammar by him.
(d) By whom given you permission to enter? (d) Grammar will be teached to us by him.
Ans: (a) Ans: (b)
Q9. The Principal has granted him a scholarship. Q16. Do you understand what I mean ?
(a) A scholarship has granted to him by the Principal. (a) What I mean....is that understood by you
(b) He has been granted a scholarship by the Principal. (b) Was what I mean understood by you ?
(c) He has granted a scholarship by the Principal. (c) Is what I mean understood by you ?
(d) A scholarship was granted to him by the Principal. (d) What I mean is understood by you ?
Ans: (b) Ans: (c)
Q10. Before festivals the shops are thronged with men, Q17. Whom does he look for ?
women and children making various purchases. (a) He is looked after for whom ?
(a) During festivals people throng the shops. (b) Who is looked after for him?
Page 445 of 516
(c) Who is looked for by him ?
(d) He is looked after by whom?
https://telegram.me/aedahamlibra
Q26. This shirt cannot be worn by me any longer.
(a) I cannot wear this shirt any longer.
Ans: (c) (b) Wearing of this shirt any longer is not possible.
Q18. They say that you did that. (c) This shirt is too worn out to be worn any longer.
(a) You are told to do that (d) This worn out shirt cannot be worn any longer.
(b) You are advised to do that Ans: (a)
(c) You did that said by them Q27. A lion does not eat grass, however hungry he may
(d) You are said to have done that be.
Ans: (d) (a) Grass is not eaten by a lion, however hungry he may
Q19. I am doing sums. be.
(a) Sums are done by me (b) Grass is not being eaten by a lion, however hungry he
(b) Sums are being done by me may be.
(c) I must be doing the sums (c) Grass is eaten not by a lion, however hungry he may
(d) Sums must be done by me be.
Ans: (b) (d) Grass is being not eaten by a lion, however hungry he
Q20. The noise of the traffic kept me awake. may be.
(a) I was kept awake by the noise of the traffic Ans: (a)
(b) The traffic kept me awake by the noise Q28. Someone saw him picking up a gun.
(c) I kept myself awake due to the noise of the traffic (a) He was seen pick up a gun by someone.
(d) I remained awake by the noise of the traffic (b) He was seen picking up a gun by someone.
Ans: (a) (c) He was seen when he was picking up a gun.
Q21. We all know that there is only one God. (d) He was seen by someone pick a gun.
(a) We are all known that there is only one God. Ans: (b)
(b) It is known to us all that there is only one God. Q29. He was obliged to resign.
(c) We have all known that there is only one God. (a) He was made to resign.
(d) Only one God is known by us all. (b) To resign was his obligation.
Ans: (b) (c) Circumstances obliged him to resign.
Q22. The people elected him Mayor. (d) Resignation obliged him.
(a) Him was elected Mayor the people. Ans: (c)
(b) He was elected Mayor by the people. Q30. Why did you not agree to my proposal ?
(c) Mayor is elected by the people. (a) Why was my proposal not agreed to ?
(d) He is elected by the people Mayor. (b) Why was my proposal not agreed by you ?
Ans: (b) (c) Why my proposal was not agreed to by you ?
Q23. Don‘t laugh at me. (d) Why was my proposal not agreed to by you ?
(a) Let me be laughed at. Ans: (d)
(b) Let me be not laughed at. Q31. It is time to take tea.
(c) I am laughed at. (a) It was time that tea was taken
(d) Let me be not laughed. (b) It is time for tea to be taken
Ans: (b) (c) It is time that tea should be taken
Q24. I saw him leaving the house. (d) It is time that tea had been taken
(a) Leaving the house he was seen by me. Ans: (b)
(b) He was seen leaving the house by me. Q32. The members should adhere to all the decisions.
(c) He had been seen leaving the house. (a) All the decisions should adhere to the members
(d) He was seen to be leaving the house. (b) All the decisions adhered to the members
Ans: (b) (c) All the decisions should be adhered to by the
Q25. Someone pulled the bull violently. members
(a) The bull had been pulled violently by someone. (d) All should adhere to the decisions of the members
(b) The bull was to be pulled violently by someone. Ans: (c)
(c) The bull had been pulled violently. Q33. His subordinates accused him of various offences.
(d) The bull was pulled violently. (a) They accused him of various offences
Ans: (d)
Page 446 of 516
various offences
https://telegram.me/aedahamlibra
(b) It was accused by his subordinates that he had done (c) You are expected to complete this work before sunset.
(d) You are expected to be completed this work before
(c) His subordinates accused that he had done various sunset.
offences Ans: (c)
(d) He was accused of various offences by his Q40. The storm did much damage.
subordinates (a) Much damage was done by the storm.
Ans: (d) (b) The storm damaged much.
Q34. Has someone made all the necessary arrangements (c) Much damage did the storm.
? (d) The storm was damaged.
(a) Has all the necessary arrangements been made by Ans: (a)
someone ? Q41. The boy has rung the bell.
(b) Have the necessary arrangements been all made by (a) The bell has been rung by the boy.
someone ? (b) The bell was being rung by the boy.
(c) Have all the necessary arrangements been made by (c) The bell was rung by the boy.
someone ? (d) The bell has been being rung by the boy.
(d) All the necessary arrangements have been made by Ans: (a)
one ? Q42. He likes people to call him Sir.
Ans: (c) (a) He likes to be called Sir by people.
Q35. We will know the outcome of these experiments (b) He likes to be call Sir by people.
after six months. (c) He likes people who call him Sir.
(a) The outcome of these experiments will be known after (d) To call him Sir is liked by people.
six months Ans: (a)
(b) The outcome will be known of these experiments Q43. We added up the money and found that it was
after six months correct.
(c) After six mont hs, we will know the outcome of these (a) The money was added up and found to be correct.
experiments (b) Correct it was found and the money was added up.
(d) These experiments will have a known outcome after (c) The money added up by us and it was correctly found.
six months (d) The money added up by us found it was correct.
Ans: (a) Ans: (a)
Q36. Don‘t speak until someone speaks to you. Q44. The telegraph wires have been cut.
(a) Don‘t speak until you are spoken to (a) Someone has been cut the telegraph wires.
(b) Don‘t speak until someone is spoken to (b) No one has cut the telegraph wires.
(c) Don‘t speak until you have been spoken to (c) The telegraph wires have cut someone.
(d) Don‘t speak until someone has been spoken to (d) Someone has cut the telegraph wires.
Ans: (a) Ans: (d)
Q37. Did the noise frighten you ? Q45. Will she tell us the truth ?
(a) Did you frighten the noise ? (a) Is the truth told to us by her ?
(b) Was the noise frightened by you ? (b) The truth will be told to us by her.
(c) Were you frightened by the noise ? (c) Will the truth be told to us by her ?
(d) Were you frighten by the noise ? (d) Will the truth be told us by her ?
Ans: (c) Ans: (c)
Q38. We are reaching the end of this exercise. Q46. Mr. Sen asked him a question.
(a) This exercise is ended by us. (a) He was asked a question Mr.
(b) The end of this exercise is being reached by us. Sen.
(c) This is our end to the exercise. (b) He was asked a question to
(d) The exercise has reached its end by us. Mr. Sen.
Ans: (b) (c) He was asked a question by
Q39. I expect you to complete this work before sunset. Mr. Sen.
(a) I expect you to be completed this work before sunset. (d) A question was being asked by Mr. Sen.
(b) I am expected you to complete this work before Ans: (c)
sunset. Q47. People speak English all over the world.
Page 447 of 516
https://telegram.me/aedahamlibra
(a) English is spoken all over the world.
(b) English was spoken all over the world.
(a) A king has been made by him.
(b) He was made a king by them.
(c) English was spoken by people. (c) They have been made kings by him.
(d) English is spoken by people. (d) He has been made a king by them.
Ans: (a) Ans: (d)
Q48. The teacher punished the boys who had not done Q55. Who taught you English ?
their home work. (a) By whom English was taught to you ?
(a) The boys who had not done their homework had been (b) By whom you were taught English ?
punished by their teacher. (c) By whom was English taught to you ?
(b) The boys were punished by their teacher who had not (d) By whom are you taught English ?
done their homework. Ans: (a)
(c) The boys who had not done their homework were Q56. Why haven‘t they allowed you to go?
punished by the teacher. (a) Why you haven‘t been allowed to go?
(d) The boys who had not done their homework were (b) Why haven‘t you been allowed to go?
being punished by the teacher. (c) Why were you not allowed to go?
Ans: (c) (d) Why you were not allowed to go?
Q49. The Principal has granted him a scholarship. Ans: (b)
(a) A scholarship has granted to him by the Principal. Q57. I expected him to give us financial aid.
(b) He has been granted a scholarship by the Principal. (a) It was expected of him to give us financial aid.
(c) He has granted a scholarship by the Principal. (b) Let it be expected that he would give us finacial aid.
(d) A scholarship was granted to him by the Principal. (c) It was expected by me that he will give us financial
Ans: (b) aid.
Q50. Somebody told me that there had been an explosion (d) He may be expected to give us financial aid.
in the Town Hall. Ans: (c)
(a) I was told by somebody about the explosion in the Q58. Tobacco manufacturers are making considerable
Town Hall. efforts to gain new clients.
(b) I was told about the explosion in the Town Hall. (a) Considerable efforts are being made by tobacco
(c) I was informed that there was an explosion in the manufacturers to gain new clients.
Town Hall. (b) Considerable efforts being made by tobacco
(d) I was told by somebody that there had been an manufacturers to gain new clients.
explosion in the Town Hall. (c) Considerable efforts are made by tobacco
Ans: (d) manufacturers to gain new clients.
Q51. Circumstances will oblige me to go. (d) To gain new clients by tobacco manufacturers
(a) I will oblige the circumstances and go. considerable efforts are being made.
(b) I shall be obliged to go by the circumstances. Ans: (a)
(c) Under the circumstances, I should go. Q59. Has anybody done all the work?
(d) I would be obliged by the circumstances to go. (a) Have all the work been done by somebody?
Ans: (b) (b) Somebody has done all the work.
Q52. We waste much time on trifles. (c) The work has been done by somebody.
(a) Much time was wasted on trifles. (d) Has all the work been done by somebody?
(b) Much time will be wasted on trifles. Ans: (d)
(c) Much time is wasted by us on trifles. Q60. Cigarette smoking causes two million deaths
(d) Much time is wasted on trifles. annually in the industrial states.
Ans: (c) (a) Cigarette smoking has been causing two million
Q53. Mohan gave the beggar an old shirt. deaths annually in the industrial states.
(a) An old shirt was given to Mohan by the beggar. (b) Two million deaths arc caused annually by cigarette
(b) An old shirt was given to the beggar by Mohan. smoking in the industrial states.
(c) The beggar was gave an old shirt by Mohan. (c) Two million deaths are being caused by cigarette
(d) An old shirt was gave to the beggar by Mohan. smoking annually in the industrial states.
Ans: (b) (d) Two million deaths have been caused annually by
Q54. They have made him a king. cigarette smoking in the industrial states.
Page 448 of 516
Q61. You surprise me.
https://telegram.me/aedahamlibra
Ans: (b) (a) He is told to get out of our house.
(b) Let him be told to get out of our house.
(a) I am to be surprised (c) He might be told to get out of our house.
(b) You are surprised (d) He should be told that he may get out of our house.
(c) I am surprised Ans: (b)
(d) Me is surprised Q70. Those who worked hard seldom obtained good
Ans: (c) marks.
Q62. The boys killed the snake with a stick. (a) Good marks were seldom being obtained by those
(a) The snake was killed by the boys with a stick. who worked hard.
(b) A stick was killed by the boys with a snake. (b) Good marks are seldom obtained by those who
(c) A snake with a stick was killed by the boys. worked hard.
(d) A snake is killed by the boys with a stick. (c) Seldom had good marks been obtained by those who
Ans: (a) worked hard.
Q63. Let me do this. (d) Good marks were seldom obtained by those who
(a) Let us do this. worked hard.
(b) This be done by me. Ans: (d)
(c) Let this be done by me. Q71. The accountant took the money from the customer.
(d) Let do this. (a) The money is taken from the customer by the
Ans: (c) accountant.
Q64. The tiger caught a fox. (b) The money was taken from the customer by the
(a) A fox has been caught by the tiger. accountant.
(b) A fox was caught by the tiger. (c) The customer was taken the money by the accountant.
(c) A fox is caught by the tiger. (d) The money had been taken from the customer by the
(d) A fox had been caught by the tiger. accountant.
Ans: (b) Ans: (b)
Q65. Someone has lit the fire. Q72. The peon refused him admittance.
(a) The fire was lit by someone. (a) He was refused admittance by the peon.
(b) You are requested to light the fire by someone. (b) Admittance is refused to him by the peon.
(c) The fire has been lit by someone. (c) Admittance was refused by the peon to him.
(d) The fire had been lit by someone. (d) Admittance is refused him by the peon.
Ans: (c) Ans: (a)
Q66. Has the price rise affected all the people? Q73. The reporter was interviewing the political leaders.
(a) Have all the people been affected by the price-rise ? (a) The political leaders were being interviewed by the
(b) Are all the people being affected by the price-rise ? reporter.
(c) Had all the people being affected by the price-rise ? (b) The political leaders was being interviewed by the
(d) Are all the people affected by the price-rise ? reporter.
Ans: (a) (c) The political leaders are being interviewed by the
Q67. They pick the flowers, fresh, every morning. reporter.
(a) The Fresh flowers are picked every morning by them. (d) The political leader is being interviewed by the
(b) The flowers are fresh and picked every morning by reporter.
them. Ans: (a)
(c) The flowers are picked fresh every morning by them. Q74. The beavers have built a perfect dam across the
(d) The picked flowers are fresh every morning by them. stream
Ans: (c) (a) A perfect dam had been built by the beavers across
Q68. Everyone looked up to him. the stream.
(a) He was looked up to by everyone. (b) A perfect dam has been built by the beavers across the
(b) He was looked up by everyone. stream.
(c) He is looked up by everyone. (c) A perfect dam have been built by the beavers across
(d) He looks up by everyone. the stream.
Ans: (a) (d) A perfect dam was be ing built by the beavers across
Q69. Tell him to get out of our house. the stream.
Page 449 of 516
https://telegram.me/aedahamlibra
Ans: (b)
Q75. You should follow all the instructions carefully.
(b) English speaks all over the world.
(c) The whole world speaks English.
(a) All the instructions are carefully followed by you. (d) People speak English all over the world.
(b) All the instructions were carefully followed by you. Ans: (a)
(c) All the instructions should be carefully followed by Q83. The boys elected Mohan captain.
you. (a) The boys were elected captain by Mohan.
(d) All the instructions can be carefully followed by you. (b) Mohan is elected captain by the boys.
Ans: (c) (c) Mohan was elected captain by the boys.
Q76. The school was damaged by the earthquake which (d) Mohan and the boys elected the captain.
caused havoc to other buildings as well. Ans: (c)
(a) The earthquake damaged the school and other Q84. They threw away the rubbish.
buildings. (a) The rubbish will be thrown away.
(b) The earthquake damaged other buildings. (b) The rubbish was being thrown away.
(c) The earthquake caused havoc to the school. (c) The rubbish was thrown away.
(d) The earthquake damaged the school besides causing (d) The rubbish thrown away.
havoc to other buildings. Ans: (c)
Ans: (a) Q85. Let him see the picture.
Q77. You don‘t need to wind this watch. (a) Let the picture be seen by him.
(a) This watch need not be wound. (b) The picture is seen by him.
(b) This watch does not wind. (c) Let him the picture be seen.
(c) This watch need not be wounded. (d) The picture is seen by him.
(d) This watch need not be winded up. Ans: (a)
Ans: (a) Q86. We have already done the exercise.
Q78. Has somebody broken the window ? (a) Already, the exercise has been done by us.
(a) Have the window been broken ? (b) The exercise has already been done by us.
(b) Had the window been broken by somebody ? (c) The exercise had been already done by us.
(c) Has the window been broken by somebody ? (d) The exercise is already done by us.
(d) Has been the window broken ? Ans: (b)
Ans: (c) Q87. The main skills we seek to develop include
Q79. The children are making a noise. analysing, interpreting and evaluating ideas.
(a) A noise is made by the children. (a) The main skills sought by us to develop include
(b) A noise is being made by the children. analysing, interpreting and evaluating ideas
(c) The children should be making a noise. (b) The main skills sought to be developed by us include
(d) A noise has been made by the children. analysing, interpreting and evaluating ideas
Ans: (b) (c) The main skills that we are seeking to be developed
Q80. The child‘s shrill wail broke the silence. include analysing, interpreting and evaluating ideas
(a) The silence was being broken by the child‘s shrill (d) The main skills include analysing, interpreting and
wail. evaluating ideas which are sought by us to develop
(b) The child‘s shrill wail was broken by the silence. Ans: (b)
(c) The silence was broken by the child‘s shrill wail. Q88. Who can question Gandhi‘s integrity ?
(d) The silence was being brok en by the child‘s shrill (a) By whom Gandhi‘s integrity can be questioned ?
wail. (b) By whom can Gandhi‘s integrity be questioned ?
Ans: (c) (c) Gandhi‘s integrity can be questioned by whom ?
Q81. The cat is running after the rat. (d) Who could have questioned Gandhi‘s integrity ?
(a) The rat was being run after by the cat. Ans: (b)
(b) The rat is being run after by the cat. Q89. He presented me a bouquet on my birthday.
(c) The rat is run after by the cat. (a) A bouquet is presented to me on my birthday by him
(d) The cat is being run after by the rat. (b) I was presented on my birthday a bouquet by him
Ans: (b) (c) I was presented a bouquet on my birthday by him
Q82. English is spoken all over the world. (d) I will be presented a bouquet on my birthday by him
(a) All over the world English speaks. Ans: (c)
Page 450 of 516
Q90. This surface feels smooth.
(a) This surface is felt smooth
https://telegram.me/aedahamlibra Ans: (c)
Q97. The people couldn‘t move me to the hospital and
(b) This surface is smooth when it is felt the doctor operated on me at home.
(c) This surface when felt is smooth (a) I couldn‘t be moved to the hospital and was operated
(d) This surface is smooth as felt on at home by the doctor.
Ans: (b) (b) I couldn‘t be moved to the hospital and I had to be
Q91. He asked me to finish the work in time. operated on at home.
(a) I was asked that I should finish the work in time. (c) I couldn‘t be moved to the hospital and I was operated
(b) He asked me that I should finish the work in time. at home by the doctor.
(c) I was asked to finish the work in time. (d) I couldn‘t be moved to the hospital by the people and
(d) I was asked to finish the work in time by him. operated on at home.
Ans: (d) Ans: (a)
Q92. Quinine tastes bitter. Q98. Why did he deprive you of the membership ?
(a) Quinine is bitter when it is tasted. (a) Why you were deprived of the membership ?
(b) Quinine is bitter tasted (b) Why were you deprived of his membership by him ?
(c) The taste of quinine is bitter. (c) Why was he deprived of his membership
(d) Quinine is tasted bitter. (d) Why were you deprived of your membership by him ?
Ans: (d) Ans: (d)
Q93. The vintage cars hold a special place in the hearts Q99. The news has been brought to us by him.
of their owners. (a) He brought us the news.
(a) A special place in the hearts of the vintage car owners (b) He has brought us the news.
is held by them. (c) He was brought the news to us.
(b) A special place was held by the vintage cars in the (d) We brought the news to him.
hearts of their owners. Ans: (b)
(c) A special place is held by the vintage cars in the Q100. Not a word was spoken by the criminal in self -
hearts of their owners. defence.
(d) A special place is being held by the vintage cars in the (a) The criminal spoke not a word in self-defence.
hearts of their owners. (b) The criminal in self-defence spoke no word.
Ans: (c) (c) The criminal did not speak a word in self-defence.
Q94. What amused you? (d) The criminal spoke in selfdefence not a word.
(a) What you are made to amuse by? Ans: (c)
(b) By what are you being amused?
(c) By what were you amused?
(d) By what have you been amused?
Direct/Indirect Speech
Ans: (c)
Q95. Smoke and flames engulfed the area and made Directions: In the following questions, a sentence ha s
rescue operations difficult. been given in Direct/Indirect Speech. Out of the four
(a) The area was engulfed in smoke and flames and made alternatives suggested, select the one which best
rescue operations difficult. expresses the same sentence in Indirect/Direct Speech.
(b) The area was engulfed in smoke and flames making Q1. He said, ―Ravi, why are you sounding so depressed
rescue operations difficult. today?‖
(c) The area had been engulfed in smoke and flames and (a) He asked Ravi why did he sound so depressed that
made rescue operations difficult. day.
(d) The a rea was engulfed in smoke and flames and (b) He asked Ravi why he was sounding so depressed
rescue operations were made difficult. that day.
Ans: (a) (c) He told Ravi why he sounded so depressed today.
Q96. They have made a film based on this novel. (d) He asked Ravi that why was he sounding so
(a) A film was based on this novel and made. depressed that day.
(b) A film have been made based on this novel. Ans: (b)
(c) A film, based on this novel, has been made
(d) A film has been based and made on this novel.
Page 451 of 516
?‖
https://telegram.me/aedahamlibra
Q2. He asked me, ―What time will the sun se t tomorrow (a) I asked him where he had
him yesterday.
lost the pen I had brought

(a) He asked me what time does the sun set the next day. (b) I asked him where he had lost the pen I have bro ught
(b) He asked me what time the sun would set tomorrow. for him the previous day.
(c) He asked me what time the sun would set the next (c) I asked him where he had lost the pen I had brought
day. for him the previous day.
(d) He asked me what time would the sun set the next (d) I asked him where had he lost the pen I had brought
day. him the previous day.
Ans: (c) Ans: (c)
Q3. ―Tinu, where have you been all these days ?‖ asked Q8. He said, ―Garima wants to take up a job while her
the Principal. husband wants her to look after the family.‖
(a) The Principal asked Tinu where he had been all those (a) He said that Garima wanted to take up a job while her
days. husband wanted her to look after the family.
(b) The Principal asked Tinu where he has been all those (b) He said that Garima wants to take up a job while her
days. husband wanted her to look after the family.
(c) The Principal asked Tinu where had he been all those (c) He told that Garima wanted to take up a job while her
days. husband wants her to look after the family.
(d) The Principal asked to Tinu where was he all those (d) He said to Garima that though she wanted to take up a
days. job while her husband wanted her to look after the
Ans: (a) family.
Q4. ―Have you finished your assignment, Minu ?‖ said Ans: (a)
her brother. Q9. She said, ―When I was a child, I wasn‘t afraid of
(a) Minu‘s brother asked Minu if you have finished your ghosts.‖
assignment. (a) She said that when she was a child she wasn‘t not
(b) Minu‘s brother said to Minu if she had finished her afraid of ghosts.
assignment. (b) She said that when she was a child she hadn‘t been
(c) Minu‘s brother asked her if she had finished her afraid of ghosts.
assignment. (c) She said that when she was a child she wasn‘t afraid
(d) Minu‘s brother asked Minu if she has finished her of ghosts.
assignment (d) She said tha t when she had been a child she wasn‘t
Ans: (c) afraid of ghosts.
Q5. He says, ―I go for a walk every morning.‖ Ans: (c)
(a) He says that he goes for a walk every morning. Q10. He said, ―Reena, do you want to buy a house in
(b) He said that he used to go for a walk every morning Noida ?‖
(c) He said that he goes for a walk every morning. (a) He told Reena if she wanted to buy a house in Noida.
(d) He says he went for a walk every morning. (b) He asked that did she want to buy a house in Noida.
Ans: (a) (c) He asked Reena if she wants to buy a house in Noida.
Q6. The Principal said, ―Why didn‘t you attend the flag (d) He asked Reena if she wanted to buy a house in
hoisting ceremony, Anuj ?‖ Noida.
(a) The Principal asked Anuj why hadn‘t he attended the Ans: (d)
flag hoisting ceremony. Q11. I said to my friend, ―Can I borrow your dictionary
(b) The Principal asked to Anuj why he hadn‘t attended for one day?‖
the flag hoisting ceremony. (a) I asked my friend if I could borrow his dictionary for
(c) The Principal asked Anuj why didn‘t he attend the one day.
flag hoisting ceremony. (b) I aske d my friend if I could borrow your dictionary
(d) The Principal asked Anuj why he didn‘t attend the for one day.
flag hoisting ceremony. (c) I asked my friend if I can borrow his dictionary for
Ans: (d) one day.
Q7. I said to him, ―Where have you lost the pen I brought (d) I asked my friend that if I can borrow his dictionary
for you yesterday ?‖ for one day.
Ans: (a)
Page 452 of 516
care of him‖
https://telegram.me/aedahamlibra
Q12. He said, ―Let Hari come with us, mother. I‘ll take (c) The pilot tol d the passengers
tighten your seat belts.
not to panic but to

(a) He requested his mother to let Hari come with them as (d) The pilot told the passengers not to panic but to
he will take care of him. tighten their seat belts.
(b) He informed his mother to let Hari come with them as Ans: (d)
he would take care of him. Q17. He said, ―Ravi, why are you sounding so depressed
(c) He told his mother to let Hari come with them as he today?‖
would take care of him. (a) He asked Ravi why did he sound so depr essed that
(d) He told to his mother let Hari come with us as he day.
would take care of him. (b) He asked Ravi why he was sounding so depressed
Ans: (c) that day.
Q13. ―No,‖ said the child, ―I won‘t kneel, for if I do, I (c) He told Ravi why he sounded so depressed today.
shall spoil my new pair of trousers.‖ (d) He asked Ravi that why was he sounding so
(a) The child said that he would not kneel for if he did so depressed that day.
he will spoil his new pair of trousers. Ans: (b)
(b) The child said that he will not kneel for if he kneels Q18. ―I shall remain here and the tailor won‘t be ab le to
he will spoil his new pair of trousers. find me,‖ said she.
(c) The child said that he would not kneel for if he did so (a) She said that she should remain there and the tailor
he would spoil his new pair of trousers. won‘t be able to find me.
(d) The child said that I will not kneel for if he did kneel, (b) She said that she should remain there and the tailor
he should spoil his new pair of trousers. would not be able to find her.
Ans: (c) (c) She said that she would remain there and the tailor
Q14. The chairman of the selection committee said, ―We would not be able to find her.
shall finalise the rest of our team after we have selected (d) She said that, she could remain here and the tailor
the skipper.‖ would not find her.
(a) The chairman of the selection committee told that Ans: (c)
they would finalise the rest of our team after we have Q19. Mother said, ―Gaurav, you will be eligible for
selected the skipper. voting when you are 18.‖
(b) The chairman of the selection committee said that we (a) Mother told Gaurav he would be eligible for voting
would finalise the rest of our team after we have selected when he was 18.
the skipper. (b) Mother told Gaurav that he could vote only after 18.
(c) The chairman of the select ion committee said that (c) Mother told Gaurav you will be eligible for voting
they would finalise the rest of their team after they when you are 18.
selected the skipper. (d) Mother told Gaurav that he would be eligible for
(d) The chairman of the selection committee said that voting when he would be 18.
they would finalise the rest of their team after they had Ans: (a)
selected the skipper. Q20. He said, ―Bravo You have done well.‖‘
Ans: (d) (a) He applauded him to say that he had done well.
Q15. He said, ―Has anybody been unkind to you ?‖ (b) He applauded him and said that you have done well.
(a) He asked me if anybody had been unkind to me. (c) He applauded him, saying that he has done well.
(b) He asked me had anybody been unkind to me. (d) He applauded him, saying that he had done well.
(c) He asked me if anybody had been unkind to you. Ans: (d)
(d) He asked me if anybody had been unkind to him. Q21. He said to me, ―You are getting lazy day by day.‖
Ans: (a) (a) He informed me that I am getting lazy day by day.
Q16. The pilot said, ―Please don‘t panic but tighten your (b) He told me that I have been getting lazy day by day.
seat belts.‖ (c) He told me that I was getting lazy day by day.
(a) The pilot told to the passengers that they should not (d) He told me that you were getting lazy day by day.
panic but tighten the seat belts. Ans: (c)
(b) The pilot told the passengers to not panic but to Q22. ―When will I be able to vote?‖ I asked my mother.
tighten their seat belts instead. (a) I asked my mother when would he be able to vote.
(b) I asked to my mother when I will be able to vote.
Page 453 of 516
https://telegram.me/aedahamlibra
(c) I asked my mother when I would be able to vote.
(d) I asked my mother when would I be able to vote.
(c) He asked us why we were all sitting about there doing
nothing.
Ans: (c) (d) He asked us why were we all sitting about there doing
Q23. I said, ―Father, when will you buy me a motor cycle nothing.
?‖ Ans: (c)
(a) I asked my father when will he buy me a motorcycle. Q28. ―May you live long and prosper‖, said the old lady
(b) I asked my father when he will buy me a motor cycle. to her son.
(c) I asked my father when would he buy me a motor (a) The old lady blessed her son with long life and wished
cycle. him prosperity.
(d) I as ked my father when he would buy me a motor (b) The old lady prayed for her son‘s long life and
cycle. prosperity.
Ans: (d) (c) The old lady prayed for her son and said that he might
Q24. He said to me, ―Is there any possibility of my live long and prosper.
getting promotion this year?‖ (d) The old lady blessed her son and prayed for his long
(a) He asked me if there was any possibility of my getting life and prosperity.
promotion this year. Ans: (c)
(b) He asked me if there was any possibility of his getting Q29. ―Many happy returns of your birthday‖, we said.
promotion this year. (a) We greeted him on his birthday.
(c) He asked me if there was any possibility of his getting (b) We said that many happy returns of your birthday.
promotion that year.‖ (c) We wished him many happy returns of his birthday.
(d) He asked me if there is any possibility of his getting (d) We prayed for many happy returns of his birthday.
promotion this year. Ans: (c)
Ans: (c) Q30. ―Fie a soldier and afraid of fighting ‖ said Babar.
Q25. The dealer said, ―Either make your purchases or (a) Babar exclaimed with surprise that a soldier should be
walk out of my shop.‖ afraid of fighting.
(a) The dealer told the customer that he would either (b) Addressing a soldier, Babar cursed him that he should
make his purchases or walk out of his shop. be afraid of fighting.
(b) The dealer ordered the customer to make his (c) Babar cursed the soldier who was afraid of fighting.
purchases and walk out of his shop. (d) Babar cursed and exclaimed that a soldier should not
(c) The dealer told the customer that he should either be afraid of fighting.
make his purchases, or walk out of his shop. Ans: (d)
(d) The dealer requested the customer to make his Q31. He said to Sita, ―When do you intend to pay back
purchases or walk out of his shop. the money you borrowed from me ?‖
Ans: (c) (a) He enquired Sita when did she intend to pay back the
Q26. He said, ―I do not wish to see any of you; go away.‖ money she borrowed from him.
(a) He said that he had not wished to see any of them and (b) He enquired Sita when she intended to pay back the
ordered them to go away. money she borrowed from him.
(b) He said that he did not wish to see any of them and (c) He enquired Sita when she intended to pay back the
ordered them to go away. money she had borrowed from him.
(c) He told that he did not wish to see any of them and (d) He enquired when Sita intended to pay back the
ordered them to go away. money she borrowed from him.
(d) He said that he does not wish to see any of us an d Ans: (c)
ordered us to go away. Q32. ―So help me Heaven ―he cried, ―I will never steal
Ans: (b) again‖.
Q27. He said to us, ―Why are you all sitting about there (a) He wanted Heaven to help him so that he would never
doing nothing ?‖ steal again.
(a) He asked us why are we all sitting about there doing (b) He exclaimed to Heaven that he would never steal
nothing again.
(b) He asked us why we are all sitting about there doing (c) He exclaimed Heaven to help him that he would never
nothing. steal again.

Page 454 of 516


steal again.
https://telegram.me/aedahamlibra
(d) He called upon Heaven to witness his resolve never to (c) The Chief Minister exclaimed with sorrow that the
earthquake was a terrible disaster.
Ans: (d) (d) The Chief Minister told in pain that the earthquake
Q33. She said, ―I'll give you 100 to keep your mouth was a terrible disaster.
shut.‖ Ans: (c)
(a) She said that she would give him 100 to keep his Q38. I said to her, ―All your faults will be pardoned if
mouth shut. you confess them.‖
(b) She offered him 100 to keep his mouth shut. (a) I told her that all her faults will be pardoned if you
(c) She suggested to him that she would give him 100 to confess them.
keep his mouth shut. (b) I told her that all her faults would be pardoned if she
(d) She proposed that she would give him 100 to keep his confessed them.
mouth shut. (c) I told her that all her faults will have to be pardoned if
Ans: (d) she confessed them.
Q34. ―Doctor‖, the patient cried, please tell me how (d) I told her that all her faults would have to be pardoned
much time I have.‖ if she confessed them.
(a) The patient pleaded the doctor how much time she Ans: (b)
will have. Q39. The teache r said to the student, ―Why were you
(b) The patient exclaimed to the doctor that how much absent yesterday?‖
time she had. (a) The teacher asked the student why had been he absent
(c) The patient pleaded with the doctor to tell her how the previous day.
much time she had. (b) The teacher asked the student why had he been absent
(d) The patient cried to the doctor that how much ti me the previous day.
had she. (c) The teacher informed the student that why he h ad
Ans: (c) been absent the next day.
Q35. ―Let‘s give a party‖, said Jaya. (d) The teacher asked the student why he had been absent
―Let‘s not,‖ said her husband. the previous day.
(a) Jaya suggested giving a party but her husband Ans: (d)
answered in the negative. Q40. ‗From one of the windows of my flat I can see the
(b) Jaya suggested giving a party but her husband Qutub Minar‘, said my friend.
opposed the idea. (a)My friend told that from one of the windows of his flat
(c) Jaya suggested giv ing a party but her husband he could see the Qutub Minar.
contradicted her. (b) My friend said that from one of the windows of his
(d) Jaya suggested giving a party but her husband flat he can see the Qutub Minar.
refused. (c) My friend said that from one of the windows of his
Ans: (b) flat he could see the Qutub Minar.
Q36. He said to her, ―Did it shake you up as much as it (d)My friend told that from one of the windows of his flat
shook me ?‖ he can see the Qutub Minar.
(a) He asked her if it had shaken her up as much as it Ans: (c)
shook him. Q41. Shanker said to his friend, ―Look out there is a
(b) He enquired of her if it shook her up as it shook him. snake behind you.‖
(c) He enquired of her if it shook her up as much as it had (a) Shanker told his friend to look out as there was a
shaken him. snake behind him.
(d) He asked her if it had shaken her up as it had shaken (b)Shanker exclaimed to his friend with surprise that
him. there was a snake behind him.
Ans: (d) (c) Seeing a snake behind him, Shanker asked his friend
Q37. The Chief Minister said, ―What a disaster the to look out.
earthquake is ‖ (d)Shanker warned his friend that there was a snake
(a) The Chief Minister cried that the earthquake was a behind him.
terrible disaster. Ans: (b)
(b) The Chief Minister expressed with surprise that the Q42. The teacher said to Ram, ―Would you like to go on
earthquake was a terrible disaster. a trip to Kashmir ?‖ Ram said, ―Yes‖
Page 455 of 516
Kashmir and Ram replied yes.
https://telegram.me/aedahamlibra
(a)The teacher asked if Ram would like to go on a trip to (c) The judge commanded that the first witness be called.
(d) The judge commanded the first witness to be called.
(b)The teacher asked Ram if he would like to go on a trip Ans: (a)
to Kashmir and Ram replied yes. Q48. The queen said to her son, ―You must go to the
(c)The teacher asked Ram if he would have liked to go on forest and remain there till your father calls you back.‖
a trip to Kashmir and Ram replied in the affirmative. (a) The queen ordered her son to go to the forest and
(d)The teacher asked Ram if he would like to go on a trip remain there till his father calls him back.
to Kashmir and Ram replied in the affirmative. (b) The queen told her son that he must go to the forest
Ans: (d) and remain there till his father called him back.
Q43. ―I Suppose, you children go out for a nice long (c) The queen told his son that you must go to the forest
walk‖, she said. and remain there till your father called you back.
(a) She proposed that the children went out for a nice (d) The queen told her son to go to the forest and
long walk. remained there till his father called him back.
(b) She advised that the children go out for a nice long Ans: (b)
walk. Q49. I said, ―When it gets dark, light the lantern and
(c) She suggested that the children go out for a nice long hang it out.‖
walk. (a) I said that when it got dark he light the lantern and
(d) She suggested that the children should go out for a hung it out.
nice long walk. (b) I requested that when it got dark he may light the
Ans: (d) lantern and hang it out.
Q44. Everybody said, ―How well she sings ‖ (c) I said that when it got dark he should light the lantern
(a)Everybody told us that she sings very well. and hang it out.
(b)Everybody exclaimed that she sings very well. (d) I told that when it got dark he should light the lantern
(c)Everybody exclaimed that she sang very well. and hang it out.
(d) Everybody told us that how she sang very well. Ans: (c)
Ans: (c) Q50. The teacher said to Ashok, ―I shall report the matter
Q45. The old man of Latur sai d, ―Alas, my only son is to the Principal if you misbehave again.‖
dead ‖ (a) The teacher told Ashok that he would be reporting the
(a)The old man of Latur exclaimed with sorrow that his matter to the Principal if he misbehaved again.
only son was dead. (b) The teacher warned Ashok that he would report the
(b) The old man of Latur told with pain that his only son matter to the Principal if he misbehaved again.
was dead. (c) The teacher threatened Ashok to report the matter to
(c)The old man of Latur expressed with anguish that his the Principal if he would misbehave again.
only son was dead. (d) The teacher cautioned Ashok about reporting the
(d) The old man of Latur sorrowfully narrated that his matter to the Principal if he misbehaved again.
only son was dead. Ans: (b)
Ans: (a) Q51. ―Mohan made this mess. Let him clear it up‖, said
Q46. I said, ―Let Mohan do his worst, he cannot harm his father.
me.‖ (a) Mohan‘s father said that Mohan had made the mess
(a) I suggested to Mohan to do his worst, he could not and proposed that he clear it up.
harm me. (b) Mohan‘s father said t hat Mohan had made the mess
(b) I told that if Mohan did his worst, he could not harm and that he was to clear it.
me. (c) Mohan‘s father said that Mohan had made the mess
(c) I declared that Mohan might do his worst, he could and suggested that he clear it.
not harm me. (d) Mohan‘s father said that Mohan had made the mess
(d) I declared that though Mohan might do his worst, he and that he should clear it.
could not harm me. Ans: (d)
Ans: (d) Q52. ―Do as you wish, but don‘t come and ask me for
Q47. ―Call the first witness‖, said the judge. help if you get into difficulties.‖
(a) The judge commanded to call the first witness. (a) He told me to do as he wished or he wouldn‘t come
(b) The judge commanded them to call the first witness. and help me if I got into difficulties.
Page 456 of 516
https://telegram.me/aedahamlibra
(b) He told me to do as I wished, but not to go and ask
him for help if I got into difficulties. Q58. I said to my friend, ―Good Morning.
Ans: (c)

(c) He ordered me to do as I wished, but not to go and ask Let us go for a picnic today.‖
him for help if, I got into difficulties. (a) I told good morning to my friend and asked to go for a
(d) He told me that unless I did as I wished he would not picnic that day.
come and help me if I got into difficulties. where had he (b) I wished my friend good morning and proposed tha t
been all those we should go for a picnic that day.
Ans: (b) (c) I wished my friend good morning and proposed that
Q53. Socrates said, ―Virtue is its own reward.‖ they should go for a picnic that day.
(a) Socrates said that virtue had its own rewards. (d) I told good morning to my friend and suggested to go
(b) Socrates says that virtue is its own reward. for a picnic today.
(c) Socrates said that virtue is its own reward. Ans: (b)
(d) Socrates said that virtue was its own reward. Q59. The new student asked the old one, ―Do you know
Ans: (c) my name?‖
Q54. He said to the interviewer, ―Could you please repeat (a) The new student asked the old one if he knew his
the question?‖ name.
(a) He requested the interviewer if he could please repeat (b) The new student asked the old one that whether he
the question. knew his name.
(b) He requested the interviewer to please repeat the (c) The new student asked the old one did he know his
question. name
(c) He requested the interviewer to repeat the question. (d) The new student asked the old one if he kn ows his
(d) He requested the interviewer if he could repeat the name
question. Ans: (a)
Ans: (c) Q60. I warned her that I could no longer tolerate her
Q55. He said, ―It used to be a lovely, quiet street.‖ coming late.
(a) He said that it used to be a lovely, quiet street. (a) I said to her, ―You can no longer tolerate my coming
(b) He pointed out that it had used to be a lovely, quiet late.‖
street. (b) I said to her, ―I can no longer tolerate your coming
(c) He said that there used to be a lovely, quiet street. late.‖
(d) He inquired whether there was a lovely, quiet street. (c) I said to her, ―He can no longer tolerate her coming
Ans: (a) late.‖
Q56. The Prime Minister said that no one would be (d) I said to her, ―I can no longer tolerate she coming
allowed to disturb the peace. late.‖
(a) The Prime Minister said, ―We shall not allow any one Ans: (b)
to disturb the peace.‖ Q61. I said to my mother, ―I will certainly take you to
(b) The Prime Minister said, ―We would not allow no one Bangalore this week.‖
to disturb the peace (a) I told my mother he would certainly take her to
(c) The Prime Minister said, ―No one will disturb the Bangalore that week.
peace.‖ (b) I to ld my mother that I would certainly take her to
(d) The Prime Minister said, ―No one can disturb the Bangalore that week.
peace.‖ (c) I told my mother that she would certainly take her to
Ans: (a) Bangalore that week.
Q57. The spectators said, ―Bravo Well done players.‖ (d) I told to my mother that I would take you to
(a) The spectators shouted that the players were doing Bangalore that week.
very well. Ans: (b)
(b) The spectators exclaimed with joy that the players Q62. ―How long does the journey take‖ my co-passenger
were doing very well. asked me.
(c) The spectators applauded the players saying that they (a) My co-passenger asked me how long does the journey
had done well. take?
(d) The spectat ors applauded the players joyfully to do (b) I asked my co -passenger how long the journey would
well. take.
Page 457 of 516
journey would take.
https://telegram.me/aedahamlibra
(c) My co -passenger wanted to know how long the
Q69. The boss said, ―It‘s time we began planning our
Ans: (a)

(d) My co-passenger asked me how long the journey did work‖.


take. (a) The boss said that it was time they had begun
Ans: (c) planning their work.
Q63. ―How clever of you to have solved the puzzle so (b) The boss said that it was time we had begun planning
quickly,‖ said the mother. our work.
(a) The mother exclaimed admiringly that it was very (c) The boss said that it was time they began planning
clever of him to have solved the puzzle so quickly. their work.
(b) The mother expressed that he was so clever to have (d) The boss said that it was time we began planning his
solved the puzzle quickly. work.
(c) The mother told that he was very clever in solving the Ans: (a)
puzzle so quickly. Q70. He said to the judge, ―I did not commit this crime.‖
(d) The mother exclaimed with joy that he was clever (a) He told the judge that he did not commit the crime.
enough to solve the puzzle so quickly. (b) He told the judge that he had not committed the
Ans: (a) crime.
Q64. He said he goes for a walk every morning. (c) He told the judge that he had not committed that
(a) He said, ―I went for a walk every morning.‖ crime.
(b) He said, ―I go for a walk every morning.‖ (d) He told the judge that he had not committed this
(c) He said, ―I will go for a walk every morning.‖ crime.
(d) He said, ―He goes for a walk every morning.‖ Ans: (c)
Ans: (b) Q71. Rahul said, ―I will do it now or never‖.
Q65. I reiterated, ―I don‘t care about the job.‖ (a) Rahul said that he will do it now or never.
(a) I reiterated I didn‘t care about the job. (b) Rahul said that he will now or never do it.
(b) I said again and again I didn‘t care about the job. (c) Rahul said that he would do it then or never
(c) I reiterated that I did not care about the job. (d) Rahul said that he would now or never do it.
(d) I repeatedly said that I cared about the job. Ans: (c)
Ans: (c) Q72. My father once said to me, ―If I can‘t trust my
Q66. I said to my brother, ―Let us go to some hill station people, then I don‘t want to be doing this.‖
for a change.‖ (a) His father once told him that if he couldn‘t trust his
(a) I suggested to my brother that they should go to some people then he didn‘t want to be doing that.
hill station for a change. (b) My father once told me that if he couldn‘t t rust his
(b) I suggested to my brother that we should go to some people then he didn‗t want to be doing that.
hill station for a change. (c) My father once told me that if he couldn‘t trust my
(c) I suggested to my brother that let us go to some hill people then he didn‗t want to be doing that.
station for a change. (d) My father once told me that if he couldn‘t trust his
(d) I suggested to my brother that let them go to some hill people then he didn‘t want to be doing this.
station for a change. Ans: (b)
Ans: (b) Q73. Doshi said to his wife, ―Please select one of these
Q67. I wondered how many discoveries went unheeded. necklaces.‖
(a) I said, ―How many discoveries have gone unheeded?‖ (a) Doshi requested his wife to select one of those
(b) I said, ―How many discoveries went unheeded?‖ necklaces.
(c) I said, ―Do discoveries go unheeded?‖ (b) Doshi said to his wife to please select one of these
(d) I said, ―How many discoveries go unheeded?‖ necklaces.
Ans: (d) (c) Doshi told his wife to please select one o f those
Q68. Gopan said to me, ―Can you do these sums for necklaces.
me?‖ (d) Doshi pleased his wife to select one of those
(a) Gopan asked me if I could do those sums for him. necklaces.
(b) Gopan asked me if I can do those sums for him. Ans: (a)
(c) Gopan asked me if I can do these sums for him. Q74. He wrote in his report, ―The rainfall has been
(d) Gopan asked if I could do these sums for him. scanty till now.‖
Page 458 of 516
https://telegram.me/aedahamlibra
(a) He reported that the rainfall has been scanty till now
(b) He reported that the rainfall had been scanty till now. Q81. The teacher said to the student ―Can you explain
Ans: (c)

(c) He reported that the rainfall has been scanty till then. this ?‖
(d) He reported that the rainfall had been scanty till then. (a) The teacher asked the student whether he can explain
Ans: (d) this.
Q75. ―Govind,‖ said the manager sternly, ―I command (b) The teacher asked the student whether he can explain
you to tell me what the old man said.‖ that.
(a) The mana ger sternly told Govind that he commanded (c) The teacher asked the student whether he could
him to tell him what the old man had said. explain this.
(b) The manager commanded sternly to Govind to tell (d) The teacher asked the student whether he could
him what the oldman had said. explain that.
(c) The manager commanded Govind to tell him what the Ans: (d)
old man said. Q82. Ramu said, ―My master is planning to build a huge
(d) The manager stern ly commanded Govind to tell him house in Khandala.‖
what the old man had said. (a) Ramu said that his master planned to build a huge
Ans: (d) house in Khandala.
Q76. I said to him, ―Where have you lost the pen I (b) Ramu said that his master is planning to build a huge
brought for you yesterday ?‖ house in Khandala.
(a) I asked him where he had lost the pen I had brought (c) Ramu said that his master‘s plan is to build a huge
him yesterday. house in Khandala.
(b) I asked him where he had lost the pen I have brought (d) Ramu said that his master was planning to build a
for him the previous day. huge house in Khandala.
(c) I asked him where he had lost the pen I had brought Ans: (d)
for him the previous day. Q83. Nandita said to Nitin, ―Can you solve this sum ?‖
(d) I asked him where had he lost the pen I had brought (a) Nandita asked Nitin if could he solve this sum.
him the previous day. (b) Nandita asked Nitin if could he solve that sum.
Ans: (c) (c) Nandita asked Nitin if he could solve this sum.
Q77. She said to Rita, ―Please help me with my (d) Nandita asked Nitin if he could solve that sum.
homework.‖ Ans: (c)
(a) She requested Rita to help her in her homework. Q84. He said, ―How lovely the scene is ‖
(b) Rita requested her to help her with her homework. (a) He exclaimed that the scene is very lovely.
(c) She requested Rita to help her with her homework. (b) He exclaimed that how the scene was lovely.
(d) She requested Rita to help her homework. (c) He exclaimed that the scene was very lovely.
Ans: (c) (d) He exclaimed that the scene is indeed lovely.
Q78. ―What a stupid fellow you are ‖ she remarked. Ans: (c)
(a) She exclaimed that what a stupid fellow he was. Q85. Bill said, ‗I am here to help you all.‖
(b) She exclaimed that he was a very stupid fellow. (a) Bill said that he is here to help you all.
(c) She exclaimed in disgust how he could be so stupid. (b) Bill said that he was there to help us all.
(d) She asked him what a stupid fellow he was. (c) Bill said that he was here to help us all.
Ans: (b) (d) Bill said that he is there to help us all.
Q79. The police said to the thief, ―Don‘t move.‖ Ans: (b)
(a) The police ordered the thief not to move. Q86. The kids yelled in a loud voice, ―We love our
(b) The police ordered the thief that he should not move. family.‖
(c) The police told the thief that he did not move. (a) The kids yelled loudly that they love our family.
(d) The police ordered to the thief to not move. (b) The kids yelled in a loud voice that they loved their
Ans: (a) family.
Q80. He said, ―I am going to college just now.‖ (c) The kids yelled in a loud voice that they loved our
(a) He said that he was going to college just now. family.
(b) He asked that he was going to college just then. (d) The kids were shouting loudly that they loved their
(c) He said that he was going to college just then. family.
(d) He asked that he was going to college just now. Ans: (b)
Page 459 of 516
tomorrow‖.
https://telegram.me/aedahamlibra
Q87. He said to me, ―Please visit my company (c) He asked me if I have finished reading the magazine.
(d) He asked me whether I finished reading the magazine.
(a) He said to me to visit his company tomorrow. Ans: (a)
(b) He requested me to visit his company tomorrow. Q94. He said, ―We have done our work.‖
(c) He requested me to visit his company the next day. (a) He said that he had done his work.
(d) He said to me to visit his company the next day. (b) He said that they have done their work.
Ans: (c) (c) He said that they had done their work.
Q88. He said, ―My parents are arriving tomorrow.‖ (d) He said that they should do their work.
(a) He said that his parents are arriving the next day. Ans: (c)
(b) He said that his parents were arriving the next day. Q95. He said that he had been waiting there for a long
(c) He said that his parents are arriving tomorrow. time.
(d) He said that his parents have arrived the next day. (a) He said, ―I have been waiting here for a long time.‖
Ans: (b) (b) He said, ―He has been waiting here for a long time.‖
Q89. He said, ―How cruel of him ‖ (c) He said, ―He has been waiting there for a long time.‖
(a) He remarked that it was very cruel of him. (d) He said, ―I am waiting here for a long time.‖
(b) He remarked on his great cruelty. Ans: (a)
(c) He remark that it was very cruel of him. Q96. ―Please, take me to the officer,‖ said the visitor.
(d) He remarked that it is very cruel of him. (a) The visitor requested them to take him to the officer.
Ans: (a) (b) The visitor told them to take him to the officer.
Q90. ―How did it get here ?‖ She wanted to know. (c) The visitor requested for the officer to be taken.
(a) She wanted to know how did it get here. (d) The visitor wanted the officer to take him there.
(b) She wanted to know how it had got there. Ans: (a)
(c) She wanted to know how did it got there. Q97. Nitin said, ―What a pleasant atmosphere this place
(d) She wanted to know how it get there. has ‖
Ans: (b) (a) Nitin exclaimed with joy that place was a pleasant
Q91. ―You are one of Pinkerton‘s men, I presume,‖ he atmosphere.
said. (b) Nitin exclaimed with joy that was a pleasant
(a) He said that he presumed me one of Pinkerton‘s men. atmosphere in that place.
(b) He said that he presumed that I was one of (c) Nitin exclaimed with joy that that place had a pleasant
Pinkerton‘s men. atmosphere.
(c) He said that he presumed me as one of Pinkerton‘s (d) Nitin exclai med with joy that what a pleasant
men. atmosphere that place had.
(d) He said that he presumes that I am one of Pinkerton‘s Ans: (c)
men. Q98. Aarati said, ―1 am in a hurry today.‖
Ans: (c) (a) Aarati said that she was in a hurry that day.
Q92. ―Away ‖ she said to the man, ―do not trouble your (b) Aarati said that I am in a hurry today.
family anymore‖. (c) Aarati said that she is in a hurry today.
(a) She asked the man to go away and not trouble the (d) Aarati said that she is in a hurry that day.
family any more. Ans: (a)
(b) She asked the man not to trouble the family that was Q99. They said, ―We are practising for the next match.‖
away, any more. (a) They said that they are practising for the next match.
(c) S he asked the man not to trouble your family any (b) They said that they practised for the next match.
more and go away. (c) They said that they were practising for the next match.
(d) She asked the man to go away and not trouble his (d) They said they were going to practise for the next
family any more. match.
Ans: (d) Ans: (c)
Q93. He asked me, ―Have you finished reading the Q100. He said, ―Long may you live.‖
magazine ?‖ (a) He suggested that my life might be long.
(a) He asked me whether I had finished reading the (b) He prayed that my life might be long.
magazine. (c) He prayed that my life may be longer.
(b) He asked me if had I finished reading the magazine. (d) He declared that my life is longer.
Page 460 of 516
https://telegram.me/aedahamlibra
Ans: (b) (c) indifferently
(d) indiscriminately
Ans: (d)
Fill In The Blanks Q9. We are happy to.......the receipt of your order No
4071 dated 13.3.96.
Directions: In these questions, sentences are given with (a) admit (b) accept
blanks to be filled in with an appropriate word (s). Four (c) acknowledge (d) respond
alternatives are suggested for each question. Ans: (c)
Choose the correct alternative out of the four alternatives. Q10. All jobs are respectable.......of their nature.
Q1. The flash of the torch.......a cobra. (a) irrelevant
(a) exposed (b) displayed (b) immaterial
(c) disclosed (d) revealed (c) irresponsible
Ans: (d) (d) irrespective
Q2. He.......himself a stiff drink before making his Ans: (d)
statement to the police officer. Q11. Nadheeka was musing ......
(a) threw (b) poured memories of the past.
(c) filled (d) sipped (a) over (b) about
Ans: (b) (c) on (d) from
Q3. It was his.......that led him to penury. Ans: (a)
(a) flamboyance Q12. The lame boy tried to climb up the staircase without
(b) arrogance ...... help.
(c) extravagance (a) little (b) any
(d) ebullience (c) some (d) many
Ans: (c) Ans: (b)
Q4. Gita was known to be a.......so nobody entrusted any Q13. Each school has its own set of rules ......all good
important work to her. pupils should follow them.
(a) joker (b) worker (a) but (b) or
(c) shocker (d) shirker (c) so (d) and
Ans: (d) Ans: (d)
Q5. The statue was so.......that people stared at it in Q14. On my return from a long holiday, I had to ......with
horror. a lot of work.
(a) grotesque (b) exquisite (a) catch on (b) catch up
(c) beatific (d) cumbersome (c) make up (d) take up
Ans: (a) Ans: (b)
Q6. He was caught red -handed and could not.......the Q15. Only one of the boys ......not done the home -work
charges. given yesterday.
(a) refute (b) refuse (a) have (b) has
(c) rebuke (d) revoke (c) can (d) could
Ans: (a) Ans: (b)
Q7. Unhappy about the treatment meted out to her, Q16. Hopes of a settlement depends on the ......of the
Shanti....... discussion.
demanded justice. (a) findings
(a) sumptuously (b) outcome
(b) voraciously (c) break through
(c) spasmodically (d) resolutions
(d) vociferously Ans: (b)
Ans: (d) Q17. We must avail ourselves ......every opportunity that
Q8. The police sprayed tear gas.......on the protesters. comes ......our way.
(a) indirectly (a) of ; in (b) on ; in
(b) intensively (c) against ; for (d) to ; to
Ans: (a)
Page 461 of 516
(a) act
https://telegram.me/aedahamlibra
Q18. It is time we ......with determination.
(b) acted
(c) will be higher your bill
(d) the higher your bill will be
(c) have acted (d) will act Ans: (d)
Ans: (b) Q28. The rich parents decided to send their two sons to
Q19. When the thief entered the house, the inmates ....in Eton, the stronghold of the.....of the rich.
the hall. (a) parents (b) ancestry
(a) were slept (c) pedigree (d) progeny
(b) were sleeping Ans: (d)
(c) slept Q29. She stood.....the crowd quite amazed.
(d) had been sleeping (a) in (b) within
Ans: (b) (c) among (d) into
Q20. 700 men worked for 10 years to......the Borobudur Ans: (a)
temple in Java to its former glory. Q30. The magnificent.....temple was constructed by the
(a) restore (b) give Chinese.
(c) create (d) revive (a) eight-centuries-old
Ans: (a) (b) eight-century‘s-old
Q21. The thief explained how he ....... (c) old-eight centuries
from the jail. (d) eight-century-old
(a) escapes Ans: (d)
(b) had escaped Q31. There were two small rooms in the beach house,
(c) escaped .....served as a kitchen.
(d) has escaped (a) the smaller of which
Ans: (b) (b) the smallest of which
Q22. He held ...... to the books passionately. (c) the smaller of them
(a) on (b) in (d) smallest of that
(c) off (d) away Ans: (c)
Ans: (a) Q32. Madhu has not been able to recall where.....
Q23. They did not know where they ....... from. (a) does she live.
(a) had come (b) have come (b) she lived.
(c) has come (d) come (c) did she live.
Ans: (a) (d) lived the girl.
Q24. There is only one member who has not paid his ...... Ans: (b)
and he has promised to pay before the end of this month. Q33. You.....mad if you think I‘m going to show my
(a) remittance (b) honorarium answer-sheet.
(c) subscription (d) allowance (a) are supposed to be
Ans: (c) (b) must be
Q25. He was an ...... swimmer, but was swept away by a (c) will be
strong ...... (d) ought to be
(a) eligible; water Ans: (b)
(b) able; stream Q34. If I hadn‘t come along at that moment, Rahim.....the
(c) old; river one arrested instead of the real thief.
(d) expert; current (a) might been
Ans: (d) (b) may have been
Q26. The teacher gives many examples to....the idea (c) can have been
contained in the poem. (d) could have been
(a) bring about (b) bring in Ans: (d)
(c) bring forth (d) bring out Q35. Wheat is not native to India and barley.....
Ans: (d) (a) isn‘t either (b) is either
Q27. The more electricity you use, ..... (c) is neither (d) isn‘t neither
(a) your bill will be higher Ans: (a)
(b) will your bill be higher Q36. It‘ll rain soon, ............ ?
Page 462 of 516
(a) won‘t it
(c) isn‘t it
https://telegram.me/aedahamlibra
(b) bought it
(d) may it
Q46. A person who ________ on his own effort has the
best chance to win.
Ans: (a) (a) makes (b) puts
Q37. My book is the new one; ............ is the torn one. (c) relies (d) runs
(a) your Ans: (c)
(b) the book of you Q47. When he died, Ramanujan ________ behind 3
(c) yours notebooks.
(d) the book your (a) was leaving (b) had left
Ans: (c) (c) left (d) leaves
Q38. They needn‘t worry, ...........? Ans: (b)
(a) isn‘t it (b) doesn‘t it Q48. For all his ________, he is a _______.
(c) don‘t it (d) need they (a) wealth, miser
Ans: (d) (b) health, player
Q39. He is indifferent ............ praise and blame. (c) tricks, cheat
(a) in (b) to (d) stupidity, fool
(c) for (d) about Ans: (a)
Ans: (b) Q49. In a classroom students are to be trained to love
Q40. King George V‘s accession ............ the throne was _______.
celebrated with great pomp. (a) each other (b) all others
(a) for (b) with (c) one another (d) altogether
(c) against (d) to Ans: (c)
Ans: (d) Q50. Mr. Obi put his whole life_______ the work.
Q41. We are all very indignant ........... the injustice done (a) on (b) in
to him. (c) into (d) upon
(a) for (b) of Ans: (b)
(c) to (d) at Q51. All decisions, ____________, are likely to be taken
Ans: (d) by consensus.
Q42. If she ............ a bird, she would fly. (a) anyhow (b) forever
(a) is (b) are (c) somehow (d) however
(c) were (d) was Ans: (d)
Ans: (c) Q52. Kumar _________ boy swam across the sea for
Q43. The greater the demand, ............ the price. safety.
(a) higher (b) high (a) a 17-years-old
(c) the higher (d) the high (b) a 17-year-old
Ans: (c) (c) a 17-year-aged
Q44. Since El izabeth Barrett Browning‘s father never (d) the 17-year-old
approved of ............ Robert Browning, the couple eloped Ans: (b)
to Italy where they lived and wrote. Q53. She is beautiful ______ brave.
(a) her to marry (a) but (b) and
(b) her marrying (c) yet (d) so
(c) she marrying Ans: (b)
(d) she to marry Q54. Gandhiji will _________ in history as one of the
Ans: (b) greatest men, that ever lived.
Q45. Canada does not require that U.S. citizens obtain (a) go by (b) go on
passports to enter the country, and ............ (c) go down (d) go through
(a) Mexico does neither Ans: (c)
(b) Mexico doesn‘t either Q55. All the appl icants _______ and the results will be
(c) neither Mexico does announced next week.
(d) either does Mexico (a) rank
Ans: (b) (b) are to rank
(c) are to be ranked
Page 463 of 516
(d) are ranking https://telegram.me/aedahamlibra
Ans: (c)
Q66. She was relieved to hear from the doctor t hat the
tumour was not malignant; it was ____ .
Q56. The speaker did not properly space out his speech, (a) exposed (b) benign
but went on _________ one point only. (c) discovered (d) sizeable
(a) stressing (b) avoiding Ans: (b)
(c) devoting (d) decrying Q67. It took the child a long time to recover ______ the
Ans: (a) shock.
Q57. Tourists always enjoyed _________ the setting Sun (a) under (b) from
in the Darjeeling Hills. (c) over (d) about
(a) to watch (b) watching Ans: (b)
(c) in seeing (d) seeing Q68. You have all come well prepared.
Ans: (b) I _____ you to pass this exam.
Q58. A _________ of Japanese artists stepped off the (a) wish (b) except
coach amidst a warm welcome. (c) hope (d) expect
(a) troop (b) troupe Ans: (d)
(c) band (d) gang Q69. Children enjoy _____ the TV programmes.
Ans: (b) (a) to see (b) to watch
Q59. Today students should be reconciled ________ the (c) to look at (d) watching
way things are changing. Ans: (d)
(a) with (b) to Q70. Just as in a family it is the willing cooperation of its
(c) for (d) at members that ___ happiness and contentment, so is it in
Ans: (a) the larger family of the nation.
Q60. Ramappa _________ as the mayor of the town and (a) brought (b) will bring
he will assume charge this Friday. (c) brings (d) bought
(a) elects (b) elected Ans: (c)
(c) is elected (d) is electing Q71. The old man could not remember where he .........
Ans: (c) his money.
Q61. As he proved inefficient the company _______ him. (a) deposits
(a) rejected (b) ejected (b) is depositing
(c) evicted (d) expelled (c) had deposited
Ans: (d) (d) will deposit
Q62. Most people are influenced by ____ advertisements Ans: (c)
and tend to believe the claims made about the product. Q72. Hamid .................. in Mumbai for three years when
(a) colourful (b) aggressive his parents came to visit.
(c) efficient (d) vigorous (a) has lived
Ans: (a) (b) had lived
Q63. The cost of _____ is rising rapidly in this city. (c) was living
(a) being (b) existing (d) had been living
(c) surviving (d) living Ans: (d)
Ans: (d) Q73. I have little time, so I can tell you only .............
Q64. The meeting is ____ because the funds have not what he said.
arrived. (a) shortly (b) briefly
(a) put in (b) put off (c) little of (d) briskly
(c) put away (d) put out Ans: (b)
Ans: (b) Q74. Two years have passed since he ............. here.
Q65. It is ______ if we can organise another exam this (a) has come (b) come
month. (c) came (d) had come
(a) doubtful (b) unlikely Ans: (c)
(c) impractical (d) unsure Q75. His wife has a twin who resembles her so much that
Ans: (a) at first he had great trouble telling ...................
(a) one from the other.
Page 464 of 516
(b) one from another.
(c) each other.
https://telegram.me/aedahamlibra (a) those
(c) them
(b) that
(d) this
(d) who is which. Ans: (a)
Ans: (d) Q84. Ten miles ___ a long way to walk.
Q76. At great length, that night they ............. the (a) are (b) is
feasibility of their plans. (c) are being (d) are not
(a) discussed about Ans: (b)
(b) discussed Q85. The mechanic ____ the vehicle since this morning.
(c) discussed regarding (a) repaired
(d) discussed on (b) repairing
Ans: (b) (c) has been repairing
Q77. Will those of you who have objection to this (d) will be repairing
proposal ......put up your hands, please ? Ans: (c)
(a) discussed Q86. I complimented him ___ his success in the
(b) being discussed examination.
(c) having discussed (a) at (b) on
(d) discussing (c) about (d) for
Ans: (b) Ans: (b)
Q78. She had frequent toothaches. So she decided to Q87. Security arrangements have been tightened up in all
................ ____ areas.
(a) remove her teeth (a) sensible (b) sensual
(b) removing her teeth (c) sensitive (d) sensational
(c) have her teeth removed Ans: (c)
(d) having her teeth removed Q88. The dramatist must cater _____ the taste of the
Ans: (c) audience.
Q79. At one time, Mr. Nigel ............ (a) to (b) over
this supermarket. (c) into (d) for
(a) was owing (b) used to own Ans: (d)
(c) had owned (d) owned Q89. Suitable steps are taken to bring ______ the cost of
Ans: (d) living.
Q80. He was ............... accept the grudgingly given (a) up (b) over
invitation. (c) on (d) down
(a) not enough sensible to Ans: (d)
(b) sensible enough to Q90. If I had worked hard, I ____ very high marks in the
(c) sensible enough not to examination.
(d) sensible enough to not (a) scored
Ans: (c) (b) would score
Q81. The little girl was knocked down by a speeding car (c) could score
and she lost her _____ immediately. (d) would have scored
(a) conscience Ans: (d)
(b) conscientious Q91. I am satisfied .......... his innocence.
(c) conscious (a) at (b) with
(d) consciousness (c) for (d) of
Ans: (d) Ans: (b)
Q82. They have had to put ____ the football match Q92. People like to work for organisation that takes
because of snow. interest in their personal and ..........
(a) of (b) off growth.
(c) up (d) on (a) social (b) physical
Ans: (b) (c) financial (d) professional
Q83. She found a few good cards in a shop and she Ans: (d)
bought ___ cards last night.
Page 465 of 516
https://telegram.me/aedahamlibra
Q93. Ram, along with the members of the family and that
of his friend, .......... a movie.
(c) arrogant (d) filthy
Ans: (b)
(a) was watching Q2. COVENANT
(b) were watching (a) case (b) coupon
(c) have been watching (c) contract (d) settlement
(d) watch Ans: (c)
Ans: (a) Q3. DEFERENCE
Q94. The manager wanted to meet you .......... the theft in (a) indifference (b) sympathy
the office. (c) respect (d) flattery
(a) referring to Ans: (c)
(b) with regard to Q4. ABROGATE
(c) with reference to (a) repeal (b) destroy
(d) in connection with (c) delay (d) dismiss
Ans: (d) Ans: (a)
Q95. I absolutely refuse to put .......... Q5. INTREPID
with that sort of conduct. (a) ambitious (b) determined
(a) up (b) on (c) talkative (d) fearless
(c) off (d) out Ans: (d)
Ans: (a) Q6. GRANDEUR
Q96. The boy was _____ of cheating in the examination. (a) magnificence (b) admiration
(a) condemned (b) accused (c) happiness (d) awe
(c) charged (d) punished Ans: (a)
Ans: (b) Q7. INCEPTION
Q97. We have two telephone operators, _____ of them (a) initiative (b) beginning
do you want ? (c) initial (d) origin
(a) who (b) whom Ans: (b)
(c) which (d) what Q8. COLOSSAL
Ans: (c) (a) famous (b) vigorous
Q98. I have no _____ motive in offering this advice. (c) energetic (d) enormous
(a) posterior (b) interior Ans: (d)
(c) ulterior (d) exterior Q9. PARADOX
Ans: (c) (a) Paradise (b) question
Q99. _____ politicians are always respected. (c) puzzle (d) challenge
(a) Conscious (b) Conscientious Ans: (c)
(c) Cautious (d) Carefree Q10. PROLIFERATE
Ans: (b) (a) progression (b) prohibit
Q100. We will have to atone _____our misdeeds. (c) stipulate (d) reproduce
(a) at (b) on Ans: (d)
(c) for (d) with Q11. CENSURE
Ans: (c) (a) criticize (b) appreciate
(c) blame (d) abuse
Ans: (a)
Synonyms Q12. DILIGENT
(a) industrious (b) indifferent
Directions: In the following questions, sentences are (c) intelligent (d) energetic
given with blanks to be filled in with appropriate and Ans: (a)
suitable word (s). Four alternatives are suggested for each Q13. CONTEMPLATION
question. Choose the correct alternative out of the four. (a) consideration
Q1. OBNOXIOUS (b) meditation
(a) depressing (b) disgusting (c) deliberation
(d) speculation
Page 466 of 516
Q14. ADULATION
https://telegram.me/aedahamlibra
Ans: (b) Q26. VINDICATE
(a) open (b) ventilate
(a) duration (b) argument (c) justify (d) recommend
(c) flattery (d) institution Ans: (c)
Ans: (c) Q27. OCCULT
Q15. QUIVER (a) religious (b) unnatural
(a) quarrel (b) quicken (c) supernatural (d) strong
(c) waver (d) tremble Ans: (c)
Ans: (d) Q28. MENACE
Q16. BEAUTIFUL (a) request (b) prayer
(a) handsome (b) alluring (c) threat (d) curse
(c) appealing (d) attractive Ans: (c)
Ans: (a) Q29. DELINEATE
Q17. MOMENTOUS (a) expand (b) portray
(a) important (b) temporary (c) explain (d) argue
(c) fleeting (d) monumental Ans: (c)
Ans: (a) Q30. ABROGATE
Q18. INFATUATION (a) elope (b) gate-crash
(a) emotion (b) passion (c) abolish (d) destroy
(c) imagination (d) compassion Ans: (c)
Ans: (b) Q31. SUCCESSIVE
Q19. CONSIGNEE (a) rapid (b) victorious
(a) delegate (c) beneficent (d) consecutive
(b) representative Ans: (d)
(c) nominee Q32. RAVAGE
(d) messenger (a) destroy (b) break
Ans: (c) (c) demolish (d) abolish
Q20. TRAMP Ans: (c)
(a) wanderer (b) cheat Q33. SUPERSTITIOUS
(c) traveller (d) pilgrim (a) pious (b) traditional
Ans: (a) (c) irrational (d) sacred
Q21. IRREVERENCE Ans: (c)
(a) disrespect (b) cruelty Q34. MENTOR
(c) unkindness (d) invalidity (a) guide (b) genius
Ans: (a) (c) stylist (d) philosopher
Q22. AMICABLE Ans: (a)
(a) poisonous (b) friendly Q35. GARNISH
(c) satisfying (d) heartening (a) paint (b) adorn
Ans: (b) (c) garner (d) banish
Q23. PRUDENT Ans: (b)
(a) skilled (b) experienced Q36. INFRUCTUOUS
(c) cautious (d) criminal (a) meaningless (b) unnecessary
Ans: (c) (c) redundant (d) fruitless
Q24. PANACEA Ans: (d)
(a) flatter (b) praise Q37. FIDELITY
(c) inactivity (d) cure-all (a) affection (b) allegiance
Ans: (d) (c) accuracy (d) loyalty
Q25. BENEVOLENCE Ans: (d)
(a) ill-will (b) morbidity Q38. GRUFF
(c) kindness (d) vision (a) hard (b) rough
Ans: (c) (c) tough (d) sturdy
Page 467 of 516
Q39. DOLEFUL
https://telegram.me/aedahamlibra
Ans: (b) (b) suspicion
(c) frustration
(a) mournful (b) sober (d) dejection
(c) regretful (d) cheerless Ans: (a)
Ans: (a) Q52. DILATE
Q40. FATAL (a) spin (b) weaken
(a) terrible (b) deadly (c) widen (d) push
(c) poisonous (d) wrong Ans: (c)
Ans: (b) Q53. DUPLICATION
Q41. ELUDE (a) breed (b) reproduction
(a) allure (b) leave (c) print (d) copying
(c) deceive (d) escape Ans: (d)
Ans: (d) Q54. PRIORITY
Q42. PREMONITION (a) urgency (b) protocol
(a) insight (b) uncertainty (c) precedence (d) necessity
(c) forewarning (d) scope Ans: (c)
Ans: (c) Q55. FLUTTER
Q43. DEROGATORY (a) soar (b) agitate
(a) injurious (b) shattering (c) change (d) float
(c) destructive (d) disparaging Ans: (d)
Ans: (d) Q56. COMMAND
Q44. NOMADIC (a) lead (b) instruct
(a) fighting (b) rare (c) manage (d) supervise
(c) strange (d) wandering Ans: (b)
Ans: (d) Q57. GNOME
Q45. RESTIVE (a) giant (b) dwarf
(a) rested (b) restless (c) native (d) alien
(c) limited (d) limitless Ans: (b)
Ans: (b) Q58. ABLUTION
Q46. ERRONEOUSLY (a) censure (b) forgiveness
(a) previously (b) effectively (c) absolution (d) washing
(c) wrongly (d) evidently Ans: (d)
Ans: (c) Q59. SURMOUNT
Q47. ESTRANGED (a) discount (b) surround
(a) jealous (b) angry (c) overcome (d) capture
(c) separated (d) suspicious Ans: (c)
Ans: (c) Q60. TORPID
Q48. CAPITULATE (a) insipid (b) stupid
(a) repeat (b) execute (c) sensitive (d) inactive
(c) summarize (d) surrender Ans: (d)
Ans: (d) Q61. SELECTION
Q49. PROPITIATE (a) denial (b) preference
(a) conform (b) appease (c) refusal (d) display
(c) influence (d) approach Ans: (b)
Ans: (b) Q62. OSTENTATION
Q50. CEREMONIAL (a) pomp (b) pretence
(a) religious (b) formal (c) abundance (d) plenty
(c) official (d) pompous Ans: (a)
Ans: (b) Q63. CONVICT
Q51. OBSESSION (a) adventurer (b) fugitive
(a) pre-occupation (c) criminal (d) impostor
Page 468 of 516
Q64. ITINERANT
https://telegram.me/aedahamlibra
Ans: (c) Q76. OBSTINATE
(a) stubborn (b) pretty
(a) frequent use of the word ‗it‘ (c) silly (d) clever
(b) anything involving repetition Ans: (a)
(c) plan for a proposed journey Q77. ALERT
(d) travelling from place to place (a) hostile (b) watchful
Ans: (d) (c) brave (d) quick
Q65. TRANSPARENT Ans: (b)
(a) verbose (b) involved Q78. ACCEDE
(c) witty (d) lucid (a) consent (b) access
Ans: (d) (c) assess (d) proceed
Q66. ENLIVEN Ans: (a)
(a) dampen (b) cheer Q79. SUPERANNUATED
(c) depress (d) subdue (a) experienced (b) accepted
Ans: (b) (c) retired (d) senile
Q67. HAUGHTY Ans: (c)
(a) humble (b) subservient Q80. AUDACITY
(c) meek (d) conceited (a) strength (b) boldness
Ans: (d) (c) asperity (d) fear
Q68. DEMENTED Ans: (b)
(a) reasonable (b) sensible Q81. DECREPITUDE
(c) idiotic (d) lucid (a) disease (b) coolness
Ans: (c) (c) crowd (d) feebleness
Q69. ILL-BRED Ans: (d)
(a) uncouth (b) well-bred Q82. TRANSITION
(c) courteous (d) mannerly (a) position (b) translation
Ans: (a) (c) change (d) movement
Q70. KNACK Ans: (c)
(a) dexterity (b) awkwardness Q83. ACCUSED
(c) clumsiness (d) disability (a) indicated (b) indicted
Ans: (a) (c) induced (d) instigated
Q71. LACONIC Ans: (b)
(a) loquacious (b) concise Q84. BECKONED
(c) rambling (d) verbose (a) accused (b) called
Ans: (b) (c) sent (d) acquitted
Q72. KNAVISH Ans: (b)
(a) honourable (b) noble Q85. GENUINE
(c) trustworthy (d) unscrupulous (a) generous (b) healthy
Ans: (d) (c) natural (d) original
Q73. ASSERTIVE Ans: (d)
(a) unassertive (b) timorous Q86. SOMBRE
(c) domineering (d) unobtrusive (a) gloomy (b) quiet
Ans: (c) (c) serious (d) sleepy
Q74. CAPRICIOUS Ans: (a)
(a) whimsical (b) unmovable Q87. REGRESS
(c) decisive (d) stable (a) deteriorate (b) Degenerate
Ans: (a) (c) backslide (d) lapse
Q75. DEXTERITY Ans: (c)
(a) incompetence (b) gaucheness Q88. TOXIC
(c) adroitness (d) ineptitude (a) bitter (b) foul-smelling
Ans: (c) (c) remedia (d) poisonous
Page 469 of 516
Q89. YARDSTICK
https://telegram.me/aedahamlibra
Ans: (d)
Antonyms
(a) summation (b) size
(c) statistics (d) standard
Directions (1-5): In these questions, choose the word
Ans: (d)
opposite in meaning to the word given in bold.
Q90. LITTLE
Q1. OSTRACISE
(a) trivial (b) petty
(a) amuse (b) welcome
(c) sample (d) simple
(c) entertain (d) host
Ans: (a)
Ans: (b)
Q91. AVARICE
Q2. DENSE
(a) generosity (b) envy
(a) scarce (b) slim
(c) greed (d) hatred
(c) sparse (d) lean
Ans: (c)
Ans: (c)
Q92. AUGUST
Q3. PARSIMONIOUS
(a) majestic (b) important
(a) prodigious (b) selfless
(c) difficult (d) huge
(c) extravagant
Ans: (a)
(d) ostentatious
Q93. DECIMATED
Ans: (c)
(a) denounced (b) destroyed
Q4. FETTER
(b) successful (d) depressed
(a) restore (b) liberate
Ans: (b)
(c) exonerate (d) distract
Q94. HURDLE
Ans: (b)
(a) obstacle (b) ban
Q5. HARMONY
(c) hedge (d) relay
(a) strife (b) annoyance
Ans: (a)
(c) cruelty (d) mischief
Q95. RARE
Ans: (a)
(a) common (b) usual
Q6. CONCILIATION
(c) scarce (d) few
(a) dispute (b) irritation
Ans: (c)
(c) separation (d) confrontation
Q96. APPOSITE
Ans: (d)
(a) contrary (b) bitter
Q7. MYTH
(c) appropriate (d) misleading
(a) truth (b) fact
Ans: (c)
(c) falsehood (d) story
Q97. SCORN
Ans: (b)
(a) ridicule (b) laugh
Q8. RELUCTANTLY
(c) condemn (d) criticize
(a) pleasingly (b) willingly
Ans: (c)
(c) satisfactorily (d) happily
Q98. IMPIOUS
Ans: (b)
(a) holy (b) mischievous
Q9. MUTILATE
(c) shrewd (d) irreverent
(a) instruct (b) induct
Ans: (d)
(c) conduct (d) mend
Q99. FREELANCE
Ans: (d)
(a) self-betrayed (b) self-centred
Q10. LAMENT
(c) self-employed (d) self-driven
(a) rejoice (b) rejuvenate
Ans: (c)
(c) complain (d) cry
Q100. SLITHER
Ans: (a)
(a) slide (b) move
Q11. ANIMOSITY
(c) shake (d) slip
(a) love (b) lust
Ans: (a)
(c) luck (d) loss
Ans: (a)

Page 470 of 516


Q12. ALTERCATION
(a) explanation
https://telegram.me/aedahamlibra
(b) challenge Q25. DISTANT
Ans: (c)

(c) compromise (d) opposition (a) far (b) close


Ans: (c) (c) imminent (d) along
Q13. COAX Ans: (b)
(a) dull (b) dissuade Q26. TRANSPARENT
(c) active (d) speed (a) clear (b) ambiguity
Ans: (b) (c) opaque (d) crystal
Q14. ERUDITE Ans: (c)
(a) educated (b) unscholarly Q27. ENCOUNTERED
(c) scholarly (d) possessive (a) avoided (b) enriched
Ans: (b) (c) faced (d) overcome
Q15. AFFLUENT Ans: (a)
(a) high (b) poor Q28. HANDY
(c) rare (d) fluent (a) cumbersome (b) handful
Ans: (b) (c) unwieldy (d) heavy
Q16. PAROCHIAL Ans: (d)
(a) narrow (b) international Q29. ELEGANCE
(c) global (d) world wide (a) pride (b) beauty
Ans: (c) (c) coarseness (d) vulgarity
Q17. FALLIBLE Ans: (d)
(a) unerring (b) reliable Q30. CAPRICIOUS
(c) falsehood (d) trustful (a) firm (b) fickle
Ans: (a) (c) indefinite (d) defian
Q18. IMPERTINENT Ans: (a)
(a) arrogance (b) appropriate Q31. EXODUS
(c) respectful (d) modest (a) influx (b) home-coming
Ans: (c) (c) return (d) restoration
Q19. ROUGH Ans: (a)
(a) refined (b) charming Q32. INQUISITIVE
(c) smooth (d) polite (a) insincere (b) indifferent
Ans: (c) (c) insensitive (d) insulting
Q20. RATIFICATION Ans: (b)
(a) disapproval (b) disagreeable Q33. CANDID
(c) denial (d) disturbing (a) outspoken (b) frank
Ans: (a) (c) devious (d) disguised
Q21. OMIT Ans: (c)
(a) exclude (b) include Q34. NADIR
(c) undertake (d) add (a) modernity (b) zenith
Ans: (b) (c) liberty (d) progress
Q22. INTEGRATION Ans: (b)
(a) unity (b) synthesis Q35. CULPABLE
(c) linking (d) fragmentation (a) defendable (b) blameless
Ans: (d) (c) careless (d) irresponsible
Q23. MINIATURE Ans: (b)
(a) large (b) small Q36. FACILITATE
(c) heavy (d) least (a) help (b) propogate
Ans: (a) (c) hinder (d) reject
Q24. EXHAUSTS Ans: (c)
(a) tires (b) empties Q37. CRITICISE
(c) invigorates (d) drains (a) finish (b) recommend
Page 471 of 516
(c) commend https://telegram.me/aedahamlibra
(d) request
Ans: (c)
(a) amusing
(c) hilarious
(b) ludicrous
(d) mean
Q38. CONFORMITY Ans: (b)
(a) deviation (b) dilution Q50. ECCENTRICITY
(c) distraction (d) diversion (a) normalcy (b) similarity
Ans: (a) (c) equality (d) clarity
Q39. AFFIRM Ans: (a)
(a) refuse (b) negate Q51. GENTLEMAN
(c) neglect (d) avoid (a) clown (b) boor
Ans: (a) (c) dud (d) buffoon
Q40. AUTONOMY Ans: (b)
(a) slavery Q52. PANICKY
(b) subordination (a) confident (b) sober
(c) dependence (c) quiet (d) calm
(d) submissiveness Ans: (d)
Ans: (c) Q53. MOURNFUL
Q41. SURVIVAL (a) playful (b) joyous
(a) evolution (b) development (c) laughable (d) humorous
(c) creation (d) extinction Ans: (b)
Ans: (d) Q54. OBSTINATE
Q42. INSERT (a) confused (b) determined
(a) depict (b) extract (c) trusted (d) flexible
(c) defame (d) enhance Ans: (d)
Ans: (b) Q55. MYOPIC
Q43. INCONGRUOUS (a) short-sighted
(a) conflicting (b) feeble-minded
(b) contradictory (c) fore-sighted
(c) ill-matched (d) far-sighted
(d) harmonious Ans: (d)
Ans: (d) Q56. PRUDENT
Q44. DEMENTED (a) silly (b) unwise
(a) sensible (b) sensitive (c) idiotic (d) poor
(c) sensual (d) sensuous Ans: (b)
Ans: (a) Q57. CONCISE
Q45. IMPROMPTU (a) extended (b) lengthy
(a) prompt (b) forced (c) protracted (d) elongated
(c) premeditated (d) inordinate Ans: (b)
Ans: (c) Q58. AFFIRMATION
Q46. HOSTILE (a) denial (b) refusal
(a) friendly (b) sudden (c) opposition (d) obstruction
(c) costly (d) unfair Ans: (a)
Ans: (a) Q59. CURTAIL
Q47. BREADTH (a) arrive (b) continue
(a) shortness (b) narrowness (c) resume (d) start
(c) lightness (d) thickness Ans: (c)
Ans: (b) Q60. AFFLUENCE
Q48. TIMOROUS (a) misery (b) stagnation
(a) trembling (b) cowardly (c) neglect (d) poverty
(c) bright (d) bold Ans: (d)
Ans: (d) Q61. AGREEMENT
Q49. SUBLIME (a) dislocation (b) discord
Page 472 of 516
(c) turbulence https://telegram.me/aedahamlibra
(d) fragmentation
Ans: (b)
(c) admiration (d) assessment
Ans: (b)
Q62. AGONY Q74. PACIFY
(a) pleasure (b) laughter (a) threaten (b) challenge
(c) bliss (d) ecstasy (c) quarrel (d) enrage
Ans: (a) Ans: (d)
Q63. RETICENT Q75. APPARENT
(a) forward (b) developed (a) ambiguous (b) clear
(c) sophisticated (c) visible (d) conspicuous
(d) communicative Ans: (a)
Ans: (d) Q76. ATHEIST
Q64. PRECISE (a) rationalist (b) theologist
(a) indecent (b) vague (c) believer (d) ritualist
(c) incorrect (d) indistinct Ans: (c)
Ans: (b) Q77. GIGANTIC
Q65. CANDID (a) weak (b) fragile
(a) cunning (b) diplomatic (c) slight (d) tiny
(c) doubtful (d) impertinent Ans: (d)
Ans: (a) Q78. ILLICIT
Q66. APOCRYPHAL (a) liberal (b) intelligent
(a) authentic (c) lawful (d) clear
(b) dubious Ans: (c)
(c) unsubstantiated Q79. CALLOUS
(d) fictitious (a) sensitive (b) soft
Ans: (a) (c) kind (d) generous
Q67. REPEL Ans: (a)
(a) attend (b) concentrate Q80. ENIGMATIC
(c) continue (d) attract (a) simple (b) reticent
Ans: (d) (c) plain (d) nervous
Q68. APALLING Ans: (c)
(a) shocking (b) consoling Q81. ABUNDANT
(c) scaring (d) horrifying (a) short (b) limited
Ans: (b) (c) petty (d) meagre
Q69. JEST Ans: (d)
(a) gravity (b) grim Q82. HARASS
(c) genial (d) sport (a) reward (b) praise
Ans: (a) (c) flatter (d) relieve
Q70. CURTAIL Ans: (d)
(a) lengthen (b) shorten Q83. CHARMING
(c) entail (d) close (a) insolent (b) indignant
Ans: (a) (c) repulsive (d) handicapped
Q71. APPLAUD Ans: (c)
(a) praise (b) loud Q84. GRUESOME
(c) censure (d) acclaim (a) attractive (b) beneficial
Ans: (c) (c) gracious (d) amicable
Q72. JADE Ans: (c)
(a) defend (b) grasp Q85. DESPISE
(c) cheer (d) harass (a) appease (b) flatter
Ans: (c) (c) admire (d) appreciate
Q73. APPRECIATION Ans: (c)
(a) appraisal (b) antipathy Q86. CUMBERSOME
Page 473 of 516
(a) heavy
(c) smooth
https://telegram.me/aedahamlibra
(b) convenient
(d) automatic
(d) irresponsible
Ans: (a)
Ans: (b) Q99. SUBLIME
Q87. LOQUACIOUS (a) inferior (b) deficit
(a) talkative (b) taciturn (c) ridiculous (d) crooked
(c) diffident (d) bashful Ans: (a)
Ans: (b) Q100. EVIDENT
Q88. INTELLIGIBLE (a) definite (b) careless
(a) dull (b) foolish (c) clear (d) obscure
(c) garbled (d) confused Ans: (d)
Ans: (d)
Q89. PHILANTHROPIST
(a) philistine (b) moralist
One-Word Substitution
(c) spendthrift (d) miser
Ans: (d) Directions: In these questions, out of the four alternatives
Q90. ANXIOUS choose the one which can be substituted for the given
(a) fearful (b) worried words/sentence.
(c) calm (d) concerned Q1. A person with a long experience of any occupation
Ans: (c) (a) veteran
Q91. ABUNDANCE (b) genius
(a) poverty (b) wretchedness (c) seasoned
(c) scarcity (d) famine (d) ambidexterous
Ans: (c) Ans: (a)
Q92. MIGRATE Q2. Words written on a tomb
(a) return (b) rehabilitate (a) epithet (b) epigraph
(c) transfer (d) settle (c) Soliloquy (d) epitaph
Ans: (a) Ans: (d)
Q93. CONVICTED Q3. Stealthily done
(a) acquitted (b) pardoned (a) devious
(c) exempted (d) liberated (b) nefarious
Ans: (a) (c) surreptitious
Q94. CURTAIL (d) villainous
(a) detail (b) enlarge Ans: (c)
(c) promote (d) exaggerate Q4. Something no longer in use
Ans: (b) (a) desolate (b) absolute
Q95. ARID (c) obsolete (d) primitive
(a) cloudy (b) juicy Ans: (c)
(c) marshy (d) wet Q5. One not concerned with right or wrong
Ans: (d) (a) moral (b) amoral
Q96. INSOLENT (c) immoral (d) immortal
(a) arrogant (b) humble Ans: (b)
(c) ashamed (d) ignorant Q6. A person who opposes war or use of military force
Ans: (b) (a) fascist (b) pacifist
Q97. CURSORY (c) narcissist (d) fatalist
(a) final (b) thorough Ans: (b)
(c) impulsive (d) customary Q7. Severely abusive writing in journals
Ans: (b) (a) imaginary (b) speculative
Q98. LACKADAISICAL (c) scurrilous (d) sarcastic
(a) enthusiastic Ans: (c)
(b) intelligent
(c) classical
Page 474 of 516
help or protection
https://telegram.me/aedahamlibra
Q8. Call upon God or any other power (like law) etc. for Ans: (b)
Q19. A person coming to a foreign land to settle there.
(a) invocation (b) involution (a) immigrant (b) emigrant
(c) inundation (d) revocation (c) tourist (d) settler
Ans: (a) Ans: (a)
Q9. Fear of being enclosed in a small closed space Q20. Something capable of being done.
(a) agoraphobia (a) probable (b) feasible
(b) claustrophobia (c) tenable (d) explicable
(c) xenophobia Ans: (b)
(d) paranoia Q21. Give and receive mutually
Ans: (b) (a) present
Q10. One who has become dependent on something or (b) reciprocate
drugs is (c) compromise
(a) adamant (b) edict (d) approve
(c) addict (d) derelict Ans: (b)
Ans: (c) Q22. One who can think about the future with
Q11. Succession of rulers belonging to one family. imagination and wisdom.
(a) dynasty (b) lineage (a) dreamer (b) seer
(c) ancestry (d) progeny (c) idealist (d) visionary
Ans: (a) Ans: (d)
Q12. To cut something into two pieces. Q23. A doctor who treats children
(a) severe (b) sever (a) paediatrician
(c) sewer (d) sow (b) pedagogue
Ans: (b) (c) pedestrian
Q13. Flat metal or Porcelain plate fixed on a wall as an (d) paedophile
ornament or memorial. Ans: (a)
(a) tabloid (b) poster Q24. One who studies election trends by means of
(c) board (d) plaque opinion polls.
Ans: (d) (a) entomologist
Q14. Act of deceiving somebody in order to make (b) psephologist
money. (c) demagogue
(a) fraud (b) robbery (d) eugenist
(c) pickpocket (d) theft Ans: (b)
Ans: (a) Q25. One who believes in offering equal opportunities to
Q15. A short poem or speech addressed to the spectators women in all spheres.
after the conclusion of a drama. (a) male chauvinist
(a) prologue (b) dialogue (b) feminist
(c) epilogue (d) monologue (c) fatalist
Ans: (c) (d) futurist
Q16. Capable of being understood in either of two or Ans: (b)
more possible senses, and therefore not definite. Q26. Killing of a child
(a) amphibious (b) ambiguous (a) homicide (b) genocide
(c) amorphous (d) confusing (c) infanticide (d) suicide
Ans: (b) Ans: (c)
Q17. A person who is unable to pay his debts. Q27. The art of good eating
(a) solvent (b) banker (a) gastronomy
(c) insolvent (d) lender (b) astronomy
Ans: (c) (c) vegetarianism
Q18. Anything which is no longer in use. (d) gourmet
(a) obscure (b) obsolete Ans: (a)
(c) pristine (d) lapsed Q28. One who is indifferent to pleasure or pain.
Page 475 of 516
(a) stoic
(c) cynic
https://telegram.me/aedahamlibra
(b) stylist
(d) psychic
(a) ligaments
(c) muscles
(b) endoderm
(d) fibre
Ans: (a) Ans: (a)
Q29. Lasting only for a moment Q39. The first model of a new device
(a) momentous (a) prototype (b) sculpture
(b) momentary (c) icon (d) photograph
(c) trivial Ans: (a)
(d) petty Q40. A building where an audience sits
Ans: (b) (a) aquarium (b) gymnasium
Q30. To seize control of a vehicle in order to force it to (c) auditorium (d) stadium
go to a new destination, or demand something. Ans: (c)
(a) attack (b) contract Q41. That which lasts for a short time
(c) hijack (d) detour (a) regular (b) transitory
Ans: (c) (c) rotatory (d) repository
Q31. One who goes to settle in another country Ans: (b)
(a) immigrant (b) alien Q42. Ready to believe anything.
(c) citizen (d) emigrant (a) credible (b) incredible
Ans: (d) (c) credulous (d) incredulous
Q32. One who hates mankind Ans: (c)
(a) philanthropist Q43. A four footed animal
(b) terrorist (a) tetrapod (b) quadruped
(c) misanthrope (c) polyped (d) double-paired
(d) misogynist Ans: (b)
Ans: (c) Q44. A person who believes in the total abolition of war
Q33. Belonging to all parts of the world (a) socialist (b) communist
(a) versatile (c) fascist (d) pacifist
(b) universal Ans: (d)
(c) cosmopolitan Q45. Constant efforts to achieve something
(d) secular (a) patience (b) vigour
Ans: (b) (c) attempt (d) perseverance
Q34. One who walks on ropes Ans: (d)
(a) funambulist Q46. One who collects coins
(b) upholsterer (a) archaeologist
(c) acrobat (b) numismatist
(d) aviator (c) philatelist
Ans: (a) (d) connoisseur
Q35. The study of the origin and history of words Ans: (b)
(a) linguistics (b) etymology Q47. A system of Government in which only one
(c) verbose (d) anthology political party is allowed to function
Ans: (b) (a) oligarchy
Q36. A person who breaks into a house in order to steal (b) dictatorship
(a) poacher (b) bandit (c) totalitarianism
(c) intruder (d) burglar (d) theocracy
Ans: (d) Ans: (c)
Q37. The study of maps Q48. Customs and habits of a particular group
(a) cartography (a) mores (b) traditions
(b) geography (c) rites (d) rituals
(c) geology Ans: (a)
(d) atlas Q49. A body of persons appointed to hear evidence and
Ans: (a) give their verdict in trials
Q38. Tough tissues in joints (a) association (b) council
Page 476 of 516
(c) bar https://telegram.me/aedahamlibra
(d) jury
Ans: (d)
(a) bacteria
(c) virus
(b) amoeba
(d) fungus
Q50. Indifference to pleasure or pain Ans: (d)
(a) perseverance (b) tolerance Q62. One who is greedy
(c) stoicism (d) reticence (a) voracious (b) avaricious
Ans: (c) (c) carnivorous (d) omnivorous
Q51. Concluding part of a literary work Ans: (a)
(a) epilogue (b) bibliography Q63. An area of land that is controlled by a ruler
(c) soliloquy (d) episode (a) colony (b) dominion
Ans: (a) (c) country (d) municipality
Q52. One who is beyond reform Ans: (a)
(a) optimistic (b) incorrigible Q64. A place where Jews worship according to their
(c) indefatigable (d) notorious religion
Ans: (b) (a) cathedral (b) synagogue
Q53. Science of diseases (c) chapel (d) demagogue
(a) philology (b) pathology Ans: (b)
(c) psychology (d) virology Q65. One who is indifferent to pleasure and pain
Ans: (b) (a) ascetic (b) esoteric
Q54. One who secretly listens to the talk of others (c) stoic (d) sceptical
(a) spy (b) detective Ans: (c)
(c) emissary (d) eavesdropper Q66. The study of religion and religious ideas and beliefs
Ans: (d) (a) theocracy (b) theosophy
Q55. One who believes in no government and therefore (c) theology (d) theism
incites disorder in a State Ans: (b)
(a) monarchist Q67. Dissection of a dead body to find the cause of
(b) anarchist death.
(c) autocrat (d) naxalite (a) biopsy (b) investigation
Ans: (b) (c) surgery (d) autopsy
Q56. A mild or indirect expression substituted for an Ans: (d)
offensive or harsh one Q68. A person without training or experience in a skill or
(a) euphemism (b) truism subject
(c) favouritism (d) altruism (a) chaplain (b) mason
Ans: (a) (c) artisan (d) novice
Q57. The murder of parent or a near relative Ans: (d)
(a) patricide (b) parricide Q69. One who stays away from school with permission
(c) matricide (d) homicide (a) pedant (b) supplicant
Ans: (b) (c) mendicant (d) truant
Q58. Animals who live in herds Ans: (d)
(a) sociable (b) gregarious Q70. The act of killing a whole group of people,
(c) carnivorous (d) social especially a whole race
Ans: (b) (a) patricide (b) parricide
Q59. A broad road bordered with trees (c) matricide (d) genocide
(a) boudoir (b) boulevard Ans: (d)
(c) avenue (d) facade Q71. A government in which all religions are honoured
Ans: (b) (a) communist (b) socialistic
Q60. Violation of something holy or sacred (c) secular (d) capitalist
(a) profanity (b) sedition Ans: (c)
(c) sacrilege (d) slander Q72. A place where government/ public records are kept
Ans: (c) (a) archive (b) museum
Q61. Simple, fast -spreading plant without flowers or (c) shelf (d) cellar
leaves, which can often cause disease Ans: (a)
Page 477 of 516
married to each other
https://telegram.me/aedahamlibra
Q73. Living together of a man and woman without being (b) misogynist
(c) philanthropist
(a) marriage (b) equipage (d) misogamist
(c) lineage (d) concubinage Ans: (a)
Ans: (d) Q85. Irresistible craving for alcoholic drinks
Q74. Too much official formality (a) megalomania
(a) bureaucracy (b) dipsomania
(b) red-tapism (c) kleptomania
(c) nepotism (d) pyromania
(d) formalism Ans: (b)
Ans: (b) Q86. One who deserts his principles or party.
Q75. Dry weather with no rainfall (a) apostle (b) proselyte
(a) draught (b) draft (c) renegade (d) critic
(c) drought (d) desert Ans: (c)
Ans: (c) Q87. One who intervenes between two or more parties to
Q76. A sly look that is lustful settle differences
(a) leer (b) lore (a) neutral (b) intermediary
(c) lark (d) littoral (c) judge (d) connoisseur
Ans: (a) Ans: (b)
Q77. A jocular person who is full of amusing anecdotes Q88. The habit of always admiring oneself
(a) wile (b) yokel (a) psychosis (b) neurosis
(c) wag (d) aeon (c) narcissism (d) paranoia
Ans: (c) Ans: (c)
Q78. Deep in thought Q89. To take back, withdraw or renounce
(a) meditation (b) pensive (a) repent (b) retrace
(c) pesky (d) purloin (c) refuse (d) recant
Ans: (b) Ans: (d)
Q79. Without risk of punishment Q90. A pen for small animals
(a) impudent (b) impunity (a) hutch (b) lair
(c) inexorable (d) imperturable (c) den (d) cage
Ans: (b) Ans: (a)
Q80. Excessive preoccupation with one's health Q91. A woman having more than one husband at the
(a) hypochondria same time
(b) malaise (a) polyandry (b) polygamy
(c) disaffected (c) polyphony (d) polygyny
(d) malinger Ans: (a)
Ans: (a) Q92. Feeding on food made both of plants and flesh
Q81. The worship of idols or images (a) carnivorous (b) omnipotent
(a) atheism (b) theism (c) omnivorous (d) optimist
(c) idolatry (d) iconoclasm Ans: (c)
Ans: (c) Q93. One who destroys images or attacks popular beliefs
Q82. Something that is poisonous or unhealthy (a) imagist (b) misanthropist
(a) trivial (b) toxic (c) iconoclast (d) masochist
(c) torpid (d) tragic Ans: (c)
Ans: (b) Q94. Violation of something holy and sacred
Q83. A remedy for all diseases (a) pollution (b) pilferage
(a) amnesia (b) panacea (c) sacrilege (d) pittance
(c) intelligentsia (d) parasol Ans: (c)
Ans: (b) Q95. A place where nuns live and work
Q84. A hater of mankind (a) church (b) school
(a) misanthrope (c) abode (d) convent
Page 478 of 516
https://telegram.me/aedahamlibra
Q96. General view of a person‘s character
Ans: (d) Q3. We shouldn‘t look down upon the wr etched of the
earth.
(a) biography (a) sympathise with
(b) profile (b) hate intensely
(c) introduction (c) be indifferent to
(d) sketch (d) regard with contempt
Ans: (b) Ans: (d)
Q97. Wild and noisy disorder Q4. Because of his misbehaviour, he is bound to face the
(a) agitation music.
(b) revolution (a) get finished
(c) pandemonium (b) get reprimanded
(d) stir (c) feel sorry
Ans: (c) (d) listen to the music
Q98. The first public speech delivered by a person Ans: (b)
(a) maiden speech Q5. The working of the factory was distru-pted on
(b) inaugural speech account of a token strike by the workers.
(c) trial speech (a) total strike
(d) marathon speech (b) carefully planned strike
Ans: (a) (c) short strike held as a warning
Q99. One who does not follow the usual rules of social (d) sudden call of strike
life Ans: (c)
(a) bohemian (b) artisan Q6. By opposing his proposal he fell foul of him.
(c) partisan (d) physician (a) quarrel with
Ans: (a) (b) felt annoyed with
Q100. Placing a thing beside another (c) agreed with
(a) impose (b) repose (d) got into trouble with
(c) juxtapose (d) expose Ans: (d)
Ans: (c) Q7. Those who work by fits and starts seldom show
good results.
(a) rarely
Idioms/Phrases (b) disinterestedly
(c) irregularly
Directions: In these questions, four alternatives are given (d) regularly
for the Idiom/Phrase given in bold in the sentence. Ans: (c)
Choose the alternat ive which best expresses the meaning Q8. The new manager t hought that he would give
of the Idiom/Phrase given in bold. employees enough rope for the first six months after
Q1. For his alleged involvement in espio-nage, he is which he would check the work done himself.
under a cloud these days. (a) many directives and orders
(a) experiencing cloudy weather (b) sufficient advice
(b) enjoying favourable luck (c) all the material they needed
(c) under suspicion (d) enough freedom for action
(d) under observation Ans: (d)
Ans: (c) Q9. He was all at sea when he began his new Job.
Q2. We have appealed to him again and again; there is no (a) happy (b) sad
use flogging a dead horse now. (c) puzzled (d) triumphant
(a) repeating our request Ans: (c)
(b) making him see reason Q10. The sweeping statement by the boss left the
(c) beating about the bush conscientious workers disgusted.
(d) wasting time in useless effort (a) rash statement
Ans: (d) (b) unpremeditated statement
(c) thoughtless statement
Page 479 of 516
(d) generalised statement https://telegram.me/aedahamlibra
Ans: (d)
(c) forming a group
(d) have good understanding
Q11. The failure of crops in successive years put the Ans: (b)
farmer in a tight corner. Q19. The heavy downpour played havoc in the coastal
(a) in a closed room area.
(b) in a small field (a) caused destruction
(c) in a difficult situation (b) caused diseases
(d) in a meadow (c) caused floods
Ans: (c) (d) caused hardship
Q12. The effort to trace the culprit was a wild goose Ans: (a)
chase. Q20. To have a green thumb means
(a) fruitful hunting (a) one‘s nails are painted green
(b) futile search (b) one is artistic
(c) ideal seeking (c) to have a natural interest in gardening
(d) genuine effort (d) one has a green tattoo on the thumb
Ans: (b) Ans: (c)
Q13. The story does not hold water. Q21. When he saw the snake he took to his heels.
(a) does not deserve appreciation (a) ran away in fear
(b) does not fulfil the requirements (b) went slowly
(c) cannot be believed (c) walked in fear
(d) cannot be valued (d) jumped fast
Ans: (c) Ans: (a)
Q14. Raj couldn‘t pay the bill, so he asked the owner to Q22. He has to abide by the hard and fast rule of the
put it on the cuff. company.
(a) on credit (a) flexible (b) strict
(b) against his credit card (c) difficult (d) honest
(c) in his bank account Ans: (b)
(d) in his friend‘s account Q23. She goes to her mother‘s house off and on.
Ans: (a) (a) frequently (b) rarely
Q15. His statement is out and out a lie. (c) occasionally (d) sometimes
(a) totally (b) simply Ans: (c)
(c) merely (d) slightly Q24. The robber murdered, the woman in cold blood
Ans: (a) for the sake of the jewels.
Q16. The luxury car that they bought turned out to be a (a) a murder done without feeling
white elephant. (b) a murder done in revenge
(a) a rare article (c) a murder done in great anger
(b) useful mode of transport (d) a murder done in enmity
(c) costly or troublesome possession Ans: (a)
(d) a proud possession Q25. Indians are going places in the field of software
Ans: (c) technology.
Q17. If you are fair and square in your wo rk you will (a) going abroad
definitely prosper. (b) going to spaces
(a) active (c) talented and successful
(b) honest (d) friendly and amicable
(c) business like Ans: (c)
(d) authoritative Q26. The poet drew on his fancy , not his knowledge of
Ans: (b) Nature, when he wrote his poem on birds.
Q18. There is no love lost between any two (a) used his understanding
neighbouring countries in the world. (b) used his knowledge
(a) stop loving (c) used his imagination
(b) not on good terms (d) used his skill
Page 480 of 516
https://telegram.me/aedahamlibra
Q27. My neighbour had to pay through his nose for a
Ans: (c) (c) confessed without reserve
(d) faced bravely
brand new car. Ans: (c)
(a) pay huge loans Q35. She tries very hard to keep up with her rich
(b) pay a reasonable price neighbours.
(c) pay an extremely high price (a) to imitate
(d) make a quick buck (b) to keep in touch
Ans: (c) (c) to avoid
Q28. Very ambitious people do not like to rest on their (d) to be on par
laurels. Ans: (b)
(a) to be unhappy Q36. He went on sowing wild oats ; he reaped suf fering
(b) to be motivated in his later life.
(c) to be impatient (a) inviting troubles as a boy
(d) to be complacent (b) warning others as a youngman
Ans: (d) (c) irresponsible pleasure seeking in young age
Q29. If he phones again, I am going to give him a piece (d) sowing grains called oats when young
of my mind. Ans: (c)
(a) to be nice to him Q37. I don‘t know why she has become stand-offish
(b) to take revenge on him recently.
(c) to reprimand him (a) angry
(d) to support him (b) hilarious
Ans: (c) (c) indifferent
Q30. The party high command wanted to stave off an (d) unmanageable
open battle. Ans: (c)
(a) postpone Q38. Why don‘t you put an end to blowing your own
(b) wait and see trumpet ?
(c) allow it to take its own course (a) playing your own trumpet to produce music
(d) prevent (b) making too much noise
Ans: (a) (c) praising your own abilities and achievements
Q31. Ramesh takes after his father. (d) None of these
(a) follows (b) imitates Ans: (c)
(c) obeys (d) resembles Q39. I knew he had an axe to grind and turned down his
Ans: (d) offer of help.
Q32. They made no bones about acknowledging their (a) a blunt axe
debt to his genius. (b) a sharp tongue
(a) did not have any hesitation in (c) a private interest to serve
(b) did not have any faith in (d) a tendency to fight
(c) demanded compensation for Ans: (c)
(d) had problems in Q40. The saint‘s life was an open book.
Ans: (a) (a) an uncomplicated one
Q33. It is evident from the minister‘s statement that (b) one that held no secrets
heads will roll in the Secretariat. (c) an example to all
(a) transfers will take place (d) an interesting biography
(b) heads will be cut off Ans: (b)
(c) people will die Q41. Reading between the lines I realised that my
(d) dismissals will occur friend wanted to keep something from me.
Ans: (d) (a) looking for meanings that are not actually expressed
Q34. During the last moments of his life, the criminal (b) reading carelessly
made a clean breast of everything he had done. (c) reading with anxiety
(a) showed his breast (d) glancing over the lines
(b) fought like a hero Ans: (a)
Page 481 of 516
devil his due.
https://telegram.me/aedahamlibra
Q42. Sometimes, it happens that we have to give the Q50. He is so furious that he would go through fire and
water to revenge himself on his foe.
(a) to give credit to even a notorious person (a) approach everybody for help
(b) to give encouragement even to the enemy (b) avail himself of any opportunity
(c) to invite the devil (c) use any conceivable method
(d) to stand in the way of the devil (d) undergo any risk
Ans: (b) Ans: (d)
Q43. The king had been made to eat humble pie. Q51. The watchdogs were asleep when the bulls ran riot.
(a) to eat slowly (a) behaved cleverly
(b) to have an excellent dish (b) acted without restraint
(c) to eat a good pie (c) wandered aimlessly
(d) to apologise (d) had the best of time
Ans: (d) Ans: (b)
Q44. He was given Hobson‘s choice by the employer. Q52. In spite of the immense pressure exerted by the
(a) excellent choice militants, the Government, has decided not to give in.
(b) no real choice at all (a) accede (b) yield
(c) choice to live or die (c) oblige (d) conform
(d) first choice Ans: (b)
Ans: (b) Q53. The young and the ol d sat cheek by jowl in the
Q45. He has a very nice manner, but you would better large audience.
take what he says with a grain of salt. (a) very near (b) very far
(a) to listen to something with considerable doubt (c) tongue tied (d) irritated
(b) to talk sensibly Ans: (a)
(c) to criticise Q54. We wanted to keep the gift as a surprise for mother
(d) to complement but my sister gave the game away.
Ans: (a) (a) lost the game
Q46. He didn‘t tell me directly, but reading between the (b) gave out the secret
lines. I think he is not happy with them. (c) played badly
(a) reading slowly and haltingly (d) withdrew from the game
(b) understanding the sense rather than the actual words Ans: (b)
(c) understanding the meaning of words and not the sense Q55. I don‘t think the law will interfere with us as we are
(d) reading superficially just trying to turn an honest penny.
Ans: (b) (a) make a legitimate living
Q47. Gopi works by fits and starts. (b) make a good living
(a) consistently (c) have dealings in white money
(b) irregularly (d) become more honest
(c) in high spirits Ans: (a)
(d) enthusiastically Q56. a dark horse
Ans: (b) (a) an unforeseen competitor
Q48. I cannot put up with your misconduct any longer. (b) a black horse
(a) excuse (b) refuse (c) a nightmare
(c) accept (d) tolerate (d) an unknown person
Ans: (d) Ans: (a)
Q49. I did not mind what he was saying, he was only Q57. to run across
talking through his hat. (a) to have an appointed meeting
(a) talking nonsense (b) to meet by chance
(b) talking ignorantly (c) to run in the playground
(c) talking irresponsibly (d) to run very fast
(d) talking insultingly Ans: (b)
Ans: (a) Q58. to get one‘s own back
(a) to get one‘s revenge
Page 482 of 516
(b) to get control over someone
(c) to get one‘s position back
https://telegram.me/aedahamlibra
Q67. I am done for.
(a) ruined (b) rewarded
(d) to get hold of someone (c) answered (d) questioned
Ans: (a) Ans: (a)
Q59. to steer clear of Q68. For a healthy and lasting friendship one must be on
(a) drive carefully the level.
(b) avoid (a) equally rich
(c) explain clearly (b) mentally compatible
(d) escape (c) honest and sincere
Ans: (b) (d) ready for sacrifices
Q60. to beat a retreat Ans: (c)
(a) to withdraw in defeat or humiliation Q69. The foolish young man soon made ducks and
(b) to withdraw after scoring a victory drakes of the vast property his father left him.
(c) to march back after a ceremonious parade (a) squandered
(d) to run away in fear (b) distributed
Ans: (d) (c) spent
Q61. to blaze a trail (d) gave in charity
(a) to lead the way as a pioneer Ans: (a)
(b) to light a track Q70. All his ventures went to the winds.
(c) to set up a fire (a) dissipated
(d) to wear a blazer whi le running (b) spread all over
Ans: (a) (c) got speed of the winds
Q62. red-letter day (d) became well-known
(a) a colourful day Ans: (a)
(b) fatal day Q71. at one‘s wit‘s/wits' end
(c) happy and significant day (a) to work hard
(d) hapless day (b) to be intelligent
Ans: (c) (c) to get puzzled
Q63. have the last laugh (d) to be stupid
(a) be of a cheerful nature Ans: (c)
(b) laugh only after understanding something Q72. to take someone to task
(c) to be victorious at the end of an argument (a) to scold someone
(d) to crack the final joke (b) to assign work to someone
Ans: (c) (c) to take someone to his place of work
Q64. turn a deaf ear (d) to praise someone for the work done
(a) disregard (b) defy Ans: (a)
(c) disobey (d) dismiss Q73. to face the music
Ans: (a) (a) to be greeted rudely
Q65. to smell a rat (b) to be offered warm hospitality
(a) to experience bad smell (c) to enjoy a music programme
(b) to misunderstand (d) to bear the consequences
(c) to see a hidden meaning Ans: (d)
(d) to suspect a trick Q74. to blow one‘s own trumpet
Ans: (d) (a) to play on one‘s own trumpet
Q66. A few days before his death, he made a clean (b) to praise one‘s own self
breast of everything. (c) to create noisy disturbances
(a) confessed (d) to have a high-pitched voice
(b) took off his shirt Ans: (b)
(c) suffered Q75. to run one down
(d) spoke ill (a) to be in a hurry
Ans: (a) (b) to be weak and tired
Page 483 of 516
(c) to disparage someone
(d) to run down a lane
https://telegram.me/aedahamlibra
Q84. My close friend got the sack from his first job
recently.
Ans: (c) (a) resigned
Q76. at snail‘s pace (b) got rid of
(a) very slowly (c) was demoted from
(b) to walk like a snail (d) was dismissed from
(c) to lack interest in work Ans: (d)
(d) in methodical manner Q85. I can no longer put up with her insolence.
Ans: (a) (a) endure (b) evade
Q77. to turn a deaf ear (c) suppress (d) assume
(a) to be hard of hearing Ans: (a)
(b) to be indifferent Q86. She is a fair-weather friend.
(c) to be attentive (a) a good friend
(d) to be obstinate (b) a friend who meets difficulties calmly
Ans: (b) (c) a friend who deserts you in difficulties
Q78. to take to one‘s heels (d) a favourable friend
(a) to run off Ans: (c)
(b) to show one‘s heels Q87. to die in harness means to die while
(c) to turn around (a) riding a horse
(d) to walk leisurely (b) in a stable
Ans: (a) (c) in a uniform
Q79. to have something up one‘s sleeves (d) still in service
(a) having a practical plan Ans: (d)
(b) having an important project Q88. to keep under wraps means to keep something
(c) having an ambitious plan (a) covered (b) protected
(d) having a secret plan (c) unpacked (d) secret
Ans: (d) Ans: (d)
Q80. to end in smoke Q89. After independence Indian agriculture rose like a
(a) to have a smoking session phoenix due to the Green Revolution.
(b) to be on fire (a) with a new life
(c) to come to nothing (b) with a start
(d) to burn slowly (c) with royal gait
Ans: (c) (d) with vengeance
Q81. As the bomb exploded people ran helter-skelter. Ans: (a)
(a) in great fear Q90. His failure at the election has been a sore point
(b) in disorderly haste with him for a long time.
(c) in haste (a) something which hurts
(d) in great sorrow (b) something that brings fear to
Ans: (b) (c) something memorable for
Q82. He was progressing by leaps and bounds because (d) something pleasurable to
of his hard work. Ans: (a)
(a) rapidly (b) slowly Q91. The student is on the verge of breakdown.
(c) peacefully (d) strongly (a) on the brink of
Ans: (a) (b) at the outset of
Q83. Our founder had done a Herculean task by (c) in the midst of
constructing this great educational institution. (d) at the risk of
(a) a work of no worth Ans: (a)
(b) an effortless job Q92. My repeated attempts to get refund from the civic
(c) a work requiring very great effort authorities were of no avail.
(d) a work requiring very great intelligence (a) unsuccessful
Ans: (c) (b) postponed
Page 484 of 516
(c) useless
(d) delayed
https://telegram.me/aedahamlibra
(c) did not benefit me
(d) did not make me proud
Ans: (c) Ans: (a)
Q93. He was progressing by leaps and bounds because
of his hardwork.
(a) rapidly (b) slowly
Spelling Mistakes
(c) peacefully (d) strongly
Ans: (a) Directions (1-5): In these questions, four words are given
Q94. to emerge out of thin air means to out of which only one is misspelt. Find that mis -spelt
(a) appear suddenly word.
(b) descend gradually Q1. (a) combination (b) exageration
(c) fall down quickly (c) hallucination (d) admonition
(d) enter from space Ans: (b)
Ans: (a) Q2. (a) sacrosanct (b) sacrelege
Q95. The news of the accident came as a bolt from the (c) sacred (d) sacrament
blue. Ans: (b)
(a) something unexpected Q3. (a) allitration (b) allowanace
(b) something unpleasant (c) almighty (d) almanc
(c) something horrible Ans: (a)
(d) something unexpected and unpleasant Q4. (a) idiosyncrasy (b) idealize
Ans: (d) (c) idiosy (d) ideology
Q96. The story of the train accident as narrated by one of Ans: (c)
the survivors mademy flesh creep. Q5. (a) jaundise (b) jasmine
(a) thrilled me (c) javelin (d) jarring
(b) horrified me Ans: (a)
(c) excited me Q6. (a) eminent (b) iminent
(d) frightened me (c) eloquent (d) exuberant
Ans: (b) Ans: (b)
Q97. He has resigned his job and burnt his boats so far Q7. (a) tailor (b) sculpter
as government service is concerned. (c) doctor (d) fitter
(a) felt dejected Ans: (b)
(b) blasted his hopes Q8. (a) mathametician
(c) ruined himself (b) statistician
(d) left no means of retreat (c) dietician
Ans: (d) (d) physician
Q98. He cannot hold a candle to his elder brother. Ans: (a)
(a) equal to Q9. (a) grotesque (b) burlesque
(b) not as clever as (c) picturesque (d) pectorial
(c) cannot be compared to Ans: (d)
(d) duller than Q10. (a) Indomitable (b) furmidable
Ans: (c) (c) amicable (d) admirable
Q99. The question of higher membership fees was Ans: (b)
brought up at the last meeting. Q11. (a) submitted (b) admitted
(a) discussed at great length (c) comitted (d) omitted
(b) introduced for discussion Ans: (c)
(c) criticised vehemently Q12. (a) brillient (b) valiant
(d) vaguely referred to (c) salient (d) radiant
Ans: (b) Ans: (a)
Q100. His arguments cut no ice with me. Q13. (a) recuperate (b) regulate
(a) had no influence on me (c) reinstate (d) seperate
(b) did not hurt me
Page 485 of 516
Q14. (a) cease
https://telegram.me/aedahamlibra
(b) seize
Ans: (d) Q28. (a) scenary
(c) visionary
(b) granery
(d) luminery
(c) beseige (d) beseach Ans: (c)
Ans: (d) Q29. (a) colaborate (b) coroborate
Q15. (a) carrier (b) carreer (c) cooperate (d) colocate
(c) courier (d) barrier Ans: (c)
Ans: (b) Q30. (a) fullfil (b) ill-will
Q16. (a) personel (b) personnel (c) fabbulous (d) usefull
(c) notional (d) nationalist Ans: (b)
Ans: (a) Q31. (a) receprocate (b) recieve
Q17. (a) impetuous (b) impetinent (c) recetation (d) receipt
(c) imperial (d) implication Ans: (d)
Ans: (b) Q32. (a) traveling (b) remitance
Q18. (a) prefer (b) defer (c) kidnaping (d) equitable
(c) difer (d) refer Ans: (d)
Ans: (c) Q33. (a) necessery (b) temparory
Q19. (a) mercenary (b) machinery (c) itinerary (d) sanguinery
(c) missionery (d) visionary Ans: (c)
Ans: (c) Q34. (a) psycholagy (b) appology
Q20. (a) seize (b) decieve (c) criminolegy (d) archaeology
(c) believe (d) reign Ans: (d)
Ans: (b) Q35. (a) asociate (b) intigrate
Q21. (a) semblence (b) samblance (c) appropriate (d) exhilerate
(c) semblance (d) samblence Ans: (c)
Ans: (c) Q36. (a) mischevious
Q22. (a) gyncology (b) miscariage
(b) gynaecology (c) misdemeanour
(c) gynaeccology (d) misnomar
(d) gyneccology Ans: (c)
Ans: (b) Q37. (a) harassment
Q23. (a) hypochondria (b) committment
(b) hyppochondria (c) breevement
(c) hypochondrea (d) temparament
(d) hyppochondrea Ans: (a)
Ans: (a) Q38. (a) capracious (b) auspicious
Q24. (a) instanttaneous (c) fallicious (d) dalicious
(b) instantaneous Ans: (b)
(c) instanttanious Q39. (a) inefable (b) inaccesible
(d) instantanious (c) infallible (d) invinscible
Ans: (b) Ans: (c)
Q25. (a) itinarrary (b) itinerrary Q40. (a) camoflage (b) tabuleau
(c) ittinerary (d) itinerary (c) milieu (d) mirase
Ans: (d) Ans: (c)
Q26. (a) entirty (b) gratious Q41. (a) existence (b) occurence
(c) discern (d) contestent (c) depandance (d) persistance
Ans: (c) Ans: (a)
Q27. (a) reminiscence Q42. (a) sponsership (b) soveriegn
(b) renounciation (c) subservient (d) sepulcher
(c) recolection Ans: (c)
(d) relaxasion Q43. (a) handicaped (b) frolicked
Ans: (a) (c) kidnaped (d) developped
Page 486 of 516
Q44. (a) coersion
https://telegram.me/aedahamlibra
(b) precision
Ans: (b)
Q59. (a) irrepairable
Ans: (b)

(c) negociation (d) explotion (b) irreparable


Ans: (b) (c) irreparrable
Q45. (a) deliquancy (b) friquency (d) irepairable
(c) discrepency (d) hesitancy Ans: (b)
Ans: (d) Q60. (a) superfluous
Q46. (a) choronology (b) chronology (b) superflous
(c) cronology (d) chronalogy (c) superfluos
Ans: (b) (d) superflouss
Q47. (a) scintilating (b) scintillatinge Ans: (a)
(c) scintillating (d) scintilliting Q61. (a) pharmaceautical
Ans: (c) (b) pharmacutical
Q48. (a) nemesys (b) nemisis (c) pharmaceutical
(c) nemesis (d) nemysis (d) farmaceutical
Ans: (c) Ans: (c)
Q49. (a) anamoly (b) anomaly Q62. (a) recomendation
(c) anamaly (d) anomoly (b) reccomendation
Ans: (b) (c) recommendation
Q50. (a) perseverence (d) reccomandation
(b) perseverance Ans: (c)
(c) perseverense Q63. (a) neccesity (b) necessity
(d) perseverents (c)nesissity (d) necessety
Ans: (b) Ans: (b)
Q51. (a) competent (b) repentent Q64. (a) adrass (b) address
(c) penitent (d) consistent (c) addres (d) adrress
Ans: (b) Ans: (b)
Q52. (a) assure (b) insure Q65. (a) refridgerator
(c) sensure (d) ensure (b) refregerator
Ans: (c) (c) refreggerator
Q53. (a) seige (b) seize (d) refrigerator
(c) receive (d) believe Ans: (d)
Ans: (a) Q66. (a) grandly (b) effectively
Q54. (a) responsibility (c) genorally (d) normally
(b) oppertunity Ans: (c)
(c) possibility Q67. (a) trickery (b) uproar
(d) generosity (c) comotion (d) argument
Ans: (b) Ans: (c)
Q55. (a) courageous (b) stampede Q68. (a) benevolent
(c) temple (d) saliloquy (b) compassionate
Ans: (d) (c) generous
Q56. (a) consience (b) conscience (d) sympathatic
(c) consciens (d) consiens Ans: (d)
Ans: (b) Q69. (a) associate (b) traipse
Q57. (a) magnificent (b) magnificant (c) bruere (d) manhandle
(c) magnificient (d) magneficent Ans: (c)
Ans: (a) Q70. (a) despatch (b) attatch
Q58. (a) rennassance (c) detach (d) catch
(b) renaissance Ans: (b)
(c) rennaiscence Q71. (a) attendence (b) preference
(d) rennaissance (c) providence (d) evidence
Page 487 of 516
Q72. (a) acquire
https://telegram.me/aedahamlibra
(b) acquatic
Ans: (a) (c) magnificient (d) proficient
Ans: (c)
(c) acquittal (d) acquiesce Q87. (a) transferred (b) referred
Ans: (b) (c) sufferred (d) deferred
Q73. (a) therapeutic Ans: (c)
(b) bureaucretic Q88. (a) proceed (b) exceed
(c) thermometer (c) superseed (d) succeed
(d) barricade Ans: (c)
Ans: (b) Q89. (a) admonish (b) aborigin
Q74. (a) idyllic (b) idiotic (c) aberration (d) agrarian
(c) prolific (d) miopic Ans: (b)
Ans: (d) Q90. (a) confident (b) confiscate
Q75. (a) emerjency (b) delegate (c) conjecture (d) conversent
(c) mandatory (d) appreciate Ans: (d)
Ans: (a) Q91. (a) preposterous
Q76. (a) excution (b) excitment (b) disasterous
(c) expedition (d) extrection (c) murderous
Ans: (c) (d) onerous
Q77. (a) externel (b) extrovart Ans: (b)
(c) introvert (d) exect Q92. (a) severity (b) cruelity
Ans: (c) (c) sincerity (d) superiority
Q78. (a) expact (b) impact Ans: (b)
(c) exite (d) impect Q93. (a) begining (b) winning
Ans: (b) (c) mining (d) running
Q79. (a) effecting (b) interesting Ans: (a)
(c) affecting (d) entertening Q94. (a) complement
Ans: (b) (b) compliment
Q80. (a) supremecy (b) suppressor (c) supplement
(c) surfiet (d) surender (d) requirment
Ans: (b) Ans: (d)
Q81. (a) spectacular (b) spactroscope Q95. (a) amelirate (b) zealot
(c) spinache (d) splended (c) penetrate (d) stain
Ans: (a) Ans: (a)
Q82. (a) sacrosant (b) sacrificeal Q96. (a) courageous
(c) sacrilege (d) sabotege (b) outrageous
Ans: (c) (c) languoreous
Q83. (a) discription (b) discretion (d) spacious
(c) dessemination Ans: (c)
(d) dessertation Q97. (a) lapped (b) murmurred
Ans: (b) (c) deterred (d) worshipped
Q84. (a) occurance Ans: (b)
(b) occassion Q98. (a) sergent (b) silhouette
(c) occupancy (c) session (d) somnambulist
(d) octogenarean Ans: (a)
Ans: (c) Q99. (a) facade (b) inept
Q85. (a) humorous (c) quotation (d) pursuasive
(b) humanetarianism Ans: (d)
(c) hulabaloo Q100. (a) demeanour
(d) hurrecane (b) deodorize
Ans: (a) (c) demonstretor
Q86. (a) efficient (b) sufficient (d) demoralize
Page 488 of 516
https://telegram.me/aedahamlibra
Ans: (c)
Q4. (a) Advocates of space
Ans: (d)
programme argue for
spending huge amounts of money on exploring Mars.
Arrangement Of Sentences (P) But there is no firm evidence of any valuable mineral
that can be extracted from Mars and transp -orted to
Directions (1-5): In these questions, the 1st and the last Earth.
sentence of the passage are numbered 1 and 6. (Q) Worst, nobody has any idea what undesirable
The rest of the passage is split into four parts and named microbes or poisonous materials we will be importing
P, Q, R and S. from Mars.
These four parts are not given in their pro per order. Read (R) They are also unrealistic about the cost of
the sentences and find out which of the four combinations transportation that will be involved in interplanetary
is correct. move-ment of men and materials.
Q1. (a) The traveller said, ―Can you tell me the way to (S) These enthusiasts argue that Mars could be a
the nearest inn ?‖ perennial source of materials for us earthlings.
(P) ―Do you want one in which you can spend the night (6) Our race to Mars is likely to be a wild goose chase.
?‖ (a) SPRQ (b) QPRS
(Q) ―Yes‖, said the peasant. (c) PRSQ (d) SRQP
(R) ―Go right down the road and turn to the left.‖ Ans: (a)
(S) ―Yes‖, replied the traveller. Q5. (a) When a light passenger plane flew off course
(6) ―thank a lot.‖ sometime ago, it crashed in the mountains and its pilot
(a) PSRQ (b) QPSR was killed.
(c) SPQR (d) QSRP (P) Snow lay thick on the ground.
Ans: (b) (Q) It was the middle of winter.
Q2. (a) Money is not the root of all evils. (R) The woman knew that nearest village was miles
(P) Or, what about the desire for power driving people to away.
horrible crimes ? (S) The only passengers, a young woman and her two
(Q) Even purposeless cruelty can be seen in many baby daughters, were unhurt.
instances of evil behaviour. (6) When it grew dark, she turned a suitcase into a bed
(R) For example, neither teachers nor parents profit in and put the children inside it, covering them with all the
any way by torturing children. clothes she could find.
(S) The evils of sexual offenders are not motivated by (a) PQRS (b) QPRS
financial gains. (c) SQPR (d) RSPQ
(6) No, we ca n only say that money is the root of some Ans: (c)
evil. Q6. (a) The lead story
(a) QRPS (b) SPQR (P) at 4 AM
(c) RSPQ (d) QPRS (Q) in tonight's news
Ans: (a) (R) concerns the fire
Q3. (a) Even the smallest insect of the living world is (S) which engulfed the Columbia College
made up of a large number of cells. (6) this morning.
(P) Furthermore, the cells in these small creatures widely (a) RSPQ (b) QRSP
differ in their structure and function. (c) QRPS (d) RPSQ
(Q) An ant, or a gnat, for example is composed of Ans: (b)
hundreds of thou-sands of cells. Q7. (a) In less than a week
(R) Even a mite has cells making up its skeletal structure. (P) the processor controlled exchange
(S) It also has cells dedicated to digestive and (Q) have restored the functioning of
reproductive func-tions. (R) which had suffered
(6) The cell system of small organisms is microscopic but (S) the telecommunication people
marvellous, isn't it ? (6) a major disaster
(a) SRQP (b) QRSP (a) PRSQ (b) SRPQ
(c) SQPR (d) QPRS (c) PQSR (d) SQPR
Ans: (d)
Page 489 of 516
https://telegram.me/aedahamlibra
Q8. (a) As my jogging-enthusiast sister
(P) was often bothered by neighbourhood dogs
(a) SPRQ
(c) QRSP
(b) RPSQ
(d) PSRQ
(Q) with a stick in hand, Ans: (b)
(R) her husband started to accompany her on a bicycle, Q14. 1. By far the most logical step
(S) on her daily run, P. to relieve the housewife of routine
(6) to ward off any attackers. Q. which can be programmed
(a) SPQR (b) PSRQ R. to carry out standard operations
(c) RPQS (d) PRSQ S. is to provide a robot
Ans: (b) 6. when switched by the housewife.
Q9. (a) An electrical circuit (a) RSPQ (b) PSQR
(P) which is (c) QSPR (d) SRQP
(Q) of wires Ans: (b)
(R) designed to Q15. 1. During the reign of the Emperor Tiberius
(S) is a circle P. called Phaedrus
(6) carry electricity. Q. an Augustan story teller
(a) SQPR (b) QPRS R. translated Aesop‘s fables into Latin
(c) RQSP (d) PRSQ S. in ancient Rome
Ans: (a) 6. and also added some tales of his own.
Q10. (a) We have pleasure (a) QRPS (b) PQRS
(P) a double room with bath (c) SQPR (d) RSPQ
(Q) for five days from September 4 to September 8, Ans: (c)
(R) that we have reserved Q16. 1. The pigeons were used
(S) in informing you P. as messengers
(6) both days inclusive Q. which were tied
(a) RQPS (b) SRPQ R. in the olden days
(c) RPSQ (d) PQSR S. to carry messages
Ans: (b) 6. to their feet.
Q11. 1 A dictionary (a) PQRS (b) SPRQ
P. arranged words (c) PRSQ (d) PRQS
Q. about which information Ans: (c)
R. containing alphabetically Q17. 1. The school has always been
S. is a book P. tradition from one
6. is given. Q. the most important
(a) RPQS (b) QRPS R. the wealth of
(c) SRPQ (d) SPRQ S. means of transferring
Ans: (c) 6. generation to the next.
Q12. 1. Agriculture (a) PSRQ (b) QSRP
P. cotton for our clothes (c) RSQP (d) QRSP
Q. raw materials like jute Ans: (b)
R. and sugarcane for our industries Q18. 1. When she got to her hous e, there was nothing to
S. gives us food retrieve.
6. and food for cattle. P. All valuables were smashed or stolen
(a) SRQP (b) RQPS Q. The curtains were burned; books were ripped to shreds
(c) QPRS (d) SPQR R. Her medals and trophies had been flung everywhere
Ans: (d) S. The house had been completely ransacked
Q13. 1. Today political freedom 6. Mrs M stood in the centre of her bedroom looking at a
P. and the right ruined copy of the Koran forcing back her tears
Q. however money and all that money (a) PQRS (b) PRQS
R. assures people equality before law (c) SPRQ (d) RSQP
S. to elect their Government Ans: (a)
6. can buy is not fairly distributed. Q19. 1. The student came late to the school
Page 490 of 516
P. He went home weeping. https://telegram.me/aedahamlibra
Q. The watchman didn‘t allow him inside the school
Q23. 1. No one can deny that peasant forms the backbone
of the nation.
R. The boy was waiting outside for sometime. P. Hence he is the most useful member of the society.
S. He then decided to go home Q. Yet this fellow is exploited by the rich.
6. It was a bad day for him R. He grows food for the whole country.
(a) QSPR (b) QSRP S. It is our duty to improve his lot.
(c) QRSP (d) QPSR 6. We should grant him the social status he deserves.
Ans: (c) (a) RPQS (b) RSPQ
Q20. 1. Oliver dozed off again and it has been bright day (c) SRPQ (d) SPQR
for hours when Oliver opened his eyes. Ans: (a)
P. He belonged to the world again. Q24. 1. Priya went to the first counter at the post office
Q. In three day‘s time, he was able to sit in any easy as she needed stamps for six rupees.
chair, well propped up with pillows, and he was still too P. She was shown the corner where gum bottle was kept.
weak to walk Q. The woman behind the counter said it was registration
R. He felt cheerful and happy counter and directed her to the last counter.
S. The crisis of the disease was safely past R. She was looking for gum to affix the stamps on the
6. Mrs. Beduin had him carried downstairs into the little envelope.
housekeeper‘s room which belonged to her. S. She thanked the lady and came to the counter on the
(a) PQRS (b) RSPQ left extreme and got the stamps.
(c) QRSP (d) SPQR 6. She went to the corner, took two drops of gum, affixed
Ans: (c) the stamps and put the letter in the post box.
Q21. 1. The fox and the crane remained friends for a long (a) PSRQ (b) RPQS
time. (c) SQPR (d) QSRP
P. She served the dishes in a beaker to the fox. The fox Ans: (d)
could not eat it because the beaker was very high. Q25. 1. It is very misleading to say that computers can
Q. The crane could not eat the dishes because of its long ‗think‘ like people.
beak. The next day it was the turn of the crane to host the P. However, they make it possible for people to ‗bottle‘
lunch for the fox. thought.
R. But the fox wanted to show that he was cleverer than Q. They have no more a mind of their own than a lawn
the crane. mower.
S. So one day he invited the crane for dinner and served R. They can not.
the dishes on a plate. S. You work out how to do a particular job, write a
6. The fox put down its head in shame and went away. program and then the computer applies your thinking to
(a) PQRS (b) QSRP that job as long as you like.
(c) RSQP (d) PSQR 6. In this sense computers are half alive because they
Ans: (c) perpetuate thinking of their creators.
Q22. 1. There are examinations at school which a pupil (a) RQPS (b) PSRQ
can pass by cramming the texts. (c) SQPR (d) QSRP
P. But for spiritual knowledge mere memory of holy texts Ans: (a)
will be of no use in passing the tests. Q26. 1. India‘s uniqueness lies in its unity in diversity.
Q. One can score in them by the power of memory. P. So the problems of India should not be viewed in
R. A competent guru alone can provide the necessary isolation.
guidance to an earnest disciple. Q. Because of this factor, there are problems here and
S. What the text says has to be reflected upon and there at times.
experienced by the speaker. R. India is a multireligious, multicultural and multilingual
6. Thus, reading, reflection and experience are the three country.
stages in gaining spiritual knowledge. S. But even small countries with monolithic society have
(a) RSPQ (b) RSQP more problems.
(c) SRPQ (d) QPSR 6. And India is poised for success in all fields.
Ans: (d) (a) PSRQ (b) QSPR
(c) SRQP (d) RQSP
Page 491 of 516
https://telegram.me/aedahamlibra
Q27. 1. My friend went to live in a village.
Ans: (b) Q. Besides being expensive, smoking does injury to one‘s
health
P. But is was a very slow animal. R. In the long run he may get something worse -lung
Q. So my friend bought a donkey for Rs. 500 cancer
R. One day his new neighbour told him that he must buy S. A smoker gets nothing but smoke for his money
a donkey. 6. Then why get that bad habit?
S. Every family there had a donkey. (a) RPQS (b) QRPS
6. It did not like to work. (c) SPRQ (d) PRQS
(a) RSQP (b) PQRS Ans: (b)
(c) SRQP (d) QRSP Q32. 1. Phobic reactions are strong, irrational fears of
Ans: (c) specific objects or situations
Q28. 1. When Galileo went home, he began to P. But there is no objective danger
experiment with the pendulum. Q. For example, when a person is extremely fear ful of
P. When he showed it to his teachers, they were birds, snakes, heights or closed places, the label phobia is
delighted. applied to the person‘s fear and avoidance
Q. It was not long before physicians were all using the R. He usually recognizes that his fear is irrational
instrument to count the heart–beats of their patients. S. A person suffering from phobic neurosis knows what
R. Soon he had invented an instrument which marked the he is afraid of
rate of pulse-beats. 6. But he cannot control it.
S. Then the clock makers began to use the pendulum to (a) QPSR (b) SRQP
keep time. (c) SQPR (d) RSQP
6. Today it has many other uses. Ans: (a)
(a) PQRS (b) QRPS Q33. 1. In this life there are no gains without pains,
(c) SPQR (d) RPQS P. No victory is a real triumph unless the foe is worthy
Ans: (d) Q. Life, indeed, would be dull if there were no difficulties
Q29. 1. People have wrong calculation s about Japan‘s R. Both winner and loser enjo y a game most if it is
population. closely contested to the last
P. No, this is not true. S. Gainers lose their zest if there is no real struggle
Q. And old people die more often than the young. 6. Whether we like it or not, life is one continuous
R. There are more old people in Japan. competition.
S. The question is whether Japan has a lower death rate. (a) PQRS (b) QSRP
6. So it is very high in Japan. (c) QRSP (d) RSPQ
(a) QRPS (b) SPRQ Ans: (c)
(c) PRQS (d) RQSP Q34. 1. One Botany professor alway s tried to convince
Ans: (c) his students that his branch of biology is superior to all
Q30. 1. Pollution is one of the evils brought about by the the others
growth of science. P. His most persuasive argument, however, came during
P. Air pollution has very harmful effects. a laboratory session
Q. They pollute the air and the atmosphere. Q. And they don‘t eat very much
R. It is making the environment, water and air dirty. R. Examining the cells of a pear, the professor cut a slice
S. Factories and industries keep throwing out smoke for the microscopic slide and took a bite of the rest of the
which contains toxic gases. specimen
6. People living in the surroundings breathe the impure S. Plants, he noted never run away or bite
air and are affected by diseases of the lungs and heart. 6. ―You won‘t be doing that in a Zoology lab,‖ he said.
(a) PSQR (b) RPSQ (a) PRSQ (b) SQPR
(c) QPSR (d) SPQR (c) PSRQ (d) SQRP
Ans: (d) Ans: (a)
Q31. 1. A bad habit is harmful, none as harmful as Q35. 1. Failure is nothing to be ashamed of for there is
smoking. hardly any man who has not failed in life, not once but
P. But habit is second nature, smokers remain smokers many times.
for life P. What is important is the way we take our failure
Page 492 of 516
Q. It has been well said that he https://telegram.me/aedahamlibra
who never made a
mistake never achieved anything of great worth
R. but once they laid their eggs
S. to their limits
R. From the little ch ild who tries to stand up to the 6. they died.
would-be conqueror who tries to conquer some new (a) S Q P R (b) R S Q P
territory, everyone has to face failure (c) S P R Q (d) R P S Q
S. If we face our failure boldly and resolve to fight again Ans: (c)
we are sure to achieve victory in the long run Q41. 1. Take a small glass phial.
6. Thus failures can prove stepping stones in our march to P. Close the jar tightly with a plastic cover.
victory. Q. Place this phial inside a glass jar.
(a) RSQP (b) PQRS R. Fill it with coloured water.
(c) RPQS (d) RQPS S. Then stopper it tightly.
Ans: (d) 6. Make a hole in the cover.
Q36. 1. One of the most widely spread bad habits (a) PSRQ (b) QSPR
P. which is now smoked or chewed by men (c) RSQP (d) SRQP
Q. and even by children Ans: (c)
R. often by women Q42. 1. In China there is no man in the moon.
S. is the use of tobacco P. They are exchanged between friends while children
6. almost all over the world. receive toy pagodas made of clay.
(a) S P R Q (b) P Q R S Q. These cakes are circular to symbolise the full moon.
(c) S R Q P (d) P Q S R R. Instead, there i s a toad in the moon as well as moon
Ans: (a) rabbits and a goddess.
Q37. 1. The landscape S. All these appear as decorations on moon cakes, baked
P. with Nature displaying to celebrate the moon‘s birthday in September.
Q. here is awesome 6. The birthday marks the end of the harvest when debts
R. that are seldom are meant to be settled.
S. a range of delights (a) RQPS (b) RPQS
6. seen together (c) RSQP (d) RPSQ
(a) P S R Q (b) Q P S R Ans: (c)
(c) R S P Q (d) Q R S P Q43. 1. An observation home is called a formicarium.
Ans: (b) P. Then, if a small ant hill is dug up carefully, you will
Q38. 1. Smoke billowed up between the plants. find the hump-backed queen.
P. Passengers were told to be ready to quit the ship. Q. You will probably discover that you have some of the
Q. The rising gale fanned the smouldering fire. strange ant guests too.
R. Everyone now knew there was a fire on board. R. It can be made of two panes of glass separated by
S. Flames broke out here and there. strips of wood around the edges.
6. Most people bore the shock bravely. S. Put her in a jar with some of her workers, larvae and
(a) S R Q P (b) Q P S R cocoons.
(c) R S P Q (d) Q S R P 6. Carefully place them all in the formicarium.
Ans: (d) (a) RPQS (b) PQSR
Q39. 1. It is far better to live for a short while (c) PSQR (d) RPSQ
P. contribution to the world Ans: (d)
Q. and make some significant Q44. 1. When Ali Baba returned he called his wife
R. that is just idled away P. She said to him, ―Have you stolen them ?‖
S. than spend a long life Q. He said, ―I have bought you some jewels‖.
6. in gossiping and playing. R. She said, ―Our days of misery are now at an end‖.
(a) R Q S P (b) S Q P R S. He said, ―Be quiet and do not frighten yourself‖.
(c) Q P S R (d) R Q P S 6. He said, ―Go to your brother‘s house and get a
Ans: (c) measure‖.
Q40. 1. The salmon fish pushed themselves (a) QSRP (b) SRPQ
P. to return to their spawning grounds (c) PSRQ (d) QPSR
Q. and fertilised them Ans: (d)
Page 493 of 516
office.
https://telegram.me/aedahamlibra
Q45. 1. The head of the family returned home from
Q49. 1. Guru is a university professor.
Ans: (c)

P. Wife told that there was no coffee powder. P. It was about strange beings called KUNUS who live in
Q. Wife again told that there was no milk either. holes in the ground.
R. Husband wanted atleast a cup of tea. Q. The book is very popular now.
S. He wanted a cup of coffee. R. Thirty years ago he wrote a strange novel called
6. Husband told finally that a glass of water was enough. ―Queen of the Mars‖.
(a) SQRP (b) SPRQ S. He is also a famous writer.
(c) QSRP (d) RPSQ (6) In a recent interview on television Prof. Guru talked
Ans: (b) about the novel.
Q46. 1. When he was quite young, Le Corbusier became (a) PRSQ (b) QPRS
interested in art. (c) SRPQ (d) RQSP
P. At the age of nineteen, he travelled around Europe. Ans: (c)
Q. But the buildings which impressed him most were Q50. 1. It was nine o‘clock in the evening and Rajan was
those of the ancient Greeks in Athens. reading.
R. At the age of thirteen, he went to an art school. P. At first he thought nothing of it.
S. Everywhere he went he admired the magnificent Q. The walls were a moving mass of big ants.
buildings of the past. R. Suddenly, he heard faint noises.
(6) After his visit to Athens Le Corbusier decided to S. When he went to his bedroom later, however, he was
become an architect. shocked by what he saw.
(a) RPSQ (b) PSRQ (6) They covered everything — the book case, the
(c) RQPS (d) QPSR shelves, the chest of drawers.
Ans: (c) (a) PSRQ (b) SRPQ
Q47. 1. I was in awe of Einstein and hesitated before (c) RPSQ (d) QSRP
approaching him about some ideas I had been working Ans: (c)
on. Q51. 1. Abraham worked very hard and had no time to
P. I entered his office and found him seated at a table, feel lonely.
calculating and smoking his pipe. P. Abraham was very fond of books.
Q. When I finally knocked on his door, a gentle voice Q. When his day‘s work in the f ields or in the forest was
said, ‗come‘. over he settled down in the evenings to read by the light
R. The single word was both a welcome and a question. of the fire.
S. Dressed in ill fitting clothes, his hair characteristically R. She used to sit by the fireside in the evenings and tell
awry, he smiled a warm welcome. him stories.
(6) His utter naturalness at once set me at ease. S. His mother had taught him to read when he was very
(a) QPRS (b) QRPS young.
(c) PQRS (d) SRQP 6. Every evening he woul d spend his time in reading all
Ans: (a) the books he could find.
Q48. 1. Nothing comes out of nothing. (a) QSRP (b) SPRQ
P. We have to work and then alone we can gain (c) PSRQ (d) PRSQ
something. Ans: (c)
Q. It is honest and conscientious labour alone that Q52. 1. The tiny bacterial plants that live in the soil help
produces result. to prepare food for the plants we cultivate.
R. Millions have been struck with the lure of lottery to P. The farmer works very hard to make the soil
utter despondency. favourable.
S. A person who thinks that luck would favour him with Q. But these soil bacteria are very necessary and helpful.
all the wants of his life without his lifting his finger even, R. There are millions of bacteria in a cubic inch of fertile
is living in a fool‘s paradise. soil.
(6) A p ainstaking man who adopts honest toil as his way S. Some kinds of bacteria are harmful.
of life, makes the most of it. 6. They donot need sunlight as do most plants.
(a) PQRS (b) PSRQ (a) SQPR (b) QSPR
(c) QPRS (d) RSPQ (c) RPSQ (d) PRQS
Page 494 of 516
https://telegram.me/aedahamlibra
Q53. 1. My uncle Martin went to live in a hamlet.
Ans: (c) (c) PQRS (d) SPRQ
Ans: (d)
P. But it was a very lazy parrot. Q57. 1. Judo champ, Tamura doesn‘t look or act tough.
Q. So martin bought a parrot. P. At 4 feet 9, Tamura is the shortest woman in the light -
R. Martin‘s neighbour told him that he must buy a parrot. weight class (106 pounds).
S. Every home there had a parrot as a custom. Q. Fans know her affectionately as ―Yawara-chan‖ after a
6. The parrot did not like to speak. spunky cartoon character.
(a) RSQP (b) SRPQ R. She wears a lucky pink ribbon and at 20, still grins
(c) PQSR (d) QPSR like a carefree teen and gushes about icecream.
Ans: (a) S. No wonder the Japanese go wild when she tosses
Q54. 1. Hailstones consist of many onion- opponents, many of whom tower over her. 6. Tamura has
like layers of ice. become a national idol in Japan.
P. The process continues until the hailstone is too heavy (a) PQRS (b) SRPQ
to be lifted and then it drops to the earth. (c) SRQP (d) QPRS
Q. In certain weather conditions small ice crystals drop to Ans: (b)
form a crystal. Q58. 1. It‘s only in the last three years that we have seen
R. Some of the moisture freezes on to the crystals the rebirth of T.B.
forming another layer. P. What bothers experts is the emergence of particularly
S. Updrafts carry the hailstones and when it drops patent strains of the T.B. bacteria that are resistent to two
another layer is formed. or more anti-
6. That is how hailstones are formed. T.B. drugs.
(a) QPRS (b) QPSR Q. This is compounded by the fact that the symptoms
(c) QRSP (d) QSRP disappear in about two months.
Ans: (a) R. Three or four anti -T.B. drugs are available at no cost
Q55. I shall tell you about the ways you can see a in Government clinics.
rainbow. S. Yet, the long period of treatment leads to a high rate of
P. Big rainbows can be seen when the sun is close to non-compliance with the treatment. 6. In such cases, a
horizon. relapse occurs and the bacteria appear in more virulent
Q. Or you can notice a rainbow in the spray from a and drug-resistant.
garden hose. (a) PQRS (b) SRQP
R. You can see a rainbow in the mist from a waterfall. (c) QPRS (d) RSPQ
S. When you stand with a light source behind you and Ans: (a)
misty water before you, you can see a rainbow. Q59. 1. ‗Acu‘ means needles in Latin.
6. Occasionally, even a full moon on a rainy night will P. The needles stimulates specific nerves that t ransmit
create a faint rainbow. electrical impulses via the spinal cord and brain to the
(a) SPRQ (b) SRQP affected area.
(c) SPQR (d) SQRP Q. Quite appropriately, then, acupuncture consists of
Ans: (a) inserting very fine needles at specific points on the skin
Q56. Nobody in the ir mind would doubt that America located near nerve endings.
has problems. R. Acupuncture also stimulates the release of chemical
P. Leave out euphoria, after the Gulf War and that is the substances from the brain centres and pitulary glands.
highest such figure for six years. S. These are connected to one another by lines called
Q. And whatever the gloomy politicians may think, channels or meridians. 6. Known as endorphins and
Americans themselves seem to sense that. encephalins, which are released and carried across the
R. But these are problems , things that can and will be blood stream, these ch emicals are the body‘s own pain –
solved— they are not the stuff of national crisis. relief mechanism.
S. The latest Time/CNN poll says 62% of Americans (a) SPRQ (b) RSQP
thought their nation was doing ―fairly well‖ or ―very (c) PQRS (d) QSPR
well‖ 6. All it needs now is for politicians to catch the Ans: (d)
new mood of optimism. Q60. 1. The dead do sometimes tell tales, if you know
(a) QPSR (b) RSQP how to look for them.
Page 495 of 516
P. The flesh of bomb victims is
sieged by chemicals.
https://telegram.me/aedahamlibra
shredded and may be (6) Thus in a democracy, the responsibilities
people are great.
of the

Q. In the autopsy rooms of the Suffolk country; the (a) SQPR (b) QPSR
medical examiner and his team were looking for clues (c) QSPR (d) QSRP
that could explain how the passengers of TWA Flight 800 Ans: (c)
died. Q64. (a) Satellites have been launched into space for
R. But most of the corpses he examined had been killed various purposes.
by the impact of hitting the water from a height of more (P) The other satellites we have launched are the
than two miles. Bhaskara, Apple and Insat-IA, IB, IC.
S. The body of a person killed by a bomb looks different (Q) We have launched our first satellite Aryabhatta on
from the body of a victim in an ordinary plane crash. 19th April, 1975.
6. The mystery of their deaths will be solved in time, but (R) Our latest achievement is the launching of PSLV
it won‘t be easy or quick. rocket.
(a) SRQP (b) PQRS (S) Therefore in satellite technology, we are able to
(c) QSPR (d) RSPQ compete with other developed countries.
Ans: (c) (6) Only a few other countries have developed satellite
Q61. (a) Fighting a fire demands a lot of strength and technology.
endurance. (a) QPRS (b) QRPS
(P) The breathing unit may weigh as much as 25 (c) SQPR (d) SQRP
kilograms. Ans: (a)
(Q) The protective clothing the firefighter wears will Q65. (a) National Integrity means National Unity for all.
weigh about 10 kilograms. (P) They are the evils of Communalism and Regionalism
(R) The fire -fighter will normally wear an oxygen mask (Q) Our Government is taking st eps to remove such
and carry an oxygen tank. tendencies.
(S) Apart from these, he will have to carry the hose and (R) But there are some evil tendencies in our society to
other tools. hamper our unity.
(6) The weight of the hose and other tools, the fire-fighter (S) The feeling of Indianness should be achieved to
carries, will be around 50 kilograms. preserve our unity.
(a) QPSR (b) QRPS (6) And this is what every Indian should aim at.
(c) QSPR (d) SPRQ (a) SRQP (b) QPRS
Ans: (b) (c) QRPS (d) QSPR
Q62. (a) We were taking tea at the Rathna Tea Stall. Ans: (a)
(P) We found a tourist bus which had just rammed into a Q66. 1: Most people are afraid of snakes.
tamarind tree on the roadside. P: There may, be some truth in this theory, because
(Q) We rushed out of the tea stall. Monkeys have a deep, instinctive fear of pythons and
(R) We helped the passengers to get out of the bus. other tree snakes.
(S) Suddenly we heard a loud noise followed by a cry for Q: But this fear is as irrational as the fear of ghosts.
help. R: Any way, sna kes have been feared and hated for
(6) We informed the hospital and also the police about thousands of years.
accident. S: The fear of snakes, according to some biologists, may
(a) SQPR (b) QSRP be an instinct passed on to us by our ancestors.
(c) PRSQ (d) RSQP 6: In the literature of many countries the snake is
Ans: (a) regarded as a symbol of evil.
Q63. (a) Democracy is the primary goal of our Indian (a) PQRS (b) QPSR
Constitution. (c) RQSP (d) QSPR
(P) If representatives do not rule according to the wishes Ans: (d)
of the people, they are changed in the next election. Q67. 1: One of the reasons why people wear clothes is to
(Q) In a democracy, the people are the rulers. protect their bodies.
(R) New representatives who are aware of the needs of P: In cold countries, on the other hand, people wear
the people take their place. woollen clothes which keep the body warm.
(S) They rule through their elected representatives.
Page 496 of 516
https://telegram.me/aedahamlibra
Q: Besides cotton and wool, new fibres such as nylon and
rayon are also used today for clothes.
Q. Then Deesa looked at his feet and examined his whole
body for sores.
R: In hot countries like India, people wear clothes made R. The animal knew it was time to return.
of cotton which are cool. S. The elephant lay down on his side, while Deesa rubbed
S: The body has to be protected from cold and heat. him with a coir scrubber.
6: We can say, therefore, that all our clothes are made 6. Both the elephant and the trainer would return home.
from three different materials- (a) SQPR (b) QSRP
animal fur or skins, plant fibres and artifical fibres. (c) QPRS (d) RQSP
(a) PQRS (b) SRPQ Ans: (a)
(c) QSRP (d) PSRQ Q72. 1.Anna had longed to see her son.
Ans: (b) P. ―He will arrive at the station at 10 O‘ clock‖, she said
Q68. 1: A man should give the same care to himself that to herself.
he gives to his car. Q. She prepared herself for it.
P: And sooner or later there comes a complete break R. She looked at the clock.
down. S. There were only five minutes left.
Q: -Everyday tens of thousands of men are trying to work 6. She rushed out of her house and hailed a taxi to reach
when their bodies and minds are in need of repair work. the station in time.
R: For worry pulls down the mind and fatigue pulls down (a) QPRS (b) SPRQ
the body. (c) PRQS (d) QSRP
S: He does not try to drive his car when there is Ans: (a)
something wrong with it, he has to put it in order. Q73. 1. Mr. Ramaswamy is a very strict man.
6: Man should realise that most worry and fatigue can be P. He earns nearly three thousand rupees a month.
prevented. Q. He also believes that it is foolish to waste one‘s time
(a) PSRQ (b) QPSR or money.
(c) RSQP (d) SQPR R. He is not a poor man.
Ans: (d) S. He believes that life means work only.
Q69. 1: In our home everyone drinks milk at least once a 6. But he wants his children to lead a simple life.
day. (a) PSQR (b) SQRP
P: All these we owe to our milkman Q: We, the children (c) RQPS (d) SRQP
get milk twice a day. Ans: (b)
R: He says, he is 18, but he is not sure. Q74. 1. It will be better
S: We also eat curd, and enjoy buttermilk. P. to a few than enrol
6: Probably he needs more milk than we. Q. to provide quality education
(a) SRPQ (b) QSPR R. them out as graduates
(c) PRQS (d) PSRQ S. in masses and churn
Ans: (b) 6. after perfunctory teaching
Q70. 1: One of his greatest successes was to improve the (a) QRSP (b) RSPQ
water supply. (c) QPSR (d) SRQP
P: The lepers could obtain it for filling a vessel at a Ans: (c)
mountain stream. Q75. 1. ‗I was born here in the old city‘ the girl told us.
Q: They carried it to the village on their sore covered P. her answer suggested that her family has roots
shoulders. Q. When we inquired
R: Water was scarce. R. as opposed to the modern towns that consist mostly of
S: They had to go some distance to wash their clothes. hotels.
6: That was one reason they remained dirty as often S. and belongs to the traditional part where the temples
(a) RPQS (b) PQRS are
(c) QPRS (d) RQPS 6. Some say people here are more ethnically pure.
Ans: (a) (a) QPSR (b) SRPQ
Q71. 1. Once a week Deesa led Moti Guj, the elephant, (c) PSRQ (d) SRQP
down to the river. Ans: (a)
P. After inspection the two would stand up. Q76. 1. Making ourselves
Page 497 of 516
P. our language
Q. part of growing into
https://telegram.me/aedahamlibra
6. Mrs. M stood in the centre of her bedroom looking at a
ruined copy of the Holy book, forcing back her tears.
R. masters of (a) PQRS (b) PRQS
S. is an important (c) SPRQ (d) RSQP
6. full manhood or womanhood Ans: (c)
(a) PSRQ (b) SQPR Q82. 1. We do n ot know whether the machines are the
(c) RPSQ (d) PRSQ masters or we are.
Ans: (c) P. They must be given or rather ‗fed‘ with coal and given
Q77. 1. The very first battle they fought petrol to drink from time to time.
P. and they had to fall back Q. Already man spends most of his time looking after and
Q. cross the border waiting upon them.
R. was lost R. Yet we have grown so depende nt on them that they
S. letting the enemy have almost become the masters now.
6. and enter the country S. It is very true that they were made for the sole purpose
(a) RQSP (b) RPSQ of being man‘s servants.
(c) QRPS (d) QPRS 6. And if they don‘t get their meals when they expect
Ans: (b) them, they will just refuse to work.
Q78. 1. A nation (a) RSQP (b) RSPQ
P. the material assets it possesses (c) SPQR (d) SRQP
Q. is not made by Ans: (d)
R. and collective determination Q83. 1. The king of the oilfield is the driller.
S. but by the will P. During the process of drilling, gas and oil may be met.
6. of the people Q. He is a very skilled man.
(a) PQRS (b) QPSR R. If this rushes out and catches fire it is dangerous.
(c) RSPQ (d) SRPQ S. Sometimes he sends his drill more than a mile.
Ans: (b) 6. This danger is well -known and steps are taken to
Q79. 1. When the Governor prevent it.
P. the bell had rung (a) PQRS (b) QSPR
Q. justice should be immediately (c) QPRS (d) QSRP
R. he ordered that Ans: (b)
S. found out why Q84. 1. Freedom is first of all a personal matter.
6. done to the horse P. A man who will not submit to the discipline of his
(a) RSPQ (b) PQSR chosen occupation is not free to be a great surgeon, an
(c) SPRQ (d) SQRP engineer, or a golfer or an executive.
Ans: (c) Q. Life imposes a drastic discipline on all living things,
Q80. 1. When you ponder over including human beings.
P. that the only hope R. We are free to eat poison or jump off a tall building,
Q. you will realize but not to escape the consequences.
R. of world peace lies S. We are bound by the laws of cause and effect.
S. the question deeply 6. Nature, morever, binds the arbitrary limits of mind and
6. in the United Nations body; we are not free to do, by whatever effort, what is
(a) QRSP (b) SPQR beyond our capacity.
(c) SQPR (d) RSPQ (a) QRSP (b) RSPQ
Ans: (c) (c) PQRS (d) SRQP
Q81. 1. When she got to her house, there was nothing to Ans: (d)
retrieve. Q85. 1. The student came late to the school.
P. All valuables were smashed or stolen. P. He went home weeping.
Q. The curtains were burned; books were ripped to Q. The watchman didn‘t allow him inside the school.
shreds. R. The boy was waiting outside for some time.
R. Her medals and trophies had been flung everywhere. S. He then decided to go home.
S. The house had been completely ransacked. 6. It was a bad day for him.
Page 498 of 516
(a) QSPR
(c) QRSP
https://telegram.me/aedahamlibra
(b) QSRP
(d) QPSR
Q91. 1. Early to bed, early to rise, makes a man healthy,
wealthy and wise.
Ans: (c) P. But for the morning tea, I had to wait for someone to
Q86. (a) Superstition and get up before me.
(P) the supposed powers Q. This saying inspired me to rise early.
(Q) thrive on R. That day I was the first to get up.
(R) magical practices S. One day I got up early in the morning.
(S) of dreams to 6. One day I realised that it was a waste of time to get up
(6) foretell the future. early and wait for the morning tea.
(a) SPQR (b) PSQR (a) QSRP (b) QPRS
(c) RSQP (d) RQPS (c) PQRS (d) SPQR
Ans: (d) Ans: (a)
Q87. (a) The stronger Q92. 1. A wood-cutter was cutting a tree on a river bank.
(P) the phosphor P. He knelt down and prayed.
(Q) the more light Q. His axe slipped and fell into the water.
(R) of electrons R. God Mercury appeared before him and asked about
(S) the beam the matter.
(6) gives out. S. He could not get it back as the river was very deep.
(a) RPQS (b) SQRP 6. He dived into the water and came up with an axe of
(c) SRQP (d) RQPS gold.
Ans: (c) (a) RPQS (b) RPSQ
Q88. (a) There are thousands of us (c) QSRP (d) QSPR
(P) former school and college friends Ans: (d)
(Q) by some of our Q93. 1. A dog stole a piece of a meat from a butcher‘s
(R) at the careers chosen shop.
(S) who are surprised P. He barked in anger.
(6) and their success in these fields. Q. He ran to the jungle with the piece of meat.
(a) SRQP (b) RQSP R. He saw his reflection.
(c) PQSR (d) SPQR S. He crossed a river on the way.
Ans: (a) 6. He lost his piece of meat.
Q89. (a) Ramani is a student of medicine. (a) QPSR (b) QSRP
(P) The hopes of millions of cancer patients and doctors (c) QPRS (d) SRPQ
rest on his research. Ans: (b)
(Q) But, of late he has become a drug addict. Q94. 1. Ramai and Samai were two poor young men.
(R) He is doing research in cancer. P. On market day they sold their labour.
(S) He has already done very useful work in this field, Q. The lived near Mahespur.
and is hopeful of finding a solution to this disease. R. On other days, they remained in the village looking for
(6) This addiction has been increasing day by day, and work.
has started affecting his work. S. They wanted regular work.
(a) QPSR (b) RSPQ 6. The headman gave them two plots.
(c) SRPQ (d) RSQP (a) QPRS (b) RPQS
Ans: (b) (c) SPQR (d) PQRS
Q90. (a) The internet has given Ans: (a)
(P) to keep in touch with friends Q95. 1. Roger wanted to become a doctor.
(Q) and even allowed them P. He put away enough money to pay his first year fees.
(R) students access to reams of information Q. They could not afford the fees.
(S) made it cheaper R. Undaunted, he got himself a job in the dockyard.
(6) to attend universities remotely S. However, he came from a poor family.
(a) RPSQ (b) RSPQ 6. Once enrolled, he was recognised as a gifted student,
(c) SRPQ (d) PRSQ and scholarships took care of the rest of this studies.
Ans: (b) (a) SRPQ (b) PRSQ
Page 499 of 516
(c) SQRP https://telegram.me/aedahamlibra
(d) QRSP
Cloze Test
Ans: (c)
Q96. 1. I went to my friend last week.
P. He politely refused to oblige me.
Directions: In the following passage, some of the words
Q. I did not speak even a single word.
have been left out. First read the passage over and try to
R. Actually I wanted his scooter for a day.
understand what it is about. Then fill in the blanks with
S. I felt ashamed of my self.
the help of the alternatives given.
6. I was mistaken in assessing a true friendship.
The library, if used properly, is invaluable in helping you
(a) RPQS (b) PRQS
develop discernment.
(c) SRPQ (d) QRSP
It is rich _____information that goes far beyond the
Ans: (a)
_____ of any one text book or course. _____ your text
Q97. 1. He wanted to adopt his father‘s profession.
book au thor does not make a particular _____ clear and
P. He was influenced by his strong desire to see India
you feel the need _____ another description in greater
free.
detail _____ in other words, go to the library and _____
Q. From now on, he was a changed man.
other books on the subject. By calling _____ two or three
R. He made up his mind.
writers dealing with the _____ topic, you will find some
S. He came in contact with Mahatma Gandhi.
_____ of certain facts. In seeking additional sources, you
6. He wished to change the lot of the naked and hungry
will have gained immeasurably, for you will have seen
masses of India.
what several experts perceive as being particularly
(a) QRPS (b) SPRQ
important on a common subject.
(c) RSPQ (d) PQSR
Q1. (a) of (b) for
Ans: (b)
(c) in (d) with
Q98. 1. Reena made a cup of tea in this manner.
Ans: (c)
P. Next, she added milk and sugar.
Q2. (a) pages (b) limits
Q. When the water was boiling she added tea-leaves.
(c) confines (d) limitations
R. She turned off the gas.
Ans: (b)
S. First she put the water to boil.
Q3. (a) If (b) While
6. Finally, she poured the tea into cup.
(c) When (d) Suppose
(a) SQPR (b) QPRS
Ans: (a)
(c) PRSQ (d) RSQP
Q4. (a) information (b) entry
Ans: (a)
(c) explanation (d) point
Q99. 1. Once upon a time, there was a little man.
Ans: (d)
P. Some people called him Rabi.
Q5. (a) for (b) of
Q. He walked like a rabbit.
(c) to (d) about
R. His face and hands were brown.
Ans: (a)
S. That is why people called him Brownie.
Q6. (a) or (b) but
6. But his real name was Thomas Cook, though he never
(c) though (d) however
cooked anything.
Ans: (a)
(a) QPSR (b) SRPQ
Q7. (a) verify (b) identify
(c) RSQP (d) RQPS
(c) check (d) collect
Ans: (c)
Ans: (c)
Q100. 1. She was an old woman with a large purse that
Q8. (a) for (b) up
had everything in it.
(c) forth (d) upon
P. It was about eleven O‘clock at night.
Ans: (b)
Q. It had a long strap.
Q9. (a) actual (b) specific
R. She carried it slung across her shoulder.
(c) correct (d) same
S. A boy ran up behind her.
Ans: (d)
6. He tried to snatch her purse.
Q10. (a) description (b) clarification
(a) PQRS (b) SQRP
(c) explanation (d) evidence
(c) QRPS (d) SRQP
Ans: (b)
Ans: (c)
Page 500 of 516
https://telegram.me/aedahamlibra
Directions: In the following passage, some of the words
have been left out. First read the passage over and try to
understand what it is about. Then fill in the blanks with
the help of the alternatives given.
understand what it is about. Then fill in the blanks with Civilization, like (_____), fall, not so much because of
the help of the alternatives given. (_____) of the enemy outside, as through the weakness
Mass communication i s the delivery of _____, ideas and and
entert-ainment to thousands or millions of _____ (_____) within, Rome fell not because of the (_____),
simultaneously; it is a force with incalculable _____ on they merely knocked (_____) something that was already
today‘s world. The _____ of mass comm -unication rests dead. The heart of Rome had (_____) beating when the
_____ the skills of the communicator to _____ the arms and legs were cut (_____). We see something of this
recipient‘s thinking, _____ stir emotions, to _____ him or (_____),. in India and China and in the case of the Arabs.
her to _____. Mass communication is the one -to-one The (_____) of Arabian civilization was sudden even as
impact of one human intelligence upon _____, carried on their rise had been.
thousand fold simultaneously among individuals who In India and China the process is longdrawn
have no direct personal contact. (_____) and it is not easy to spot it.
Q11. (a) items Q21. (a) states (b) kingdom
(b) news (c) empires (d) buildings
(c) advertisements Ans: (c)
(d) cartoons Q22. (a) weakness (b) strength
Ans: (b) (c) power (d) tenacity
Q12. (a) People Ans: (b)
(b) pictures Q23. (a) disease (b) decease
(c) advertisements (c) decay (d) fortitude
(d) items Ans: (a)
Ans: (a) Q24. (a) barbarity (b) barbarous
Q13. (a) worth (b) impact (c) barbarians (d) Romans
(c) force (d) value Ans: (d)
Ans: (b) Q25. (a) off (b) on
Q14. (a) dynamism (b) force (c) down (d) no word
(c) success (d) power Ans: (c)
Ans: (c) Q26. (a) seized (b) rotated
Q15. (a) against (b) under (c) ceased (d) started
(c) to (d) upon Ans: (c)
Ans: (d) Q27. (a) down (b) off
Q16. (a) stimulate (b) agitate (c) up (d) no word
(c) understand (d) minimize Ans: (b)
Ans: (a) Q28. (a) process (b) procedure
Q17. (a) for (b) to (c) spectacle (d) scene
(c) with (d) from Ans: (c)
Ans: (b) Q29. (a) downfall
Q18. (a) encourage (b) persuade (b) death
(c) discourage (d) endanger (c) collapse
Ans: (a) (d) dismemberment
Q19. (a) direction (b) function Ans: (a)
(c) action (d) mission Q30. (a) out (b) up
Ans: (b) (c) upon (d) no word
Q20. (a) another (b) people Ans: (a)
(c) man (d) women Directions: In the following passage, some of the words
Ans: (a) have been left out. First read the passage over and try to
Directions: In the following passage, some of the words understand what it is about. Then fill in the blanks with
have been left out. First read the passage over and try to the help of the alternatives given and indicate the correct
alternative.
Page 501 of 516
https://telegram.me/aedahamlibra
Man has been tampering _____ the Ecosphere for a very
_____ time and is forced to _____ that the environmental
rabbits. _____ he is lord of all the animals. He sets them
to work, he gives back to them the bare _____ that will
resources are _____. Environmental problems are _____ prevent them from _____ and the rest he keeps for
social problems. They begin with people as _____ and himself.
end with people as victims. Unplanned use of resource Q41. (a) area (b) scene
_____ resulted in the depletion of fossil fuels, _____ of (c) place (d) light
air and water, deforestation which has resulted in _____ Ans: (b)
imbalance and draining away of national wealth _____ Q42. (a) overwork (b) work
heavy expenditure on oil and power generation. (c) while (d) waste
_____ CLOZE TEST Ans: (b)
Q31. (a) to (b) in Q43. (a) body (b) worm
(c) with (d) for (c) human (d) creature
Ans: (c) Ans: (d)
Q32. (a) short (b) long Q44. (a) drinking (b) producing
(c) small (d) tall (c) eating (d) sleeping
Ans: (b) Ans: (b)
Q33. (a) see (b) look Q45. (a) does (b) do
(c) do (d) recognise (c) did (d) has done
Ans: (a) Ans: (a)
Q34. (a) plenty (b) scarce Q46. (a) too idle (b) too strong
(c) minute (d) enough (c) too weak (d) too quick
Ans: (b) Ans: (c)
Q35. (a) really (b) coldly Q47. (a) slow (b) fast
(c) badly (d) happily (c) idle (d) dull
Ans: (a) Ans: (b)
Q36. (a) effect (b) result Q48. (a) Yet (b) But
(c) cause (d) wisdom (c) Then (d) Thus
Ans: (c) Ans: (a)
Q37. (a) have (b) had Q49. (a) Maximum (b) Minimum
(c) being (d) has (c) Average (d) Capacity
Ans: (d) Ans: (b)
Q38. (a) revolution (b) pollution Q50. (a) Producing (b) Creating
(c) resolution (d) evolution (c) Eating (d) Striving
Ans: (b) Ans: (d)
Q39. (a) ecological (b) biological Directions: In the following passage, some of the words
(c) logical (d) chronological have been left out. First read the passage over and try to
Ans: (a) understand what it is about. Then fill in the blanks with
Q40. (a) by (b) in the help of the alternatives given.
(c) out (d) through Pythons are _____ non -venomous snakes found _____ in
Ans: (d) damp places or rocky ledges near marshes or in dense
Directions: In t he following passage, some of the words _____. They are considered to be the most primitive
have been left out. First read the passage over and try to _____ the living snakes, _____ they retain the traces of
understand what it is about. Then fill in the blanks with hind limbs of a _____ era in the form ___ __ horny claw-
the help of the alternatives given. Mark you answer in the like spurs, which are present _____ either side of _____
Answer Sheet. anus. These hind limbs are functionless _____ female
It is summed up in a singl e word – Man. Man is the only pythons.
real enemy we have. Remove man from the _____ and Q51. (a) small (b) tiny
the root cause of hunger and _____ is abolished forever. (c) cute (d) huge
Man is the only _____ that consumes without _____. He Ans: (d)
does not give milk, he _____ not lay eggs, he is _____ to Q52. (a) mostly (b) rarely
pull th e plough, he cannot run _____ enough to catch (c) occasionally (d) hardly
Page 502 of 516
Q53. (a) cities
https://telegram.me/aedahamlibra
(b) jungles
Ans: (a) (c) into (d) over
Ans: (b)
(c) hills (d) towns Q65. (a) swordfishes
Ans: (b) (b) penknives
Q54. (a) between (b) in (c) featherweights
(c) among (d) over (d) ploughshares
Ans: (c) Ans: (b)
Q55. (a) so (b) because Q66. (a) miracle (b) masterpiece
(c) and (d) but (c) legend (d) model
Ans: (b) Ans: (b)
Q56. (a) by gone (b) latest Q67. (a) battleaxe (b) spear
(c) present (d) recent (c) sword (d) stick
Ans: (a) Ans: (a)
Q57. (a) in (b) for Q68. (a) slaughter (b) penalise
(c) on (d) of (c) reform (d) belabour
Ans: (d) Ans: (a)
Q58. (a) over (b) above Q69. (a) bomb (b) science
(c) on (d) in (c) weaponry (d) research
Ans: (c) Ans: (c)
Q59. (a) the (b) a Q70. (a) fabricate (b) generate
(c) an (d) that (c) formulate (d) emulate
Ans: (a) Ans: (d)
Q60. (a) with (b) among Directions: In the following passage, some of the words
(c) on (d) in have been left out. First read the passage over and try to
Ans: (d) understand what it is about. Then fill in the blanks with
Directions: In the following passage, some of the words the help of the alternatives given.
have been left out. First read the passage ov er and try to Happy is the man who (_____) the habit of reading when
understand what it is about. Then fill in the blanks with he is young. He has secured a life -long source of (_____)
the help of the alternatives given. instruction and inspiration. So long as he has his beloved
Man‘s basic aggressiveness is a fact and is the emotional books he need (_____) feel lonely. He always has a
factor for all the bloody violence that has marked human pleasant
history since the beginning. His technology has never (_____) of leisure moments. He is the (_____) of wealth
been as perfectly _____ as in the weapons he makes to more precious than gold. (_____) indeed is the man who
_____ his brothers. The plough has changed very _____ does not read and (_____) is his life. (_____) gives the
in design from the time man took _____ cultivation. But highest kind of pleasure. When we are (_____) it is a
swords have not become _____ and the rifle, with which healthy recreation to lose (_____) in the company of
one man kills an other, is a _____ in comparison with the books.
stone and _____ with which he used to _____ his Q71. (a) owns (b) buys
adversaries in the earliest stage of civilisation. (c) acquires (d) takes
Nuclear _____ has designs that nature can never _____. Ans: (c)
They are wonders of technological creation. Q72. (a) pleasure (b) satisfaction
Q61. (a) operated (b) implemented (c) sadness (d) dejection
(c) employed (d) deployed Ans: (a)
Ans: (c) Q73. (a) always (b) ever
Q62. (a) shoot (b) injure (c) sometimes (d) never
(c) wound (d) kill Ans: (d)
Ans: (d) Q74. (a) source (b) occupation
Q63. (a) much (b) little (c) possession (d) relief
(c) slightly (d) radically Ans: (c)
Ans: (b) Q75. (a) possessor (b) loser
Q64. (a) in (b) to (c) master (d) heir
Page 503 of 516
Q76. (a) Rich
https://telegram.me/aedahamlibra
(b) Lucky
Ans: (a) (c) achieving (d) changing
Ans: (c)
(c) Poor (d) Bad Q87. (a) with (b) in
Ans: (c) (c) of (d) from
Q77. (a) full (b) vacuum Ans: (d)
(c) empty (d) deep Q88. (a) departments (b) institutions
Ans: (c) (c) concerns (d) projects
Q78. (a) Writing (b) Speaking Ans: (b)
(c) Listening (d) Reading Q89. (a) was (b) had
Ans: (d) (c) were (d) have
Q79. (a) weak (b) fresh Ans: (a)
(c) sick (d) tired Q90. (a) rural (b) revenue
Ans: (d) (c) private (d) government
Q80. (a) himself (b) ourselves Ans: (d)
(c) yourselves (d) themselves Directions: In the following passage, there are blanks‘
Ans: (b) each of which has been numbered. These numbers are
Directions: In the following passage, some of the words printed below the passage and against each four words
have been left out. First r ead the passage over and try to are suggested, one of which fits the blanks appropriately.
understand what it is about. Then fill in the blanks with Find out the appropriate word in each case.
the help of the alternatives given. Gandhiji once said, ―I would say that if the village
The educational institutions established by the British and perishes, India will perish too. India will be _____ more
the Christian missionaries were primarily designed to India.
propagate and pr omote the English language and the Her own mission in the world will get _____ The _____
western _____. of the v illage is possible only when it is no more _____
Their aim was also to produce such _____ who could Industrialization on a mass scale will _____ lead to
man the lower levels of _____ British administrative passive or active exploitation of the villagers as the
hierarchy and remain ever loyal _____ the British rulers. problems _____ competition and marketing come in.
The British educational policy _____ with em inent Therefore, we have to _____ on the village being
success in the matter of _____ its objectives. The selfcontained, manu-facturing mainly for
majority of people _____ middle classes who went to use. Provided this character of the village industry is
these educational _____, did acquire some knowledge _____ there would be no objection to villagers using even
and skill which _____ sufficient enough to work as babus the modern machines and tools that they can make and
in these _____ offices. _____ to use. Only, they _____ not be used as a means of
Q81. (a) range (b) trade exploitation of others‖.
(c) culture (d) pride Q91. (a) certainly (b) scarcely
Ans: (c) (c) much (d) no
Q82. (a) Indians Ans: (d)
(b) North-Indians Q92. (a) lost (b) extension
(c) South-Indians (c) elevated (d) flourished
(d) Rajputs Ans: (a)
Ans: (a) Q93. (a) rehabilitation (b) pruning
Q83. (a) the (b) a (c) revival (d) devastation
(c) an (d) now Ans: (c)
Ans: (a) Q94. (a) denuded (b) exploited
Q84. (a) of (b) with (c) contaminated (d) populated
(c) for (d) to Ans: (b)
Ans: (d) Q95. (a) passionately (b) surprisingly
Q85. (a) served (b) met (c) scarcely (d) necessarily
(c) planned (d) started Ans: (d)
Ans: (b) Q96. (a) forming (b) enhancing
Q86. (a) performing (b) conducting (c) between (d) of
Page 504 of 516
Q97. (a) concentrate
https://telegram.me/aedahamlibra
(b) ponder
Ans: (d) (c) Masala Dosa
(d) synthetic fibre
(c) imagine (d) ensure Ans: (d)
Ans: (a) Q2. The synthetic fibre has
Q98. (a) regained (b) neglected (a) always been popular in India.
(c) maintained (d) thwarted (b) become popular during the last twenty years.
Ans: (c) (c) never been popular in India.
Q99. (a) prepare (b) afford (d) been as popular as other kinds of fibre.
(c) hesitate (d) propose Ans: (b)
Ans: (b) Q3. The latest symbol of modernity for the rural people is
Q100. (a) can (b) could (a) the bicycle. (b) the wristwatch.
(c) need (d) should (c) the transistor. (d) the synthetic cloth.
Ans: (d) Ans: (d)
Q4. The term Neo-rich means
(a) the aristocracy.
Comprehension Test (b) the industrialists.
(c) the newly rich people.
Directions (1-10): In these questions, you ha ve two brief (d) the common people.
passages with five questions following each passage. Ans: (c)
Read the passages carefully and choose the best answer Q5. The tone of the passage is
to each question out of the four alternatives. (a) tragic (b) ironic
PASSAGE–I What one wonders, is the lowest common (c) sombre (d) satiric
denominator of Indian culture today. The attractive Hema Ans: (c)
Malini The songs of Vividh Bharati Or the mouth - PASSAGE-II Most people who bother with the matter at
watering Masala Dosa Delectable as these may be, each all would admit that the English language is in a bad way,
yields pride of place to that false symbol of a new era–the but it is generally assumed that we cannot by conscious
synthetic fibre. action do anything about it. Our civilization is decadent
In less than twenty years, the nylon sari and the terylene and our language — so the argument runs —must
shirt have swept the countryside, penetrated to the inevitably share in the general collapse. It follows that
farthest corners of the land and persuaded every common any stru ggle against the abuse of language is a
man, woman and child that the key to success in the sentimental archaism, like preferring candles to electric
present day world lies in artificial fibres: glass nylon, light or handsome cabs to aeroplanes. Underneath this
crepe nylon, tery mixes, polysters and what have you. lies the half -conscious belief that language is a natural
More than the bicycles, the wristwatch or the transistor growth and not an instrument which we shape fo r our
radio, synthetic clothes have come to represent the first own purposes.
step away from the village square. The village lass Now it is clear that the decline of a language must
treasures the flashy nylon sari in her trousseau most ultimately have political and economic causes: it is not
dearly; the village youth gets a great kick out of his cheap simply due to the bad influence of this or that individual
terrycot shirt and trousers, the nearest he can approximate writer. But an effect can become a cause, reinforcing the
to the expensive synthetic sported by his wealthy citybred original c ause and producing the same effect in an
contemporaries. And the Neo –rich craze for ‗phoren‘is intensified form, and so on indefinitely. A man may take
nowhere more apparent than in the price t hat people will to drink because he feels himself to be a failure, and then
pay for smuggled, stolen, begged, borrowed secondhand fails all the more completely because he drinks. It is
or thrown away synthetics. Alas, even the unique rather the same thing that is happening to the English
richness of the traditional tribal costume is being fast language. It becomes ugly and inaccurate because our
eroded by the deadening uniformity of nylon. thoughts are foolish, but the slovenliness of our language
Q1. The lowest common denominator of the Indian makes it easier for us to have foolish thoughts. The point
culture today is is that the process is reversible. Modern English,
(a) Hema Malini especially written Eng lish, is full of bad habits which
(b) songs of Vividh Bharati spread by imitation and which can be avoided if one is
willing to take the necessary trouble. If one gets rid of
Page 505 of 516
https://telegram.me/aedahamlibra
these habits, one can think more clearly, and to think
clearly is a necessary first step towards political
Science has enabled man to bring forces of nature under
control and to use them for h is own advantage. It has
regeneration: so that the fight against bad English is not brought the distant parts of the world close together. Our
frivolous and is not the exclusive concern of professional knowledge of the universe has been much widened on
writers. account of the untiring efforts of the astronomers like
Q6. Many people believe that nothing can be done about Jeans and Eddington.
the English language because Remarkable cures of human diseases have been possible
(a) bad habits spread by imitation. owing to the discovery of some wonderful medicines.
(b) we live in a decadent civilization. Q11. The main idea of the passage is
(c) there are too many bad writers. (a) the impact of science can be felt in every sphere of
(d) people are too lazy to change their bad habits. life
Ans: (d) (b) science is an anathema
Q7. The author believes that (c) nothing is beyond the purview of science
(a) it‘s now too late to do anything about the problem. (d) science can work miracles
(b) language is a natural growth and cannot b e shaped for Ans: (a)
our own purposes. Q12. The mode of approach is
(c) the decline in the language can be stopped. (a) logical. (b) anatomical.
(d) the process of an increasingly bad language cannot be (c) descriptive. 4) expository.
stopped. Ans: (d)
Ans: (c) Q13. What has enabled man to harness the forces of
Q8. The author believes that the first stage towards the nature to the advantage of mankind?
political regeneration of the language would be (a) Arts. (b) Oratory.
(a) taking the necessary trouble to avoid bad habits. (c) Bravery. (d) Science.
(b) avoiding being frivolous about it. Ans: (d)
(c) clear thinking. Q14. Science has proved a great boon for
(d) for professional writers to help. (a) scientists. (b) artists.
Ans: (a) (c) explorers. (d) mankind.
Q9. The author believes that Ans: (d)
(a) English is becoming ugly. Q15. The most appropriate title for the passage will be
(b) bad language habits are inevitable. (a) Science is a curse
(c) our thoughts are becoming uglier because we are (b) Science, a great boon
making the language uglier. (c) Achievements of science
(d) our civilization is decadent so nothing can be done to (d) None of these
stop the decline of the language. Ans: (b)
Ans: (c) PASSAGE-II ―Science cannot reduce the magic of a
Q10. What causes bad language in the end ? sunset to arithmetic, nor can it express friendship with a
(a) The bad influence of individual writers. formula‖ observed the eminent medical researcher, Dr.
(b) The imitation of bad language habits. Lous Orr. He added, ―also beyond science‘s mastery of
(c) Political and economic causes. nature are love and laughter, pain and loneliness and
(d) An assumption that nothing can be done about it. insights into truth and beauty‖. This distancing of science
Ans: (c) from the human condition perhaps explains why most
Directions: You have two brief passages with five foreign tourists visiting Britain flock predictably to see
questions following each passage. Read the pas sages the hallowed homes of playwrights, writers and poets, but
carefully and choose the best answer to each question out choose to ignore the habitations where its eminent
of the four alternatives. scientists lived and worked.
PASSAGE-I The achievement of science in the twentieth Q16. Why is it that science cannot express friendship
century has been very great. Its influence can be felt in with a formula?
every sphere of life. From the small pins and needles to (a) Science and friendship cannot co-exist.
the huge iron sheets and joints, most of the things we (b) It is abstract term which cannot be grappled by
require for our everyday use, come out of factories where science.
scientific principles are utilized for practical ends. (c) Friendship is beyond science‘s mastery.
Page 506 of 516
https://telegram.me/aedahamlibra
(d) Friendship is unknown to scientists.
Ans: (c)
oil. Both cod liver oil and halibut oil are given to sick
children and other invalids who need certain vitamins.
Q17. The word magic refers to Vegetable oil has been known from very old times. No
(a) evening dusk. household can get on without it, for it is used in cooking.
(b) the sunrise. Perfumes may be made from the oils of certain flowers.
(c) solar and lunar eclipse. Soaps are made from vegetable and animal product an d
(d) setting of the sun, with all its beauty. the oils of certain flowers.
Ans: (d) Q21. The main source of animal oil is
Q18. Which of the following are beyond science‘s reach, (a) fish. (b) whale.
according to the passage? (c) sea weeds. (d) plants.
(a) Love and laughter, pain and loneliness. Ans: (b)
(b) Derivation of a formula. Q22. Vegetable oil is mainly used for
(c) Complexity of time and tide. (a) eating. (b) cooking.
(d) Work of the mind. (c) frying. (d) lubricating.
Ans: (a) Ans: (b)
Q19. The verb flock refers to. Q23. The ....... of fish yeilds nourishing oil.
(a) tourists in Britain. (a) liver (b) stomach
(b) local people. (c) eyes (d) head
(c) large number of foreign tourists visiting homes of Ans: (a)
playwrights, writers, poets. Q24. The thick protective covering of fat on a whale is
(d) Indian tourists. called a
Ans: (c) (a) skin. (b) cells.
Q20. Why according to the author do tourists prefer to (c) blubber. (d) fins.
visit hallowed homes of playwrights, writers and poets Ans: (c)
rather than visiting the habitation of eminent scientists? Q25. ....... are made from vegetable, animal products and
(a) The houses of playwright and writers are the oils of certain flowers.
welldecorated and are full of splendour (a) Perfumes (b) Cosmetics
(b) Science cannot explain human emotions. Hence, (c) Cooking medium (d) Soaps
people have a soft corner for those who produce a Ans: (d)
splendid display of emotions in their work. Directions: You have two brief passages with five
(c) Scientists are loathsome. questions following each passage. Read the passages
(d) Houses of scientists are untidy and not well - carefully and choose the best answer to each question out
preserved. of the four alternatives.
Ans: (b) PASSAGE-I Vacations were once the prerogative of the
Directions: You have a brief passage with 5 questions privileged few, even as late as the nineteenth century.
following the passage. Read the passage carefully and Now they are considered the right of all, except for such
choose the best answe r to each question out of the four unfortunate masses as in China, for whom life, except for
alternatives. sleep and brief periods of rest, is uninterrupted toil.
PASSAGE There are three main groups of oils -animal, They are more necessary now than before because the
vegetable and mineral. Great quantities of animal oil average life is well rounded and has become increasingly
comes from whales, those enormous creatures of the sea, departmenta-lised. The idea of vacations, as we conceive
which are the largest of the animals remainin g in the it must be incomprehensible to primitive pe ople. Rest of
world. To protect the whales from the cold of the Arctic some kind has of course always been a part of the rhythm
seas, nature has provided them with a thick covering of of human life, but earlier ages did not find it necessary to
fat, called blubber. organise it in the way that modern man has done.
When the whale is killed, the blubber is stripped off and Holidays, feast days, were sufficient.
boiled down. It produces a great quantity of oil which can With modern man‘s increasing tension s, with the useless
be made into food for human consumption. A few other quality of so much of his work, this break in the year‘s
creatures yield oil, but none so much as the whale. The routine became steadily more necessary. Vacations
livers of the cod and halibut, two kinds of fish, yield became necessary for the purpose of renewal and repair.
nourishing Q26. The author‘s main purpose in this passage is to
Page 507 of 516
https://telegram.me/aedahamlibra
(a) explore the history of vacations.
(b) tell why vacations have become more common.
(c) The S.S. Titanic sank near Newfoundland
(d) The S.S. Titanic was the fastest ship afloat in 1912
(c) contrast holidays and festive occasions with vacation. Ans: (a)
(d) demonstrate that vacations are not really necessary. Q32. All of the following contributed to the large death
Ans: (a) toll except
Q27. According to the passage, we need vacations now (a) panic (b) fire
more than ever because we have (c) speed (d) The Carpathia
(a) a more carefree nature Ans: (d)
(b) much more free time Q33. How many days was the S.S. Titanic at sea before
(c) little diversity in our work sinking ?
(d) a higher standard of living (a) 2 (b) 4
Ans: (a) (c) 6 (d) 12
Q28. It is implied in the passage that our lives are very Ans: (a)
(a) habitual (b) patriotic Q34. Maiden voyage is closest in meaning to
(c) varied (d) independent (a) inaugural (b) most elegant
Ans: (c) (c) longest (d) final
Q29. As used in the passage the word prerogative (line) Ans: (a)
(a) habit (b) privilege Q35. What does this passage convey?
(c) request (d) hope (a) The S.S. Titanic proved itself the most seaworthy
Ans: (b) vessel in 1912
Q30. The contemporary attitude towards vacations is (b) Attempts to rescue the S.S. Titanic‘s survivors were
best expressed by which of the following proverbs ? not successful
(a) A penny saved is penny earned. (c) Overconfidence by builders and owners was greatly
(b) The devil finds work for idle hands. responsible for the sinking of the vessel
(c) All work and no play makes Jack a dull boy. (d) A fire and panic were the only causes for the sinking
(d) Many hands make light work. of the ship
Ans: (c) Ans: (c)
PASSAGE-II In an effort to produce the largest, fastest Directions: You have two brief passages with 5 questions
and most luxurious ship afloat, the British built the S.S. following each passage. Read the passages carefully and
Titanic. It was so superior to anything else on the seas choose the best answer to each question out of the four
that it was dubbed ‗unsinkable‘. So sure of this were the alternatives.
owners that they provided only twenty life boats and PASSAGE-I The Printing Press has made knowledge
rafts, less than one -half the number needed for the 2,227 available to the vast multitude of people -Pray, what kind
passengers on board. of knowledge is it ? Is it of any permanent character ?
Many passenger s were aboard the night it rammed an Books have become common and, when we say that
iceberg only two days at sea and more than halfway books like the Se xton Blake series sell like hot cakes, we
between England and its New -York destination. Because have an index of the nature of knowledge which a typical
the luxury liner was travelling so fast, it was impossible person in a vast multitude seeks. Let me tell you of an
to avoid the ghostly looking iceberg. An unextinguished incident that took place in America a few years ago. An
fire also contributed to the ship‘s submersion. Panic American publisher printed a million copies o f the works
increased the number of casualties as people jumped into of Charles Dickens in the hope that he could easily sell
the icy water or fought to be among the few to board the them on the name of the author. But to his
life boats. Four hours after the mishap, another ship, the disappointment, not even the widest publicity and
‗Carpathia‘, rescued 705 survivors. advertisement could enable him to sell the books. Being
The infamous S. S. Titanic had enjoyed only two days of sorely tired, he hit on a plan. He tor e off the cover pages,
sailing glory on its maiden voyage in 1912 before substituted covers containing sensational love headings
plunging into 12,000 feet of water near the coast of for the titles and again advertised the new books.
Newfoundland where it lies today. In a week, all the books were sold out. We are not
Q31. All of the following are true except that concerned here with the moral of the bookseller‘s action.
(a) Only a third of those aboard perished What we have to n ote is that only books of a sensational
(b) The Carpathia rescued the survivors type are really sought for by the ordinary folk who have a
Page 508 of 516
https://telegram.me/aedahamlibra
great aversion to serious study. So, you will see that the
grand argument that the Printing Press has made
Q41. What is the main theme of the passage ?
(a) Concept of time in pre-industrial world.
knowledge available even to the masses is certainl y (b) The Greek concept of time.
fallacious and quite misleading. To put it correctly, it has (c) Awareness of time in the modern industrial world.
created a taste for a low order of books. (d) The orientals and their awareness of time.
Q36. Sexton Blake series are big sellers because they Ans: (c)
(a) disseminate knowledge. Q42. The orientals are alien to
(b) are informative. (a) the business of amusement.
(c) satisfy a typically serious reader. (b) the notion of time as a collection of minutes.
(d) are sensational. (c) industrialization.
Ans: (d) (d) the fine art of doing nothing.
Q37. The American publisher had chosen the works of Ans: (b)
Charles Dickens to Q43. A person who belongs to pre-industrial world
(a) give wide publicity to Dickens‘ works. (a) knows the utility of time.
(b) offer the readers what best he could. (b) knows how to derive happiness by making use of time
(c) counter the trash. carefully.
(d) make money easily. (c) does not care about each minute.
Ans: (d) (d) cares much for every minute.
Q38. What is the main contention of the passage ? Ans: (c)
(a) To stress the popularity of the printing press. Q44. According to the author
(b) To point out the disappointment of serious readers. (a) the orientals are very punctual.
(c) To shed light on the morale of the publishers. (b) the Americans or the Englishmen are punctual.
(d) To bring out the evil impacts of the printing press. (c) the Greek and the orientals are very punctual.
Ans: (d) (d) the Indians are very punctual.
Q39. The author‘s contention makes us feel that he Ans: (b)
(a) is unilateral in his argument. Q45. The orient in the passage refers to
(b) is balanced. (a) China and Japan. (b) Japan and England.
(c) is a typical critic. (c) England and America. (d) America alone.
(d) argues convincingly. Ans: (a)
Ans: (d) Directions: You have two brief passages with 5 questions
Q40. Who is Charles Dickens ? following each passage. Read the passages carefully and
(a) A playwright. (b) An epic poet. choose the best answer to each question out of the four
(c) A short story writer. (d) A novelist. alternatives.
Ans: (d) PASSAGE-I In the technological systems of tomorrow -
PASSAGE-II Our awareness of time has reached such a fast, fluid and self -regulating-machines will deal with the
pitch of intensity that we suffer acutely whenever our flow of physical materials; men with the flow of
travels take us into some corner of the world where information and insight.
people are not interested in minutes and seconds. The Machines will increasingly perfo rm tasks. Machines and
unpunctuality of the or ient, for example is appalling to men both, instead of being concentrated in gigantic
those who come freshly from a land of fixed meal -times factories and factory cities, will be scattered across the
and regular train services. globe, linked together by amazingly sensitive, near -
For a modern American or Englishman, waiting is a instantaneous communications. Human work will move
psychological torture. An Indian accepts the blank hours out of the factory and mass office into the community
with resignation, even with sati sfaction. He has not lost and the home. Machines will be synchronized, as some
the fine art of doing nothing. Our notion of time as a already are, to the billionth of a second; men will be de -
collection of minutes, each of which must be filled with synchronized. The factory whistle will vanish. Even the
some business or amusement, is wholly alien to the clock, ―the key machine of the modern industrial age‖ as
Greek. For the man who lives in a pre -industrial world, Lewis Mumford called it a generation ago, will lose some
time moves at a slow and easy pace; he does not care of its power over humans, as distinct from purely
about each minute, for the good reason that he has not technological affairs. Simultaneously, the organisation
been made conscious of the existence of minutes. needed to control technology shift from bureaucracy to
Page 509 of 516
https://telegram.me/aedahamlibra
Ad-hocracy, from permanence to tran sience, and from a
concern with the present to a focus on the future.
not honestly like it, or will persuade themselves that they
like it. And that does a great deal of harm. The people
In such a world, the most valued attributes of the who cannot like the book run the risk of two things
industrial age become handicaps. The technology of happening to them; either they are put off the idea of the
tomorrow requires not millions of lightly lettered men, book-let us suppose the book was David Copperfield -
ready to work in unison at e ndlessly repetitive jobs, it either they are put off the idea of classical novels, or they
requires not men who take orders in unblinking fashion, take a dislike to Dickens, and decide firmly never to
aware that the price of bread is mechanical submission to waste their time on anything of the sort again; or they get
authority, but men who can make critical judgments, who a guilty conscience about the whole thing, they feel that
can weave their way through novel environments, who they do not like what they ought to like and that therefore
are q uick to spot new relationships in the rapidly there is something wrong with them.
changing reality. It requires men who, in C.P. Snow‘s They are quite mist aken, of course. There is nothing
compelling terms, ―have the future in their bones‖. wrong with them. The mistake has all been on the
Q46. The technological system of tomorrow will be teacher‘s side. What has happened is that they have been
marked by shoved up against a book before they were ready for it. It
(a) dehumanization. (b) perfection. is like giving a young child food only suitable for an
(c) automation. (d) unpredictability. adult. Result: indigestion, violent stomach -ache, and a
Ans: (c) rooted dislike of that article of food evermore.
Q47. The future man, according to this passage, must be Q51. The passage is about what
(a) most adaptative and intelligent. (a) we should do to make children read.
(b) most capable of dealing with the changing reality. (b) we should not do when we ask children to read.
(c) more concerned with the present than the future. (c) teachers should teach in the classroom.
(d) trained and obedient. (d) treatment is to be given for indigestion.
Ans: (b) Ans: (a)
Q48. Near -instantaneous communications may be Q52. The writer says that teachers should
regarded as a symbol of (a) prevent children from reading any book.
(a) anachronization. (b) mischronization. (b) compel children to read moral stories.
(c) desynchronization. (d) synchronization. (c) stop compelling children to read books recommended
Ans: (d) by them.
Q49. If a person believes that the price of bread is (d) carefully supervise what children read.
mechanical submission to authority, he is Ans: (c)
(a) a believer in devotion to duty. Q53. According to the author many boys and girls read
(b) a believer in taking things for granted. books to
(c) a believer in doing what he is told, right or wrong. (a) win the favour of their teachers.
(d) a believer in the honesty of machines. (b) spend money in a useful way.
Ans: (c) (c) express their gratitude to their teachers.
Q50. The type of society which the auth or has mentioned (d) show others that they are lovers of books.
makes a plea for Ans: (a)
(a) a mind assimilative of modern scientific ideas. Q54. The mistake has been on the teacher‘s side . Here
(b) a critical mind having insight into future. the mistake refers to
(c) a mind well-versed in cultural heritage. (a) making the children to please the teacher.
(d) a mind with firm principles of life. (b) asking the children to read books which teachers do
Ans: (b) not like.
PASSAGE-II A reason why peo ple at school read books (c) discouraging children from reading more books.
is to please their teacher. The teacher has said that this, (d) recommending them the books intended for adults.
that, or the other is a good book, and that it is a sign of Ans: (d)
good taste to enjoy Q55. Indigestion and violent stomach -ache will be the
it. So a number of boys and girls, anxious to please their result if the child
teacher, get the book and read i t. Two or three of them (a) reads books not suitable for his age.
may genuinely like it, for their own sake, and be grateful (b) does not read any book.
to the teacher for putting it in their way. But many will (c) is forced to eat food meant for adults.
Page 510 of 516
https://telegram.me/aedahamlibra
(d) is not taken to doctor regularly.
Ans: (a)
(a) understand shades of meaning not readily available in
written responses.
Directions: You have two brief passages with 5 questions (b) observe the physical stature of the interviewee.
following each passages. Read the passages carefully and (c) listen to the voice of the interviewee directly.
choose the best answer to each question out of the four (d) compel the interviewees to express their opinions in
alternatives. writing.
PASSAGE-I The interview may be conducted by letter Ans: (a)
and by telephone, as well as in person. Letter and Q60. The author used ‗individual opinions‘, ‗attitudes‘
telephone interviews are less satisfactory. Direct contact and ‗preferences‘ as examples of
with an individual and a face -to-face relationship often (a) objective data about the interviewees.
provide a stimulating situation for both inter viewer and (b) abstract philosophical concepts irrelevant to the
interviewee. Personal reaction and interaction aid not interview process.
only in rapport but also in obtaining nuances and (c) psychological properties particular to a given
additional information by the reactions which are more interviewee.
fully observed in a face-to-face relationship. (d) likes and dislikes common to interviewers and
Adequate preparation for the interview is a ―must‖. interviewees.
Careful planning saves not only time but also energy of Ans: (c)
both parties concerned. The interview is used to obtain PASSAGE-II Among the natural resources which can be
facts or subjective data such as individual opinions, called upon in national plan for development, possibly
attitudes, and preferences. Interviews are used to check the most important is human labour; without productive
on questionnaires w hich may have been used to obtain labour force, including effective leadership and
data, or when a problem being investigated is complex, or intelligent middle ma nagement, no amount of foreign
when the information needed to solve it cannot be assistance or natural wealth can ensure successful
secured easily in any other way. People will often give development and modernisation. One essential factor is
information orally but will not put it in writing. usually overlooked or ignored. The forgotten factor is the
Q56. The intention of the writer of this passage is to role of women. Development will be handicapped as long
(a) warn the readers against conducting interviews. as wo men remain second -class citizens, uneducated,
(b) instruct people on the best means of conducting without any voice in family or community decisions,
interviews. without legal or economic status, married when they are
(c) tell people how to make friends with interviewers. still practically children, and thenceforth producing one
(d) advise people on the use of letters and telephone. baby after another, often only to see half o f them die
Ans: (b) before they are of school age. We can enhance
Q57. According to the author the best way to conduct development by improving ‗woman power‘ by giving
interviews is women opportunity to develop themselves.
(a) to talk to the interviewees over telephone. Q61. The most important natural resource for national
(b) to write letters to the interviewees. development is
(c) to observe the interviewees from a distance. (a) human labour.
(d) to have a direct conversation with the interviewees. (b) effective leadership.
Ans: (d) (c) intelligent middle management.
Q58. If I want to interview someone, (d) foreign assistance.
(a) all I need to do is to just drop in and have a talk with Ans: (a)
the person. Q62. Human power means
(b) I ought to plan and prepare for the interview well in (a) only men.
advance. (b) only women.
(c) I have to ring up the person and ask him/her all the (c) people including children.
questions I want to. (d) both men and women.
(d) establishing good rapport with the person will be Ans: (d)
enough. Q63. The ignored and overlooked for factor in the
Ans: (b) development of the nation is
Q59. Face-to-face interaction with the interviewees (a) role of women.
enables the interviewer to (b) role of middle management.
Page 511 of 516
(c) role of child labour.
(d) role of foreign assistance.
https://telegram.me/aedahamlibra
(d) By being conservative.
Ans: (a)
Ans: (a) Q69. What does the absence of any bad advance indicate
Q64. Women have hardly ?
(a) any voice in family or community decisions. (a) A penchant for risks
(b) any economic or legal status. (b) Immense conservatism
(c) any voice in family or community decision or legal or (c) Financial independence
economic status. (d) A deep-seated social commitment
(d) any voice to decide about themselves. Ans: (b)
Ans: (c) Q70. What would happen if novel and pra gmatic
Q65. Woman power is techniques are ignored ?
(a) an essential power in the development of the nation. (a) Will put the banks in danger.
(b) an essential power in child production. (b) Will undermine the banks‘ social commitment.
(c) an essential power in marriages. (c) Will reveal the untapped talent.
(d) an essential power in the death of children. (d) Will result in inefficient portfolio management.
Ans: (a) Ans: (b)
Directions: You have one brief passage with five Directions: You have two brief passages wi th 5 questions
questions following the passage. Read the passage following each passage. Read the passages carefully and
carefully and choose the best answer to each question out choose the best answer to each question out of the four
of the four alternatives. alternatives.
The public sector banks are witnessing in India a period PASSAGE-I In the past 50 years, doctors across the
of transition and are at crossroads, where they without world have accepted the practice to prescribe antibiotics
giving up social responsibility, should also remain at the first sign of a trivial infection or treat patients with
healthy. a handful of antibiotics. These days it is not uncommon
They need to undertake risky experiments, yet perform it to see practitioners prescribing multiple antibiotics
innovatively in a way it does not fail. They should make without any real indication or relevance for such a
forays into new areas which are rarely tread by them and combination of drugs.
lose no emerging opportunities. It should be understood Antibiotics ha ve traditionally been known as miracle
that absence of any bad advance is no sign of efficient drugs, but there is growing evidence that they are
banking system. It only indicates immense conservatism. overworked miracles, especially in countries like ours
However this is no guarantee for profit. There should be a where there is easy access to drugs across the counter,
balance between liquidity and risk. Past sins should be including antibiotics. We cannot think of a return to p re-
forgotten. Novel and pragmatic techniques should be antibiotic days. Yet the unbridled use of these agents is
adopted without which banks would be in danger. inexorably propelling us in that direction.
Q66. What, according to the author, are the publi c sector Q71. Antibiotics are called overworked miracles
banks witnessing ? because
(a) A period of profit (b) A period of change (a) they performed miracles.
(c) A period of certainty (d) A loss-making period (b) they are hardly used.
Ans: (b) (c) they are over-used.
Q67. In addition to being socially responsible, what does (d) they exhausted their miracles.
the author want the banks to be ? Ans: (c)
(a) Customer friendly Q72. ‗We cannot think of a return to pre-antibiotic days‘
(b) Able to attract foreign investors means
(c) Financially healthy (a) antibiotics became indispensable.
(d) Senseless risk-takers (b) we must go back to pre-antibiotic days.
Ans: (c) (c) we cannot stop using antibiotics.
Q68. How can the banks take risks without risking a (d) we can stop using antibiotics.
failure ? Ans: (c)
(a) By being innovative. Q73. The passage tells us that
(b) By soliciting the help of the government. (a) the antibiotics work miracles.
(c) By being financially healthy. (b) the antibiotics are available at the counters.
Page 512 of 516
https://telegram.me/aedahamlibra
(c) the use of antibiotics is uncontrollable.
(d) antibiotics are used indiscriminately.
Q77. Commensal relationship is a type of symbiosis in
which the relationship is beneficial
Ans: (d) (a) to one and harmless to other.
Q74. The passage discusses the use of (b) to one and harmful to other.
(a) drugs in general. (c) to both.
(b) miracle drugs. (d) to both for a very short time.
(c) antibiotics. Ans: (a)
(d) combination of different drugs. Q78. The passage talks about how animals
Ans: (c) (a) help each other.
Q75. These days it is not uncommon to prescribe (b) live together.
antibiotics‘ means (c) take advantage of the weaker ones.
(a) it is rare to prescribe antibiotics. (d) are related to each other.
(b) it is a common practice to prescribe antibiotics. Ans: (d)
(c) it is not a common practice to prescribe antibiotics. Q79. Parasites
(d) it is compulsory to prescribe antibiotics. (a) are neither beneficial nor harmful to animals they are
Ans: (b) with.
Passage II Among Nature‘s most intriguing phenomena (b) benefit at the expense of the animals they live with.
are the partnerships formed by any different species. The (c) are beneficial to the animals they live with.
name used for these relati onships, Symbiosis, comes (d) harm the animals they live with.
from Greek meaning "living together". Not all symbiotic Ans: (b)
relationships are the same. There are some called Q80. Remora feeds
commensal relationships, in which one partner gains a (a) on the shark it travels with.
benefit while the other gains little or none but is not (b) on the left-over parts of the shark‘s prey.
harmed. One ex ample is the relationship between two (c) by detaching itself to attack the prey.
types of fish – remoras and sharks. The remora, which is (d) on a whale or another type of fish.
long and often striped, attaches itself to a shark Ans: (b)
(sometimes to another type of fish or a whale), using a Directions: You have two brief passages with five
sucker on its head. When the shark makes a kill, the questions following each passage. Read the passages
hitchhiker briefly detaches itself to feed on the scraps. carefully and choose the best answer to each question out
Another type of symbiotic relationship is parasitism, in of the four alternatives.
which one partner benefits at the expense of others. Ticks PASSAGE–I Power and possession have been central
and tapeworms are among familiar parasites. pursuits of modern civilisation for a long time. They
The third type of symbiotic relationshi p, called blocked out or distorted other features of the western
mutualism, is a true partnership in which both partners renaissance (revival) which promised so much for
benefit. humanity. What people have been and are still being
The relationship may be limited as when zebras and taught to prize are money, success, control over the lives
wildbeast graze together on the vast African grasslands. of others, acquisition of more and more objects. Modern
Each species can survive on its own, but together their social, political, and economic systems, whether
chances of detecting predators are improved because each capitalist, fascist or communist, reject in their working
contributes a specially keen sense. (Zebras have the better the basic principle th at the free and creative unfoldment
eyesight; wildbeast, hearing and sense of smell). In a few of every man, woman and child is the true measure of the
cases partners are so interdependent that one cannot worth of any society. Such unfoldment requires
survive without the other. Most mutualistic relationships understanding and imagination, integrity and compassion,
probably lie somewhere in betwe cooperation among people and harmony between the
Q76. Remora attaches itself to the shark or whale human species and the rest of nature. Acquisitiveness and
(a) by entwining its long body around the bigger fish. the pursuit of power have made the modern man an
(b) by biting into the fish‘s body with its teeth. aggressor against everything that is non -human; an
(c) with an adhesive organ found in its head. exploiter and oppressor of those who are poor, meek and
(d) with a hook like structure in its head. unorganised; a pathological type which hates and
Ans: (c) distrusts the world and suffers from both acute loneliness
and false pride.
Page 513 of 516
https://telegram.me/aedahamlibra
Q81. The author appears to be advocating which of the
following approaches to be adopted by society.
doubly hard. On the other hand, the threat of debarment,
will almost certainly impact adversely on his/ her
(a) Capitalistic (b) Communists performance. Of course, linking pr e-boards to the boards
(c) Humanistic (d) Authoritarian is only one of the problems with our school system.
Ans: (c) Q86. Which is the good news for parents, according to
Q82. Which of the following best describes the the passage?
behaviour of modern man? (a) School will take the responsibility of preparing
(a) Imaginative and sympathetic. students for the board.
(b) Cruel and greedy. (b) School will provide study facilit ies to the poor
(c) Conscientious and co-operative. student.
(d) Percepting and creative. (c) Schools will enforce discipline to ensure higher
Ans: (b) attendance of students.
Q83. According to the passage, why has modern man (d) No students can be barred from the Boards without
turned out as an enemy of everything that is nonhuman? prior clearance from the CBSE.
(a) He has been dominated by drives of acquisitiveness Ans: (d)
and power. Q87. What is the ruling of the CBSE ?
(b) He consciously practises spirit of co-operation. (a) Students must pass the pre -board exam before
(c) Non -humans have refused co -operation to human appearing for the Board exam.
beings. (b) Schools should follow the practice of
(d) He hates and distrusts other human beings. performancelinked debarment.
Ans: (c) (c) Schools should maintain the performance record of
Q84. Which of the following statements is not true in students at high level.
context of the given passage ? (d) Schools must motivate students to work hard.
(a) Power and possession go hand in hand. Ans: (a)
(b) The modern man is not individualist. Q88. What is the faulty assumption of schools, according
(c) There is a need for a new renaissance. to the passage?
(d) Poor and weak people are oppressed by the modern (a) Students who do not do well at pre -boards will be
man. motivated to work hard.
Ans: (d) (b) Pre-boards are generally easy and therefore students
Q85. Which of the following is one of the requirements take them lightly.
bringing out the best is man ? (c) Students who fare poorly at the pre -board will fail at
(a) Money. (b) Success. the boards.
(c) Power. (d) Understanding. (d) Learning by note is a better method of learning.
Ans: (d) Ans: (c)
PASSAGE-II Child psychology is certainly not a strong Q89. Which of the following according to the passage is
point wi th most Indian schools; why else would they the problem with our school system ?
inflict a double trauma on a student forming badly in the (a) Providing study facilities to the students.
pre-boards by banning her from taking the board exams. (b) Linking pre -board performance of students to the
Often with fatal results as evidenced by reports of student boards.
suicides in the run -up to the boards. N ow the Central (c) Teacher‘s lack of knowledge of child psychology.
Board of Secondary Education (CBSE) has stepped in (d) Attracting talented students
and put the brakes on this discriminatory practice, ruling Ans: (d)
that no student can be barred from the Boards without Q90. According to the passage, parents had to live with
prior clearance from the CBSE. This is good news for the threat of
parents and students, ma ny of whom have had to live (a) falling grades of their wards.
with the threat of performancelinked department. While (b) not getting their wards admitted in the quality
the school‘s logic is that in order to attract talented schools.
students, they need to maintain their performance records (c) schools not treating their wards with the attitude of
at high levels. Chances are that a student faring poorly at counsellor.
the pre -boards will replicate this at the boards is faulty. (d) linking performance of their wards in pre -boards to
Chances are that the student will be spurred to work the debarment.
Page 514 of 516
https://telegram.me/aedahamlibra
Ans: (d)
Directions: You have two brief passages wit h 5 questions
being comes from the assumption that our lives are easier
than those of earlier generations.
following each passage. Read the passages carefully and The lives of the so-called primitive peoples are thought to
choose the best answer to each question out of the four be harsh –their existence dominated by the ‗incessant
alternatives. quest for food‘. In fact, primitives did very little work. By
PASSAGE–I In 776 BC the First Olympic Games were contemporary standards we‘d have to judge them very
held at the foot of Mount Olympus to honour the Greek‘s lazy.
chief God Zeus. The Greeks emphasized, physical fitness The k ey to understanding why these ‗stone-age people‘
and strength in their education of youth. Therefore failed to act like us – increasing their work effort to get
contests in running, jumping, discus and javelin more things –is that they had limited desires. In the race
throwing, boxing and horse and chariot racing were held between wanting and having, they had kept their wanting
in individual cities, and the winners competed every f our low–and, in this way ensured thei r own kind of
years at Mount Olympus. Winners were greatly honoured satisfaction.
by having poems sung about their deeds. Originally these They were materially poor by contemporary standards,
were held as games of friendship, and any wars in but in at least one dimension – time–we have to count
progress were halted to allow the games to take place. them richer.
The Greeks attached so much importan ce to these games Q96. What is the basis for progress and growth according
that they calculated time in four -year cycles called to the writer ?
‗Olympiads‘ dating from 776 BC. (a) Faith in progress is deep-rooted in our culture.
Q91. Where were the First Olympic Games held? (b) We have been taught that progress in necessary.
(a) Mount Olympus (b) Mount Olympiad (c) Material progress has given us strength.
(c) Mount Orels (d) Mount of Greeks (d) We have assumed to progress.
Ans: (a) Ans: (a)
Q92. Why were the Olympic Games held ? Q97. What does the writer attribute to modern economics
(a) To stop wars ?
(b) To crown the best athletes (a) That our lives are easier than before.
(c) To honour Zeus (b) The progress is a natural process.
(d) To sing songs about athletes (c) That material progress leads to higher satisfaction and
Ans: (c) well-being.
Q93. Approximately how many years ago did these (d) That it forces us to assume progress.
games originate ? Ans: (c)
(a) 776 years (b) 2279 years Q98. What is the writer‘s image of the primitive people ?
(c) 1207 years (d) 2781 years (a) Their life was harsh.
Ans: (d) (b) They did no work.
Q94. Which of the following contests was not held ? (c) They were lazy.
(a) Discus throwing (b) Skating (d) Search for food was their primary focus in life.
(c) Boxing (d) Running Ans: (c)
Ans: (b) Q99. What is the key to understanding the primitive
Q95. The values connected with Olympic Games were people‘s behaviour according to the passage ?
(a) physical fitness, education of youth and friendship. (a) They had no desires.
(b) health, contests and singing. (b) They had everything they needed.
(c) running, jumping, throwing and boxing. (c) They had limited desires.
(d) four-year cycles, war-time, young age and friendship. (d) They kept their wants high.
Ans: (a) Ans: (c)
PASSAGE–II Faith in progress is deep within our Q100. How does the writer appreciate the primitives ?
culture. We have been taught to believe that our lives are (a) They have a low degree of wants.
better than the lives of those who came before us. The (b) They are the masters of their time owing to their
ideology of modern economics suggests that material contentedness.
progress has yielded enhanced satisfaction and well - (c) They are materially poor.
being. But much of our confidence about our own well - (d) They are highly satisfied.
Ans: (a)
Page 515 of 516
https://telegram.me/aedahamlibra

Page 516 of 516

S-ar putea să vă placă și